Anda di halaman 1dari 488

Law 321_Corporation LAW_ Case Digest

In Partial Fulfilment
Of the Requirements for the Subject
Law 321 (Corporation Law)

Corporation Law
Case Digest

Submitted to:

Atty. Maria Lulu G. Reyes

Submitted by:

ARUMIN, Lesley Jane B.


BAGUIDUDOL, Valentin Jr. G.
BAGUILAT, Lauriz G.
BUENO, Marc Crisante C.
CAMSOL, Haryeth M.
LUBANTE, Jessica B.
ORALLO, Joanna Marie C.
ORAS, Phylian Corazon W.
SANTOS, Hyacinth B.
SECTEL, Florence O.
TUGUIC, Joshua B.

Date Submitted:

March 8, 2014

1|Page
Law 321_Corporation LAW_ Case Digest

CORPORATION CODE OF THE PHILIPPINES


(Batas Pambansa Blg. 68)

I. FORMATION AND ORGANIZATION OF CORPORATIONS

A. General Principles

1. History of Business Organizations


2. Constitutional Basis, Art. XII, Sec. 16
3. Definitions of Corporation (Sec. 2)
4. Attributes of Corporation
 Petron v. NCBA, 516 S 168
___________________________________20
 APT v. CA, 300 S 582
________________________________________21
 Mambulao Lumber v. PNB, 22 S 359
 Hanil v. CA, 362 S 1
 Bache and Co. v. Ruiz , 637 S 823
 Sulo ng Bayan v. Araneta, 72 S 347

B. Classification of Corporations

1. Private v. Public Corporation


 Boy Scout of the Phil. v. COA, June 7, 2011
____________________26
 Liban v. Gordon, July 15, 2009
 Baluyot v. Holganza, 325 S 526
 Vet. Fel. Of the Phil. v. Reyes, 483 S 526
 MIA v. CA, 495 S 591

2. Stock (Sec. 3) v. non-Stock (Sec. 3 and 87)


3. Open v. Close Corporation (Sec. 96 et. seq.)
4. Domestic v. foreign Corporation (Sec. 123 et. seq.)
5. Special Charter Corporation
6. Educational (Sec. 106 et. seq.)
7. Religious sole and aggregate (Sec. 109 et. seq.)

C. Stages in the Formation/Organization of a Corporation

1. Promotion
 March II Marketing v. Joson, December 12, 2011
_______________32
 Cagayan Fishing v. Sandiko, 65 P 223
 Caram v. CA, 151 S 372

2|Page
Law 321_Corporation LAW_ Case Digest

 Pioneer Insurance v. CA, 175 S 668


 Rizal Light v. Municipality of Morong, 25 S 258

2. Incorporation
3. Organization

D. Articles of Incorporation (Sec. 14 and 15): Contents


 Lanuza v. CA, 454 S 54
______________________________________37

1. Corporate Name (Sec. 18)


 Alonso v. Cebu, 417 S 115
___________________________________38
 Industrial Refractories v. CA, 390 S 252
 Ang mga Kaanib sa Iglesia ng Diyos v. Iglesia, December 12,
2001
 Universal Mills v. Universal Textile Mills, 78 S 62
 Lyceum of the Phil. v. CA, 219 S 610
 Indiana Aerospace University v. CHED, April 4, 2001
 Philips Export BV v. CA, 206 S 457

2. Primary Purpose (Sec. 14)


 Gala v. Ellice, 418 S 431
_____________________________________45
 Heirs of Pael v. CA, December 7, 2001
 Uy Siulong v. Director, 40 P 541
 Asuncion v. De Yriarte, 28 P 67

3. Secondary Purpose/s (Sec. 14)

4. Principal Office/Domicile (Sec. 14)


 Davao Light and Power Co. v. CA, August 20, 2001
_____________49
 Clavecilla Radio Sytem v. Antillon, 19 S 379
 Sy v. Tyson Enterprise, 119 S 367
 Young Auto Supply v. CA, 223 S 670

5. Term (Sec. 11 in rel. to Sec. 37, 81 and 120)


 Alhambra Cigar and Cigarette Mfg. v. SEC, 24 S 269
_____________53

6. Incorporators (Secs. 10 and 5)

7. Incorporating Directors (Sec. 14)

3|Page
Law 321_Corporation LAW_ Case Digest

8. Capital Stock
a) Authorized (Sec. 12)
b) Subscribed (Sec. 13)
c) Paid-up (Sec 13)

 MISCI-NACUSIP Local Chapter v. NWPC, 269 S 173


______________54

d) Outstanding (Sec. 143)


e) Minimum Requirements for Incorporation

9. Classification of Shares (Sec. 6)


a) Common v. Preferred
b) Par value v. No par value shares
c) Voting v. Non-voting
d) Founder’s Shares (Sec. 7)
e) Redeemable Preferred (Sec. 8)
f) Treasury (Sec. 9)

10. Subscribers (Sec. 14)


11. Treasurer-in-trust (Sec. 15)
12. Special Provisions
a) “No Transfer” Clause

13. Amendment and/or rejection of Articles of Incorporation (Secs. 16


and 17)
 Republic Planters Bank v. CA, 216 S 738
_______________________55

E. Commencement of Corporate Existence (Sec. 19)/Theory of Concession

F. Doctrine of Corporate Entity

G. Doctrine of Piercing the Veil of Corporate Fiction: Instances

1. Public Convenience Cases;


2. Fraud Cases;
3. Alter Ego/Instrumentality Cases.
 PNB v. Hydro Resources, March 13, 2013
______________________57
 Ramirez v. Mar Fishing, Inc., June 13, 2012
 Sarona v. NLRC, January 18, 2012
 Gold Line Tours v. heirs of Lacsa, June 18, 2012
 Hacianda Luisita v. Presidential Agrarian Council, January 22,
2011
 Pantranco Employees Assoc., et al. v. NLRC, March 17, 2009

4|Page
Law 321_Corporation LAW_ Case Digest

 Cagayan Valley Drug Corp v. CIR, 545 S 10


 Heirs of Pajarillo v. CA, 537 S 96
 Petron v. NLRC, 505 S 596
 China Banking v. Dyne-Sem, 494 S 493
 Marubeni v. Lirag, August 10, 2001
 Francisco v. Mejia, August 14, 2001
 PNB v. Andrada Electric, 382 S 244
 AZCOR Mfg. v. NLRC, 303 S 26
 Claparols v. CIR, 65 S 613
 CIR v. Norton and Harrison, August 31, 1964
 Concept Builders v. NLRC, 257 S 149
 Complex Electronics Employees Assoc. v. NLRC, 310 S 403
 Cordon v. Balicanta, October 4, 2002
 Delpher Trades v. IAC, January 2, 1988
 Del Rosario v. NLRC, July 24, 1990
 First International Bank v. CA, 252 S 259
 Francisco Motors v. CA, 309 S 73
 Laguio v. NLRC, 262 S 709
 Lim v. CA, 323 S 102
 Matuguina Integrated Wood Products v. CA, 263 S 490
 Manila Hotel Corp. v. NLRC, October 13, 2000
 Norton and Harrison v. Collector, 11 S 74
 San Juan Structural v. CA, 296 S 634
 Tan Boon Bee v. Jarencio, 163 S 205
 Telephone Eng’g and Service Co. v. WCC, 104 S 354
 Umali v. CA, September 13, 1990
 Vlason Enterprises v. CA, 310 S 26
 Villa Rey transit v. Ferrer, October 29, 1968

H. De Facto Corporation (Sec. 20)


 Hall v. Piccio, 86 P 603
______________________________________90

I. Corporation by Estoppel (Sec. 21)


 International Express v. CA, 343 S 74
__________________________91
 Lim Tiong v. PFGI, Inc., 317 S 728
 Albert v. University Publishing, 13 S 84

J. Non-user of Charters v. Continuous Inoperation (Sec. 22)


 Loyola Grand Villas v. CA, 276 S 681
__________________________94

II. BOARD OF DIRECTORS (SEC. 22 ET. SEQ.)

A. Nature of Office

5|Page
Law 321_Corporation LAW_ Case Digest

B. Requirements

1. Qualifications/Qualifying shares (Sec. 24)


 Villafuerte v. Moreno, October 2, 2009
________________________95
 Baguio v. CA, 26S 366
 Detective and Protective Bureau v. Cloribel, 26 S 255
 Grace Christian HS. CA, 281 S 133
 Lee v. CA, 205 S 752

2. Disqualifications (Sec. 27)


 Brias v. Hord, 24 P 286
_____________________________________100

3. Residence
4. Nationality
C. Election (Sec. 24)
1. Quorum
2. Voting
 Aurbach v. Sanitary Wares, 180 S 131
________________________101
 Bataan Shipyard v. PCGG, 150 S 181

D. Report on Election (Sec. 26)


 Premium Marble v. CA, 264 S 11
_____________________________103

E. Term of Office/Holdover
 Seneres v. COMELEC and Robles, April 16, 2009
_______________104

F. How removed (Sec. 28)


 Lambert v. Fox, 26 P 588
___________________________________105

G. How Vacancy filled (Sec. 29)


 Valle Verde Country Club v. Africa, September 4, 209
__________106

H. How Compensated (Sec. 30)


 Singson, et al. v. COA, August 9, 2010
________________________107
 Western institute v. Salas, 278 S 216
 Central Coop Exchange v. Tibe, 33 S 593

6|Page
Law 321_Corporation LAW_ Case Digest

 Lingayen Gulf v. Baltazar, 93 P 404

I. Authority of the Board of Directors (Sec. 24)


 La Buga’al v. Ramos, 421 S 148
_________________________111
 Shipside v. CA, 352 S 334
 ABS-CBN v. CA, 301 S 573
 Asset Privatization Trust v. CA, 300 S 582
 BA Savings Bank v. Sia, 336 S 484
 Montelibano v. Bacolod Murcia, 5 S 36
 Powers v. Marshall, May 9, 1988
 Premium Marble v. CA, 264 S 11
 Ramirez v. Orientalist, 38 P 634

J. Delegation of Authority to Corporate Officers

1. Corporate Officers/meaning of “Office” vis-à-vis Employment


 Real v. Sangu Phil., January 19, 2011
_________________________120
 Matling v. Coros, October 13, 2010
 Manila Metal v. PNB, 511 S 444
 Ongkiko v. NLRC, 270 S 613
 Lao v. CA, 325 S 694
 De Tevera v. Phil. Tuberculosis Society, 112 S 243

2. Corporate Officers (Sec. 25); Qualifications and Disqualifications;


Authority and Liabilities
 Matling v. Coros, October 13, 2010
__________________________126
 Okol v. Slimmers World, December 11, 2011
 Gomez v. PNOC DMC, November 27, 2009
 E.B. Villarosa and Partners, Co. v. Benito, 312 S 65
 SSPC v. Bardaje, 522 S 155
 Cagayan Valley Drug Corp v. CIR, 545 S 10
 Pabon v. NLRC, 296 S 8
 Vlason Enterprise v. CA, 310 S 26
 Prime White Cement v. IAC, 220 S 103
 Louis Vuitton SA v. Villanueva, 216 S 121
3. Executive Committee (Sec. 35)

4. “Doctrine of Apparent Authority’


 Banate v. Philippine Countryside, July 13, 2010
________________136
 Sargasso v. PPA, July 5, 2010
 Associated Bank v. Sps. Ponstroller, 3 September 2009
 Acuna v. Batac Producers, 20 S 326

7|Page
Law 321_Corporation LAW_ Case Digest

 Board of Liquidators v. Kalaw, 20 S 987


 Francisco v. GSIS, 7 S 577
 Rural Bank v. Ocfemia, 325 S 99

K. Three-Fold Duties of Directors and Officers: Diligence, Loyalty and Obedience

1. Duties (Sec. 31): Business Judgment Rule

2. Personal Liability of Directors and other Corporate Officers


 Ever Electrical v. Samahang Manggagawa, 13 June 2012
________143
 Harpoon v. Francisco, 2 March 2011
 Ty v. NBI, 15 December 2010
 Queensland-Tokyo Commodities v. George, 8 September 2010
 Wensha Spa Center v. Yung, 16 August 2010
 Cebu Mactan v. Masahiro, 17 July 2009
 David v. National Federation of Labor Unions, 21 April 2009
 Soriano v. People, BSP and PDIC, 30 June 2009
 Cebu Country Club v. Elizagaque, 542 SCRA 65
 Caltex Inc. v. NLRC, 536 SCRA 175
 Atrium Management v. CA, 353 SCRA 23
 ARB Construction v. CA 332 SCRA 426
 Lim v. CA, 232 SCRA 102
 Francisco v. Mejia, 14 August 2001
 DBP v. CA, 16 August 2001
 AHS Philippines v. CA, 257 SCRA 319
 Complex Electronics v. NLRC, 310 SCRA 403
 Crisologo-Jose v. CA, 15 September 1989
 FCY Construction v. CA, 324 SCRA 270
 Llamado v. CA, 270 SCRA 423
 MAM Realty Development v. NLRC, 244 SCRA 797
 Naguiat v. NLRC, 269 SCRA 564
 Progress Homes v. NLRC, 269 SCRA 274
 REAHS Corporation v. NLRC, 271 SCRA 247
 Santos v. NLRC, 254 SCRA 673
 Sia v. People, 121 SCRA 655
 Tramat Mercantile v. CA, 238 SCRA 14

3. Self-Dealing Director/Officer
 Cojuangco v. Republic, 12 April 2011
_________________________170
 Mead v. McCullough, 21 P 95
 Prime White Cement v. IAC, 220 SCRA 103

4. Contracts between Corporations with Interlocking Directors

8|Page
Law 321_Corporation LAW_ Case Digest

 Palting v. San Jose Petroleum, 18 SCRA 924


____________________173
 DBP v. CA, 363 SCRA 307

5. Disloyalty
 Gokongwei Jr. V. SEC, 89 SCRA 336
__________________________175
 Strong v. Repide, 41 P 947

6. Watered Stocks
 Lirag Textile Mills v. SSS, 31 August 1987
______________________177
 Nava v. Peers Marketing, 25 November 1976

7. Derivative Suit: Remedies to Enforce Personal Liability


 Ang v. Ang, 19 June 2013
__________________________________179
 Legaspi Towers 300 v. Muer, et. al, 18 July 2012
 Lisam Enterprises v. BDO, 23 April 2012
 STRADEC v. Radstock & PNCC, 4 December 2009
 Yu v. Yukayguan, 18 June 2009
 Gochan v. Young, 12 March 2001
 Western Institute v. Salas, 278 SCRA 216
 First International Bank v. CA, 252 SCRA 259
 Commart Philippines v. SEC, 198 SCRA 73
 Chase v. Buencamino, 136 SCRA 367
 San Miguel Corporation v. Kahn, 11 August 1989
 Everett v. Asia Banking, 49 P 512
 Gamboa v. Victoriano, 90 SCRA 40
 Reyes v. Tan, 3 SCRA 198
 Pascual v. Orozco, 19 P 84

III. POWERS OF CORPORATION (SECS. 36, ET. SEQ.)

A. In General (Sec. 36 in rel. to Arts. 44-46 of the Civil Code of the


Philippines)

1. Theory of Special Capacities v. Theory of General Capacities


 Acebedo Optical v. CA, 31 March 2000
_______________________193

2. Express, Implied and Incidental Powers, Distinguished


 Pilipinas v. SEC, 356 SCRA 193
_______________________________194

9|Page
Law 321_Corporation LAW_ Case Digest

 Luneta Motors v. Santos, 5 SCRA 809


 Teresa Electric v. PSC, 21 SCRA 199
 Powers v. Marshall, 9 May 1988

3. Power to Have/Use Corporate Name and Seal


 Laureano Investment v. CA, 272 SCRA 253
____________________198

4. Power to sue and be sued


 Tam v. Hon. Makasiar, 29 January 2001
_______________________199
 Bitong v. CA, 292 SCRA 503
 Special Services Corporation v. Centro La Paz, 28 April 1983
 R Transport Corporation v. CA, 241 SCRA 76

5. Power to acquire, dispose, encumber property


Art. XII, Section 2-3, 1987 Constitution
 Director of Lands v. CA, 14 March 1988
_______________________203

6. Power to Make Donations


 Pirovano v. Dela Rama Steamship Co., 96 P 335
________________204

7. Other Powers

B. To Increase or Decrease Capital Stock (Sec. 38)


 Madrigal V. Zamora, 51 S 355
______________________________________205
 Philtrust v. Rivera, 44 P 469

C. To Incur, Create, Or Increase Bonded Indebtedness (Sec. 38)

D. To Deny Pre-Emptive Rights (Sec. 39)


 Datu Benito v. SEC, 123 S 722 and
__________________________________207
 Dee v. SEC, 199 S 238 as clarified through
 SEC Letter Opinion, 10 March 2000
 PCGG v. SEC, 30 June 1988
 Republic v. Sandiganbayan, 4 December 2000

E. To Sell Or Otherwise Dispose Of All or Substantially All Of Corporate


Assets (Sec. 40)
In relation to Bulk Sales Law

10 | P a g e
Law 321_Corporation LAW_ Case Digest

 PNB v. Andrada Electric, 381 S 244


__________________________________211
 Islamic Directorate v. CA, 272 S 454
 Edward J. Nell Co. v. Pacific Farms, 15 S 415
 Esguerra v. CA, 3 February 1997
 Lopez Realty v. Fontecha, 247 S 183

F. To Invest Corporate Funds In Another Corporation or Business (Sec. 42)


 Gokongwei v. SEC, 89 S 336
_______________________________________216
 Dela Rama v. Ma-ao Sugar, 7 S 247

G. To Acquire Own Shares (Sec. 41)


 Boman Environmental v. CA, 22 November 1988
_____________________218
 Steinberg v. Velasco, 52 P 953

H. To Declare Dividends

1. Kinds: Cash, Stock, Property, Scrip


 Conjuangco v. Republic, 24 April 2009
________________________220

2. Declaration, Payment and Record Dates


 Cojuangco, et al v. Sandiganbayan, 24 April 2009 _______supra
(220)

3. Limitation on Retention of Surplus Profits


 Steinberg v. Velasco, 52 P 953
_______________________________222
 Nielson v. Lepanto, 26 S 540
 CIR v. Manning, 66 S 14
 Madrigal v. Zamora, 151 S 355
 Republic Planters v. Agana, 269 S 1
 Bitong v. CA, 292 S 503
 CIR v. CA, 301 S 152

I. To Enter into a Management Contract (Sec. 44)


 Aurbach v. Sanitary Wares, 180 S 131 (joint venture)
__________________229
 PNB v. Producers’ Warehouse, 42 P 608
 Nielson and Co. v. Lepanto Mining, 26 S 541
 Tuason v. Bolanos, 28 May 1954

J. Ultra Vires Acts (Sec. 45)

11 | P a g e
Law 321_Corporation LAW_ Case Digest

 Heirs of Pael v. CA, 371 S 587


______________________________________233
 Pilipinas Loan v. SEC, 356 S 193
 Crisologo v. Ca, 117 S 594
 Carlos v. Mindoro Sugar, 57 P 343
 Pirovano v. Dela Rama Steamship Co., 96 P 335
 Republic v. Acoje Mining, 7 S 361
 Republic v. Security Credit and Acceptance Corp., 19 S 58

IV. BY-LAWS (Sec. 46, et. seq)

A. Function
 Nakpil v. IBC, 370 S 653
___________________________________________240
 PMI Colleges v. NLRC, 277 S 462
 Loyola Grand Villas v. CA, 276 S 681
 Citibank NA v. Chua, 220 S 75

B. Kinds

C. When to adopt and file (Sec. 46)


 Loyola Grand Villas v. Ca, 276 S 681
_________________________________244

D. Contents (Sec. 47)

1. SEC policy on date of annual stockholders’ meeting

2. Authority to elect additional by-laws officers


 Fleischer v. Botica Nolasco, 47 P 583
__________________________245
 Gokongwei Jr. v. SEC, 89 S 336
 Government v. El Hogar Filipino, 50 P 399

E. Amendment and/or rejection of By Laws


 Salafranca v. PhilAmLife, 300 S 469
_________________________________248

V. MEETINGS OF STOCKHOLDERS AND THE BOARD OF DIRECTORS

A. Kinds (Sec. 49)


 Pena v CA, 193 S 717
________________________________________249

12 | P a g e
Law 321_Corporation LAW_ Case Digest

B. When and where held (Secs. 50, 51 and 52 in rel. to Sec 93)

C. Notice required (Secs. 50 and 53)


 Board of Liquidators v. Tan, 105 P 426
_______________________________250

D. Quorum required (Secs. 25 and 52)


 Javellana v. Tayo, 29 December 1962
_______________________________251

E. Who presides (Sec. 54)

F. Who could attend and vote (Secs. 25 and 58)


 Sales v. SEC, 13 January 1989
______________________________________252
 Ponce v. Encarnacion, 94 P 81
 Lopez v. Ericta, 45 S 539

VI. VOTING

A. Who May Exercise


 Gamboa v. Teves, 28 June 2011
____________________________________255
 COCOFED, et al. vs. Republic, 11 February 2010
______________________257
 Republic v. COCOFED, 372 S 462
 Lee v. CA, 205 S 752
 Republic v. Sandiganbayan, 402 S 84

B. Pledgors, mortgagors, executors, receivers and administrators (Sec. 55)

C. Joint owners of stack, ITF shares, and/or shares (Sec. 56)


D. Non-voting shares (Sec. 6)
E. Treasury Shares (Sec. 57 in rel. to Sec. 9)
F. Proxies (Sec. 58)

G. Voting trust agreement (Sec. 59)


 Cordon v. Balicanta, 4 October 2002
________________________________261
 NIDC v. Aquino, 163 S 153
 Lambert v. Fox, 26 P 588

VII. CAPITAL STRUCTURE STOCKS AND STOCKHOLDERS

A. Capital Stock, Meaning

13 | P a g e
Law 321_Corporation LAW_ Case Digest

1. Distinguished from Capital

2. Authorized (Sec. 12)

3. Subscribed (Sec. 13)

4. Paid-up (Sec. 13)

5. Outstanding (Sec. 143)

6. Pre-requisites to Incorporation (Sec. 13)

7. As Legal/Stated Capital: Trust Fund Doctrine


 PLDT v. NTC, 539 S 365
_____________________________________264
 NTC v CA, 370 P 538 (1999)

8. As Nationality Basis: Control Test vs. “Grandfather” Rule

9. Voting Control Test v. Beneficial Control Test


R.A. 7042, Foreign Investment Act, as amended
SEC Letter-Opinion dated 28 November 2007
 Gamboa v. Teves, 28 June 2011 and 9 October 2012
___________266
 Express Investment v. BayanTel, 5 December 2012
_____________268
 Redmont Consolidated v. McArthur Mining, SEC En Banc Case
No. 09-09-177, 25 March 2010
_____________________________________270
 Agan v. PIATCO, 21 January 2004
____________________________271

B. Classification of Shares (Sec. 6)

1. Par Value v. No Par Value Shares


2. Voting v. Non-voting
 Gamboa v. Teves, 28 June 2011 _______________________supra
(266)
 Castillo v. Balinghasay, 18 October 2004
______________________272
 Sales v. SEC, 169 S 109

3. Common v. Preferred
4. Kinds of Preferred Shares
5. Founders’ Shares (Sec. 7)

14 | P a g e
Law 321_Corporation LAW_ Case Digest

6. Redeemable Preferred (Sec. 8)


 Republic Planters’ Bank v. Agana, 269 S 1
_____________________274

7. Treasury (Sec. 9)
 CIR v. Manning, 66 S 14
_____________________________________275
 San Miguel Corporation v. Sandiganbayan, 14 September 2000

C. Trust Fund Doctrine


 National Telecommunications Commission v. SEC, 311 S 509
________277
 Ong v. Tiu, 401 S 1

D. What is an “issue”

E. What is a “subscription” (Sec. 60)


 Ong v. Tiu, 401 S 1
____________________________________________279
 Bayla v. Silang Traffic, 73 P 557
 Salmon, Dexter and Co. v. Unson, 47 P 649
 Sunset View Condominium v. Campos, 104 S 295
 Velasco v. Poizat, 37 P 802

F. Acquisition and Ownership of Shares in a Corporation; Extent of


Proprietary Right/Doctrine of Limited Liability

 Cojuangco v. Republic, 12 April 2011


____________________________284
 Espiritu v. Petron, 24 November 2009
 Crisostomo v. SEC, 179 S 146
 Garcia v. Lim, 59 P 562
 Magsaysay-Labrador v. CA, 180 S 266
 Nicolas v. CA, 27 March 1998
 Ramos v. CA, 179 S 719
 Saw v. CA, 195 S 740

G. Pre-incorporation Subscriptions (Sec. 61)

H. Consideration for Stocks (Sec. 62)


 Apodaca v. NLRC, 172 S 442
____________________________________292
 Fua Cun v. Summers, 44 P 705

15 | P a g e
Law 321_Corporation LAW_ Case Digest

 National Exchange Co. v. Dexter, 51 P 601


 Nielson and Co. v. Lepanto Mining, 26 S 541
 Trillana v. Quezon College, 93 P 383

I. Unpaid Subscriptions

1. Interest on unpaid subscriptions (Sec. 66)


2. Right of unpaid shares (Sec. 72)
3. Collection of unpaid subscription

a) Call: When necessary (Sec. 67)


 Garcia v. Suarez, 67 P 441
_________________________297
 PNB v. Bitulok Sawmill, 23 S 1366
 Velasco v. Poizat, 37 P 802

b) Court action (Sec. 70)


 Lumanlan v. Cura, 59 P 746
________________________300
 Edward Keller v. COB Group Marketing, 16 January
1986

4. How shares become delinquent (Sec. 67)

5. Effect of Delinquency (Sec. 71 in rel. to Sec. 43)


 Valley Golf & Country Club, Inc. v. Caram, 16 April 2009
______302
 Calatagan Golf Club, Inc. v. Clemente, Jr., 16 April 2009

6. Delinquency Sale (Sec. 68)


7. Grounds to Question Delinquency Sale (Sec. 69)

J. Issuance of Certificates of Stock (Sec. 64)


 Fua Cun v. Summers, 44 P 705 as compared with
__________________304
 Baltazar v. Lingayen Gulf, 14 S 522
 Tan v. SEC, 206 S 740
 Embassy Farms v. CA, 188 S 492

K. Right to Transfer of Shares/Validity of Restrictions on Right (Sec. 98 in


rel. to S15)
 Makati Sports Club v. Cheng, 16 June 2010
_______________________308
 Fleischer v. Botica Nolasco, 47 P 583
 Padgett and Babcock v. Templeton, 59 P 232
 Rural Bank of Salinas v. CA, 210 S 510

16 | P a g e
Law 321_Corporation LAW_ Case Digest

 Thompson v. CA, 298 S 280


 Yuchengco v. Velayo, 115 S 307
 Lim Tay v. CA, 293 S 634

L. Transfer of Shares of Stock and Registration (Sec. 63)


 Musni Puno v. Puno Enterprises, 11 September 2009
_______________315
 Cojuangco, et al v. Sandiganbayan, 24 April 2009
 Republic v. Sandiganbayan, 402 S 84
 Rural Bank of LIpa v. CA, 366 S 188
 BLTB v. Bitanga, 10 August 2001
 Abejo v. Dela Cruz, 149 S 643
 Batong Buhay Gold Mines v. CA, 147 S 4
 Chemphil Export v. CA, 251 S 257
 Chua Guan v. Samahang Magsasaka , 62 P 472
 CIR v. Anglo-California Bank, 106 P 903
 Delos Santos v. Republic, 96 P 577
 De Erquiaga v. CA, 27 September 1989
 Garcia v. Jomouad, 26 January 2000
 Lopez v. CA, 114 S 671
 Monserrat v. Ceron, 58 P 469
 Puyat v. De Guzman, 113 S 31
 Razon v. IAC, 207 S 234
 Rivera v. Florendo, 144 S643
 Santamaria v. Hongkong and Shanghai Bank, 89 P781
 Torres v. CA, 278 S 793
 Won v. Wack-wack Golf and Country Club, 104 P 466

M. Lost or Destroy Certificates (Sec. 73)


 PHILEX Mining v. Reyes, 118 S602
_______________________________336

VIII. CORPORATE BOOKS AND RECORDS

A. Books to be kept (Sec. 74)

1. Stock and transfer book


 Bitong v. CA, 292 S 503
__________________________________337

2. Stock transfer agent (Sec. 74)


3. Minutes book
4. Record of Business transactions

B. Inspection of corporate books and records (Sec. 74)

17 | P a g e
Law 321_Corporation LAW_ Case Digest

 Sy, et al. v. 30 March 2009


______________________________________338
 Africa v. PCGG, 205 S39
 RP V. Sandiganbayan, 199 S 39
 Gokongwei v. SEC, 89 S 336
 Gonzales v. PNB, 122 S 489
 Pardo v. Hercules Lumber, 47 P 964
 Philpotts v. Philippine Manufacturing Co, 40 P 471
 Republic v. Sandiganbayan, 199 S 39

C. Right to financial statements (Sec. 75)

IX. MERGER AND CONSOLIDATION

A. Corporate Combinations, Purposes and Methods


B. “Constituent” corporation vs. “consolidated” corporation (Sec. 76)
C. Corporate approvals required (Sec. 77)
D. Plan of merger or consolidation (Sec. 76)
E. Articles of merger or consolidation (Sec. 78)

F. Effects of merger or consolidation (Sec. 80)


 BPI v. BPI Employees Union, 18 August 2010
______________________346
 PNB v. Andrada Electric, 381 S 244
 Babst v. CA, 135 S 37
 Associated Bank v. CA, 290 S 639
 Alger Electric v. CA, 135 S 37
 CIR v. Norton and Harrison, 11 S 714
 CIR v. Rufino, 27 February 1987
 CIR v. Bio Hong, 8 April 1991

X. APPRAISAL RIGHT

A. Instances of appraisal right (Sec. 81)


B. Requirements of exercise of appraisal right (Sec. 82 and 86)
C. Effect of demand (Sec. 83)
D. Who bears costs of appraisal (Sec. 85)
E. Notation on stock certificates of dissenting stockholders (Sec. 86)

XI. NON-STOCK CORPORATIONS

A. Purposes (Sec. 88)


 Chinese YMCA v. Ching, S 460
__________________________________354
 CIR v. Club Filipino, 5 S 321

18 | P a g e
Law 321_Corporation LAW_ Case Digest

B. Distribution of Income (Sec. 87)


C. Scope to right to vote (Sec. 89)

D. Voting (Sec. 89)


 Litonjua v. CA, 286 S136
_______________________________________356
 PPSTA v. Apostol, 55 S 743

E. Transferability of interest or membership (Sec. 90)


F. Governing Board (Sec. 92)
1. Number
2. Term

G. Election of Officers (Sec. 92)


H. Place of meetings (Sec. 93)
I. Distribution of assets in case of dissolution (Sec. 94)

XII. CLOSE CORPORATIONS

A. Requirements for formation (Sec. 96)


 Dulay Enterprises v. CA, 225 S 658
_______________________________358
 San Juan Structural Steel v. CA, 296 S 63
 Naguiat v. NLRC, 269 S 54

B. Restrictions on formation of close corporations (Sec. 96)

C. Distinctions from regular corporations


1. Management (Sec. 97)
2. Meetings (Sec. 101)
3. Voting (Sec. 97)
4. Quorum (Sec. 97)
5. Board authority (Sec. 97)
6. Pre-emptive rights (Sec. 102)
7. Buy back of shares
8. Resolutions of deadlocks (Sec. 104)

D. Provisional director (Sec. 104)


E. Appraisal right in regular corporations vs. withdrawal right of a
stockholder of a close corporation (Sec. 105)

XIII. SPECIAL CORPORATIONS

A. Educational corporations
1. Distinguished from ordinary stocks / Non-Stock Corporations

19 | P a g e
Law 321_Corporation LAW_ Case Digest

2. Art. IV, Sec. 28 (3) in rel. to Art. XIV, Sec. 4 (2)(3)(4), 1987
Constitution
B. Religious corporations

1. Art. IV, Sec. 28 (3) and Art. 29 (2), 1987 Constitution


 RP v. IAC, 15 January 1988
_______________________________361
 Director of Lands v. CA, 14 March 1988

2. Corporation sole

3. Corporation aggregate
 IEMELIF, Inc., et al. v. Bishop Lazaro, et al., 6 July 2010
_______363
 IEMELIF, Inc., et al. v. Juane, 18 September 2009

XIV. DISSOLUTION OF CORPORATIONS

A. Methods

1. Voluntary

a. Where creditors are not affected (Sec. 118)


 Vesagas v. CA, 5 December 2001
___________________365

b. Where creditors are affected (Sec. 119)


 Avon Dale Garments v. NLRC, 246 S 733
____________366
 Daguhoy Enterprises v. Ponce, 96 P 15

2. Involuntary (Sec. 121 in rel. to Sec. 6, par. N, PD 902-A)


 PNB v. CFI of Pasig, 209 S 294
____________________________368

3. Shortening of corporate term (Sec. 120 in rel. to Secs. 16 and 37)

B. Liquidation (Sec. 122)

1. Methods
 Metropolitan v. Centro Development, 13 June 2012
_________369
 Metropolitan Bank Inc. v. Riverside Mills, 8 September 2010
 Yam v. CA, 303 S 1
 Alhambra Cigar and Cigarette Mfg. v. 24 S 269
 Chungka Bio v. IAC, 26 July 1988

20 | P a g e
Law 321_Corporation LAW_ Case Digest

 Republic v. Marsman Dev., 27 April 1972


 Tan Tiong Bio v. CIR, 4S 986

2. Duration
 Reynolds, Phil. V. ca, 169 s 220
___________________________376
 Mambulao v. PNB, 22 S 359

3. Powers of corporation at liquidation


 Aguirre v. FQB7, 9 January 2013
__________________________378
 Catmon Sales v. Liquidator, 15 January 2010
 Knecht v. United Cigarette, 384 S 45
 Chua v. NLRC, 190 S 558
 Clemente v. CA, 242 S 717
 Gelano v. CA, 103 S 90
 Reburian v. CA, 301 S 344
 Republic Planters Bank v. CA, 216 S 738

XV. FOREIGN CORPORATIONS

A. Definition and Rights


 Avon v. Court of Appeals, August 29, 1997
________________________386
 San Jose Petroleum v. Court of Appeals, 18 SCRA 591

B. Requirements for the Establishment of a Branch/License to do Business in


the Philippines

1. Documentary
 Georg Grotjahn vs. Isnani, 235 SCRA 216
__________________388

2. Deposit

3. Appointment of Resident Agent


 New York Marine Managers vs. CA, 249 S 417
______________389

C. Applicable laws

D. Amendment of License
 Aetna Casualty vs. Pacific Star, 29 December 1977
__________________390
 Bulakhidas vs. Navarro, 7 April 1986

21 | P a g e
Law 321_Corporation LAW_ Case Digest

 Shmid and Oberly vs RJL, 18 October 1988

E. “Doing business” with or without license: Suits By or Against Foreign


Corporation
 AM No. 11-3-6-SC New Rule on Service Summons on
Foreign Juridical Entities
 Steelcase, Inc. vs. Desing International, 18 April 2012
_______________393
 PDIC vs. Citibank, 11 April 2012
 Cargill, Inc. vs. Intra Strata, 15 March 2010
 Sehwani vs. In and Out Burger, 536 S 225
 MR Holdings vs. Bajar, 380 S 617
 Commissioner of Customs vs. KMK Gani, 182 S 591
 Communications an Materials Designs vs. CA, 260 S 144
 Columbia Pictures vs. Court of Appeals, 261 S 144
 Eriks PTE Ltd. Vs. CA, 276 S 567
 Far East International vs. Nnkai Kogyo, 6 S 725
 Facilities Management vs. Dela Osa, 89 S 131
 HB Zachray and Co. vs. Court of Appeals, 232 S 29
 Hutchison Ports vs. SBMA, 31 August 2000
 La Chemise Lacoste vs. Fernandez, 129 S 373
 Marubeni Nederlands vs. Tensuan, 28 September 1990
 Phil. Columbia vs. Lantin, 39 S 376
 Philip Morris vs. Fortune Tobacco, 493 S 333
 Puma vs. IAC, 158 S 233
 SBMA vs. Universal International, 14 September 2000
 Tibe vs. Reyes, 39 S 304
 Universal Rubber vs. Court of Appeals, 130 S 104
 Van Zuiden vs. GTVL Industries, 523 S 233
SECURITIES AND EXCHANGE COMMISSION LAW
(P.D. No. 902-A, as Amended by R.A. No. 8799 or Securities Regulation Code)

I. STRUCTURE OF THE SECURITIES AND EXCHANGE COMMISSION

II. ENTITIES UNDER ABSOLUTE JURISDICTION, SUPERVISION AND CONTROL OF THE


SEC

A. Corporations, partnerships or associations which are grantees of primary


franchises
B. Investment Houses
C. Financing Companies

III. POWERS AND FUNCTIONS OF THE SEC


 SEC vs. PFEC, 495 S 579
_______________________________________________415
 Arranza vs. B.F. Homes, Inc., 19 June 2000

22 | P a g e
Law 321_Corporation LAW_ Case Digest

 Quasha vs. SEC, 83 S 557


 Traders’ Royal Bank vs. Court of Appeals, 26 September 1989
 VICMAR Development vs. Court of Appeals, 185 S 634

IV. ORIGINAL AND EXCLUSIVE JURISDICTION OF THE REGIONAL TRIAL COURTS


 Orendain vs. BF Homes, 506 S 634
_____________________________________420
 Pascual vs. Court of Appeals, 339 S 117

A. Devices of schemes amounting to fraud of misrepresentation


 Fabia vs. Court of Appeals, 388 S 574
____________________________422
 A & A continental vs. SEC, 225 S 314
 Alleje vs. Court of Appeals, 240 S 495
 Banez vs. Dimensional Construction , 140 S 249
 Sesbreno vs. Court of Appeals, 240 S 606

B. Controversies arising out of intra-corporate of partnership relations


 Aguirre vs. FQB7, 9 January 2013
_______________________________427
 Go, Lim, et al. vs. Distinction Properties, 25 April 2012
 Strategic Alliance vs. Star Infrastructure, 17 November 2010
 GD Express vs. Court of Appeals, 8 May 2009
 Iglesia vs. Juane 18 September 2009
 GD Express Worldwide N.V. vs. Court of Appeals, 8 May 2009
 Intestate Estate of Ty vs. Court of Appeals, 356 S 661
 Fabia vs. Court of Appeals, 363 S 427
 Vesagas vs. Court of Appeals, 371 S 508
 Abejo vs. Dela Cruz, 149 S 654
 Aguinaldo vs. SEC, 163 S 262
 Pereyra vs. IAC, 181 S 244
 Mainland Construction vs. Molvilla, 250 S 290
 SEC vs. Court of Appeals, 201 S 124
 Sunsetview Condominum vs. Campos, 104 S 295
 Western Institute of Technology vs. Salas, 21 August 1997

C. Controversies in the Election or appointment of corporate officers


 Real v. Sangu Phil., 19 January 2011
_____________________________442
 March II marketing v. Joson, 12 December 2011
 Matling v. Coros, 13 October 2010
 Garcia v. Eastern Telecom, 1 April 2009
 De Rossi v. NLRC, 314 S 245
 Espino v. NRC, 240 S 52
 Estrada v. NLRC, 262 S 709
 Islamic Directorate v. CA 272 S 454
 Ongkiko v. NLRC, 270 S 613

23 | P a g e
Law 321_Corporation LAW_ Case Digest

 Paguio v. NLRC, 253 S 166


 Pearson and George v. NLRC, 253 S 136
 Apodaca v. NLRC, 172 S 442
 PSBA v. Leano, 127 S 778
 Tabang v. NLRC, 266 S 462
 Union Motors v. NLRC, 314 S 531

D. Petitions for declaration in the state of suspension of payments


R.A. 10142 – The Financial Rehabilitation and Insolvency Act of 2010
A.M. No. 12-12-11-SC 2013 Financial Rehabilitation Rules of Procedure
 Express Investment v. Bayantel, 5 December 2012
_________________457
 Advent Capital v. Alcantara, 25 January 2012
 Siochi Fichery v. BPI, 19 October 2011
 Panililio v. RTC, 2 February 2011
 Castillo v. Uniwide Warehouse, 30 April 2010
 Pacific Wide v. Puerto Asul, 25 November 2009
 PNB and ECPIB v. CA, 20 January 2009
 Pryce Corp. v. CA, 543 S 657
 Uniwide v. Jandecs Corp. 541 S 158
 BPI v. SEC, 541 S 294
 Ching v. Land Bank of the Philippines, 201 S 191
 PCIB v. CA, 18 April 1989
 Radiola-Toshiba v. IAC, 18 July 1991
 RCBC v. IAC, 213 S 223
 Rubberworld v. NLRC, 305 S 722
 Union Bank v. CA, 19 May 1998

24 | P a g e
Law 321_Corporation LAW_ Case Digest

SECURITIES REGULATIONS CODE


(Republic Act No. 8799)

I. OVERVIEW OF THE FINANCIAL MARKETS

A. Capital Markets
1. Equities Capital (e.g. stock market)
2. Debt Capital (e.g. money market or bond market)

B. Non-Capital Markets
1. Commodities Market
2. Foreign Exchange Market
3. Options Market

II. REGISTRATION OF SECURITIES

A. Securities Defined (Sec. 3.1)


B. Elements of an Investment Contract
 Securities and Exchange Commission v. W.J. howey Co., 328 U.S.
293 (1946)
 Securities and Exchange Commission v. prosperity.Com, Inc., 25
January 2012
 Power homes Unlimited v. SEC and Manero, 2 February 2008
 SFC v. Performance, 495 S 579
 Suzuki v. De Guzman, 496 S 651
 Baviera v. Paglinawan, 515 S 170

C. What securities are required to be registered (Sec. 8)


 Timeshare Realty v. Lao, 544 S 254
 Makati Stock Exchange v. SEC, 14 S 620
 La Orden v. Stiver and Philtrust, 93 P 341

25 | P a g e
Law 321_Corporation LAW_ Case Digest

 Philippine Stock Exchenge v. SEC, 281 S 232

D. Exempt Securities (Sec. 9)

E. Exempt Transactions 9Sec 10)


 Timeshare Realty Co. v. Lao, 522 S 254
 Nestle Phils. V. CA, 203 S 504

F. Public Companies
 Philippine Veterans Bank v. Callangan, 3 August 2011

G. Registration Statement (Sec. 12)


1. Contents (Sec. 12.1-2.3)
2. Attachments
3. Signature (Sec. 12.4)

H. Grounds for Rejection of Registration Statement (Sec. 13)

I. False Registration (Sec. 56)


J. Limitations on Actions for False Registration Statement (Sec. 62)

III. TRADING IN SECURITIES

A. Margin Requirements (Sec. 48) and Restrictions on Borrowings (Sec. 49)

 Carolina Industries v. CMS Stock Brokerage, 97 S 734

B. Brokers/Dealers: Chinese Wall; Self-Regulatory Organizations; Stock


Exchange
C. Regulation of Options Trading (Sec. 25)
1. Option
2. Put
3. Call
4. Straddle or Spread

D. Manipulations of Security Prices; Devices and Practices (Sec. 24)


1. Boiler Room Operations
2. Wash Sales
3. Daisy Chain
4. Painting the Tape
5. Marketing the Close
6. Hype and Dump
7. Short Sale
8. Matched Order
9. Stop-Loss Order

26 | P a g e
Law 321_Corporation LAW_ Case Digest

E. Fraudulent Transactions (Sec. 26)

 Phil. Asso. Of Stock Transfer and Agencies v. CA


 SEC v. CA, 246 S 738
 Onapal v. CA, 218 S 281

F. Insider’s Duty to Disclose When Trading (Sec. 27)


1. Insider Defined
2. Material or Significant Facts
 Strong v. Repide, 41 P 947

3. Disclosure Regulations for Publicly-Listed Shares


 Union Bank of the Phil. v. SEC, June 2001

IV. PROTECTION OF SHAREHOLDER’S INTEREST

A. Tender Offers (Sec. 19)


 CEMCO v. National Life, 7 August 2007
B. Transactions of Directors, Officers and Principal Stockholders (Sec 23)

V. LIABILITIES

A. Administrative Sanctions (Sec. 54)


B. Civil Liabilities (Secs. 58-61)
1. Amount and Kinds of Damages (Sec. 63.1)
2. Solidary Liability (Secs. 63.2 and 63.3)
3. Limitation of Actions (Sec. 62)
C. Criminal Liabilities (Sec. 73)
D. Settlements/Nolo Contendere or Consent Decree (Sec. 55)

CORPORATION CODE
(Batas Pambansa Blg. 68)

27 | P a g e
Law 321_Corporation LAW_ Case Digest

Formation and Organization of Corporations


Attributes of Corporation

PETRON CORPORATION
vs.
NATIONAL COLLEGE OF BUSINESS AND ARTS
G.R. No. 155683. February 16, 2007

FACTS:

The V. Mapa properties owned by Felipe and Enrique Monserrat, Jr., were
mortgaged to DBP as part of the security for the loan of P5.2 million by MYTC and
Monserrat Co. MYTC mortgaged four parcels of land located in Manila. One-half of
Felipe‘s undivided interest in the V. Mapa properties was levied upon in execution of a
money judgment rendered by the RTC in the Manila case. DBP challenged the levy
through a third-party claim asserting that the V. Mapa properties were mortgaged to it
and were, for that reason, exempt from levy or attachment. The RTC quashed it.
MYTC and the Monserrats got DBP to accept a dacion en pago arrangement whereby
MYTC conveyed to the bank the four mortgaged Quiapo properties as full settlement of
their loan obligation. But despite this agreement, DBP did not release the V. Mapa
properties from the mortgage. Felipe, acting for himself and as Enrique‘s attorney-in-
fact, sold the V. Mapa properties to respondent NCBA. The Monserrats failed to comply
with this undertaking. This instigated the civil action filed by NCBA.
During the pendency of the case, ½ of Enrique‘s undivided interest in the V.
Mapa properties was levied on in execution of a judgment of the Makati case holding
him liable to Petron on a 1972 promissory note. Petron, the highest bidder, acquired
both Felipe‘s and Enrique‘s undivided interests in the property. Petron intervened in
the NCBA case.

ISSUE:

Whether or not Petron should be held liable for exemplary damages and
attorney‘s fees.

RULING:

NO.

Article 2208(5) contemplates a situation where one refuses unjustifiably and in


evident bad faith to satisfy another‘s plainly valid, just and demandable claim,
compelling the latter needlessly to seek redress from the courts. In such a case, the
law allows recovery of money the plaintiff had to spend for a lawyer‘s assistance in
suing the defendant – expenses the plaintiff would not have incurred if not for the
defendant‘s refusal to comply with the most basic rules of fair dealing. It does not
mean, however, that the losing party should be made to pay attorney‘s fees merely
because the court finds his legal position to be erroneous and upholds that of the
other party, for that would be an intolerable transgression of the policy that no one
should be penalized for exercising the right to have contending claims settled by a
court of law. In fact, even a clearly untenable defense does not justify an award of
attorney‘s fees unless it amounts to gross and evident bad faith.
No gross and evident bad faith could be imputed to Petron merely for
intervening in NCBA‘s suit against DBP and the Monserrats in order to assert what it
believed and had good reason to believe. The rule in this jurisdiction is that the
plaintiff must show that he is entitled to moral, temperate or compensatory damages
before the court may even consider the question of whether exemplary damages
should be awarded. No exemplary damages may be awarded without the plaintiff‘s
right to moral, temperate, liquidated or compensatory damages having first been
established.
ASSET PRIVATIZATION TRUST
vs.

28 | P a g e
Law 321_Corporation LAW_ Case Digest

COURT OF APPEALS, JESUS S. CABARRUS, SR., JESUS S. CABARRUS, JR.,


JAIME T. CABARRUS, JOSE MIGUEL CABARRUS, ALEJANDRO S. PASTOR, JR.,
ANTONIO U. MIRANDA, and MIGUEL M. ANTONIO, as Minority Stock-Holders of
Marinduque Mining and Industrial Corporation
G.R. No. 121171. December 29, 1998

FACTS:

MMIC, PNB and DBP executed a Mortgage Trust Agreement whereby MMIC, as
mortgagor, agreed to constitute a mortgage in favor or PNB and DBP as mortgagees,
over all MMIC's assets. Article IV of the Mortgage Trust Agreement provides for Events
of Default, which expressly includes the event that the MORTGAGOR shall fail to pay
any amount secured by this Mortgage Trust Agreement when due. In various requests
for advances/remittances of loans if huge amounts, Deeds of Undertaking, Promissory
Notes, Loan Documents, Deeds of Real Estate Mortgages, MMIC invariably committed
to pay either on demand or under certain terms the loans and accommodations
secured from or guaranteed by both DBP and PNB. Because of the tremendous loans
obtained, a financial restructuring plan (FRP) designed to reduce MMIC's interest
expense through debt conversion to equity was drafted SGV, however, it was never
adopted.
The various loans and advances made by DBP and PNB to MMIC had become
overdue and since any restructuring program relative to the loans was no longer
feasible, and in compliance with the directive of Presidential Decree No. 385, DBP and
PNB as mortgagees of MMIC assets, decided to exercise their right to extrajudicially
foreclose the mortgages in accordance with the Mortgage Trust Agreement. The assets
were eventually transferred to APT. SHs of MMIC thereafter filed a derivative suit
against DBP and PNB praying that the foreclosure be annulled, that the FRP be
followed and damages. In arbitration proceedings, MMIC obtained a favorable
decision. Court of Appeals denied due course and dismissed the petition for certiorari.

ISSUE:

Whether or not the MMIC is entitled to moral damages.

RULING:

NO.

As a rule, a corporation exercises its powers, including the power to enter into
contracts, through its board of directors. While a corporation may appoint agents to
enter into a contract in its behalf, the agent should not exceed his authority. In the
case at bar, there was no showing that the representatives of PNB and DBP in MMIC
even had the requisite authority to enter into a debt-for-equity swap. And if they had
such authority, there was no showing that the banks, through their board of directors,
had ratified the FRP.
Further, how the MMIC could be entitled to a big amount of moral damages
when its credit reputation was not exactly something to be considered sound and
wholesome. Under Article 2217 of the Civil Code, moral damages include besmirched
reputation which a corporation may possibly suffer. A corporation whose overdue and
unpaid debts to the Government alone reached a tremendous amount of P22 Billion
Pesos cannot certainly have a solid business reputation to brag about.

MAMBULAO LUMBER COMPANY


vs.

29 | P a g e
Law 321_Corporation LAW_ Case Digest

PHILIPPINE NATIONAL BANK and ANACLETO HERALDO Deputy Provincial


Sheriff of Camarines Norte
G.R. No.L-22973.January 30, 1968

FACTS:

Plaintiff applied for an industrial loan of P155, 000.00 with the PNB and the
former offered real estate, machinery, logging and transportation equipment as
collaterals. The application was approved for a loan of P100, 000.00 only. To secure
the payment of the loan, the plaintiff mortgaged to defendant PNB a parcel of land,
together with the buildings and improvements existing thereon, situated in the
province of Camarines Norte, and covered by TCT No. 381 of the land records of said
province, as well as various sawmill equipment, rolling unit and other fixed assets of
the plaintiff, all situated in its compound in the aforementioned municipality.
PNB released from the approved loan the sum of P27, 500.00, for which the
plaintiff signed a promissory note wherein it promised to pay to the PNB. PNB made
another release of P15, 500.00 as part of the approved loan granted to the plaintiff and
so on the said date, the latter executed another promissory note. Plaintiff failed to pay
the amortization on the amounts released to and received by it. Repeated demands
were made upon the plaintiff to pay its obligation but it failed or otherwise refused to
do so. Upon inspection and verification made by employees of the PNB, it was found
that the plaintiff had already stopped operation.
PNB initiated steps to have the properties extrajudicially foreclosed. The
Plaintiff opposed. The foreclosure sale of the parcel of land, together with the
buildings and improvements thereon, was held and the said property was sold to the
PNB for the sum of P56, 908.00, subject to the right of the plaintiff to redeem the
same within a period of one year. PNB sold the properties to Mariano Bundok. The
Security guard of the properties refused to let PNB‘s successor in interest to retrieve
properties inside the premises of the property bought by them.
RTC sentenced the Mambulao Lumber Company to pay to the defendant PNB.
Mambulao therefore appealed.

ISSUE:

Whether or not a corporation can be awarded moral damages.

RULING:

NO.

An artificial person like herein appellant corporation cannot experience physical


sufferings, mental anguish, fright, serious anxiety, wounded feelings, moral shock or
social humiliation which are basis or moral damages.
A Corporation may have a good reputation if besmirched, may also be a ground
for the award of moral damages. The same cannot be considered under the facts of
this case, however, not only because it is admitted that herein appellant had already
ceased in its business operation at the time of the foreclosure sale of the chattels, but
also for the reason that whatever adverse effects of the foreclosure sale of the chattels
could have upon its reputation or business standing would undoubtedly be the same
whether the sale was conducted at Camarines Norte, or in Manila which is the place
agreed upon by the parties in the mortgage contract.
But for the wrongful acts of herein appellee bank and the deputy sheriff of
Camarines Norte in proceeding with the sale in utter disregard of the agreement to
have the chattels sold in Manila as provided for in the mortgage contract, to which
their attentions were timely called by herein appellant, and in disposing of the chattels
in gross for the miserable amount of P4, 200.00, herein appellant should be awarded
exemplary damages in the sum of P10, 000.00. The circumstances of the case also
warrant the award of P3, 000.00 as attorney's fees for herein appellant.

HANIL DEVELOPMENT CO., LTD.


vs.

30 | P a g e
Law 321_Corporation LAW_ Case Digest

COURT OF APPEALS AND M.R. ESCOBAR EXPLOSIVE ENGINEERS, INC.


G.R. No. 113176.July 30, 2001

FACTS:

MPWH awarded petitioner Hanil the contract to construct the 200-kilometer


Iligan-Cagayan de Oro-Butuan Highway Project. Hanil sub-let the rock-blasting work
portion of the contract to private respondent Escobar. For the duration of the
contract, it worked on the segments of the construction undertaking designated in the
agreement as A-2, B-2, B-3, B-4, and C-1. It was fully paid for the areas A-2 and B-4.
It claimed, however, that Hanil still partially owes it one million three hundred forty
one thousand seven hundred twenty-seven and 40/100 (P1, 341, 727.40) pesos for
blastings done in the B-2, B-3 and C-1 areas. The claim was predicated on the theory
that the rocks it caused to explode in the contested areas were solid in nature, and
therefore the volume should be computed using the cross-section approach.
Escobar filed an action for recovery of a sum of money with damages against
Hanil in the CFI. CFI ordered Hanil to pay P1, 341, 727.40 for the value of rocks
blasted by Escobar; 10% of the amount due for attorney's fees; and the costs of suit.
CFI garnished the bank accounts of Hanil and levied its equipment. CFI also granted
Escobar's Ex-parte Motion to Deposit Cash praying that the Finance Manager of the
NAPOCOR be directed to withdraw Hanil's funds from the NAPOCOR and deposit the
same with the Clerk of Court. Hanil challenged the Orders before the CA, who voided
said orders.

ISSUE:

Whether or not Hanil should be awarded a much higher grant of nominal


damages and attorney‘s fees and whether they are entitled to moral and exemplary
damages.

RULING:

NO.

As to the temperate damages in form of nominal damages, Hanil is not entitled


for it failed to prove that it deserves a grant of a higher amount. Thus, P20, 000.00 is
just. Hanil failed to prove the actual value of pecuniary injury which it sustained as a
consequence of Escobar's institution of an unfounded civil suit. The testimony of one
of its witnesses presented in the CFI, to the effect that "the filing of the complaint
affected Hanil's reputation and that it affected the management and engineers working
in the site," is not enough proof. The institution of the suit, unfounded though it may
be, does not always lead to pecuniary loss as to warrant an award of actual or
temperate damages. The link between the cause (the suit) and the effect (the loss)
must be established by the required proof.
Its demand for payment of moral damages must also fail. The rule is that moral
damages cannot be granted in favor of a corporation. Being an artificial person and
having existence only in legal contemplation, a corporation has no feelings, no
emotions, and no senses. It cannot, therefore, experience physical suffering, mental
anguish, fright, serious anxiety, wounded feelings or moral shock or social
humiliation, which can be suffered only by one having a nervous system. Hanil's
prayer for exemplary damages must likewise be denied. It must be remembered that
this kind of damages cannot be recovered as a matter of right. Its allowance rests in
the sound discretion of the court, and only upon a showing of its legal foundation.
Under the Civil Code, the claimant must first establish that he is entitled to moral,
temperate, compensatory or liquidated damages before it may be imposed in his favor.
Hanil failed to do so, hence, it cannot claim exemplary damages.

BACHE & CO. (PHIL.), INC. and FREDERICK E. SEGGERMAN


vs.

31 | P a g e
Law 321_Corporation LAW_ Case Digest

HON. JUDGE VIVENCIO M. RUIZ, MISAEL P. VERA, in his capacity as


Commissioner of Internal Revenue, et al.
G.R. No.L-32409. February 27, 1971

FACTS:

Misael P. Vera, CIR wrote a letter to respondent Judge Vivencio M. Ruiz


requesting the issuance of a search warrant against petitioners for violation of Section
46(a) of the National Internal Revenue Code, in relation to all other pertinent
provisions thereof, particularly Sections 53, 72, 73, 208 and 209, and authorizing
Revenue Examiner Rodolfo de Leon, one of herein respondents, to make and file the
application for search warrant which was attached to the letter. De Leon and his
witness, respondent Arturo Logronio, went to the Court of First Instance of Rizal. They
brought with them the following papers: respondent Vera‘s aforesaid letter-request; an
application for search warrant already filled up but still unsigned by respondent De
Leon; an affidavit of respondent Logronio subscribed before respondent De Leon; a
deposition in printed form of respondent Logronio already accomplished and signed by
him but not yet subscribed; and a search warrant already accomplished but still
unsigned by respondent Judge. Judge was hearing a certain case so he instructed his
Deputy Clerk of Court to take the depositions of respondents De Leon and Logronio.
After the session had adjourned, respondent Judge was informed that the depositions
had already been taken. Respondent Judge signed respondent de Leon‘s application
for search warrant and respondent Logronio‘s deposition, Search Warrant No. 2-M-70
was then sign by respondent Judge and accordingly issued.
BIR agents served the search warrant petitioners at the offices of petitioner
corporation. Petitioners‘ lawyers protested the search on the ground that no formal
complaint or transcript of testimony was attached to the warrant. The agents
nevertheless proceeded with their search which yielded six boxes of documents.

ISSUES:

Whether or not a corporation is entitled to protection against unreasonable


search and seizures.

RULING:

YES.

Although, for the reasons above stated, the Supreme Court is of the opinion
that an officer of a corporation which is charged with a violation of a statute of the
state of its creation, or of an act of Congress passed in the exercise of its constitutional
powers, cannot refuse to produce the books and papers of such corporation, the Court
does not wish to be understood as holding that a corporation is not entitled to
immunity against unreasonable searches and seizures. A corporation is, after all, but
an association of individuals under an assumed name and with a distinct legal entity.
In organizing itself as a collective body it waives no constitutional immunities
appropriate to such body. Its property cannot be taken without compensation. It can
only be proceeded against by due process of law, and is protected against unlawful
discrimination.

SULO NG BAYAN INC.


vs.

32 | P a g e
Law 321_Corporation LAW_ Case Digest

GREGORIO ARANETA, INC., PARADISE FARMS, INC., NATIONAL WATERWORKS


& SEWERAGE AUTHORITY, HACIENDA CARETAS, INC, and REGISTER OF DEEDS
OF BULACAN
G.R. No.L-31061. August 17, 1976

FACTS:

Sulo ng Bayan, Inc. filed an accion de revindicacion with the CFI against
defendants-appellees to recover the ownership and possession of a large tract of land.
The complaint specifically alleged that plaintiff is a corporation organized and existing
under the laws of the Philippines, with its principal office and place of business at San
Jose del Monte, Bulacan; that its membership is composed of natural persons residing
at San Jose del Monte, Bulacan; that the members of the plaintiff corporation, through
themselves and their predecessors-in-interest, had pioneered in the clearing of the
fore-mentioned tract of land, cultivated the same since the Spanish regime and
continuously possessed the said property openly and public under concept of
ownership adverse against the whole world.

ISSUE:

Whether or not plaintiff corporation may institute an action in behalf of its


individual members for the recovery of certain parcels of land allegedly owned by said
members; for the nullification of the transfer certificates of title issued in favor of
defendants appellees covering the aforesaid parcels of land; for a declaration of
"plaintiff's members as absolute owners of the property" and the issuance of the
corresponding certificate of title; and for damages.

RULING:

NO.

It is a doctrine well-established and obtains both at law and in equity that a


corporation is a distinct legal entity to be considered as separate and apart from the
individual stockholders or members who compose it, and is not affected by the
personal rights, obligations and transactions of its stockholders or members. The
property of the corporation is its property and not that of the stockholders, as owners,
although they have equities in it. Properties registered in the name of the corporation
are owned by it as an entity separate and distinct from its members. Conversely, a
corporation ordinarily has no interest in the individual property of its stockholders
unless transferred to the corporation, "even in the case of a one-man corporation. The
mere fact that one is president of a corporation does not render the property which he
owns or possesses the property of the corporation, since the president, as individual,
and the corporation are separate similarities. Similarly, stockholders in a corporation
engaged in buying and dealing in real estate whose certificates of stock entitled the
holder thereof to an allotment in the distribution of the land of the corporation upon
surrender of their stock certificates were considered not to have such legal or equitable
title or interest in the land, as would support a suit for title, especially against parties
other than the corporation.
It must be noted, however, that the juridical personality of the corporation, as
separate and distinct from the persons composing it, is but a legal fiction introduced
for the purpose of convenience and to subserve the ends of justice. This separate
personality of the corporation may be disregarded, or the veil of corporate fiction
pierced, in cases where it is used as a cloak or cover for fraud or illegality, or to work -
an injustice, or where necessary to achieve equity.
Clearly, no right of action exists in favor of plaintiff corporation, for as shown
heretofore it does not have any interest in the subject matter of the case which is
material and, direct so as to entitle it to file the suit as a real party in interest.

Private v. Public Corporation

33 | P a g e
Law 321_Corporation LAW_ Case Digest

BOY SCOUTS OF THE PHILIPPINES


vs.
COMMISSION ON AUDIT
G.R. No. 177131.June 7, 2011

FACTS:

The COA maintains that the functions of the BSP that include, among others,
the teaching to the youth of patriotism, courage, self-reliance, and kindred virtues, are
undeniably sovereign functions enshrined under the Constitution and discussed by
the Court in Boy Scouts of the Philippines v. National Labor Relations Commission.
The COA contends that any attempt to classify the BSP as a private corporation would
be incomprehensible since no less than the law which created it had designated it as a
public corporation and its statutory mandate embraces performance of sovereign
functions. The COA claims that the only reason why the BSP employees fell within the
scope of the Civil Service Commission even before the 1987 Constitution was the fact
that it was a government-owned or controlled corporation; that as an attached agency
of the Department of Education, Culture and Sports (DECS), the BSP is an agency of
the government; and that the BSP is a chartered institution under Section 1(12) of the
Revised Administrative Code of 1987, embraced under the term government
instrumentality. The COA concludes that being a government agency, the funds and
property owned or held by the BSP are subject to the audit authority of the COA
pursuant to Section 2(1), Article IX (D) of the 1987 Constitution.
BSP claims that it has a unique characteristic which "neither classifies it as a
purely public nor a purely private corporation"; that it is not a quasi-public
corporation; and that it may belong to a different class altogether.

ISSUE:

Whether or not the BSP is public corporation.

RULING:

YES.

BSP is a public corporation and its funds are subject to the COA‘s audit
jurisdiction. It is a public corporation or a government agency or instrumentality with
juridical personality, which does not fall within the constitutional prohibition in Article
XII, Section 16, notwithstanding the amendments to its charter. Not all corporations,
which are not government owned or controlled, are ipso facto to be considered private
corporations as there exist another distinct class of corporations or chartered
institutions which are otherwise known as "public corporations." These corporations
are treated by law as agencies or instrumentalities of the government which are not
subject to the tests of ownership or control and economic viability but to different
criteria relating to their public purposes/interests or constitutional policies and
objectives and their administrative relationship to the government or any of its
Departments or Offices.
Note that the Administrative Code of 1987 designates the BSP as one of the
attached agencies of the Department of Education, Culture and Sports ("DECS"). An
"agency of the Government" is defined as referring to any of the various units of the
Government including a department, bureau, office, and instrumentality, government-
owned or -controlled corporation, or local government or distinct unit therein. BSP still
remains an instrumentality of the national government. It is a public corporation
created by law for a public purpose, attached to the DECS pursuant to its Charter and
the Administrative Code of 1987. It is not a private corporation which is required to be
owned or controlled by the government and be economically viable to justify its
existence under a special law.

34 | P a g e
Law 321_Corporation LAW_ Case Digest

DANTE V. LIBAN, REYNALDO M. BERNARDO, and SALVADOR M. VIARI


vs.
RICHARD J. GORDON
G.R. No. 175352.July 15, 2009

FACTS:

Petitioners filed with this Court a Petition to Declare Richard J. Gordon as


Having Forfeited His Seat in the Senate. Petitioners are officers of the Board of
Directors of the Quezon City Red Cross Chapter while respondent is Chairman of the
Philippine National Red Cross (PNRC) Board of Governors.
During respondent‘s incumbency as a member of the Senate of the Philippines,
he was elected Chairman of the PNRC during the February 23, 2006 meeting of the
PNRC Board of Governors. Petitioners allege that by accepting the chairmanship of the
PNRC Board of Governors, respondent has ceased to be a member of the Senate as
provided in Section 13, Article VI of the Constitution, which reads: ―No Senator or
Member of the House of Representatives may hold any other office or employment in
the Government, or any subdivision, agency, or instrumentality thereof, including
government-owned or controlled corporations or their subsidiaries, during his term
without forfeiting his seat. Neither shall he be appointed to any office which may have
been created or the emoluments thereof increased during the term for which he was
elected.‖

ISSUE:

Whether or not the office of the PNRC Chairman is a government office or an


office in a government-owned or controlled corporation for purposes of the prohibition
in Section 13, Article VI of the Constitution.

RULING:

NO.

PNRC is a Private Organization Performing Public Functions. The Republic of


the Philippines, adhering to the Geneva Conventions, established the PNRC as a
voluntary organization for the purpose contemplated in the Geneva Convention of 27
July 1929. The PNRC must not appear to be an instrument or agency that implements
government policy; otherwise, it cannot merit the trust of all and cannot effectively
carry out its mission as a National Red Cross Society. It is imperative that the PNRC
must be autonomous, neutral, and independent in relation to the State. To ensure and
maintain its autonomy, neutrality, and independence, the PNRC cannot be owned or
controlled by the government. Indeed, the Philippine government does not own the
PNRC. The PNRC does not have government assets and does not receive any
appropriation from the Philippine Congress. The PNRC is financed primarily by
contributions from private individuals and private entities obtained through
solicitation campaigns organized by its Board of Governors. The government does not
control the PNRC. Under the PNRC Charter, as amended, only six of the thirty
members of the PNRC Board of Governors are appointed by the President of the
Philippines.
The PNRC is not government-owned but privately owned. The vast majority of
the thousands of PNRC members are private individuals, including students. Under
the PNRC Charter, those who contribute to the annual fund campaign of the PNRC are
entitled to membership in the PNRC for one year. Thus, the PNRC is a privately owned,
privately funded, and privately run charitable organization. Hence, the office of the
PNRC Chairman is not a government office or an office in a government-owned or
controlled corporation for purposes of the prohibition in Section 13, Article VI of the
1987 Constitution. However, since the PNRC Charter is void insofar as it creates the
PNRC as a private corporation, the PNRC should incorporate under the Corporation
Code and register with the Securities and Exchange Commission if it wants to be a
private corporation.

35 | P a g e
Law 321_Corporation LAW_ Case Digest

DANTE V. LIBAN, REYNALDO M. BERNARDO and SALVADOR M. VIARI


vs.
RICHARD J. GORDON
G. R. No. 175352.January 18, 2011

ISSUE:

Whether or not considering the PNRC‘s structure is sui generis; it is a class of


its own, while it is performing humanitarian functions as an auxiliary to government,
it is a neutral entity separate and independent of government control, yet it does not
qualify as strictly private in character.

RULING:

YES.

A National Society partakes of a sui generis character. It is a protected


component of the Red Cross movement under Articles 24 and 26 of the First Geneva
Convention, especially in times of armed conflict. These provisions require that the
staff of a National Society shall be respected and protected in all circumstances. Such
protection is not ordinarily afforded by an international treaty to ordinary private
entities or even non-governmental organizations (NGOs). This sui generis character is
also emphasized by the Fourth Geneva Convention which holds that an Occupying
Power cannot require any change in the personnel or structure of a National Society.
National societies are therefore organizations that are directly regulated by
international humanitarian law, in contrast to other ordinary private entities,
including NGOs.
The PNRC, as a National Society of the International Red Cross and Red
Crescent Movement, can neither "be classified as an instrumentality of the State, so as
not to lose its character of neutrality" as well as its independence, nor strictly as a
private corporation since it is regulated by international humanitarian law and is
treated as an auxiliary of the State. The PNRC is one of the National Red Cross and
Red Crescent Societies, which, together with the International Committee of the Red
Cross (ICRC) and the IFRC and RCS, make up the International Red Cross and Red
Crescent Movement (the Movement). They constitute a worldwide humanitarian
movement.
Based on the above, the sui generis status of the PNRC is now sufficiently
established. Although it is neither a subdivision, agency, or instrumentality of the
government, nor a government-owned or -controlled corporation or a subsidiary
thereof, as succinctly explained in the Decision of July 15, 2009, so much so that
respondent, under the Decision, was correctly allowed to hold his position as
Chairman thereof concurrently while he served as a Senator, such a conclusion does
not ipso facto imply that the PNRC is a "private corporation" within the contemplation
of the provision of the Constitution, that must be organized under the Corporation
Code. As correctly mentioned by Justice Roberto A. Abad, the sui generis character of
PNRC requires us to approach controversies involving the PNRC on a case-to-case
basis.
In sum, the PNRC enjoys a special status as an important ally and auxiliary of
the government in the humanitarian field in accordance with its commitments under
international law. This Court cannot all of a sudden refuse to recognize its existence,
especially since the issue of the constitutionality of the PNRC Charter was never raised
by the parties.
Thus, Respondent Richard J. Gordon‘s Motion for Clarification and/or for
Reconsideration and movant-intervenor PNRC‘s Motion for Partial Reconsideration of
the Decision in G.R. No. 175352 dated July 15, 2009 are GRANTED. The
constitutionality of R.A. No. 95, as amended, the charter of the Philippine National Red
Cross, was not raised by the parties as an issue and should not have been passed
upon by this Court. The structure of the PNRC is sui generis¸ being neither strictly
private nor public in nature. R.A. No. 95 remains valid and constitutional in its
entirety.

36 | P a g e
Law 321_Corporation LAW_ Case Digest

FRANCISCA S. BALUYOT
vs.
PAUL E. HOLGANZA and the OFFICE OF THE OMBUDSMAN (VISAYAS)
represented by its Deputy Ombudsman for the Visayas ARTURO C. MOJICA,
Director VIRGINIA PALANCA-SANTIAGO, and Graft Investigation Officer I ANNA
MARIE P. MILITANTE
G.R. No. 136374.February 9, 2000

FACTS:

During a spot audit in 1977, the auditors from the Philippine National Red
Cross (PNRC) headquarters discovered a case shortage in the funds of its Bohol
chapter. The chapter administrator, petitioner Baluyot, was held accountable and
thereafter, respondent Holganza as member of the board Bohol chapter, filed a
complaint with the Ofc. of the Ombudsman for malversation. Upon recommendation
of respondent Militante, an administratiave docket of dishonesty was also opened
against Baluyot. Baluyot raised the defense that the Ombudsman had no jurisdiction
as he had authority only over government owned or controlled corporations which the
PNRC was not. She gives as evidence of its private character 1) it does not receive
budgetary support from the government and all money given to it by the latter and its
instrumentalities become private funds of the organization. 2) funds for the payment
of personnel‘s salaries and other emoluments come from yearly fund campaigns,
private contributions and rentals from its properties. 3) it is not audited by COA.
PNRC, petitioner claims falls under the International Federation of Red Cross, Swiss-
based organization.

ISSUE:

Whether or not PNRC is a government owned or controlled corporation.

RULING:

YES.

PNRC is a government owned and controlled corporation, with an original


charter under RA No. 95, as amended. The test to determine whether a corporation is
government owned or controlled or private in nature is simple. Is it created by its own
charter for the exercise of a public function, or by incorporation under the general
corporation law? Those with special charters are government corporations subject to
its provisions, and its employees are under the jurisdiction of the Civil Service
Commission, and are compulsory members of the GSIS.
The PNRC was not ―impliedly converted to a private corporation‖ simply
because its charter was amended to vest in it the authority to secure loans, be
exempted from payment of all duties, taxes, fees and other charges of all kinds on all
importations and purchases for its exclusive use, on donations for its disaster relief
work and other services and in its benefits and fund raising drives.‖ Clearly then,
public respondent has jurisdiction over the matter.

37 | P a g e
Law 321_Corporation LAW_ Case Digest

THE VETERANS FEDERATION OF THE PHILIPPINES represented by Esmeraldo


R. Acorda
vs.
Hon. ANGELO T. REYES in his capacity as Secretary of National Defense; and
Hon. EDGARDO E. BATENGA in his capacity as Undersecretary for Civil Relations
and Administration of the Department of National Defense
G. R. No. 155027.February 28, 2006

FACTS:

Petitioner claims that it is not a public nor a governmental entity but a private
organization, and advances this claim to prove that the issuance of DND Department
Circular No. 04 is an invalid exercise of respondent Secretary‘s control and
supervision. Petitioner claims that its funds are not public funds because no
budgetary appropriations or government funds have been released to the VFP directly
or indirectly from the DBM, and because VFP funds come from membership dues and
lease rentals earned from administering government lands reserved for the VFP.

ISSUE:

Whether or not the VFPA is a private corporation.

RULING:

NO.

The functions of petitioner corporation enshrined in Section 4 of Rep. Act No.


2640 should most certainly fall within the category of sovereign functions. The
protection of the interests of war veterans is not only meant to promote social justice,
but is also intended to reward patriotism. All of the functions in Section 4 concern the
well-being of war veterans, our countrymen who risked their lives and lost their limbs
in fighting for and defending our nation. It would be injustice of catastrophic
proportions to say that it is beyond sovereignty‘s power to reward the people who
defended her.
Like the holding of the National Centennial Celebrations, the functions of the
VFP are executive functions, designed to implement not just the provisions of Rep. Act
No. 2640, but also, and more importantly, the Constitutional mandate for the State to
provide immediate and adequate care, benefits and other forms of assistance to war
veterans and veterans of military campaigns, their surviving spouses and orphans.
The fact that no budgetary appropriations have been released to the VFP does
not prove that it is a private corporation. The DBM indeed did not see it fit to propose
budgetary appropriations to the VFP, having itself believed that the VFP is a private
corporation. If the DBM, however, is mistaken as to its conclusion regarding the
nature of VFP‘s incorporation, its previous assertions will not prevent future budgetary
appropriations to the VFP. The erroneous application of the law by public officers does
not bar a subsequent correct application of the law.
Since petitioner VFP is a public corporation. As such, it can be placed under
the control and supervision of the Secretary of National Defense, who consequently
has the power to conduct an extensive management audit of Petitioner Corporation.

38 | P a g e
Law 321_Corporation LAW_ Case Digest

MANILA INTERNATIONAL AIRPORT AUTHORITY


vs.
COURT OF APPEALS, CITY OF PARAÑAQUE, CITY MAYOR OF PARAÑAQUE,
SANGGUNIANG PANGLUNGSOD NG PARAÑAQUE, CITY ASSESSOR OF
PARAÑAQUE, and CITY TREASURER OF PARAÑAQUE
G.R. No. 155650. July 20, 2006

FACTS:

MIAA operates the Ninoy Aquino International Airport (NAIA) Complex in


Parañaque City under E.O. No. 903, otherwise known as the Revised Charter of the
Manila International Airport Authority ("MIAA Charter"). Executive Order No. 903 was
issued on 21 July 1983 by then President Ferdinand E. Marcos. Subsequently, E.O.
Nos. 909 and 298 amended the MIAA Charter.
As operator of the international airport, MIAA administers the land,
improvements and equipment within the NAIA Complex. The MIAA Charter transferred
to MIAA approximately 600 hectares of land, including the runways and buildings and
provided that no portion of the land transferred to MIAA shall be disposed of through
sale or any other mode unless specifically approved by the President of the
Philippines.
In 2001, the Parañaque city government issued notices of levy and warrants of
levy on MIAA‘s Airport Lands and Buildings for its failure to pay real estate taxes plus
penalties amounting to Php624.5 million for the taxable years 1992 to 2001. The city
government then put the subject properties up for sale at a public auction. Thus,
MIAA brought a case for prohibition to the Court of Appeals against the City of
Parañaque.

ISSUES:

Whether or not MIAA is a public corporation owned by the State.

RULING:

YES.

MIAA is a government instrumentality vested with corporate powers to perform


efficiently its governmental functions. MIAA is like any other government
instrumentality; the only difference is that MIAA is vested with corporate powers.
When the law vests in a government instrumentality corporate powers, the
instrumentality does not become a corporation. Unless the government
instrumentality is organized as a stock or non-stock corporation, it remains a
government instrumentality exercising not only governmental but also corporate
powers. Thus, MIAA exercises the governmental powers of eminent domain, police
authority and the levying of fees and charges. At the same time, MIAA exercises "all
the powers of a corporation under the Corporation Law, insofar as these powers are
not inconsistent with the provisions of this Executive Order."
Many government instrumentalities are vested with corporate powers but they
do not become stock or non-stock corporations, which is a necessary condition before
an agency or instrumentality is deemed a government-owned or controlled
corporation. Examples are the Mactan International Airport Authority, the Philippine
Ports Authority, the University of the Philippines and Bangko Sentral ng Pilipinas. All
these government instrumentalities exercise corporate powers but they are not
organized as stock or non-stock corporations as required by Section 2(13) of the
Introductory Provisions of the Administrative Code. These government
instrumentalities are sometimes loosely called government corporate entities. However,
they are not government-owned or controlled corporations in the strict sense as
understood under the Administrative Code, which is the law defining the legal
relationship and status of government entities.

39 | P a g e
Law 321_Corporation LAW_ Case Digest

Promotion

MARC II MARKETING, INC. and LUCILA V. JOSON


vs.
ALFREDO M. JOSON
G.R. No. 171993.December 12, 2011

FACTS:

Marc II Marketing, Inc. and Lucila Joson is assailing the decision of the CA for
reversing and settling aside the Resolution of the National Labor Relations
Commission. Marc II Marketing, Inc. is a corporation duly organized and existing
under and by virtue of the laws of the Philippines. It is primarily engaged in buying,
marketing, selling and distributing in retail or wholesale for export or import
household appliances and products and other items. Petitioner Lucila is the President
and majority stockholder of the corporation. Before Marc II Marketing, Inc. was
officially incorporated, Alfredo has already been engaged by Lucila, in her capacity as
President, to work as General Manager of the corporation and it was formalized
through the execution of a Management Contract dated in 1994 under Marc
Marketing, Inc., as Marc II Marketing, Inc. was yet to be incorporated. For occupying
the said position, respondent was among the corporation‘s corporate officers by the
express provision of Section 1, Article IV of its by-laws.
Alfredo was appointed as one of its officers with the designation or title of
General Manager to function as a managing director with other duties and
responsibilities that the Board may provide and authorized. However, in 1997, Marc II
Marketing Inc. decided to stop and cease its operation as evidenced by an Affidavit of
Non-Operation due to poor sales collection aggravated by the inefficient management
of its affairs. Alfredo was informed of the cessation of its business operations and the
termination of his services as General Manager. He filed action for reinstatement and
money claim against petitioners.

ISSUE:

Whether or not Marc II Marketing Inc.‘s Board of Directors could create a


position for corporate officers through an enabling clause found in its corporate by-
laws.

RULING:

YES.

The Court held that in the context of PD 902-A, corporate officers are those
officers of a corporation who are given that character either by the Corporation Code
or by the corporation‘s by-laws. Section 25 of the Corporation Code specifically
enumerated who are these corporate officers, namely: president, secretary, treasurer
and such other officers as may be provided for in the by-laws. A careful examination of
Marc II Marketing Inc.‘s by-laws, particularly paragraph 1, Section 1of Article IV
explicitly revealed that its corporate officers are composed only of chairman, president,
one/more vice president, treasurer and secretary. The position of general manager was
not among those enumerated. Meanwhile, paragraph 2, Section 1 of Article IV of the
corporation‘s by-laws empowered its Board of Directors to appoint such officers as it
may determine necessary or proper, making this an enabling provision for approving a
resolution to make the position of general manager a corporate officer. All of these acts
were done without first amending its by-laws so as to include the General Manager in
its roster of corporate officers. Though the Board of Directors may create appointive
positions other than the positions of corporate officers, the persons occupying such
positions cannot be viewed as corporate officers under Section 25 of the Corporation
Code. The said provision of the Corporation Code safeguards the constitutionally
enshrined right of every employee to security of tenure and prevents the creation of a
corporate officer position by a simple inclusion in the corporate by-laws of an enabling
clause empowering the Board of Directors.

40 | P a g e
Law 321_Corporation LAW_ Case Digest

CAGAYAN FISHING DEVELOPMENT CO., INC.


vs.
TEODORO SANDIKO
G.R. No. L-43350. December 23, 1937

FACTS:

Manuel Tabora is the registered owner of four parcels of land. To guarantee the
payment of a loan in the sum of P8,000, Manuel Tabora executed in favor of the
Philippine National Bank a first mortgage on the four parcels of land above-mentioned.
A second mortgage in favor of the same bank was executed by Tabora over the same
lands to guarantee the payment of another loan amounting to P7,000. A third
mortgage on the same lands was executed in favor of Severina Buzon to whom Tabora
was indebted in the sum of P2,9000. These mortgages were registered and annotations
thereof appear at the back of transfer certificate of title No. 217.
The board of directors of plaintiff adopted a resolution authorizing its president,
Jose Ventura, to sell the four parcels of lands in question to Teodoro Sandiko for
P42,000. The defendant having failed to pay the sum stated in the promissory note,
plaintiff, brought this action in the Court of First Instance of Manila praying that
judgment be rendered against the defendant for the sum of P25,300, with interest at
legal rate from the date of the filing of the complaint, and the costs of the suits.

ISSUE:

Whether or not the transfers were valid.

RULING:

NO.

The contract here was entered into not between Manuel Tabora and a non-
existent corporation but between the Manuel Tabora as owner of the four parcels of
lands on the one hand and the same Manuel Tabora, his wife and others, as mere
promoters of a corporations on the other hand. For reasons that are self-evident, these
promoters could not have acted as agent for a projected corporation since that which
no legal existence could have no agent. A corporation, until organized, has no life and
therefore no faculties. It is, as it were, a child in ventre sa mere. This is not saying that
under no circumstances may the acts of promoters of a corporation be ratified by the
corporation if and when subsequently organized. There are, of course, but under the
peculiar facts and circumstances of the present case we decline to extend the doctrine
of ratification which would result in the commission of injustice or fraud to the candid
and unwary.
If the plaintiff corporation could not and did not acquire the four parcels of land
here involved, it follows that it did not possess any resultant right to dispose of them
by sale to the defendant, Teodoro Sandiko.
Some of the members of this court are also of the opinion that the transfer from
Manuel Tabora to the Cagayan Fishing Development Company, Inc., which transfer is
evidenced by Exhibit A, was subject to a condition precedent (condicion suspensiva),
namely, the payment of the mortgage debt of said Tabora to the Philippine National
Bank, and that this condition not having been complied with by the Cagayan Fishing
Development Company, Inc., the transfer was ineffective. However, having arrived at
the conclusion that the transfer by Manuel Tabora to the Cagayan Fishing
Development Company, Inc. was null because at the time it was affected the
corporation was non-existent, we deem it unnecessary to discuss this point.

41 | P a g e
Law 321_Corporation LAW_ Case Digest

FERMIN Z. CARAM, JR. and ROSA O. DE CARAM


vs.
THE HONORABLE COURT OF APPEALS and ALBERTO V. ARELLANO
G.R. No. L-48627. June 30, 1987

FACTS:

The petitioners claim that this order has no support in fact and law because
they had no contract whatsoever with the private respondent regarding the above-
mentioned services. Their position is that as mere subsequent investors in the
corporation that was later created, they should not be held solidarily liable with the
Filipinas Orient Airways, a separate juridical entity, and with Barretto and Garcia,
their co-defendants in the lower court, ** who were the ones who requested the said
services from the private respondent.

ISSUE:

Whether or not the petitioners should be held liable.

RULING:

NO.

The petitioners were not involved in the initial stages of the organization of the
airline. They were merely among the financiers whose interest was to be invited and
who were in fact persuaded, on the strength of the project study, to invest in the
proposed airline.
There was no showing that the Airline was a fictitious corporation and did not
have a separate juridical personality to justify making the petitioners, as principal
stockholders thereof, responsible for its obligations. As a bona fide corporation, the
Airline should alone be liable for its corporate acts as duly authorized by its officers
and directors. Granting that the petitioners benefited from the services rendered,
such is no justification to hold them personally liable therefor. Otherwise, all the
other stockholders of the corporation, including those who came in late, and
regardless of the amount of their shareholdings, would be equally and personally
liable also with the petitioner for the claims of the private respondent.
Petitioners cannot be held personally liable for the compensation claimed by the
private respondent for the services performed by him in the organization of the
corporation. To repeat, the petitioners did not contract such services. It was only the
results of such services that Barretto and Garcia presented to them and which
persuaded them to invest in the proposed airline. The most that can be said is that
they benefited from such services, but that surely is no justification to hold them
personally liable therefor.
A promoter could not have acted as agent for a corporation that had no legal
existence. A corporation, until organized, has no life therefore no faculties. The
corporation had no juridical personality to enter into a contract.

42 | P a g e
Law 321_Corporation LAW_ Case Digest

PIONEER INSURANCE & SURETY CORPORATION


vs.
THE HON. COURT OF APPEALS, BORDER MACHINERY & HEAVY EQUIPMENT,
INC., (BORMAHECO), CONSTANCIO M. MAGLANA and JACOB S. LIM
G.R. No. 84197. July 28, 1989

FACTS:

In 1965, Jacob S. Lim, owner-operator of Southern Airlines (SAL), a single


proprietorship entered into a sales contract with regarding Japan Domestic Airlines
(JDA) regarding 2 DC-#A type aircrafts, 1 set of necessary spare parts where a Total
of $ 190,000 in instalments are to be paid. Pioneer Insurance and Surety Corp. as
surety executed its surety bond in favor of JDA on behalf of its principal Lim. Border
Machinery and Heavy Equipment Co, Inc. of Francisco and Modesto Cervantes and
Constancio Maglana contributed funds for the transaction based on the
misrepresentation of Lim that they will form a new corporation to expand his
business.
Lim as owner operator for SAL executed in favor of Pioneer a deed of chattel
mortgage as security. A restructuring of obligation to change the maturity was done
twice without the knowledge of other defendants made the surety of JDA prescribed so
not entitled to reimbursement. Upon default on the 2/8 payments, Pioneer paid for
him and filed a petition for the foreclosure of chattel mortgage as security. CA
affirmed Trial of Merits: Only Lim is liable to pay

ISSUE:

Whether or not there is failure of respondents to incorporate leading to a de


facto partnership.

RULING:

NO.

Partnership inter se does not necessarily exist, for ordinarily CANNOT be made
to assume the relation of partners as between themselves, when their purpose is that
no partnership shall exists. It should be implied only when necessary to do justice
between the parties (i.e. only pretend to make others liable). Lim never intended to
form a corporation.

43 | P a g e
Law 321_Corporation LAW_ Case Digest

RIZAL LIGHT & ICE CO., INC.


vs.
THE MUNICIPALITY OF MORONG, RIZAL and THE PUBLIC SERVICE
COMMISSION
G.R. No. L-20993.September 28, 1968

FACTS:

The bulk of petitioner's arguments assailing the personality of Morong Electric


dwells on the proposition that since a franchise is a contract, 23 at least two competent
parties are necessary to the execution thereof, and parties are not competent except
when they are in being. Hence, it is contended that until a corporation has come into
being, in this jurisdiction, by the issuance of a certificate of incorporation by the
Securities and Exchange Commission (SEC) it cannot enter into any contract as a
corporation.
The certificate of incorporation of the Morong Electric was issued by the SEC on
October 17, 1962, so only from that date, not before, did it acquire juridical
personality and legal existence. Petitioner concludes that the franchise granted to
Morong Electric on May 6, 1962 when it was not yet in esse is null and void and
cannot be the subject of the Commission's consideration. On the other hand, Morong
Electric argues, and to which argument the Commission agrees, that it was a de facto
corporation at the time the franchise was granted and, as such, it was not
incapacitated to enter into any contract or to apply for and accept a franchise. Not
having been incapacitated, Morong Electric maintains that the franchise granted to it
is valid and the approval or disapproval thereof can be properly determined by the
Commission.

ISSUE:

Whether the lack or corporate existence on the part of Morong rendered the
franchise valid.

RULING:

YES.

The incorporation of (Morong) and its acceptance of the franchise as shown by


this action in prosecuting the application filed with the Commission for approval of
said franchise, not only perfected a contract between the municipality and Morong but
also cured the deficiency pointed out by the petition. The fact that Morong did not
have a corporate existence on the day the franchise was granted does not render the
franchise invalid, as Morong later obtained its certificate of incorporation and accepted
the franchise.
The two decisions of the Public Service Commission, appealed from, should be,
as they are hereby affirmed, with costs in the two cases against petitioner Rizal Light
& Ice Co., Inc.

44 | P a g e
Law 321_Corporation LAW_ Case Digest

Articles of Incorporation

JESUS V. LANUZA, MAGADYA REYES, BAYANI REYES and ARIEL REYES


vs.
COURT OF APPEALS, SECURITIES AND EXCHANGE COMMISSION, DOLORES
ONRUBIA, ELENITA NOLASCO, JUAN O. NOLASCO III, ESTATE OF FAUSTINA M.
ONRUBIA, PHILIPPINE MERCHANT MARINE SCHOOL, INC.
G.R. No. 131394.March 28, 2005

FACTS:

Philippine Merchant Marine School, Inc. (PMMSI) had seven hundred founders‘
shares and seventy-six common shares as its initial capital stock subscription
reflected in the articles of incorporation. However, private respondents and their
predecessors who were in control of PMMSI registered the company‘s stock and
transfer book for the first time in 1978, recording thirty-three (33) common shares as
the only issued and outstanding shares of PMMSI.
Sometime in 1979, a special stockholders‘ meeting was called and held on the
basis of what was considered as a quorum of twenty-seven common shares,
representing more than two-thirds of the common shares issued and outstanding. In
1982, the heirs of one of the original incorporators, Juan Acayan, filed a petition with
the SEC for the registration of their property rights over one hundred (120) founders‘
shares and twelve (12) common shares owned by their father. The SEC held that the
heirs were entitled to the claimed shares and called for a special stockholders‘ meeting
to elect a new set of officers. As a result, the shares of Acayan were recorded in the
stock and transfer book.
A special stockholders‘ meeting was held to elect a new set of directors. Private
respondents thereafter filed a petition with the SEC questioning the validity of the 06
May 1992 stockholders‘ meeting, alleging that the quorum for the said meeting should
not be based on the 165 issued and outstanding shares as per the stock and transfer
book, but on the initial subscribed capital stock of seven hundred seventy-six (776)
shares, as reflected in the 1952 Articles of Incorporation.

ISSUE:

Whether or not the basis of quorum for a stockholders‘ meeting is the


outstanding capital stock as indicated in the articles of incorporation.

RULING:

YES.

The stock and transfer book of PMMSI cannot be used as the sole basis for
determining the quorum as it does not reflect the totality of shares which have been
subscribed, more so when the articles of incorporation show a significantly larger
amount of shares issued and outstanding as compared to that listed in the stock and
transfer book. A stock and transfer book is one which records the names and
addresses of all stockholders arranged alphabetically, the instalments paid and
unpaid on all stock for which subscription has been made, and the date of payment
thereof; a statement of every alienation, sale or transfer of stock made, the date thereof
and by and to whom made; and such other entries as may be prescribed by law.
To base the computation of quorum solely on the deficient stock and transfer
book, and completely disregarding the issued and outstanding shares as indicated in
the articles of incorporation would work injustice to the owners and/or successors in
interest of the said shares.
It is to be explained, that if at the onset of incorporation a corporation has 771
shares subscribed, the Stock and Transfer Book should likewise reflect 771 shares.
Any sale, disposition or even reacquisition of the company of its own shares, in which
it becomes treasury shares, would not affect the total number of shares in the Stock
and Transfer Book. All that will change are the entries as to the owners of the shares
but not as to the amount of shares already subscribed.

45 | P a g e
Law 321_Corporation LAW_ Case Digest

Corporate Name

FRANCISCO M. ALONSO, substituted by his heirs


vs.
CEBU COUNTRY CLUB, INC.
G.R. No. 130876.January 31, 2002

FACTS:

Petitioner died pendente lite and substituted by his legal heirs, a lawyer by
profession, the only son and sole heir of the late Tomas N. Alonso and Asuncion
Medalle, who died on June 16, 1962 and August 18, 1963, respectively. Cebu Country
Club, Inc. is a non-stock, non-profit corporation duly organized and existing under
Philippine Laws the purpose of which is to cater to the recreation and leisure of its
members.
Sometime in 1992, petitioner discovered documents and records of Friar Lands
Sale Certificate Register/Instalment Record Certificate No. 734, Sales Certificate No.
734 and Assignment of Sales Certificate showing that his father acquired Lot No. 727
of the Banilad Friar Lands Estate from the Government of the Philippine Islands in or
about the year 1911 in accordance with the Friar Lands Act (Act No. 1120). The
documents show that one Leoncio Alburo, the original vendee of Lot No. 727, assigned
his sales certificate to petitioner‘s father on December 18, 1911, who completed the
required instalment payments thereon under Act No. 1120 and was consequently
issued Patent No. 14353 on March 24, 1926. On March 27, 1926, the Director of
Lands, acting for and in behalf of the government, executed a final deed of sale in
favor of petitioner‘s father Tomas N. Alonso. It appears, however, that the deed was not
registered with the Register of Deeds because of lack of technical requirements, among
them the approval of the deed of sale by the Secretary of Agriculture and Natural
Resources, as required by law.

ISSUE:

Whether or not the Court of Appeals erred in sustaining respondent‘s claim of


ownership over Lot No. 727.

RULING:

YES.

Under Act No. 1120, which governs the administration and disposition of friar
lands, the purchase by an actual and bona fide settler or occupant of any portion of
friar land shall be "agreed upon between the purchaser and the Director of Lands,
subject to the approval of the Secretary of Agriculture and Natural Resources (mutatis
mutandis)."
The instruments do not bear the signature of the Director of Lands and the
Secretary of the Interior. They also do not bear the approval of the Secretary of
Agriculture and Natural Resources. The approval by the Secretary of Agriculture and
Commerce of the sale of friar lands is indispensable for its validity, hence, the absence
of such approval made the sale null and void ab initio. Necessarily, there can be no
valid titles issued on the basis of such sale or assignment. Consequently, petitioner
Francisco‘s father did not have any registered title to the land in question. Having
none, he could not transmit anything to his sole heir, petitioner Francisco Alonso or
the latter‘s heirs.
Hence, the Court ruled that neither Tomas N. Alonso nor his son Francisco M.
Alonso or the latter‘s heirs are the lawful owners of Lot No. 727 in dispute. Neither has
the respondent Cebu Country Club, Inc. been able to establish a clear title over the
contested estate. The reconstitution of a title is simply the re-issuance of a lost
duplicate certificate of title in its original form and condition. It does not determine or
resolve the ownership of the land covered by the lost or destroyed title. A reconstituted
title, like the original certificate of title, by itself does not vest ownership of the land or
estate covered thereby.

46 | P a g e
Law 321_Corporation LAW_ Case Digest

INDUSTRIAL REFRACTORIES CORPORATION OF THE PHILIPPINES,


vs.
COURT OF APPEALS, SECURITIES AND EXCHANGE COMMISSION and
REFRACTORIES CORPORATION OF THE PHILIPPINES
G.R. No. 122174.October 3, 2002

FACTS:

Refractories Corporation of the Philippines (RCP) is a corporation duly organized


for the purpose of engaging in the business of manufacturing, producing, selling,
exporting and otherwise dealing in any and all refractory bricks, its by-products and
derivatives. On June 22, 1977, it registered its corporate and business name with the
Bureau of Domestic Trade. IRCP was originally under the name "Synclaire
Manufacturing Corporation". It amended its Articles of Incorporation on August 23,
1985 to change its corporate name to "Industrial Refractories Corp. of the Philippines".
It is engaged in the business of manufacturing all kinds of ceramics and other
products, except paints and zincs. Both companies are the only local suppliers of
monolithic gunning mix.
Discovering that petitioner was using such corporate name, respondent RCP
filed on April 14, 1988 with the Securities and Exchange Commission (SEC) a petition
to compel petitioner to change its corporate name on the ground that its corporate
name is confusingly similar with that of petitioner‘s such that the public may be
confused or deceived into believing that they are one and the same corporation.

ISSUE:

Whether or not the corporate names are confusingly similar.

RULING:

YES.

"Refractories are structural materials used at high temperatures to industrial


furnaces. They are supplied mainly in the form of brick of standard sizes and of
special shapes. Refractories also include refractory cements, bonding mortars, plastic
firebrick, castables, ramming mixtures, and other bulk materials such as dead-burned
grain magneside, chrome or ground ganister and special clay."
As regards the first requisite, it has been held that the right to the exclusive use
of a corporate name with freedom from infringement by similarity is determined by
priority of adoption. In this case, respondent RCP was incorporated on October 13,
1976 and since then has been using the corporate name "Refractories Corp. of the
Philippines". Meanwhile, petitioner was incorporated on August 23, 1979 originally
under the name "Synclaire Manufacturing Corporation". It only started using the
name "Industrial Refractories Corp. of the Philippines" when it amended its Articles of
Incorporation on August 23, 1985, or nine (9) years after respondent RCP started
using its name. Thus, being the prior registrant, respondent RCP has acquired the
right to use the word "Refractories" as part of its corporate name.
Anent the second requisite, in determining the existence of confusing similarity
in corporate names, the test is whether the similarity is such as to mislead a person
using ordinary care and discrimination and the Court must look to the record as well
as the names themselves. Petitioner‘s corporate name is "Industrial Refractories Corp.
of the Phils.", while respondent‘s is "Refractories Corp. of the Phils." Obviously, both
names contain the identical words "Refractories", "Corporation" and "Philippines". The
only word that distinguishes petitioner from respondent RCP is the word "Industrial"
which merely identifies a corporation‘s general field of activities or operations. We need
not linger on these two corporate names to conclude that they are patently similar that
even with reasonable care and observation, confusion might arise. It must be noted
that both cater to the same clientele, i.e.¸ the steel industry. In fact, the SEC found
that there were instances when different steel companies were actually confused
between the two, especially since they also have similar product packaging.

47 | P a g e
Law 321_Corporation LAW_ Case Digest

ANG MGA KAANIB SA IGLESIA NG DIOS KAY KRISTO HESUS, HSK SA BANSANG
PILIPINAS INC.
vs.
IGLESIA NG DIOS KAY CRISTO JESUS, HALIGI AT SUHAY NG KATOTOHANAN
GR 137592, 12 DECEMBER 2001

FACTS:

The Iglesia ng Dios Kay Cristo Jesus, Haligi at Suhay ng Katotohanan (IDCJ-
HSK; Church of God in Christ Jesus, the Pillar and Ground of Truth), is a non-stock
religious society or corporation registered in 1936. Sometime in 1976, one Eliseo
Soriano and several other members of said corporation disassociated themselves from
the latter and succeeded in registering on 30 March 1977 a new non-stock religious
society or corporation, named Iglesia ng Dios Kay Kristo Hesus, Haligi at Saligan ng
Katotohanan (IDKJ-HSK). On 16 July 1979, IDCJ-HSK filed with the SEC a petition to
compel IDKJ-HSK to change its corporate name.
The SEC rendered judgment in favor of IDCJ-HSK, ordering IDKJ-HSK to
change its corporate name to another name that is not similar or identical to any
name already used by a corporation, partnership or association registered with the
Commission.

ISSUE:

Whether the corporate names of AK[IDKH-HSK]BP and IDCH-HSK are


confusingly similar.

RULING:

YES.

The additional words "Ang Mga Kaanib " and "Sa Bansang Pilipinas, Inc." in
AK[IDKH-HSK]BP's name are merely descriptive of and also referring to the members,
or kaanib, of IDCH-HSK who are likewise residing in the Philippines. These words can
hardly serve as an effective differentiating medium necessary to avoid confusion or
difficulty in distinguishing AK[IDKH-HSK]BP from IDCH-HSK. This is especially so,
since both AK[IDKH-HSK]BP and IDCH-HSK are using the same acronym — H.S.K.;
not to mention the fact that both are espousing religious beliefs and operating in the
same place. Parenthetically, it is well to mention that the acronym H.S.K. used by
AK[IDKH-HSK]BP stands for "Haligi at Saligan ng Katotohanan." Then, too, the records
reveal that in holding out their corporate name to the public, AK[IDKH-HSK]BP
highlights the dominant words "IGLESIA NG DIOS KAY KRISTO HESUS, HALIGI AT
SALIGAN NG KATOTOHANAN," which is strikingly similar to IDCH-HSK's corporate
name, thus making it even more evident that the additional words "Ang Mga Kaanib"
and "Sa Bansang Pilipinas, Inc.", are merely descriptive of and pertaining to the
members of IDCH-HSK. Significantly, the only difference between the corporate names
of AK[IDKH-HSK]BP and IDCH-HSK are the words SALIGAN and SUHAY. These words
are synonymous — both mean ground, foundation or support. Hence, the Court ruled
that the corporate names Universal are indisputably so similar that even under the
test of "reasonable care and observation" confusion may arise.
The wholesale appropriation by AK[IDKH-HSK]BP of IDCH-HSK's corporate
name cannot find justification under the generic word rule. A contrary ruling would
encourage other corporations to adopt verbatim and register an existing and protected
corporate name, to the detriment of the public. The fact that there are other non-stock
religious societies or corporations using the names Church of the Living God, Inc.,
Church of God Jesus Christ the Son of God the Head, Church of God in Christ & By
the Holy Spirit, and other similar names, is of no consequence. It does not authorize
the use by AK[IDKH-HSK]BP of the essential and distinguishing feature of IDCH-HSK's
registered and protected corporate name.

48 | P a g e
Law 321_Corporation LAW_ Case Digest

UNIVERSAL MILLS CORPORATION


vs.
UNIVERSAL TEXTILE MILLS, INC.
G.R. No. L-28351, July 28, 1977

FACTS:

Universal Textile Mills, Inc. was organized on December 29, 1953, as a textile
manufacturing firm for which it was issued a certificate of registration on January 8,
1954. The Universal Mills Corporation, on the other hand, was registered with the
Commission on October 27, 1954, under its original name, Universal Hosiery Mills
Corporation, having as its primary purposes the "manufacture and production of
hosieries and wearing apparel of all kinds." On May 24, 1963, it filed an amendment
to its articles of incorporation changing its name to Universal Mills Corporation, its
present name, for which it was issued the certificate of approval on June 10, 1963.
The immediate cause of this complaint was the occurrence of a fire which
gutted petitioner‘s spinning mills in Pasig, Rizal. Universal Textile Mills, Inc. alleged
that as a result of this fire and because of the similarity of petitioner's name to that of
the former, the news items appearing in the various metropolitan newspapers carrying
reports on the fire created uncertainty and confusion among its bankers, friends,
stockholders and customers prompting respondent to make announcements,
clarifying the real Identity of the corporation whose property was burned.
The Commission then issued an order enjoining Universal Mills Corporation
from using its present corporate name because it is confusingly and deceptively
similar with Universal Textile Mills, Inc.

ISSUE:

Whether or not the order of the SEC is proper.

RULING:

YES.

The corporate names in question are not Identical, but they are indisputably so
similar that even under the test of "reasonable care and observation as the public
generally are capable of using and may be expected to exercise" invoked by appellant.
The Supreme Court ruled that confusion will usually arise, considering that under the
second amendment of its articles of incorporation on August 14, 1964, appellant
included among its primary purposes the "manufacturing, dyeing, finishing and selling
of fabrics of all kinds" in which respondent had been engaged for more than a decade
ahead of petitioner.
And since respondent is not claiming damages in this proceeding, it is, of
course, immaterial whether or not appellant has acted in good faith, but the SC
cannot perceive why of all names, petitioner had to choose a name already being used
by another firm engaged in practically the same business for more than a decade
enjoying well-earned patronage and goodwill, when there are so many other
appropriate names it could possibly adopt without arousing any suspicion as to its
motive and, more importantly, any degree of confusion in the mind of the public which
could mislead even its own customers, existing or prospective.

49 | P a g e
Law 321_Corporation LAW_ Case Digest

LYCEUM OF THE PHILIPPINES, INC.


vs.
COURT OF APPEALS, ET AL.
G.R. No. 101897 March 5, 1993

FACTS:

Lyceum of the Philippines is an educational institution duly registered with the


Securities and Exchange Commission. Petitioner instituted proceedings before the
SEC to compel the private respondents, which are also educational institutions, to
delete the word "Lyceum" from their corporate names and permanently to enjoin them
from using "Lyceum" as part of their respective names. The SEC hearing officer
rendered a decision sustaining petitioner's claim to an exclusive right to use the word
"Lyceum." The hearing officer relied upon the SEC ruling in the Lyceum of Baguio, Inc.
case and held that the word "Lyceum" was capable of appropriation and that petitioner
had acquired an enforceable exclusive right to the use of that word.
On appeal, however, by private respondents the SEC En Banc did not consider
the word "Lyceum" to have become so identified with petitioner as to render use
thereof by other institutions as productive of confusion about the identity of the
schools concerned in the mind of the general public. Unlike its hearing officer, the
SEC En Banc held that the attaching of geographical names to the word "Lyceum"
served sufficiently to distinguish the schools from one another, especially in view of
the fact that the campuses of petitioner and those of the private respondents were
physically quite remote from each other.

ISSUE:

Whether or not the word Lyceum has not acquired a secondary meaning.

RULING:

NO.

The Articles of Incorporation of a corporation must, among other things, set out
the name of the corporation. The policy underlying the prohibition in Section 18
against the registration of a corporate name which is "identical or deceptively or
confusingly similar" to that of any existing corporation or which is "patently deceptive"
or "patently confusing" or "contrary to existing laws," is the avoidance of fraud upon
the public which would have occasion to deal with the entity concerned, the evasion of
legal obligations and duties, and the reduction of difficulties of administration and
supervision over corporations.
"Under the doctrine of secondary meaning, a word or phrase originally
incapable of exclusive appropriation with reference to an article in the market,
because geographical or otherwise descriptive might nevertheless have been used so
long and so exclusively by one producer with reference to this article that, in that
trade and to that group of the purchasing public, the word or phrase has come to
mean that the article was his produce. The appellant failed to satisfy the requisites. No
evidence was ever presented in the hearing before the Commission which sufficiently
proved that the word 'Lyceum' has indeed acquired secondary meaning in favor of the
appellant.

50 | P a g e
Law 321_Corporation LAW_ Case Digest

INDIANA AEROSPACE UNIVERSITY


vs.
COMMISSION ON HIGHER EDUCATION
G.R. NO. 139371, 04 APRIL 2001

FACTS:

In 1996, Dr. Reynaldo B. Vera, Chairman, Technical Panel for Engineering,


Architecture, and Maritime Education (TPRAM) of [CHED], received a letter dated
October 18, 1998 (Annex 'C') from Douglas R. Macias, Chairman, Board of
Aeronautical Engineering, Professional Regulatory Commission (PRC) and Chairman,
Technical Committee for Aeronautical Engineering (TPRAME) inquiring whether
petitioner had already acquired University status in view of the latter's advertisement
in the Manila Bulletin.
In a letter dated October 24, 1996, Dr. Vera formally referred the aforesaid
letter to Chairman Alcala with a request that the concerned Regional Office of [CHED]
be directed to conduct appropriate investigation on the alleged misrepresentation by
petitioner. Thereafter, CHED referred the matter to its Regional Director in Cebu City,
requesting said office to conduct an investigation and submit its report. The Report
stated that there was a violation [committed by] his institution when it used the term
university unless the school had complied [with] the basic requirement of being a
university as prescribed in CHED Memorandum Order No. 48, s. 1996.'

ISSUE:

Whether or not the Petitioner can use the word ―University‖.

RULING:

YES.

The Court found that there was no grave abuse of discretion in the RTC's denial
of the Motion to Dismiss, as contained in the August 14, 1998 Order. The CA erred in
ruling otherwise. The trial court stated in its Decision that petitioner was an
educational institution, originally registered with the Securities and Exchange
Commission as the "Indiana School of Aeronautics, Inc." That name was subsequently
changed to "Indiana Aerospace University" after the Department of Education, Culture
and Sports had interposed no objection to such change.
Respondent issued a formal Cease and Desist Order directing petitioner to stop
using the word "university" in its corporate name. The former also published an
announcement in the March 21, 1998 issue of Freeman, a local newspaper in Cebu
City, that there was no institution of learning by that name. The counsel of respondent
was quoted as saying in the March 28, 1998 issue of the newspaper Today that
petitioner had been ordered closed by the respondent for illegal advertisement, fraud
and misrepresentation of itself as a university. Such acts, according to the RTC
undermined the public's confidence in petitioner as an educational institution. This
was a clear statement of a sufficient cause of action.
When a motion to dismiss is grounded on the failure to state a cause of action,
a ruling thereon should be based only on the facts alleged in the complaint. The court
must pass upon this issue based solely on such allegations, assuming them to be
true. For it to do otherwise would be a procedural error and a denial of plaintiff's right
to due process.

51 | P a g e
Law 321_Corporation LAW_ Case Digest

PHILIPS EXPORT B.V., PHILIPS ELECTRICAL LAMPS,INC. and PHILIPS


INDUSTRIAL DEVELOPMENT, INC.
vs.
COURT OF APPEALS, SECURITIES & EXCHANGE COMMISSION and STANDARD
PHILIPS CORPORATION
G.R. No. 96161. February 21, 1992

FACTS:

Petitioner is a foreign corporation organized under the laws of Netherlands


although not engaged in any business here in the Philippines, is the registered owner
of the trademarks Philips and Philips Shield Emblem. Petitioners Philips Electrical
Lamps, Inc, and Philips Industrial Development Inc are the authorized users of the
trademark of petitioner Philips BV. All petitioner corporations belong to the Philips
Group of Companies.
Private respondent Standard Philips Corporation was issued a certificate of
registration by the respondent Securities and Exchange Commission. Petitioner filed
a letter complaint with the SEC asking for the cancellation of the word ―Philips‖ from
private respondent‘s corporate name in view of its prior registration with the Bureau of
Patent alleging that private respondent‘s use of the word Philips amounts to an
infringement and clear violation of petitioner‘s exclusive right to use the same
considering that both parties are engaged in the same business.

ISSUE:

Whether or not Standard Philips‘ use of the word PHILIPS amounts to an


infringement and clear violation of Petitioner's exclusive right to use the same
considering that both parties engage in the same business.

RULING:

YES.

The requisite no less exists in this case. In determining the existence of


confusing similarity in corporate names, the test is whether the similarity is such as to
mislead a person using ordinary care and discrimination. In so doing, the Court must
look to the record as well as the names themselves. While the corporate names of
Petitioners and Private Respondent are not identical, a reading of Petitioner's corporate
names, to wit: PHILIPS EXPORT B.V., PHILIPS ELECTRICAL LAMPS, INC. and
PHILIPS INDUSTRIAL DEVELOPMENT, INC., inevitably leads one to conclude that
"PHILIPS" is, indeed, the dominant word in that all the companies affiliated or
associated with the principal corporation, PEBV, are known in the Philippines and
abroad as the PHILIPS Group of Companies.
It is settled that proof of actual confusion need not be shown. It suffices that
confusion is probably or likely to occur. Under the Guidelines in the Approval of
Corporate and Partnership Names formulated by the SEC, the proposed name "should
not be similar to one already used by another corporation or partnership. If the
proposed name contains a word already used as part of the firm name or style of a
registered company, the proposed name must contain two other words different from
the company already registered". Private Respondents' name, however, contains only
a single word, that is, "STANDARD", different from that of Petitioners inasmuch as the
inclusion of the term "Corporation" or "Corp." merely serves the purpose of
distinguishing the corporation from partnerships and other business organizations.
It is obvious that private respondent‘s choice of Philips as part of its corporate
name tends to show said respondent‘s intention to ride on the popularity and
established goodwill of the said petitioner‘s business throughout the world. The
subsequent appropriator of the name or one-confusingly similar thereto usually seeks
an unfair advantage, a free ride on another‘s goodwill. Besides, there is showing that
private respondent not only manufactured and sold ballasts for fluorescent lamps with
their corporate name printed thereon but also advertised the same as Standard
Philips.

52 | P a g e
Law 321_Corporation LAW_ Case Digest

Primary Purpose

ALICIA E. GALA, GUIA G. DOMINGO and RITA G. BENSON


vs.
ELLICE AGRO-INDUSTRIAL CORPORATION, MARGO MANAGEMENT AND
DEVELOPMENT CORPORATION, RAUL E. GALA, VITALIANO N. AGUIRRE II,
ADNAN V. ALONTO, ELIAS N. CRESENCIO, MOISES S. MANIEGO, RODOLFO B.
REYNO, RENATO S. GONZALES, VICENTE C. NOLAN, NESTOR N. BATICULON
G.R. No. 156819. December 11, 2003

FACTS:

On March 28, 1979, the Ellice Agro-Industrial Corporation was formed and
organized. The total subscribed capital stock of the corporation was P3.5 Million with
35,000 shares. Additional shares were acquired and subscribed from said
corporation. Subsequently, on September 16, 1982, the Margo Management and
Development Corporation (Margo) was incorporated. The total subscribed capital
stock of Margo was 20,000 shares at P200, 000.00. Several transfers of shares of
Ellice to Margo were made by the stockholders and some payments of subscription
were made by transferring parcels of land by the Gala Spouses.
In essence, petitioners want this Court to disregard the separate juridical
personalities of Ellice and Margo for the purpose of treating all property purportedly
owned by said corporations as property solely owned by the Gala spouses. The
petitioners‘ contention in support of this theory is that the purposes for which Ellice
and Margo were organized should be declared as illegal and contrary to public policy.
They claim that the respondents never pursued exemption from land reform coverage
in good faith and instead merely used the corporations as tools to circumvent land
reform laws and to avoid estate taxes. Specifically, they point out that respondents
have not shown that the transfers of the land in favor of Ellice were executed in
compliance with the requirements of Section 13 of R.A. 3844. Furthermore, they
alleged that respondent corporations were run without any of the conventional
corporate formalities.

ISSUE:

Whether or not the purpose of the creation of the two corporations is illegal and
against public policy.

RULING:

NO.

Impugning the legality of the purposes for which Ellice and Margo were
organized, amount to collateral attacks which are prohibited in this jurisdiction. The
best proof of the purpose of a corporation is its articles of incorporation and by-laws.
The articles of incorporation must state the primary and secondary purposes of the
corporation, while the by-laws outline the administrative organization of the
corporation, which, in turn, is supposed to insure or facilitate the accomplishment of
said purpose. A perusal of the Articles of Incorporation of Ellice and Margo shows no
sign of the allegedly illegal purposes that petitioners are complaining of. If a
corporation‘s purpose, as stated in the Articles of Incorporation, is lawful, then the
SEC has no authority to inquire whether the corporation has purposes other than
those stated, and mandamus will lie to compel it to issue the certificate of
incorporation.
With regard to their claim that Ellice and Margo were meant to be used as mere
tools for the avoidance of estate taxes, suffice it say that the legal right of a taxpayer to
reduce the amount of what otherwise could be his taxes or altogether avoid them, by
means which the law permits, cannot be doubted.
Thus, even if Ellice and Margo were organized for the purpose of exempting the
properties of the Gala spouses from the coverage of land reform legislation and
avoiding estate taxes, the court cannot disregard their separate juridical personalities.

53 | P a g e
Law 321_Corporation LAW_ Case Digest

HEIRS OF ANTONIO PAEL and ANDREA ALCANTARA and CRISANTO PAEL


vs.
COURT OF APPEALS, JORGE H. CHIN and RENATO B. MALLARI
G.R. No. 133547.December 7, 2001

FACTS:

PFINA acquired the properties from the Heirs of Pael by virtue of a deed of
assignment dated January 25, 1983. It filed a motion to intervene before the Court of
Appeals; however, before it filed its motion for intervention, or for a long period of
fifteen (15) years, PFINA and the Heirs of Pael were totally silent about the alleged deed
of assignment. No steps were taken by either of them to register the deed or secure
transfer certificate of title evidencing the change of ownership during this long period
of time.
At the time PFINA acquired the disputed properties in 1983, its corporate name
was PFINA Mining and Exploration, Inc., a mining company which had no valid
grounds to engage in the highly speculative business of urban real estate
development.

ISSUE:

Whether or not the 1983 transfer produces legal effect.

RULING:

NO.

As correctly ruled by the courts, the alleged transfer in 1983 was not only
dubious and fabricated; it could produce no legal effect as the Paels were no longer
owners of the land they allegedly assigned.
The Court highlighted the citation in the comment of Intervenor U.P.,
specifically citing the decision in Roberto A. Pael et al. v. Court of Appeals, et al.,
supra, wherein the title of the Paels was declared to be of dubious origin and a
fabrication. Hence, since respondents derive their titles from a defective title, their
titles should also be null and void.
The motion for intervention of the University of the Philippines is GRANTED.
The case is REMANDED to the Court of Appeals for reception of evidence on the
conflicting claims over the property covered by TCT Nos. 52928 and 52929 between
the intervernor University of the Philippines, on the one hand, and respondents Jorge
H. Chin and Renato B. Mallari, on the other hand. The motions for reconsideration
filed by petitioners are DENIED for lack of merit. This denial is FINAL and no further
pleadings from petitioners will be entertained.

54 | P a g e
Law 321_Corporation LAW_ Case Digest

UY SIULIONG, MARIANO LIMJAP, GACU UNG JIENG, EDILBERTO CALIXTO and


UY CHO YEE
vs.
THE DIRECTOR OF COMMERCE AND INDUSTRY
G.R. No.L-15429. December 1, 1919

FACTS:

Prior to the presentation of the petition the petitioners had been associated
together as partners, which partnership was known as "mercantil regular colectiva,
under the style and firm name of "Siuliong y Cia. That the petitioners herein, who had
theretofore been members of said partnership of "Siuliong y Cia.," desired to dissolve
said partnership and to form a corporation composed of the same persons as
incorporators, to be known as "Siulong y Compañia, Incorporada.
While the articles of incorporation of "Siuliong y Cia., Inc." states that its
purpose is to acquire and continue the business, with some of its objects or purposes,
of Siuliong & Co., it will be found upon an examination of the purposes enumerated in
the proposed articles of incorporation of "Siuliong y Cia., Inc.," that some of the
purposes of the original partnership of "Siuliong y Cia." have been omitted.

ISSUE:

Whether or not a corporation can engage in other purposes other than that
stated in the purpose clause of its articles of incorporation.

RULING:

YES.

A corporation may be organized under the laws of the Philippine Islands for
mercantile purposes, and to engage in such incidental business as may be necessary
and advisable to give effect to, and aid in, the successful operation and conduct of the
principal business. All of the power and authority included in the articles of
incorporation of "Siuliong y Cia., Inc.," enumerated above in paragraph 4 of the
Articles of Incorporation are only incidental to the principal purpose of said proposed
incorporation, to wit: "mercantile business." The purchase and sale, importation and
exportation of the products of the country, as well as of foreign countries, might make
it necessary to purchase and discount promissory notes, bills of exchange, bonds,
negotiable instruments, stock, and interest in other mercantile and industrial
associations. It might also become important and advisable for the successful
operation of the corporation to act as agent for insurance companies as well as to buy,
sell and equip boats and to buy and sell other establishments, and industrial and
mercantile businesses. The proposed articles of incorporation do not authorize the
petitioners to engage in a business with more than one purpose, the Court do not
mean to be understood as having decided that corporations under the laws of the
Philippine Islands may not engage in a business with more than one purpose. Such
an interpretation might work a great injustice to corporations organized under the
Philippine laws. Such an interpretation would give foreign corporations, which are
permitted to be registered under the laws here and which may be organized for more
than one purpose, a great advantage over domestic corporations. It was not the
intention of the legislature to give foreign corporations such an advantage over
domestic corporations.

55 | P a g e
Law 321_Corporation LAW_ Case Digest

NORBERTO ASUNCION, ET AL.


vs.
MANUEL DE YRIARTE
G.R. No. 9321.September 24, 1914

FACTS:

This is an action to obtain a writ of mandamus to compel the Chief of the


Division of Achieves of the Executive Bureau to file a certain articles of incorporation.
The Chief of the Division of Archives refused to file the articles of incorporation upon
the ground that the object of the corporation, as stated in the articles, was not lawful
and that, in pursuance of section 6 of Act No. 1459, they were not registerable.
The proposed incorporators began an action in the CFI of Manila to compel the
Chief of the Division of Archives to receive and register said articles of incorporation
and to do any and all acts necessary for the complete incorporation of the persons
named in the articles.

ISSUE:

Whether or not the purposes of the corporation as stated in the articles of


incorporation are lawful within the meaning of the Corporation Law.

RULING:

YES.

When on the face of the articles of incorporation presented for registration it is


shown that it is organized for a purpose contrary to law or public policy, the same may
be denied outright registration.
The object of the proposed corporation, as appears from the articles offered for
registration, is to make of the barrio of Pulo or San Miguel a corporation which will
become the owner of and have the right to control and administer any property
belonging to the municipality of Pasig found within the limits of that barrio. This
clearly cannot be permitted. Otherwise municipalities as now established by law could
be deprived of the property which they now own and administer. Each barrio of the
municipality would become under the scheme proposed, a separate corporation, would
take over the ownership, administration, and control of that portion of the municipal
territory within its limits. This would disrupt, in a sense, the municipalities of the
Islands by dividing them into a series of smaller municipalities entirely independent of
the original municipality. The object of the proposed corporation is clearly repugnant
to the provisions of the Municipal Code and the governments of municipalities as they
have been organized thereunder.

56 | P a g e
Law 321_Corporation LAW_ Case Digest

Principal Office/Domicile

DAVAO LIGHT & POWER CO., INC.


vs.
THE HON. COURT OF APPEALS, HON. RODOLFO M. BELLAFLOR, Presiding
Judge of Branch 11, RTC-Cebu and FRANCISCO TESORERO
G.R. No. 111685.August 20, 2001

FACTS:

In 1992 Davao Light & Power Co., Inc. filed a complaint for damages against
private respondent Francisco Tesorero before the Regional Trial Court of Cebu for
damages in the amount of P11, 000,000.00. In turn, the latter filed a motion to
dismiss claiming among others that the venue was improperly laid since the principal
place of business of the plaintiff is Davao City as indicated in the lease executed by
petitioner, and the same determines the venue of the action, instead of Banilad City
which the company indicated in its complaint. The trial court granted the said motion.
Petitioner‘s motion for reconsideration was denied, as well as its appeal to the Court of
Appeals. Hence, this petition.

ISSUE:

Whether or not the company‘s principal place is in Davao City.

RULING:

YES.

Davao City is the Principal place of business which determines venue. A


corporation has no residence in the same sense in which this term is applied to a
natural person. But for practical purposes, a corporation is in a metaphysical sense a
resident of the place where its principal office is located as stated in the articles of
incorporation. The Corporation Code precisely requires each corporation to specify in
its articles of incorporation the "place where the principal office of the corporation is to
be located which must be within the Philippines". The purpose of this requirement is
to fix the residence of a corporation in a definite place, instead of allowing it to be
ambulatory.
The same considerations apply to the instant case. It cannot be disputed that
petitioner's principal office is in Cebu City, per its amended articles of incorporation
and by-laws. However, Tesorero is not a party to any of the contracts presented before
the court. Those documents were between the petitioner and NAPOCOR and therefore
estoppel may not lie against the private respondent. He is a stranger to those
documents even if he says that by being a member of the public for whose benefit the
electric generating contracts were entered into. There is no estoppel because there is
no showing that he relied on the representations made by the petitioner.

57 | P a g e
Law 321_Corporation LAW_ Case Digest

CLAVECILLIA RADIO SYSTEM


vs.
HON. AGUSTIN ANTILLON, as City Judge of the Municipal Court of Cagayan de
Oro Cityand NEW CAGAYAN GROCERY
G.R. No.L-22238.February 18, 1967

FACTS:

On June 22, 1963, the New Cagayan Grocery filed a complaint against the
Clavecilla Radio System alleging, in effect, that on March 12, 1963, the following
message, addressed to the former, was filed at the latter's Bacolod Branch Office for
transmittal thru its branch office at Cagayan de Oro: ―NECAGRO CAGAYAN DE ORO
(CLAVECILLA): REURTEL WASHED NOT AVAILABLE REFINED TWENTY FIFTY IF
AGREEABLE SHALL SHIP LATER REPLY POHANG.‖ The Cagayan de Oro branch office
having received the said message omitted, in delivering the same to the New Cagayan
Grocery, the word "NOT" between the words "WASHED" and "AVAILABLE," thus
changing entirely the contents and purport of the same and causing the said
addressee to suffer damages. After service of summons, the Clavecilla Radio System
filed a motion to dismiss the complaint on the grounds that it states no cause of action
and that the venue is improperly laid. The New Cagayan Grocery interposed an
opposition to which the Clavecilla Radio System filed its rejoinder. Thereafter, the City
Judge, on September 18, 1963, denied the motion to dismiss for lack of merit and set
the case for hearing.
Hence, the Clavecilla Radio System filed a petition for prohibition with
preliminary injunction with the Court of First Instance praying that the City Judge,
Honorable Agustin Antillon, be enjoined from further proceeding with the case on the
ground of improper venue. The respondents filed a motion to dismiss the petition but
this was opposed by the petitioner. Later, the motion was submitted for resolution on
the pleadings.

ISSUE:

Whether or not the place is the proper venue to sue Clavecilla Radio System?

RULING:

NO.

In this case, the suit for damages filed with the city court is based upon tort
and not upon a written contract. Section 1 of Rule 4 of the New Rules of Court,
governing venue of actions in inferior courts, provides in its paragraph (b)(3) that when
"the action is not upon a written contract, then in the municipality where the
defendant or any of the defendants resides or may be served with summons." Settled
is the principle in corporation law that the residence of a corporation is the place
where its principal office is established. Since it is not disputed that the Clavecilla
Radio System has its principal office in Manila, it follows that the suit against it may
properly be filed in the City of Manila.
The appellee maintain, however, that with the filing of the action in Cagayan de
Oro City, venue was properly laid on the principle that the appellant may also be
served with summons in that city where it maintains a branch office. The term "may
be served with summons" does not apply when the defendant resides in the
Philippines for, in such case, he may be sued only in the municipality of his residence,
regardless of the place where he may be found and served with summons. As any
other corporation, the Clavecilla Radio System maintains a residence which is Manila
in this case, and a person can have only one residence at a time (See Alcantara vs.
Secretary of the Interior, 61 Phil. 459; Evangelists vs. Santos, 86 Phil. 387). The fact
that it maintains branch offices in some parts of the country does not mean that it can
be sued in any of these places. To allow an action to be instituted in any place where
a corporate entity has its branch offices would create confusion and work untold
inconvenience to the corporation.

58 | P a g e
Law 321_Corporation LAW_ Case Digest

JOHN SY and UNIVERSAL PARTS SUPPLY CORPORATION


vs.
TYSON ENTERPRISES, INC., JUDGE GREGORIO G. PINEDA of the Court of First
Instance of Rizal, Pasig Branch XXI and COURT OF APPEALS
G.R. No.L-56763. December 15, 1982

FACTS:

On August 29, 1979, Tyson Enterprises, Inc. filed against John Sy and
Universal Parts Supply Corporation, residents of Bacolod, a complaint for the
collection of money in Pasig, Rizal. However, there is no allegation in the complaint as
to the office or place of business of plaintiff Tyson Enterprises, Inc., which is located in
Manila. What is alleged is the postal address or residence of Dominador Ti, the
president and general manager of plaintiff firm, which is in San Juan, Rizal.
Defendant Sy and Universal Parts Supply Corporation filed a motion to dismiss
on the ground of improper venue. The plaintiff opposed the motion to dismiss which
the trial court denied. On appeal, the Appellate Court dismissed the petition. It ruled
that the parties did not intend Manila as the exclusive venue of the actions arising
under their transactions and that since the action was filed in Pasig, which is near
Manila, no useful purpose would be served by dismissing the same and ordering that
it be filed in Manila.

ISSUE:

Whether or not venue was improperly laid in this case.

RULING:

YES.

The place of business of plaintiff Tyson Enterprises, Inc., which for purposes of
venue is considered as its residence is in Manila and not in Rizal. The residence of its
president is not the residence of the corporation because a corporation has a
personality separate and distinct from that of its officers and stockholders.
Consequently, the collection suit should have been filed in Manila, the residence of
plaintiff corporation and the place designated in its sales invoice, or it could have been
filed also in Bacolod City, the residence of defendant Sy.
The decision of the Court of Appeals and the order of respondent judge denying
the motion to dismiss are reversed and set aside. The writ of prohibition is granted.
Civil Case No. 34302 should be considered dismissed without prejudice to refiling it in
the Court of First Instance of Manila or Bacolod City at the election of plaintiff which
should be allowed to withdraw the documentary evidence submitted in that case. All
the proceedings in said case, including the decision, were also set aside.
The decision of the Court of Appeals and the order of respondent judge denying
the motion to dismiss are reversed and set aside.

59 | P a g e
Law 321_Corporation LAW_ Case Digest

YOUNG AUTO SUPPLY CO. AND NEMESIO GARCIA


vs.
THE HONORABLE COURT OF APPEALS (THIRTEENTH DIVISION) AND GEORGE
CHIONG ROXAS
G.R. No. 104175. June 25, 1993

FACTS:

On October 28, 1987, Young Auto Supply Co. Inc. (YASCO) represented by
Nemesio Garcia, its president, Nelson Garcia and Vicente Sy, sold all of their shares of
stock in Consolidated Marketing & Development Corporation (CMDC) to Roxas. The
purchase price was P8,000,000.00 payable as follows: a down payment of
P4,000,000.00 and the balance of P4,000,000.00 in four postdated checks of
P1,000,000.00 each. The first check of P4, 000,000.00, representing the down
payment, was honored by the drawee bank but the four other checks representing the
balance of P4, 000,000.00 were dishonored.
On June 10, 1988, petitioners filed a complaint against Roxas in the Regional
Trial Court, Branch 11, Cebu City, praying that Roxas be ordered to pay petitioners
the sum of P3, 400,000.00 or that full control of the three markets be turned over to
YASCO and Garcia. The complaint also prayed for the forfeiture of the partial payment
of P4, 600,000.00 and the payment of attorney's fees and costs.

ISSUE:

Whether the proper venue is in Pasay City.

RULING:

NO.

The Court of Appeals erred in holding that the venue was improperly laid in
Cebu City. Young Auto Supply Co., Inc. ("YASCO") is a domestic corporation duly
organized and existing under Philippine laws with principal place of business at M.J.
Cuenco Avenue, Cebu City. It also has a branch office at 1708 Dominga Street, Pasay
City, Metro Manila. The Article of Incorporation of YASCO states that the place where
the principal office of the corporation is to be established or located is at Cebu City,
Philippines.
A corporation has no residence in the same sense in which this term is applied
to a natural person. But for practical purposes, a corporation is in a metaphysical
sense a resident of the place where its principal office is located as stated in the
articles of incorporation. The Corporation Code precisely requires each corporation to
specify in its articles of incorporation the "place where the principal office of the
corporation is to be located which must be within the Philippines" The purpose of this
requirement is to fix the residence of a corporation in a definite place, instead of
allowing it to be ambulatory. With the finding that the residence of YASCO for
purposes of venue is in Cebu City, where its principal place of business is located, it
becomes unnecessary to decide whether Garcia is also a resident of Cebu City and
whether Roxas was in estoppel from questioning the choice of Cebu City as the venue.
Hence, it should be in Cebu City.

60 | P a g e
Law 321_Corporation LAW_ Case Digest

Term

ALHAMBRA CIGAR & CIGARETTE MANUFACTURING COMPANY, INC.


vs.
SECURITIES & EXCHANGE COMMISSION
G.R. No. L-23606.July 29, 1968

FACTS:

Incorporated under Philippine laws on January 15, 1912, petitioner Alhambra


Cigars Mfg. Co (ACCMI) was to exist for fifty (50) years from incorporation. Its term of
existence expired on January 15, 1962. On that date, it ceased transacting business
and entered into a state of liquidation.
Thereafter, a new corporation — Alhambra Industries, Inc. was formed to carry
on the business of Alhambra. On May 1, 1962, Alhambra's stockholders, by
resolution, named Angel S. Gamboa trustee to take charge of its liquidation. On June
20, 1963,within Alhambra's three-year statutory period for liquidation — Republic Act
3531 was enacted into law amending Section 18 of the Corporation Law and enabling
domestic private corporations to extend their corporate life beyond the period fixed by
the articles of incorporation for a term not to exceed fifty years in any one instance.
On July 15, 1963 Alhambra's board of directors resolved to amend paragraph
"Fourth" of its articles of incorporation to extend its corporate life for an additional fifty
years, or a total of 100 years from its incorporation. Its stockholders, representing
more than two-thirds of its subscribed capital stock, voted to approve the foregoing
resolution. SEC, however, returned said amended articles of incorporation with the
ruling that RA 3531 which took effect only on June 20, 1963, cannot be availed of by
the said corporation, for the reason that its term of existence had already expired
when the said law took effect; in short, said law has no retroactive effect."

ISSUE:

Whether or not a corporation may extend its life by amendment of its articles of
incorporation effected during the three-year statutory period for liquidation when its
original term of existence had already expired.

RULING:

NO.

Provided by Section 77 of the Corporation Law, the continuance of a "dissolved"


corporation as a body corporate for three years has for its purpose the final closure of
its affairs, and no other; the corporation is specifically enjoined from continuing the
business for which it was established. The liquidation of the corporation's affairs set
forth in Section 77 became necessary precisely because its life had ended. For this
reason alone, the corporate existence and juridical personality of that corporation to
do business may no longer be extended. The provisions of RA 3531 merely empower a
corporation to act in liquidation, and not to extend its corporate existence.

61 | P a g e
Law 321_Corporation LAW_ Case Digest

Paid-up Capital Stock

MSCI-NACUSIP Local Chapter


vs.
NATIONAL WAGES AND PRODUCTIVITY COMMISSION and MONOMER SUGAR
CENTRAL, INC.
G.R. No. 125198. March 3, 1997

FACTS:

On January 11, 1990, Asturias Sugar Central, Inc. (ASCI), executed a


Memorandum of Agreement with Monomer Trading Industries, Inc. (MTII), whereby
MTII shall acquire the assets of ASCI by way of a Deed of Assignment provided that an
entirely new organization in place of MTII shall be organized, which new corporation
shall be the assignee of the assets of ASCI.
By virtue of this Agreement, a new corporation was organized and incorporated
on February 15, 1990 under the corporate name Monomer Sugar Central, Inc. or
MSCI, the private respondent herein. On January 16, 1991, MSCI applied for
exemption from the coverage of Wage Order No. RO VI-01 issued by the Board on the
ground that it is a distressed employer.
On January 16, 1991, MSCI applied for exemption from the coverage of Wage
Order No. RO VI-01 issued by the Board on the ground that it is a distressed
employer. In support thereto, MSCI submitted its audited financial statements and
income tax returns duly stamped "received" by the Bureau of Internal Revenue (BIR)
and the Securities and Exchange Commission (SEC) for the period beginning February
15, 1990 and ending August 31, 1990, including the quarterly financial statements
and income tax returns for the two quarters ending November 30, 1990 and February
28, 1991.

ISSUE:

Whether or not the correct paid-up capital of MSCI for the pertinent period
covered by the application for exemption is P5 million, not P64,688,528.00.

RULING:

YES.

It is P5 million. The Supreme Court held that in the case under consideration,
there is no dispute, and the Board even mentioned in its August 17, 1993 Decision,
that MSCI was organized and incorporated on February 15, 1990 with an authorized
capital stock of P60 million, P20 million of which was subscribed. Of the P20 million
subscribed capital stock, P5 million was paid-up. This fact is only too glaring for the
Board to have been misled into believing that MSCI'S paid-up capital stock was P64
million plus and not P5 million.
Power to increase or decrease capital stock; incur, create or increase bonded
indebtedness. No corporation shall increase or decrease its capital stock or incur,
create or increase any bonded indebtedness unless approved by a majority vote of the
board of directors and, at a stockholders' meeting duly called for the purpose, two-
thirds (2/3) of the outstanding capital stock shall favor the increase or diminution of
the capital stock, or the incurring, creating or increasing of any bonded indebtedness.
The above requirements, which are condition precedents before the capital
stock of a corporation may be increased, were unquestionably not observed in this
case. Henceforth, the paid-up capital stock of MSCI for the period covered by the
application for exemption still stood at P5 million. The losses, therefore, amounting to
P3,400,738.00 for the period February 15, 1990 to August 31, 1990 impaired MSCI's
paid-up capital of P5 million by as much as 68%. Likewise, the losses incurred by
MSCI for the interim period from September 1, 1990 to November 30, 1990, as found
by the Commission, per MSCI's quarterly income statements, amounting to
P13,554,337.33 impaired the company's paid-up capital of P5 million by a whopping
271.08%, more than enough to qualify MSCI as a distressed employer.

62 | P a g e
Law 321_Corporation LAW_ Case Digest

Amendment and/or Rejection of Articles of Incorporation

REPUBLIC PLANTERS BANK


vs.
COURT OF APPEALS and FERMIN CANLAS
G.R. No. 93073. December 21, 1992

FACTS:

On September 18, 1961, private respondent Corporation secured a loan from


petitioner in the amount of P120,000.00. As part of the proceeds of the loan, preferred
shares of stocks were issued to private respondent Corporation, through its officers
then, private respondent Adalia F. Robes and one Carlos F. Robes. In other words,
instead of giving the legal tender totaling to the full amount of the loan, which is
P120,000.00, petitioner lent such amount partially in the form of money and partially
in the form of stock certificates. Said stock certificates were in the name of private
respondent Adalia F. Robes and Carlos F. Robes, who subsequently, however,
endorsed his shares in favor of Adalia F. Robes. Said certificates of stock bear the
following terms and conditions:
The Preferred Stock shall have the following rights, preferences, qualifications
and limitations, to wit: Of the right to receive a quarterly dividend of One Per Centum
(1%), cumulative and participating. That such preferred shares may be redeemed, by
the system of drawing lots, at any time after two (2) years from the date of issue at the
option of the Corporation
On January 31, 1979, private respondents proceeded against petitioner and
filed a Complaint anchored on private respondents' alleged rights to collect dividends
under the preferred shares in question and to have petitioner redeem the same under
the terms and conditions of the stock certificates.

ISSUES:

Whether or not there is a difference between a preferred share from a


redeemable share.
Whether or not petitioner can be compelled by defendant to redeem the
preferred shares issued to the private respondent.

RULING:

YES.

A preferred share of stock is one which entitles the holder thereof to certain
preferences over the holders of common stock. The preferences are designed to induce
persons to subscribe for shares of a corporation. Preferred shares take a multiplicity
of forms. The most common forms may be classified into two: (1) preferred shares as to
assets; and (2) preferred shares as to dividends. The former is a share which gives the
holder thereof preference in the distribution of the assets of the corporation in case of
liquidation; the latter is a share the holder of which is entitled to receive dividends on
said share to the extent agreed upon before any dividends at all are paid to the holders
of common stock. There is no guaranty, however, that the share will receive any
dividends.
Under the old Corporation Law in force at the time the contract between the
petitioner and the private respondents was entered into, it was provided that "no
corporation shall make or declare any dividend except from the surplus profits arising
from its business, or distribute its capital stock or property other than actual profits
among its members or stockholders until after the payment of its debts and the
termination of its existence by limitation or lawful dissolution." Similarly, the present
Corporation Code provides that the board of directors of a stock corporation may
declare dividends only out of unrestricted retained earnings. The Code, in Section 43,
adopting the change made in accounting terminology, substituted the phrase
"unrestricted retained earnings," which may be a more precise term, in place of
"surplus profits arising from its business" in the former law.

63 | P a g e
Law 321_Corporation LAW_ Case Digest

Thus, the declaration of dividends is dependent upon the availability of surplus


profit or unrestricted retained earnings, as the case may be.
Preferences granted to preferred stockholders, moreover, do not give them a lien
upon the property of the corporation nor make them creditors of the corporation, the
right of the former being always subordinate to the latter. Dividends are thus payable
only when there are profits earned by the corporation and as a general rule, even if
there are existing profits, the board of directors has the discretion to determine
whether or not dividends are to be declared. Shareholders, both common and
preferred, are considered risk takers who invest capital in the business and who can
look only to what is left after corporate debts and liabilities are fully paid.
Redeemable shares, on the other hand, are shares usually preferred, which by
their terms are redeemable at a fixed date, or at the option of either issuing
corporation, or the stockholder, or both at a certain redemption price. Redemption by
the corporation of its stock is, in a sense, a repurchase of it for cancellation. The
present Code allows redemption of shares even if there are no unrestricted retained
earnings on the books of the corporation. This is a new provision which in effect
qualifies the general rule that the corporation cannot purchase its own shares except
out of current retained earnings. However, while redeemable shares may be redeemed
regardless of the existence of unrestricted retained earnings, this is subject to the
condition that the corporation has, after such redemption, assets in its books to cover
debts and liabilities inclusive of capital stock. Redemption, therefore, may not be made
where the corporation is insolvent or if such redemption will cause insolvency or
inability of the corporation to meet its debts as they mature.

YES.

While it is true that the very wordings of the terms and conditions in said stock
certificates clearly allows redemption, the option to do was clearly vested in the
petitioner bank. The redemption therefore is clearly the type known as "optional".
Thus, except as otherwise provided in the stock certificate, the redemption rests
entirely with the corporation and the stockholder is without right to either compel or
refuse the redemption of its stock. Furthermore, the terms and conditions set forth
therein use the word "may". It is a settled doctrine in statutory construction that the
word "may" denotes discretion, and cannot be construed as having a mandatory effect.
CA decision is reversed and set aside. Judgement is hereby rendered declaring
private respondent Fermin Canlas jointly and severally liable on all the nine
promissory notes with the following sums and at 16% interest per annum from the
dates indicated.

64 | P a g e
Law 321_Corporation LAW_ Case Digest

Alter Ego/Instrumentality Cases

PHILIPPINE NATIONAL BANK


vs.
HYDRO RESOURCES CONTRACTORS CORPORATION
G.R. No. 167561
ASSET PRIVATIZATION TRUST
vs.
HYDRO RESOURCES CONTRACTORS CORPORATION
G.R. No. 167603
DEVELOPMENT BANK OF THE PHILIPPINES
vs.
HYDRO RESOURCES CONTRACTORS CORPORATION
G.R. No. 167530. March 13, 2013

FACTS:

A contract was entered into between Hydro and NIA for the project of the latter.
The contract price is to be payable partly in Philippine peso and US dollars. Once the
project was being executed, there was depreciation in value of Peso resulting to price
differential. In order to resolve the issue, the administrator of NIA, Mr Tek, and Hydro
made a joint computation of the amount corresponding to the foreign currency
differential. The computation showed that NIA owed Hydro for the differential. When a
demand was made by Hydro against NIA, NIA refused to pay contending that Mr Tek
has no authority to participate into a joint computation of the foreign currency
differential and that Mr Tek has no authority to bind NIA.

ISSUE:

Whether or not the corporate entity of PNB and DBP must be pierced.

RULING:

NO.

A corporation is an artificial entity created by operation of law. It possesses the


right of succession and such powers, attributes, and properties expressly authorized
by law or incident to its existence. It has a personality separate and distinct from that
of its stockholders and from that of other corporations to which it may be connected.
As a consequence of its status as a distinct legal entity and as a result of a conscious
policy decision to promote capital formation, a corporation incurs its own liabilities
and is legally responsible for payment of its obligations.40 In other words, by virtue of
the separate juridical personality of a corporation, the corporate debt or credit is not
the debt or credit of the stockholder. This protection from liability for shareholders is
the principle of limited liability.
Equally well-settled is the principle that the corporate mask may be removed or
the corporate veil pierced when the corporation is just an alter ego of a person or of
another corporation. For reasons of public policy and in the interest of justice, the
corporate veil will justifiably be impaled only when it becomes a shield for fraud,
illegality or inequity committed against third persons.
However, the rule is that a court should be careful in assessing the milieu
where the doctrine of the corporate veil may be applied. Otherwise an injustice,
although unintended, may result from its erroneous application. Thus, cutting
through the corporate cover requires an approach characterized by due care and
caution.
Hence, any application of the doctrine of piercing the corporate veil should be
done with caution. A court should be mindful of the milieu where it is to be applied. It
must be certain that the corporate fiction was misused to such an extent that
injustice, fraud, or crime was committed against another, in disregard of its rights.
The wrongdoing must be clearly and convincingly established; it cannot be presumed.

65 | P a g e
Law 321_Corporation LAW_ Case Digest

VIVIAN T. RAMIREZ, ALBERTO B. DIGNO, DANILO M. CASQUITE, JUMADIYA A.


KADIL, FAUJIA SALIH, ANTONIO FABIAN, ROMEL DANAG, et.al.
vs.
MAR FISHING CO., INC., MIRAMAR FISHING CO., INC., ROBERT BUEHS AND
JEROME SPITZ
G.R. No. 168208. June 13, 2012

FACTS:

Mar Fishing Co., Inc. (Mar Fishing), engaged in the business of fishing and
canning of tuna, sold its principal assets to co-respondent Miramar through public
bidding. Proceeds of the sale were paid to the Trade and Investment Corp. to cover Mar
Fishing‘s outstanding obligation in the amount of ₱ 897,560,041.00.
In view of that transfer, Mar Fishing issued a Memorandum informing all its
workers that the company would cease to operate by the end of the month. It notified
the DOLE of the closure of its business operations. Then, Mar Fishing‘s labor union,
Mar Fishing Workers Union – NFL – and Miramar entered into a Memorandum of
Agreement for the acquiring company, Miramar, to absorb Mar Fishing‘s regular rank
and file employees whose performance was satisfactory, without loss of seniority rights
and privileges previously enjoyed. Unfortunately, petitioners, who worked as rank and
file employees, were not hired or given separation pay by Miramar, so they filed
Complaints for illegal dismissal with money claims before the Arbitration Branch of
the NLRC.

ISSUE:

Whether or not Mar Fishing and Miramar are solidarily liable to the employees.

RULING:

NO.

Mar Fishing, and not Miramar, is required to compensate petitioners. Indeed,


the back wages and retirement pay earned from the former employer cannot be filed
against the new owners or operators of an enterprise.
Miramar and Mar Fishing are separate and distinct entities, based on the
marked differences in their stock ownership. Also, the fact that Mar Fishing‘s officers
remained as such in Miramar does not by itself warrant a conclusion that the two
companies are one and the same. The mere showing that the corporations had a
common director sitting in all the boards without more does not authorize
disregarding their separate juridical personalities.
Neither can the veil of corporate fiction between the two companies be pierced
by the rest of petitioners‘ submissions, namely, the alleged take-over by Miramar of
Mar Fishing‘s operations and the evident similarity of their businesses. Since piercing
the veil of corporate fiction is frowned upon, those who seek to pierce the veil must
clearly establish that the separate and distinct personalities of the corporations are set
up to justify a wrong, protect a fraud, or perpetrate a deception. This, unfortunately,
petitioners have failed to do.

66 | P a g e
Law 321_Corporation LAW_ Case Digest

TIMOTEO H. SARONA
vs.
NATIONAL LABOR RELATIONS COMMISSION, ROYALE SECURITY AGENCY
(FORMERLY SCEPTRE SECURITY AGENCY) and CESAR S. TAN
G.R. No. 185280. January 18, 2012

FACTS:

Petitioner, a security guard in Sceptre since April 1976, was asked by Sceptre‘s
operations manager to submit a resignation letter as a requirement for an application
in Royale and to fill up an employment application form for the said company. He was
then assigned at Highlight Metal Craft Inc. from July 29 to August 8, 2003 and was
later transferred to Wide Wide World Express Inc.
On September 2003, he was informed that his assignment at WWWE Inc. was
withdrawn because Royale has been allegedly replaced by another security agency
which he later discovered to be untrue. Nevertheless, he was once again assigned at
Highlight Metal sometime in September 2003and when he reported at Royale‘s office
on October 1, 2003, he was informed that he would no longer be given any assignment
as instructed by Sceptre‘s general manager.
He thus filed a complaint for illegal dismissal.

ISSUE:

Whether or not Royale‘s corporate fiction should be pierced for the purpose of
compelling it to recognize the petitioner‘s length of service with Sceptre and for holding
it liable for the benefits that have accrued to him arising from his employment with
Sceptre.

RULING:

YES.

The doctrine of piercing the corporate veil is applicable on alter ego cases,
where a corporation is merely a farce since it is a mere alter ego or business conduit of
a person, or where the corporation is so organized and controlled and its affairs are so
conducted as to make it merely an instrumentality, agency, conduit or adjunct of
another corporation.
The respondents‘ scheme reeks of bad faith and fraud and compassionate
justice dictates that Royale and Sceptre be merged as a single entity, compelling
Royale to credit and recognize the petitioner‘s length of service with Sceptre. The
respondents cannot use the legal fiction of a separate corporate personality for ends
subversive of the policy and purpose behind its creation or which could not have been
intended by law to which it owed its being.
Also, Sceptre and Royale have the same principal place of business. As early as
October 14, 1994, Aida and Wilfredo became the owners of the property used by
Sceptre as its principal place of business by virtue of a Deed of Absolute Sale they
executed with Roso. Royale, shortly after its incorporation, started to hold office in the
same property. These, the respondents failed to dispute. Royale also claimed a right to
the cash bond which the petitioner posted when he was still with Sceptre. If Sceptre
and Royale are indeed separate entities, Sceptre should have released the petitioner‘s
cash bond when he resigned and Royale would have required the petitioner to post a
new cash bond in its favor.
The way on how petitioner was made to resign from Sceptre then later on made
an employee of Royale, reflects the use of the legal fiction of the separate corporate
personality and is an implication of continued employment. Royale is a continuation
or successor or Sceptre since the employees of Sceptre and of Royale are the same and
said companies have the same principal place of business.

67 | P a g e
Law 321_Corporation LAW_ Case Digest

GOLD LINE TOURS, INC.


vs.
HEIRS OF MARIA CONCEPCION LACSA
G.R. No. 159108. June 18, 2012

FACTS:

Concepcion boarded a Gold Line passenger bus owned and operated by Travel
& Tours Advisers, Inc. Before reaching their destination, the Gold Line bus collided
with a passenger jeepney and as a result, a metal part of the jeepney was detached
and struck Concepcion in the chest, causing her instant death. Then, Concepcion‘s
heirs, represented by Teodoro Lacsa, instituted in the RTC a suit against Travel &
Tours Advisers Inc. to recover damages arising from breach of contract of carriage.

ISSUE:

Whether or not the proposition of the third party claimant by the petitioner
where Travel & Tours Advises, Inc. has an existence separate and/or distinct from
Gold Line Tours, Inc.

RULING:

NO.

The two corporations are liable to the death of Ma. Concepcion Lacsa. The
Court was not persuaded by the proposition of the third party claimant that a
corporation has an existence separate and/or distinct from its members insofar as
this case at bar is concerned, for the reason that whenever necessary for the interest
of the public or for the protection of enforcement of their rights, the notion of legal
entity should not and is not to be used to defeat public convenience, justify wrong,
protect fraud or defend crime.
Where the main purpose in forming the corporation was to evade one‘s
subsidiary liability for damages in a criminal case, the corporation may not be heard
to say that it has a personality separate and distinct from its members, because to
allow it to do so would be to sanction the use of fiction of corporate entity as a shield
to further an end subversive of justice.
This is what the third party claimant wants to do including the defendant in
this case, to use the separate and distinct personality of the two corporations as a
shield to further an end subversive of justice by avoiding the execution of a final
judgment of the court.
The RTC thus rightly ruled that petitioner might not be shielded from liability
under the final judgment through the use of the doctrine of separate corporate
identity. Truly, this fiction of law could not be employed to defeat the ends of justice.

68 | P a g e
Law 321_Corporation LAW_ Case Digest

HACIENDA LUISITA, INCORPORATED


LUISITA INDUSTRIAL PARK CORPORATION and RIZAL COMMERCIAL BANKING
CORPORATION
vs.
PRESIDENTIAL AGRARIAN REFORM COUNCIL; SECRETARY NASSER
PANGANDAMAN OF THE DEPARTMENT OF AGRARIAN REFORM; ALYANSA NG
MGA MANGGAGAWANG BUKID NG HACIENDA LUISITA, RENE GALANG, NOEL
MALLARI, and JULIO SUNIGA1 and his SUPERVISORY GROUP OF THE
HACIENDA LUISITA, INC. and WINDSOR ANDAYA
G.R. No. 171101. November 22, 2011

FACTS:

On March 17, 1988, during the administration of President Corazon Cojuangco


Aquino, the Office of the Solicitor General moved to withdraw the government‘s case
against Tadeco, et al. The CA dismissed the case, subject to the PARC‘s approval of
Tadeco‘s proposed stock distribution plan (SDP) in favor of its farmworkers. [Under EO
229 and later RA 6657, Tadeco had the option of availing stock distribution as an
alternative modality to actual land transfer to the farmworkers.] On August 23, 1988,
Tadeco organized a spin-off corporation, herein petitioner HLI, as vehicle to facilitate
stock acquisition by the farmworkers. For this purpose, Tadeco conveyed to HLI the
agricultural land portion (4,915.75 hectares) and other farm-related properties of
Hacienda Luisita in exchange for HLI shares of stock.
On May 9, 1989, some 93% of the then farmworker-beneficiaries (FWBs)
complement of Hacienda Luisita signified in a referendum their acceptance of the
proposed HLI‘s Stock Distribution Option Plan (SODP). On May 11, 1989, the SDOA
was formally entered into by Tadeco, HLI, and the 5,848 qualified FWBs. This attested
to by then DAR Secretary Philip Juico. The SDOA embodied the basis and mechanics
of HLI‘s SDP, which was eventually approved by the PARC after a follow-up
referendum conducted by the DAR on October 14, 1989, in which 5,117 FWBs, out of
5,315 who participated, opted to receive shares in HLI.
On August 15, 1995, HLI applied for the conversion of 500 hectares of land of
the hacienda from agricultural to industrial use, pursuant to Sec. 65 of RA 6657. The
DAR approved the application on August 14, 1996, subject to payment of three
percent (3%) of the gross selling price to the FWBs and to HLI‘s continued compliance
with its undertakings under the SDP, among other conditions.

ISSUE:

Whether or not the revocation of the HLI‘s SDP valid.

RULING:

YES.

The PARC did NOT gravely abuse its discretion in revoking the subject SDP and
placing the hacienda under CARP‘s compulsory acquisition and distribution scheme.
The revocation of the approval of the SDP is valid: (1) the mechanics and
timelines of HLI‘s stock distribution violate DAO 10 because the minimum individual
allocation of each original FWB of 18,804.32 shares was diluted as a result of the use
of ―man days‖ and the hiring of additional farmworkers; (2) the 30-year timeframe for
HLI-to-FWBs stock transfer is contrary to what Sec. 11 of DAO 10 prescribes.
It is clear as day that the original 6,296 FWBs, who were qualified beneficiaries
at the time of the approval of the SDP, suffered from watering down of shares. As
determined earlier, each original FWB is entitled to 18,804.32 HLI shares. The
original FWBs got less than the guaranteed 18,804.32 HLI shares per beneficiary,
because the acquisition and distribution of the HLI shares were based on ―man days‖
or ―number of days worked‖ by the FWB in a year‘s time.

69 | P a g e
Law 321_Corporation LAW_ Case Digest

PANTRANCO EMPLOYEES ASSOCIATION (PEA-PTGWO) and PANTRANCO


RETRENCHED EMPLOYEES ASSOCIATION (PANREA)
vs.
NATIONAL LABOR RELATIONS COMMISSION (NLRC), PANTRANCO NORTH
EXPRESS, INC. (PNEI), PHILIPPINE NATIONAL BANK (PNB), PHILIPPINE
NATIONAL BANK-MANAGEMENT AND DEVELOPMENT CORPORATION (PNB-
MADECOR), and MEGA PRIME REALTY AND HOLDINGS CORPORATION (MEGA
PRIME)
G.R. No. 170689. March 17, 2009

FACTS:

The Gonzales family owned two corporations, namely, the PNEI and Macris
Realty Corporation. PNEI provided transportation services to the public, and had its
bus terminal at the corner of Quezon and Roosevelt Avenues in Quezon City. The
terminal stood on four valuable pieces of real estate registered under the name of
Macris. The Gonzales family later incurred huge financial losses despite attempts of
rehabilitation and loan infusion. In March 1975, their creditors took over the
management of PNEI and Macris. By 1978, full ownership was transferred to one of
their creditors, the National Investment Development Corporation (NIDC), a subsidiary
of the PNB.
In 1985, NIDC sold PNEI to North Express Transport, Inc. (NETI), a company
owned by Gregorio Araneta III. In 1992, PNEI applied with the Securities and
Exchange Commission (SEC) for suspension of payments.

ISSUE:

Whether PNEI employees can attach the properties (specifically the Pantranco
properties) of PNB, PNB-Madecor and Mega Prime to satisfy their unpaid labor claims
against PNEI.

RULING:

NO.

First, the subject property is not owned by the judgment debtor, that is, PNEI.
Nowhere in the records was it shown that PNEI owned the Pantranco properties.
Settled is the rule that the power of the court in executing judgments extends only to
properties unquestionably belonging to the judgment debtor alone. To be sure, one
man‘s goods shall not be sold for another man‘s debts. A sheriff is not authorized to
attach or levy on property not belonging to the judgment debtor, and even incurs
liability if he wrongfully levies upon the property of a third person.
Second, PNB, PNB-Madecor and Mega Prime are corporations with personalities
separate and distinct from that of PNEI. PNB is sought to be held liable because it
acquired PNEI through NIDC at the time when PNEI was suffering financial reverses.
PNB-Madecor is being made to answer for petitioners‘ labor claims as the owner of the
subject Pantranco properties and as a subsidiary of PNB. Mega Prime is also included
for having acquired PNB‘s shares over PNB-Madecor.
The general rule is that a corporation has a personality separate and distinct
from those of its stockholders and other corporations to which it may be connected.
This is a fiction created by law for convenience and to prevent injustice. Obviously,
PNB, PNB-Madecor, Mega Prime, and PNEI are corporations with their own
personalities. Neither can we merge the personality of PNEI with PNB simply because
the latter acquired the former. Settled is the rule that where one corporation sells or
otherwise transfers all its assets to another corporation for value, the latter is not, by
that fact alone, liable for the debts and liabilities of the transferor.
Lastly, while we recognize that there are peculiar circumstances or valid
grounds that may exist to warrant the piercing of the corporate veil, none applies in
the present case whether between PNB and PNEI; or PNB and PNB-Madecor.

70 | P a g e
Law 321_Corporation LAW_ Case Digest

CAGAYAN VALLEY DRUG CORPORATION


vs.
COMMISSIONER OF INTERNAL REVENUE
G.R. No. 151413. February 13, 2008

FACTS:

Petitioner granted 20% sales discounts to qualified senior citizens on purchases


of medicine pursuant to RA 7432 and its IRR. Petitioner filed with the BIR a claim for
tax refund/tax credit of the full amount of the 20% sales discount it granted to senior
citizens for the year 1995, in accordance with RA 7432.
The BIR‘s inaction on petitioner‘s claim for refund/tax credit compelled
petitioner to file a petition for review before the CTA, until the case reached the CA.
The CA dismissed the petition because the person who signed the verification and
certification of absence of forum shopping, a certain Jacinto J. Concepcion, President
of petitioner, failed to adduce proof that he was duly authorized by the board of
directors to do so.

ISSUE:

Whether or not it is valid when the petitioner‘s president signs the subject
verification and certification sans the approval of its Board of Directors.

RULING:

NO.

A corporation has a separate and distinct personality from its directors and
officers and can only exercise its corporate powers through the board of directors.
Thus, it is clear that an individual corporate officer cannot solely exercise any
corporate power pertaining to the corporation without authority from the board of
directors.
Only individuals vested with authority by a valid board resolution may sign the
certificate of non-forum shopping on behalf of a corporation. The action can be
dismissed if the certification was submitted unaccompanied by proof of the signatory‘s
authority. Hence, the power to sue and be sued in any court or quasi-judicial tribunal
is necessarily lodged with the said board.
There is a complete listing of authorized signatories to the verification and
certification required by the rules, the determination of the sufficiency of the authority
was done on a case to case basis. The rationale applied in the foregoing cases is to
justify the authority of corporate officers or representatives of the corporation to sign
the verification or certificate against forum shopping, being in a position to verify the
truthfulness and correctness of the allegations in the petition.

71 | P a g e
Law 321_Corporation LAW_ Case Digest

THE HEIRS OF THE LATE PANFILO


vs.
PAJARILLO VS. CA, NLRC, et al.
G.R. No. 155056-57. October 19, 2007

FACTS:

Private respondents were employed as drivers, conductors and conductresses


by Panfilo. In sum, each of the private respondents earned an average daily
commission of about P150.00 a day. They were not given emergency cost of living
allowance, 13th month pay, legal holiday pay and service incentive leave pay.The
following were deducted from the private respondents‘ daily commissions.
Thereafter, private respondents and several co-employees formed a union called
―SAMAHAN NG MGA MANGGAGAWA NG PANFILO V. PAJARILLO‖. The Department
of Labor and Employment issued a Certificate of Registration in favor of the
respondent union. Upon learning of the formation of respondent union, Panfilo and
his children ordered some of the private respondents to sign a document affirming
their trust and confidence in Panfilo and denying any irregularities on his part. Other
private respondents were directed to sign a blank document which turned out to be a
resignation letter. Private respondents refused to sign the said documents; hence,
they were barred from working or were dismissed without hearing and notice. Panfilo
and his children and relatives also formed a company union where they acted as its
directors and officers.
On 25 August 1987, respondent union and several employees filed a Complaint
for unfair labor practice and illegal deduction before the Labor Arbiter with ―Panfilo V.
Pajarillo Liner‖ as party-respondent. After hearing and submission by both parties of
their respective position papers and memoranda, Labor Arbiter Manuel P. Asuncion
rendered a Decision dated 28 December 1992, dismissing the consolidated complaints
for lack of merit. Respondent union appealed to the NLRC. On 18 June 1996, the
NLRC reversed the decision of Arbiter Asuncion and ordered the reinstatement and
payment of backwages, ECOLA, 13th month pay, legal holiday pay and service
incentive leave pay to, private respondents.

ISSUE:

Whether the Honorable Court of Appeals seriously erred in piercing the veil of
corporate entity of Pvp Pajarillo Liner Inc.

RULING:

NO.

Hence, when the notion of separate juridical personality is used to defeat public
convenience, justify wrong, protect fraud or defend crime, or is used as a device to
defeat labor laws, this separate personality of the corporation may be disregarded or
the veil of the corporate fiction pierced. This is true likewise when the corporation is
merely an adjunct, a business conduit or an alter ego of another corporation. The
corporate mask may be lifted and the corporate veil may be pierced when a
corporation is but the alter ego of a person or another corporation. It is clear from the
foregoing that P.V. Pajarillo Liner Inc. was a mere continuation and successor of the
sole proprietorship of Panfilo. It is also quite obvious that Panfilo transformed his sole
proprietorship into a family corporation in a surreptitious attempt to evade the
charges of respondent union. Given these considerations, Panfilo and P.V. Pajarillo
Liner Inc. should be treated as one and the same person for purposes of liability.

72 | P a g e
Law 321_Corporation LAW_ Case Digest

PETRON CORPORATION AND PETER C. MALIGRO


vs.
NATIONAL LABOR RELATIONS COMMISSION AND CHITO S. MANTOS
G.R. No. 154532. October 27, 2006

FACTS

On May 15, 1990, Petron, through its Cebu District Office, hired the herein
private respondent Chito S. Mantos, an Industrial Engineer, as a managerial,
professional and technical employee with initial designation as a Bulk Plant
Engineering Trainee. He attained regular employment status on November 15, 1990
and was later on designated as a Bulk Plant Relief Supervisor, remaining as such for
the next five years while being assigned to the different plants and offices of Petron
within the Visayas area.
It was while assigned at Petron‘s Cebu District Office with petitioner Peter
Maligro as his immediate superior, when Mantos, thru a Notice of Disciplinary Action
was suspended for 30 days from November 1 to 30, 1996 for violating company rules
and regulations regarding AWOL, not having reported for work during the period
August 5 to 27, 1996. Subsequently, his services was terminated effective December
1, 1996, by reason of his continued absences from August 28, 1996 onwards, as well
as for Insubordination/Discourtesy for making false accusations against his superior.
Meanwhile, contending that he has been constructively dismissed as of August 5,
1996, Mantos filed with the NLRC-RAB, Cebu City, a complaint for illegal dismissal.

ISSUES:

Whether or not Maligro is solidarily liable with Petron.

RULING:

NO.

The NLRC erred in holding petitioner Peter Maligro jointly and severally liable
with petitioner Petron for the money claims of the private respondent.
Settled is the rule in this jurisdiction that a corporation is invested by law with
a legal personality separate and distinct from those acting for and in its behalf and, in
general, from the people comprising it. Thus, obligations incurred by corporate
officers acting as corporate agents are not theirs but the direct accountabilities of the
corporation they represent. True, solidary liabilities may at times be incurred by
corporate officers, but only when exceptional circumstances so warrant.
In the present case, the apparent basis for the NLRC in holding petitioner
Maligro solidarily liable with Petron were its findings that (1) the Investigation
Committee was created a day after the summons in NLRC RAB was received, with
Maligro no less being the chairman thereof; and (2) the basis for the charge of
insubordination was the private respondent‘s alleged making of false accusations
against Maligro.
Those findings, however, cannot justify a finding of personal liability on the part
of Maligro inasmuch as said findings do not point to Maligro‘s extreme personal hatred
and animosity with the respondent. It cannot, therefore, be said that Maligro was
motivated by malice and bad faith in connection with private respondent‘s dismissal
from the service.

73 | P a g e
Law 321_Corporation LAW_ Case Digest

CHINA BANKING CORPORATION


vs.
DYNE-SEM ELECTRONICS CORPORATION
G.R. No. 149237. June 11, 2006

FACTS:

On June 19 and 26, 1985, Dynetics, Inc. (Dynetics) and Elpidio O. Lim
borrowed a total of P8,939,000 from petitioner China Banking Corporation evidenced
by six promissory notes. The borrowers failed to pay when the obligations became due
prompting the petitioner to institute a complaint for sum of money against them.
Summons was not served on Dynetics, however, because it had already closed down.
An amended complaint was filed by petitioner impleading respondent Dyne-Sem
Electronics Corporation (Dyne-Sem) and its stockholders Vicente Chuidian, Antonio
Garcia and Jacob Ratinoff. According to petitioner, respondent was formed and
organized to be Dynetics alter ego as established by the following circumstances: (a)
Dynetics, Inc. and respondent are both engaged in the same line of business of
manufacturing, producing, assembling, processing, importing, exporting, buying,
distributing, marketing and testing integrated circuits and semiconductor devices; (b)
the principal office and factory site of Dynetics, Inc. located at Avocado Road, FTI
Complex, Taguig, Metro Manila, were used by respondent as its principal office and
factory site; (c) respondent acquired some of the machineries and equipment of
Dynetics, Inc. from banks which acquired the same through foreclosure; (d)
respondent retained some of the officers of Dynetics, Inc.

ISSUE:

Whether the Doctrine of Piercing the Veil of Corporate Fiction is applicable in


the present case.

RULING:

YES.

The general rule is that a corporation has a personality separate and distinct
from that of its stockholders and other corporations to which it may be connected.
This is a fiction created by law for convenience and to prevent injustice. Nevertheless,
being a mere fiction of law, peculiar situations or valid grounds may exist to warrant
the disregard of its independent being and the piercing of the corporate veil.
The veil of separate corporate personality may be lifted when such personality is used
to defeat public convenience, justify wrong, protect fraud or defend crime; or used as a
shield to confuse the legitimate issues; or when the corporation is merely an adjunct,
a business conduit or an alter ego of another corporation or where the corporation is
so organized and controlled and its affairs are so conducted as to make it merely an
instrumentality, agency, conduit or adjunct of another corporation; or when the
corporation is used as a cloak or cover for fraud or illegality, or to work injustice, or
where necessary to achieve equity or for the protection of the creditors. In such cases,
the corporation will be considered as a mere association of persons. The liability will
directly attach to the stockholders or to the other corporation. To disregard the
separate juridical personality of a corporation, the wrongdoing must be proven clearly
and convincingly.
In this case, petitioner failed to prove that Dyne-Sem was organized and
controlled, and its affairs conducted, in a manner that made it merely an
instrumentality, agency, conduit or adjunct of Dynetics, or that it was established to
defraud Dynetics creditors, including petitioner.

74 | P a g e
Law 321_Corporation LAW_ Case Digest

MARUBENI CORPORATION, RYOICHI TANAKA, RYOHEI KIMURA


and SHOICHI ONE
vs.
FELIX LIRAG
G.R. No. 130998. August 10, 2001

FACTS:

Marubeni Corporation is a foreign corporation organized under the laws of


Japan. It was doing business in the Philippines through its duly licensed, wholly
owned subsidiary, Marubeni Philippines Corporation. Petitioners Ryoichi Tanaka,
Ryohei Kimura and Shoichi One were officers of Marubeni assigned to its Philippine
branch.
On January 27, 1989, Lirag filed with the RTC of Makati a complaint for
specific performance and damages in the sum of P6M as commission pursuant to an
oral consultancy agreement with Marubeni for obtaining government contracts of
various projects. Lirag claimed that on February 2, 1987, petitioner Ryohei Kimura
hired his consultancy group for the purpose of obtaining government contracts of
various projects. The agreement was merely oral because of the mutual trust between
Marubeni and the Lirag family which dates back to the 1960s. One of the projects
handled by respondent Lirag, the Bureau of Post project, amounting to
P100,000,000.00 was awarded to the ―Marubeni-Sanritsu tandem. Despite repeated
demands of his 6% commission was never paid.
Marubeni claimed that Ryohei Kimura did not have the authority to enter into
such agreement in their behalf. Only the general manager, upon issuance of a SPA by
the principal office in Tokyo, Japan, could enter into any contract in behalf of the
corporation. They also claimed that Marubeni never participated in the Bureau of Post
project nor benefited from such project.

ISSUE:

Whether or not there was a consultancy agreement to make Lirag entitled to


commission.

RULING:

NO.

The only basis of Lirag in claiming from Marubeni was because he claims that
they are sister companies since Marubeni was the supplier and contractor of the
Sanritsu. Not because two foreign companies came from the same country and closely
worked together on certain projects would the conclusion arise that one was the
conduit of the other, thus piercing the veil of corporate fiction.
The separate personality of the corporation may be disregarded only when the
corporation is used as a cloak or cover for fraud or illegality, or to work injustice, or
where necessary for the protection of creditors. Aside from the self-serving testimony
of respondent regarding the existence of a close working relationship between
Marubeni and Sanritsu, there was nothing that would support the conclusion that
Sanritsu was an agent of Marubeni.
Any agreement entered into because of the actual or supposed influence which
the party has, engaging him to influence executive officials in the discharge of their
duties, which contemplates the use of personal influence and solicitation rather than
an appeal to the judgment of the official on the merits of the object sought is contrary
to public policy. Consequently, the agreement, assuming that the parties agreed to
the consultancy, is null and void as against public policy. Therefore, it is
unenforceable before a court of justice.

75 | P a g e
Law 321_Corporation LAW_ Case Digest

ADALIA B. FRANCISCO and MERRYLAND DEVELOPMENT CORPORATION


vs.
RITA C. MEJIA, as Executrix of ANDREA CORDOVA VDA. DE GUTIERREZ
G.R. No. 141617. August 14, 2001

FACTS:

Gutierrez was the registered owner of a parcel of land which was later
subdivided into five lots. In 1964, Gutierrez and Cardale Financing and Realty
Corporation executed a Deed of Sale with Mortgage relating to the four of the five lots
for the consideration of P800,000.00. Upon the execution of the deed, Cardale paid
Gutierrez P171,000.00. To secure payment of the balance of the purchase price,
Cardale constituted a mortgage on three of the four parcels of land.
In 1968, owing to Cardale's failure to settle its mortgage obligation, Gutierrez
filed a complaint for rescission of the contract with the Quezon City Regional Trial
Court. In 1969, during the pendency of the rescission case, Gutierrez died and was
substituted by her executrix, respondent Rita C. Mejia.
In the meantime, the mortgaged parcels of land became delinquent in the
payment of real estate taxes, which culminated in their levy and auction sale in
satisfaction of the tax arrears. The highest bidder for the three parcels of land was
petitioner Merryland Development Corporation, whose President and majority
stockholder is Francisco.

ISSUES:

Whether or not the corporate fiction of Cardale will be pierced.


Whether or not the corporate entity of Merryland must be pierced.

RULING:

YES.

Under the doctrine of piercing the veil of corporate entity, when valid grounds
therefore exist, the legal fiction that a corporation is an entity with a juridical
personality separate and distinct from its members or stockholders may be
disregarded. In such cases, the corporation will be considered as a mere association of
persons. The members or stockholders of the corporation will be considered as the
corporation, that is, liability will attach directly to the officers and stockholders. The
doctrine applies when the corporate fiction is used to defeat public convenience, justify
wrong, protect fraud, or defend crime, or when it is made as a shield to confuse the
legitimate issues, or where a corporation is the merealter ego or business conduit of a
person, or where the corporation is so organized and controlled and its affairs are so
conducted as to make it merely an instrumentality, agency, conduit or adjunct of
another corporation.

NO.

Merryland cannot be solidarily liable with Francisco. The only act imputable to
Merryland in relation to the mortgaged properties is that it purchased the same and
this by itself is not a fraudulent or wrongful act. No evidence has been adduced to
establish that Merryland was a mere alter ego or business conduit of Francisco. Time
and again it has been reiterated that mere ownership by a single stockholder or by
another corporation of all or nearly all of the capital stock of a corporation is not of
itself sufficient ground for disregarding the separate corporate personality. Neither has
it been alleged or proven that Merryland is so organized and controlled and its affairs
are so conducted as to make it merely an instrumentality, agency, conduit or adjunct
of Cardale. Even assuming that the businesses of Cardale and Merryland are
interrelated, this alone is not justification for disregarding their separate personalities,
absent any showing that Merryland was purposely used as a shield to defraud
creditors and third persons of their rights. Thus, Merryland's separate juridical
personality must be upheld.

76 | P a g e
Law 321_Corporation LAW_ Case Digest

PHILIPPINE NATIONAL BANK & NATIONAL SUGAR DEVELOPMENT


CORPORATION (NASUDECO)
vs.
ANDRADA ELECTRIC & ENGINEERING COMPANY
GR No. 142936. April 17, 2002

FACTS:

Respondent is a partnership duly organized, existing, and operating under the


laws of the Philippines is a semi-government corporation duly organized, existing and
operating under the laws of the Philippines; whereas, NASUDECO is also a semi-
government corporation and the sugar arm of the PNB; and the defendant Pampanga
Sugar Mills (PASUMIL in short), is a corporation organized, existing and operating
under the 1975 laws of the Philippines; that the plaintiff is engaged in the business of
general construction for the repairs and/or construction of different kinds of
machineries and buildings.
On August 26, 1975, PNB acquired the assets of the defendant PASUMIL that
were earlier foreclosed by the DBP. PNB organized the defendant NASUDECO in
September, 1975, to take ownership and possession of the assets and ultimately to
nationalize and consolidate its interest in other PNB controlled sugar mills; that prior
to October 29, 1971, the defendant PASUMIL engaged the services of defendant for
electrical rewinding and repair, most of which were partially paid by the defendant
PASUMIL, leaving several unpaid accounts with the plaintiff; that finally, on October
29, 1971, the plaintiff and the defendant PASUMIL entered into a construction
contract.
The defendant PASUMIL and the defendant PNB, and now the defendant
NASUDECO, failed and refused to pay the plaintiff their just, valid and demandable
obligation based on the contract. Defendant prayed that judgment be rendered against
the defendants PNB, NASUDECO, and PASUMIL.

ISSUE:

Whether or not the Veil of Corporate Fiction should be pierced in this case.

RULING:

NO.

The absence of the elements in the present case precludes the piercing of the
corporate veil. First, other than the fact that petitioners acquired the assets of
PASUMIL, there is no showing that their control over it warrants the disregard of
corporate personalities. Second, there is no evidence that their juridical personality
was used to commit a fraud or to do a wrong; or that the separate corporate entity was
farcically used as a mere alter ego, business conduit or instrumentality of another
entity or person. Third, respondent was not defrauded or injured when petitioners
acquired the assets of PASUMIL.
Being the party that asked for the piercing of the corporate veil, respondent had
the burden of presenting clear and convincing evidence to justify the setting aside of
the separate corporate personality rule. However, it utterly failed to discharge this
burden; it failed to establish by competent evidence that petitioner‘s separate
corporate veil had been used to conceal fraud, illegality or inequity.

77 | P a g e
Law 321_Corporation LAW_ Case Digest

AZCOR MANUFACTURING INC., FILIPINAS PASO and/or ARTURO


ZULUAGA/Owner
vs.
NATIONAL LABOR RELATIONS COMMISSION (NLRC) AND CANDIDO CAPULSO
G.R. No. 117963. February 11, 1999

FACTS:

Candido Capluso has been working for petitioner for more than 12 years as a
ceramics worker. On February 1991, Capulso requested to go on sick leave, it
appearing that his illness was directly caused by his occupation. Upon recovering,
Capulso was not allowed to resume work and was not reinstated after having tried five
times. He filed a complaint for constructive illegal dismissal and illegal deduction
against AZCOR and Arturo Zuluaga.
AZCOR moved to dismiss the complaint alleging that no employer- employee
relationship existed. Petitioner further added that Capulso became an employee of Fil
Paso on March 1990 but voluntarily resigned after a year as evidenced by a letter of
resignation allegedly tendered by Capulso.
The Labor Arbiter dismissed the complaint for lack of merit and ordered AZCOR
to refund the deducted salaries. On Appeal, the NLRC ruled that the Contract of
Employment stated that the work to be done by Capulso was with Fil Paso and added
the fact that the latter denied having executed and signed the said resignation letters.
Pending the trial of AZCOR‘s petition for Certiorari, Capulso succumbed to asthma
and heart disease.

ISSUE:

Whether the petitioners are jointly liable for backwages in favor of the heirs
being separate and distinct entities.

RULING:

YES.

Capulso was led into believing that while he was working with Filipinas Paso,
his real employer was AZCOR. Petitioners never dealt with him openly and in good
faith, nor was he informed of the developments within the company, i.e., his alleged
transfer to Filipinas Paso and the closure of AZCOR's manufacturing operations
beginning 1 March 1990. AZCOR manifested for the first time before the Court that it
had already ceased its business operations. Understandably, Capulso sued AZCOR
alone and was constrained to implead Filipinas Paso as additional respondent only
when it became apparent that the latter also appeared to be his employer.
In the case, the corporate fiction was used as a means to perpetrate a social
injustice or as a vehicle to evade obligations or confuse the legitimate issues. Such
corporate fiction would be discarded and the two (2) corporations would be merged as
one, the first being merely considered as the instrumentality, agency, conduit or
adjunct of the other.

78 | P a g e
Law 321_Corporation LAW_ Case Digest

EDUARDO CLAPAROLS, ROMULO AGSAM and/or


CLAPAROLS STEEL AND NAIL PLANT
vs.
COURT OF INDUSTRIAL RELATIONS, ALLIED WORKERS' ASSOCIATION
and/or DEMETRIO GARLITOS, et al.
G.R. No. L-30822. July 31, 1975

FACTS:

In a case filed by private respondents against petitioners for unfair labor


practices, CIR held petitioners liable for reinstatement and back wages from the date
of their dismissal up to their actual reinstatement. Motion for execution was granted
and an examination of petitioners‘ payrolls and other records for the computation of
the back wages. When respondents returned to work, the company accountant
refused on the ground that there was no order from the plant owner.
This was on the ground that the records of Claparols Steel Corp. (CSC) show
that it was established on July 1, 1957 succeeding the CSNP which ceased operations
on June 30, 1957, and that the CSC stopped operation on Dec. 7, 1962. Petitioners
filed an opposition alleging that they cannot personally reinstate respondents because
of the present status of the corporation; back wages should only be limited to 3
months; and that since it ceased to operate on Dec. 7, 1962, reinstatement should
only be up to that date. Respondents opposed and alleged, among others, that CSNP
and CSC is one and the same corporation controlled by petitioner Claparols, with the
latter corporation succeeding the former.

ISSUE:

Whether or not CSNP and CSC is one and the same corporation.

RULING:

YES.

Respondent Court‘s findings that indeed the CSNP, which ceased operation in
June 30, 1957, was succeeded by the CSC effective next day, July 1, 1957 up top
December 7, 1962, when the latter finally ceased to operate, were not disputed by
petitioners. It is very clear that the latter was a continuation and successor of the first
entity, and its emergence was skillfully timed to avoid the financial liability that
already attached to its predecessor. Both corporations were owned and controlled by
petitioner Eduardo Claparols and there was no break in the succession and continuity
of the same business. This ―avoiding-the-liability‖ scheme is very patent, considering
that 90% of the subscribed shares of the CSC were owned by Claparols himself, and
all the assets of the dissolved CSNP were turned over to the CSC.

79 | P a g e
Law 321_Corporation LAW_ Case Digest

COMMISSIONER OF INTERNAL REVENUE


vs.
NORTON and HARRISON COMPANY
G.R. No. L-17618. August 31, 1964

FACTS:

Norton and Harrison is a corporation organized to carry on and conduct a


general wholesale and retail mercantile establishment in the Philippines. Jackbilt is,
likewise, a corporation organized primarily for the purpose of making, producing and
manufacturing concrete blocks. Norton and Jackbilt entered into an agreement
whereby Norton was made the sole and exclusive distributor of concrete blocks
manufactured by Jackbilt. Pursuant to this agreement, whenever an order for concrete
blocks was received by the Norton & Harrison Co. from a customer, the order was
transmitted to Jackbilt which delivered the merchandise direct to the customer.
Apparently, due to this transaction, the CIR, after conducting an investigation,
assessed the respondent Norton & Harrison for deficiency sales tax making as basis
thereof the sales of Norton to the Public. As Norton and Harrison did not conform with
the assessment, the matter was brought to the Court of Tax Appeals.
The Commissioner of Internal Revenue contends that since Jackbilt was owned
and controlled by Norton & Harrison, the corporate personality of Jackbilt should be
disregarded for sales tax purposes, and the sale of Jackbilt blocks by petitioner to the
public must be considered as the original sales from which the sales tax should be
computed. The Norton & Harrison Company contended otherwise -that is, the
transaction subject to tax is the sale from Jackbilt to Norton.

ISSUE:

Whether or not the corporate personality of Norton and Jackbilt be disregarded.

RULING:

YES.

It has been settled that the ownership of all the stocks of a corporation by
another corporation does not necessarily breed an identity of corporate interest
between the two companies and be considered as a sufficient ground for disregarding
the distinct personalities. However, in the case at bar, we find sufficient grounds to
support the theory that the separate identities of the two companies should be
disregarded. Among these circumstances are: Norton and Harrison owned all the
outstanding stocks of Jackbilt; of the 15,000 authorized shares of Jackbilt, 14,993
shares belonged to Norton and Harrison and one each to seven others; Norton
constituted Jackbilt's board of directors in such a way as to enable it to actually direct
and manage the other's affairs by making the same officers of the board for both
companies; Norton financed the operations of the Jackbilt, and this is shown by the
fact that the loans obtained from the RFC and Bank of America were used in the
expansion program of Jackbilt, to pay advances for the purchase of equipment,
materials rations and salaries of employees of Jackbilt and other sundry expenses;
Norton treats Jackbilt employees as its own. Evidence shows that Norton paid the
salaries of Jackbilt employees and gave the same privileges as Norton employees, an
indication that Jackbilt employees were also Norton's employees. Furthermore services
rendered in any one of the two companies were taken into account for purposes of
promotion; Compensation given to board members of Jackbilt, indicate that Jackbilt is
merely a department of Norton. The offices of Norton and Jackbilt are located in the
same compound. Payments were effected by Norton of accounts for Jackbilt and vice
versa. Payments were also made to Norton of accounts due or payable to Jackbilt and
vice versa.
Hence, the corporate personality of the two corporations must be disregarded.

80 | P a g e
Law 321_Corporation LAW_ Case Digest

CONCEPT BUILDERS, INC.


vs.
THE NATIONAL LABOR RELATIONS COMMISSION
G.R. No. 108734. May 29, 1996

FACTS:

Petitioner, a domestic corporation, with principal office at 355 Maysan Road,


Valenzuela, Metro Manila, is engaged in the construction business. Private
respondents were employed by said company as laborers, carpenters and riggers.
Eventually, respondent‘s services were terminated. The Labor Arbiter then
rendered judgment ordering petitioner to reinstate private respondents and to pay
them back wages. A writ of execution was then issued but was partially satisfied
because the sheriff reported all the employees inside petitioner's premises at 355
Maysan Road, Valenzuela, Metro Manila, claimed that they were employees of Hydro
Pipes Philippines, Inc and not by respondent. Subsequently, a certain Dennis
Cuyegkeng filed a third-party claim with the Labor Arbiter alleging that the properties
sought to be levied upon by the sheriff were owned by Hydro (Phils.), Inc. of which he
is the Vice-President.
Private respondents filed a "Motion for Issuance of a Break-Open Order,"
alleging that HPPI and petitioner corporation were owned by the same
incorporator/stockholders. They also alleged that petitioner temporarily suspended its
business operations in order to evade its legal obligations to them and that private
respondents were willing to post an indemnity bond to answer for any damages which
petitioner and HPPI may suffer because of the issuance of the break-open order.

ISSUE:

Whether or not petitioner corporation and HPPI are one and the same.

RULING:

YES.

It is a fundamental principle of corporation law that a corporation is an entity


separate and distinct from its stockholders and from other corporations to which it
may be connected. But, this separate and distinct personality of a corporation is
merely a fiction created by law for convenience and to promote justice. So, when the
notion of separate juridical personality is used to defeat public convenience, justify
wrong, protect fraud or defend crime, or is used as a device to defeat the labor laws,
this separate personality of the corporation may be disregarded or the veil of corporate
fiction pierced. This is true likewise when the corporation is merely an adjunct, a
business conduit or an alter ego of another corporation.
The test in determining the applicability of the doctrine of piercing the veil of
corporate fiction is as follows: a. Control, not mere majority or complete stock control,
but complete domination, not only of finances but of policy and business practice in
respect to the transaction attacked so that the corporate entity as to this transaction
had at the time no separate mind, will or existence of its own; b. Such control must
have been used by the defendant to commit fraud or wrong, to perpetuate the violation
of a statutory or other positive legal duty or dishonest and unjust act in contravention
of plaintiff's legal rights; and c. The aforesaid control and breach of duty must
proximately cause the injury or unjust loss complained of.
HPPI is obviously a business conduit of Petitioner Corporation and its
emergence was skillfully orchestrated to avoid the financial liability that already
attached to Petitioner Corporation.

81 | P a g e
Law 321_Corporation LAW_ Case Digest

COMPLEX ELECTRONICS EMPLOYEES ASSOCIATION (CEEA)


vs.
THE NATIONAL LABOR RELATIONS COMMISSION
G.R. No. 121315. July 19, 1999

FACTS:

Complex was engaged in the manufacture of electronic products. It was actually


a subcontractor of electronic products where its customers gave their job orders, sent
their own materials and consigned their equipment to it. Thus, there was the AMS
Line for the Adaptive Micro System, Inc., the Heril Line for Heril Co., Ltd., the Lite-On
Line for the Lite-On Philippines Electronics Co., etc.
The rank and file workers of Complex were organized into a union known as the
Complex Electronics Employees Association, herein referred to as the Union.
Due to its financial reverses, Complex regretfully informed the employees that it was
left with no alternative but to close down the operations of the Lite-On Line. The Union
on the other hand filed a notice of strike with the NCMB. In the evening of April 6,
1992, the machinery, equipment and materials being used for production at Complex
were pulled-out from the company premises and transferred to the premises of Ionics
at Cabuyao, Laguna. The following day, a total closure of company operation was
effected at Complex.
A complaint was, thereafter, filed with the Labor Arbitration Branch of the
NLRC for unfair labor practice. Ionics was impleaded as a party defendant because the
officers and management personnel of Complex were also holding office at Ionics with
Lawrence Qua as the President of both companies.
Complex, on the other hand, averred that since the time the Union filed its
notice of strike, there was a significant decline in the quantity and quality of the
products in all of the production lines. Fearful that the machinery, equipment and
materials would be rendered inoperative and unproductive due to the impending strike
of the workers, the customers ordered their pull-out and transfer to Ionics. Ionics
contended that it was an entity separate and distinct from Complex and had been in
existence 8 years before the labor dispute arose at Complex. While admitting that
Lawrence Qua, the President of Complex was also the President of Ionics, the latter
denied having Qua as their owner since he had no recorded subscription of
P1,200,00.00 in Ionics as claimed by the Union.

ISSUE:

Whether or not Complex and Ionics are one and the same.

RULING:

YES.

Ionics may be engaged in the same business as that of Complex, but this fact
alone is not enough reason to pierce the veil of corporate fiction of the corporation.
Well-settled is the rule that a corporation has a personality separate and distinct from
that of its officers and stockholders. This fiction of corporate entity can only be
disregarded in certain cases such as when it is used to defeat public convenience,
justify wrong, protect fraud, or defend crime. To disregard said separate juridical
personality of a corporation, the wrongdoing must be clearly and convincingly
established.
As to the additional documentary evidence which consisted of a newspaper
clipping filed by petitioner Union, we agree with respondent Ionics that the
photo/newspaper clipping itself does not prove that Ionics and Complex are one and
the same entity. The photo/newspaper clipping merely showed that some plants of
Ionics were recertified to ISO 9002 and does not show that there is a relation between
Complex and Ionics except for the fact that Lawrence Qua was also the president of
Ionics. However, as we have stated above, the mere fact that both of the corporations
have the same president is not in itself sufficient to pierce the veil of corporate fiction
of the two corporations.

82 | P a g e
Law 321_Corporation LAW_ Case Digest

ROSAURA P. CORDON
vs.
JESUS BALICANTA
A.C. No. 2797. October 4, 2002

FACTS:

When her husband died, herein complainant Rosaura Cordon and her daughter
Rosemarie inherited the properties left by the said decedent. All in all, complainant
and her daughter inherited 21 parcels of land located in Zamboanga City. The lawyer
who helped her settle the estate of her late husband was respondent Jesus Balicanta.
Respondent enticed complainant and her daughter to organize a corporation
that would develop the said real properties into a high-scale commercial complex.
Relying on these apparently sincere proposals, complainant and her daughter
assigned 19 parcels of land to Rosaura Enterprises, Incorporated, a newly-formed and
duly registered corporation in which they assumed majority ownership. The subject
parcels of land were then registered in the name of the corporation.
Thereafter, respondent single-handedly ran the affairs of the corporation in his
capacity as Chairman of the Board, President, General Manager and Treasurer. The
respondent also made complainant sign a document which turned out to be a voting
trust agreement. Respondent likewise succeeded in making complainant sign a special
power of attorney to sell and mortgage some of the parcels of land she inherited from
her deceased husband. She later discovered that respondent transferred the titles of
the properties to a certain Tion Suy Ong who became the new registered owner
thereof. Respondent never accounted for the proceeds of said transfers.
In 1981, respondent, using a spurious board resolution, contracted a loan from
the Land Bank of the Philippines in the amount of P2,220,000 using as collateral 9 of
the real properties that the complainant and her daughter contributed to the
corporation. The respondent ostensibly intended to use the money to construct the
Baliwasan Commercial Center.

ISSUE:

Whether or not responden‘s acts will bind the Petitioners.

RULING:

NO.

This Court confirms the duly supported findings of the IBP Board that
respondent committed condemnable acts of deceit against his client. The fraudulent
acts he carried out against his client followed a well thought of plan to misappropriate
the corporate properties and funds entrusted to him. At the very outset, he embarked
on his devious scheme by making himself the President, Chairman of the Board,
Director and Treasurer of the corporation, although he knew he was prohibited from
assuming the position of President and Treasurer at the same time.
Also, respondent denies that he acted as Corporate Secretary aside from being
the Chairman, President and Treasurer of the corporation. Yet respondent submitted
to the investigating commission documents which were supposed to be in the official
possession of the Corporate Secretary alone such as the stock and transfer book and
minutes of meetings.
After a thorough review of the records, we find that respondent committed grave
and serious misconduct that casts dishonor on the legal profession. His
misdemeanors reveal a deceitful scheme to use the corporation as a means to convert
for his own personal benefit properties left to him in trust by complainant and her
daughter. Based on the aforementioned findings, this Court believes that the gravity of
respondent‘s offenses cannot be adequately matched by mere suspension as
recommended by the IBP. Instead, his wrongdoings deserve the severe penalty of
disbarment, without prejudice to his criminal and civil liabilities for his dishonest acts.

83 | P a g e
Law 321_Corporation LAW_ Case Digest

DELPHER TRADES CORPORATION, and DELPHIN PACHECO


vs.
INTERMEDIATE APPELLATE COURT
G.R. No. L-69259. January 26, 1988

FACTS:

Delfin Pacheco and his sister, Pelagia Pacheco, were the owners of real estate
property. The said co-owners leased to Construction Components International Inc.
the same property and providing that during the existence or after the term of this
lease the lessor should he decide to sell the property leased shall first offer the same to
the lessee and the letter has the priority to buy under similar conditions.
Subsequently, lessee assigned its rights and obligations under the contract of lease in
favor of Hydro Pipes Philippines, Inc.
A deed of exchange was executed between Delfin and Pelagia Pacheco and
defendant Delpher Trades Corporation whereby the former conveyed to the latter the
leased property for 2,500 shares of stock of defendant corporation with a total value of
P1,500,000.00. On the ground that it was not given the first option to buy the leased
property pursuant to the proviso in the lease agreement, respondent Hydro Pipes
Philippines, Inc., filed an amended complaint for reconveyance of the property in its
favor under conditions similar to those whereby Delpher Trades Corporation acquired
the property from Pelagia Pacheco and Delphin Pacheco.
Respondents on the other hand stated that there was no transfer of ownership
over the properties.

ISSUE:

Whether or not there was an effective transfer of property in this case.

RULING:

NO.

After incorporation, one becomes a stockholder of a corporation by subscription


or by purchasing stock directly from the corporation or from individual owners thereof.
In the case at bar, in exchange for their properties, the Pachecos acquired 2,500
original unissued no par value shares of stocks of the Delpher Trades Corporation.
Consequently, the Pachecos became stockholders of the corporation by subscription
"The essence of the stock subscription is an agreement to take and pay for original
unissued shares of a corporation, formed or to be formed.‖ It is significant that the
Pachecos took no par value shares in exchange for their properties.
It is to be stressed that by their ownership of the 2,500 no par shares of stock,
the Pachecos have control of the corporation. Their equity capital is 55% as against
45% of the other stockholders, who also belong to the same family group.
In effect, the Delpher Trades Corporation is a business conduit of the Pachecos.
What they really did was to invest their properties and change the nature of their
ownership from unincorporated to incorporated form by organizing Delpher Trades
Corporation to take control of their properties and at the same time save on
inheritance taxes.
The "Deed of Exchange" of property between the Pachecos and Delpher Trades
Corporation cannot be considered a contract of sale. There was no transfer of actual
ownership interests by the Pachecos to a third party. The Pacheco family merely
changed their ownership from one form to another. The ownership remained in the
same hands. Hence, the private respondent has no basis for its claim of a light of first
refusal under the lease contract.

84 | P a g e
Law 321_Corporation LAW_ Case Digest

FRANCISCO V. DEL ROSARIO


vs.
NATIONAL LABOR RELATIONS COMMISSION
G.R. No. 85416. July 24, 1990

FACTS:

In POEA Case No. 85-06-0394, the POEA promulgated a decision dismissing


the complaint for money claims for lack of merit. The decision was appealed to the
NLRC, which reversed the POEA decision and ordered Philsa Construction and
Trading Co., Inc., the recruiter and Arieb Enterprises, the foreign employer to jointly
and severally pay private respondent their salary differentials and vacation leave
benefits.
A writ of execution was issued by the POEA but it was returned unsatisfied as
Philsa was no longer operating and was financially incapable of satisfying the
judgment. Private respondent moved for the issuance of an alias writ against the
officers of Philsa. This motion was opposed by the officers, led by petitioner, the
president and general manager of the corporation.
Petitioner appealed to the NLRC. On September 23, 1988, the NLRC dismissed
the appeal on the theory that the corporate personality of Philsa should be
disregarded. According to the NLRC, Philsa Construction & Trading Co., Inc. and
Philsa International Placement & Services Corp are one and the same because both
corporations has the same set of directors and officers. Petitioner's motion for
reconsideration was denied.
Thus, this petition was filed, alleging that the NLRC gravely abused its discretion.

ISSUE:

Whether or not the NLRC acted with grave abuse of discretion.

RULING:

YES.

Under the law a corporation is bestowed juridical personality, separate and


distinct from its stockholders. But when the juridical personality of the corporation is
used to defeat public convenience, justify wrong, protect fraud or defend crime, the
corporation shall be considered as a mere association of persons and its responsible
officers and/or stockholders shall be held individually liable. For the same reasons, a
corporation shall be liable for the obligations of a stockholder, or a corporation and its
successor-in-interest shall be considered as one and the liability of the former shall
attach to the latter.
But for the separate juridical personality of a corporation to be disregarded, the
wrongdoing must be clearly and convincingly established. It cannot be presumed.
Thus, at the time Philsa allowed its license to lapse in 1985 and even at the time it
was delisted in 1986, there was yet no judgment in favor of private respondent. An
intent to evade payment of his claims cannot therefore be implied from the expiration
of Philsa's license and its delisting. Likewise, substantial identity of the incorporators
of the two corporations does not necessarily imply fraud.
In this case, not only has there been a failure to establish fraud, but it has also
not been shown that petitioner is the corporate officer responsible for private
respondent's predicament. It must be emphasized that the claim for differentials and
benefits was actually directed against the foreign employer. Philsa became liable only
because of its undertaking to be jointly and severally bound with the foreign employer,
an undertaking required by the rules of the POEA, together with the filing of cash and
surety bonds, in order to ensure that overseas workers shall find satisfaction for
awards in their favor.

85 | P a g e
Law 321_Corporation LAW_ Case Digest

FIRST PHILIPPINE INTERNATIONAL BANK


vs.
COURT OF APPEALS
G.R. No. 115849. January 24, 1996

FACTS:

In the course of its banking operations, the defendant Producer Bank of the
Philippines acquired six parcels of land. The original plaintiffs, Demetrio Demetria and
Jose O. Janolo, wanted to purchase the property and thus initiated negotiations for
that purpose.
Negotiations happened between the parties. However, petitioner bank reneged
their agreement because it offered the same lot to different buyers. Plaintiffs then filed
a suit for specific performance with damages against the bank, its Manager Rivers and
Acting Conservator Encarnacion. The basis of the suit was that the transaction had
with the bank resulted in a perfected contract of sale.
Subsequently, Henry L. Co, filed a motion to intervene in the trial court,
alleging that as owner of 80% of the Bank's outstanding shares of stock, he had a
substantial interest in resisting the complaint. The trial court issued an order denying
the motion to intervene on the ground that it was filed after trial had already been
concluded. Henry Co did not appeal the denial of his motion for intervention.
During the pendency of the proceedings in the Court of Appeals, Henry Co and
several other stockholders of the Bank, filed an action purportedly a "derivative suit"
with the RTC Branch 134, against Encarnacion, Demetria and Janolo to declare any
perfected sale of the property as unenforceable and to stop Ejercito from enforcing or
implementing the sale. In his answer, Janolo argued that the Second Case was barred
by litis pendentia by virtue of the case then pending in the Court of Appeals.

ISSUE:

Whether or not the juridical personalities of the two corporations be pierced.

RULING:

YES.

In addition to the many cases where the corporate fiction has been disregarded,
we now add the instant case, and declare herewith that the corporate veil cannot be
used to shield an otherwise blatant violation of the prohibition against forum-
shopping. Shareholders, whether suing as the majority in direct actions or as the
minority in a derivative suit, cannot be allowed to trifle with court processes,
particularly where, as in this case, the corporation itself has not been remiss in
vigorously prosecuting or defending corporate causes and in using and applying
remedies available to it. To rule otherwise would be to encourage corporate litigants to
use their shareholders as fronts to circumvent the stringent rules against forum
shopping.

86 | P a g e
Law 321_Corporation LAW_ Case Digest

FRANCISCO MOTORS
vs.
CA and SPOUSES GREGORIO and LIBRADA MANUEL
G.R. No. 100812. Jun 25, 1999

Facts:

This case arose from the decision o the trial court granting the counter claim of
the herein private respondents. Such counterclaim is based from the fact that
Gregorio Manuel, while he was petitioner‘s Assistant Legal Officer, he represented
members of the Francisco family in the intestate estate proceedings of the late Benita
Trinidad. However, even after the termination of the proceedings, his services were not
paid. Said family members, he said, were also incorporators, directors and officers of
petitioner. Hence to counter petitioner‘s collection suit, he filed a permissive
counterclaim for the unpaid attorney‘s fees.

ISSUE:

Whether or not the petitioner corporation is liable for the attorney‘s fee owing to
the respondents.

RULING:

NO.

Petitioner argued that being a corporation, it should not be held liable therefore
because these fees were owed by the incorporators, directors and officers of the
corporation in their personal capacity as heirs of Benita Trinidad. Petitioner stressed
that the personality of the corporation, vis-à-vis the individual persons who hired the
services of private respondent, is separate and distinct, hence, the liability of said
individuals did not become an obligation chargeable against petitioner.
In this case, the piercing of the corporate veil was not applied because rationale
behind piercing a corporation‘s identity in a given case is to remove the barrier
between the corporation from the persons comprising it to thwart the fraudulent and
illegal schemes of those who use the corporate personality as a shield for undertaking
certain proscribed activities. However, in the case at bar, instead of holding certain
individuals or persons responsible for an alleged corporate act, the situation has been
reversed. It is the petitioner as a corporation which is being ordered to answer for the
personal liability of certain individual directors, officers and incorporators concerned.
Furthermore, according to private respondent Gregorio Manuel his services were
solicited as counsel for members of the Francisco family to represent them in the
intestate proceedings over Benita Trinidad‘s estate. These estate proceedings did not
involve any business of petitioner.
The personality of the corporation and those of its incorporators, directors and
officers in their personal capacities ought to be kept separate in this case. The claim
for legal fees against the concerned individual incorporators, officers and directors
could not be properly directed against the corporation without violating basic
principles governing corporations.

87 | P a g e
Law 321_Corporation LAW_ Case Digest

SOL LAGUIO, RENE LAOLAO, ANNALIZA ENSANDO, EDELIZA ASAS, LILIA


MARAY, EVELYN UNTALAN,* ROSARIO CHICO, REYNALDO GARCIA, MERLITA DE
LOS SANTOS,* JOSEPHINE DERONG,* GEMMA TIBALAO BANTOLO, LUCY
ALMONTE,* CRISPINA VANQUARDIA, NARCISA VENZON, NORMA ELEGANTE,*
AMELIA MORENO,* ABNER PETILOS, NARCISO HILAPO, DOLORES OLAES,
MELINDA LLADOC, ERNA AZARCON, and APRIL TOY, INC. WORKERS UNION –
ALAB
vs.
NATIONAL LABOR RELATIONS COMMISSION, WELL WORLD TOYS, INC., APRIL
TOYS, INC., YU SHENG LING, JENN L. WANG, EUCLIFF CHENG, CHI SHENG LIN,
NENITA C. AGUIRRE, MA. THERESA R. CADIENTE and GLICERIA R. AGUIRRE
G.R. No. 108936. October 4, 1996

FACTS:

Private respondent April Toy, Inc. is a domestic corporation, for the purpose of
"manufacturing, importing, exporting, buying , selling, sub-contracting or otherwise
dealing in, at wholesale and retail," stuffed toys. On December 20, 1989, or after
almost a year of operation, April posted a memorandum 2 within its premises and
circulated a copy of the same among its employees informing them of its dire financial
condition. April decided to shorten its corporate term "up to February 28, 1990,‖
In view of April's cessation of operations, petitioners who initially composed of
seventy-seven employees below filed a complaint for "illegal
shutdown/retrenchment/dismissal and unfair labor practice." On June 21, 1990,
petitioners amended their complaint to implead private respondent Well World Toys,
Inc. (Well World for brevity), a corporation also engaged in the manufacture of stuffed
toys for export.
Petitioners further alleged that the original incorporators and principal officers
of April were likewise the original incorporators of Well World, thus both corporations
should be treated as one corporation liable for their claims. The Labor Arbiter found
as valid the closure of April, and treated April and Well World as two distinct
corporations.

ISSUE:

Whether or not April and Well World are two distinct corporations.

RULING:

YES.

The two corporations have two different set of officers managing their respective
affairs in two separate offices. It is basic that a corporation is invested by law with a
personality separate and distinct from those of the persons composing it as well as
from that of any other legal entity to which it may be related. Mere substantial identity
of the incorporators of the two corporations does not necessarily imply fraud, 15 nor
warrant the piercing of the veil of corporate fiction. In the absence of clear and
convincing evidence that April and Well World's corporate personalities were used to
perpetuate fraud, or circumvent the law said corporations were rightly treated as
distinct and separate from each other.

88 | P a g e
Law 321_Corporation LAW_ Case Digest

RUFINA LUY LIM


vs.
COURT OF APPEALS, AUTO TRUCK TBA CORPORATION, SPEED DISTRIBUTING,
INC., ACTIVE DISTRIBUTORS, ALLIANCE MARKETING CORPORATION, ACTION
COMPANY, INC.
G.R. No. 124715, January 24, 2000

FACTS:

Petitioner Rufina Luy Lim is the surviving spouse of late Pastor Y. Lim whose
estate is the subject of probate proceedings. The respondent herein is the owner of the
properties subject of this. Said properties were included in the inventory of estate late
Pastor Lim. Thus he respondents moved for the exclusion of said properties which was
denied by the trial court. Petitioner contended upon filing an amended petition that
the properties were actually owned by Pastor Lim and the same were registered under
his name, hence they should be included in the inventory of his estate, and that
during his lifetime, he organized and wholly-owned the five corporations, which are
the private respondents in the instant case.

ISSUE:

Whether or not the doctrine of piercing the corporate veil is applicable.

RULING:

NO.

The test in determining the applicability of the doctrine of piercing the veil of
corporate fiction is as follows: 1) Control, not mere majority or complete stock control,
but complete domination, not only of finances but of policy and business practice in
respect to the transaction attacked so that the corporate entity as to this transaction
had at the time no separate mind, will or existence of its own; (2) Such control must
have been used by the defendant to commit fraud or wrong, to perpetuate the violation
of a statutory or other positive legal duty, or dishonest and unjust act in contravention
of plaintiffs legal right; and (3) The aforesaid control and breach of duty must
proximately cause the injury or unjust loss complained of. The absence of any of these
elements prevents piercing the corporate veil.
In this case there is no showing that the elements are present. Furthermore, it
was proven that said properties were registered in the name of the corporation, hence
the same were owned by the corporation despite the fact that, assuming true, it was
Pastor Lim who organized the corporation.

89 | P a g e
Law 321_Corporation LAW_ Case Digest

MATUGUINA INTEGRATED WOOD PRODUCTS, INC.


vs.
The HON. COURT OF APPEALS, DAVAO ENTERPRISES CORPORATION, The HON.
MINISTER, (NOW SECRETARY) of NATURAL RESOURCES AND PHILLIP CO.
G.R. No. 98310. October 24, 1996

FACTS:

On June 28, 1973, the Acting Director of the Bureau of Forest Development
issued Provisional Timber License (PTL) No. 30, covering an area of 5,400 hectares to
Ms. Milagros Matuguina who was then doing business under the name of MLE, a sole
proprietorship venture. A portion, covering 1,900 hectares, of the said area was
located within the territorial boundary of Gov. Generoso in Mati, Davao Oriental, and
adjoined the timber concession of Davao Enterprises Corporation (DAVENCOR), the
private respondent.
On July 17, 1975, Milagros Matuguina and petitioner MIWPI executed a Deed of
Transfer 5 transferring all of the former's rights, interests, ownership and participation
in Provincial Timber License No. 30 to the latter for and in consideration of 148,000
shares of stocks in MIWPI.
On July 28, 1975, pending approval of the request to transfer the PTL to
MIWPI, DAVENCOR, through its Assistant General Manager, complained to the
District Forester at Mati, Davao Oriental that Milagros Matuguina/MLE had
encroached into and was conducting logging operations in DAVENCOR's timber
concession.

ISSUE:

Whether or not MLE and MIWPI are separate and distinct corporations.

RULING:

YES.

It is settled that a corporation is clothed with personality separate and distinct


from that of the persons composing it. It may not generally be held liable for that of
the persons composing it. It may not be held liable for the personal indebtedness of its
stockholders or those of the entities connected with it. Conversely, a stockholder
cannot be made to answer for any of its financial obligations even if he should be its
president. But when the juridical personality of the corporation is used to defeat
public convenience, justify wrong, protect fraud or defend crime, the corporation shall
be considered as a mere association of persons, and its responsible officers and/or
stockholders shall be individually. For the same reasons, a corporation shall be liable
for the obligations of a stockholder, or a corporation and its successor-in-interest shall
be considered as one and the liability of the former shall attach to the latter.
But for the separate juridical personality of a corporation to be disregarded, the
wrongdoing must be clearly and convincingly established. It cannot be presumed.
In the case at bar, there is, insufficient basis for the appellate court's ruling
that MIWPI is the same as Matuguina. The alleged control of Plaintiff Corporation was
not evident in any particular corporate acts of Plaintiff Corporation, wherein Maria
Milagros Matuguina Logging Enterprises is using Plaintiff Corporation, executed acts
or powers directly involving Plaintiff Corporation. Also, mere ownership by a single
stockholder or by another corporation of all or nearly all of the capital stocks of the
corporation, is not itself a sufficient warrant for disregarding the fiction of separate
personality.

90 | P a g e
Law 321_Corporation LAW_ Case Digest

THE MANILA HOTEL CORP. AND MANILA HOTEL INTL. LTD.


vs.
NATIONAL LABOR RELATIONS COMMISSION, ARBITER CEFERINA J. DIOSANA
AND MARCELO G. SANTOS
G.R. No. 120077. October 13, 2000

FACTS:

MHICL is a corporation duly organized and existing under the laws of Hong
Kong. MHC is an ―incorporator‖ of MHICL, owning 50% of its capital stock. By virtue of
a ―management agreement‖ with the Palace Hotel (Wang Fu Company Limited), MHICL
trained the personnel and staff of the Palace Hotel at Beijing, China.
Respondent Santos accepted an employment offer from Palace Hotel. On
November 5, 1988, respondent Santos left for Beijing, China. He started to work at the
Palace Hotel. A year later he received a letter stating that his employment is being
terminated due to business reverses brought about by the political upheaval in China.
On February 20, 1990, respondent Santos filed a complaint for illegal dismissal.

ISSUE:

Whether or not the doctrine of piercing the corporate veil is available to make
MHC liable for damages.

RULING:

NO.

MHC, as a separate and distinct juridical entity cannot be held liable. True,
MHC is an incorporator of MHICL and owns fifty percent (50%) of its capital stock.
However, this is not enough to pierce the veil of corporate fiction between MHICL and
MHC.
Piercing the veil of corporate entity is an equitable remedy. It is resorted to
when the corporate fiction is used to defeat public convenience, justify wrong, protect
fraud or defend a crime. It is done only when a corporation is a mere alter ego or
business conduit of a person or another corporation.
In Traders Royal Bank v. Court of Appeals, the court held that ―the mere
ownership by a single stockholder or by another corporation of all or nearly all of the
capital stock of a corporation is not of itself a sufficient reason for disregarding the
fiction of separate corporate personalities.‖
The tests in determining whether the corporate veil may be pierced are: First,
the defendant must have control or complete domination of the other corporation‘s
finances, policy and business practices with regard to the transaction attacked. There
must be proof that the other corporation had no separate mind, will or existence with
respect the act complained of. Second, control must be used by the defendant to
commit fraud or wrong. Third, the aforesaid control or breach of duty must be the
proximate cause of the injury or loss complained of. The absence of any of the
elements prevents the piercing of the corporate veil.
It is basic that a corporation has a personality separate and distinct from those
composing it as well as from that of any other legal entity to which it may be related.
Clear and convincing evidence is needed to pierce the veil of corporate fiction. In this
case, the court found no evidence to show that MHICL and MHC are one and the same
entity.

91 | P a g e
Law 321_Corporation LAW_ Case Digest

SAN JUAN STRUCTURAL AND STEEL FABRICATORS, INC.


vs.
COURT OF APPEALS, MOTORICH SALES CORPORATION, NENITA LEE
GRUENBERG, ACL DEVELOPMENT CORP. and JNM REALTY AND DEVELOPMENT
CORP.
G.R. No. 129459. September 29, 1998

FACTS:

A parcel of land was sold by Nenita Lee Gruenberg, the corporate treasurer of
defendant corporation Motorich Sale in favor of San Juan Structural and Steel
Fabricators, Inc. However, the latter failed to execute the necessary Transfer of
Rights/Deed of Assignment in favor of plaintiff-appellant. Hence a case for damages
was filed. The defendant corporation questions the validity of the contract entered by
its treasurer in its behalf without authorization from the corporation‘s Board.

ISSUE:

Whether or not the doctrine of piercing the veil of corporate fiction be applied to
Motorich.

RULING:

NO.

The contract cannot bind Motorich, because it never authorized or ratified such
sale. A corporation is a juridical person separate and distinct from its stockholders or
members. Accordingly, the property of the corporation is not the property of its
stockholders or members and may not be sold by the stockholders or members
without express authorization from the corporation‘s board of directors. The
corporation may act only through its board of directors, or, when authorized either by
its bylaws or by its board resolution, through its officers or agents in the normal
course of business. The general principles of agency govern the relation between the
corporation and its officers or agents, subject to the articles of incorporation, bylaws,
or relevant provisions of law.
As to the piercing of the corporate veil, the same is not applicable. In the
present case, the Court finds no reason to pierce the corporate veil of Respondent
Motorich. Petitioner utterly failed to establish that said corporation was formed, or
that it is operated, for the purpose of shielding any alleged fraudulent or illegal
activities of its officers or stockholders; or that the said veil was used to conceal fraud,
illegality or inequity at the expense of third persons, like petitioner.

92 | P a g e
Law 321_Corporation LAW_ Case Digest

TAN BOON BEE & CO., INC.


vs.
THE HONORABLE HILARION U. JARENCIO, PRESIDING JUDGE OF BRANCH
XVIII of the Court of First Instance of Manila, GRAPHIC PUBLISHING, INC., and
PHILIPPINE AMERICAN CAN DRUG COMPANY
G.R. No. L-41337. June 30, 1988

FACTS:

Petitioner herein, doing business under the name and style of Anchor Supply
Co., sold on credit to herein private respondent Graphic Publishing, Inc. (GRAPHIC)
paper products amounting to P55,214.73. On December 20, 1972, GRAPHIC made
partial payment by check to petitioner in the total amount of P24,848.74; and on
December 21, 1972, a promissory note was executed to cover the balance of
P30,365.99. In the said promissory note, it was stipulated that the amount will be
paid on monthly installments and that failure to pay any installment would make the
amount immediately demandable with an interest of 12% per annum. On September
6, 1973, for failure of GRAPHIC to pay any installment, petitioner filed a complaint for
collection of Sum of Money.
A decision was rendered and became final and executory, where one (1) unit
printing machine identified as "Original Heidelberg Cylinder Press" Type H 222, NR
78048, found in the premises of GRAPHIC was levied. However, a third party claim
was filed by Philippine American Drug Company (PADCO), hence after trial the levy
was rendered to be without force.

ISSUE:

Whether or not the properties of PADCO could be levied due to the allegation
that it is mere an adjunct or conduit of Graphic.

RULING:

YES.

In the instant case, petitioner's evidence established that PADCO was never
engaged in the printing business; that the board of directors and the officers of
GRAPHIC and PADCO were the same; and that PADCO holds 50% share of stock of
GRAPHIC. Petitioner likewise stressed that PADCO's own evidence shows that the
printing machine in question had been in the premises of GRAPHIC since May, 1965,
long before PADCO even acquired its alleged title on July 11, 1966 from Capitol
Publishing. That the said machine was allegedly leased by PADCO to GRAPHIC on
January 24, 1966, even before PADCO purchased it from Capital Publishing on July
11, 1966, only serves to show that PADCO's claim of ownership over the printing
machine is not only farce and sham but also unbelievable.
Considering the aforestated principles and the circumstances established in
this case, respondent judge should have pierced PADCO's veil of corporate identity.

93 | P a g e
Law 321_Corporation LAW_ Case Digest

TELEPHONE ENGINEERING & SERVICE COMPANY, INC.


vs.
WORKMEN'S COMPENSATION COMMISSION, PROVINCIAL SHERIFF OF RIZAL
and LEONILA SANTOS GATUS, for herself and in behalf of her minor children,
Teresita, Antonina and Reynaldo, all surnamed GATUS
G.R. No. L-28694. May 13, 1981

FACTS:

Utilities Management Corporation (UMACOR), the sister company of the


petitioner hired the late Pacifica L. Gatus as Purchasing Agent. The latter died due to
liver cirrhosis with malignant degeneration. His widow, respondent Leonila S. Gatus,
filed a "Notice and Claim for Compensation" Workmen's Compensation Section,
alleging therein that her deceased husband was an employee of TESCO, and that he
died of liver cirrhosis.
TESCO, in its reply, contended that the cause of the illness contracted by Gatus
was in no way aggravated by the nature of his work. TESCO takes the position that
there was no employer-employee relationship between them, the deceased having been
an employee of UMACOR and not of TESCO.

ISSUE:

Whether or not the contentions of TESCO is tenable.

RULING:

NO.

The court ruled that indeed TESCO is estopped from raising the defense of non-
existence of employer-employee relationship because such was raised only in the
petition for the first time. It was considered by the court as a mere afterthought to
evade liability. It was also seen that in its initial pleadings it did not deny that it is the
employer of the decedent. Petitioner even admitted that TESCO and UMACOR are
sister companies operating under one single management and housed in the same
building. Although respect for the corporate personality as such, is the general rule,
there are exceptions. In appropriate cases, the veil of corporate fiction may be pierced
as when the same is made as a shield to confuse the legitimate issues.

94 | P a g e
Law 321_Corporation LAW_ Case Digest

BUENAFLOR C. UMALI, MAURICIA M. VDA. DE CASTILLO, VICTORIA M.


CASTILLO, BERTILLA C. RADA, MARIETTA C. ABAÑEZ, LEOVINA C. JALBUENA
and SANTIAGO M. RIVERA
vs.
COURT OF APPEALS, BORMAHECO, INC. and PHILIPPINE MACHINERY PARTS
MANUFACTURING CO., INC.
G.R. No. 89561. September 13, 1990

FACTS:

Santiago Rivera is the nephew of plaintiff Mauricia Meer Vda. de Castillo. The
Castillo family is the owners of a parcel of land located in Lucena City which was given
as security for a loan from the Development Banks of the Philippines. For their failure
to pay the amortization, foreclosure of the said property was about to be initiated. This
problem was made known to Santiago Rivera, who proposed to them the conversion
into subdivision of the four (4) parcels of land adjacent to the mortgaged property to
raise the necessary fund. The idea was accepted by the Castillo family and to carry out
the project, a Memorandum of Agreement was executed by and between Slobec Realty
and Development, Inc., represented by its President Santiago Rivera and the Castillo
family. In this agreement, Santiago Rivera obliged himself to pay the Castillo family
the sum of P70,000.00 immediately after the execution of the agreement and to pay
the additional amount of P400,000.00 after the property has been converted into a
subdivision. Rivera, armed with the agreement, approached Mr. Modesto Cervantes,
President of defendant Bormaheco, and proposed to purchase from Bormaheco two (2)
tractors Model D-7 and D-8. Subsequently, a Sales Agreement was executed on
December 28, 1970, which was accepted by the latter and executed Sales Agreement.
The balance of the consideration was secured by a surety bond from ICP (Insurance
Corporation of the Phil.) which was in turn secured by a mortagage, the properties of
the Castillos.

ISSUE:

Whether or not the doctrine of piercing the veil of corporate fiction is applicable.

RULING:

NO.

Petitioners seek to pierce the veil of corporate entity of Bormaheco, ICP and PM
Parts, alleging that these corporations employed fraud in causing the foreclosure and
subsequent sale of the real properties belonging to petitioners.
In the instant case, petitioners do not seek to impose a claim against the
individual members of the three corporations involved; on the contrary, it is these
corporations which desire to enforce an alleged right against petitioners. Assuming
that petitioners were indeed defrauded by private respondents in the foreclosure of the
mortgaged properties, this fact alone is not, under the circumstances, sufficient to
justify the piercing of the corporate fiction, since petitioners do not intend to hold the
officers and/or members of respondent corporations personally liable therefore.
Petitioners are merely seeking the declaration of the nullity of the foreclosure sale,
which relief may be obtained without having to disregard the aforesaid corporate
fiction attaching to respondent corporations. Secondly, petitioners failed to establish
by clear and convincing evidence that private respondents were purposely formed and
operated, and thereafter transacted with petitioners, with the sole intention of
defrauding the latter.
The mere fact, therefore, that the businesses of two or more corporations are
interrelated is not a justification for disregarding their separate personalities, absent
sufficient showing that the corporate entity was purposely used as a shield to defraud
creditors and third persons of their rights.

95 | P a g e
Law 321_Corporation LAW_ Case Digest

VLASON ENTERPRISES CORPORATION


vs.
COURT OF APPEALS and DURAPROOF SERVICES, represented by its General
Manager, Cesar Urbino Sr.
G.R. Nos. 121662-64. July 6, 1999

FACTS:

Poro Point Shipping Services, then acting as the local agent of Omega Sea
Transport Company of Honduras & Panama, a Panamanian company, (hereafter
referred to as Omega), requested permission for its vessel M/V Star Ace, which had
engine trouble, to unload its cargo and to store it at the Philippine Ports Authority
(PPA) compound in San Fernando, La Union while awaiting transhipment to
Hongkong. The request was approved by the Bureau of Customs. Despite the
approval, the customs personnel boarded the vessel when it docked on January 7,
1989, on suspicion that it was the hijacked M/V Silver Med owned by Med Line
Philippines Co., and that its cargo would be smuggled into the country. The district
customs collector seized said vessel and its cargo pursuant to Section 2301, Tariff and
Customs Code.
They entered into a salvage agreement with private respondent to secure and
repair the vessel which was destroyed by the typhoons that hit the province at the
agreed consideration of $1 million and ―fifty percent (50%) of the cargo after all
expenses, cost and taxes.‖ Subsequently, the seizure was lifted for want of fraud. To
enforce its preferred salvor‘s lien, herein Private Respondent Duraproof Services filed
with the Regional Trial Court of Manila a Petition for Certiorari, Prohibition and
Mandamus assailing the actions of Commissioner Mison and District Collector Sy.

ISSUE:

Whether or not the doctrine of piercing the corporate veil is applicable.

RULING:

NO.

In the present case, Bebero was the secretary of Angliongto, who was president
of both VSI and petitioner, but she was an employee of VSI, not of petitioner. The
piercing of the corporate veil cannot be resorted to when serving summons.
Doctrinally, a corporation is a legal entity distinct and separate from the members and
stockholders who compose it. However, when the corporate fiction is used as a means
of perpetrating a fraud, evading an existing obligation, circumventing a statute,
achieving or perfecting a monopoly or, in generally perpetrating a crime, the veil will be
lifted to expose the individuals composing it. None of the foregoing exceptions has
been shown to exist in the present case. Quite the contrary, the piercing of the
corporate veil in this case will result in manifest injustice.

96 | P a g e
Law 321_Corporation LAW_ Case Digest

VILLA REY TRANSIT, INC.


vs.
EUSEBIO E. FERRER, PANGASINAN TRANSPORTATION CO., INC. and PUBLIC
SERVICE COMMISSION
EUSEBIO E. FERRER and PANGASINAN TRANSPORTATION CO., INC.

PANGASINAN TRANSPORTATION CO., INC.


vs.
JOSE M. VILLARAMA
G.R. No. L-23893. October 29, 1968

FACTS:

Prior to 1959, Jose M. Villarama was an operator of a bus transportation, under


the business name of Villa Rey Transit, pursuant to certificates of public convenience
granted him by the Public Service Commission (PSC, for short) in Cases Nos. 44213
and 104651, which authorized him to operate a total of thirty-two (32) units on
various routes or lines from Pangasinan to Manila, and vice-versa. On January 8,
1959, he sold the aforementioned two certificates of public convenience to the
Pangasinan Transportation Company, Inc. (Pantranco), for P350,000.00 with the
condition, among others, that the seller (Villarama) "shall not for a period of 10 years
from the date of this sale, apply for any TPU service identical or competing with the
buyer."
Barely three months thereafter, or on March 6, 1959, a corporation called Villa
Rey Transit, Inc. was organized. In less than a month after its registration, it bought
five certificates of public convenience, forty-nine buses, tools and equipment from one
Valentin Fernando. Before the PSC could take final action on said application for
approval of sale, however, the Sheriff of Manila, on July 7, 1959, levied on two of the
five certificates of public convenience involved therein pursuant to a writ of execution
issued by the Court of First Instance of Pangasinan in Civil Case No. 13798, in favor of
Eusebio Ferrer, plaintiff, judgment creditor, against Valentin Fernando, defendant,
judgment debtor. The Sheriff made and entered the levy in the records of the PSC. On
July 16, 1959, a public sale was conducted by the Sheriff of the said two certificates of
public convenience.

ISSUE:

Whether or not the doctrine of piercing the corporate veil is applicable.

RULING:

YES.

The doctrine that a corporation is a legal entity distinct and separate from the
members and stockholders who compose it is recognized and respected in all cases
which are within reason and the law. 29 When the fiction is urged as a means of
perpetrating a fraud or an illegal act or as a vehicle for the evasion of an existing
obligation, the circumvention of statutes, the achievement or perfection of a monopoly
or generally the perpetration of knavery or crime, the veil with which the law covers
and isolates the corporation from the members or stockholders who compose it will be
lifted to allow for its consideration merely as an aggregation of individuals.
Upon the foregoing considerations, the Court so held that the preponderance of
evidence have shown that the Villa Rey Transit, Inc. is an alter ego of Jose M.
Villarama, and that the restrictive clause in the contract entered into by the latter and
Pantranco is also enforceable and binding against the said Corporation. For the rule is
that a seller or promissor may not make use of a corporate entity as a means of
evading the obligation of his covenant. Where the Corporation is substantially the
alter ego of the covenantor to the restrictive agreement, it can be enjoined from
competing with the covenantee.

97 | P a g e
Law 321_Corporation LAW_ Case Digest

De Facto Corporation

C. ARNOLD HALL and BRADLEY P. HALL, petitioners,


vs.
EDMUNDO S. PICCIO, Judge of the Court of First Instance of Leyte, FRED
BROWN, EMMA BROWN, HIPOLITA CAPUCIONG, in his capacity as receiver of the
Far Eastern Lumber and Commercial Co., Inc., respondents.
G.R. No. L-2598. June 29, 1950

FACTS:

In 1947, the petitioners and the respondents signed and acknowledged in Leyte,
the article of incorporation of the Far Eastern Lumber and Commercial Co., Inc.,
organized to engage in a general lumber business to carry on as general contractors,
operators and managers, etc. Attached to the article was an affidavit of the treasurer
stating that 23,428 shares of stock had been subscribed and fully paid with certain
properties transferred to the corporation described in a list appended thereto.
Immediately after the execution of said articles of incorporation, the corporation
proceeded to do business with the adoption of by-laws and the election of its officers.
In 1947, the said articles of incorporation were filed in the office of the SEC for
the issuance of the corresponding certificate of incorporation. Thereafter, pending
action on the articles of incorporation by the SEC, the respondents filed before the
Court of First Instance of Leyte a civil case, alleging among other things that the Far
Eastern Lumber and Commercial Co. was an unregistered partnership; that they
wished to have it dissolved because of bitter dissension among the members,
mismanagement and fraud by the managers and heavy financial losses. The
petitioners alleged that the court had no jurisdiction over the civil case decree the
dissolution of the company, because it being a de facto corporation, dissolution thereof
may only be ordered in a quo warranto proceeding instituted in accordance with
section 19 of the Corporation Law.

ISSUES:

Whether or not the Far Eastern Lumber and Commercial Co., Inc. is a de facto
corporation.

RULING:

NO.

Inasmuch as the Far Eastern Lumber and Commercial Co., is a de facto


corporation, section 19 of the Corporation Law applies, and therefore the court had
not jurisdiction to take cognizance of said civil case.
There are least two reasons why this section does not govern the situation. (1)
First, not having obtained the certificate of incorporation, the Far Eastern Lumber and
Commercial Co. — even its stockholders — may not probably claim "in good faith" to
be a corporation.
Under our statue it is to be noted that it is the issuance of a certificate of
incorporation by the Director of the Bureau of Commerce and Industry (now SEC)
which calls a corporation into being. The immunity if collateral attack is granted to
corporations "claiming in good faith to be a corporation under this act." Such a claim
is compatible with the existence of errors and irregularities; but not with a total or
substantial disregard of the law. Unless there has been an evident attempt to comply
with the law the claim to be a corporation "under this act" could not be made "in good
faith."
(2) Second, this is not a suit in which the corporation is a party. This is a
litigation between stockholders of the alleged corporation, for the purpose of obtaining
its dissolution. Even the existence of a de jure corporation may be terminated in a
private suit for its dissolution between stockholders, without the intervention of the
state.

98 | P a g e
Law 321_Corporation LAW_ Case Digest

Corporation by Estoppel

INTERNATIONAL EXPRESS TRAVEL & TOUR SERVICES


vs.
HON. COURT OF APPEALS, HENRI KAHN, PHILIPPINE FOOTBALL FEDERATION
G.R. No. 119002. October 19, 2000

FACTS:

Petitioner International Express Travel and Tour Services, Inc., through its
managing director, wrote a letter to the Philippine Football Federation (Federation),
through its president private respondent Henri Kahn, wherein the former offered its
services as a travel agency to the latter, which was accepted. Petitioner secured the
airline tickets for the trips of the athletes and officials of the Federation which
amounted to P449,654.83. For failure to pay the unpaid amount after demands, the
petitioner filed a collection case against Henri Kahn in his personal capacity and as
President of the Federation and impleaded the Federation as an alternative defendant.
Kahn denied liability and averred that it merely acted as the agent of the Federation
and did not guaranty the payment of the purchased tickets. The trial court ruled
against Kahn.

ISSUE:

Whether or not Kahn is personally liable.

RULING:

YES.

Kahn avers that he should not be made personally liable because it should be
the Federation, as a corporation having juridical existence, which must be held liable.
He merely acted as an agent of the latter.
The Court was not persuaded. It ruled that under R.A. 3135, and the
Department of Youth and Sports Development under P.D. 604, for a Federation to
acquire juridical existence it is a requirement that the federation must be recognized
by the accrediting organization, the Philippine Amateur Athletic Federation. And Kahn
failed to prove that such requirement was complied with by the Federation. It is a
settled principal in corporation law that any person acting or purporting to act on
behalf of a corporation which has no valid existence assumes such privileges and
becomes personally liable for contract entered into or for other acts performed as such
agent.1 As president of the Federation, Henri Kahn is presumed to have known about
the corporate existence or non-existence of the Federation.

99 | P a g e
Law 321_Corporation LAW_ Case Digest

LIM TONG LIM


vs.
PHILIPPINE FISHING GEAR INDUSTRIES, INC.
1999 Nov 3, G.R. No. 136448

FACTS:

On behalf of "Ocean Quest Fishing Corporation," Antonio Chua and Peter Yao
entered into a Contract for the purchase of fishing nets of various sizes from the
Philippine Fishing Gear Industries, Inc. They claimed that they were engaged in a
business venture with Petitioner Lim Tong Lim, who however was not a signatory to
the agreement. They, however, failed to pay; hence, private respondent filed a
collection suit against Chua, Yao and Petitioner Lim Tong Lim with a prayer for a writ
of preliminary attachment. The suit was brought against the three in their capacities
as general partners, on the allegation that "Ocean Quest Fishing Corporation" was a
nonexistent corporation
Yao and Chua admitted liability while Lim filed his answer. Trial court rendered
decision ruling that Philippine Fishing Gear Industries was entitled to the Writ of
Attachment and that Chua, Yao and Lim, as general partners, were jointly liable to
pay respondent.

ISSUE:

Whether or not Lim should be made jointly liable with Yao and Chua.

RULING:

YES.

Lim asserts that he should not be made liable because there was no
partnership existing between them.
The court ruled that there exist a partnership between them. It is clear that
Chua, Yao and Lim had decided to engage in a fishing business, which they started by
buying boats worth P3.35 million, financed by a loan secured from Jesus Lim who was
petitioner's brother. In their Compromise Agreement, they subsequently revealed their
intention to pay the loan with the proceeds of the sale of the boats, and to divide
equally among them the excess or loss. These boats, the purchase and the repair of
which were financed with borrowed money, fell under the term "common fund" under
Article 1767. The contribution to such fund need not be cash or fixed assets; it could
be an intangible like credit or industry. That the parties agreed that any loss or profit
from the sale and operation of the boats would be divided equally among them also
shows that they had indeed formed a partnership.
Moreover, it is clear that the partnership extended not only to the purchase of
the boat, but also to that of the nets and the floats. The fishing nets and the floats,
both essential to fishing, were obviously acquired in furtherance of their business. It
would have been inconceivable for Lim to involve himself so much in buying the boat
but not in the acquisition of the aforesaid equipment, without which the business
could not have proceeded.

100 | P a g e
Law 321_Corporation LAW_ Case Digest

MARIANO A. ALBERT
vs.
UNIVERSITY PUBLISHING CO., INC.
G.R. No. L-19118, January 30, 1965

FACTS:

In the original case, the court had awarded P P15,000.00 in favor of the
petitioner for damages arising out of a breach of contract. Such breach of contract
arose when the publishing company failed to pay the petitioner the agreed amount for
latter to have the exclusive right to publish his revised Commentaries on the Revised
Penal Code and for his share in previous sales of the book's first edition. The order
became final and executory. A writ of execution was issued against the company,
however the petitioner petitioned for a writ of execution against Jose M. Aruego, as the
real defendantstating, plaintiff's counsel and the Sheriff of Manila discovered that
there is no such entity as University Publishing Co., Inc. and no such entity is
registered with the SEC.
This case asks the court whether or not the judgment may be executed against
Jose M. Aruego, supposed President of University Publishing Co., Inc., as the real
defendant.

ISSUE:

Whether or not the judgment may be executed against Jose M. Aruego,


supposed President of University Publishing Co., Inc., as the real defendant.

RULING:

NO.

The Court ruled that the doctrine of corporation by estoppel was not applicable.
Although the rule is that a person acting or purporting to act on behalf of a
corporation which has no valid existence assumes such privileges and obligations and
becomes personally liable for contracts entered into or for other acts performed as
such agent, in this case, Aruego was not named as a defendant. Since he was not
named, he could not be served and be made liable for the claim because to do so
would violate his right to due process. He was not given the chance to defend himself
and be heard during trial.
Wherefore, the order was reversed and set aside and was remanded lower court
to hold supplementary proceedings for the purpose of carrying the judgment into effect
against University Publishing Co., Inc. and/or Jose M. Aruego.

101 | P a g e
Law 321_Corporation LAW_ Case Digest

Non-User of Charter vs. Continuous Inoperation

LOYOLA GRAND VILLAS HOMEOWNERS (SOUTH) ASSOCIATION, INC.


vs.
HON. COURT OF APPEALS
1997 Aug 7, G.R. No. 117188

FACTS:

LGVHAI was organized as the association of homeowners and residents of the


Loyola Grand Villas. It was registered with the Home Financing Corporation. For
unknown reasons, however, LGVHAI did not file its corporate by-laws. Sometime in
1988, the officers of the LGVHAI tried to register its by-laws. They failed to do so. They
later discovered that there were two other organizations within the subdivision the
North Association and the South Association. According to private respondents, a non-
resident and Soliven himself respectively headed these associations. They also
discovered that these associations had five (5) registered homeowners each who were
also the incorporators, directors and officers thereof. None of the members of the
LGVHAI was listed as member of the North Association while three (3) members of
LGVHAI were listed as members of the South Association. When they inquired as to
the status of LGVHAI, the head of the legal department of the HIGC, informed him that
LGVHAI had been automatically dissolved for two reasons. First, it did not submit its
by-laws within the period required by the Corporation Code and, second, there was
non-user of corporate charter because HIGC had not received any report on the
association's activities. These prompted the LGVHAI to lodge complaint with HIGC
questioning its act of revoking its certificate of registration without due notice and
hearing and concomitantly prayed for the cancellation of the certificates of registration
of the North and South Associations by reason of the earlier issuance of a certificate of
registration in favor of LGVHAI.

ISSUE:

Whether or not the failure of a corporation to file its by-laws within one month
from the date of its incorporation, as mandated by Section 46 of the Corporation Code,
result in its automatic dissolution.

RULING:

NO.

Although the Corporation Code requires the filing of by-laws, it does not
expressly provide for the consequences of the non-filing of the same within the period
provided for in Section 46. Even under the express grant of power and authority under
Presidential Decree No. 902-A, there can be no automatic corporate dissolution simply
because the incorporators failed to abide by the required filing of by-laws embodied in
Section 46 of the Corporation Code. There is no outright "demise" of corporate
existence. Proper notice and hearing are cardinal components of due process in any
democratic institution, agency or society. In other words, the incorporators must be
given the chance to explain their neglect or omission and remedy the same.

102 | P a g e
Law 321_Corporation LAW_ Case Digest

BOARD OF DIRECTORS
Qualifications/Qualifying Shares

REP. LUIS R. VILLAFUERTE, et al.


vs.
GOV. OSCAR S. MORENO, et al.
G.R. No. 186566, October 2, 2009

FACTS:

As a result of the Tokyo Communique, which unified the feuding Basketball


Association of the Philippines ("BAP") and the newly formed Pilipinas Basketbol ("PB"),
the Samahang Basketbol ng Pilipinas, Inc. ("SBP") was established and its constitutive
documents consisting of the Articles of Incorporation were signed by the five (5)
incorporators, which include petitioner Pangilinan. On the same day, the
incorporators likewise passed and signed its by-laws. Subsequently, the three-man
panel met in Bangkok, Thailand where it forged and executed a Memorandum of
Agreement ("Bangkok Agreement") integrating therein the final terms and conditions of
the unity and merger of BAP and PB. Then came the nomination and election of its
transitory officers for the years 2007-2008 the results of which had led to the
proclamation of respondent Villafuerte as Chairman. Petitioner raised its opposition
and did not recognize the election of respondent Villafuerte as Chairman of BAP-SBP
on account of the alleged failure of the latter to qualify for the said position. As a result
of this, two elections were held by the different factions for the positions in the Board
of Trustees. Petitioners filed before the Regional Trial Court of Manila a petition 5 for
declaration of nullity of the election of respondents as members of the Board of
Trustees and Officers of BAP-SBP. The trial court rendered decision in favor of the
petitioners.

ISSUE:

Whether or not Villafuerte is qualified as a Director.

RULING:

NO.

Respondents asserted that Villafuerte never assumed the position of Chairman


of the BAP-SBP because he failed to qualify for the same; that before Villafuerte could
legally assume the Chairmanship of BAP-SBP, he must first be elected a member of
the Board of Trustees.
As correctly pointed out by CA, petitioner Villafuerte‘s nomination must of
necessity be understood as being subject to or in accordance with the qualifications
set forth in the By-Laws of the BAP-SBP. Since the said by-laws require the Chairman
of the Board of Trustees to be a trustee himself, petitioner Villafuerte was not qualified
since he had neither been elected nor appointed as one of the trustees of BAP-SBP. In
other words, petitioner Villafuerte never validly assumed the position of Chairman
because he failed in the first place to qualify therefore.

103 | P a g e
Law 321_Corporation LAW_ Case Digest

CONSTANCIO T. BAGUIO
vs.
LAS PALMAS INTERNATIONAL MANPOWER CORPORATION, SPOUSES DONALDO
PALMA AND CONSUELO P. PALMA and CYNTHIA C. CALAPRE
G.R. No. 93417, September 14, 1993

FACTS:

Petitioner claims that he bought 600 shares of stocks from the respondent
corporation for the amount of P60, 000.00. He further avers that the respondents
failed, for such a long time, to deliver the certificates of stocks corresponding to the
stocks he bought. He thus sought for the reimbursement of return of the P60, 000.00
he allegedly paid to the corporation. The respondents denied receiving any payment
from the petitioner. Petitioner forwards the fact that the corporation had adopted a
resolution recognizing him as being a stockholder owning 600 shares, and was further
appointed as vice-president, hence the respondents cannot deny the fact that they
have received the payment. There was no receipt presented because when petitioner
asked for one, respondents Palmas assured him that the board resolution and the
secretary's certificate were better evidence of payment than an ordinary receipt. He
was likewise told that the stock certificate would be issued in December 1982, after
the board meeting. Respondents Palmas used the money to pay their employees,
whose salaries had not been paid for several months. The trial court ruled for the
petitioner, but the CA reversed the same.

ISSUE:

Whether or not petitioner actually paid respondent Palmas the sum of


P60,000.00, the price of the shares of stock sold to him.

RULING:

NO.

The court ruled that there was no payment. The resolution adopted by the
Board does not speak of any sales transaction and receipt of payment. It merely states
that the petitioner was accepted as a stockholder to the corporation. Even assuming
that a transaction between the petitioners and the spouses Palma transpired, the
corporation had nothing to do with the business transactions entered by its officers in
their personal capacity and petitioner.
Furthermore, petitioner cannot claim that being a member of the board of
directors and occupying the position of Vice-President-International necessarily imply
that he must have owned duly-paid shares of stock. The election of a person to the
board of directors of a corporation does not necessarily mean that he has paid for the
shares recorded in his name. In most cases, nominee directors do not pay for the
qualifying shares assigned to them. Likewise, the Corporation Code does not require
that one elected or appointed as vice-president of a corporation should be the owner of
shares of stock of the corporation.

104 | P a g e
Law 321_Corporation LAW_ Case Digest

DETECTIVE & PROTECTIVE BUREAU, INC.


vs.
HON. CLORIBEL and FAUSTO S. ALBERTO
1968 Nov 29, G.R. No. L-23428

FACTS:

Plaintiff filed against herein private defendant a complaint for accounting with
preliminary injunction and receivership. It alleged that defendant was managing
director of plaintiff corporation from 1952 until January 14, 1964; that in June 1963,
defendant illegally seized and took control of all the assets as well as the books,
records, vouchers and receipts of the corporation from the accountant-cashier,
concealed them illegally and refused to allow any member of the corporation to see
and examine the same; that on January 14, 1964, the stockholders, in a meeting,
removed defendant as managing director and elected Jose de la Rosa in his stead; that
defendant not only had refused to vacate his office and to deliver the assets and books
to Jose de la Rosa, but also continued to perform unauthorized acts for and in behalf
of plaintiff corporation; that defendant had been required to submit a financial
statement and to render an accounting of his administration from 1952 but defendant
has failed to do so; and that it continued disposing properties of the corporation
contrary to a Board resolution.
The writ of preliminary injunction was granted upon posting a bond. However,
the respondent filed a counter-bond which was granted, and the order for preliminary
injunction was lifted. Hence, petition for certiorari under Rule 65 was filed.

ISSUE:
Whether or not a writ of preliminary injunction against respondent should be
granted.

RULING:

NO.

Petitioner contended that respondent Alberto had arrogated to himself the


powers of the Board of Directors of the corporation because he refused to vacate the
office and surrender the same to Jose de la Rosa who had been elected managing
director by the Board to succeed him. This assertion, however, was disputed by
respondent Alberto who stated that Jose de la Rosa could not be elected managing
director because he did not own any stock in the corporation.
The Court ruled that there is in the record no showing that Jose de la Rosa
owned a share of stock in the corporation. If he did not own any share of stock,
certainly he could not be a director pursuant to the mandatory provision of Section 30
of the Corporation Law, which in part provides: "Sec. 30. Every director must own in
his own right at least one share of the capital stock of the stock corporation of which
he is a director, which stock shall stand in his name on the books of the corporation."
If the managing director-elect was not qualified to become managing director,
respondent Fausto Alberto could not be compelled to vacate his office and cede the
same to the managing director-elect because the by-laws of the corporation provides in
Article IV, Section 1 that "Directors shall serve until the election and qualification of
their duly qualified successor."

105 | P a g e
Law 321_Corporation LAW_ Case Digest

GRACE CHRISTIAN HIGH SCHOOL


vs.
THE COURT OF APPEALS, GRACE VILLAGE ASSOCIATION, INC., ALEJANDRO G.
BELTRAN, and ERNESTO L. GO
1997 Oct 23, G.R. No. 108905

FACTS:

Petitioner Grace Christian High School is an educational institution offering


preparatory, kindergarten and secondary courses at the Grace Village in Quezon City.
Private respondent Grace Village Association, Inc., on the other hand, is an
organization of lot and/or building owners, lessees and residents at Grace Village,
while private respondents Alejandro G. Beltran and Ernesto L. Go were its president
and chairman of the committee on election. For 15 years the petitioner had been
occupying a permanent seat in the Board of Directors of the respondent. However, the
latter decided to ―reexamine‖ the right of petitioner's representative to continue as an
unelected member of the board. As the board denied petitioner's request to be allowed
representation without election, petitioner brought an action for mandamus in the
Home Insurance and Guaranty Corporation. Its action was dismissed by the hearing
officer whose decision was subsequently affirmed by the appeals board. Petitioner
appealed to the Court of Appeals, which in turn upheld the decision of the HIGC's
appeals board. Hence this petition for review.

ISSUE:

Whether or not the petitioner has acquired a vested right to be a permanent


director in the association under the drafted by –laws, but which were not submitted
to the members for approval.

RULING:

NO.

The present Corporation Code states that the board of directors of corporations
must be elected from among the stockholders or members. There may be corporations
in which there are unelected members in the board but it is clear that in the examples
cited by petitioner the unelected members sit as ex officio members, i.e., by virtue of
and for as long as they hold a particular office. But in the case of petitioner, there is
no reason at all for its representative to be given a seat in the board. Nor does
petitioner claim a right to such seat by virtue of an office held. In fact it was not given
such seat in the beginning. It was only in 1975 that a proposed amendment to the by-
laws sought to give it one.
Since the provision in question is contrary to law, the fact that for fifteen years
it has not been questioned or challenged but, on the contrary, appears to have been
implemented by the members of the association cannot forestall a later challenge to its
validity. Neither can it attain validity through acquiescence because, if it is contrary to
law, it is beyond the power of the members of the association to waive its invalidity.
For that matter the members of the association may have formally adopted the
provision in question, but their action would be of no avail because no provision of the
by-laws can be adopted if it is contrary to law.
Also, petitioner cannot claim a vested right to sit in the board on the basis of
"practice." Practice, no matter how long continued, cannot give rise to any vested right
if it is contrary to law. Even less tenable is petitioner's claim that its right is
"coterminus with the existence of the association."

106 | P a g e
Law 321_Corporation LAW_ Case Digest

RAMON C. LEE and ANTONIO DM. LACDAO


vs.
THE HON. COURT OF APPEALS, SACOBA MANUFACTURING CORP., PABLO
GONZALES, JR. and THOMAS GONZALES
G.R. No. 93695, February 4, 1992

FACTS:

A complaint for a sum of money was filed by the International Corporate Bank,
Inc. against the private respondents who, in turn, filed a third party complaint against
ALFA and the petitioners. The trial court denied the motion to dismiss the 3rd party
complaint filed by petitioners and ordered the respondents to serve summons to ALFA.
Initially the summons was served to ALFA through the DBP as a consequence of the
petitioner's letter informing the court that the summons for ALFA was erroneously
served upon them considering that the management of ALFA had been transferred to
the DBP. On the other hand, the DBP claimed that it was not authorized to receive
summons on behalf of ALFA since the DBP had not taken over the company which has
a separate and distinct corporate personality and existence. Private respondents filed a
Manifestation and Motion for the Declaration of Proper Service of Summons which the
trial court granted, and which was opposed by the petitioners contending that there
was improper service of summons because they were no longer officers of ALFA by
virtue of a voting trust agreement.

ISSUE:

Whether or not the petitioners are correct.

RULING:

YES.

The petitioners argue that by virtue of the voting trust agreement the petitioners
can no longer be considered directors of ALFA. They cited that to be directors, the
Corporation Code requires that it must own at least 1 one (1) share of the capital stock
of the corporation of which he is a director which share shall stand in his name on the
books of the corporation. The voting trust agreement effectively transferred to DBP, as
the trustee, legal ownership of the stock covered by the agreement and the latter
became the stockholder of record with respect to the said shares of stocks. Since the
petitioners no longer had in their names even a single share in the corporation, they
ceased to be qualified as directors, hence they are no longer authorized to receive
summons. Being so, the service of summons upon the petitioners was invalid.

107 | P a g e
Law 321_Corporation LAW_ Case Digest

Disqualifications

ENRIQUE P. BRIAS Y ROXAS


vs.
JOHN S. HORD, et al.
1913 Feb 5, G.R. No. 8387

FACTS:

The petitioner was a duly elected member of the Board of BPI. When he
requested before Hord, the President of the company, an examination of the books and
finances of the company, the same was denied, even after repeated demands.
Thereafter, he alleged that the respondents made it appear that the petitioner had
tendered a resignation and declared that his position was vacant. Hence the latter filed
this complaint demanding that he be reinstated from his former office.

ISSUE:

Whether or not the petitioner is entitled to the relief sought.

HELD:

YES.

Based from the documentary and testimonial evidence there is no clear showing
that the petitioner had actually resigned. The testimonies of the respondents posed
several and fatal inconsistencies while the testimony of the petitioner more or less
proves what really transpired during the meeting. With these, the petitioner is still
entitled to his position and his request for examination of the corporate books must be
granted.

108 | P a g e
Law 321_Corporation LAW_ Case Digest

Voting

WOLFGANG AURBACH, et al.


vs.
SANITARY WARES MANUFACTURING CORPORATION, et al.
1989 Dec 15, G.R. No. 75875

FACTS:

In 1961, Saniwares, a domestic corporation was incorporated for the primary


purpose of manufacturing and marketing sanitary wares. One of the incorporators,
Mr. Baldwin Young went abroad to look for foreign partners, European or American
who could help in its expansion plans.
ASI, a foreign corporation domiciled in Delaware, United States entered into an
Agreement with Saniwares and some Filipino investors whereby ASI and the Filipino
investors agreed to participate in the ownership of an enterprise which would engage
primarily in the business of manufacturing in the Philippines and selling here and
abroad vitreous china and sanitary wares. The parties agreed that the business
operations in the Philippines shall be carried on by an incorporated enterprise and
that the name of the corporation shall initially be "Sanitary Wares Manufacturing
Corporation."
The conflict arose when there had dissentions from ASI for the proposed export
expansion by the other stockholders. When the next annual election of the Board
came, further conflicts arose on the manner of voting, it resulted to the uncertainty as
to who were duly elected. The contending groups of Lagdameo Group and ASI Group
claim claimed to be the legitimate directors of the corporation.

ISSUE:

Whether or not Petitioners were the duly elected members of the Board.

HELD:

NO.

The Court ruled that Wolfgang Aurbach, John Griffin, David P Whittingham,
Ernesto V. Lagdameo, Baldwin Young, Raul A. Boncan, Ernesto R. Lagdameo, Jr.,
Enrique Lagdameo, and George F. Lee as the duly elected directors of Saniwares at the
March 8, 1983 annual stockholders‘ meeting were the duly elected members of the
Board. Under their agreement, both parties were given the right their shares
cumulatively. ASI, however, should not be allowed to interfere in the voting within the
Filipino group. Otherwise, ASI would be able to designate more than the three
directors it is allowed to designate under the Agreement, and may even be able to get a
majority of the board seats, a result which is clearly contrary to the contractual intent
of the parties. The foreign Group (ASI) was limited to designate three directors . This is
the allowable participation of the ASI Group. Hence, in future dealings, this limitation
of six to three board seats should always be maintained as long as the joint venture
agreement exists considering that in limiting 3 board seats in the 9-man board of
directors there are provisions already agreed upon and embodied in the parties'
Agreement to protect the interests arising from the minority status of the foreign
investors.

109 | P a g e
Law 321_Corporation LAW_ Case Digest

BATAAN SHIPYARD & ENGINEERING CO., INC. (BASECO)


vs.
PRESIDENTIAL COMMISSION ON GOOD GOVERNMENT, et al.
1987 May 27, G.R. No. 75885

FACTS:

This case arose from a sequestration order issued by the PCGG under authority
given by the president. Such sequestration order was sent and received by petitioner.
Pursuant to this sequestration orders, take over orders were also issued to protect
public interest and to prevent the disposal or dissipation of business enterprises and
properties taken over by the government of the Marcos Administration or by entities or
persons close to former President Marcos, until the transactions leading to such
acquisition by the latter can be disposed of by the appropriate authorities. However,
among other facts, the petitioner questions the exercise of PCGGs right of ownership
and management when it terminated several contracts without the consent of both
parties, to enter contracts, and to operate its quarry business, and especially its right
ot vote during stockholders‘ meetings.

ISSUE:

Whether or not PCGG may vote in stockholders‘ meetings.

RULING:

YES.

PCGG may properly exercise the prerogative to vote sequestered stock of


corporations, granted to it by the President of the Philippines through a Memorandum
dated June 26, 1986. That Memorandum authorizes the PCGG, pending the outcome
of proceedings to determine the ownership of sequestered shares of stock, to vote such
shares of stock as it may have sequestered in corporations at all stockholders'
meetings called for the election of directors, declaration of dividends, amendment of
the Articles of Incorporation, etc. Moreover, in the case at bar, there was adequate
justification to vote the incumbent directors out of office and elect others in their stead
because the evidence showed prima facie that the former were just tools of President
Marcos and were no longer owners of any stock in the firm, if they ever were at all.

110 | P a g e
Law 321_Corporation LAW_ Case Digest

Report on Election

PREMIUM MARBLE RESOURCES, INC.


vs.
THE COURT OF APPEALS and INTERNATIONAL CORPORATE BANK
1996 Nov 4, G.R. No. 96551

FACTS:

Herein petitioner filed a case for damages against respondent for allowing
clearance of checks by unauthorized officers of the former, to the former‘s prejudice.
However this case was opposed by some members of the petitioner on the ground that
the filing of the complaint was not authorized by the Board. Hence, a resolution of this
case was necessary to litigate the claim of the petitioner for damages against the
respondent bank.

ISSUE:

Whether or not the filing was authorized by a duly constituted Board of


Directors of the petitioner corporation.

RULING:

NO.

The petitioners asserted that the Board authorized such filing. However, from
the records of the case as well as that of the corporation, no evidence was seen and
shown that the results of the election where the supposed members of the Board who
allegedly authorized the filing were filed with the Securities and Exchange
Commission. The Corporation Code mandates that within thirty (30) days after the
election of the directors, trustees and officers of the corporation, the secretary, or any
other officer of the corporation, shall submit to the Securities and Exchange
Commission, the names, nationalities and residences of the directors, trustees and
officers elected. Failure to comply with such requirement, the elected members cannot
be considered as the duly constituted and elected members of the Board. Hence, being
not duly constituted, the filing of the case was not authorized by the Board.

111 | P a g e
Law 321_Corporation LAW_ Case Digest

Term of Office/Holdover

DR. HANS CHRISTIAN M. SEÑERES


vs.
COMMISSION ON ELECTIONS and MELQUIADES A. ROBLES
G.R. No. 178678, April 16, 2009

FACTS:

Private respondent Robles was elected president and chairperson of Buhay, a


party-list group duly registered with COMELEC. The constitution of BUHAY provides
for a three-year term for all its party officers, without re-election. Robles again signed
and filed a Certificate of Nomination of BUHAY‘s nominees for the 2007 elections,
however such certificate was denied by petitioner alleging that he was the acting
president and secretary-general of BUHAY, having assumed that position since August
17, 2004 when Robles vacated the position. Señeres further claimed that the
nominations made by Robles were, for lack of authority, null and void owing to the
expiration of the latter‘s term as party president.
On May 10, 2007, the National Council of BUHAY adopted a resolution
expelling Señeres as party member for his act of submitting a Certificate of
Nomination for the party. Subsequently, Robles was adjudged as the duly authorized
representative of Buhay. Aggrieved, petitioner filed this complaint.

ISSUE:

Whether or not respondent Robles is the duly authorized representative of


BUHAY.

RULING:

YES.

Petitioner Señeres maintains that at the time the Certificate of Nomination was
submitted, Robles‘ term as President of BUHAY had already expired, thus effectively
nullifying the Certificate of Nomination and the nomination process.
The Court was mot amenable. As a general rule, officers and directors of a
corporation hold over after the expiration of their terms until such time as their
successors are elected or appointed. Sec. 23 of the Corporation Code contains a
provision to this effect, thus: the board of directors or trustees to be elected from
among the holders of stocks, or where there is no stock, from among the members of
the corporation, who shall hold office for one (1) year until their successors are elected
and qualified.
The holdover doctrine accords validity to what would otherwise be deemed as
dubious corporate acts and gives continuity to a corporate enterprise in its relation to
outsiders. The voting members of BUHAY duly elected Robles as party President in
October 1999. And although his regular term as such President expired in October
2002, no election was held to replace him and the other original set of
officers. Further, the constitution and by-laws of BUHAY do not expressly or impliedly
prohibit a hold-over situation. As such, since no successor was ever elected or
qualified, Robles remained the President of BUHAY in a "hold-over" capacity.

112 | P a g e
Law 321_Corporation LAW_ Case Digest

How Removed

LEON J. LAMBERT
vs.
T. J. FOX
1914 Jan 29, G.R. No. 7991

FACTS:

Due to financial crisis the petitioner and the defendant were able to acquire the
bulk of the stocks of John R. Edgar & Co. as the latter‘s creditors. Hence, upon
incorporating said company, the parties entered into an agreement that either of them
will not sell or transfer their respective shares till after one year from the date of
agreement. However, less than a year, defendant Fox sold his stock in the said
corporation to E. D. McCullough of the firm of E. C. McCullough & Co. of Manila, a
strong competitor of the said John R. Edgar & Co., Inc. This sale was made by the
defendant against the protest of the plaintiff and with the warning that he would be
held liable under the contract hereinabove set forth and in accordance with its terms.
In fact, the defendant Fox offered to sell his shares of stock to the plaintiff for the
same sum that McCullough was paying for them less P1, 000, the penalty specified in
the contract.
The trial Court rendered judgment in favor of defendant.

ISSUE:

Whether or not the stipulation not to sell is valid.

RULING:

YES.

The suspension of the power to sell has a beneficial purpose, results in the
protection of the corporation as well as of the individual parties to the contract, and is
reasonable as to the length of time of the suspension.
The intention of parties to a contract must be determined, in the first instance,
from the words of the contract itself. It is to be presumed that persons mean what they
say when they speak plain English. Interpretation and construction should by the
instruments last resorted to by a court in determining what the parties agreed to.
Where the language used by the parties is plain, then construction and interpretation
are unnecessary and, if used, result in making a contract for the parties.
In this jurisdiction, there is no difference between a penalty and liquidated
damages, so far as legal results are concerned. Whatever differences exists between
them as a matter of language, they are treated the same legally. In either case the
party to whom payment is to be made is entitled to recover the sum stipulated without
the necessity of proving damages. Indeed one of the primary purposes in fixing a
penalty or in liquidating damages is to avoid such necessity.

113 | P a g e
Law 321_Corporation LAW_ Case Digest

How Vacancy Filled

VALLE VERDE COUNTRY CLUB, INC., et al.


vs.
VICTOR AFRICA
G.R. No. 151969, September 4, 2009

FACTS:

During the Annual Stockholders‘ Meeting of petitioner Valle Verde Country


Club, Inc. (VVCC), the following were elected as members of the VVCC Board of
Directors: Ernesto Villaluna, Jaime C. Dinglasan, Eduardo Makalintal, Francisco
Ortigas III, Victor Salta, Amado M. Santiago, Jr., Fortunato Dee, Augusto Sunico, and
Ray Gamboa. In the years 1997, 1998, 1999, 2000, and 2001, however, the requisite
quorum for the holding of the stockholders‘ meeting could not be obtained.
Consequently, the above-named directors continued to serve in the VVCC Board in a
hold-over capacity. Two of the said members resigned (Makalintal and Dinglasan).
After the resignation of Dinglasan, Eric Roxas was elected. Makalintal was replaced by
Jose Ramirez.
Respondent Africa, a member of VVCC, questioned the election of Roxas and
Ramirez as members of the VVCC Board with the Securities and Exchange
Commission (SEC) and the Regional Trial Court. Africa alleged that the election of
Roxas was contrary to Section 29, in relation to Section 23, of the Corporation Code of
the Philippines. The respective trial courts ruled in favor of Africa.

ISSUE:

Whether or not the elections were valid.

RULING:

YES.

Section 23of the Corporation Code declares that "the board of directors shall
hold office for one (1) year until their successors are elected and qualified," we
construe the provision to mean that the term of the members of the board of directors
shall be only for one year; their term expires one year after election to the office. The
holdover period – that time from the lapse of one year from a member‘s election to the
Board and until his successor‘s election and qualification – is not part of the director‘s
original term of office, nor is it a new term; the holdover period, however, constitutes
part of his tenure. Corollary, when an incumbent member of the board of directors
continues to serve in a holdover capacity, it implies that the office has a fixed term,
which has expired, and the incumbent is holding the succeeding term.
After the lapse of one year from his election as member of the VVCC Board in
1996, Makalintal‘s term of office is deemed to have already expired. That he continued
to serve in the VVCC Board in a holdover capacity cannot be considered as extending
his term. This holdover period, however, is not to be considered as part of his term,
which, as declared, had already expired.
With the expiration of Makalintal‘s term of office, a vacancy resulted which, by
the terms of Section 29of the Corporation Code, must be filled by the stockholders of
VVCC in a regular or special meeting called for the purpose. As correctly pointed out
by the RTC, when remaining members of the VVCC Board elected Ramirez to replace
Makalintal, there was no more unexpired term to speak of, as Makalintal‘s one-year
term had already expired. Pursuant to law, the authority to fill in the vacancy caused
by Makalintal‘s leaving lies with the VVCC‘s stockholders, not the remaining members
of its board of directors.

114 | P a g e
Law 321_Corporation LAW_ Case Digest

How Compensated

GABRIEL C. SINGSON, et al.


vs.
COMMISSION ON AUDIT
G.R. No. 159355, August 9, 2010

FACTS:

Petitioners are the members of the Board of Philippine International Convention


Center, Inc. (PICCI). By virtue of the PICCI By-Laws, petitioners were authorized to
receive P1,000.00 per diem each for every meeting attended. An amended resolution
further granted the Members of the additional monthly RATA, in the amount
of P1,500.00, to each of the petitioners. However, payment for such grants were
denied. The disallowance was questioned but it was upheld by herein respondent.
Hence this petition.

ISSUE:

Whether or not the grant of the compensation as well as the monthly RATA are
valid.

RULING:

NO.

Section 30 of the Corporation Code, which authorizes the stockholders to grant


compensation to its directors, states: In the absence of any provision in the by-laws
fixing their compensation, the directors shall not receive any compensation, as such
directors, except for reasonable per diems; Provided, however, that any such
compensation (other than per diems) may be granted to directors by the vote of the
stockholders representing at least a majority of the outstanding capital stock at a
regular or special stockholders‘ meeting. In no case shall the total yearly compensation
of directors, as such directors, exceed ten (10%) percent of the net income before
income tax of the corporation during the preceding year.
From this, it is clear that the directors of a corporation shall not receive any
compensation for being members of the board of directors, except for reasonable per
diems. The two instances where the directors are to be entitled to compensation shall
be when it is fixed by the corporation‘s by-laws or when the stockholders, representing
at least a majority of the outstanding capital stock, vote to grant the same at a regular
or special stockholder‘s meeting, subject to the qualification that, in any of the two
situations, the total yearly compensation of directors, as such directors, shall in no
case exceed ten (10%) percent of the net income before income tax of the corporation
during the preceding year.
In this regard, the Court upholds the findings of respondent that petitioners‘
right to compensation as members of the PICCI Board of Directors is limited only to
per diem of P1,000.00 for every meeting attended, by virtue of the PICCI By-Laws.

115 | P a g e
Law 321_Corporation LAW_ Case Digest

WESTERN INSTITUTE OF TECHNOLOGY, INC.,


vs.
HOMERO L. VILLASIS, DIMAS ENRIQUEZ, PRESTON F. VILLASIS & REGINALD F.
VILLASIS v. RICARDO T. SALAS, et al.
1997 Aug 21, G.R. No. 113032

FACTS:

The petitioners are the minority stockholders, while the respondents are the
majority stockholders of the corporation. The petitioners alleged that a meeting was
held in its principal office and a prior notice was distributed to the members.
Subsequently a resolution was passed granting monthly compensation for services
rendered by its officers.
A complaint was filed against the respondents by the petitioners contending
that the grant of compensation is prohibited. The trial court rendered decision in favor
of the respondents.

ISSUE:

Whether or not the grant of compensation was valid.

RULING:

YES.

Under section 30, there are two (2) ways by which members of the board can be
granted compensation apart from reasonable per diems: (1) when there is a provision
in the by-laws fixing their compensation; and (2) when the stockholders representing a
majority of the outstanding capital stock at a regular or special stockholders' meeting
agree to give it to them. The proscription under said section pertains to compensations
granted to members of the Board. But they are not prohibited to be compensated if
these members of the Board act as officers of the corporation, more particularly as
Chairman, Vice-Chairman, Treasurer and Secretary of Western Institute of
Technology.

116 | P a g e
Law 321_Corporation LAW_ Case Digest

CENTRAL COOPERATIVE EXCHANGE, INC.


vs.
CONCORDIO TIBE, SR. and THE HONORABLE COURT OF APPEALS
1970 Jun 30, G.R. No. L-27972

FACTS:

The petitioner is a national federation of farmers' cooperative marketing


associations, or FACOMAS, scattered throughout the country. Under its by- laws "The
compensation, if any, and the per diems for attendance at meetings of the members of
the Board of Directors shall be determined by the members at any annual meeting in
special meeting of the Exchange called for the purpose."
In the annual stockholders‘ meeting it was resolved that the members of the
Board of Directors attending the CCE board meetings be entitled to actual
transportation expenses plus the per diems of P30.00 and actual expenses while
waiting. In this regard, Tibe collected the said amounts however the petitioner refused
to give on the ground that the resolutions are invalid. The trial court ruled in favor of
Tibe.

ISSUE:

Whether or not the resolutions are valid.

RULING:

NO.

The Court ruled that resolutions are contrary to the By-Laws of the federation
and, therefore, are not within the power of the board of directors to enact. The By-
Laws, in the aforequoted Section 8, explicitly reserved unto the stockholders the power
to determine the compensation of members of the board of directors, and the
stockholders did restrict such compensation to "actual transportation expenses plus
the per diems of P30.00 and actual expenses while waiting." Even without the express
reservation of said power, the directors are not entitled to compensation under the
Corporation Code.
The directors, in assigning themselves additional duties, such as the visitation
of FACOMAS, acted within their power, but, by voting for themselves compensation for
such additional duties, they acted in excess of their authority, as expressed in the By-
Laws.

117 | P a g e
Law 321_Corporation LAW_ Case Digest

LINGAYEN GULF ELECTRIC POWER COMPANY, INC.


vs.
IRINEO BALTAZAR
G.R. No. L-4824, June 30, 1953

FACTS:

The respondent subscribed stocks of the petitioner. After paying several


amount, the respondent failed to pay its outstanding balance, even after a demand
made by the corporation. The latter hence opted to collect the unpaid balance of the
subscription made. However, the respondent refused to pay on the contention that he
has been released from his liability under Resolution No. 17. Furthermore, he
countered that, as the President of the corporation, he was entitled to compensation.
The trial court rendered judgment in favor of respondent.

ISSUE:

Whether or not the respondent is entitled to compensation.

RULING:

NO.

It is clear that he is not entitled to the same. The by-laws of the company are
silent as to the salary of the President. And, while resolutions of the incorporators and
stockholders provide salaries for the general manager, secretary-treasurer and other
employees, there was no provision for the salary of the President. On the other hand,
other resolutions provide for per diems to be paid to the President and the directors of
each meeting attended, P10 for the President and P8 for each director, which were
later increased to P25 and P15, respectively. This leads to the conclusions that the
President and the board of directors were expected to serve without salary, and that
the per diems paid to them were sufficient compensation for their services.
Furthermore, for defendant's several years of service as President and up to the filing
of the action against him, he never filed a claim for salary.

118 | P a g e
Law 321_Corporation LAW_ Case Digest

Authority of the Board of Directors

LA BUGAL-B'LAAN TRIBAL ASSOCIATION, INC., et al.


vs.
VICTOR O. RAMOS, Secretary, Department of Environment and Natural
Resources (DENR); HORACIO RAMOS, Director, Mines and Geosciences Bureau
(MGB-DENR); RUBEN TORRES, Executive Secretary; and WMC (PHIL.), INC.
G.R. No. 127882. December 1, 2004

FACTS:

On July 25, 1987, then President Corazon C. Aquino issued Executive Order
(E.O.) No. 279authorizing the DENR Secretary to accept, consider and evaluate
proposals from foreign-owned corporations or foreign investors for contracts or
agreements involving either technical or financial assistance for large-scale
exploration, development, and utilization of minerals, which, upon appropriate
recommendation of the Secretary, the President may execute with the foreign
proponent.
Subsequently, then President Fidel V. Ramos approved R.A. No. 7942 to "govern
the exploration, development, utilization and processing of all mineral resources."R.A.
No. 7942 defines the modes of mineral agreements for mining operations,outlines the
procedure for their filing and approval, assignment/transferand withdrawal, and fixes
their terms. Similar provisions govern financial or technical assistance agreements.
On August 15, 1995, then DENR Secretary Victor O. Ramos issued DENR
Administrative Order (DAO) No. 95-23, s. 1995, otherwise known as the Implementing
Rules and Regulations of R.A. No. 7942. This was later repealed by DAO No. 96-40, s.
1996 which was adopted on December 20, 1996.
On January 10, 1997, counsels for petitioners sent a letter to the DENR
Secretary demanding that the DENR stop the implementation of R.A. No. 7942 and
DAO No. 96-40.

ISSUE:

Whether or not the law in force when the WMC FTAA was executed, not come
into effect.

RULING:

YES.

It bears noting that there is nothing in E.O. No. 200 that prevents a law from
taking effect on a date other than even before the 15-day period after its publication.
Where a law provides for its own date of effectivity, such date prevails over that
prescribed by E.O. No. 200. Indeed, this is the very essence of the phrase "unless it is
otherwise provided" in Section 1 thereof. Section 1, E.O. No. 200, therefore, applies
only when a statute does not provide for its own date of effectivity.
As noted, "service contracts" is a term that assumes different meanings to
different people. The commissioners may have been using the term loosely, and not in
its technical and legal sense, to refer, in general, to agreements concerning natural
resources entered into by the Government with foreign corporations. These loose
statements do not necessarily translate to the adoption of the 1973 Constitution
provision allowing service contracts.
In any case, the constitutional provision allowing the President to enter into
FTAAs with foreign-owned corporations is an exception to the rule that participation in
the nation's natural resources is reserved exclusively to Filipinos. Accordingly, such
provision must be construed strictly against their enjoyment by non-Filipinos. As
Commissioner Villegas emphasized, the provision is "very restrictive."Commissioner
Nolledo also remarked that "entering into service contracts is an exception to the rule
on protection of natural resources for the interest of the nation and, therefore, being
an exception, it should be subject, whenever possible, to stringent rules." Indeed,
exceptions should be strictly but reasonably construed.

119 | P a g e
Law 321_Corporation LAW_ Case Digest

SHIPSIDE INCORPORATED
vs.
THE HON. COURT OF APPEALS, HON. REGIONAL TRIAL COURT, BRANCH 26 &
the REPUBLIC OF THE PHILIPPINES
G.R. No. 143377, February 20, 2001

FACTS:

Originally four lots were owned Rafael Galvez. He subsequently sold lot 1 and 4
in favor of Filipina Mamaril, Cleopatra Llana, Regina Bustos, and Erlinda Balatbat in a
deed of sale. Mamaril later sold lot 1 to Lepanto Consolidated Mining Company. Later,
unknown to Lepanto, the RTC declared the OCT registered in the name of Galvez as
null and void and ordered the cancellation thereof. On October 28, 1963, Lepanto
Consolidated Mining Company sold to herein petitioner Lots No. 1 and 4. Meanwhile
the decision of the CA became final and executory and a writ was issued, however said
writ remained unsatisfied for 24 years.
Office of the Solicitor General filed a complaint for revival of judgment and
cancellation of titles before the Regional Trial Court of the First Judicial Region
against the successors of Galvez and herein petitioner and its motion for
reconsideration was likewise turned down. The CA affirmed the same, hence this
petition.

ISSUE:

Whether or not the filing of the petition was authorized by the BOD of
petitioner.

RULING:

YES.

The Court of Appeals dismissed the petition for certiorari on the ground that
Lorenzo Balbin, the resident manager for petitioner, who was the signatory in the
verification and certification on non-forum shopping, failed to show proof that he was
authorized by petitioner's board of directors to file such a petition.
It was clear from the record that when the general manager filed the petition,
there was no proof attached as to the authorization by the Board. However, when the
petitioner filed its motion for reconsideration a resolution or secretary‘s certification
stating that that on October 11, 1999, or ten days prior to the filing of the petition,
Balbin had been authorized by petitioner's board of directors to file said petition. The
Court accepted this certification, although belatedly presented, as a valid
authorization. The Court was reiterated that belated submission of a verification is
allowed the same being not a mandatory and jurisdictional requirement, and as to the
non-forum shopping the same was considered to be valid because the case of the
petitioner must be litigated based on its merit and must not be dismissed based on
technical and procedural infirmities, which were actually cured.

120 | P a g e
Law 321_Corporation LAW_ Case Digest

ABS-CBN
vs.
COURT OF APPEALS
G.R. No. 128690. January 21, 1999

FACTS:

In 1990, ABS CBN and Viva executed a Film Exhibition Agreement whereby
Viva gave ABS CBN an exclusive right to exhibit some Viva films. Said agreement
contained a stipulation that ABS shall have the right of first refusal to the next 24 Viva
films for TV telecast, provided that such right shall be exercised by ABS from the
actual offer in writing. Hence, through this agreement, Viva offered ABS a list of 36
films from which ABS may exercise its right of first refusal. ABS however, through VP
Concio, did not accept the list since she could only tick off 10 films. This rejection was
embodied in a letter. In 1992, Viva again approached ABS with a list consisting of 52
original films where Viva proposed to sell these airing rights for P60M. Viva‘s Vic del
Rosario and ABS‘ general manager Eugenio Lopez III met at the Tamarind Grill to
discuss this package proposal. What transcribed at that meeting was subject to
conflicting versions.
According to Lopez, he and del Rosario agreed that ABS was granted exclusive
film rights to 14 films for P36M, and that this was put in writing in a napkin, signed
by Lopez and given to del Rosario. On the other hand, del Rosario denied the
existence of the napkin in which Lopez wrote something, and insisted that what he
and Lopez discussed was Viva‘s film package of the 52 original films for P60M stated
above, and that Lopez refused said offer, allegedly signifying his intent to send a
counter proposal. When the counter proposal arrived, Viva‘s BoD rejected it; hence,
he sold the rights to the 52 original films to RBS.
Thus, ABS filed before RTC a complaint for specific performance with prayer for
TRO against RBS and Viva. RTC issued the TRO enjoining the airing of the films
subject of controversy. After hearing, RTC rendered its decision in favor of RBS and
Viva contending that there was no meeting of minds on the price and terms of the
offer. The agreement between Lopez and del Rosario was subject to Viva BoD
approval, and since this was rejected by the board, then, there was no basis for ABS‘
demand that a contract was entered into between them. That the 1990 Agreement
with the right of first refusal was already exercised by Ms. Concio when it rejected the
offer, and such 1990 Agreement was an entirely new contract other than the 1992
alleged agreement at the Tamarind Grill.

ISSUE:

Whether or not there was a perfected contract between Lopez and del Rosario.

RULING:

NO.

Contracts that are consensual in nature are perfected upon mere meeting of the
minds. Once there is concurrence between the offer and the acceptance upon the
subject matter, consideration, and terms of payment, a contract is produced. The
offer must be certain. To convert the offer into a contract, the acceptance must be
absolute and must not qualify the terms of the offer; it must be plain, unequivocal,
unconditional, and without variance of any sort from the proposal. A qualified
acceptance, or one that involves a new proposal, constitutes a counter offer and is a
rejection of the original offer. Consequently, when something is desired which is not
exactly what is proposed in the offer, such acceptance is not sufficient to generate
consent because any modification or variation from the terms of the offer annuls the
offer.
In the case at bar, when Del Rosario met with Lopez at the Tamarind Grill, the
package of 52 films was Viva‘s offer to enter into a new Exhibition Agreement. But
ABS, through its counter proposal sent to Viva, actually made a counter offer. Clearly,
there was no acceptance. The acceptance should be unqualified.

121 | P a g e
Law 321_Corporation LAW_ Case Digest

ASSET PRIVATIZATION TRUSTS


vs.
COURT OF APPEALS, JESUS S. CABARRUS, SR., et al.
G.R. No. 121171. December 29, 1998

FACTS:

By virtue of an agreement, the exclusive right to explore, develop and exploit


nickel, cobalt and other minerals in the Surigao mineral reservation was granted to
Marinduque Mining and Industrial Corporation (MMIC), with respondent Jesus S.
Cabarrus, Sr. as President and among its original stockholders.
The Philippine Government undertook to support the financing of MMIC by
purchase of MMIC debenture bonds and extension of guarantees. Subsequently,
MMIC, PNB and DBP executed a Mortgage Trust Agreement 3 whereby MMIC, as
mortgagor, agreed to constitute a mortgage in favor or PNB and DBP as mortgagees,
over all MMIC's assets. MMIC defaulted in its loans with the bank amounting to more
or less P21M. Thus, a financial restructuring plan (FRP) designed to reduce MMIC's
interest expense through debt conversion to equity was drafted and was approved by
the Board of Directors of the MMIC. However, the proposed FRP had never been
formally adopted, approved or ratified by either PNB or DBP.
Hence, the properties were foreclosed with PNB as the lone bidder and were
assigned to three newly formed corporations, namely, Nonoc Mining Corporation,
Maricalum Mining and Industrial Corporation, and Island Cement Corporation. In
1986, these assets were transferred to the Asset Privatization Trust (APT). , Jesus S.
Cabarrus, Sr., together with the other stockholders of MMIC, filed a derivative suit
against DBP and PNB before the RTC of Makati, Branch 62, for Annulment of
Foreclosures, Specific Performance and Damages. The parties entered into a
compromise agreement. However, when submitted for approval, the Arbitration
Committee rendered judgment in favor of MMIC.

ISSUE:

Whether or not there was a valid foreclosure of the properties.

RULING:

YES.

The FRP was not adopted by PNB and DBP hence there was no valid
restructuring program undertaken and the option of the banks to foreclose the
properties were never divested. Although there were allegations that representatives of
PNB and DBP were part of the drafting of the FRP there was no showing that the
representatives of PNB and DBP in MMIC even had the requisite authority to enter
into a debt-for-equity swap. And if they had such authority, there was no showing that
the banks, through their board of directors, had ratified the FRP. Hence, without such
proof, the Court ruled that those representatives, singly or collectively, are not
themselves PNB or DBP. They are individuals with personalities separate and distinct
from the banks they represent. PNB and DBP have different boards with different
members who may have different decisions. And estoppel cannot be used to impose
upon them the decision of the board of another company. Otherwise the rights of
entirely separate distinct and autonomous legal entities like PNB and DBP with
thousands of stockholders will be suppressed and rendered nugatory. Wherefore, the
decisions of the appellate court were reversed.

122 | P a g e
Law 321_Corporation LAW_ Case Digest

BA SAVINGS BANK
vs.
ROGER T. SIA, TACIANA U. SIA and JOHN DOE
G.R. No. 131214, July 27, 2000

FACTS:

A petition for certiorari was filed by herein petitioner bank. However, the CA
denied due course the same on the ground that the certificate of non-forum shopping
was signed by a lawyer. A Motion for Reconsideration was subsequently filed by the
petitioner, attached to which was a BA Savings Bank Corporate Secretary‘s Certificate.
The Certificate showed that the petitioner‘s Board of Directors approved a Resolution
on May 21, 1996, authorizing the petitioner‘s lawyers to represent it in any action or
proceeding before any court, tribunal or agency; and to sign, execute and deliver the
Certificate of Non-forum Shopping, among others. Said motion was denied on the
ground that Supreme Court Revised Circular No. 28-91 requires that it is the
petitioner, not the counsel, who must certify under oath to all of the facts and
undertakings required therein.

ISSUE:

Whether or not the CA was correct.

RULING:

YES.

A corporation exercises powers through its board of directors and/or its duly
authorized officers and agents. Physical acts, like the signing of documents, can be
performed only by natural persons duly authorized for the purpose by corporate
bylaws or by a specific act of the board of directors. In this case, the corporation‘s
board of directors issued a Resolution specifically authorizing its lawyers ―to act as
their agents in any action or proceeding before the Supreme Court, the Court of
Appeals, or any other tribunal or agency and to sign, execute and deliver in connection
therewith the necessary pleadings, motions, verification, affidavit of merit, certificate of
non-forum shopping and other instruments necessary for such action and
proceeding.‖ The Resolution was sufficient to vest such persons with the authority to
bind the corporation and was specific enough as to the acts they were empowered to
do.
Circular 28-91 requires the parties themselves to sign the certificate of non-
forum shopping. However, such requirement cannot be imposed on artificial persons,
like corporations, for the simple reason that they cannot personally do the task
themselves. In this case, the corporation very well exercised its power to authorize a
representative to act on its behalf.

123 | P a g e
Law 321_Corporation LAW_ Case Digest

ALFREDO MONTELIBANO and ALEJANDRO MONTELIBANO


vs.
THE HON. COURT OF APPEALS and BACOLOD-MURCIA MILLING COMPANY, INC.
G.R. No. 85757, July 8, 1991

FACTS:

Alfredo and Alejandro Montelibano, together with other planters, entered into
contracts with Bacolod-Murcia Milling Co., Inc., for the milling of sugar cane at a
sharing ratio of 55% for the planters and 45% for the miller. The contracts were to be
in force for thirty (30) years starting with the 1920-21 crops. A proposal was made to
amend the milling contracts by increasing the planters' share to 60% of the
manufactured sugar and molasses and giving them other concessions besides, but the
term of the contracts was extended to 45 years instead of 30. On August 30, 1936, the
milling company's Board of Directors adopted a resolution granting further
concessions to the planters over and above those contained in the amended milling
contract. Subsequently, the Montelibanos sued the milling company alleging that the
three other centrals in the province were granting increased participation to their
planters; therefore, pursuant to paragraph 9 of the August 20, 1936 Resolution,
Bacolod-Murcia Milling Co., Inc. was obligated to grant similar concessions to the
Montelibanos.
The milling company opposed the claim on the ground that, among others, it
was a donation which was not within the power of the Board of Directors to grant. The
trial court dismissed the action, but on appeal to the Supreme Court reversed the
lower court.

ISSUE:

Whether or not the reversal was proper.

RULING:

YES.

The Court ruled that the August 20, 1936 resolution, passed in good faith by
the board of directors, was valid and binding and formed an integral part of the
amended milling contracts, the milling company having agreed to give concessions to
the planters, precisely to induce them to agree to an extension of their contracts.
Petitioner filed two motions for reconsideration; however, the doctrine of res judicata
had set in. Wherefore, the appeal was denied.

124 | P a g e
Law 321_Corporation LAW_ Case Digest

ANTHONY POWERS, et al.


vs.
DONALD I. MARSHALL, et al.
G.R. No. L-48064, May 9, 1988

FACTS:

A letter was passed to the members of the International School, Inc. indicating
that that the Board of Trustees had decided to embark on a program to construct new
buildings and remodel existing ones to accommodate the increasing enrollment in the
school, and that it was necessary for the school to raise P35,000,000.00 for this
purpose. The Board intended to raise the needed funds primarily through
subscriptions to capital notes and prepayment certificates, and any deficiency from
these sources would be covered by collecting a so-called "development fees" of P2,625
from each enrollee starting with the school year 1975-1976 and continuing up to the
school year 1986-1987. An implementing letter was issued indicating therein that the
conditions stated are pre-enrollment requirements.
Fourteen (14) plaintiffs, all associate members of the International School, Inc.,
brought an action for injunction in the Court of First Instance of Rizal. A TRO was
granted in favor of the defendants. Subsequently a decision was rendered dismissing
the complaint for lack of cause of action.

ISSUE:

Whether or not the actions of the BOT are valid.

RULING:

YES.

Section 2 (b) of P.D. No. 732 granting certain rights to the International School,
Inc., expressly authorized the Board of Trustees "upon consultation with the Secretary
of Education and Culture, ... to determine the amount of fees and assessments which
may be reasonably imposed upon its students, to maintain or conform to the school
standard of education." Such consultation had been made with the Secretary of
Education and Culture who expressed his conformity with the reasonableness of the
assessment of P2,625.00 per student for the whole school year to carry out its
development program. Since the collection of the development fee had been approved
by the Board of Trustees of the International School, Inc., it was a valid exercise of
corporate power by the Board, and said assessment was binding upon all the
members of the corporation.

125 | P a g e
Law 321_Corporation LAW_ Case Digest

PREMIUM MARBLE RESOURCES, INC.


vs.
THE COURT OF APPEALS and INTERNATIONAL CORPORATE BANK
1996 Nov 4, G.R. No. 96551

FACTS:

Herein petitioner filed a case for damages against respondent for allowing
clearance of checks by unauthorized officers of the former, to the former‘s prejudice.
However this case was opposed by some members of the petitioner on the ground that
the filing of the complaint was not authorized by the Board. Hence, a resolution of this
case was necessary to litigate the claim of the petitioner for damages against the
respondent bank.

ISSUE:

Whether or not the filing was authorized by a duly constituted Board of


Directors of the petitioner corporation.

RULING:

NO.

The petitioners asserted that the Board authorized such filing. However, from
the records of the case as well as that of the corporation‘s, no evidence was seen and
shown that the results of the election where the supposed members of the Board who
allegedly authorized the filing were filed with the Securities and Exchange
Commission. The Corporation Code mandates that within thirty (30) days after the
election of the directors, trustees and officers of the corporation, the secretary, or any
other officer of the corporation, shall submit to the Securities and Exchange
Commission, the names, nationalities and residences of the directors, trustees and
officers elected. Failure to comply with such requirement, the elected members cannot
be considered as the duly constituted and elected members of the Board. Hence, being
not duly constituted, the filing of the case was not authorized by the Board.

126 | P a g e
Law 321_Corporation LAW_ Case Digest

J. F. RAMIREZ
vs.
THE ORIENTALIST CO., and RAMON J. FERNANDEZ
G.R. No. 11897, September 24, 1918

FACTS:

Orientalist Companywas engaged in the business of maintaining and


conducting a theatre in the city of Manila for the exhibition of cinematographic films.
Later on it accepted an offer from Jose Ramirez, the son of herein petitioner that the
latter will supply films that will be managed by the respondent. However, when the
films arrived, Orientalist was without fund to pay the cost and expenses incident to
each shipment. In effect the company‘s president B. Hernandez paid said obligations
and treated the films by him as his own property; and they in fact never came into the
actual possession of the Orientalist Company as owner at all, though it is true
Hernandez rented the films to the Orientalist Company and they were exhibited by it
in the Oriental Theater under an arrangement which was made between him and the
theater's manager. However, subsequent deliveries were no longer paid by any of the
concerned party.

ISSUE:

Whether or not the contract was entered into with the authorization of its
Board.

RULING:

YES.

Although there were no evidence as to the authority of Ramon Fernandez to


enter into said contract, the Court had observed that when the defendant corporation
failed to question the validity of the contract, it resulted to eliminating the question of
his authority from the case. This is a case where an officer of a corporation has made
a contract in its name, that the corporation should be required, if it denies his
authority, to state such defense in its answer. By this means the plaintiff is apprised
of the fact that the agent's authority is contested; and he is given an opportunity to
adduce evidence showing either that the authority existed or that the contract was
ratified and approved. Failure to question such timely and appropriately question such
authority results to the admission of such fact.

127 | P a g e
Law 321_Corporation LAW_ Case Digest

Corporate Officers/Meaning of “Office” vis-à-vis Employment

RENATO REAL
vs.
SANGU PHILIPPINES, INC. and/ or KIICHI ABE
G.R. No. 168757, January 19, 2011

FACTS:

Petitioner Renato Real was the Manager of Respondent Corporation Sangu


Philippines, Inc. engaged in the business of providing manpower for general services,
like janitors, janitresses and other maintenance personnel, to various clients.
Subsequently, the janitors, etc employed by the latter filed their respective complaint
for illegal dismissal. Petitioner, on the other hand, was removed from his position via
Board resolution, and thus also filed an illegal dismissal case against the corporation.
The LA ruled for the petitioner and his co-complainants, which was reversed by the
NLRC. The ruling of the NLRC was affirmed by the CA.

ISSUE:

Whether or not petitioner is a corporate officer of the corporation.

RULING:

NO.

He is not a corporate officer. It has been consistently held that an ‗office‘ is


created by the charter of the corporation and the officer is elected (or appointed) by the
directors or stockholders. Clearly here, respondents failed to prove that petitioner was
appointed by the board of directors. Although they had been reiterating that the
petitioner was employed as a manager, there was no indication as to how he was put
into such position. For respondents‘ failure to substantiate its claim the petitioner was
deemed to be not a coporate officer, hence jurisdiction properly lies with the LA.

128 | P a g e
Law 321_Corporation LAW_ Case Digest

MATLING INDUSTRIAL AND COMMERCIAL CORPORATION, RICHARD K.


SPENCER, CATHERINE SPENCER, AND ALEX MANCILLA
vs.
RICARDO R. COROS
G.R. No. 157802, October 13, 2010

FACTS:

The respondent filed a case for illegal dismissal against Matling and some of its
corporate officers (petitioners) in the NLRC. He was the Vice President for Finance and
Administration when he was dismissed. The petitioner opposed said complaint on the
ground that the issue at hand is an intra- corporate dispute which falls under the
jurisdiction of the SEC. The respondent opposed the same contending that his being
in the position was doubtful that he had not been formally elected as such. The LA
ruled in favor of petitioner but the same was reversed by the NLRC, and to which the
CA affirmed.

ISSUE:

Whether or not respondent is a corporate officer.

RULING:

NO.

The petitioners contend that the position of Vice President for Finance and
Administration was a corporate office, having been created by Matling‘s President
pursuant to the by-Law.
However, the Court explained that an "office" is created by the charter of the
corporation and the officer is elected by the directors or stockholders. On the other
hand, an employee occupies no office and generally is employed not by the action of
the directors or stockholders but by the managing officer of the corporation who also
determines the compensation to be paid to such employee. In this case, respondent
was appointed vice president for nationwide expansion by Malonzo, petitioner‘'s
general manager, not by the board of directors of petitioner. Also his compensation
was paid by Malonzo. Thus, respondent was an employee, not a "corporate officer.
Also, the Board of Directors of Matling could not validly delegate the power to
create a corporate office to the President, in light of Section 25 of the Corporation Code
requiring the Board of Directors itself to elect the corporate officers. Verily, the power
to elect the corporate officers was a discretionary power that the law exclusively vested
in the Board of Directors, and could not be delegated to subordinate officers or
agents.The office of Vice President for Finance and Administration created by Matling‘s
President pursuant to the by- law was an ordinary, not a corporate, office.

129 | P a g e
Law 321_Corporation LAW_ Case Digest

MANILA METAL CONTAINER CORPORATION, REYNALDO C. TOLENTINO


vs.
PHILIPPINE NATIONAL BANK, DMCI-PROJECT DEVELOPERS, INC.
G.R. No. 166862, December 20, 2006

FACTS:

Petitioner is the owner of a parcel of land. To secure a loan he obtained from


PNB he executed a REM over said land. For failure to pay the loan, PNB sought the
foreclosure of the REM. After the public auction, the petitioner requested PNB to grant
him an extension to redeem the property. He failed to redeem the property. Later on,
the PNB agreed to let the petitioner purchase the property for a certain amount, and a
downpayment was then given. Subsequently, however, the bank informed the
petitioner that it was increasing the purchase price. Hence, a case was filed by
petitioner.

ISSUE:

Whether or not the letter by the respondent accepting the petitioner‘s offer was
valid.

RULING:

NO.

There is no evidence that the SAMD was authorized by respondent's Board of


Directors to accept petitioner's offer and sell the property. Any acceptance by the
SAMD of petitioner's offer would not bind respondent. A corporation can only execute
its powers and transact its business through its Board of Directors and through its
officers and agents when authorized by a board resolution or its by-laws. Absent such
valid delegation/authorization, the rule is that the declarations of an individual
director relating to the affairs of the corporation, but not in the course of, or connected
with the performance of authorized duties of such director, is held not binding on the
corporation.

130 | P a g e
Law 321_Corporation LAW_ Case Digest

BIENVENIDO ONGKINGCO, as President and GALERIA DE MAGALLANES


CONDOMINIUM ASSOCIATION, INC.
vs.
NATIONAL LABOR RELATIONS COMMISSION and FEDERICO B. GUILAS
G.R. No. 119877, March 31, 1997

FACTS:

Petitioner Galeria de Magallanes Condominium Association, Inc. is a non-stock,


non-profit corporation with a primary purpose of holding title to the common areas of
the Galeria de Magallanes Condominium Project and to manage and administer the
same for the use and convenience of the residents and/or owners. Petitioner
Bienvenido Ongkingco was the president of Galeria at the time private respondent filed
his complaint. Subsequently, Galeria's Board of Directors appointed private
respondent Federico B. Guilas as Administrator/Superintendent. Respondent,
however, was no longer re-appointed as Administrator; hence he filed a case for illegal
dismissal. Petitioners filed a motion to dismiss alleging that it is the SEC, and not the
labor arbiter, which has jurisdiction over the subject matter of the complaint. The LA
granted the motion to dismiss, which decision was reversed by the NLRC.

ISSUE:

Whether or not respondent was a corporate officer.

RULING:

YES.

Private respondent is an officer of Petitioner Corporation and not its mere


employee. The by-laws of the Galeria de Magallanes Condominium Association
specifically include the Superintendent/Administrator in its roster of corporate
officers. He was appointed directly by the Board of Directors not by any managing
officer of the corporation and his salary was, likewise, set by the same Board. Having
thus determined, his dismissal or non-appointment is clearly an intra-corporate
matter and jurisdiction, therefore, properly belongs to the SEC and not the NLRC.
Despite not being elected, P.D. 902-A Sec. 5(c) expressly covers both election and
appointment of corporate directors, trustees, officers and managers.

131 | P a g e
Law 321_Corporation LAW_ Case Digest

ANDRES LAO
vs.
COURT OF APPEALS, THE ASSOCIATED ANGLO-AMERICAN TOBACCO
CORPORATION and ESTEBAN CO,
G.R. No. 47013, February 17, 2000

FACTS:

On April 6, 1965, The Associated Anglo-American Tobacco Corporation entered


into a "Contract of Sales Agent" with Andres Lao. Under the contract, Lao agreed to
sell cigarettes manufactured and shipped by the Corporation to his business. Lao
would in turn remit the sales proceeds to the Corporation. For his services, Lao would
receive commission depending on the kind of cigarettes sold, fixed monthly salary, and
operational allowance. For several months, Lao had religiously complied with his
obligations, however until about seven (7) months later, Lao failed to accomplish his
monthly sales report. Re was reminded of his enormous accounts and the difficulty of
obtaining a tally thereon despite Lao's avowal of regular remittances of his collections.
Because of this he was summoned to the office of the corporation where his liability
amounted to P525,053.47. The corporation stopped providing Lao with the products.
Subsequently, Andres, Jose and Tomas Lao brought a complaint for accounting and
damages with writ of preliminary injunction against the Corporation. The CFI rendered
judgment in favor of petitioners. Meanwhile, on June 24, 1974 and during the
pendency of Civil Case No. 4452, Esteban Co, representing the Corporation as its new
vice-president, filed an affidavit of complaint alleging that Lao failed to remit the
amount of P224,585.82 which he allegedly misappropriated and converted to his
personal use, which resulted to the filing of an estafa case against Lao. In turn Lao
filed a case for malicious prosecution against the corporation. In this case, Co was
made liable solidarily with the corporation for malicious prosecution.

ISSUE:

Whether or not Co should be made liable solidarily with the corporation for
malicious prosecution.

RULING:

YES.

Co asserted that he should not be held jointly and severally liable with the
Corporation because in filing the affidavit-complaint against respondent Lao, he was
acting as the executive vice-president of the Corporation and his action was within the
scope of his authority as such corporate officer. Based from the records, Co was the
vice-president of the corporation when he filed the affidavit- complaint. The
corporation failed tomake an issue out of his authority to file said case. Upon well-
established principles of pleading, lack of authority of an officer of a corporation to
bind it by contract executed by him in its name, is a defense which should have been
specially pleaded by the Corporation
The Corporation's failure to interpose such a defense could only mean that the
filing of the affidavit-complaint by petitioner Co was with the consent and authority of
the Corporation. In the same vein, petitioner Co may not be held personally liable for
acts performed in pursuance of an authority and therefore, holding him solidarily
liable with the Corporation for the damages awarded to respondent Lao does accord
with law and jurisprudence.

132 | P a g e
Law 321_Corporation LAW_ Case Digest

MITA PARDO DE TAVERA


vs.
PHILIPPINE TUBERCULOSIS SOCIETY, INC., et al
GR. No. L-48928, February 25, 1982

FACTS:

The plaintiff alleges that she is a doctor of Medicine by profession and a


recognized specialist in the treatment of tuberculosis and that she was a member of
the Board of Directors of the defendant Society, in representation of the Philippine
Charity Sweepstakes Office. She was also duly appointed as Executive Secretary of the
Society. However, she was removed from such position without any cause. This was
denied by the respondents. The appellate courts rendered decision in favor of
respondents.

ISSUE:

Whether or not the petitioner is an officer of the corporation.

RULING:

NO.

The Court ruled that there was no clear indication that the petitioner was appointed to
a permanent position. Although the minutes of the organizational meeting show that
the Chairman mentioned the need of appointing a "permanent" Executive Secretary,
such statement alone cannot characterize the appointment of petitioner without a
contract of employment definitely fixing her term. Without such term, the appointment
was deemed to be temporary, and is subject to the pleasure of the Board or of the
appointing body. Hence, when the Board opts to replace the incumbent, technically
there is no removal but only expiration of term and in an expiration of term, there is
no need of prior notice, due hearing or sufficient grounds before the incumbent can be
separated from office.

133 | P a g e
Law 321_Corporation LAW_ Case Digest

Corporate Officers; Qualifications and Disqualifications; Authority and Liabilities

MATLING INDUSTRIAL AND COMMERCIAL CORPORATION, RICHARD K.


SPENCER, CATHERINE SPENCER, AND ALEX MANCILLA
vs.
RICARDO R. COROS
G.R. No. 157802, October 13, 2010

FACTS:

The respondent filed a case for illegal dismissal against Matling and some of its
corporate officers (petitioners) in the NLRC. He was the Vice President for Finance and
Administration when he was dismissed. The petitioner opposed said complaint on the
ground that the issue at hand is an intra- corporate dispute which falls under the
jurisdiction of the SEC. The respondent opposed the same contending that his being
in the position was doubtful that he had not been formally elected as such. The LA
ruled in favor of petitioner but the same was reversed by the NLRC, and to which the
CA affirmed.

ISSUE:

Whether or not respondent is a corporate officer.

RULING:

NO.

The petitioners contend that the position of Vice President for Finance and
Administration was a corporate office, having been created by Matling‘s President
pursuant to the by-Law.
However, the Court explained that an "office" is created by the charter of the
corporation and the officer is elected by the directors or stockholders. On the other
hand, an employee occupies no office and generally is employed not by the action of
the directors or stockholders but by the managing officer of the corporation who also
determines the compensation to be paid to such employee. In this case, respondent
was appointed vice president for nationwide expansion by Malonzo, petitioner‘'s
general manager, not by the board of directors of petitioner. Also his compensation
was paid by Malonzo. Thus, respondent was an employee, not a "corporate officer.
Also, the Board of Directors of Matling could not validly delegate the power to
create a corporate office to the President, in light of Section 25 of the Corporation Code
requiring the Board of Directors itself to elect the corporate officers. Verily, the power
to elect the corporate officers was a discretionary power that the law exclusively vested
in the Board of Directors, and could not be delegated to subordinate officers or
agents.The office of Vice President for Finance and Administration created by Matling‘s
President pursuant to the by- law was an ordinary, not a corporate, office.
In this case the Court enumerated the minimum set of officers who are the
president, vice president, secretary and treasurer and modern corporation statutes
usually designate them as the officers of the corporation. However, other offices are
sometimes created by the charter or by-laws of a corporation, or the board of directors
may be empowered under the by-laws of a corporation to create additional offices as
may be necessary.

134 | P a g e
Law 321_Corporation LAW_ Case Digest

LESLIE OKOL
vs.
SLIMMERS WORLD INTERNATIONAL, BEHAVIOR MODIFICATIONS, INC., and
RONALD JOSEPH MOY
G.R. No. 160146, December 11, 2009

FACTS:

Leslie Okol, a Vice President of Slimmers World, was terminated from


employment after an incident with the Bureau of Customs regarding equipment
belonging to/consigned to Slimmers World. As such, Okolfiled a complaint with the
Arbitration branch of the NLRC against Slimmers World for illegal suspension, illegal
dismissal, unpaid commissions, damages, and attorney‘s fees, with prayer for
reinstatement and payment of backwages.
Slimmers World filed a Motion to Dismiss the case, asserting that the NLRC had
no jurisdiction over the subject matter of the complaint. Slimmers World‘s motion was
sustained, with the labor arbiter ruling that since Okol was the vice president at the
time of her dismissal, being a corporate officer, the dispute was an intra-corporate
controversy falling outside the jurisdiction of the arbitration branch. On appeal, the
NLRC reversed the LA decision and ordered Slimmers World to reinstate Okol. The CA
subsequently set aside the NLRC decision and ruled that the case was an intra-
corporate controversy, and falls within the jurisdiction of the regular courts pursuant
to RA 8799.

ISSUE:

Whether Okol was a corporate officer of Slimmers World.

RULING:

YES.

Okol was a corporate officer at the time of her dismissal. According to the
Amended By-Laws of Slimmers World which enumerate the power of the board of
directors as well as the officers of the corporation, the general management of the
corporation shall be vested in a board of five directors who shall be stockholders and
who shall be elected annually by the stockholders and who shall serve until the
election and qualification of their successors and like the Chairman of the Board and
the President, the Vice President shall be elected by the Board of Directors from its
own members. The Vice President shall be vested with all the powers and authority
and is required to perform all the duties of the President during the absence of the
latter for any cause. The Vice President will perform such duties as the Board of
Directors may impose upon him from time to time. This clearly shows that Okol was a
director and officer of Slimmers World.
An office is created by the charter of the corporation and the officer is elected by
the directors and stockholders. On the other hand, an employee usually occupies no
office and generally is employed not by action of the directors or stockholders but by
the managing officer of the corporation who also determines the compensation to be
paid to such employee.

135 | P a g e
Law 321_Corporation LAW_ Case Digest

GLORIA V. GOMEZ
vs.
PNOC DEVELOPMENT AND MANAGEMENT CORPORATION (PDMC) - (formerly
known as FILOIL DEVELOPMENT AND MANAGEMENT CORPORATION [FDMC])
G.R. No. 174044, November 27, 2009

FACTS:

Petitioner Gloria V. Gomez used to work as Manager of the Legal Department of


Petron Corporation, then a government-owned corporation. With Petron‘s
privatization, she availed of the company‘s early retirement program and left that
organization on April 30, 1994. On the following day, May 1, 1994, however, Filoil
Refinery Corporation (Filoil), also a government-owned corporation, appointed her its
corporate secretary and legal counsel, with the same managerial rank, compensation,
and benefits that she used to enjoy at Petron. However, the privatization did not
materialize so Gomez continued to serve as corporate secretary of respondent PDMC.
On March 29, 1999 the new board of directors of respondent PDMC removed
petitioner Gomez as corporate secretary. Further, at the board‘s meeting on October
21, 1999 the board questioned her continued employment as administrator. In
answer, she presented the former president‘s May 24, 1998 letter that extended her
term. Dissatisfied with this, the board sought the advice of its legal department,
which expressed the view that Gomez‘s term extension was an ultra vires act of the
former president. It reasoned that, since her position was functionally that of a vice-
president or general manager, her term could be extended under the company‘s by-
laws only with the approval of the board. The legal department held that her ―de facto‖
tenure could be legally put to an end. Petitioner Gomez for her part conceded that as
corporate secretary, she served only as a corporate officer. But, when they named her
administrator, she became a regular managerial employee. Consequently, the
respondent PDMC‘s board did not have to approve either her appointment as such or
the extension of her term in 1998.

ISSUE:

Whether or not Gomez is an ordinary employee whose complaint is within the


jurisdiction of the NLRC.

RULING:

YES.

The relationship of a person to a corporation, whether as officer or agent or


employee, is not determined by the nature of the services he performs but by the
incidents of his relationship with the corporation as they actually exist. That the
employee served concurrently as corporate secretary for a time is immaterial. A
corporation is not prohibited from hiring a corporate officer to perform services under
circumstances which will make him an employee. Indeed, it is possible for one to have
a dual role of officer and employee. NLRC has jurisdiction over a complaint filed by
one who served both as corporate officer and employee, when the money claims were
made as an employee and not as a corporate officer.

136 | P a g e
Law 321_Corporation LAW_ Case Digest

E. B. VILLAROSA & PARTNER CO., LTD.


vs.
HON. HERMINIO I. BENITO, in his capacity as Presiding Judge, RTC, Branch 132,
Makati City and IMPERIAL DEVELOPMENT CORPORATION
G.R. No. 136426, August 6, 1999

FACTS:

Petitioner E.B. Villarosa & Partner Co., Ltd. is a limited partnership. Petitioner
and private respondent executed a Deed of Sale with Development Agreement wherein
the former agreed to develop certain parcels of land belonging to the latter into a
housing subdivision for the construction of low cost housing units. They further
agreed that in case of litigation regarding any dispute arising therefrom, the venue
shall be in the proper courts of Makati. Subsequently, a complaint for breach of
contract was filed by the respondent against the plaintiff allegedly for failure of the
latter to comply with its contractual obligation in that, other than a few unfinished low
cost houses, there were no substantial developments. Summons, together with the
complaint, were served upon the defendant, through its Branch Manager Engr.
Wendell Sabulbero. The respondent moved for dismissal on the ground that there was
improper service of summons.
The trial court rendered decision denying the motion to dismiss. Hence this petition.

ISSUE:

Whether or not there was imporoper service of summons.

RULING:

YES.

Section 13, Rule 14 of the Rules of Court which provided that: Service upon
private domestic corporation or partnership, If the defendant is a corporation
organized under the laws of the Philippines or a partnership duly registered, service
may be made on the president, manager, secretary, cashier, agent, or any of its
directors.
The Court ruled that under such provision, it is clear upon whom the service of
summons should be made. The designation of persons or officers who are authorized
to accept summons for a domestic corporation or partnership is now limited and more
clearly specified. The rule now states "general manager" instead of only "manager";
"corporate secretary" instead of "secretary"; and "treasurer" instead of "cashier." The
phrase "agent, or any of its directors" is conspicuously deleted in the new rule. In this
case, since the summons was served upon a branch manager, who is not authorized
to accept the same, there was improper service of summons.

137 | P a g e
Law 321_Corporation LAW_ Case Digest

SUPREME STEEL PIPE CORPORATION and REGAN SY


vs. ROGELIO BARDAJE
G.R. No. 170811, April 24, 2007

FACTS:

Petitioner Supreme Steel Pipe Corporation (SSPC) was primarily engaged in the
business of manufacturing steel pipes. It employed respondent Rogelio Bardaje as a
warehouseman on March 14, 1994. SSPC employees were required to wear a uniform
(a yellow t-shirt with a logo and the marking "Supreme") while at work.
Due to an incident, his employment was terminated on the ground of multiple
infractions of company rules. He thus filed a case for illegal dismissal. The LA ruled for
the respondent, while the NLRC reversed said decision. The CA ruled favoring the LA.

ISSUE:

Whether or not Regan Sy, the president of SSPC, may be held solidarily liable
with the latter.

RULING:

NO.

It appears that respondent impleaded SSPC President Regan Sy only because


he is an officer/agent of the company. However, the court ruled that he cannot be
made solidarily liable because for the termination of respondent‘s employment, since
there is no showing that the dismissal was attended with malice or bad faith. The rule
still stand that the liabilities of a corporation should not be directly imputed to its
officers and it shall be borne entirely by the corporation itself.

138 | P a g e
Law 321_Corporation LAW_ Case Digest

CAGAYAN VALLEY DRUG CORPORATION


vs.
COMMISSIONER OF INTERNAL REVENUE
G.R. No. 151413, February 13, 2008

FACTS:

Petitioner is a duly licensed retailer of medicine and other pharmaceutical


products. In compliance with Revenue Regulation No. (RR) 2-94, petitioner treated the
20% sales discounts granted to qualified senior citizens in 1995 as deductions from
the gross sales in order to arrive at the net sales, instead of treating them as tax credit
as provided by Section 4 of RA 7432. however, petitioner filed with the Bureau of
Internal Revenue (BIR) a claim for tax refund/tax credit of the full amount of the 20%
sales discount it granted to senior citizens for the year 1995, allegedly totalling to
P123,083.00. Because of the BIR‘s inaction, the petitioner filed a petition for review
before the CTA, which denied its claim. The CA also denied the same on procedural
grounds, such that the person who signed the verification and certification of absence
of forum shopping, a certain Jacinto J. Concepcion, President of petitioner, failed to
adduce proof that he was duly authorized by the board of directors to do so.

ISSUE:

Whether petitioner‘s president can sign the subject verification and certification
without the approval of its Board of Directors.

RULING:

YES.

In several cases the court has recognized the authority of some corporate
officers to sign the verification and certification against forum shopping. In these
cases, the court allowed the: (1) the Chairperson of the Board of Directors, (2) the
President of a corporation, (3) the General Manager or Acting General Manager, (4)
Personnel Officer, and (5) an Employment Specialist in a labor case, to sign said
documents, without need of a board resolution.
Also in this case, an authorization was belatedly submitted. Although belated,
the court still accepts it as a valid and which it had cured the procedural infirmities of
the case.

139 | P a g e
Law 321_Corporation LAW_ Case Digest

SALOME PABON and VICENTE CAMONAYAN


vs.
NATIONAL LABOR RELATIONS COMMISSION and SENIOR MARKETING
CORPORATION
G.R. No. 120457 September 24, 1998

FACTS:

On May 24, 1994 and June 22, 1994, complaints for illegal dismissal and non-
payment of benefits were filed by petitioners Salome Pabon and Vicente Camonayan
against private respondent Senior Marketing Corporation (SMC) and its Field Manager,
R-Jay Roxas Summons and notices of hearings were sent to Roxas at private
respondent's provincial office in 13 Valley Homes, Patul Road, Santiago, Isabela which
were received by its bookkeeper, Mina Villanueva.
On September 15, 1994, the Labor Arbiter rendered a judgment by default after
finding that private respondent tried to evade all the summons and orders of hearing
by refusing to claim all the registered mail addressed to it.

ISSUE:

Whether or not Petitioners herein are authorized to receive summons in behalf


of the corporation.

RULING:

YES.

Bookkeeper can be considered as an agent of private respondent corporation


within the purview of Section 13, Rule 14 of the old Rules of Court. The rationale of all
rules with respect to service of process on a corporation is that such service must be
made to an agent or a representative so integrated with the corporation sued as to
make it a priori supposable that he will realize his responsibilities and know what he
should do with any legal papers served on him. The bookkeeper's task is one under
consideration. The job of a bookkeeper is so integrated with the corporation that his
regular recording of the corporation's "business accounts" and "essential facts about
the transactions of a business or enterprise" safeguards the corporation from possible
fraud being committed adverse to its own corporate interest.
Although it may be true that the service of summons was made on a person not
authorized to receive the same in behalf of the petitioner, nevertheless since it appears
that the summons and complaint were in fact received by the corporation through its
said clerk, the Court finds that there was substantial compliance with the rule on
service of summons. Indeed the purpose of said rule as above stated to assure service
of summons on the corporation had thereby been attained. The need for speedy justice
must prevail over technicality.

140 | P a g e
Law 321_Corporation LAW_ Case Digest

VLASON ENTERPRISES CORPORATION


vs.
COURT OF APPEALS and DURAPROOF SERVICES, represented by its General
Manager, Cesar Urbino Sr.
G.R. Nos. 121662-64. July 6, 1999

FACTS:

Poro Point Shipping Services, then acting as the local agent of Omega Sea
Transport Company of Honduras & Panama, a Panamanian company, (hereafter
referred to as Omega), requested permission for its vessel M/V Star Ace, which had
engine trouble, to unload its cargo and to store it at the Philippine Ports Authority
(PPA) compound in San Fernando, La Union while awaiting transshipment to Hong
Kong. The request was approved by the Bureau of Customs. Despite the approval, the
customs personnel boarded the vessel when it docked on January 7, 1989, on
suspicion that it was the hijacked M/V Silver Med owned by Med Line Philippines Co.,
and that its cargo would be smuggled into the country. The district customs collector
seized said vessel and its cargo pursuant to Section 2301, Tariff and Customs Code.
They entered into a salvage agreement with private respondent to secure and
repair the vessel which was destroyed by the typhoons that hit the province at the
agreed consideration of $1 million and ―fifty percent (50%) of the cargo after all
expenses, cost and taxes.‖ Subsequently, the seizure was lifted for want of fraud.

ISSUE:

Whether or not summons was properly served with Vlason Corporation.

RULING:

NO.

Sec 4 and 5 of the Rules of Court ideally requires a movant to address and
serve on the counsel of the adverse party the notice of hearing of its motion. Service of
a copy of a motion must contain a notice of the time and the place of hearing. There
are, however, exceptions to the rule: Where a rigid application will result in a manifest
failure or miscarriage of justice, especially if a party successfully shows that the
alleged defect in the questioned final and executory judgment is not apparent on its
face or from the recitals contained therein; Where the interest of substantial justice
will be served; Where the resolution of the motion is addressed solely to the sound and
judicious discretion of the court; Where the injustice to the adverse party is not
commensurate to the degree of his failure to comply with prescribed procedure
In this case, Vlason was not informed of any cause of action against it. It was
not validly summoned. Its vessels that it used for its salvaging business was levied
upon and sold in execution to satisfy a supposed judgment against it. To allow this to
happen simply because of its failure to comply with the notice requirement would
result into manifest injustice.

141 | P a g e
Law 321_Corporation LAW_ Case Digest

Prime White Cement Corporation


vs.
Intermediate Appellate Court
GR 68555, 19 March 1993

FACTS:

On or about 16 July 1969, Alejandro Te and Prime White Cement Corporation


(PWCC) thru its President, Mr. Zosimo Falcon and Justo C. Trazo, as Chairman of the
Board, entered into a dealership agreement whereby Te was obligated to act as the
exclusive dealer and/or distributor of PWCC of its cement products in the entire
Mindanao area for a term of 5 years.
Right after Te entered into the dealership agreement, he placed an
advertisement in a national, circulating newspaper the fact of his being the exclusive
dealer of PWWC's white cement products in Mindanao area, more particularly, in the
Manila Chronicle dated 16 August 1969 and was even congratulated by his business
associates, so much so, he was asked by some of his businessmen friends and close
associates if they can be his sub-dealer in the Mindanao area.

ISSUE:

Whether the "dealership agreement" referred by the President and Chairman of


the Board of PWCC is a valid and enforceable contract.

RULING:

NO.

The ―dealership agreement‖ is not valid and unenforceable. Under the


Corporation Law, which was then in force at the time the case arose, as well as under
the present Corporation Code, all corporate powers shall be exercised by the Board of
Directors, except as otherwise provided by law. Although it cannot completely abdicate
its power and responsibility to act for the juridical entity, the Board may expressly
delegate specific powers to its President or any of its officers.
In the absence of such express delegation, a contract entered into by its
President, on behalf of the corporation, may still bind the corporation if the board
should ratify the same expressly or impliedly. Implied ratification may take various
forms — like silence or acquiescence; by acts showing approval or adoption of the
contract; or by acceptance and retention of benefits flowing therefrom. Furthermore,
even in the absence of express or implied authority by ratification, the President as
such may, as a general rule, bind the corporation by a contract in the ordinary course
of business, provided the same is reasonable under the circumstances. These rules
are basic, but are all general and thus quite flexible. They apply where the President or
other officer, purportedly acting for the corporations, is dealing with a third person,
i.e., a person outside the corporation. The situation is quite different where a director
or officer is dealing with his own corporation. Herein, Te was not an ordinary
stockholder; he was a member of the Board of Directors and Auditor of the corporation
as well. He was what is often referred to as a "self-dealing" director.

142 | P a g e
Law 321_Corporation LAW_ Case Digest

LOUIS VUITTON S.A.


vs.
JUDGE FRANCISCO DIAZ VILLANUEVA, presiding Judge, Branch 36, The
Metropolitan Trial Court at Quezon City, Metro Manila
A.M. No. MTJ-92-643 November 27, 1992

FACTS:

In Criminal Case No. XXXVI-62431, entitled "People of the Philippines vs. Jose
V. Rosario", Louis Vuitton, S.A. accused the latter of unfair competition as defined by
paragraph 1 of Article 189, Revised Penal Code.
Complainant also assailed respondent judge's findings that there was no unfair
competition because the elements of the crime were not met, and that he seized
articles did not come close to the appearance of a genuine Louis Vuitton product, the
counterfeit items having been poorly, done.

ISSUE:

Whether or not respondent judge is guilty of knowingly rendering a manifestly


unjust judgment.

RULING:

NO.

The ground which was relied upon by the trial court in acquitting the accused
finds basis in the well-settled doctrine that a corporation has a distinct personality
from that of its stockholders/owners. A corporation is vested by law with a personality
of its own, separate and distinct from that of its stockholders and from that of its
officers who manage and run its affairs. This decision is assailed to be unjust mainly
because it did not consider the Prosecution's Memorandum with Motion and Motion
for Early Resolution filed by private prosecutor, herein complainant, on February 8,
1991 and February 11, 1991, respectively. According to complainant, had respondent
judge taken the former motion into account, he would not have acquitted the accused,
Jose V. Rosario. Instead, he would have been held guilty for giving others an
opportunity engage in unfair competition as prescribed by Article 189 of the Revised
Penal Code.
In the first place, it would not have made any difference because Jose v. Rosario
was charged as owner/proprietor. COD is not a single proprietorship but one that is
run and owned by a corporation, Rosario Bros., Inc., of which the accused is
stockholder and Executive Vice-President. A stockholder generally does not have a
hand in the management of the corporate affairs. On the other hand, the Vice-
President had no inherent power to bind the corporation.
As general rule, his duties must be specified in the by-laws. In the criminal
case, the information did not specify his duties as Executive Vice-President. The trial
court had no basis for holding that as such, the accused entered into a contract with
the concessionaire thereby giving the latter an opportunity to practice unfair
competition. Whereas, Section 23 of the Corporation Code is explicit that the directors,
acting as a body, exercise corporation powers and conduct the corporation's business.

143 | P a g e
Law 321_Corporation LAW_ Case Digest

Doctrine of Apparent Authority

VIOLETA BANATE et. al.


vs.
PHILIPPINE COUNTRYSIDE RURAL BANK
GR 163825, 13 July 2010

FACTS:

Sometime in November 1997 the spouses Maglasang and the spouses Cortel
asked PCRB‘s permission to sell the properties which they mortgaged with the bank.
They likewise requested that the said properties be released from the mortgage since
the two other loans were adequately secured by the other mortgages. The spouses
Maglasang and the spouses Cortel claimed that the PCRB, acting through its Branch
Manager, Pancrasio Mondigo, verbally agreed to their request but required first the full
payment of the subject loan. They thereafter sold to petitioner Violeta Banate the
subject properties for P1,750,000.00 and used the amount to pay the subject loan
with PCRB. After settling the subject loan, PCRB gave the owner‘s duplicate certificate
of title of Lot 12868-H-3-C to Banate, who was able to secure a new title in her name.
It, however, carried the mortgage lien in favor of PCRB, prompting the petitioners to
request from PCRB a Deed of Release of Mortgage. As PCRB refused to comply with
the petitioners‘ request, the petitioners instituted an action for specific performance
before the RTC to compel PCRB to execute the release deed.
Accordingly, PCRB claimed that full payment of the three loans, obtained by the
spouses Maglasang, was necessary before any of the mortgages could be released; the
settlement of the subject loan merely constituted partial payment of the total
obligation. Thus, the payment does not authorize the release of the subject properties
from the mortgage lien.

ISSUE:

Whether or not Mondigo, as branch manager of PCRB, has the authority to


modify the original mortgage contract on behalf of the company.

RULING:

NO.

He is not authorized to modify the mortgage contract that would in effect cause
novation. Under the doctrine of apparent authority, acts and contracts of the agent, as
are within the apparent scope of the authority conferred on him, although no actual
authority to do such acts or to make such contracts has been conferred, bind the
principal. The principal‘s liability, however, is limited only to third persons who have
been led reasonably to believe by the conduct of the principal that such actual
authority exists, although none was given. In other words, apparent authority is
determined only by the acts of the principal and not by the acts of the agent. There
can be no apparent authority of an agent without acts or conduct on the part of the
principal; such acts or conduct must have been known and relied upon in good faith
as a result of the exercise of reasonable prudence by a third party as claimant, and
such acts or conduct must have produced a change of position to the third party‘s
detriment.
In the present case, the decision of the trial court was utterly silent on the
manner by which PCRB, as supposed principal, has ―clothed‖ or ―held out‖ its branch
manager as having the power to enter into an agreement, as claimed by petitioners. No
proof of the course of business, usages and practices of the bank about, or knowledge
that the board had or is presumed to have of, its responsible officers‘ acts regarding
bank branch affairs, was ever adduced to establish the branch manager‘s apparent
authority to verbally alter the terms of mortgage contracts. Neither was there any
allegation, much less proof, that PCRB ratified Mondigo‘s act or is estopped to make a
contrary claim.

144 | P a g e
Law 321_Corporation LAW_ Case Digest

SARGASSO CONSTRUCTION
vs.
PHILIPPINE PORTS AUTHORITY
GR 170530, 05 July 2010

FACTS:

Plaintiff Sargasso Construction and Development Corporation, Pick and Shovel,


Inc. and Atlantic Erectors, Inc., a joint venture, was awarded the construction of Pier 2
and the rock causeway (R.C. Pier 2) for the port of San Fernando, La Union, after a
public bidding conducted by the defendant PPA.
Plaintiff offered to undertake the reclamation between the Timber Pier and Pier
2 of the Port of San Fernando, La Union, as an extra work to its existing construction
of R.C. Pier 2 and Rock Causeway for a price of P36,294,857.03. The offer was
unacceptable to PPA and thereafter asked for its reduction to P30,794,230.89.
On August 26, 1993, a Notice of Award signed by PPA General Manager Rogelio
Dayan was sent to plaintiff for the phase I Reclamation Contract in the amount of
P30,794,230.89 and instructing it to ―enter into and execute the contract agreement
with this Office‖ and to furnish the documents representing performance security and
credit line. PPA Management further set a condition [that] ―the acceptance by the
contractor that mobilization/demobilization cost shall not be included in the contract
and that escalation shall be reckoned upon approval of the Supplemental Agreement.
Hence, then General Manager Carlos L. Agustin presented for consideration by the
PPA Board of Directors the contract proposal for the reclamation project.
At its meeting held on September 9, 1994, the Board decided not to approve the
contract proposal. The Board noted that the Pier 2 Project was basically for the
construction of a pier while the supplemental agreement refers to reclamation. Thus
there is no basis to compare the terms and conditions of the reclamation project with
the original contract (Pier 2 Project) of Sargasso.‖ Plaintiff filed a complaint for specific
performance and damages.

ISSUE:

Whether or not the Notice of Award made by the General Manager of PPA
binding with the entity.

RULING:

NO.

There is no perfected contract between the parties in this case. Likewise, the
General Manager of the PPA is not authorized to enter into contracts on behalf of the
agency. Petitioner‘s invocation of the doctrine of apparent authority is misplaced. This
doctrine, in the realm of government contracts, has been restated to mean that the
government is NOT bound by unauthorized acts of its agents, even though within the
apparent scope of their authority. Under the law on agency, however, ―apparent
authority‖ is defined as the power to affect the legal relations of another person by
transactions with third persons arising from the other‘s manifestations to such third
person such that the liability of the principal for the acts and contracts of his agent
extendsto those which are within the apparent scope of the authority conferred on
him, although no actual authority to do such acts or to make such contracts has been
conferred.
The existence of apparent authority may be ascertained through (1) the general
manner in which the corporation holds out an officer or agent as having the power to
act or, in other words, the apparent authority to act in general, with which it clothes
him; or (2) the acquiescence in his acts of a particular nature, with actual or
constructive knowledge thereof, whether within or beyond the scope of his ordinary
powers. It requires presentation of evidence of similar act(s) executed either in its favor
or in favor of other parties. In this case, not a single act of respondent, acting through
its Board of Directors, was cited as having clothed its general manager with apparent
authority to execute the contract with it.

145 | P a g e
Law 321_Corporation LAW_ Case Digest

ASSOCIATED BANK (now UNITED OVERSEAS BANK [PHILS.])


vs.
SPOUSES RAFAEL and MONALIZA PRONSTROLLER,
G.R. No. 148444, July 14, 2008

FACTS:

On April 21, 1988, spouses Vaca executed a Real Estate Mortgage (REM) in
favor of the petitioner over their parcel of residential land located at Quezon City. For
failure of the spouses Vaca to pay their obligation, the subject property was sold at
public auction with the petitioner as the highest bidder. The spouses Vaca, however,
commenced an action for the nullification of the real estate mortgage and the
foreclosure sale. During the pendency of the aforesaid cases, Respondents Rafael and
Monaliza Pronstroller offered to purchase the property for P7,500,000.00. Said offer
was made through Atty. Jose Soluta, Jr., petitioner‘s Vice-President, Corporate
Secretary and a member of its Board of Directors. Petitioner accepted respondents‘
offer of P7.5 million. Consequently, respondents paid petitioner P750,000.00, or 10%
of the purchase price, as down payment. On March 18, 1993, petitioner, through Atty.
Soluta, and respondents, executed a Letter-Agreement.

ISSUE:

Whether or not the petitioner is bound by the July 14, 1993 Letter-Agreement
signed by Atty. Soluta under the doctrine of apparent authority.

RULING:

YES.

The general rule is that, in the absence of authority from the board of directors,
no person, not even its officers, can validly bind a corporation. The power and
responsibility todecide whether the corporation should enter into a contract that will
bind the corporation islodged in the board of directors. However, just as a natural
person may authorize another to do certain acts for and on his behalf, the board may
validly delegate some of its functions and powers to officers, committees and agents.
The authority of such individuals to bind the corporation is generally derived from law,
corporate bylaws or authorization from the board, either expressly or impliedly, by
habit, custom, or acquiescence, in the general course of business.The authority of a
corporate officer or agent in dealing with third persons may be actual or apparent.
The doctrine of ―apparent authority,‖ with special reference to banks, had long
been recognized in this jurisdiction. Apparent authority is derived not merely from
practice. Its existence may be ascertained through 1) the general manner in which the
corporation holds out an officer or agent as having the power to act, or in other words,
the apparent authority to act in general, with which it clothes him; 2) the acquiescence
in his acts of a particular nature, with actual or constructive knowledge thereof,
within or beyond the scope of his ordinary powers. Accordingly, the authority to act for
and to bind a corporation may be presumed from acts of recognition in other
instances, wherein the power was exercised without any objection from its board or
shareholders.

146 | P a g e
Law 321_Corporation LAW_ Case Digest

EMILIANO ACUÑA
vs.
BATAC PRODUCERS
GR L-20333, 30 June 1963

FACTS:

Acuña entered into a contract with Batac wherein he agreed to advance


P20,000.00 to the company for its tobacco planting and drying, provided that he shall
be assigned as the company‘s representative in Manila and supervise the transport
and delivery of the goods in the said place. Batac‘s Board of Directors are amenable
with the idea and thereafter issued a resolution authorizing Manager Leon Verano to
enter into the agreement on behalf of the corporation.
The necessary contract between Acuña and Verano was entered into, with some
of the Board of Directors acting as witness. Acuña then inquired if the contract needs
to be ratified by the Board, in which the counsel for Batac answered in the negative.
Acuña thereafter proceeded to perform his part of the contrac, including the
advancement of the amount promised, which was accepted by Batac.
Batac‘s BoD, however, disapproved the contract. Acuña insisted on its
performance, but the corporation refused, stating that the contract is not binding by
reason that it was not ratified by the board.

ISSUE:

Whether or not the contract between Acuña and Verano is binding with the
corporation.

RULING:

YES.

A perusal of the complaint reveals that it contains sufficient allegations


indicating such approval or at least subsequent ratification. On the first point note the
following averments: that on May 9th the plaintiff met with each and all of the
individual defendants (who constituted the entire Board of Directors) and discussed
with them extensively the tentative agreement and he was made to understand that it
was acceptable to them, except as to plaintiff's remuneration; that it was finally agreed
between plaintiff and all said Directors that his remuneration would be P0.30 per kilo
(of tobacco); and that after the agreement was formally executed he was assured by
said Directors that there would be no need of formal approval by the Board. It should
be noted in this connection that although the contract required such approval it did
not specify just in what manner the same should be given.
On the question of ratification the complaint alleges that plaintiff delivered to
the defendant corporation the sum of P20,000.00 as called for in the contract; that he
rendered the services he was required to do; that he furnished said defendant 3,000
sacks at a cost of P6,000.00 and advanced to it the further sum of P5,000.00; and that
he did all of these things with the full knowledge, acquiescence and consent of each
and all of the individual defendants who constitute the Board of Directors of the
defendant corporation. There is abundant authority in support of the proposition that
ratification may be expressed or implied, and that implied ratification may take diverse
forms, such as by silence or acquiescence; by acts showing approval or adoption of the
contract; or by acceptance and retention of benefits flowing therefrom.

147 | P a g e
Law 321_Corporation LAW_ Case Digest

BOARD OF LIQUIDATORS
vs.
HEIRS OF MAXIMO KALAW
GR L-18805, 14 August 1967

FACTS:

National Coconut Corporation (NACOCO) is with Maximo Kalaw as its General


Manager and Chairman of the BOD. Under his tenure NACOCO entered into different
contracts involving the trade of coconuts. It failed, however, due to natural calamities
that greatly affected the production of coconuts. This led to some customers of
NACOCO suing the corporation for undelivered coconuts due to them under the
contracts that they signed. This was settled by NACOCO by paying the customers.
Thereafter, NACOCO seeks to recover the above sum of P1,343,274.52 from
general manager and board chairman Maximo M. Kalaw, and directors Juan Bocar,
Casimiro Garcia and Leonor Moll. It charges Kalaw with negligence under Article 1902
of the old Civil Code (now Article 2176, new Civil Code); and defendant board
members, including Kalaw, with bad faith and/or breach trust for having approved the
contracts.

ISSUE:

Whether or not Kalaw may be held liable by NACOCO for the debts the
corporation incurred under his administration.

RULING:

NO.

They were done with implied authority from the BOD. These previous contracts,
it should be stressed, were signed by Kalaw without prior authority from the board.
Said contracts were known all along to the board members. Nothing was said by them.
The aforesaid contracts stand to prove one thing. Obviously NACOCO board met the
difficulties attendant to forward sales by leaving the adoption of means to end, to the
sound discretion of NACOCO's general manager Maximo M. Kalaw.
Settled jurisprudence has it that where similar acts have been approved by the
directors as a matter of general practice, custom, and policy, the general manager may
bind the company without formal authorization of the board of directors. In varying
language, existence of such authority is established, by proof of the course of
business, the usages and practices of the company and by the knowledge which the
board of directors has, or must be presumed to have, of acts and doings of its
subordinates in and about the affairs of the corporation.
Authorities, great in number, are one in the idea that "ratification by a
corporation of an unauthorized act or contract by its officers or others relates back to
the time of the act or contract ratified, and is equivalent to original authority;" and
that "[t]he corporation and the other party to the transaction are in precisely the same
position as if the act or contract had been authorized at the time." The language of one
case is expressive: "The adoption or ratification of a contract by a corporation is
nothing more nor less than the making of an original contract. The theory of corporate
ratification is predicated on the right of a corporation to contract, and any ratification
or adoption is equivalent to a grant of prior authority.

148 | P a g e
Law 321_Corporation LAW_ Case Digest

TRINIDAD FRANCISCO
vs.
GOVERNMENT SERVICE INSURANCE SYSTEM
GR L-18287, 30 March 1963

FACTS:

On 10 October 1956, the plaintiff, Trinidad J. Francisco, in consideration of a


loan in the amount of P400,000.00, out of which the sum of P336,100.00 was released
to her, mortgaged in favor of the defendant, GSIS, a parcel of land containing an area
of 18,232 square meters, with twenty-one (21) bungalows, known as Vic-Mari
Compound, located at Baesa, Quezon City, payable within ten (10) years in monthly
installments of P3,902.41, and with interest of 7% per annum compounded monthly.
On 6 January 1959, the System extrajudicially foreclosed the mortgage on the
ground that up to that date the plaintiff-mortgagor was in arrears on her monthly
installments in the amount of P52,000.00. Payments made by the plaintiff at the time
of foreclosure amounted to P130,000.00. The System itself was the buyer of the
property in the foreclosure sale.
Trinidad‘s father, Vicente Francisco, offered to pay part of the arrears with
P30,000.00 with the remainder of the arrears plus the balance of the loan shall be
paid with the proceeds of the rents of the property which shall be administered by
GSIS. GSIS informed the plaintiff through a telegraph made its General Manager
Rodolfo Andal and accepted the tender of P30,000.00 but did not take over the
administration of the property. Meanwhile, Trinidad received the rents amounting to
P44,000.00 and remitted the same to GSIS, who issued a receipt thereto.
GSIS, however, notified the appellant that the time for redemption is about to
expire and requested the Franciscos to offer them a system of payment. Trinidad
questioned the action, standing by the validity of the proposal they made with GSIS,
whose acceptance of the payment amounted to estoppel. he defendant answered,
pleading that the binding acceptance of Francisco's offer was the resolution of the
Board, and that Andal's telegram, being erroneous, should be disregarded.

ISSUE:

Whether or not the approval of the proposal made by GSIS is valid.

RULING:

YES.

The terms of the offer were clear, and over the signature of defendant's general
manager, Rodolfo Andal, plaintiff was informed telegraphically that her proposal had
been accepted. There was nothing in the telegram that hinted at any anomaly, or gave
ground to suspect its veracity, and the plaintiff, therefore, cannot be blamed for
relying upon it. There is no denying that the telegram was within Andal's apparent
authority, but the defense is that he did not sign it, but that it was sent by the Board
Secretary in his name and without his knowledge. Assuming this to be true, how was
appellee to know it? Corporate transactions would speedily come to a standstill were
every person dealing with a corporation held duty-bound to disbelieve every act of its
responsible officers, no matter how regular they should appear on their face
If a private corporation intentionally or negligently clothes its officers or agents
with apparent power to perform acts for it, the corporation will be estopped to deny
that such apparent authority is real, as to innocent third persons dealing in good faith
with such officers or agents. Knowledge of facts acquired or possessed by an officer or
agent of a corporation in the course of his employment, and in relation to matters
within the scope of his authority, is notice to the corporation, whether he
communicates such knowledge or not.

149 | P a g e
Law 321_Corporation LAW_ Case Digest

RURAL BANK OF MILAOR (CAMARINES SUR)


vs.
FRANCISCA OCFEMIA, et. al.
GR 137686, 08 February 2000

FACTS:

The spouses Felicisimo Ocfemia and Juanita Arellano Ocfemia were not able to
redeem the mortgaged properties consisting of seven (7) parcels of land from Milaor
and so the mortgage was foreclosed and thereafter ownership thereof was transferred
to the bank. Out of the seven (7) parcels that were foreclosed, five (5) of them are in
the possession of the Ocfemias because these were sold by the [petitioner] bank to the
parents of Marife Ocfemia Niño as evidenced by a Deed of Sale executed in January
1988.
Marife went to the Register of Deeds of Camarines Sur with the Deed of Sale
(Exh. C) in order to have the same registered. The Register of Deeds, however,
informed her that the document of sale cannot be registered without a board
resolution of the Bank. Marife then went to the bank, showed to it the Deed of Sale,
the tax declaration and receipt of tax payments and requested the bank for a board
resolution so that the property can be transferred to the name of Marife‘s parents
Renato Ocfemia and Francisca Ocfemia.
The bank, after requiring so many requirements and making so many alibis to
Marife, refused to issue the board resolution. It claims that its bank manager Fe Tena
did not have authority to sell the properties to the Ocfemias therefore rendering the
deed of sale invalid.

ISSUE:

Whether or not the bank manager has authority to act on behalf of the bank.

RULING:

YES.

There was an apparent authority bestowed with Tena. The bank acknowledged,
by its own acts or failure to act, the authority of Fe S. Tena to enter into binding
contracts. After the execution of the Deed of Sale, respondents occupied the properties
in dispute and paid the real estate taxes due thereon. If the bank management
believed that it had title to the property, it should have taken some measures to
prevent the infringement or invasion of its title thereto and possession thereof.
Likewise, Tena had previously transacted business on behalf of the bank, and
the latter had acknowledged her authority. A bank is liable to innocent third persons
where representation is made in the course of its normal business by an agent like
Manager Tena, even though such agent is abusing her authority. Clearly, persons
dealing with her could not be blamed for believing that she was authorized to transact
business for and on behalf of the bank.
In this light, the bank is estopped from questioning the authority of the bank
manager to enter into the contract of sale. If a corporation knowingly permits one of its
officers or any other agent to act within the scope of an apparent authority, it holds
the agent out to the public as possessing the power to do those acts; thus, the
corporation will, as against anyone who has in good faith dealt with it through such
agent, be estopped from denying the agent‘s authority.
More so, the bank is in default for failing to answer the complaint of the
Ocfemias within the reglamentary period without any justifiable excuse.

150 | P a g e
Law 321_Corporation LAW_ Case Digest

Personal Liability of Directors and Other Corporate Officers

EVER ELECTRICAL MANUFACTURING, INC., (EEMI) and VICENTE GO


vs.
SAMAHANG MANGGAGAWA NG EVER ELECTRICAL/NAMAWU LOCAL
G.R. No. 194795. June 13, 2012.

FACTS:

EEMI is a corporation engaged in the business of manufacturing electrical parts


and supplies. EEMI was one of those who suffered huge losses during the Asian
financial crisis. In 1996, EEMI obtained a loan from UCPB. As security, EEMI‘s land
and improvements, including the factory, were mortgaged to UCPB. EEMI‘s business
suffered further losses due to the continued entry of cheaper goods from China and
other Asian countries. Later, Orient Bank, where EEMI invested 500MIO, went out of
business. As aresult, EEMI defaulted on its loan with UCPB. In an attempt to save the
company, EEMI entered into a dacion en pago arrangement with UCPB which, in
effect, transferred ownership of the company‘s property to UCPB. Since UCPB‘s policy
prohibited EEMI from leasing the premises directly with UCPB, UCPB agreed to lease
it to an affiliate corporation, EGO Electrical Supply Co, Inc. (EGO), for and in behalf of
EEMI. However, UCPB later instituted an unlawful detainer suit against EGO, wherein
UCPB won. The Sheriff implemented the writ of execution by closing the premises and,
as a result, EEMI‘s employees were prevented from entering the factory. Aggrieved,
UNION filed a complaint for illegal dismissal with prayer for payment of 13th month
pay, separation pay, damages, and attorney‘s fees.

ISSUE:

Whether or not Vicente Go should be held solidarily liable with EEMI.

RULING:

NO.

As a general rule, corporate officers should not be held solidarily liable with the
corporation for separation pay for it is settled that a corporation is invested by law
with a personality separate and distinct from those of the persons composing it as well
as from that of any other legal entity to which it may be related. Mere ownership by a
single stockholder or by another corporation of all or nearly all of the capital stock of a
corporation is not of itself sufficient ground for disregarding the separate corporate
personality.
A corporation is invested by law with a personality separate and distinct from
those of the persons composing it as well as from that of any other legal entity
to which it may be related. Corporate directors and officers become solidarily liable
with the corporation for the termination of employees done with malice or bad faith.
It stressed that bad faith does not connote bad judgment or negligence; it
imports a dishonest purpose or some moral obliquity and conscious doing of wrong; it
means breach of a known duty through some motive or interest or ill will; it partakes
of the nature of fraud.

151 | P a g e
Law 321_Corporation LAW_ Case Digest

HARPOON MARINE SERVICES, Inc. and JOSE LIDO T. ROSIT


vs.
FERNAN H. FRANCISCO
G.R. No. 167751. March 2, 2011

FACTS:

Respondent averred that he was unceremoniously dismissed by petitioner Rosit.


He was informed that the company could no longer afford his salary and that he would
be paid his separation pay and accrued commissions. Respondent nonetheless
continued to report for work. A few days later, however, he was barred from entering
the company premises. Relying on the promise of petitioner Rosit, respondent went to
the office on June 30, 2001 to receive his separation pay and commissions, but
petitioner Rosit offered only his separation pay. Respondent refused to accept it and
also declined to sign a quitclaim. After several unheeded requests, respondent,
through his counsel, sent a demand letter dated September 24, 2001 to petitioners
asking for payment of P70,000.00, which represents his commissions for the seven
boats constructed and repaired by the company under his supervision. In a letter-
reply dated September 28, 2001, petitioners denied that it owed respondent any
commission, asserting that they never entered into any contract or agreement for the
payment of commissions. Hence, on October 24, 2001, respondent filed an illegal
dismissal complaint praying for the payment of his backwages, separation pay, unpaid
commissions, moral and exemplary damages and attorney‘s fees.

ISSUE:

Whether or not the President is solidarily liable with the corporation.

RULING:

NO.

Though the Court found that Respondent was illegally dismissed, it held that
the President of the Petitioner Corporation should not be held solidarily liable with
Petitioner Corporation. Obligations incurred by corporate officers, acting as such
corporate agents, are not theirs but the direct accountabilities of the corporation they
represent. Thus, they should not be generally held jointly and solidarily liable with the
corporation. The general rule is grounded on the theory that a corporation has a legal
personality separate and distinct from the persons comprising it.
As exceptions to the general rule, solidary liability may be imposed: (1) When
directors and trustees or, in appropriate cases, the officers of a corporation –(a) vote
for or assent to [patently] unlawful acts of the corporation; (b) act in bad faith or with
gross negligence in directing the corporate affairs; (c) are guilty of conflict of interest to
the prejudice of the corporation, its stockholders or members, and other persons; (2)
When the director or officer has consented to the issuance of watered stock or who,
having knowledge thereof, did not forthwith file with the corporate secretary his
written objection thereto; (3) When a director, trustee or officer has contractually
agreed or stipulated to hold himself personally and solidarily liable with the
corporation; (4) When a director, trustee or officer is made, by specific provision of law,
personally liable for his corporate action. To warrant the piercing of the veil of
corporate fiction, the officer‘s bad faith or wrongdoing must be established clearly and
convincingly as bad faith is never presumed.

152 | P a g e
Law 321_Corporation LAW_ Case Digest

ARNEL TY, et al.


vs.
NBI AGENT MARTIN JAMIL, et al.
GR 182147, 15 December 2010

FACTS:

Acting on a complaint filed by various LPG manufacturers, the NBI conducted a


surveillance on the activities of Omni Gas, which is being accused of selling LPG tanks
without the required permit and below its required standards. Agents De Jemil and
Kawada attested to conducting surveillance of Omni and brought eight branded LPG
cylinders of Shellane, Petron Gasul, Totalgaz, and Superkalan Gaz to Omni for
refilling. The branded LPG cylinders were refilled, for which agents paid P1,582 as
evidenced by Sales Invoice No. 90040issued by Omni on April 15, 2004. The refilled
LPG cylinders were without LPG valve seals and one of the cylinders was actually
underfilled, as found by LPG Inspector Noel N. Navio of the Liquefied Petroleum Gas
Industry Association (LPGIA) who inspected the eight branded LPG cylinders.
The NBI's test-buy yielded positive results for violations of BP 33, Section 2(a) in
relation to Secs. 3(c) and 4, i.e., refilling branded LPG cylinders without authority; and
Sec. 2(c) in relation to Sec. 4, i.e., under delivery or under filling of LPG cylinders.
Petitioners Arnel Ty, Marie Antonette Ty, Jason Ong, Willy Dy, and Alvin Ty
questioned the case against that, claiming that being mere directors, they are not
liable to the case filed against Omni for they are not in charge of the management of
the said entity.

ISSUE:

Whether or not petitioners may be held liable for the actions of Omni.

RULING:

NO.

Only Arnel Ty may be held liable in his capacity as president of Omni, but not
the other directors. The corporate powers of a corporation are reposed in the board of
directors under the first paragraph of Sec. 23 of the Corporation Code, it is of common
knowledge and practice that the board of directors is not directly engaged or charged
with the running of the recurring business affairs of the corporation. Depending on
the powers granted to them by the Articles of Incorporation, the members of the board
generally do not concern themselves with the day-to-day affairs of the corporation,
except those corporate officers who are charged with running the business of the
corporation and are concomitantly members of the board, like the President. Section
25of the Corporation Code requires the president of a corporation to be also a member
of the board of directors.
Evidently, petitioner Arnel, as President, who manages the business affairs of
Omni, can be held liable for probable violations by Omni of BP 33, as amended. The
fact that petitioner Arnel is ostensibly the operations manager of Multi-Gas
Corporation, a family owned business, does not deter him from managing Omni as
well. It is well-settled that where the language of the law is clear and unequivocal, it
must be taken to mean exactly what it says. As to the other petitioners, unless
otherwise shown that they are situated under the catch-all "such other officer charged
with the management of the business affairs," they may not be held liable under BP
33, as amended, for probable violations. Consequently, with the exception of petitioner
Arnel, the charges against other petitioners must perforce be dismissed or dropped.
Also, under BP 33 (which regulates the production and sale of LPG), Directors
are not among those enumerated as criminally liable for the acts of the corporation.

153 | P a g e
Law 321_Corporation LAW_ Case Digest

QUEENSLAND-TOKYO COMMODITIES et al.


vs.
THOMAS GEORGE
GR 172727, 08 September 2010

FACTS:

QTCI is a duly licensed broker engaged in the trading of commodity futures. In


1995, Guillermo Mendoza, Jr. (Mendoza) and Oniler Lontoc (Lontoc) of QTCI met with
respondent Thomas George (respondent), encouraging the latter to invest with QTCI.
On July 7, 1995, upon Mendoza's prodding, respondent finally invested with QTCI. On
the same day, Collado, in behalf of QTCI, and respondent signed the Customer's
Agreement. Forming part of the agreement was the Special Power of Attorney executed
by respondent, appointing Mendoza as his attorney-in-fact with full authority to trade
and manage his account.
On June 20, 1996, the Securities and Exchange Commission (SEC) issued a
Cease-and-Desist Order against QTCI. Alarmed by the issuance of the CDO,
respondent demanded from QTCI the return of his investment, but it was not heeded.
QTCI claimed that they were not aware of, nor were they privy to, any
arrangement which resulted in the account of respondent being handled by unlicensed
brokers. They pointed out that respondent transacted business with QTCI for almost
a year, without questioning the license or the authority of the traders handling his
account, rendering him estopped. It was only after it became apparent that QTCI
could no longer resume its business transactions by reason of the CDO that
respondent raised the alleged lack of authority of the brokers or traders handling his
account.

ISSUE:

Whether or not QTCI should be held liable for the loss incurred by George in the
investment he made with the corporation.

RULING:

YES.

It recognized Mendoza and Collado as its brokers. Petitioners did not object to,
and in fact recognized, Mendoza's appointment as respondent's attorney-in-fact.
Collado, in behalf of QTCI, concluded the Customer's Agreement despite the fact that
the appointed attorney-in-fact was not a licensed dealer. Worse, petitioners permitted
Mendoza to handle respondent's account.
Doctrine dictates that a corporation is invested by law with a personality
separate and distinct from those of the persons composing it, such that, save for
certain exceptions, corporate officers who entered into contracts in behalf of the
corporation cannot be held personally liable for the liabilities of the latter. Personal
liability of a corporate director, trustee, or officer, along (although not necessarily) with
the corporation, may validly attach, as a rule, only when - (1) he assents to a patently
unlawful act of the corporation, or when he is guilty of bad faith or gross negligence in
directing its affairs, or when there is a conflict of interest resulting in damages to the
corporation, its stockholders, or other persons; (2) he consents to the issuance of
watered down stocks or who, having knowledge thereof, does not forthwith file with
the corporate secretary his written objection thereto; (3) he agrees to hold himself
personally and solidarily liable with the corporation; or (4) he is made by a specific
provision of law personally answerable for his corporate action.
Romeo Lau, as president of [petitioner] QTCI, cannot feign innocence on the
existence of these unlawful activities within the company, especially so that Collado,
himself a ranking officer of QTCI, is involved in the unlawful execution of customers
orders. Lau, being the chief operating officer, cannot escape the fact that had he
exercised a modicum of care and discretion in supervising the operations of QTCI, he
could have detected and prevented the unlawful acts of Collado and Mendoza.

154 | P a g e
Law 321_Corporation LAW_ Case Digest

WENSHA SPA CENTER and/or XU ZHI JIE


vs.
LORETA YUNG
GR 185122, 16 August 2010

FACTS:

Loreta stated that she used to be employed by Manmen Services Co., Ltd. where
Xu was a client. Xu was apparently impressed by Loreta's performance. After he
established Wensha, he convinced Loreta to transfer and work at Wensha. Loreta was
initially reluctant to accept Xu's offer because her job at Manmen was stable and she
had been with Manmen for seven years. But Xu was persistent and offered her a
higher pay. Enticed, Loreta resigned from Manmen and transferred to Wensha as Xu's
personal assistant and interpreter. Loreta introduced positive changes to Wensha
which resulted in increased business. This pleased Xu so that she was promoted to
the position of Administrative Manager.
Wensha and Xu denied illegally terminating Loreta's employment. They claimed
that two months after Loreta was hired, they received various complaints against her
from the employees so that on August 10, 2004, they advised her to take a leave of
absence for one month while they conducted an investigation on the matter. Based on
the results of the investigation, they terminated Loreta's employment on August 31,
2004 for loss of trust and confidence.
The Court ruled that indeed Loreta was illegally dismissed because Wensha
failed to substantially prove its claim that she committed wrongdoings with Wensha‘s
employees, and that Loreta‘s testimony as to her termination because her feng shui
aura does not match that of Xu is consistent. Xu failed to duly prove a valid ground
for the loss of trust and confidence with Loreta. Question lies if Xu should be held
liable together with Wensha.

ISSUE:

Whether or not Xu is liable together with the corporation.

RULING:

NO.

Xu is not liable together with the corporation. Elementary is the rule that a
corporation is invested by law with a personality separate and distinct from those of
the persons composing it and from that of any other legal entity to which it may be
related. "Mere ownership by a single stockholder or by another corporation of all or
nearly all of the capital stock of a corporation is not of itself sufficient ground for
disregarding the separate corporate personality."
In labor cases, corporate directors and officers may be held solidarily liable with
the corporation for the termination of employment only if done with malice or in bad
faith.Bad faith does not connote bad judgment or negligence; it imports a dishonest
purpose or some moral obliquity and conscious doing of wrong; it means breach of a
known duty through some motive or interest or ill will; it partakes of the nature of
fraud.
There is no finding of bad faith or malice on the part of Xu. There is, therefore,
no justification for such a ruling. To sustain such a finding, there should be an
evidence on record that an officer or director acted maliciously or in bad faith in
terminating the services of an employee.Moreover, the finding or indication that the
dismissal was effected with malice or bad faith should be stated in the decision itself.

155 | P a g e
Law 321_Corporation LAW_ Case Digest

CEBU MACTAN MEMBERS CENTER


vs.
MASAHIRO TSUKAHARA
GR 159624, 17 July 2009

FACTS:

In February 1994, petitioner Cebu Mactan Members Center, Inc. (CMMCI),


through Mitsumasa Sugimoto (Sugimoto), the President and Chairman of the Board of
Directors of CMMCI, obtained a loan amounting to P6,500,000 from respondent
Masahiro Tsukahara. As payment for the loan, CMMCI issued seven postdated checks
of CMMCI payable to Tsukahara. On 13 April 1994, CMMCI, through Sugimoto,
obtained another loan amounting to P10,000,000 from Tsukahara. Sugimoto executed
and signed a promissory note in his capacity as CMMCI President and Chairman, as
well as in his personal capacity.
Upon maturity, the seven checks were presented for payment by Tsukahara,
but the same were dishonored by PNB, the drawee bank. After several failed attempts
to collect the loan amount totaling P16,500,000, Tsukahara filed the instant case for
collection of sum of money against CMMCI and Sugimoto.
Tsukahara alleged that the amount of P16,500,000 was used by CMMCI for the
improvement of its beach resort, which included the construction of a wave fence, the
purchase of air conditioners and curtains, and the provision of salaries of resort
employees. He also asserted that Sugimoto, as the President of CMMCI, "has the
power to borrow money for said corporation by any legal means whatsoever and to
sign, endorse and deliver all checks and promissory notes on behalf of the
corporation."
CMMCI, on the other hand, denied borrowing the amount from Tsukahara, and
claimed that both loans were personal loans of Sugimoto. The company also
contended that if the loans were those of CMMCI, the same should have been
supported by resolutions issued by CMMCI's Board of Directors.

ISSUE:

Whether or not CMMCI is liable to Tsukahara.

RULING:

YES.

It is because Sugimoto‘s actions are binding with CMMCI. A corporation, being


a juridical entity, may act through its board of directors which exercises almost all
corporate powers, lays down all corporate business policies, and is responsible for the
efficiency of management. The general rule is that, in the absence of authority from
the board of directors, no person, not even its officers, can validly bind a corporation.
In this case, the corporate by-laws of CMMCI explicitly empowered the President to
enter into loans with third persons on behalf of the corporation without the necessity
of a board resolution. By-laws of a corporation should be construed and given effect
according to the general rules governing the construction of contracts. They, as the
self-imposed private laws of a corporation, have, when valid, substantially the same
force and effect as laws of the corporation, as have the provisions of its charter insofar
as the corporation and the persons within it are concerned. They are in effect written
into the charter and in this sense; they become part of the fundamental law of the
corporation. And the corporation and its directors (or trustees) and officers are bound
by and must comply with them.
The corporation is now estopped from denying the authority of its president to
bind the former into contractual relations.

156 | P a g e
Law 321_Corporation LAW_ Case Digest

ARMANDO DAVID
vs.
NFLU, MARIVELES APPAREL CORPORATION LABOR UNION
GR 148263, 148271-72, 21 April 2009

FACTS:

MAC hired David as IMPEX and Treasury Manager on 16 September 1988.


David began serving as MAC's President in May 1990. David served as President in the
nature of a nominee as he did not own any of MAC's shares. David tendered his
irrevocable resignation from MAC on 30 September 1993. David's resignation was
made effective on 15 October 1993.
In a complaint for illegal dismissal dated 12 August 1993, National Federation
of Labor Unions (NAFLU) and Mariveles Apparel Corporation Labor Union (MACLU)
alleged that MAC ceased operations on 8 July 1993 without prior notice to its
employees. MAC allegedly gave notice of its closure on the same day that it ceased
operations. MACLU and NAFLU further alleged that, at the time of MAC's closure,
employees who had rendered one to two weeks work were not paid their corresponding
salaries.
Atty. Joshua Pastores, as MAC's counsel, submitted a position paper dated 21
February 1994 and argued that Carag and David should not be held liable because
MAC is owned by a consortium of banks. Carag's and David's ownership of MAC
shares only served to qualify them to serve as officers in MAC.

ISSUE:

Whether or not David may be held liable for the illegal dismissal of MAC
employees.

RULING:

NO.

It is improper to hold David liable for MAC's obligations to its employees.


However, Article 212(e) of the Labor Code, by itself, does not make a corporate officer
personally liable for the debts of the corporation because Section 31 of the Corporation
Code is still the governing law on personal liability of officers for the debts of the
corporation. Section 31 of the Corporation Code provides: Directors or trustees who
willfully and knowingly vote for or assent to patently unlawful acts of the corporation
or who are guilty of gross negligence or bad faith in directing the affairs of the
corporation or acquire any personal or pecuniary interest in conflict with their duty as
such directors, or trustees shall be liable jointly and severally for all damages resulting
therefrom suffered by the corporation, its stockholders or members and other persons.
There was no showing of David willingly and knowingly voting for or assenting to
patently unlawful acts of the corporation, or that David was guilty of gross negligence
or bad faith.
Also, the NLRC never gained jurisdiction over David for there was an invalid
service of summons.

157 | P a g e
Law 321_Corporation LAW_ Case Digest

HILARIO SORIANO AND ROSARIO ILAGAN


vs.
PEOPLE, BANGKO SENTRAL, PDIC
GR 159517-18, 30 June 2009

FACTS:

Hilario P. Soriano (Soriano) and Rosalinda Ilagan (Ilagan) were the President
and General Manager, respectively, of the Rural Bank of San Miguel (Bulacan), Inc.
(RBSM). Allegedly, on June 27, 1997 and August 21, 1997, during their incumbency
as president and manager of the bank, petitioners indirectly obtained loans from
RBSM. They falsified the loan applications and other bank records, and made it
appear that Virgilio J. Malang and Rogelio Mañaol obtained loans of P15,000,000.00
each, when in fact they did not.
Criminal charges were filed against them. They sought for its dismissal because
their action does not amount to any criminal action, and if it does, it will only render
them liable civilly. Also, their single act could not amount to multiple offenses.

ISSUE:

Whether or not the petitions may be held liable for their actions.

RULING:

YES.

They committed grave abuse of discretion in the exercise of their duties. As


aptly pointed out by the BSP in its memorandum, there are differences between the
two (2) offenses. A DOSRI violation consists in the failure to observe and comply with
procedural, reportorial or ceiling requirements prescribed by law in the grant of a loan
to a director, officer, stockholder and other related interests in the bank, i.e. lack of
written approval of the majority of the directors of the bank and failure to enter such
approval into corporate records and to transmit a copy thereof to the BSP supervising
department. The elements of abuse of confidence, deceit, fraud or false pretenses, and
damage, which are essential to the prosecution for estafa, are not elements of a DOSRI
violation. The filing of several charges against Soriano was, therefore, proper.

158 | P a g e
Law 321_Corporation LAW_ Case Digest

CEBU COUNTRY CLUB et al.


vs. RICARDO ELIZAGAQUE
GR 160273, 18 January 2008

FACTS:

Sometime in 1987, San Miguel Corporation, a special company proprietary


member of CCCI, designated respondent Ricardo F. Elizagaque, its Senior Vice
President and Operations Manager for the Visayas and Mindanao, as a special non-
proprietary member. The designation was thereafter approved by the CCCI‘s Board of
Directors.
In 1996, respondent filed with CCCI an application for proprietary membership.
As the price of a proprietary share was around the P5 million range, Benito Unchuan,
then president of CCCI, offered to sell respondent a share for only P3.5 million.
Respondent, however, purchased the share of a certain Dr. Butalid for only P3 million.
Consequently, on September 6, 1996, CCCI issued Proprietary Ownership Certificate
No. 1446 to respondent. On August 1, 1997, respondent received a letter from Julius
Z. Neri, CCCI‘s corporate secretary, informing him that the Board disapproved his
application for proprietary membership.
On August 6, 1997, Edmundo T. Misa, on behalf of respondent, wrote CCCI a
letter of reconsideration. As CCCI did not answer, respondent, on October 7, 1997,
wrote another letter of reconsideration. Still, CCCI kept silent. On November 5,
1997, respondent again sent CCCI a letter inquiring whether any member of the Board
objected to his application. Again, CCCI did not reply.

ISSUE:

Whether or not the Board of Directors of Cebu Country Club are liable to
Elizagawue for damages.

RULING:

YES.

There is bad faith among the members of the board. As shown by the records,
the Board adopted a secret balloting known as the ―black ball system‖ of voting
wherein each member will drop a ball in the ballot box. A white ball represents
conformity to the admission of an applicant, while a black ball means disapproval.
Pursuant to Section 3(c), as amended, cited above, a unanimous vote of the directors
is required. When respondent‘s application for proprietary membership was voted
upon during the Board meeting on July 30, 1997, the ballot box contained one (1)
black ball. Thus, for lack of unanimity, his application was disapproved.
Obviously, the CCCI Board of Directors, under its Articles of Incorporation, has
the right to approve or disapprove an application for proprietary membership. But
such right should not be exercised arbitrarily.
It is thus clear that respondent was left groping in the dark wondering why his
application was disapproved. He was not even informed that a unanimous vote of the
Board members was required. When he sent a letter for reconsideration and an
inquiry whether there was an objection to his application, petitioners apparently
ignored him. Certainly, respondent did not deserve this kind of treatment. Having
been designated by San Miguel Corporation as a special non-proprietary member of
CCCI, he should have been treated by petitioners with courtesy and civility. At the very
least, they should have informed him why his application was disapproved.
The exercise of a right, though legal by itself, must nonetheless be in
accordance with the proper norm. When the right is exercised arbitrarily, unjustly or
excessively and results in damage to another, a legal wrong is committed for which the
wrongdoer must be held responsible.

159 | P a g e
Law 321_Corporation LAW_ Case Digest

CALTEX (CHEVRON) PHILIPPINES, INC.


vs.
NLRC and ROMEO STO. TOMAS
GR 159641, 15 October 2007

FACTS:

In a letter dated October 21, 1996, Caltex informed the Department of Labor
and Employment (DOLE) of its plan to implement a redundancy program in its
Marketing Division and some departments in its Batangas Refinery for the period
starting October 1996 to December 1998. The letter alleged that the redundancy
program is a response to the market situation which constrained petitioner to
rationalize and simplify its business processes.
Santo Tomas was notified of his termination effective July 31, 1997 due to the
redundancy of his position and awarded him a separation package in the amount of
P559,458.90. On June 8, 1998, respondent filed with the Labor Arbiter a complaint for
illegal dismissal against petitioner and its President and Chief Executive Officer, Mr.
Clifton Hon. Private respondent alleged that: being petitioner‘s regular employee, he is
entitled to security of tenure; he did not commit any serious misconduct, willful
disobedience, gross and habitual neglect of duty or fraud and willful breach of trust to
warrant the penalty of dismissal from employment; there was no independent proof or
evidence presented by petitioner to substantiate its claim of redundancy nor was he
afforded due process as he was not given any opportunity to present his side; he was
dismissed due to his active participation in union activities; petitioner opened
positions for hiring some of which offered jobs that are the same as what private
respondent was performing; petitioner failed to give written notice to him and DOLE at
least one month before the intended date of termination as required by the Labor
Code.

ISSUE:

Whether or not Santo Tomas was illegally dismissed.

RULING:

YES.

Caltex failed to prove the necessity of the redundancy program. It is the rule
that the characterization of an employee‘s services as no longer necessary or
sustainable, and therefore, properly terminable, is an exercise of business judgment
on the part of the employer, and that the wisdom or soundness of such
characterization or decision is not subject to discretionary review. However, such
characterization may be rejected if the same is found to be in violation of law or is
arbitrary or malicious.
In the instant case, there was no substantial evidence presented by petitioner to
justify private respondent's dismissal due to redundancy. As correctly found by the
CA, petitioner‘s evidence to show redundancy merely consisted of a copy of petitioner‘s
letter to the DOLE informing the latter of its intention to implement a redundancy
program and nothing more. The letter which merely stated that petitioner undertook a
review, restructuring and streamlining of its organization which resulted in
consolidation, abolition and outsourcing of certain functions; and which resulted in
identified and redundant positions instead of simplifying its business process
restructuring, does not satisfy the requirement of substantial evidence, that is, the
amount of evidence which a reasonable mind might accept as adequate to justify a
conclusion.
Petitioner failed to demonstrate the superfluity of private respondent‘s position
as there was nothing in the records that would establish any concrete and real factors
recognized by law and relevant jurisprudence, such as overhiring of workers,
decreased volume of business, or dropping of a particular product line or service
activity previously manufactured or undertaken by the enterprise, which were adopted
by petitioner in implementing the redundancy program.

160 | P a g e
Law 321_Corporation LAW_ Case Digest

ATRIUM MANAGEMENT CORPORATION


vs.
COURT OF APPEALS, E.T. HENRY AND CO., et al.
GR 109491, 28 February 2001

FACTS:

Hi-Cement Corporation through its corporate signatories, petitioner Lourdes M.


de Leon, treasurer, and the late Antonio de las Alas, Chairman, issued checks in favor
of E.T. Henry and Co. Inc., as payee. E.T. Henry and Co., Inc., in turn, endorsed the
four checks to petitioner Atrium Management Corporation for valuable consideration.
Upon presentment for payment, the drawee bank dishonored all four checks for the
common reason "payment stopped". Atrium, thus, instituted an action after its
demand for payment of the value of the checks was denied.
At the trial, Atrium presented as its witness Carlos C. Syquia who testified that
in February 1981, Enrique Tan of E.T. Henry approached Atrium for financial
assistance, offering to discount four RCBC checks in the total amount of P2 million,
issued by Hi-Cement in favor of E.T. Henry. Atrium agreed to discount the checks,
provided it be allowed to confirm with Hi-Cement the fact that the checks represented
payment for petroleum products which E.T. Henry delivered to Hi-Cement. Carlos C.
Syquia identified two letters, dated February 6, 1981 and February 9, 1981 issued by
Hi-Cement through Lourdes M. de Leon, as treasurer, confirming the issuance of the
four checks in favor of E.T. Henry in payment for petroleum products.
Lourdes M. de Leon claimed she is not solidarilly liable with Hi-Cement for the
amount of the check and that Atrium was an ordinary holder, not a holder in due
course of the rediscounted checks.

ISSUE:

Whether or not de Leon may be held liable.

RULING:

YES.

Due to negligence. Lourdes M. de Leon and Antonio de las Alas as treasurer and
Chairman of Hi-Cement were authorized to issue the checks. However, Ms. de Leon
was negligent when she signed the confirmation letter requested by Mr. Yap of Atrium
and Mr. Henry of E.T. Henry for the rediscounting of the crossed checks issued in
favor of E.T. Henry. She was aware that the checks were strictly endorsed for deposit
only to the payee's account and not to be further negotiated. What is more, the
confirmation letter contained a clause that was not true, that is, "that the checks
issued to E.T. Henry were in payment of Hydro oil bought by Hi-Cement from E.T.
Henry". Her negligence resulted in damage to the corporation. Hence, Ms. de Leon may
be held personally liable therefor.
"Personal liability of a corporate director, trustee or officer along (although not
necessarily) with the corporation may so validly attach, as a rule, only when: He
assents (a) to a patently unlawful act of the corporation, or (b) for bad faith or gross
negligence in directing its affairs, or (c) for conflict of interest, resulting in damages to
the corporation, its stockholders or other persons; He consents to the issuance of
watered down stocks or who, having knowledge thereof, does not forthwith file with
the corporate secretary his written objection thereto; He agrees to hold himself
personally and solidarily liable with the corporation; or He is made, by a specific
provision of law, to personally answer for his corporate action.
However, as to the claim of Atrium, it cannot be upheld because it is not a
holder of the check in due course due to the fact that the same was crossed in favor of
E.T. Henry, and therefore only payable to the latter‘s account.

161 | P a g e
Law 321_Corporation LAW_ Case Digest

ARB CONSTRUCTION and MARK MOLINA


vs.
COURT OF APPEALS, TBS SECURITY AND INVESTIGATION
GR 126554, 31 May 2000

FACTS:

On 15 August 1993 TBS Security and Investigation Agency (TBSS) entered into
two (2) Service Contracts with ARBC wherein TBSS agreed to provide and post security
guards in the five (5) establishments being maintained by ARBC. The contract shall be
effective for one (1) year and shall be considered renewed for the same period unless
the same is terminated after a notice is given to the parties thirty (30) days in advance.
In a letter dated 23 February 1994 ARBC informed TBSS of its desire to
terminate the Service Contracts effective thirty (30) days after receipt of the letter.
Also, in a letter dated 22 March 1994, ARBC through its Vice President for Operations,
Mark Molina, informed TBSS that it was replacing its security guards with those of
Global Security Investigation Agency (GSIA).
In response to both letters, TBSS informed ARBC that the latter could not
preterminate the Service Contracts nor could it post security guards from GSIA as it
would run counter to the provisions of their service contracts. Nevertheless, Molina
decreased the security guards to only one (1) as a right provided under the service
contract. TBSS thereafter filed a case for breach of contract against ARBC and Mark
Molina.

ISSUE:

Whether or not Mark Molina should be held liable together with ARBC.

RULING:

NO.

He merely acted within his capacity as an officer of the corporation. It is basic


that a corporation is invested by law with a personality separate and distinct from
those of the persons composing it as well as from that of any other legal entity to
which it may be related. As a general rule, a corporation may not be made to answer
for acts or liabilities of its stockholders or those of the legal entities to which it may be
connected and vice versa. However, the veil of corporate fiction may be pierced when it
is used as a shield to further an end subversive of justice; or for purposes that could
not have been intended by the law that created it; or to defeat public convenience,
justify wrong, protect fraud, or defend crime; or to perpetuate deception; or as an alter
ego, adjunct or business conduit for the sole benefit of the stockholders.
On the basis hereof, petitioner Molina could not be held jointly and severally
liable for any obligation which petitioner ARBC may be held accountable for, absent
any proof of bad faith or malice on his part. Corollarily, it is also incorrect on the part
of the Court of Appeals to conclude that there was a sufficient cause of action against
Molina as to make him personally liable for his actuations as Vice President for
Operations of ARBC.

162 | P a g e
Law 321_Corporation LAW_ Case Digest

RUFINA LUY LIM


vs.
COURT OF APPEALS, AUTO TRUCK TBA CORP., et al.
GR 124715, 24 January 2000

FACTS:

On 11 June 1994, Pastor Y. Lim died intestate. Herein petitioner, as surviving


spouse and duly represented by her nephew George Luy, filed on 17 March 1995, a
joint petition for the administration of the estate of Pastor Y. Lim before the Regional
Trial Court of Quezon City.
Private respondent corporations, whose properties were included in the
inventory of the estate of Pastor Y. Lim, then filed a motion6 for the lifting of lis
pendens and motion for exclusion of certain properties from the estate of the decedent.
Although the defendant corporations dealt and engaged in business with the
public as corporations, all their capital, assets and equity were however, personally
owned by the late Pastor Y Lim. Hence the alleged stockholders and officers appearing
in the respective articles of incorporation of the above business entities were mere
dummies of Pastor Y. Lim, and they were listed therein only for purposes of
registration with the Securities and Exchange Commission.
Petitioner argues that the parcels of land covered under the Torrens system and
registered in the name of private respondent corporations should be included in the
inventory of the estate of the decedent Pastor Y. Lim, alleging that after all the
determination by the probate court of whether these properties should be included or
not is merely provisional in nature, thus, not conclusive and subject to a final
determination in a separate action brought for the purpose of adjudging once and for
all the issue of title.

ISSUE:

Whether or not the properties of the corporation-defendants be included in the


estate of the deceased.

RULING:

NO.

They hold separate personalities from the deceased. In as much as the real
properties included in the inventory of the estate of the late Pastor Y. Lim are in the
possession of and are registered in the name of private respondent corporations,
which under the law possess a personality separate and distinct from their
stockholders, and in the absence of any cogency to shred the veil of corporate fiction,
the presumption of conclusiveness of said titles in favor of private respondents should
stand undisturbed.
Notwithstanding that the real properties were duly registered under the Torrens
system in the name of private respondents, and as such were to be afforded the
presumptive conclusiveness of title, the probate court obviously opted to shut its eyes
to this gleamy fact and still proceeded to issue the impugned orders.
Mere ownership by a single stockholder or by another corporation of all or
nearly all of the capital stock of a corporation is not of itself a sufficient reason for
disregarding the fiction of separate corporate personalities.

163 | P a g e
Law 321_Corporation LAW_ Case Digest

ADALIA FRANCISCO and MERRYLAND DEVELOPMENT


vs. RITA MEJIA
GR 141617, 14 August 2001

FACTS:

On 21 December 1964, Gutierrez and Cardale Financing and Realty


Corporation (Cardale) executed a Deed of Sale with Mortgage for the consideration of
P800,000.00 to be paid in several installments within five years from the date of the
deed, at an interest of nine percent per annum ―based on the successive unpaid
principal balances.‖ Thereafter, the titles of Gutierrez were cancelled new ones were
issued in favor of Cardale.
On 26 August 1968, owing to Cardale‘s failure to settle its mortgage obligation,
Gutierrez filed a complaint for rescission of the contract. During the pendency of the
rescission case, Gutierrez died and was substituted by her executrix, respondent Rita
C. Mejia (Mejia). However, Cardale, which was represented by petitioner Adalia B.
Francisco (Francisco) in her capacity as Vice-President and Treasurer of Cardale, lost
interest in proceeding with the presentation of its evidence and the case lapsed into
inactive status for a period of about fourteen years.
In the meantime, the mortgaged parcels of land became delinquent in the
payment of real estate taxes, which culminated in their levy and auction sale in
satisfaction of the tax arrears. The highest bidder for the three parcels of land was
petitioner Merryland Development Corporation, whose President and majority
stockholder is Francisco.
Mejia filed for damages against Francisco who controlled Cardale and
Merryland and that she had employed fraud by intentionally causing Cardale to
default in its payment of real property taxes on the mortgaged properties so that
Merryland could purchase the same by means of a tax delinquency sale.

ISSUE:

Whether or not Francisco is liable for damages.

RULING:

YES.

The totality of the circumstances appertaining conduce to the inevitable


conclusion that petitioner Francisco acted in bad faith. The events leading up to the
loss by the Gutierrez estate of its mortgage security attest to this. It has been
established that Cardale failed to comply with its obligation to pay the balance of the
purchase price for the four parcels of land it bought from Gutierrez. This prompted
Gutierrez to file an action for rescission of the Deed of Sale with Mortgage, but the
case dragged on for about fourteen years when Cardale, as represented by Francisco,
who was Vice-President and Treasurer of the same, lost interest in completing its
presentation of evidence
That Merryland acquired the property at the public auction only serves to shed
more light upon Francisco‘s fraudulent purposes. Based on the findings of the Court
of Appeals, Francisco is the controlling stockholder and President of Merryland. Thus,
aside from the instrumental role she played as an officer of Cardale, in evading that
corporation‘s legitimate obligations to Gutierrez, it appears that Francisco‘s actions
were also oriented towards securing advantages for another corporation in which she
had a substantial interest.
Under the doctrine of piercing the veil of corporate entity, when valid grounds
therefore exist, the legal fiction that a corporation is an entity with a juridical
personality separate and distinct from its members or stockholders may be
disregarded. In such cases, the corporation will be considered as a mere association of
persons. The members or stockholders of the corporation will be considered as the
corporation, that is, liability will attach directly to the officers and stockholders.

164 | P a g e
Law 321_Corporation LAW_ Case Digest

DEVELOPMENT BANK OF THE PHILIPPINES


vs.
COURT OF APPEALS, REMINGTON INDUSTRIAL SALES
GR 126200, 16 August 2001

FACTS:

Between July 1981 and April 1984, Marinduque Mining entered into 3
mortgage agreements with PNB and DBP involving its real properties located in
Surigao del Norte, Negros Occidental, and Rizal, as well as its equipments located
therein. Marinduque failed to pay its loans, causing the foreclosure of the said
mortgages. PNB and DBP thereafter gained control of the said properties.
In the meantime, between July 16, 1982 to October 4, 1983, Marinduque
Mining purchased and caused to be delivered construction materials and other
merchandise from Remington Industrial Sales Corporation. The purchases remained
unpaid as of August 1, 1984 when Remington filed a complaint for a sum of money
and damages against Marinduque Mining for the value of the unpaid construction
materials and other merchandise purchased by Marinduque Mining, as well as
interest, attorney‘s fees and the costs of suit.
Remington‘s original complaint was amended to include PNB, DBP, Maricalum
Mining Corporation and Island Cement Corporation as co-defendants. Remington
asserted that Marinduque Mining, PNB, DBP, Nonoc Mining, Maricalum Mining and
Island Cement must be treated in law as one and the same entity by disregarding the
veil of corporate fiction since the personnel, key officers and rank-and-file workers and
employees of co-defendants NMIC, Maricalum and Island Cement creations of co-
defendants PNB and DBP were the personnel of co-defendant MMIC such that
practically there has only been a change of name for all legal purpose and intents.

ISSUE:

Whether or not the take over of PNB and DBP over Marinduque Mining is in bad
faith.

RULING:

NO.

Their actions are mandated under the law. Where the corporations have
directors and officers in common, there may be circumstances under which their
interest as officers in one company may disqualify them in equity from representing
both corporations in transactions between the two. Thus, where one corporation was
‗insolvent and indebted to another, it has been held that the directors of the creditor
corporation were disqualified, by reason of self-interest, from acting as directors of the
debtor corporation in the authorization of a mortgage or deed of trust to the former to
secure such indebtedness In the same manner that when the corporation is insolvent,
its directors who are its creditors cannot secure to themselves any advantage or
preference over other creditors. They cannot thus take advantage of their fiduciary
relation and deal directly with themselves, to the injury of others in equal right.
Directors of insolvent corporation, who are creditors of the company, can not
secure to themselves any preference or advantage over other creditors in the payment
of their claims. It is not good morals or good law. The governing body of officers
thereof are charged with the duty of conducting its affairs strictly in the interest of its
existing creditors, and it would be a breach of such trust for them to undertake to give
any one of its members any advantage over any other creditors in securing the
payment of his debts in preference to all others. When validity of these mortgages, to
secure debts upon which the directors were indorsers, was questioned by other
creditors of the corporation, they should have been classed as instruments rendered
void by the legal principle which prevents directors of an insolvent corporation from
giving themselves a preference over outside creditors.

165 | P a g e
Law 321_Corporation LAW_ Case Digest

AMERICAN HOSPITAL SUPPLIES/PHILIPPINES et al.


vs.
COURT OF APPEALS, ALFONSO BAYANI
GR 111807, 14 June 1996

FACTS:

American Hospital Supplies was engaged in the sale and manufacture of


medicines and pharmaceuticals in the country and did substantial business with
government hospitals. On 1 June 1970 it hired Alfonso Bayani as an Area Manager for
Visayas and Mindanao, and later appointed him Manager of its Cebu branch. On 30
January 1978 private respondent was dismissed from the service. At that time he was
receiving a monthly compensation of P3,180.00.
On 5 May 1978 private respondent filed a complaint for damages before the
trial court alleging that in the course of their business petitioners were directly
encouraging, abetting and promoting bribery in the guise of "commissions,"
"entertainment expenses" and "representation expenses" which were given to various
government hospital officials in exchange for favorable recommendations, approvals
and actual purchases of medicines and pharmaceuticals. For his refusal to take direct
and personal hand in giving "bribe money" he was dismissed. He then implicated AHS
President Gervacio Amistoso and Vice President Constancio Halili as responsible for
his illegal dismissal.

ISSUE:

Whether or not Amistoso and Halili be held solidarily liable with the
corporation.

RULING:

NO.

Corporate officers are not personally liable for money claims of discharged
corporate employees unless they acted with evident malice and bad faith in
terminating their employment. In the case at bar, while petitioners Amistoso and
Halili may have had a hand in the relief of respondent. Bayani, there are no
indications of malice and bad faith on their part. We take exception to the conclusion
of respondent Court of Appeals that "the manner by which Halili and Amistoso acted is
characterized by bad faith and malice, thus binding them personally liable to plaintiff-
appellee,'' On the contrary it is apparent that the relief order was a business judgment
on the part of the officers, with the best interest of the corporation in mind, based on
their opinion that respondent Bayani had failed to perform the duties expected of him.
Hence both the trial court and respondent Court of Appeals committed a reversible
error in holding petitioners Amistoso and Halili jointly and solidarily liable with
Petitioner Corporation.

166 | P a g e
Law 321_Corporation LAW_ Case Digest

COMPLEX ELECTRONICS EMPLOYEES ASSOCIATION


vs.
NLRC, et al.
GR 121315, 19 July 1999

FACTS:

Complex informed its Lite-On personnel that a request from Lite On Philippines
to lower their selling price by 10% was not feasible as they were already incurring
losses at the present prices of their products. Under such circumstances, Complex
regretfully informed the employees that it was left with no alternative but to close
down the operations of the Lite-On Line. The Union, however, decried the decision and
voted to declare a strike. Labor unrest within the company eventually ensued.
In the evening of April 6, 1992, the machinery, equipment and materials being
used for production at Complex were pulled-out from the company premises and
transferred to the premises of Ionics Circuit, Inc. at Cabuyao, Laguna. The following
day, a total closure of company operation was effected at Complex.
A complaint was, thereafter, filed with the Labor Arbitration Branch of the
NLRC for unfair labor practice, illegal closure/illegal lockout, money claims for
vacation leave, sick leave, unpaid wages, 13th month pay, damages and attorney's
fees. Ionics was impleaded as a party defendant because the officers and management
personnel of Complex were also holding office at Ionics with Lawrence Qua as the
President of both companies.
Ionics contended that it was an entity separate and distinct from Complex and
had been in existence since July 5, 1984 or eight (8) years before the labor dispute
arose at Complex. Like Complex, it was also engaged in the semi-conductor business
where the machinery, equipment and materials were consigned to them by their
customers. While admitting that Lawrence Qua, the President of Complex was also
the President of Ionics, the latter denied having Qua as their owner since he had no
recorded subscription of P1,200,000.00 in Ionics as claimed by the Union.

ISSUE:

Whether or not Lawrence Qua should be held liable for the alleged illegal
transfer of machineries of Complex to Ionics.

RULING:

NO.

It is settled that in the absence of malice or bad faith, a stockholder or an


officer of a corporation cannot be made personally liable for corporate liabilities. The
fact that the pull-out of the machinery, equipment and materials was effected during
nighttime is not per se an indicia of bad faith on the part of respondent Qua since he
had no other recourse, and the same was dictated by the prevailing mood of unrest as
the laborers were already vandalizing the equipment, bent on picketing the company
premises and threats to lock out the company officers were being made. Such acts of
respondent Qua were, in fact, made pursuant to the demands of Complex's customers
who were already alarmed by the pending labor dispute and imminent strike to be
stage by the laborers, to have their equipment, machinery and materials pull out of
Complex. As such, these acts were merely done pursuant to his official functions and
were not, in any way, made with evident bad faith.
As to the juridical personality of the corporations, Ionics may be engaged in the
same business as that of Complex, but this fact alone is not enough reason to pierce
the veil of corporate fiction of the corporation. Well-settled is the rule that a
corporation has a personality separate and distinct from that of its officers and
stockholders. Likewise, mere ownership by a single stockholder or by another
corporation of all or nearly all of the capital stock of a corporation is not of itself
sufficient ground for disregarding the separate corporate personality.

167 | P a g e
Law 321_Corporation LAW_ Case Digest

ERNESTINA CRISOLOGO-JOSE
vs.
COURT OF APPEALS, RICARDO SANTOS, JR.
GR 80599, 15 September, 1989

FACTS:

In 1980, Ricardo S. Santos, Jr. was the vice-president of Mover Enterprises,


Inc. in-charge of marketing and sales; and the president of the said corporation was
Atty. Oscar Z. Benares. On April 30, 1980, Atty. Benares, in accommodation of his
clients, the spouses Jaime and Clarita Ong, a check drawn against Traders Royal
Bank, dated June 14, 1980, in the amount of P45,000.00 payable to Ernestina
Crisologo-Jose. Since the check was under the account of Mover Enterprises, Inc., the
same was to be signed by its president, Atty. Oscar Z. Benares, and the treasurer of
the said corporation. However, since at that time, the treasurer of Mover Enterprises
was not available, Atty. Benares prevailed upon Santos, Jr., to sign the aforesaid
check as an alternate signatory, who did sign the same.
It appears that the check to Crisologo-Jose in consideration of the waiver or
quitclaim by said defendant over a certain property which the Government Service
Insurance System (GSIS) agreed to sell to the clients of Atty. Oscar Benares, the
spouses Jaime and Clarita Ong, with the understanding that upon approval by the
GSIS of the compromise agreement with the spouses Ong, the check will be encashed
accordingly. However, since the compromise agreement was not approved within the
expected period of time, the aforesaid check was replaced by Atty. Benares with
another Traders Royal Bank check dated August 10, 1980, in the same amount. This
replacement check was also signed by Atty. Benares and by Santos, Jr. When Jose
deposited this replacement check with her account, it was dishonored for insufficiency
of funds. A subsequent redepositing of the said check was likewise dishonored by the
bank for the same reason.

ISSUE:

Whether or not Movers Enterprises should be held liable to the bounced checks
which are personal liabilities of Atty. Bañares.

RULING:

NO.

The provision of the Negotiable Instruments Law which holds an


accommodation party liable on the instrument to a holder for value, although such
holder at the time of taking the instrument knew him to be only an accommodation
party, does not include nor apply to corporations which are accommodation parties.
This is because the issue or indorsement of negotiable paper by a corporation without
consideration and for the accommodation of another is ultra vires. Hence, one who
has taken the instrument with knowledge of the accommodation nature thereof cannot
recover against a corporation where it is only an accommodation party.
By way of exception, an officer or agent of a corporation shall have the power to
execute or indorse a negotiable paper in the name of the corporation for the
accommodation of a third person only if specifically authorized to do so.. Since such
accommodation paper cannot thus be enforced against the corporation, especially
since it is not involved in any aspect of the corporate business or operations, the
inescapable conclusion in law and in logic is that the signatories thereof shall be
personally liable therefor, as well as the consequences arising from their acts in
connection therewith.
Instead, Jose should direct her claim against Bañares and Santos. Santos,
however, is exculpated from criminal liability under BP 22 for he successfully and
legally consigned the amount of the check with the Court within the reglamentary
period.

168 | P a g e
Law 321_Corporation LAW_ Case Digest

FCY CONSTRUCTION GROUP and FRANCIS YU


vs.
COURT OF APPEALS, HON. JOSE DE LA RAMA
GR 123358, 01 February 2000

FACTS:

On June 29, 1993, Ley Construction and Development Corporation filed a


complaint for collection of a sum of money with application for preliminary attachment
against petitioner FCY Construction Group, Inc. and Francis C. Yu. Ley alleged that it
had a joint venture agreement with petitioner FCY Construction Group, Inc. (wherein
petitioner Francis C. Yu served as President) over the Tandang Sora Commonwealth
Flyover government project for which it had provided funds and construction
materials. The Complaint was filed in order to compel petitioners to pay its half share
in the collections received in the project as well as those yet to be received therein. In
support of its application for a writ of attachment, private respondent alleged that
petitioners were guilty of fraud in incurring the obligation and had fraudulently
misapplied or converted the money paid them, to which it had an equal share.
FCY denied the allegation, and also moved for the dropping of Francis Yu as one
of the defendants, claiming that the hornbook law that corporate personality is a
shield against personal liability of its officers.

ISSUE:

Whether or not Francis Yu may be held solidarily liable with FCY.

RULING:

NO.

FCY has a separate juridical entity from that of Francis. Francis Yu cannot be
made liable in his individual capacity if he indeed entered into and signed the contract
in his official capacity as President, in the absence of stipulation to that effect, due to
the personality of the corporation being separate and distinct from the persons
composing it. However, while Yu cannot be held solidarily liable with petitioner
corporation merely because he is the President thereof and was involved in the
transactions with private corporation, there exists instances when corporate officers
may be held personally liable for corporate acts.
Personal liability of a corporate director, trustee or officer along (although not
necessarily) with the corporation may so validly attach, as a rule. The attendance of
these circumstances, however, cannot be determined at this stage and should properly
be threshed out during the trial on the merits. Also, there was no fraud on the part of
FCY in the performance of its obligations with Ley, therefore rendering attachment as
improper.

169 | P a g e
Law 321_Corporation LAW_ Case Digest

RICARDO LLAMADO
vs.
COURT OF APPEALS, PEOPLE OF THE PHILIPPINES
GR 99032, 26 March 1997

FACTS:

Private complainant, Leon Gaw, delivered to accused the amount of


P180,000.00, with the assurance of Aida Tan, the secretary of the accused in the
corporation, that it will be repaid on 4 November 1983. Upon delivery of the money,
accused Ricardo Llamado took it and placed it inside a deposit box. Accused Jacinto
Pascual and Ricardo Llamado signed Philippine Trust Company Check No. 047809,
postdated 4 November 1983, in the amount of P186,500.00 in the presence of private
complainant.
The aforesaid check was issued in payment of the cash money delivered to the
accused by private complainant, plus interests thereon for sixty (60) days in the
amount of P6,500.00.
On 4 November 1983, private complainant deposited the check in his current
account with the Equitable Banking Corporation which later informed the
complainant that said check was dishonored by the drawee bank because payment
was stopped, and that the check was drawn against insufficient funds. Private
complainant was also notified by the Equitable Banking Corporation that his current
account was debited for the amount of P186,500.00 because of the dishonor of the
said check.
Private complainant returned to Aida Tan to inform her of the dishonor of the
check. Aida Tan received the check from private complainant with the assurance that
she will have said check changed with cash. However, upon his return to Aida Tan,
the latter informed him that she had nothing to do with the check.
Llamado alleges that he should not be held personally liable for the amount of
the check because it was a check of the Pan Asia Finance Corporation and he signed
the same in his capacity as Treasurer of the corporation.

ISSUE:

Whether or not Llamado should be held liable under BP 22.

RULING:

YES.

He is mere act of signing the check held him liable under BP 22. Where the
check is drawn by a corporation, company or entity, the person or persons who
actually signed the check in behalf of such drawer shall be liable under this Act.

170 | P a g e
Law 321_Corporation LAW_ Case Digest

MAM REALTY CORPORATION


vs.
NLRC, CELSO BALBASTRO
GR 114787, 02 June 1995

FACTS:

Celso B. Balbastro filed a case against MAM Realty Development Corporation


("MAM") and its Vice President Manuel P. Centeno, for unfair labor practice in violation
of the Labor Code. Balbastro alleged that he was employed by MAM as a pump
operator in 1982 and had since performed such work at its Rancho Estate, Marikina,
Metro manila.
MAM countered that Balbastro had previously been employed by Francisco
Cacho and co., Inc., the developer of Rancho Estates. Sometime in May 1982, his
services were contracted by MAM for the operation of the Rancho Estates' water pump.
He was engaged, however, not as an employee, but as a service contractor, at an
agreed fee of P1,590.00 a month. Similar arrangements were likewise entered into by
MAM with one Rodolfo Mercado and with a security guard of Rancho Estates III
Homeowners' Association. Under the agreement, Balbastro was merely made to open
and close on a daily basis the water supply system of the different phases of the
subdivision in accordance with its water rationing scheme. He worked for only a
maximum period of three hours a day, and he made use of his free time by offering
plumbing services to the residents of the subdivision. He was not at all subject to the
control or supervision of MAM for, in fact, his work could so also be done either by
Mercado or by the security guard. On 23 May 1990, prior to the filing of the complaint,
MAM executed a Deed of Transfer, 1 effective 01 July 1990, in favor of the Rancho
Estates Phase III Homeowners Association, Inc., conveying to the latter all its rights
and interests over the water system in the subdivision.
NLRC found the corporation guilty as charged, and likewise held Centeno liable
together with said corporation.

ISSUE:

Whether or not Centeno should be held liable together with MAM Realty.

RULING:

NO.

A corporation, being a juridical entity, may act only through its directors,
officers and employees. Obligations incurred by them, acting as such corporate
agents, are not theirs but the direct accountabilities of the corporation they represent.
True, solidarily liabilities may at times be incurred but only when exceptional
circumstances. In labor cases, for instance, the Court has held corporate directors and
officers solidarily liable with the corporation for the termination of employment of
employees done with malice or in bad faith.
In the case at bench, there is nothing substantial on record that can justify,
prescinding from the foregoing, petitioner Centeno's solidary liability with the
corporation. Nothing states that he acted in bad faith.
Although the Court found that there is an employer-employee relationship
between Balbastro and MAM Realty, the case was remanded to NLRC for the
recomputation of Balbastro‘s monetary awards, such as backwages and wage
differentials.

171 | P a g e
Law 321_Corporation LAW_ Case Digest

SERGIO NAGUIAT
vs.
CLARK FIELD TAXI, INC.
GR 116123, 13 March 1997

FACTS:

Petitioner CFTI held a concessionaire's contract with the Army Air Force
Exchange Services ("AAFES") for the operation of taxi services within Clark Air Base.
Sergio F. Naguiat was CFTI's president, while Antolin T. Naguiat was its vice-
president. Like Sergio F. Naguiat Enterprises, Incorporated ("Naguiat Enterprises"), a
trading firm, it was a family-owned corporation.
Individual respondents were previously employed by CFTI as taxicab drivers.
Due to the phase-out of the US military bases in the Philippines, from which Clark Air
Base was not spared, the AAFES was dissolved, and the services of individual
respondents were officially terminated on November 26, 1991.
The AAFES Taxi Drivers Association ("drivers' union"), through its local
president, Eduardo Castillo, and CFTI held negotiations as regards separation benefits
that should be awarded in favor of the drivers. They arrived at an agreement that the
separated drivers will be given P500.00 for every year of service as severance pay. Most
of the drivers accepted said amount in December 1991 and January 1992. However,
individual respondents herein refused to accept theirs.
Instead, after disaffiliating themselves from the drivers' union and filed a
complaint against "Sergio F. Naguiat doing business under the name and style Sergio
F. Naguiat Enterprises, Inc., and CFTI with Antolin T. Naguiat as vice president and
general manager, as party respondent.

ISSUE:

Whether or not Sergio Naguiat may be held liable for the claims instituted by
the taxi drivers against his company.

RULING:

YES.

As provided for under the fifth paragraph of Section 100 of the Corporation
Code specifically imposes personal liability upon the stockholder actively managing or
operating the business and affairs of the close corporation.
In fact, in posting the surety bond required by this Court for the issuance of a
temporary restraining order enjoining the execution of the assailed NLRC Resolutions,
only Sergio F. Naguiat, in his individual and personal capacity, principally bound
himself to comply with the obligation thereunder, i.e., "to guarantee the payment to
private respondents of any damages which they may incur by reason of the issuance
of a temporary restraining order sought, if it should be finally adjudged that said
principals were not entitled thereto.
The Court here finds no application to the rule that a corporate officer cannot
be held solidarily liable with a corporation in the absence of evidence that he had
acted in bad faith or with malice. In the present case, Sergio Naguiat is held solidarily
liable for corporate tort because he had actively engaged in the management and
operation of CFTI, a close corporation.
Antolin Naguiat, however, could not be held liable. Although he carried the title
of "general manager" as well, it had not been shown that he had acted in such
capacity. Furthermore, no evidence on the extent of his participation in the
management or operation of the business was preferred. In this light, he cannot be
held solidarily liable for the obligations of CFTI and Sergio Naguiat to the private
respondents.

172 | P a g e
Law 321_Corporation LAW_ Case Digest

PROGRESS HOMES and ERMELO ALMEDA


vs.
NLRC, et al.
GR 106212, 07 March 1997

FACTS:

Private respondents allegedly were among the workers employed by Progress


Homes in their construction and development of the subdivision from 1986 to 1988.
They were paid varying salaries.
Forty of these workers, including private respondents, filed before the NLRC
Arbitration Branch a petition for reinstatement, salary adjustment, ECOLA, overtime
pay and 13th month pay. Petitioners amicably settled the case with thirty-three of the
laborers, leaving private respondents as the only claimants. Private respondents
alleged that they worked as laborers and carpenters for 8.5 hours a day at a salary
below the minimum wage and that when they demanded payment of the benefits due
them, they were summarily dismissed and barred from entering the workplace. It also
denied that private respondents were regular employees claiming that they were only
project employees and that there was no employer-employee relationship between
them.
The Labor Arbiter and the NLRC ruled that Progress is liable to the
respondents, with Almeda jointly and severally liable.

ISSUE:

Whether or not Almeda may be held liable.

RULING:

NO.

It amounted to grave abuse of discretion. The Court has held that corporate
directors and officers are solidarily liable with the corporation for the termination of
employment of employees only if the termination is done with malice or in bad faith.
The Labor Arbiter's decision failed to disclose why Almeda was made personally
liable. There appears no evidence on record that he acted maliciously or in bad faith in
terminating the services of private respondents. Almeda, therefore, should not have
been made personally answerable for the payment of private respondents' salaries.
The decision of the Labor Arbiter and the NLRC, however, should be set aside
because of denial of due process on the part of Progress Homes.

173 | P a g e
Law 321_Corporation LAW_ Case Digest

REAHS CORP., SEVERO CASTULO, et al.


vs.
NLRC, BONIFACIO RED, et al.
GR 117473, 15 April 1997

FACTS:

Private respondents sued Reahs Corp. for unfair labor practice and illegal
dismissal. They claim that they were unlawfully dismissed and were not awarded nor
given any separation pay.
On the other hand, respondents allege that sometime in 1986, a certain Ms
Soledad Domingo, the sole proprietress and operator of Rainbow Sauna located at 316
Araneta Avenue, Quezon City, offered to sell her business to respondent Reah's
Corporation After the sale, all the assets of Ms Domingo were turned over to
respondent Reah's, which put a sing-along coffee shop and massage clinic; that
complainant Red started his employment on the first week of December 1988 as a
room boy at P50.00/day and was given living quarters inside the premises as he
requested; that sometime in March 1989, complainant Red asked permission to go to
Bicol for a period of ten (10) days, which was granted, and was given an advance
money of P1,200.00 to bring some girls from the province to work as attendants at the
respondent's massage clinic, that it was only on January 1, 1990 that complainant
Red returned and was re-hired under the same terms and conditions of his previous
employment with the understanding that he will have to refund the P1,200.00 cash
advance given to him; that due to poor business, increase in the rental cost and the
failure of Meralco to reconnect the electrical services in the establishment, it suffered
losses leading to its closure.
The NLRC ruled in favor of respondents. Together with the corporation, the
NLRC also held Castulo, Romeo Pascua, and Daniel Valenzuela solidarily liable due to
their capacity as Chairman, Board Member and Accountant, and Acting Manager,
respectively.

ISSUE:

Whether or not Pascua, Castulo, and Valenzuela, may be held liable.

RULING:

YES.

They acted in bad faith in dismissing the respondents. As a general rule


established by legal fiction, the corporation has a personality separate and distinct
from its officers, stockholders and members. Hence, officers of a corporation are not
personally liable for their official acts unless it is shown that they have exceeded their
authority. This fictional veil, however, can be pierced by the very same law which
created it when "the notion of the legal entity is used as a means to perpetrate fraud,
an illegal act, as a vehicle for the evasion of an existing obligation, and to confuse
legitimate issues". Under the Labor Code, for instance, when a corporation violates a
provision declared to be penal in nature, the penalty shall be imposed upon the guilty
officer or officers of the corporation.
In the case at bar, the thrust of petitioners' arguments was aimed at confining
liability solely to the corporation, as if the entity were an automaton designed to
perform functions at the push of a button. The issue, however, is not limited to
payment of separation pay under Article 283 but also payment of labor standard
benefits such as underpayment of wages, holiday pay and 13th month pay to two of
the private respondents. While there is no sufficient evidence to conclude that
petitioners have indiscriminately stopped the entity's business, at the same time,
petitioners have opted to abstain from presenting sufficient evidence to establish the
serious and adverse financial condition of the company.

174 | P a g e
Law 321_Corporation LAW_ Case Digest

BENJAMIN SANTOS
vs.
NLRC, MELVIN MILLENA
GR 101699, 13 March 1996

FACTS:

Melvin Millena, on 01 October 1985, was hired to be the project accountant for
Mana Mining and Development Corp.‘s mining operations in Gatbo, Bacon, Sorsogon.
On 12 August 1986, private respondent sent to Mr. Gil Abaño, the MMDC corporate
treasurer, a memorandum calling the latter's attention to the failure of the company to
comply with the withholding tax requirements of, and to make the corresponding
monthly remittances to, the Bureau of Internal Revenue ("BIR") on account of delayed
payments of accrued salaries to the company's laborers and employees.
Albaño responded that the mining operations in Sorsogon shall be stopped
pending the end of the wet season and the normalization of the peace and order
situation in the province. Therefore, MMDC is dispensing the services of Millena
because of lack of work load.
Private respondent expressed "shock" over the termination of his employment.
He complained that he would not have resigned from the Sycip, Gorres & Velayo
accounting firm, where he was already a senior staff auditor, had it not been for the
assurance of a "continuous job" by MMDC's Engr. Rodillano E. Velasquez. Private
respondent requested that he be reimbursed the "advances" he had made for the
company and be paid his "accrued salaries/claims
With his demands left unheeded, Millena filed a complaint for illegal dismissal,
unpaid salaries, 13th month pay, overtime pay, separation pay and incentive leave pay
against MMDC and its two top officials, namely, herein petitioner Benjamin A. Santos
(the President) and Rodillano A. Velasquez (the executive vice-president).

ISSUE:

Whether or not the impleaded officials of MMDC may be held liable.

RULING:

NO.

It was not proven that they acted in bad faith. A corporation is a juridical entity
with legal personality separate and distinct from those acting for and in its behalf and,
in general, from the people comprising it. Nevertheless, being a mere fiction of law,
peculiar situations or valid grounds can exist to warrant, albeit done sparingly, the
disregard of its independent being and the lifting of the corporate veil. The Court also
has collated the settled instances when, without necessarily piercing the veil of
corporate fiction, personal civil liability can also be said to lawfully attach to a
corporate director, trustee or officer.
The case of petitioner is way off these exceptional instances. It is not even
shown that petitioner has had a direct hand in the dismissal of private respondent
enough to attribute to him (petitioner) a patently unlawful act while acting for the
corporation. It is undisputed that the termination of petitioner's employment has,
instead, been due, collectively, to the need for a further mitigation of losses, the onset
of the rainy season, the insurgency problem in Sorsogon and the lack of funds to
further support the mining operation in Gatbo.

175 | P a g e
Law 321_Corporation LAW_ Case Digest

JOSE SIA
vs.
PEOPLE OF THE PHILIPPINES
GR L-30896, 28 April 1983

FACTS:

Jose O. Sia sometime prior to 24 May, 1963, was General Manager of the Metal
Manufacturing Company of the Philippines, Inc. engaged in the manufacture of steel
office equipment; on 31 May, 1963, because his company was in need of raw materials
to be imported from abroad, he applied for a letter of credit to import steel sheets from
Mitsui Bussan Kaisha, Ltd. of Tokyo, Japan, the application being directed to the
Continental Bank, herein complainant, and his application having been approved, the
letter of credit was opened on 5 June, 1963 in the amount of $18,300. The goods
arrived sometime in July, 1963 according to accused himself, now from here on there
is some debate on the evidence; according to Complainant Bank, there was permitted
delivery of the steel sheets only upon execution of a trust receipt, while according to
the accused, the goods were delivered to him sometime before he executed that trust
receipt in fact they had already been converted into steel office equipment by the time
he signed said trust receipt. But there is no question - and this is not debated — that
the bill of exchange issued for the purpose of collecting the unpaid account thereon
having fallen due neither accused nor his company having made payment thereon
notwithstanding demands, and the accounts having reached the sum in pesos of
P46,818.68 after deducting his deposit valued at P28,736.47.

ISSUE:

Whether or not Sia should be held liable.

RULING:

NO.

The bank is transacting with Metal Manufacturing and not with him. The case
cited by the Court of Appeals in support of its stand - Tan Boon Kong case, supra -
may however not be squarely applicable to the instant case in that the corporation was
directly required by law to do an act in a given manner, and the same law makes the
person who fails to perform the act in the prescribed manner expressly liable
criminally. The performance of the act is an obligation directly imposed by the law on
the corporation. Since it is a responsible officer or officers of the corporation who
actually perform the act for the corporation, they must of necessity be the ones to
assume the criminal liability; otherwise this liability as created by the law would be
illusory, and the deterrent effect of the law, negated.
In the present case, a distinction is to be found with the Tan Boon Kong case in
that the act alleged to be a crime is not in the performance of an act directly ordained
by law to be performed by the corporation. The act is imposed by agreement of parties,
as a practice observed in the usual pursuit of a business or a commercial transaction.
The offense may arise, if at all, from the peculiar terms and condition agreed upon by
the parties to the transaction, not by direct provision of the law. The intention of the
parties, therefore, is a factor determinant of whether a crime was committed or
whether a civil obligation alone intended by the parties. With this explanation, the
distinction adverted to between the Tan Boon Kong case and the case at bar should
come out clear and meaningful. In the absence of an express provision of law making
the petitioner liable for the criminal offense committed by the corporation of which he
is a president as in fact there is no such provisions in the Revised Penal Code under
which petitioner is being prosecuted, the existence of a criminal liability on his part
may not be said to be beyond any doubt. In all criminal prosecutions, the existence of
criminal liability for which the accused is made answerable must be clear and certain.
The maxim that all doubts must be resolved in favor of the accused is always of
compelling force in the prosecution of offenses.

176 | P a g e
Law 321_Corporation LAW_ Case Digest

TRAMAT MERCANTILE, INC. and DAVID HONG


vs.
COURT OF APPEALS, MELCHOR DE LA CUESTA
GR 111008, 07 November 1994

FACTS:

On 09 April 1984, Melchor de la Cuesta, doing business under the name and
style of "Farmers Machineries," sold to Tramat Mercantile, Inc. one unit Hinomoto
Tractor Model MB 1100D powered by a 13 H.P. diesel engine. In payment, David Ong,
Tramat's president and manager, issued a check for P33,500.00 (apparently replacing
an earlier postdated check for P33,080.00). Tramat, in turn, sold the tractor, together
with an attached lawn mower fabricated by it, to the Metropolitan Waterworks and
Sewerage System/NAWASA for P67,000.00. David Ong caused a stop payment of the
check when NAWASA refused to pay the tractor and lawn mower after discovering
that, aside from some stated defects of the attached lawn mower, the engine (sold by
de la Cuesta) was a reconditioned unit.
On 28 May 1985, de la Cuesta filed an action for the recovery of P33,500.00, as
well as attorney's fees of P10,000.00, and the costs of suit. Ong, in his answer,
averred, among other things, that de la Cuesta had no cause of action; that the
questioned transaction was between plaintiff and Tramat Mercantile, Inc., and not
with Ong in his personal capacity; and that the payment of the check was stopped
because the subject tractor had been priced as a brand new, not as a reconditioned
unit.

ISSUE:

Whether or not Ong should be held liable for the unpaid tractor.

RULING:

NO.

It is an error to hold David Ong jointly and severally liable with TRAMAT to de
la Cuesta under the questioned transaction. Ong had acted, not in his personal
capacity, but as an officer of a corporation, TRAMAT, with a distinct and separate
personality. As such, it should only be the corporation, not the person acting for and
on its behalf, that properly could be made liable thereon.
Tramat, however, should be held liable for the unpaid tractor because at the
time of the purchase, the appellants did not reveal to the appellee the true purpose for
which the tractor would be used. Granting that the appellants informed the appellee
that they would be reselling the unit to the MWSS, an entity admittedly not engaged in
farming, and that they ordered the tractor without the power tiller, an indispensable
accessory if the tractor would be used in farming, these in themselves would not
constitute the required implied notice to the appellee as seller.

177 | P a g e
Law 321_Corporation LAW_ Case Digest

Self-Dealing Director/Officer

REPUBLIC OF THE PHILIPPINES


vs.
EDUARDO CONJUANGCO
GR 166859, 12 April 2011

FACTS:

Danding Cojuangco is being accused of using public funds to finance his


acquisition of shares in the San Miguel Corporation. Through the coconut levy fund,
was is being accused of buying out shareholders in the corporation in order to
become a substantial shareholder himself. To carry out his scheme, he used dummy
shareholders who shall be trustors of the shares on his behalf.
Contention rises on his culpability as a public official during the time that he
bought the shares. It is claimed by the Sandiganbayan that he was able to amass vast
shares of the corporation through the use of the coconut levy fund, which is public in
nature. Therefore, it but apparent that he be held liable for his actions in taking
control of the corporation.

ISSUE:

Whether or not Conjuangco illegally used ill-gotten wealth to buy shares of


SMC.

RULING:

NO.

The funds are in fact loaned from UCPB, which was organized as a depositary of
the coconut levy funds of the corporation. Also, the Government failed to adduce
substantial evidence linking Cojuangco to the use of Marcos ill-gotten wealth.
All these judicial pronouncements demand two concurring elements to be
present before assets or properties were considered as ill-gotten wealth, namely: (a)
they must have ―originated from the government itself,‖ and (b) they must have been
taken by former President Marcos, his immediate family, relatives, and close
associates by illegal means.
But settling the sources and the kinds of assets and property covered by E.O.
No. 1 and related issuances did not complete the definition of ill-gotten wealth. The
further requirement was that the assets and property should have been amassed by
former President Marcos, his immediate family, relatives, and close associates both
here and abroad. In this regard, identifying former President Marcos, his immediate
family, and relatives was not difficult, but identifying other persons who might be the
close associates of former President Marcos presented an inherent difficulty, because
it was not fair and just to include within the term close associates everyone who had
had any association with President Marcos, his immediate family, and relatives.
It does not suffice, as in this case, that the respondent is or was a government
official or employee during the administration of former Pres. Marcos. There must be a
prima facie showing that the respondent unlawfully accumulated wealth by virtue of
his close association or relation with former Pres. Marcos and/or his wife. This is so
because otherwise the respondent‘s case will fall under existing general laws and
procedures on the matter.

178 | P a g e
Law 321_Corporation LAW_ Case Digest

CHARLES W. MEAD
vs.
E.C. MCCULLOUGH, et. al.
GR 6217, 26 December 2011

FACTS:

On March 15, 1902, the plaintiff (Mead will be referred to as the plaintiff in this
opinion unless it is otherwise stated) and the defendant organized the "Philippine
Engineering and Construction Company.
Shortly after the organization, the directors held a meeting and elected the
plaintiff as general manager. The plaintiff held this position with the company for nine
months, when he resigned to accept the position of engineer of the Canton and
Shanghai Railway Company.
The contract and work undertaken by the company during the management of
Mead were the wrecking contract with the Navy Department at Cavite for the raising of
the Spanish ships sunk by Admiral Dewey; the contract for the construction of certain
warehouses for the quartermaster department; the construction of a wharf at Fort
McKinley for the Government; The supervision of the construction of the Pacific
Oriental Trading Company's warehouse; and some other odd jobs not specifically set
out in the record.
Shortly after the plaintiff left the Philippine Islands for China, the other
directors, the defendants in this case, held a meeting on December 24, 1903, for the
purpose of discussing the condition of the company at that time and determining what
course to pursue.
The assignees of the wrecking contract, including McCullough, formed was not
known as the "Manila Salvage Association." This association paid to McCullough
$15,000 Mexican Currency cash for the assignment of said contract. In addition to
this payment, McCullough retained a one-sixth interest in the new company or
association.

ISSUE:

Whether or not the respondents are self-dealing directors.

RULING:

NO.

While a corporation remains solvent, there is no reason why a director or


officer, by the authority of a majority of the stockholders or board of managers, may
not deal with the corporation, loan it money or buy property from it, in like manner as
a stranger. So long as a purely private corporation remains solvent, its directors are
agents or trustees for the stockholders. They owe no duties or obligations to others.
But the moment such a corporation becomes insolvent, its directors are trustees of all
the creditors, whether they are members of the corporation or not, and must manage
its property and assets with strict regard to their interest; and if they are themselves
creditors while the insolvent corporation is under their management, they will not be
permitted to secure to themselves by purchasing the corporate property or otherwise
any personal advantage over the other creditors. Nevertheless, a director or officer may
in good faith and for an adequate consideration purchase from a majority of the
directors or stockholders the property even of an insolvent corporation, and a sale
thus made to him is valid and binding upon the minority.

179 | P a g e
Law 321_Corporation LAW_ Case Digest

PRIME WHITE CEMENT


vs.
INTERMEDIATE APPELLATE COURT, ALEJANDRO TE
GR 68555, 19 March 1993

FACTS:

On the 16th day of July, 1969, plaintiff and defendant corporation thru its
President, Mr. Zosimo Falcon and Justo C. Trazo, as Chairman of the Board, entered
into a dealership agreement (Exhibit A) whereby said plaintiff was obligated to act as
the exclusive dealer and/or distributor of the said defendant corporation of its cement
products in the entire Mindanao area.
Prime, however, amended the agreement made with Te, forcing the latter to
demand the performance of the conditions stated in the original contract. Aside from
that, Prime entered into a dealership contract with Napoleon Co, therefore violating
the exclusive rights of Te in Mindanao. Te thereafter filed for specific performance
against Prime.
Prime questioned the validity of the contract, claiming it is null and void due to
the fact that Te is a Director and the Auditor of the cement company.

ISSUE:

Whether or not the dealership contract between Prime and Te is valid.

RULING:

NO.

The requisites for the approval of a contract with a ‗self dealing director‘ was not
satisfied. A director of a corporation holds a position of trust and as such, he owes a
duty of loyalty to his corporation. In case his interests conflict with those of the
corporation, he cannot sacrifice the latter to his own advantage and benefit. As
corporate managers, directors are committed to seek the maximum amount of profits
for the corporation. This trust relationship "is not a matter of statutory or technical
law. It springs from the fact that directors have the control and guidance of corporate
affairs and property and hence of the property interests of the stockholders."
A director's contract with his corporation is not in all instances void or voidable.
If the contract is fair and reasonable under the circumstances, it may be ratified by
the stockholders provided a full disclosure of his adverse interest is made.
Granting arguendo that the "dealership agreement" involved here would be valid
and enforceable if entered into with a person other than a director or officer of the
corporation, the fact that the other party to the contract was a Director and Auditor of
the petitioner corporation changes the whole situation. First of all, the contract was
neither fair nor reasonable. The "dealership agreement" entered into in July, 1969,
was to sell and supply to respondent Te 20,000 bags of white cement per month, for
five years starting September, 1970, at the fixed price of P9.70 per bag. Respondent Te
is a businessman himself and must have known, or at least must be presumed to
know, that at that time, prices of commodities in general, and white cement in
particular, were not stable and were expected to rise. At the time of the contract,
petitioner corporation had not even commenced the manufacture of white cement, the
reason why delivery was not to begin until 14 months later. He must have known that
within that period of six years, there would be a considerable rise in the price of white
cement. In fact, respondent Te's own Memorandum shows that in September, 1970,
the price per bag was P14.50, and by the middle of 1975, it was already P37.50 per
bag. Despite this, no provision was made in the "dealership agreement" to allow for an
increase in price mutually acceptable to the parties. Instead, the price was pegged at
P9.70 per bag for the whole five years of the contract. Fairness on his part as a
director of the corporation from whom he was to buy the cement, would require such a
provision.

180 | P a g e
Law 321_Corporation LAW_ Case Digest

Contracts Between Corporations with Interlocking Directors

PEDRO PALTING
vs.
SAN JOSE PETROLEUM, INC.
GR L-14441, 17 December 1966

FACTS:

San Jose Petroleum filed with the Philippine Securities and Exchange
Commission a sworn registration statement, for the registration and licensing for sale
in the Philippines Voting Trust Certificates representing 2,000,000 shares of its capital
stock with a par value of $0.35 a share, at P1.00 per share
Pedro R. Palting and others, allegedly prospective investors in the shares of San
Jose Petroleum, filed with the Securities and Exchange Commission an opposition to
the registration and licensing of the securities on the grounds that (1) the tie-up
between the issuer, San Jose Petroleum, a Panamanian corporation, and San Jose Oil,
a domestic corporation, violates the Constitution of the Philippines, the Corporation
Law and the Petroleum Act of 1949; (2) the issuer has not been licensed to transact
business in the Philippines; (3) the sale of the share of the issuer is fraudulent, and
works or tends to work a fraud upon Philippine purchasers; and (4) the issuer as an
enterprise, as well as its business, is based upon unsound business principles.

ISSUE:

Whether or not San Jose Petroleum can validly engage in business in the
Philippines.

RULING:

NO.

It does not have the required percentage of Filipino capital to validly exercise its
business in the Philippines. In the two lists of stockholders, there is no indication of
the citizenship of these stockholders, or of the total number of authorized stocks of
each corporation for the purpose of determining the corresponding percentage of these
listed stockholders in relation to the respective capital stock of said corporation.
These provisions are in direct opposition to the corporation law and corporate
practices in this country. These provisions alone would outlaw any corporation locally
organized or doing business in this jurisdiction. Consider the unique and unusual
provision that no contract or transaction between the company and any other
association or corporation shall be affected except in case of fraud, by the fact that any
of the directors or officers of the company may be interested in or are directors or
officers of such other association or corporation; and that none of such contracts or
transactions of this company with any person or persons, firms, associations or
corporations shall be affected by the fact that any director or officer of this company is
a party to or has an interest in such contract or transaction or has any connection
with such person or persons, firms, associations or corporations: and that any and all
persons who may become directors or officers of this company are hereby relieved of
all responsibility which they would otherwise incur by reason of any contract entered
into which this company either for their own benefit, or for the benefit of any person,
firm, association or corporation in which they may be interested.

181 | P a g e
Law 321_Corporation LAW_ Case Digest

DEVELOPMENT BANK OF THE PHILIPPINES


vs.
COURT OF APPEALS, REMINGTON INDUSTRIAL SALES
GR 126200, 16 August 2001

FACTS:

Between July 1981 and April 1984, Marinduque Mining entered into 3
mortgage agreements with PNB and DBP involving its real properties located in
Surigao del Norte, Negros Occidental, and Rizal, as well as its equipment located
therein. Marinduque failed to pay its loans, causing the foreclosure of the said
mortgages. PNB and DBP thereafter gained control of the said properties.
In the meantime, between July 16, 1982 to October 4, 1983, Marinduque
Mining purchased and caused to be delivered construction materials and other
merchandise from Remington Industrial Sales Corporation. The purchases remained
unpaid as of August 1, 1984 when Remington filed a complaint for a sum of money
and damages against Marinduque Mining for the value of the unpaid construction
materials and other merchandise purchased by Marinduque Mining, as well as
interest, attorney‘s fees and the costs of suit.
Remington‘s original complaint was amended to include PNB, DBP, Maricalum
Mining Corporation (Maricalum Mining) and Island Cement Corporation (Island
Cement) as co-defendants. Remington asserted that Marinduque Mining, PNB, DBP,
Nonoc Mining, Maricalum Mining and Island Cement must be treated in law as one
and the same entity by disregarding the veil of corporate fiction since the personnel,
key officers and rank-and-file workers and employees of co-defendants NMIC,
Maricalum and Island Cement creations of co-defendants PNB and DBP were the
personnel of co-defendant MMIC such that practically there has only been a change of
name for all legal purpose and intents.

ISSUE:

Whether or not the takeover of PNB and DBP over Marinduque Mining is in bad
faith.

RULING:

NO.

Their actions are mandated under the law. Where the corporations have
directors and officers in common, there may be circumstances under which their
interest as officers in one company may disqualify them in equity from representing
both corporations in transactions between the two. Thus, where one corporation was
‗insolvent and indebted to another, it has been held that the directors of the creditor
corporation were disqualified, by reason of self-interest, from acting as directors of the
debtor corporation in the authorization of a mortgage or deed of trust to the former to
secure such indebtedness In the same manner that when the corporation is insolvent,
its directors who are its creditors cannot secure to themselves any advantage or
preference over other creditors. They cannot thus take advantage of their fiduciary
relation and deal directly with themselves, to the injury of others in equal right. If they
do, equity will set aside the transaction at the suit of creditors of the corporation or
their representatives, without reference to the question of any actual fraudulent intent
on the part of the directors, for the right of the creditors does not depend upon fraud
in fact, but upon the violation of the fiduciary relation to the directors.
Directors of insolvent corporation, who are creditors of the company, can not
secure to themselves any preference or advantage over other creditors in the payment
of their claims. It is not good morals or good law. The governing body of officers
thereof are charged with the duty of conducting its affairs strictly in the interest of its
existing creditors, and it would be a breach of such trust for them to undertake to give
any one of its members any advantage over any other creditors in securing the
payment of his debts in preference to all others.

182 | P a g e
Law 321_Corporation LAW_ Case Digest

Disloyalty

JOHN GOKONGWEI
vs.
SEC, ANDRES SORIANO, et al.
GR L-45911, 11 April 1979

FACTS:

Gokonwei alleged that on September 18, 1976, individual respondents amended


by bylaws of San Miguel Corporation, basing their authority to do so on a resolution of
the stockholders adopted on March 13, 1961, when the outstanding capital stock of
respondent corporation was only P70,139.740.00, divided into 5,513,974 common
shares at P10.00 per share and 150,000 preferred shares at P100.00 per share. At the
time of the amendment, the outstanding and paid up shares totalled 30,127,043, with
a total par value of P301,270,430.00. It was contended that according to section 22 of
the Corporation Law and Article VIII of the by-laws of the corporation, the power to
amend, modify, repeal or adopt new by-laws may be delegated to the Board of
Directors only by the affirmative vote of stockholders representing not less than 2/3 of
the subscribed and paid up capital stock of the corporation, which 2/3 should have
been computed on the basis of the capitalization at the time of the amendment. Since
the amendment was based on the 1961 authorization, petitioner contended that the
Board acted without authority and in usurpation of the power of the stockholders.
It was claimed that prior to the questioned amendment, petitioner had all the
qualifications to be a director of respondent corporation, being a substantial
stockholder thereof; that as a stockholder, petitioner had acquired rights inherent in
stock ownership, such as the rights to vote and to be voted upon in the election of
directors; and that in amending the by-laws, respondents purposely provided for
petitioner's disqualification and deprived him of his vested right as afore-mentioned,
hence the amended by-laws are null and void.

ISSUE:

Whether or not SMC‘s BoD acted in bad faith in making the amendment which
disqualified Gokongwei from being elected as Director.

RULING:

NO.

SMC is merely protecting its interest from Gokongwei, who owns companies in
direct competition with SMC‘s business. Although in the strict and technical sense,
directors of a private corporation are not regarded as trustees, there cannot be any
doubt that their character is that of a fiduciary insofar as the corporation and the
stockholders as a body are concerned. As agents entrusted with the management of
the corporation for the collective benefit of the stockholders, they occupy a fiduciary
relation, and in this sense the relation is one of trust. It springs from the fact that
directors have the control and guidance of corporate affairs and property; hence of the
property interests of the stockholders. Equity recognizes that stockholders are the
proprietors of the corporate interests and are ultimately the only beneficiaries thereof
It is obviously to prevent the creation of an opportunity for an officer or director
of San Miguel Corporation, who is also the officer or owner of a competing corporation,
from taking advantage of the information which he acquires as director to promote his
individual or corporate interests to the prejudice of San Miguel Corporation and its
stockholders, that the questioned amendment of the by-laws was made.
Certainly, where two corporations are competitive in a substantial sense, it
would seem improbable, if not impossible, for the director, if he were to discharge
effectively his duty, to satisfy his loyalty to both corporations and place the
performance of his corporation duties above his personal concerns.

183 | P a g e
Law 321_Corporation LAW_ Case Digest

ELEANOR ERICA STRONG, ET AL.


vs.
FRANCISCO GUTIERREZ REPIDE
G.R. No. L-7154. February 21, 1912

FACTS:

Eleanor Strong was the owner of 800 shares of the capital stock of Philippine
Sugar Estate Development Company. Gutierrez Rapide, owner of three-fourths shares
of the company‘s stock , 1 of the 5 directors of the company and was elected by the
board as administrator general of such company, took steps to purchase the 800
shares owned by Strong, which he knew were in the possession of F. Stuart Jones, as
her agent. Instead of seeing Jones, who had an office next door, Repide employed one
Kauffman. Kaufmann, in turn, employed Mr. Sloan, a broker, to purchase the stock
for him. Kauffman told Sloan that the stock to be purchased was for a member of his
wife‘s family. This action by Repide was due to the negotiations initiated by the
government where the latter will purchase the company‘s lands (together with other
friar lands) at a price which greatly enhance the value of the stock.
As a result of the negotiations, Jones, assuming he had the power and without
consulting Strong, sold the 800 shares. Strong filed a case to recover the shares from
Repide on the ground that the shares had been sold and delivered by Strong‘s agent
without authority to do so and on the ground that Repide fraudulently concealed from
Strong‘s agent the facts affecting the value of the stock so sold and delivered.

ISSUE:

Whether or notRepide, acting in good faith, has the duty to disclose to the agent
of Strong the facts bearing upon or which might affect the value of the stocks.

RULING:

YES.

The Court ruled that there is no relationship of a fiduaciary nature exists


between a director and a shareholder in a business corporation. There are cases,
however, where, by reason of special facts, such duty of a director to disclose to a
shareholder the knowledge which he may possess regarding the value of the shares of
the company before he purchases any from a shareholder. Some special facts are
present in this case such as the fact the Repide is not only a director of the
corporation but an owner of three-fourths shares of its stock. He was the chief
negotiator for the sale of all the lands and was acting substantially as the agent of the
shareholders by reason of his ownership of the shares. Thus, a purchase of stock in a
corporation by a director and owner of three-fourths of the entire capital stock, who
was also administrator general of the company and engaged in the negotiations which
finally led to the sale of company‘s lands to the government at a price which greatly
enhanced the value of the stock, was fraudulent as procured by insidious machination
where he employed an agent to make the purchase, concealing both his identity as
purchaser and his knowledge of the state of the negotiations and their probable
successful result

184 | P a g e
Law 321_Corporation LAW_ Case Digest

Watered Stocks

LIRAG TEXTILE MILLS and BASILIO LIRAG


vs.
SSS, HON. PACIFICO DE CASTRO
GR L-33205, 31 August 1987

FACTS:

That on September 4, 1961, the SSS and Lirag Textile Mills, Inc. and Basilio
Lirag entered into a Purchase Agreement under which the plaintiff agreed to purchase
from the said defendant preferred shares of P1,000,000.00 subject to the conditions
set forth in such agreement. Pursuant to the Purchase Agreement of September 4,
1961, SSS, on January 31, 1962, paid Lirag Textile Mills, Inc. the sum of P500,000.00
for which the said defendant issued to plaintiff 5,000 preferred shares with a par value
of P100.00 per share.
To guarantee the redemption of the stocks purchased by the plaintiff, the
payment of dividends, as well as the other obligations of the Lirag Textile Mills, Basilio
signed the Purchase Agreement of September 4, 1961 not only as president of the
defendant corporation, but also as surety so that should the Lirag Textile Mills, Inc.
fail to perform any of its obligations in the said Purchase Agreement, the surety shall
immediately pay to the vendee the amounts then outstanding.
Notwithstanding such letters of demand to the defendant Basilio L. Lirag, Stock
Certificates Nos. 128 and 139 issued to plaintiff are still unredeemed and no dividends
have been paid on said stock certificates;

ISSUE:

Whether or not Lirag Textile is liable to SSS.

RULING:

YES.

It failed to comply with its contractual stipulations. The Purchase Agreement is,
indeed, a debt instrument. Its terms and conditions unmistakably show that the
parties intended the repurchase of the preferred shares on the respective scheduled
dates to be an absolute obligation which does not depend upon the financial ability of
petitioner corporation. This absolute obligation on the part of petitioner corporation is
made manifest by the fact that a surety was required to see to it that the obligation is
fulfilled in the event of the principal debtor's inability to do so. The unconditional
undertaking of petitioner corporation to redeem the preferred shares at the specified
dates constitutes a debt which is defined "as an obligation to pay money at some fixed
future time, or at a time which becomes definite and fixed by acts of either party and
which they expressly or impliedly, agree to perform in the contract.
A stockholder sinks or swims with the corporation and there is no obligation to
return the value of his shares by means of repurchase if the corporation incurs losses
and financial reverses, much less guarantee such repurchase through a surety.

185 | P a g e
Law 321_Corporation LAW_ Case Digest

RICARDO NAVA
vs.
PEERS MARKETING CORP., RENATO CUSI and AMPARO CUSI
GR L-28120, 25 November 1976

FACTS:

Teofilo Po as an incorporator subscribed to eighty shares of Peers Marketing


Corporation at one hundred pesos a share or a total par value of eight thousand
pesos. Po paid two thousand pesos or twenty-five percent of the amount of his
subscription. No certificate of stock was issued to him or, for that matter, to any
incorporator, subscriber or stockholder.
On April 2, 1966 Po sold to Ricardo A. Nava for two thousand pesos twenty of
his eighty shares. In the deed of sale Po represented that he was "the absolute and
registered owner of twenty shares" of Peers Marketing Corporation.
Nava requested the officers of the corporation to register the sale in the books of
the corporation. The request was denied because Po has not paid fully the amount of
his subscription. Nava was informed that Po was delinquent in the payment of the
balance due on his subscription and that the corporation had a claim on his entire
subscription of eighty shares which included the twenty shares that had been sold to
Nava.

ISSUE:

Whether or not Peers may be compelled by mandamus to register the stocks in


Nava‘s name.

RULING:

NO.

There‘s no certificate of stock issued in favor of Po. Shares of stock may be


transferred by delivery to the transferee of the certificate properly indorsed. "Title may
be vested in the transferee by delivery of the certificate with a written assignment or
indorsement thereof" There should be compliance with the mode of transfer prescribed
by law.
The usual practice is for the stockholder to sign the form on the back of the
stock certificate. The certificate may thereafter be transferred from one person to
another. If the holder of the certificate desires to assume the legal rights of a
shareholder to enable him to vote at corporate elections and to receive dividends, he
fills up the blanks in the form by inserting his own name as transferee. Then he
delivers the certificate to the secretary of the corporation so that the transfer may be
entered in the corporation's books. The certificate is then surrendered and a new one
issued to the transferee.
That procedure cannot be followed in the instant case because, as already
noted, the twenty shares in question are not covered by any certificate of stock in Po's
name. Moreover, the corporation has a claim on the said shares for the unpaid
balance of Po's subscription. A stock subscription is a subsisting liability from the
time the subscription is made. The subscriber is as much bound to pay his
subscription as he would be to pay any other debt. The right of the corporation to
demand payment is no less incontestable.
In this case no stock certificate was issued to Po. Without the stock certificate,
which is the evidence of ownership of corporate stock, the assignment of corporate
shares is effective only between the parties to the transaction.

186 | P a g e
Law 321_Corporation LAW_ Case Digest

Derivative Suit: Remedies to Enforce Personal Liability

JUANITO ANG, for and in behalf of SUNRISE MARKETING (BACOLOD), INC.


vs.
SPOUSES ROBERTO and RACHEL ANG
G.R. No. 201675. June 19, 2013

FACTS:

Sps. Roberto and Rachel Ang took over the active management of [SMBI].
Through the employment of sugar coated words, they were able to successfully
manipulate the stocks sharings between themselves at 50-50 under the condition that
the procedures mandated by the Corporation Code on increase of capital stock be
strictly observed (valid Board Meeting). No such meeting of the Board to increase
capital stock materialized. It was more of an accommodation to buy peace.
Juanito claimed that payments to Nancy and Theodore ceased sometime after
2006. On 24 November 2008, Nancy and Theodore, through their counsel here in the
Philippines, sent a demand letter to "Spouses Juanito L. Ang/Anecita L. Ang and
Spouses Roberto L. Ang/Rachel L. Ang" for payment of the principal amounting to
$1,000,000.00 plus interest at ten percent (10%) per annum, for a total of
$2,585,577.37 within ten days from receipt of the letter. 12 Roberto and Rachel then
sent a letter to Nancy and Theodore‘s counsel on 5 January 2009, saying that they are
not complying with the demand letter because they have not personally contracted a
loan from Nancy and Theodore.

ISSUE:

Whether or not the Honorable Court of Appeals erred in ordering the dismissal
of the Complaint on the ground that the case is not a derivative suit.

RULING:

NO.

The Complaint is not a derivative suit. A derivative suit is an action brought by


a stockholder on behalf of the corporation to enforce corporate rights against the
corporation‘s directors, officers or other insiders. Under Sections 23 and 36 of the
Corporation Code, the directors or officers, as provided under the by-laws, have the
right to decide whether or not a corporation should sue. Since these directors or
officers will never be willing to sue themselves, or impugn their wrongful or fraudulent
decisions, stockholders are permitted by law to bring an action in the name of the
corporation to hold these directors and officers accountable. In derivative suits, the
real party ininterest is the corporation, while the stockholder is a mere nominal party.

187 | P a g e
Law 321_Corporation LAW_ Case Digest

LISAM ENTERPRISES, INC. represented by LOLITA A. SORIANO, and LOLITA A.


SORIANO
vs.
BANCO DE ORO UNIBANK, INC. (formerly PHILIPPINE COMMERCIAL
INTERNATIONAL BANK),* LILIAN S. SORIANO, ESTATE OF LEANDRO A.
SORIANO, JR., REGISTER OF DEEDS OF LEGASPI CITY, and JESUS L. SARTE
G.R. No. 143264 April 23, 2012

FACTS:

On August 13, 1999, petitioners filed a Complaint against respondents for


Annulment of Mortgage with Prayer for Temporary Restraining Order & Preliminary
Injunction with Damages with the RTC of Legaspi City. Petitioner Lolita A. Soriano
alleged that she is a stockholder of petitioner Lisam Enterprises, Inc. (LEI) and a
member of its Board of Directors, designated as its Corporate Secretary.
On or about 28 March 1996, defendant Lilian S. Soriano and the late Leandro
A. Soriano, Jr., as husband and wife Spouses Soriano, in their personal capacity and
for their own use and benefit, obtained a loan from defendant PCIB (Legaspi Branch)
(Banco de Oro Unibank, Inc.) in the total amount of P20 Million.

ISSUE:

Whether or not the case will prosper.

RULING:

YES.

The courts should be liberal in allowing amendments to pleadings to avoid a


multiplicity of suits and in order that the real controversies between the parties are
presented, their rights determined, and the case decided on the merits without
unnecessary delay. This liberality is greatest in the early stages of a lawsuit, especially
in this case where the amendment was made before the trial of the case, thereby
giving the petitioners all the time allowed by law to answer and to prepare for trial.
The Court enumerated the requisites for filing a derivative suit, as follows: a)
the party bringing the suit should be a shareholder as of the time of the act or
transaction complained of, the number of his shares not being material; b) he has
tried to exhaust intra-corporate remedies, i.e., has made a demand on the board of
directors for the appropriate relief but the latter has failed or refused to heed his plea;
and c) the cause of action actually devolves on the corporation, the wrongdoing or
harm having been, or being caused to the corporation and not to the particular
stockholder bringing the suit.
A reading of the amended complaint will reveal that all the foregoing requisites
had been alleged therein. Hence, the amended complaint remedied the defect in the
original complaint and now sufficiently states a cause of action.

188 | P a g e
Law 321_Corporation LAW_ Case Digest

STRATEGIC ALLIANCE DEV. CORPORATION


vs.
RADSTOCK SECURITIES
GR 178158, 04 December 2009

FACTS:

CCDCP Mining Corporation (CDCP Mining), an affiliate of CDCP, obtained loans


from Marubeni Corporation of Japan (Marubeni). A CDCP official issued letters of
guarantee for the loans although there was no CDCP Board Resolution authorizing the
issuance of such letters of guarantee. CDCP Mining secured the Marubeni loans when
CDCP and CDCP Mining were still privately owned and managed.
In 1983, CDCP‘s name was changed to Philippine National Construction
Corporation (PNCC) in order to reflect that the Government already owned 90.3% of
PNCC and only 9.70% is under private ownership. Meanwhile, the Marubeni loans to
CDCP Mining remained unpaid.
On 20 October 2000 and 22 November 2000, the PNCC Board of Directors
(PNCC Board) passed Board Resolutions admitting PNCC‘s liability to Marubeni.
Previously, for two decades the PNCC Board consistently refused to admit any liability
for the Marubeni loans.
In January 2001, Marubeni assigned its entire credit to Radstock Securities
Limited (Radstock), a foreign corporation. Radstock immediately sent a notice and
demand letter to PNCC.
PNCC and Radstock entered into a Compromise Agreement. Under this
agreement, PNCC shall pay Radstock the reduced amount of P6,185,000,000.00 in full
settlement of PNCC‘s guarantee of CDCP Mining‘s debt allegedly totaling
P17,040,843,968.00 (judgment debt as of 31 July 2006). To satisfy its reduced
obligation, PNCC undertakes to (1) "assign to a third party assignee to be designated
by Radstock all its rights and interests" to the listed real properties of PNCC; (2) issue
to Radstock or its assignee common shares of the capital stock of PNCC issued at par
value which shall comprise 20% of the outstanding capital stock of PNCC; and (3)
assign to Radstock or its assignee 50% of PNCC‘s 6% share, for the next 27 years, in
the gross toll revenues of the Manila North Tollways Corporation.
Luis Sison, a stockholder and former PNCC President and Chairman, filed a
derivative suit questioning the legality of the compromise agreement.

ISSUE:

Whether or not Sison‘s derivative suit is valid.

RULING:

YES.

Sison has legal standing to challenge the Compromise Agreement. Although


there was no allegation that Sison filed the case as a derivative suit in the name of
PNCC, it could be fairly deduced that Sison was assailing the Compromise Agreement
as a stockholder of PNCC.
A derivative action is a suit by a stockholder to enforce a corporate cause of
action. Under the Corporation Code, where a corporation is an injured party, its power
to sue is lodged with its board of directors or trustees. However, an individual
stockholder may file a derivative suit on behalf of the corporation to protect or
vindicate corporate rights whenever the officials of the corporation refuse to sue, or are
the ones to be sued, or hold control of the corporation. In such actions, the
corporation is the real party-in-interest while the suing stockholder, on behalf of the
corporation, is only a nominal party.
In this case, the PNCC Board cannot conceivably be expected to attack the
validity of the Compromise Agreement since the PNCC Board itself approved the
Compromise Agreement. In fact, the PNCC Board steadfastly defends the Compromise
Agreement for allegedly being advantageous to PNCC.

189 | P a g e
Law 321_Corporation LAW_ Case Digest

ANTHONY YU et al.
vs.
JOSEPH YUKAYGUAN et al.
GR 177549, 18 June 2009

FACTS:

Petitioners and the respondents were all stockholders of Winchester Industrial


Supply, Inc. On 15 October 2002, respondents filed against petitioners a verified
Complaint forAccounting, Inspection of Corporate Books and Damages through
Embezzlement and Falsification of Corporate Records and Accounts2[6] before the
RTC of Cebu. The said Complaint was filed by respondents, in their own behalf and as
a derivative suit on behalf of Winchester, Inc., and was docketed as SRC Case No.
022-CEB. The factual background of the Complaint was stated in the attached
Affidavit executed by respondent Joseph.
According to respondents, Winchester, Inc. was established and incorporated
on 12 September 1977, with petitioner Anthony as one of the incorporators, holding
1,000 shares of stock worth P100,000.00. Petitioner Anthony paid for the said shares
of stock with respondent Joseph‘s money, thus, making the former a mere trustee of
the shares for the latter.
The case at bar was initiated before the RTC by respondents as a derivative
suit, on their own behalf and on behalf of Winchester, Inc., primarily in order to
compel petitioners to account for and reimburse to the said corporation the corporate
assets and funds which the latter allegedly misappropriated for their personal benefit.

ISSUE:

Whether or not the derivative suit is valid.

RULING:

YES.

The general rule is that where a corporation is an injured party, its power to
sue is lodged with its board of directors or trustees. Nonetheless, an individual
stockholder is permitted to institute a derivative suit on behalf of the corporation
wherein he holds stocks in order to protect or vindicate corporate rights, whenever the
officials of the corporation refuse to sue, or are the ones to be sued, or hold the control
of the corporation. In such actions, the suing stockholder is regarded as a nominal
party, with the corporation as the real party in interest. A derivative action is a suit by
a shareholder to enforce a corporate cause of action. The corporation is a necessary
party to the suit. And the relief which is granted is a judgment against a third person
in favor of the corporation.
Glaringly, a derivative suit is fundamentally distinct and independent from
liquidation proceedings. They are neither part of each other nor the necessary
consequence of the other. There is totally no justification for the Court of Appeals to
convert what was supposedly a derivative suit instituted by respondents, on their own
behalf and on behalf of Winchester, Inc. against petitioners, to a proceeding for the
liquidation of Winchester, Inc.
While it may be true that the parties earlier reached an amicable settlement, in
which they agreed to already distribute the assets of Winchester, Inc., and in effect
liquidate said corporation, it must be pointed out that respondents themselves
repudiated said amicable settlement before the RTC, even after the same had been
partially implemented; and moved that their case be set for pre-trial. Attempts to
again amicably settle the dispute between the parties before the Court of Appeals were
unsuccessful.

190 | P a g e
Law 321_Corporation LAW_ Case Digest

VIRGINIA GOCHAN et al.


vs.
RICHARD YOUNG, et al.
GR 131889, 12 March 2001

FACTS:

Felix Gochan and Sons Realty Corporation (Gochan Realty, for brevity) was
registered with the SEC on June, 1951, with Felix Gochan, Sr., Maria Pan Nuy Go
Tiong, Pedro Gochan, Tomasa Gochan, Esteban Gochan and Crispo Gochan as its
incorporators.
Felix Gochan Sr.'s daughter, Alice, mother of respondents, inherited 50 shares
of stock in Gochan Realty from the former. She died in 1955, leaving the 50 shares to
her husband, John Young, Sr.
In 1962, the Regional Trial Court of Cebu adjudicated 6/14 of these shares to
her children, Richard Young, David Young, Jane Young Llaban, John Young Jr., Mary
Young Hsu and Alexander Thomas Young.
Having earned dividends, these stocks numbered 179 by 20 September 1979.
Five days later (25 September), at which time all the children had reached the age of
majority, their father John Sr., requested Gochan Realty to partition the shares of his
late wife by cancelling the stock certificates in his name and issuing in lieu thereof,
new stock certificates in the names of the children.

ISSUE:

Whether or not respondents have the legal personality to file a derivative suit on
behalf of the corporation.

RULING:

NO.

Where corporate directors have committed a breach of trust either by their


frauds, ultra vires acts, or negligence, and the corporation is unable or unwilling to
institute suit to remedy the wrong, a single stockholder may institute that suit, suing
on behalf of himself and other stockholders and for the benefit of the corporation, to
bring about a redress of the wrong done directly to the corporation and indirectly to
the stockholders.
In the present case, the Complaint alleges all the components of a derivative
suit. The allegations of injury to the Spouses Uy can coexist with those pertaining to
the corporation. The personal injury suffered by the spouses cannot disqualify them
from filing a derivative suit on behalf of the corporation. It merely gives rise to an
additional cause of action for damages against the erring directors. This cause of
action is also included in the Complaint filed before the SEC.
The Spouses Uy have the capacity to file a derivative suit in behalf of and for
the benefit of the corporation. The reason is that, as earlier discussed, the allegations
of the Complaint make them out as stockholders at the time the questioned
transaction occurred, as well as at the time the action was filed and during the
pendency of the action.
As to the Intestate Estate of John Young, Sr., permitting an executor or
administrator to represent or to bring suits on behalf of the deceased, do not prohibit
the heirs from representing the deceased. These rules are easily applicable to cases in
which an administrator has already been appointed. But no rule categorically
addresses the situation in which special proceedings for the settlement of an estate
have already been instituted, yet no administrator has been appointed. In such
instances, the heirs cannot be expected to wait for the appointment of an
administrator; then wait further to see if the administrator appointed would care
enough to file a suit to protect the rights and the interests of the deceased; and in the
meantime do nothing while the rights and the properties of the decedent are violated
or dissipated.

191 | P a g e
Law 321_Corporation LAW_ Case Digest

WESTERN INSTITUTE OF TECHNOLOGY, et al.


vs.
RICARDO SALAS
GR 113032, 21 August 1997

FACTS:

The minority stockholders of WIT, sometime on June 1, 1986 in the principal


office of WIT at La Paz, Iloilo City, a Special Board Meeting was held. In attendance
were other members of the Board including one of the petitioners Reginald Villasis.
Prior to aforesaid Special Board Meeting, copies of notice thereof, dated May 24, 1986,
were distributed to all Board Members, regarding the compensation of the school‘s
officers, which was eventually passed.
A few years later, that is, on March 13, 1991, petitioners Homero Villasis,
Prestod Villasis, Reginald Villasis and Dimas Enriquez filed an affidavit-complaint
against private respondents before the Office of the City Prosecutor of Iloilo, as a result
of which two (2) separate criminal informations, one for falsification of a public
document under Article 171 of the Revised Penal Code and the other for estafa under
Article 315, par. 1(b) of the RPC, were filed before Branch 33 of the Regional Trial
Court of Iloilo City. The charge for falsification of public document was anchored on
the private respondents' submission of WIT's income statement for the fiscal year
1985-1986 with the Securities and Exchange Commission (SEC) reflecting therein the
disbursement of corporate funds for the compensation of private respondents based
on Resolution No. 4, series of 1986, making it appear that the same was passed by the
board on March 30, 1986, when in truth, the same was actually passed on June 1,
1986, a date not covered by the corporation's fiscal year 1985-1986 (beginning May 1,
1985 and ending April 30, 1986).
WIT questioned the legal standing of the petitioners to sue on its behalf,
claiming it did not give them authority to do do. Petitioner, however, contended that
the case is a derivative suit.

ISSUE:

Whether or not the case at bar is a derivative suit.

RULING:

NO.

A derivative suit is an action brought by minority shareholders in the name of


the corporation to redress wrongs committed against it, for which the directors refuse
to sue. It is a remedy designed by equity and has been the principal defense of the
minority shareholders against abuses by the majority. Here, however, the case is not a
derivative suit but is merely an appeal on the civil aspect of Criminal Cases Nos.
37097 and 37098 filed with the RTC of Iloilo for estafa and falsification of public
document. Among the basic requirements for a derivative suit to prosper is that the
minority shareholder who is suing for and on behalf of the corporation must allege in
his complaint before the proper forum that he is suing on a derivative cause of action
on behalf of the corporation and all other shareholders similarly situated who wish to
join. This is necessary to vest jurisdiction upon the tribunal in line with the rule that it
is the allegations in the complaint that vests jurisdiction upon the court or quasi-
judicial body concerned over the subject matter and nature of the action. This was not
complied with by the petitioners either in their complaint before the court a quo nor in
the instant petition which, in part, merely states that "this is a petition for review on
certiorari on pure questions of law to set aside a portion of the RTC decision in
Criminal Cases Nos. 37097 and 37098" since the trial court's judgment of acquittal
failed to impose any civil liability against the private respondents. By no amount of
equity considerations, if at all deserved, can a mere appeal on the civil aspect of a
criminal case be treated as a derivative suit.

192 | P a g e
Law 321_Corporation LAW_ Case Digest

FIRST PHILIPPINE INTERNATIONAL BANK


vs.
COURT OF APPEALS, CARLOS EJERCITO, et al.
GR 115849, 24 January 1996

FACTS:

Janolo and Demetria and Producers Bank, through its bank manager Mercurio
Rivera, entered into a contract to sell involving parcels of land in Laguna owned by the
bank. The sale, however, was disapproved by the bank‘s conservator (the bank is
under receivership). Ejercito insisted that there was already a perfected contract
between him and the bank, considering that the offer that he made was already
approved by the bank‘s board of directors. He then instituted a case for specific
performance against the bank.
On July 11, 1992, during the pendency of the proceedings in the Court of
Appeals, Henry Co and several other stockholders of the Bank filed an action,
purportedly a ―derivative suit‖, against Encarnacion, Demetria and Janolo ―to declare
any perfected sale of the property as unenforceable and to stop Ejercito from enforcing
or implementing the sale.‖ In his answer, Janolo argued that the Second Case was
barred by litis pendentia by virtue of the case then pending in the Court of Appeals.
During the pre-trial conference in the Second Case, plaintiffs filed a Motion for Leave
of Court to Dismiss the Case Without Prejudice. ―Private respondent opposed this
motion on the ground, among others, that plaintiff‘s act of forum shopping justifies the
dismissal of both cases, with prejudice.

ISSUE:

Whether or not the case filed by the stockholders of the bank is a ‗derivative
suit‘.

RULING:

NO.

An individual stockholder is permitted to institute a derivative suit on behalf of


the corporation wherein he holds stock in order to protect or vindicate corporate
rights, whenever the officials of the corporation refuse to sue, or are the ones to be
sued or hold the control of the corporation. In such actions, the suing stockholder is
regarded as a nominal party, with the corporation as the real party in interest.
In the face of the damaging admissions taken from the complaint, petitioners,
quite strangely, sought to deny that the Second Case was a derivative suit, reasoning
that it was brought, not by the minority shareholders, but by Henry Co et al., who not
only own, hold or control over 80% of the outstanding capital stock, but also
constitute the majority in the Board of Directors of petitioner Bank. That being so,
then they really represent the Bank. So, whether they sued ―derivatively‖ or directly,
there is undeniably an identity of interests/entity represented.

193 | P a g e
Law 321_Corporation LAW_ Case Digest

COMMART PHILIPPINES, INC.


vs.
SEC, ALICE MAGLUTAC
GR 85318, 03 June 1991

FACTS:

Commart (Phils.), Inc., (Commart for short) is a corporation organized by two


brothers, Jesus and Mariano Maglutac, to engage in the brokerage business for the
importation of fertilizers and other products/commodities.
Sometime in June 1984, the two brothers agreed to go their separate ways, with
Mariano being persuaded to sell to Jesus his shareholdings in Commart amounting to
25% of the outstanding capital stock. As part of the deal, a "Cooperative Agreement"
was signed, between Commart (represented by Jesus) and Mariano, in which, among
others, Commart ceded to Mariano or to an "acceptable entity" he may create, a
portion of its business, with a pledge of mutual cooperation for a certain period so as
to enable Mariano to get his own corporation off the ground, so to speak.
Mariano's wife, Alice M. Maglutac, has been for years a stockholder and director of
Commart, did not dispose of her shareholdings, and thus continued as such even after
the sale of Mariano's equity.
As broker and indentor, Commart's principal income came from commissions
paid to it in U.S. dollars by foreign suppliers of fertilizers and other commodities
imported by Planters Products, Inc. and other local importers.

ISSUE:

Whether or not Alice has the legal standing to file the derivative suit.

RULING:

YES.

A derivative suit has been the principal defense of the minority shareholder
against abuses by the majority. It is a remedy designed by equity for those situations
where the management, through fraud, neglect of duty, or other cause, declines to
take the proper and necessary steps to assert the corporation's rights. Indeed, to grant
to Commart the light of withdrawing or dismissing the suit, at the instance of majority
stockholders and directors who themselves are the persons alleged to have committed
breaches of trust against the interest of the corporation, would be to emasculate the
right of minority stockholders to seek redress for the corporation. To consider the
Notice of Dismissal filed by Commart as quashing the complaint filed by Alice
Maglutac in favor of the corporation would be to defeat the very nature and function of
a derivative suit and render the right to institute the action illusory.

194 | P a g e
Law 321_Corporation LAW_ Case Digest

ELTON CHASE
vs.
DR. VICTOR BUENCAMINO
GR L-20395, 13 May 1985

FACTS:

Plaintiff Elton Chase, on the other hand, was the owner of Production
Manufacturing Company, of Portland, Oregon, USA, a corporation primarily dedicated
to the operation of a machine shop and heat-treating plant for the production of
tractor parts.
Sometime in 1954, Chase was notified by the Highway Commission of the State
of Oregon that his factory was going to be in the path of a proposed highway. He was
then advised to sell or face expropriation and warned to remove his plant within a
year. His distributor Craig Carrol told him of a Dr. Buencamino of Manila who he said
was interested in establishing a manufacturing plant in the Philippines. Craig Carrol
contacted Buencamino who told him to contact his associate William Cranker in the
United States. 8 Thereafter, a series of negotiations took place both here in Manila,
and in the United States, between Chase on the one hand, and Cranker and
Buencamino, on the other, for the purchase of Chase's factory (Production
Manufacturing Company) and the establishment of a new factory in Manila which was
to be called the American Machinery Engineering Parts, Inc. (Amparts for short). These
negotiations culminated in a final agreement to the effect that - Elton Chase was to be
paid One Hundred Thousand Dollars ($100,000.00) and he would also be given a one-
third interest in Amparts, with the other two, Dr. Buencamino and Cranker, as the
owner of the other two-thirds (2/3) interest, 1/3 interest each; that in exchange for
said $100,000.00 and the 1/3 interest, Chase was to transfer to Amparts his tractor
plant, ship his machineries to Manila, assuming all costs of dismantling, preserving
and crating for shipment to Manila, install said machineries at Amparts plant with the
aid of five technicians and finally, he has to be the production manager of Amparts.
Chase had shipped his machineries and had them installed in the Amparts
plant in Pasig, Rizal. Amparts then began operation with Dr. Buencamino as
President, William Cranker as Manager and Elton Chase as Production Manager. For
sometime the three maintained harmonious relations but later on distrust came in
until finally Chase tendered his letter of resignation as Production Manager. He then
filed a derivative suit against Buencamino and Chase, who allegedly stole from the
corporation. He sought for the dissolution of the corporation.

ISSUE:

Whether or not the corporation may be dissolved.

RULING:

NO.

The case is of derivative in nature, therefore, it was filed for the benefit of the
corporation. The Court grant a dissolution because the action is a derivative one for
the benefit of Amparts and not for the personal benefit of Chase, and Amparts can not
be benefited by its extinction; as to the ouster of Dr. Buencamino from management, it
should not be forgotten that Dr. Buencamino is not only a manager, but is in fact 2/3
owner of Amparts and to oust him from management would amount to his
disenfranchisement as owner of the majority of the enterprise apart from the fact that
it is also established in the proofs that Amparts is already picking up and has been a
going concern after Cranker left unto him the direction of its affairs; the Court
therefore having in mind all these finds that the solution most equitable and just
would be to limit its decision to imposing a monetary judgment upon the guilty parties
for the benefit of Amparts.

195 | P a g e
Law 321_Corporation LAW_ Case Digest

SAN MIGUEL CORPORATION, represented by EDUARDO DE LOS ANGELES


vs.
ERNEST KAHN, ANDRES SORIANO III, BENIGNO TODA, JR., ANTONIO ROXAS,
ANTONIO PRIETO, FRANCISCO EIZMENDI, JR., EDUARDO SORIANO, RALPH
KAHN and RAMON DEL ROSARIO, JR.
G.R. No. 85339. August 11, 1989

FACTS:

33,133,266 shares of the outstanding capital stock of SMC were acquired 14


other corporations, and were placed under a Voting Trust Agreement in favor of the
late Andres Soriano, Jr. However, 33,133,266 SMC shares were sequestered by the
PCGG, on the ground that the stock belonged to Eduardo Cojuangco, Jr., allegedly a
close associate and dummy of former President Marcos. SMC promptly suspended
payment of the other installments of the price to the 14 seller corporations.
On December, 1986, the SMC Board, by Resolution No. 86-122, "decided to
assume the loans incurred by Neptunia for the down payment ((P500M)) on the
33,133,266 shares." The Board opined that there was "nothing illegal in this
assumption (of liability for the loans)," since Neptunia was "an indirectly wholly owned
subsidiary of SMC," there "was no additional expense or exposure for the SMC Group,
and there were tax and other benefits which would redound to the SMC group of
companies. However, at the meeting of the SMC Board, Eduardo de los Angeles, one of
the PCGG representatives in the SMC board, impugned said Resolution No. 86-122.

ISSUE:

Whether or not de los Angeles can file a derivative suit in behalf of the
corporation.

RULING:

YES.

The Court ruled that it is claimed that since de los Angeles 20 shares represent
only .00001644% of the total number of outstanding shares (1 21,645,860), he cannot
be deemed to fairly and adequately represent the interests of the minority
stockholders. The implicit argument — that a stockholder, to be considered as
qualified to bring a derivative suit, must hold a substantial or significant block of
stock — finds no support whatever in the law. The requisites for a derivative suit are
as follows: (a) the party bringing suit should be a shareholder as of the time of the act
or transaction complained of, the number of his shares not being material; (b) he has
tried to exhaust intra-corporate remedies, i.e., has made a demand on the board of
directors for the appropriate relief but the latter has failed or refused to heed his
plea; and (c) the cause of action actually devolves on the corporation, the wrongdoing
or harm having been, or being caused to the corporation and not to the particular
stockholder bringing the suit.
The bona fide ownership by a stockholder of stock in his own right suffices to
invest him with standing to bring a derivative action for the benefit of the corporation.
The number of his shares is immaterial since he is not suing in his own behalf, or for
the protection or vindication of his own particular right, or the redress of a wrong
committed against him, individually, but in behalf and for the benefit of the
corporation.

196 | P a g e
Law 321_Corporation LAW_ Case Digest

HARRIE S. EVERETT, CRAL G. CLIFFORD, ELLIS H. TEAL and GEORGE W.


ROBINSON
vs.
THE ASIA BANKING CORPORATION, NICHOLAS E. MULLEN, ERIC BARCLAY,
ALFRED F. KELLY, JOHN W. MEARS and CHARLES D. MACINTOSH
G.R. No. L-25241. November 3, 1926

FACTS:

In order more effectually to plunder the Company and to defraud these


plaintiffs the said defendants, Mullen, Barclay, Mears and Macintosh, made, executed
and filed in the Bureau of Commerce and Industry of the Philippine Islands, articles of
incorporation of a corporation called the "Philippine Motors Corporation," having its
principal office in the City of Manila, a capital stock of P25,000, of which the sum of
P5,000, was alleged to have been subscribed and paid as follows: the defendant
Barclay P200, defendant Mears P1,200, defendant Kelly P1,200, defendant Macintosh
P1,200, defendant Mullen P1,200, the treasurer thereof being the defendant Mears.
And these plaintiffs beg leave to refer to the original articles of Incorporation on file in
the said Bureau for greater certainty.
That at the time of such incorporation each and every one of the last above
named defendants was an officer or employee of the defendant Bank. That these
plaintiffs have nor information nor means of obtaining information as to whether the
money alleged to have been described by them for their shares of stock was of their
personal funds and property or whether it was money furnished them by the Bank of
purpose moneys such incorporation was a fraud upon these plaintiffs for the reason
that it was intended for the sole purpose of taking over the assets of the Company and
said defendants were enabled to effectuate such intent by reason of their positions as
officers and employees of the Bank.

ISSUE:

Whether or not plaintiffs have the capacity to sue.

RULING:

YES.

Invoking the well-known rule that shareholders cannot ordinarily sue in equity
to redress wrongs done to the corporation, but that the action must be brought by the
Board of Directors, the appellees argue — and the court below held — that the
corporation Teal and Company is a necessary party plaintiff and that the plaintiff
stockholders, not having made any demand on the Board to bring the action, are not
the proper parties plaintiff. But, like most rules, the rule in question has its
exceptions. It is alleged in the complaint and, consequently, admitted through the
demurrer that the corporation Teal and Company is under the complete control of the
principal defendants in the case, and, in these circumstances, it is obvious that a
demand upon the Board of Directors to institute an action and prosecute the same
effectively would have been useless, and the law does not require litigants to perform
useless acts.
The conclusion of the court below that the plaintiffs, not being stockholders in
the Philippine Motors Corporation, had no legal right to proceed against that
corporation in the manner suggested in the complaint evidently rest upon a
misconception of the character of the action. In this proceeding it was necessary for
the plaintiffs to set forth in full the history of the various transactions which
eventually led to the alleged loss of their property and, in making a full disclosure,
references to the Philippine Motors Corporation appear to have been inevitable. It is to
be noted that the plaintiffs seek no judgment against the corporation itself at this
stage of the proceedings.

197 | P a g e
Law 321_Corporation LAW_ Case Digest

RICARDO L. GAMBOA, LYDIA R. GAMBOA, HONORIO DE 1A RAMA, EDUARDO DE


LA RAMA, and the HEIRS OF MERCEDES DE LA RAMA-BORROMEO
vs.
HON. OSCAR R. VICTORIANO as Presiding Judge of the Court of First Instance of
Negros Occidental, Branch II, BENJAMIN LOPUE, SR., BENJAMIN LOPUE, JR.,
LEONITO LOPUE, and LUISA U. DACLES
G.R. No. L-40620. May 5, 1979

FACTS:

The herein petitioners were sued by herein defendants to nullify the issuance of
823 shares of stock of the Inocentes de la Rama, Inc. in favor of the petitioners.
On April 4, 1972, the respondents, are the owners of 1,328 shares of stock of
the Inocentes de la Rama, Inc., a domestic corporation, with an authorized capital
stock of 3,000 shares, with a par value of P100.00 per share, 2,177 of which were
subscribed and issued, thus leaving 823 shares unissued. Then President and Vice-
President of the corporation, respectively, the defendants Mercedes R. Borromeo,
Honorio de la Rama, and Ricardo Gamboa, remaining members of the board of
directors of the corporation, in order to forestall the takeover by the plaintiffs of the
afore-named corporation, surreptitiously met and elected Ricardo L. Gamboa and
Honorio de la Rama as president and vice-president of the corporation, respectively,
and passed a resolution authorizing the sale of the 823 unissued shares of the
corporation to the defendants, at par value, after which the petitioners were elected to
the board of directors of the corporation.
The respondents claimed that the sale of the unissued 823 shares of stock of
the corporation was in violation of the plaintiffs' and pre-emptive rights and made
without the approval of the board of directors representing 2/3 of the outstanding
capital stock, and is in disregard of the strictest relation of trust existing between the
defendants, as stockholders. The respondents prayed that a writ of preliminary
injunction be issued restraining the defendants from committing, or continuing the
performance of an act tending to prejudice, diminish or otherwise injure the plaintiffs'
rights in the corporate properties and funds of the corporation, and from disposing,
transferring, selling, or otherwise impairing the value of the 823 shares of stock
illegally issued. The respondent court granted the prayer.

ISSUES:

Whether or not the proper action is a derivative suit.

RULING:

YES.

An individual stockholder is permitted to institute a derivative suit on behalf of


the corporation wherein he holds stock in order to protect or vindicate corporate
rights, whenever the officials of the corporation refuse to sue, or are the ones to be
sued or hold the control of the corporation. In such actions, the suing stockholder is
regarded as a nominal party, with the corporation as the real party in interest. 12 In
the case at bar, however, the plaintiffs are alleging and vindicating their own
individual interests or prejudice, and not that of the corporation. At any rate, it is yet
too early in the proceedings since the issues have not been joined. Besides, misjoinder
of parties is not a ground to dismiss an action.

198 | P a g e
Law 321_Corporation LAW_ Case Digest

CATALINA R. REYES
vs.
HON. BIENVENIDO A. TAN, as Judge of the Court of First Instance of Manila,
Branch XIII and FRANCISCA R. JUSTINIANI
G.R. No. L-16982. September 30, 1961

FACTS:

The corporation, Roxas-Kalaw Textile Mills, Inc., was organized on June 5, 1954
by defendants Cesar K. Roxas, Adelia K. Roxas, Benjamin M. Roxas, Jose Ma.
Barcelona and Morris Wilson, for and on behalf of the following primary principals
with the following shareholdings: Adelia K. Roxas, 1200 Class A shares; I. Sherman,
900 Class A shares; Robert W. Born, 450 Class A shares and Morris Wilson, 450 Class
A shares; that the respondent holds both Class A and Class B shares and number and
value thereof are is follows: Class A — 50 shares, Class B — 1,250 shares.
On May 8, 1957, the Board of Directors approved a resolution designating one
Dayaram as co-manager and Morris Wilson was likewise designated as co-manager
with responsibilities for the management of the factory only‘. An office in New York
was opened for the purpose of supervising purchases, which purchases must have the
unanimous agreement of Cesar K. Roxas, New York resident member of the board of
directors, Robert Born and Wadhumal Dalamal or their respective representatives.
Several purchases aggregating $289,678.86 were made in New York for raw materials
and shipped to the Philippines, which shipment were found out to consist not of raw
materials but already finished products, for which reasons the Central Bank of the
Philippines stopped all dollar allocations for raw materials for the corporation which
necessarily led to the paralyzation of the operation of the textile mill and its business.

ISSUES:

Whether or not a derivative suit will prosper.

RULING:

NO.

The claim that respondent Justiniani did not take steps to remedy the illegal
importation for a period of two years is without merit. During that period of time
respondent had the right to assume and expect that the directors would remedy the
anomalous situation of the corporation brought about by their own wrong doing. Only
after such period of time had elapsed could respondent conclude that the directors
were remiss in their duty to protect the corporation property and business. The fraud
consisted in importing finished textile instead of raw cotton for the textile mill; the
fraud, therefore, was committed by the manager of the business and was consented to
by the directors, evidently beyond reach of respondent as treasurer for that period.
The directors permitted the fraudulent transaction to go unpunished and
nothing appears to have been done to remove the erring purchasing managers. In a
way the appointment of a receiver may have been thought of by the court below so
that the dollar allocation for raw material may be revived and the textile mill placed on
an operating basis.

199 | P a g e
Law 321_Corporation LAW_ Case Digest

CANDIDO PASCUAL
vs.
EUGENIO DEL SAZ OROZCO, ET AL.
G.R. No. L-5174. March 17, 1911

FACTS:

This action was brought by the plaintiff Pascual, in his own right as a
stockholder of the bank, for the benefit of the bank, and all the other stockholders
thereof. The Banco Español-Filipino is a banking corporation, constituted as such by
royal decree of the Crown of Spain in the year 1854, the original grant having been
subsequently extended and modified by royal decree of July 14, 1897, and by Act No.
1790 of the Philippine Commission.
It is alleged in the amended complaint that the only compensation
contemplated or provided for the managing officers of the bank was a certain per cent
of the net profits resulting from the bank's operations, as set forth in article 30 of its
reformed charter or statutes.
The gist of the first and second causes of action is as follows: The defendants
constitute a majority of the present board of directors of the bank, who alone can
authorize an action against them in the name of the corporation. It appears that
during the years 1903, 1904, 1905, and 1907 the defendants and appellees, without
the knowledge, consent, or acquiescence of the stockholders, deducted their respective
compensation from the gross income instead of from the net profits of the bank,
thereby defrauding the bank and its stockholders of approximately P20,000 per
annum.
The second cause of action sets forth that defendants' and appellees' immediate
predecessors in office in the bank during the years 1899, 1900, 1901, and 1902,
committed the same illegality as to their compensation as is charged against the
defendants themselves. In the four years immediately following the year 1902, the
defendants and appellees were the only officials or representatives of the bank who
could and should investigate and take action in regard to the sums of money thus
fraudulently appropriated by their predecessors. They were the only persons interested
in the bank who knew of the fraudulent appropriation by their predecessors.
The court below sustained the demurrer as to the first and second causes of action on
the ground that in actions of this character the plaintiff must aver in his complaint
that he was the owner of stock in the corporation at the time of the occurrences
complained of, or else that the stock has since devolved upon him by operation of law.

ISSUE:

Whether or not the petitioner has a cause of action to file a derivative suit.

RULING:

YES.

As to the first cause of action: In suits of this character the corporation itself and
not the plaintiff stockholder is the real party in interest. The rights of the individual
stockholder are merged into that of the corporation. It is a universally recognized
doctrine that a stockholder in a corporation has no title legal or equitable to the
corporate property; that both of these are in the corporation itself for the benefit of all
the stockholders. So it is clear that the plaintiff, by reason of the fact that he is a
stockholder in the bank (corporation) has a right to maintain a suit for and on behalf
of the bank, but the extent of such a right must depend upon when, how, and for what
purpose he acquired the shares which he now owns.
As to the Second cause of action: It affirmatively appears from the complaint
that the plaintiff was not a stockholder during any of the time in question in this
second cause of action. Upon the question whether or not a stockholder can maintain
a suit of this character upon a cause of action pertaining to the corporation when it
appears that he was not a stockholder at the time of the occurrence of the acts
complained of and upon which the action is based, the authorities do not agree.

200 | P a g e
Law 321_Corporation LAW_ Case Digest

POWERS OF CORPORATION
Theory of Special Capacities v. Theory of General Capacities

ACEBEDO OPTICAL COMPANY, INC.


vs.
THE HONORABLE COURT OF APPEALS, Hon. MAMINDIARA MANGOTARA, in his
capacity as Presiding Judge of the RTC, 12th Judicial Region, Br. 1, Iligan City;
SAMAHANG OPTOMETRIST Sa PILIPINAS — Iligan City Chapter, LEO T.
CAHANAP, City Legal Officer, and Hon. CAMILO P. CABILI, City Mayor of Iligan
G.R. No. 100152. March 31, 2000

FACTS:

Petitioner applied with the Office of the City Mayor of Iligan for a business
permit. After consideration of petitioner's application and the opposition interposed
thereto by local optometrists, respondent City Mayor issued Business Permit No. 5342
subject to the following conditions that since it is a corporation, Acebedo cannot put
up an optical clinic but only a commercial store; it cannot examine and/or prescribe
reading and similar optical glasses for patients, because these are functions of optical
clinics; it cannot sell reading and similar eyeglasses without a prescription having first
been made by an independent optometrist (not its employee) or independent optical
clinic and can only sell directly to the public, without need of a prescription, Ray-Ban
and similar eyeglasses; it cannot advertise optical lenses and eyeglasses, but can
advertise Ray-Ban and similar glasses and frames; and is allowed to grind lenses but
only upon the prescription of an independent optometrist.
Private respondent Samahan ng Optometrist Sa Pilipinas (SOPI), Iligan Chapter,
through its Acting President, Dr. Frances B. Apostol, lodged a complaint against the
petitioner before the Office of the City Mayor, alleging that Acebedo had violated the
conditions set forth in its business permit and requesting the cancellation and/or
revocation of such permit.

ISSUES:

Whether or not the act of the Respondent Mayor was lawful.

RULING:

NO.

The authority of city mayors to issue or grant licenses and business permits is
beyond cavil. However, the power to grant or issue licenses or business permits must
always be exercised in accordance with law, with utmost observance of the rights of all
concerned to due process and equal protection of the law. In the case under
consideration, the business permit granted by respondent City Mayor to petitioner was
burdened with several conditions.
Distinction must be made between the grant of a license or permit to do
business and the issuance of a license to engage in the practice of a particular
profession. The first is usually granted by the local authorities and the second is
issued by the Board or Commission tasked to regulate the particular profession. A
business permit authorizes the person, natural or otherwise, to engage in business or
some form of commercial activity. A professional license, on the other hand, is the
grant of authority to a natural person to engage in the practice or exercise of his or her
profession.
A business permit is issued primarily to regulate the conduct of business and
the City Mayor cannot, through the issuance of such permit, regulate the practice of a
profession, like that of optometry. Such a function is within the exclusive domain of
the administrative agency specifically empowered by law to supervise the profession,
in this case the Professional Regulations Commission and the Board of Examiners in
Optometry.

201 | P a g e
Law 321_Corporation LAW_ Case Digest

Express, Implied and Incidental Powers, Distinguished

PILIPINAS LOAN COMPANY, INC.


vs.
HON. SECURITES AND EXCHANGE COMMISSION AND FILIPINAS PAWNSHOP,
INC.
G.R. No. 104720. April 4, 2001

FACTS:

Private respondent Filipinas Pawnshop, Inc. is a duly organized corporation


registered with the Securities and Exchange Commission on February 9, 1959. The
articles of incorporation of private respondent states that its primary purpose is to
extend loans at legal interest on the security of either personal properties or on the
security of real properties, and to finance installment sales of motor vehicles, home
appliances and other chattels.
Petitioner is a lending corporation duly registered with the SEC on July 27,
1989. Based on its articles of incorporation, the primary purpose of petitioner is ―to
act as a lending investor or, otherwise, to engage in the practice of lending money or
extending loans on the security of real or personal, tangible or intangible properties
whether as pledge, real or chattel mortgage or otherwise, xxx without however,
engaging in pawnbroking as defined under PD 114."
Private respondent filed a complaint with the Prosecution and Enforcement
Department (PED) of the SEC and alleged that: (1) petitioner, contrary to the
restriction set by the Commission, has been operating and doing business as a
pawnbroker, pawnshop or "sanglaan" in the same neighborhood where private
respondent has had its own pawnshop for 30 years in violation of its primary purpose
and without the imprimatur of the Central Bank to engage in the pawnshop business
thereby causing unjust and unfair competition with private respondent. Petitioner
denied that it is engaged in the pawnshop business, alleging that it is a lending
investor duly registered with the Central Bank.

ISSUES:

Whether or not petitioner violated its primary franchise.

RULING:

YES.

A corporation, under the Corporation Code, has only such powers as are
expressly granted to it by law and by its articles of incorporation, those which may be
incidental to such conferred powers, those reasonably necessary to accomplish its
purposes and those which may be incident to its existence.
In the case at bar, the limit of the powers of petitioner as a corporation is very
clear, it is categorically prohibited from "engaging in pawnbroking as defined under PD
114". Hence, in determining what constitutes pawnbrokerage, the relevant law to
consider is PD 114.
Indispensable therefore to the determination of whether or not petitioner had
violated its articles of incorporation, was an inquiry by the SEC if petitioner was
holding out itself to the public as a pawnshop. It must be stressed that the
determination of whether petitioner violated PD 114 was merely incidental to the
regulatory powers of the SEC, to see to it that a corporation does not go beyond the
powers granted to it by its articles of incorporation.

202 | P a g e
Law 321_Corporation LAW_ Case Digest

LUNETA MOTOR COMPANY


vs.
A.D. SANTOS, INC., ET AL.
G.R. No. L-17716. July 31, 1962

FACTS:

On December 31, 1941, to secure payment of a loan evidenced by a promissory


note executed by Nicolas Concepcion and guaranteed by one Placido Esteban in favor
of petitioner, Concepcion executed a chattel mortgage covering the above mentioned
certificate in favor of petitioner.
Thereafter, he constituted a second mortgage on the same certificate to secure
payment of a subsequent loan obtained by Concepcion from the Rehabilitation
Finance Corporation (now Development Bank of the Philippines). This second
mortgage was approved by the respondent Commission, subject to the mortgage lien
in favor of petitioner. The certificate was later sold to Francisco Benitez, Jr., who
resold it to Rodi Taxicab Company. Both sales were made with assumption of the
mortgage in favor of the RFC, and were also approved provisionally by the
Commission, subject to petitioner's lien.
On October 10, 1953 petitioner filed an action to foreclose the chattel mortgage
executed in its favor by Concepcion. While the above case was pending, the RFC also
instituted foreclosure proceedings on its second chattel mortgage, and as a result of
the decision in its favor therein rendered, the certificate of public convenience was sold
at public auction in favor of Amador D. Santos for P24,010.00 on August 31, 1956.
Santos immediately applied with the Commission for the approval of the sale, and the
same was approved on January 26, 1957, subject to the mortgage lien in favor of
petitioner.

ISSUE:

Whether or not the purpose for which petitioner was organized and the
transaction of its lawful business reasonably and necessarily requires acquisition and
holds the certificate and operates as a common carrier by land.

RULING:

NO.

Under Section 13 (5) of the Corporation Law, a corporation created thereunder


may purchase, hold, etc., and otherwise deal in such real and personal property is the
purpose for which the corporation was formed may permit, and the transaction of its
lawful business may reasonably and necessarily require.
Petitioner‘s corporate purposes are to carry on a general mercantile and
commercial business, etc., and that it is authorized in its articles of incorporation to
operate and otherwise deal in and concerning automobiles and automobile
accessories' business in all its multifarious ramification and to operate, etc., and
otherwise dispose of vessels and boats, etc., and to own and operate steamship and
sailing ships and other floating craft and deal in the same and engage in the Philippine
Islands and elsewhere in the transportation of persons, merchandise and chattels by
water; all this incidental to the transportation of automobiles.
The Court finds that Petitoner‘s articles of incorporation are precisely the best
evidence that it has no authority at all to engage in the business of land
transportation and operate a taxicab service. That it may operate and otherwise deal
in automobiles and automobile accessories; that it may engage in the transportation of
persons by water does not mean that it may engage in the business of land
transportation — an entirely different line of business. If it could not thus engage in
the line of business, it follows that it may not acquire a certificate of public
convenience to operate a taxicab service, such as the one in question, because such
acquisition would be without purpose and would have no necessary connection with
petitioner's legitimate business.

203 | P a g e
Law 321_Corporation LAW_ Case Digest

TERESA ELECTRIC AND POWER CO., INC.


vs.
PUBLIC SERVICE COMMISSION and FILIPINAS CEMENT CORPORATION
G.R. No. L-21804. September 25, 1967

FACTS:

The petitioner Teresa Electric Light and Power Co., Inc. is a domestic
corporation operating an electric plant in Teresa, Rizal, under a subsisting certificate
of public convenience and necessity issued on June 2, 1960, while the respondent
Filipinas is likewise a domestic corporation engaged in the manufacture and sale of
cement.
On May 24, 1962, Filipinas filed an application with the Public Service
Commission for a certificate of public convenience to install, maintain and operate an
electric plant in sitio Kaysapon of barrio Pamanaan, municipality of Teresa, Rizal, for
the purpose of supplying electric power and light to its cement factory and its
employees living within its compound.
Petitioner opposed alleging that it is the duly authorized operator of an electric
light, heat and power service in Teresa, Rizal and that Filipinas is not authorized by its
articles of incorporation to operate an electric plant; that the Municipal Council of
Teresa had not authorized it either to operate the proposed service since Filipinas'
principal business does not come within the jurisdiction of the respondent
Commission.

ISSUES:

Whether or not under its articles of incorporation Filipinas is authorized to


operate and maintain an electric plant.

RULING:

YES.

The Articles of Incorporation of Filipinas (paragraph 7) provide for authority to


secure from any governmental, state, municipality, or provincial, city or other
authority, and to utilize and dispose of in any lawful manner, rights, powers,
privileges, franchises and concessions — obviously necessary or at least related to the
operation of its cement factory. Moreover, said Articles of Incorporation also provide
that the corporation may generally perform any and all acts connected with the
business of manufacturing Portland cement or arising therefrom or incidental thereto.
It cannot be denied that the operation of an electric light, heat and power plant is
necessarily connected with the business of manufacturing cement. Moreover, it has
been established in this case that petitioner was in no condition to supply the power
needs of Filipinas, because its load capacity was only 200 kilowatts while Filipinas
was in need of 6,000 Kilowatts power to operate its cement factory.

204 | P a g e
Law 321_Corporation LAW_ Case Digest

ANTHONY POWERS, BERTEL FASSNACHT, RICHARD I GUARDIAN, JOANN


KELLY, LANDLESS, AMADO MACASAET, JAVIER MACICIORATUSHI NAKAI KAY
NG, JAMES ROBERSON, FREDERICK SEGGERMAN, ARTHUR YANG, EZRA TOEG,
ISIDRO CO, In behalf of themselves and 316 other Associate Members all other
Associate Members similarly situated, and in behalf of and for the benefit of the
INTERNATIONAL SCHOOL, INC.
vs.
DONALD I. MARSHALL, CHARLES ANGEVINE, CARLOS D. ARGUELLES, BRYCE F.
BASTIAN, GABRIEL DIMACHE, JOSE FLORENTO, JAMES T. HODGE, ROSEMARY
IYAS, EUSEBIO R. LUZURIAGA, THOMAS C. NIBLOCK Board of Trustees of the
International School, Inc., and MAX SNYDER Superintendent, International
School, Inc.
G.R. No. L-48064. May 9, 1988

FACTS:

On July 16, 1975, the fourteen (14) plaintiffs, all associate members of the
International School, Inc., brought an action for injunction against the ten (10)
members of the Board of Trustees of the school, praying that said Trustees be enjoined
from collecting a "development fee" of P2,625.00 per child-enrollee per school year for
a period of twelve (12) years, beginning with the school year 1975-1976, as a pre-
requisite for re-enrollment in said school.
The suit was precipitated by a letter addressed to the parents of the students,
giving notice that the Board of Trustees had decided to embark on a program to
construct new buildings and remodel existing ones. The Board intended to raise the
needed funds primarily through subscriptions to capital notes and prepayment
certificates, and any deficiency from these sources would be covered by collecting a so-
called "development fees" of P2,625 from each enrollee starting with the school year
1975-1976 and continuing up to the school year 1986-1987.
The trial court issued an order temporarily restraining the defendants or their
authorized representatives and agents from executing and/or enforcing in any manner
the development program and after the submission of the parties' memoranda the trial
court issued an order dismissing the complaint for lack of valid cause of action

ISSUE:

Whether or not the Board of Trustees of the International School was


authorized to adopt the development plan for which the disputed fee was being
collected from the students.

RULING:

YES.

Section 2 of Article 3 of the By-Laws of the International School, Inc. provides:


The Board of Trustees, in addition to the powers conferred by these By-Laws, shall
have the right to such powers and do such acts as may be lawfully exercised or
performed by the corporation, subject to applicable laws and to the provisions of the
articles of incorporation and the By-Laws.
Section 2 (b) of P.D. No. 732 granting certain rights to the International School,
Inc., expressly authorized the Board of Trustees, upon consultation with the Secretary
of Education and Culture, to determine the amount of fees and assessments which
may be reasonably imposed upon its students, to maintain or conform to the school
standard of education." Such consultation had been made with the Secretary of
Education and Culture who expressed his conformity with the reasonableness of the
assessment of P2,625.00 per student for the whole school year to carry out its
development program. Since the collection of the development fee had been approved
by the Board of Trustees of the International School, Inc., it was a valid exercise of
corporate power by the Board, and said assessment was binding upon all the
members of the corporation.

205 | P a g e
Law 321_Corporation LAW_ Case Digest

Power to Have/Use Corporate Name and Seal

LAUREANO INVESTMENT & DEVELOPMENT CORPORATION


vs.
THE HONORABLE COURT OF APPEALS and BORMAHECO, INC.
G.R. No. 100468. May 6, 1997

FACTS:

The Spouses Reynaldo Laureano and Florence Laureano are majority


stockholders of petitioner Corporation who entered into a series of loan and credit
transactions with Philippine National Cooperative Bank. To secure payment of the
loans, they executed Deeds of Real Estate Mortgage dated December 11, 1962,
January 9, 1963, July 2, 1963 and September 5, 1964, for the following amounts:
P100,000.00, P20,000.00, P70,000.00 and P13,424.04, respectively. Spouses
Laureano failed to pay their indebtedness, thus PNCB applied for extrajudicial
foreclosure of the real estate mortgages. The bank was the purchaser of the properties
in question in the foreclosure sale and titles thereof were consolidated in PNCB's name
on February 20, 1984. PNCB did not secure a writ of possession nor did it file
ejectment proceedings against the Laureano spouses, because there were then
pending cases involving the titles of ownership of subject two lots, which are situated
at Bel-Air Subdivision, Makati, Metro Manila.
Private respondent Bormaheco, Inc. became the successor of the obligations
and liabilities of PNCB over subject lots by virtue of a Deed of Sale/Assignment.
Transfer Certificate of Title Nos. 157724 and 157725 over the lots in question were
issued on October 12, 1988 in the name of Bormaheco.
Five (5) days after securing titles over the said properties, Petitioner Corporation
filed on January 18, 1989 its Motion for Intervention and to Admit Attached Complaint
in Intervention in said. Respondent Bormaheco filed its Motion to Strike out the
Complaint in Intervention and all related pleadings filed by LIDECO Corporation. The
motion was granted stating that Intervenor LIDECO Corporation and LAUREANO
INVESTMENT AND DEVELOPMENT CORPORATION are two (2) separate and distinct
entities, therefore, no way whatsoever that LIDECO Corporation's interests will be
adversely affected by the outcome of the instant case. Thus, intervenor LIDECO lacks
personality to intervene in the instant

ISSUES:

Whether or not Respondent Bormaheco, Inc. is estopped from contesting the


legal personality to sue of "Lideco Corporation".

RULING:

NO.

Bad faith implies a conscious and intentional design to do a wrongful act for a
dishonest purpose or moral obliquity; bad faith contemplates a state of mind
affirmatively operating with furtive design or ill will.
Other than its bare allegations that private respondent acted in bad faith,
petitioner failed to show that the former acted consciously and deliberately to achieve
a dishonest purpose or moral obliquity, or was motivated by ill will. Rather, as
discussed above, no false representation was contrived nor concealment made by
private respondent. Neither did it deliberately convey facts other than, or inconsistent
with, what it now asserts and upon which petitioner had relied or acted upon due to
the representations of private respondent.

206 | P a g e
Law 321_Corporation LAW_ Case Digest

Power to Sue and be Sued

TAM WING TAK


vs.
HON. RAMON P. MAKASIAR (in his Capacity as Presiding Judge of the Regional
Trial Court of Manila, Branch 35) and ZENON DE GUIA (in his capacity as Chief
State Prosecutor)
G.R. No. 122452. January 29, 2001

FACTS:

On November 11, 1992, petitioner, in his capacity as director of Concord-World


Properties, Inc., (Concord for brevity), a domestic corporation, filed an affidavit-
complaint with the Quezon City Prosecutor's Office, charging Vic Ang Siong with
violation of B.P. Blg. 22 alleging that a check for the amount of P83,550,000.00,
issued by Vic Ang Siong in favor of Concord, was dishonored when presented for
encashment.
Vic Ang Siong sought the dismissal of the case on two grounds: First, that
petitioner had no authority to file the case on behalf of Concord, the payee of the
dishonored check, since the firm's board of directors had not empowered him to act on
its behalf. Second, he and Concord had already agreed to amicably settle the issue
after he made a partial payment of P19,000,000.00 on the dishonored check.
The City Prosecutor dismissed the case. Petitioner moved for reconsideration but the
City Prosecutor denied such. On November 8, 1994, petitioner appealed the dismissal
of his complaint and the Chief State Prosecutor dismissed the appeal for having been
filed out of time.

ISSUES:

Whether or not petitioner is the proper party to institute the case.

RULING:

NO.

In general, mandamus may be resorted to only where one's right is founded


clearly in law and not when it is doubtful. The exception is to be found in criminal
cases where mandamus is available to compel the performance by the public
prosecutor of an ostensibly discretionary function, where by reason of grave abuse of
discretion on his part, he willfully refuses to perform a duty mandated by law. Thus,
mandamus may issue to compel a prosecutor to file information when he refused to do
so in spite of the prima facie evidence of guilt.
First, with respect to the agreement between Concord and Victor Ang Siong to
amicably settle their difference, we find this resort to an alternative dispute settlement
mechanism as not contrary to law, public policy, or public order. Efforts of parties to
solve their disputes outside of the courts are looked on with favor, in view of the
clogged dockets of the judiciary.
Second, it is not disputed in the instant case that Concord, a domestic
corporation, was the payee of the bum check, not petitioner. Therefore, it is Concord,
as payee of the bounced check, which is the injured party. Since petitioner was neither
a payee nor a holder of the bad check, he had neither the personality to sue nor a
cause of action against Vic Ang Siong.
Petitioner failed to show any proof that he was authorized or deputized or
granted specific powers by Concord's board of director to sue Victor Ang Siong for and
on behalf of the firm. Petitioner as a minority stockholder and member of the board of
directors had no such power or authority to sue on Concord's behalf. Nor can we
uphold his act as a derivative suit. For a derivative suit to prosper, it is required that
the minority stockholder suing for and on behalf of the corporation must allege in his
complaint that he is suing on a derivative cause of action on behalf of the corporation
and all other stockholders similarly situated who may wish to join him in the suit.

207 | P a g e
Law 321_Corporation LAW_ Case Digest

NORA A. BITONG
vs.
COURT OF APPEALS (FIFTH DIVISION), EUGENIA D. APOSTOL, JOSE A.
APOSTOL, MR. & MS. PUBLISHING CO., LETTY J. MAGSANOC, AND ADORACION
G. NUYDA
G.R. No. 123553. July 13, 1998
NORA A. BITONG
vs.
COURT OF APPEALS (FIFTH DIVISION) and EDGARDO B. ESPIRITU
(CA-G.R. No. 33873) July 13, 1998

FACTS:

Bitong alleged that she was the treasurer and member of the BoD of Mr. & Mrs.
Corporation. She filed a complaint with the SEC to hold respondent spouses Apostol
liable for fraud, misrepresentation, disloyalty, evident bad faith, conflict of interest and
mismanagement in directing the affairs of the corporation to the prejudice of the
stockholders. She alleges that certain transactions entered into by the corporation
were not supported by any stockholder‘s resolution. The complaint sought to enjoin
Apostol from further acting as president-director of the corporation and from
disbursing any money or funds.
Apostol contends that Bitong was merely a holder-in-trust of the JAKA shares
of the corporation, hence, not entitled to the relief she prays for. SEC Hearing Panel
issued a writ enjoining Apostol. After hearing the evidence, SEC Hearing Panel
dissolved the writ and dismissed the complaint filed by Bitong. Bitong appealed to the
SEC en banc which reversed SEC Hearing Panel decision. Apostol filed petition for
review with the CA. CA reversed SEC en banc ruling holding that Bitong was not the
owner of any share of stock in the corporation and therefore, not a real party in
interest to prosecute the complaint.

ISSUE:

Whether or not Bitong was the real party in interest.

RULING:

NO.

It could be gleaned that Bitong was not a bona fide stockholder of the
corporation. Several corporate documents disclose that the true party in interest was
JAKA. Although her buying of the shares were recorded in the Stock and Transfer
Book of the corporation, and as provided by Sec. 63 of the Corp Code that no transfer
shall be valid except as between the parties until the transfer is recorded in the books
of the corporation, and upon its recording the corporation is bound by it and is
estopped to deny the fact of transfer of said shares, this provision is not conclusive
even against the corporation but are prima facie evidence only.
Parol evidence may be admitted to supply the omissions in the records, explain
ambiguities, or show what transpired where no records were kept, or in some cases
where such records were contradicted.
The certificate of stock itself once issued is a continuing affirmation or
representation that the stock described therein is valid and genuine and is at least
prima facie evidence that it was legally issued in the absence of evidence to the
contrary. However, this presumption may be rebutted. However, the books and
records of a corporation are not conclusive even against the corporation but are prima
facie evidence only. The effect of entries in the books of the corporation which purport
to be regular records of the proceedings of its board of directors or stockholders can be
destroyed by testimony of a more conclusive character than mere suspicion that there
was an irregularity in the manner in which the books were kept.

208 | P a g e
Law 321_Corporation LAW_ Case Digest

SPECIAL SERVICES CORPORATION


vs.
CENTRO LA PAZ (SAMAHANG ESPIRITISTA SA LUNDUYANG LA PAZ), A CHAPTER
OF UNION ESPIRITISTA CRISTIANA DE FILIPINAS, INC.
G.R. No. L-44100. April 28, 1983

FACTS:

On October 10, 1972, judgment was rendered in favor of petitioner against one
Alejandro Estudillo in the amount of P94,727.52, in an action for Replevin with Sum
of Money and a writ of execution was thereafter issued but which has remained
unsatisfied.
By virtue of an alias writ of execution issued on December 15, 1972, the Sheriff
of Manila caused the annotation of a notice of levy on Transfer Certificate of Title No.
51837, in respect of the rights, interest and participation of said Alejandro Estudillo,
one of the registered owners indicated in said title.
On July 23, 1973, "Centro La Paz (Samahang Espiritista sa Lunduyang La Paz)
a Chapter of Union Espiritista Cristiana de Filipinas, Inc.," as plaintiff, instituted for
Damages and Preliminary Injunction against herein petitioner and the Sheriff of
Manila with the Court of First Instance, Branch IV, Manila, the same Court which
rendered judgment in the replevin case. CENTRO reiterated ownership of the
properties in question and emphasized that the registered owners thereof had publicly
acknowledged their possession of said properties in the concept of trustees.

ISSUE:

Whether or not Centro La Paz which is merely a Chapter of Union Espiritista de


Filipinas, Inc. has a juridical personality of its own in accordance with the provisions
of our laws.

RULING:

YES.

Although it was CENTRO that was actively prosecuting the case, in substance,
it was representing the mother organization, the Union Espiritista Cristiana de
Filipinas, Inc., which is the real party in interest and is itself named in the Complaint.
It is an organization that is duly registered with the Securities and Exchange
Commission, and thus possessed of a juridical personality to sue and be sued.
Admittedly, the trust was not registered in accordance with section 65 of Act
496 (the former Land Registration Law). The absence of said registration, however,
cannot be taken against CENTRO inasmuch as, if the public auction sale had actually
been held, with petitioner as the successful buyer, petitioner could not have been
considered a purchaser for value and in good faith at said sale since it had knowledge
of CENTRO's claim, particularly when the latter had filed a third-party-claim with the
Sheriff of Manila before the scheduled auction sale, which knowledge was equivalent
to registration of the several "Acknowledgments" in the Registry of Deeds.
The conclusion follows that inasmuch as Estudillo has no interest in the
properties in question, there is nothing that petitioner can levy upon. The power of a
Court in the execution of its judgment extends only over properties unquestionably
belonging to the judgment debtor.

209 | P a g e
Law 321_Corporation LAW_ Case Digest

R. TRANSPORT CORPORATION
vs.
HON. COURT OF APPEALS, Former 15th Division, Manila, HON. SALVADOR S.
ABAD SANTOS, as Presiding Judge, Regional Trial Court of, Metro Manila, Branch
65 and FLOSERIDA L. CASTAÑEDA
G.R. No. 111187. February 1, 1995

FACTS:

On November 22, 1991, private respondent filed a complaint for damages


arising from breach of contract of carriage against petitioner. In an Order dated
January 28, 1991, the trial court upon ex parte motion of private respondent, declared
petitioner in default and appointed a commissioner to receive evidence ex parte.
Petitioner filed a Motion to Dismiss and to Stop Ex Parte Reception of Evidence. It
asserted that it was not properly served with summons and consequently, the trial
court did not acquire jurisdiction over its person. It argued that none of the officers
enumerated in Section 13, Rule 14 of the Revised Rules of Court (namely, the
corporation's president, manager, secretary, cashier, agent or any of its directors)
received any summons. The trial court denied petitioner's motion and allowed private
respondent to adduce its evidence ex parte.
The Court of Appeals dismissed the petitioner‘s petition for certiorari ruling that
the trial court did not commit any grave abuse of discretion in declaring the petitioner
in default and in denying petitioner's motion for reconsideration.

ISSUE:

Whether or not there was valid service of summons.

RULING:

YES.

As a general rule, service of summons must be made on the persons named in


Section 13, Rule 14 of the Revised Rules of Court which provides: If the defendant is a
corporation organized under the laws of the Philippines or a partnership duly
registered, service may be made on the president, manager, secretary, cashier, agent
or any of its directors.
Thus service on persons other than those mentioned in said Rule has been held
as improper. Through the years, the rule on service of summons has been liberalized.
Such liberalization is to give life to the rationale behind Section 13 of Rule 14. Service
of summons on persons other than those enumerated in Section 13 of Rule 14 have
been held proper on the theory that those persons served were holding positions of
responsibility and could appreciate the importance of the papers handed them, and
could be expected to deliver the papers to the proper officer. These individuals were
considered "agents" within the contemplation of Section 13 of Rule 14. Thus, the
Court holds that service of summons on petitioner's Operations Manager was valid. He
is an officer who may be relied upon to appreciate the importance of the papers served
on him.
The fact that service was made at petitioner's bus terminal at the address
stated in the summons and not at its office in Makati does not render the service of
summons invalid. Petitioner is engaged in the transportation business, operating over
100 buses. Its central bus terminal is located at Sucat, Parañaque, from where it
conducts the bulk of its business. It was at that terminal where petitioner's Operations
Manager was found and upon whom service was made.

210 | P a g e
Law 321_Corporation LAW_ Case Digest

Power to Acquire, Dispose, Encumber Property

THE DIRECTOR OF LANDS


vs.
THE HONORABLE COURT OF APPEALS and IGLESIA NI CRISTO
G.R. No. L-56613. March 14, 1988

FACTS:

On November 28, 1973, private respondent Iglesia ni Cristo filed an application


for registration in its name of a parcel of land with an area of 379 square meters
located at Poblacion, Municipality of Amadeo, Cavite. In its application, private
respondent alleged inter alia that it was the owner in fee simple of the land afore-
described, having acquired title thereto by virtue of a Deed of Absolute Sale executed
in 1947 by Aquelina de la Cruz in its favor and that applicant. Private respondent
prayed that should the Land Registration Act not be applicable, the provisions of
Chapter VIII of Commonwealth Act No. 141, as amended by Republic Act No. 6236 be
applied as applicant and its predecessors-in-interest had been in possession of the
land for more than thirty [30] years and had introduced improvements thereon.
The Republic of the Philippines, represented by the Director of Lands, opposed
the application on the following grounds: 1] the applicant and its predecessors-in-
interest did not possess sufficient title to acquire ownership in fee simple of the parcel
of land applied for; 2] neither the applicant nor its predecessors-in-interest have been
in open, continuous, exclusive and notorious possession and occupation of the land in
question; and, 3] the subject parcel of land is a portion of the public domain not
subject to private appropriation.

ISSUES:

Whether or not the respondent is prohibited from acquiring private land as


provided under the Constitution.

RULING:

YES.

Taking the year 1936 as the reckoning point, the 30-year period of open,
continuous, exclusive and notorious possession and occupation required by law was
completed in 1966. The completion by private respondent of this statutory 30-year
period has dual significance in the light of Section 48[b] of Commonwealth Act No.
141, as amended and prevailing jurisprudence: [1] at this point, the land in question
ceased by operation of law to be part of the public domain; and [2] private respondent
could have its title thereto confirmed through the appropriate proceedings as under
the Constitution then in force, private corporations or associations were not prohibited
from acquiring public lands, but merely prohibited from acquiring, holding or leasing
such type of land in excess of 1,024 hectares.
If in 1966, the land in question was converted ipso jure into private land, it
remained so in 1974 when the registration proceedings were commenced. This being
the case, the prohibition under the 1973 Constitution would have no application.
Otherwise construed, if in 1966, private respondent could have its title to the land
confirmed, then it had acquired a vested right thereto, which the 1973 Constitution
can neither impair nor defeat.

211 | P a g e
Law 321_Corporation LAW_ Case Digest

Power to Make Donations

MARIA CLARA PIROVANO ET AL.


vs.
THE DE LA RAMA STEAMSHIP CO.
G.R. No. L-5377. December 29, 1954

FACTS:

Plaintiffs herein are the minor children of the late Enrico Pirovano represented
by their mother and judicial guardian Estefania R. Pirovano. They seek to enforce
certain resolutions adopted by the Board of Directors and stockholders of the
defendant company giving to said minor children of the proceeds of the insurance
policies taken on the life of their deceased father Enrico Pirovano with the company as
beneficiary. Defendant's main defense is: that said resolutions and the contract
executed pursuant thereto are ultra vires, and, if valid, the obligation to pay the
amount given is not yet due and demandable.
Plaintiff-appellant Pirovano is the owner of 3424 shares of stocks in defendant-
appellee Corporation which declared a dividend of P100 per share. Appellant wants to
recover from appellee the sum of P221, 975 after deducting the sum of P120, 424
which she had withdrawn or received from appellee for advances she received after the
death of her father, the late Esteban de la Rama.
Appellant‘s theory is that the cash advances to her for her personal use and
that of her children were assumed by Esteban de la Rama. She claims that the
advances made to her by appellees were debited from the account of Hijos de I. de la
Rama, another corporation practically owned by Esteban de la Rama. She further
claims that the appellee can only deduct from the amount of dividend she is entitled
to, the amount of cash advances which was not assumed by her father. The
withdrawals by the appellant were made during the period 1940 to 1949 during which
the appellee made a deed of trust with Hijos. The deed of trust was made to
circumvent the prohibition of declaring dividends during the period.

ISSUE:

Whether or not the donation made by the corporation of the proceeds of the
insurance is a valid act.

RULING:

YES.

The Articles of Incorporation of Dela Rama Steamship provided that under (g)
the company may invest and deal with moneys of the company not immediately
required, in such a manner as from time to time may be determined, and under (i)… to
lend money or to aid in any other manner any person association, or corporation of
which any obligation or in which any interest is held by the corporation or in the
affairs of prosperity of which the corporation has a lawful interest.
The corporation was thus given broad and almost unlimited powers to carry out
the purposes for which it was organized. The word ―deal‖ is broad enough to include
any manner of disposition, and thus the donation comes within the scope of this
broad power. The company was in fact very much solvent as it was able to declare and
issue dividends to its stockholders, and shows that the excess funds which were not
needed by the company which was donated to the children was justified under the
AOI. Under the second broad power, the corporation knew well its scope such that
none lifted a finger to dispute its validity. The company gave the donation not only
because it was indebted to him but also because it was fit and proper to make
provisions for the children and out of a sense of gratitude.

212 | P a g e
Law 321_Corporation LAW_ Case Digest

To Increase or Decrease Capital Stock

MADRIGAL & COMPANY, INC.


vs.
HON. RONALDO B. ZAMORA, PRESIDENTIAL ASSISTANT FOR LEGAL AFFAIRS,
THE HON. SECRETARY OF LABOR, and MADRIGAL CENTRAL OFFICE
EMPLOYEES UNION
G.R. No. L-48237. June 30, 1987

MADRIGAL & COMPANY, INC.


vs.
HON. MINISTER OF LABOR and MADRIGAL CENTRAL OFFICE EMPLOYEES
UNION
No. L-49023. June 30, 1987

FACTS:

The petitioner was engaged, among several other corporate objectives, in the
management of Rizal Cement Co., Inc.Admittedly, the petitioner and Rizal Cement Co.,
Inc. are sister companies.Both are owned by the same or practically the same
stockholders.On December 28, 1973, the respondent, the Madrigal Central Office
Employees Union, sought for the renewal of its collective bargaining agreement with
the petitioner, which was due to expire on February 28, 1974.Specifically, it proposed
a wage increase of P200.00 a month, an allowance of P100.00 a month, and other
economic benefits.The petitioner, however, requested for a deferment in the
negotiations.
On July 29, 1974, by an alleged resolution of its stockholders, the petitioner
reduced its capital stock from 765,000 shares to 267,366 shares.This was effected
through the distribution of the marketable securities owned by the petitioner to its
stockholders in exchange for their shares in an equivalent amount in the
corporation.On August 22, 1975, by yet another alleged stockholders' action, the
petitioner reduced its authorized capitalization from 267,366 shares to 110,085
shares, again, through the same scheme.

ISSUE:

Whether or not the decrease in the ACS of petitioner is valid.

RULING:

NO.

The Court ruled that what clearly emerges from the recorded facts is that the
petitioner, awash with profits from its business operations but confronted with the
demand of the union for wage increases, decided to evade its responsibility towards
the employees by a devised capital reduction. While the reduction in capital stock
created an apparent need for retrenchment, it was, by all indications, just a mask for
the purge of union members, who, by then, had agitated for wage increases.
As such shareholder, the dividends paid to it were its own money, which may
then be available for wage increments. It is not a case of a corporation distributing
dividends in favor of its stockholders, in which case, such dividends would be the
absolute property of the stockholders and hence, out of reach by creditors of the
corporation. Here, the petitioner was acting as stockholder itself, and in that case, the
right to a share in such dividends, by way of salary increases, may not be denied its
employees.
Accordingly, the Court is convinced that the petitioner's capital reduction efforts
were, to begin with, a subterfuge, a deception as it were, to camouflage the fact that it
had been making profits, and consequently, to justify the mass layoff in its employee
ranks, especially of union members.

213 | P a g e
Law 321_Corporation LAW_ Case Digest

PHILIPPINE TRUST COMPANY, as assignee in insolvency of "La Cooperativa


Naval Filipina"
vs.
MARCIANO RIVERA
G.R. No. L-19761. January 29, 1923

FACTS:

In 1918 the Cooperativa Naval Filipina was duly incorporated under the laws of
the Philippine Islands, with a capital of P100,000, divided into one thousand shares of
a par value of P100 each. Among the incorporators of this company was numbered the
defendant Mariano Rivera, who subscribed for 450 shares representing a value of
P45,000, the remainder of the stock being taken by other persons.
In the course of time the company became insolvent and went into the hands of
the Philippine Trust Company, as assignee in bankruptcy; and by it this action was
instituted to recover one-half of the stock subscription of the defendant, which
admittedly has never been paid.
The reason given for the failure of the defendant to pay the entire subscription
is a meeting of its stockholders occurred, at which a resolution was adopted to the
effect that the capital should be reduced by 50 per centum and the subscribers
released from the obligation to pay any unpaid balance of their subscription in excess
of 50 per centum of the same.

ISSUE:

Whether or not the reduction of the company‘s capital by 50 per centum and
the subscribers released from the obligation to pay any unpaid balance of their
subscription in excess of 50 per centum is valid.

RULING:

NO.

The Court ruled that defendant was still liable for the unpaid balance of his
subscription. It is established doctrine that subscription to the capital of a corporation
constitute a find to which creditors have a right to look for satisfaction of their claims
and that the assignee in insolvency can maintain an action upon any unpaid stock
subscription in order to realize assets for the payment of its debts.
A corporation has no power to release an original subscriber to its capital stock
from the obligation of paying for his shares, without a valuable consideration for such
release; and as against creditors a reduction of the capital stock can take place only in
the manner an under the conditions prescribed by the statute or the charter or the
articles of incorporation. Moreover, strict compliance with the statutory regulations is
necessary. In the case before us the resolution releasing the shareholders from their
obligation to pay 50 per centum of their respective subscriptions was an attempted
withdrawal of so much capital from the fund upon which the company's creditors were
entitled ultimately to rely and, having been effected without compliance with the
statutory requirements, was wholly ineffectual.

214 | P a g e
Law 321_Corporation LAW_ Case Digest

To Deny Pre-Emptive Rights

DATU TAGORANAO BENITO


vs.
SECURITIES AND EXCHANGE COMMISSION and JAMIATUL PHILIPPINE-AL
ISLAMIA, INC.
G.R. No. L-56655. July 25, 1983

FACTS:

On February 6, 1959, the Articles of Incorporation of respondent Jamiatul


Philippine-Al Islamia, Inc. (originally Kamilol Islam Institute, Inc.) were filed with the
Securities and Exchange Commission (SEC) and were approved on December 14,
1962. The corporation had an authorized capital stock of P200,000.00 divided into
20,000 shares at a par value of P10.00 each. Of the authorized capital stock, 8,058
shares worth P80,580.00 were subscribed and fully paid for. Petitioner Datu
Tagoranao Benito subscribed to 460 shares worth P4,600.00.
On October 28, 1975, the respondent corporation filed a certificate of increase
of its capital stock from P200,000.00 to P1,000,000.00. Thus, P110,980.00 worth of
shares were subsequently issued by the corporation from the unissued portion of the
authorized capital stock of P200,000.00. Of the increased capital stock of
P1,000,000.00, P160,000.00 worth of shares were subscribed by Mrs. Fatima A.
Ramos, Mrs. Tarhata A. Lucman and Mrs. Moki-in Alonto.
Petitioner Datu Tagoranao filed a petition alleging that the additional issue
(worth P110,980.00) of previously subscribed shares of the corporation was made in
violation of his pre-emptive right to said additional issue and that the increase in the
authorized capital stock of the corporation from P200,000.00 to P1,000,000.00 was
illegal considering that the stockholders of record were not notified of the meeting
wherein the proposed increase was in the agenda.
Respondents denied the material allegations of the petition and claimed that
petitioner has no cause of action and that the stock certificates covering the shares
alleged to have been sold to petitioner were only given to him as collateral for the loan
of Domocao Alonto and Moki-in Alonto. The SEC affirmed the sale.

ISSUE:

Whether or not the issuance of the unissued shares was subject to the pre-
emptive right of the stockholders.

RULING:

NO.

The Court held that the questioned issuance of the unsubscribed portion of the
capital stock worth P110,980.00 is not invalid even if assuming that it was made
without notice to the stockholders as claimed by petitioner. The power to issue shares
of stocks in a corporation is lodged in the board of directors and no stockholders'
meeting is necessary to consider it because additional issuance of shares of stocks
does not need approval of the stockholders.
Petitioner bewails the fact that in view of the lack of notice to him of such
subsequent issuance, he was not able to exercise his right of pre-emption over the
unissued shares. However, the general rule is that pre-emptive right is recognized only
with respect to new issue of shares, and not with respect to additional issues of
originally authorized shares. This is on the theory that when a corporation at its
inception offers its first shares, it is presumed to have offered all of those which it is
authorized to issue. An original subscriber is deemed to have taken his shares
knowing that they form a definite proportionate part of the whole number of
authorized shares. When the shares left unsubscribed are later re-offered, he cannot
therefore claim a dilution of interest.

215 | P a g e
Law 321_Corporation LAW_ Case Digest

PEDRO LOPEZ DEE


vs.
SECURITIES AND EXCHANGE COMMISSION, HEARING OFFICER EMMANUEL
SISON, NAGA TELEPHONE CO., INC., COMMUNICATION SERVICES, INC.,
LUCIANO MAGGAY, AUGUSTO FEDERIS, NILDA RAMOS, FELIPA JAVALERA,
DESIDERIO SAAVEDRA
G.R. No. L-60502. July 16, 1991

JUSTINO DE JESUS, SR., PEDRO LOPEZ DEE, JULIO LOPEZ DEE, and VICENTE
TORDILLA, JR.
vs.
INTERMEDIATE APPELLATE COURT, LUCIANO MAGGAY, NILDA I. RAMOS,
DESIDERIO SAAVEDRA, AUGUSTO FEDERIS, ERNESTO MIGUEL,
COMMUNICATION SERVICES, INC., and NAGA TELEPHONE COMPANY, INC.
G.R. No. L-63922. July 16, 1991

FACTS:

Naga Telephone Company, Inc. was organized in 1954, the authorized capital
was P100,000.00. In 1974 Naga Telephone Co., Inc. (Natelco) decided to increase its
authorized capital to P3,000,000.00. As required by the Public Service Act, Natelco
filed an application for the approval of the increased authorized capital with the then
Board of Communications on which a decision was rendered approving the application
subject to certain conditions, among which was: That the issuance of the shares of
stocks will be for a period of one year from the date hereof, "after which no further
issues will be made without previous authority from this Board."
Natelco filed its Amended Articles of Incorporation with the Securities and
Exchange Commission. When the amended articles were filed with the SEC, the
original authorized capital of P100,000.00 was already paid. Of the increased capital of
P2,900,000.00 the subscribers subscribed to P580,000.00 of which P145,000 was
fully paid.

ISSUE:

Whether or not Natelco stockholders have a right of preemption to the 113,800


shares in question.

RULING:

NO.

The general rule is that pre-emptive right is recognized only with respect to new
issues of shares, and not with respect to additional issues of originally authorized
shares. This is on the theory that when a corporation at its inception offers its first
shares, it is presumed to have offered all of those which it is authorized to issue. An
original subscriber is deemed to have taken his shares knowing that they form a
definite proportionate part of the whole number of authorized shares. When the shares
left unsubscribed are later re-offered, he cannot therefore claim a dilution of interest.
Thus, the questioned issuance of the 113,800 stocks is not invalid even assuming that
it was made without notice to the stockholders as claimed by the petitioner. The power
to issue shares of stocks in a corporation is lodged in the board of directors and no
stockholders meeting is required to consider it because additional issuance of shares
of stocks does not need approval of the stockholders. Consequently, no pre-emptive
right of Natelco stockholders was violated by the issuance of the 113,800 shares to
CSI.
Accordingly, it is clear that since the trial judge in the lower court did not have
jurisdiction in issuing the questioned restraining order, disobedience thereto did not
constitute contempt, as it is necessary that the order be a valid and legal one. It is an
established rule that the court has no authority to punish for disobedience of an order
issued without authority.

216 | P a g e
Law 321_Corporation LAW_ Case Digest

PRESIDENTIAL COMMISSION ON GOOD GOVERNMENT


vs.
HON. BENJAMIN M. AQUINO, JR., as Presiding Judge, Regional Trial Court, NCJR
Branch LXXII Malabon, Metro Manila, and MARCELO FIBERGLASS
CORPORATION
G.R. No. 77816. June 30, 1988

MARCELO FIBERGLASS CORPORATION


vs.
PRESIDENTIAL COMMISSION ON GOOD GOVERNMENT
G.R. No. 78753. June 30, 1988

FACTS:

On June 18, 1982, Edward T. Marcelo as president of private respondent


Marcelo Fiberglass Corporation [hereinafter referred to as MFC] entered into a Contract
to Buy and Sell with the Philippine Navy represented by Rear Admiral Simeon M.
Alejandro, then Flag Officer in Command, for the construction and delivery by the
former of fifty-five 551 units of fiberglass high-speed patrol boats at P7,200,000 each
plus spare parts amounting to P29,700,000 for a total contract price of P425,700,000.
It was stipulated in the contract that the patrol boats would be delivered within thirty-
six [36] months from the date the Philippine Navy pays to private respondent the
stipulated down payment of thirty per cent [30%] of the contract price.
To facilitate funding of the initial down payment agreed upon under the
contract, MFC through Edward Marcelo, secured presidential approval for the
issuance of a guarantee 'by the national government in acquiring either a foreign
currency loan in behalf of the Philippine Navy with a foreign bank or offshore banking
unit, or a peso term loan to be negotiated by the Philippine National Bank, also, in
behalf of the Philippine Navy.

ISSUES:

Whether or not the issuance and implementation of the writ of


sequestration violates the constitutional rights of private respondent against
impairment of obligation of contracts and deprivation of property without due process
of law.

RULING:

NO.

The Court sustains petitioner's stand and holds that regional trial courts and
the Court of Appeals for that matter have no jurisdiction over the Presidential
Commission on Good Government in the exercise of its powers under the applicable
Executive Orders and Article XVIII, Section 26 of the Constitution and therefore may
not interfere with and restrain or set aside the orders and actions of tile Commission.
Under Section 2 of the President's Executive Order No. 14 issued on May 7, 1986, all
cases of the Commission regarding "the Funds, Moneys, Assets and Properties Illegally
Acquired or Misappropriated by Former President Ferdinand Marcos, Mrs. Imelda
Romualdez Marcos, then, Close Relatives, Subordinates, Business Associate,
Dummies, Agents or Nominees' whether civil or criminal.
The attempt to remove special civil actions from the Sandiganbayan's exclusive
jurisdiction is of no avail if they similarly involve the powers and functions of the
Presidential Commission on Good Government.
The matters involved in these cases are orders of the PCGG issued in the
exercise of its powers and functions for they involve the sequestration of the assets of
private respondent Marcelo Fiberglass Corporation and Edward T. Marcelo, its
president. The propriety of said sequestration and any incident arising from, incidental
or related to such sequestration is within the exclusive jurisdiction of the
Sandiganbayan.

217 | P a g e
Law 321_Corporation LAW_ Case Digest

REPUBLIC OF THE PHILIPPINES


vs.
SANDIGANBAYAN (3RD DIVISION), JOSE L. AFRICA, UNIMOLCO, ROBERTO
BENEDICTO, ANDRES AFRICA and SMART COMMUNICATIONS
G.R. No. 128606. December 4, 2000

FACTS:

Eastern Telecommunications Philippines, Inc. (ETPI) was one of the


corporations sequestered by the Presidential Commission on Good Government
(PCGG). Among its stockholders were Roberto S. Benedicto and Universal Molasses
Corporation (UNIMOLCO).
Sometime in 1990, PCGG and Benedicto entered into a compromise agreement
whereby Benedicto ceded to the government 204,000 shares of stock in ETPI,
representing his fifty-one percent (51%) equity therein. The other forty-nine percent
(49%), consisting of 196,000 shares of stock, were released from sequestration and
adjudicated by final judgment to Benedicto and UNIMOLCO. Furthermore, the
government agreed to withdraw the cases filed against Benedicto and free him from
further criminal prosecution.
Meanwhile, on motion of petitioner, through the PCGG, the Sandiganbayan
issued a Resolution, dated May 7, 1996, authorizing the entry in the Stock and
Transfer Book of ETPI of the transfer of ownership of 204,000 shares of stock to
petitioner, to be taken out of the shareholdings of UNIMOLCO.
PCGG issued Resolution No. 96-142 enjoining all stockholders of ETPI from
selling shares of stock therein without the written conformity of the PCGG.
Subsequently, on July 24, 1996, UNIMOLCO and Smart Communications executed a
Deed of Absolute Sale whereby UNIMOLCO sold its 196,000 shares of stock in ETPI to
Smart. Prior to the sale, Smart was not a stockholder of ETPI.

ISSUE:

Whether or not petitioner was denied of his pre-emptive right because of the
defective notice.

RULING:

NO.

The records of the case clearly show that the written notice by UNIMOLCO, the
Offeror, of its intention to sell its 196,000 shares of stock was duly received on April
24, 1996 by the President and Chairman of the Board of ETPI.
Moreover, the purpose of the notice requirement in Article 10 of the ETPI
Articles of Incorporation is to give the stockholders knowledge of the intended sale of
shares of stock of the corporation, in order that they may exercise their preemptive
right. Where it is shown that a stockholder had actual knowledge of the intended sale
within the period prescribed to exercise the right, the notice requirement had been
sufficiently met.
In the case at bar, PCGG had actual knowledge of UNIMOLCO‘s offer to sell its
shares of stock. In fact, it issued Resolution No. 96-142 enjoining the sale of the said
shares of stock to Smart. Petitioner, thus, cannot feign lack of notice. PCGG had no
more authority to enjoin the sale of UNIMOLCO‘s 196,000 shares of stock, as it
endeavored to do in Resolution No. 96-142. As correctly found by the Sandiganbayan,
since the 196,000 shares of stock had already been adjudicated by final judgment to
Benedicto and UNIMOLCO, PCGG could no longer exercise power and authority over
the same.

218 | P a g e
Law 321_Corporation LAW_ Case Digest

To Sell Or Otherwise Dispose Of All or Substantially All Of Corporate Assets

PHILIPPINE NATIONAL BANK & NATIONAL SUGAR DEVELOPMENT


CORPORATION
vs.
ANDRADA ELECTRIC & ENGINEERING COMPANY
G.R. No. 142936. April 17, 2002

FACTS:

The Plaintiff (herein respondent) alleged that it is a partnership duly organized,


existing, and operating under the laws of the Philippines, while the herein petitioner
Philippine National Bank (PNB), is a semi-government corporation duly organized,
existing and operating under the laws of the Philippines; whereas, the other
defendant, the National Sugar Development Corporation (NASUDECO), is also a semi-
government corporation and the sugar arm of the PNB; and the defendant Pampanga
Sugar Mills (PASUMIL), is a corporation organized, existing and operating under the
1975 laws of the Philippines.
The respondent is engaged in the business of general construction for the
repairs and/or construction of different kinds of machineries and buildings. On
August 26, 1975, the defendant PNB acquired the assets of the defendant PASUMIL
that were earlier foreclosed by the Development Bank of the Philippines (DBP) under
LOI No. 311. But prior to October 29, 1971, the defendant PASUMIL engaged the
services of plaintiff for electrical rewinding and repair, most of which were partially
paid by the defendant PASUMIL, leaving several unpaid accounts with the plaintiff;
that finally, on October 29, 1971, the plaintiff and the defendant PASUMIL entered
into a contract.
Out of the total obligation of P777,263.80, the defendant PASUMIL had paid
only P264,000.00, leaving an unpaid balance of P513,263.80. Petitioners herein failed
and refused to pay the plaintiff their just, valid and demandable obligation; that the
President of the NASUDECO is also the Vice-President of the PNB, inasmuch as PNB
and NASUDECO now owned and possessed the assets of the defendant PASUMIL.
Accordingly, the plaintiff prayed that judgment be rendered against the defendants
PNB, NASUDECO, and PASUMIL, jointly and severally.

ISSUE:

Whether or not petitioners should be held liable for the corporate debts of
PASUMIL for taking over of the latter‘s foreclosed assets.

RULING:

NO.

As a rule, a corporation that purchases the assets of another will not be liable
for the debts of the selling corporation, provided the former acted in good faith and
paid adequate consideration for such assets, except when any of the following
circumstances is present: (1) where the purchaser expressly or impliedly agrees to
assume the debts, (2) where the transaction amounts to a consolidation or merger of
the corporations, (3) where the purchasing corporation is merely a continuation of the
selling corporation, and (4) where the transaction is fraudulently entered into in order
to escape liability for those debts.
Equally well-settled is the principle that the corporate mask may be removed or
the corporate veil pierced when the corporation is just an alter ego of a person or of
another corporation. For reasons of public policy and in the interest of justice, the
corporate veil will justifiably be impaled only when it becomes a shield for fraud,
illegality or inequity committed against third persons.

219 | P a g e
Law 321_Corporation LAW_ Case Digest

ISLAMIC DIRECTORATE OF THE PHILIPPINES, MANUEL F. PEREA and


SECURITIES & EXCHANGE COMMISSION
vs.
COURT OF APPEALS and IGLESIA NI CRISTO
G.R. No. 117897. May 14, 1997

FACTS:

Petitioner IDP-Tamano Group alleges that sometime in 1971, Islamic leaders of


all Muslim major tribal groups in the Philippines headed by Dean Cesar Adib Majul
organized and incorporated the ISLAMIC DIRECTORATE OF THE PHILIPPINES (IDP),
the primary purpose of which is to establish an Islamic Center in Quezon City for the
construction of a "Mosque (prayer place), Madrasah (Arabic School), and other
religious infrastructures" so as to facilitate the effective practice of Islamic faith in the
area.
The Libyan government donated money to the IDP to purchase land at Culiat,
Tandang Sora, Quezon City, to be used as a Center for the Islamic populace. The land,
with an area of 49,652 square meters, was covered by two titles: Transfer Certificate of
Title Nos. RT-26520 (176616) and RT-26521 (170567), both registered in the name of
IDP.
According to the petitioner, in 1972, after the purchase of the land by the
Libyan government in the name of IDP, Martial Law was declared by the late President
Ferdinand Marcos. Thereafter, two Muslim groups sprung, the Carpizo Group and the
Abbas Group, both groups claimed to be the legitimate IDP. Significantly, on October
3, 1986, the SEC, in a suit between these two contending groups, came out with a
Decision in SEC Case No. 2687 declaring the election of both the Carpizo Group and
the Abbas Group as IDP board members to be null and void.

ISSUE:

Whether or not the Deed of Sale executed by Carpizo Group is valid.

RULING:

NO.

This is precisely what the SEC did in SEC Case No. 4012 when it adjudged the
election of the Carpizo Group to the IDP Board of Trustees to be null and void.
Consequently, the Carpizo Group is bereft of any authority whatsoever to bind IDP in
any kind of transaction including the sale or disposition of ID property.
Nothing thus becomes more settled than that the IDP-Carpizo Group with
whom private respondent INC contracted is a fake Board. Premises considered, all acts
carried out by the Carpizo Board, particularly the sale of the Tandang Sora property,
allegedly in the name of the IDP.
The Carpizo Group-INC sale is further deemed null and void ab initio because of
the Carpizo Group's failure to comply with Section 40 of the Corporation Code
pertaining to the disposition of all or substantially all assets of the corporation.
The Tandang Sora property, appears from the records, constitutes the only
property of the IDP. Hence, its sale to a third-party is a sale or disposition of all the
corporate property and assets of IDP falling squarely within the contemplation of the
foregoing section. For the sale to be valid, the majority vote of the legitimate Board of
Trustees, concurred in by the vote of at least 2/3 of the bona fide members of the
corporation should have been obtained.
These twin requirements were not met as the Carpizo Group which voted to sell
the Tandang Sora property was a fake Board of Trustees, and those whose names and
signatures were affixed by the Carpizo Group together with the sham Board Resolution
authorizing the negotiation for the sale were, from all indications, not bona fide
members of the IDP as they were made to appear to be. All told, the disputed Deed of
Absolute Sale executed by the fake Carpizo Board and private respondent INC was
intrinsically void ab initio.

220 | P a g e
Law 321_Corporation LAW_ Case Digest

THE EDWARD J. NELL COMPANY


vs.
PACIFIC FARMS, INC.
G.R. No. L-20850. November 29, 1965

FACTS:

On October 9, 1958, appellant secured against Insular Farms, Inc. a judgment


for the sum of P1,853.80 representing the unpaid balance of the price of a pump sold
by appellant to Insular Farms with interest on said sum. A writ of execution, issued
after the judgment had become final, was, on August 14, 1959, returned unsatisfied,
stating that Insular Farms had no leviable property. Appellant then filed with said
court the present action against Pacific Farms, Inc. for the collection of the judgment
aforementioned, upon the theory that appellee is the alter ego of Insular Farms, which
appellee has denied.
In due course, the municipal court rendered judgment dismissing petitioner‘s
complaint. Defendant appealed, with the same result, to the court of First Instance
and, subsequently, to the Court of Appeals. Hence this appeal by certiorari, upon the
ground that the Court of Appeals had erred in not holding the defendant liable for said
unpaid obligation of the Insular Farms.

ISSUE:

Whether or not the defendant is liable for the unpaid obligation of the Insular
Farms.

RULING:

NO.

Generally where one corporation sells or otherwise transfers all of its assets to
another corporation, the latter is not liable for the debts and liabilities of the
transferor, except: (1) where the purchaser expressly or impliedly agrees to assume
such debts; (2) where the transaction amounts to a consolidation or merger of the
corporations; (3) where the purchasing corporation is merely a continuation of the
selling corporation; and (4) where the transaction is entered into fraudulently in order
to escape liability for such debts."
In the case at bar, there is neither proof nor allegation that defendant had
expressly or impliedly agreed to assume the debt of Insular Farms in favor of
petitioner, or that the defendant is a continuation of Insular Farms, or that the sale of
either the shares of stock or the assets of Insular Farms to the defendant had been
entered into fraudulently, in order to escape liability for the debt of the Insular Farms
in favor of petitioner. Moreover, defendant purchased the shares of stock of Insular
Farms as the highest bidder at an auction sale held at the instance of a bank to which
said shares had been pledged as security for an obligation of Insular Farms in favor of
said bank. It has also been established that the defendant had paid P285,126.99 for
said shares of stock, apart from the sum of P10,000.00 it, likewise, paid for other
assets of Insular Farms.

221 | P a g e
Law 321_Corporation LAW_ Case Digest

JULIETA V. ESGUERRA
vs.
COURT OF APPEALS and SURESTE PROPERTIES, INC.
G.R. No. 119310. February 3, 1997

FACTS:

Julieta Esguerra filed a complaint for administration of conjugal partnership or


separation of property against her husband Vicente Esguerra, Jr. and V. Esguerra
Construction Co., Inc. (VECCI) and other family corporations as defendants before the
trial court.
The parties entered into a compromise agreement. By virtue of said agreement,
Esguerra Bldg. I was sold and the net proceeds distributed according to the
agreement. The controversy arose with respect to Esguerra Building II. Herein
petitioner started claiming one-half of the rentals of the said building which VECCI
refused. Thus, petitioner filed a motion with respondent court praying that VECCI be
ordered to remit one-half of the rentals to her. The trial court ruled in favor of
petitioner.
Meanwhile, Esguerra Bldg. II was sold to private respondent Sureste Properties.
Inc. for P150,000,000.00 prompting Julieta V. Esguerra to file a motion seeking the
nullification of the sale on the ground that VECCI is not the lawful and absolute owner
thereof and that she has not been notified nor consulted as to the terms and
conditions of the sale. The trial court ruled that the sale to Sureste was valid.

ISSUES:

Whether or not the sale of Esguerra Building II is a valid exercise of corporate


power.

RULING:

YES.

VECCI's sale of all the properties mentioned in the judicially-approved


compromise agreement was done on the basis of its Corporate Secretary's Certification
of these two resolutions. The partial decision did not require any further board or
stockholder resolutions to make VECCI's sale of these properties valid. Being regular
on its face, the Secretary's Certification was sufficient for private respondent Sureste
Properties, Inc. to rely on. It did not have to investigate the truth of the facts contained
in such certification. Otherwise, business transactions of corporations would become
tortuously slow and unnecessarily hampered. Ineluctably, VECCI's sale of Esguerra
Building II to private respondent was not ultra vires but a valid execution of the trial
court's partial decision.
Based on the foregoing, the sale is also deemed to have satisfied the
requirements of Section 40 of the Corporation Code.

222 | P a g e
Law 321_Corporation LAW_ Case Digest

LOPEZ REALTY, INC., AND ASUNCION LOPEZ GONZALES


vs.
FLORENTINA FONTECHA, ET AL., AND THE NATIONAL LABOR RELATIONS
COMMISSION
G.R. No. 76801. August 11, 1995

FACTS:

Lopez Realty, Inc., is a corporation engaged in real estate business, while


petitioner Asuncion Lopez Gonzales is one of its majority shareholders. Sometime in
1978, Arturo Lopez submitted a proposal relative to the distribution of certain assets
of Petitioner Corporation among its three (3) main shareholders. The proposal had
three (3) aspects, viz: (1) the sale of assets of the company to pay for its obligations; (2)
the transfer of certain assets of the company to its three (3) main shareholders, while
some other assets shall remain with the company; and (3) the reduction of employees
with provision for their gratuity pay. The proposal was deliberated upon and approved
in a special meeting of the board of directors held on April 17, 1978.
It appears that petitioner corporation approved two (2) resolutions providing for
the gratuity pay of its employees, viz: (a) Resolution No. 6, Series of 1980 resolving to
set aside, twice a year, a certain sum of money for the gratuity pay of its retiring
employees and to create a Gratuity Fund for the said contingency; and (b) Resolution
No. 10, Series of 1980, setting aside the amount of P157,750.00 as Gratuity Fund
covering the period from 1950 up to 1980.
On August 17, 1981, the remaining members of the Board of Directors, namely:
Rosendo de Leon, Benjamin Bernardino, and Leo Rivera, convened a special meeting
and passed a resolution which provides that: (a) Those who will be laid off be given the
full amount of gratuity; (b) Those who will be retained will receive 25% of their gratuity
(pay) due on September 1, 1981, and another 25% on January 1, 1982, and 50% to be
retained by the office in the meantime.
Private respondents were the retained employees of petitioner corporation. In a
letter, dated August 31, 1981, private respondents requested for the full payment of
their gratuity pay. Their request was granted in a special meeting held on September
1, 1981.

ISSUES:

Whether or not the subject resolutions requires for their validity stockholders‘
approval.

RULING:

YES.

The Court is not persuaded that the subject resolutions had no force and effect
in view of the non-approval thereof during the Annual Stockholders' Meeting held on
March 1, 1982. To strengthen their position, petitioners cite section 28 1/2 of the
Corporation Law (Section 40 of the Corporation Code).
The cited provision is not applicable to the case at bench as it refers to the sale,
lease, exchange or disposition of all or substantially all of the corporation's assets,
including its goodwill. In such a case, the action taken by the board of directors
requires the authorization of the stockholders on record.
It will be observed that, except for Arturo Lopez, the stockholders of petitioner
corporation also sit as members of the board of directors. Under the circumstances in
field, it will be illogical and superfluous to require the stockholders' approval of the
subject resolutions. Thus, even without the stockholders' approval of the subject
resolutions, petitioners are still liable to pay private respondents' gratuity pay.
Petition is dismissed.

223 | P a g e
Law 321_Corporation LAW_ Case Digest

To Invest Corporate Funds In Another Corporation or Business

JOHN GOKONGWEI, JR.


vs.
SECURITIES AND EXCHANGE COMMISSION, ANDRES M. SORIANO, JOSE M.
SORIANO, ENRIQUE ZOBEL, ANTONIO ROXAS, EMETERIO BUNAO, WALTHRODE
B. CONDE, MIGUEL ORTIGAS, ANTONIO PRIETO, SAN MIGUEL CORPORATION,
EMIGDIO TANJUATCO, SR., and EDUARDO R. VISAYA
G.R. No. L-45911. April 11, 1979

FACTS:

On October 22, 1976, petitioner, as stockholder of respondent San Miguel


Corporation, filed with the Securities and Exchange Commission (SEC) a petition for
"declaration of nullity of amended by-laws, cancellation of certificate of filing of
amended by- laws, injunction and damages with prayer for a preliminary injunction"
against the majority of the members of the Board of Directors and San Miguel
Corporation as an unwilling petitioner.
Petitioner alleged that on September 18, 1976, individual respondents amended
by bylaws of the corporation, basing their authority to do so on a resolution of the
stockholders adopted on March 13, 1961, when the outstanding capital stock of
respondent corporation was only P70,139.740.00, divided into 5,513,974 common
shares at P10.00 per share and 150,000 preferred shares at P100.00 per share. At the
time of the amendment, the outstanding and paid up shares totalled 30,127,047 with
a total par value of P301,270,430.00. It was contended that according to section 22 of
the Corporation Law and Article VIII of the by-laws of the corporation, the power to
amend, modify, repeal or adopt new by-laws may be delegated to the Board of
Directors only by the affirmative vote of stockholders representing not less than 2/3 of
the subscribed and paid up capital stock of the corporation, which 2/3 should have
been computed on the basis of the capitalization at the time of the amendment. Since
the amendment was based on the 1961 authorization, petitioner contended that the
Board acted without authority and in usurpation of the power of the stockholders.

ISSUES:

Whether or not respondent SEC committed grave abuse of discretion in allowing


discussion of Item 6 of the Agenda of the Annual Stockholders' Meeting on May 10,
1977, and the ratification of the investment in a foreign corporation of the corporate
funds, allegedly in violation of section 17-1/2 of the Corporation Law.

RULING:

NO.

Section 17-1/2 of the Corporation Law allows a corporation to "invest its funds
in any other corporation or business or for any purpose other than the main purpose
for which it was organized" provided that its Board of Directors has been so authorized
by the affirmative vote of stockholders holding shares entitling them to exercise at
least two-thirds of the voting power. If the investment is made in pursuance of the
corporate purpose, it does not need the approval of the stockholders. It is only when
the purchase of shares is done solely for investment and not to accomplish the
purpose of its incorporation that the vote of approval of the stockholders holding
shares entitling them to exercise at least two-thirds of the voting power is necessary.
As stated by Respondent Corporation, the purchase of beer manufacturing facilities by
SMC was an investment in the same business stated as its main purpose in its
Articles of Incorporation, which is to manufacture and market beer. It appears that the
original investment was made in 1947-1948, when SMC, then San Miguel Brewery,
Inc., purchased a beer brewery in Hongkong (Hongkong Brewery & Distillery, Ltd.) for
the manufacture and marketing of San Miguel beer thereat. Restructuring of the
investment was made in 1970-1971 thru the organization of SMI in Bermuda as a tax
free reorganization.

224 | P a g e
Law 321_Corporation LAW_ Case Digest

RAMON DE LA RAMA et.al


vs.
MA-AO SUGAR CENTRAL CO., INC., J. AMADO ARANETA, MRS. RAMON S.
ARANETA, ROMUALDO M. ARANETA, and RAMON A. YULO
G.r.No. L-17504 & l-17506; February 28, 1969

FACTS:

In 1950 the MSCCI through its President, J. Amado, subscribed for P300k
worth of capital stock of the Philippine Fiber Processing Co., Inc. (PFPC). Payments of
the subscription were made on 3 installments, but at the time the first two payments
were made there was no board resolution authorizing the investment; and that it was
only on November 26, 1951, that J. Amado was so authorized by the BOD, by the way,
making the third payment made in March 1952 authorized.
In addition, 355k shares of PFPC, owned by Luzon Industrial Corporation (LIC)
were transferred on May 31, 1952, to MSCCI. Again, the investment was made without
prior board resolution, the authorizing resolution having been subsequently approved
only on June 4, 1952. A derivative suit was filed by 4 minority SHs of MSCCI which
stated 5 causes of action: (1) for alleged illegal and ultra-vires acts consisting of self-
dealing, irregular loans, and unauthorized investments; (2) for alleged gross
mismanagement; (3) for alleged forfeiture of corporate rights warranting dissolution;
(4) for alleged damages and attorney's fees; and (5) for receivership.

ISSUE:

Whether or not a corporation can invest in another corporation.

RULING:

YES.

The law requiring the votes does not apply in the case because of MSCCI‘s
contention that since said PFPC was engaged in the manufacture of sugar bags it was
perfectly legitimate for MSCCI either to manufacture sugar bags or invest in another
corporation engaged in said manufacture.
SC also quoted the interpretation of Professor Guevara, a well-known authority
in Commercial Law: A private corporation, in order to accomplish its purpose as stated
in its articles of incorporation, and subject to the limitations imposed by the
Corporation Law, has the power to acquire, hold, mortgage, pledge or dispose of
shares, bonds, securities, and other evidences of indebtedness of any domestic or
foreign corporation. Such an act, if done in pursuance of the corporate purpose, does
not need the approval of the stockholders; but when the purchase of shares of another
corporation is done solely for investment and not to accomplish the purpose of its
incorporation, the vote of approval of the stockholders is necessary. In any case, the
purchase of such shares or securities must be subject to the limitations established by
the Corporation Law; namely, (a) that no agricultural or mining corporation shall in
anywise be interested in any other agricultural or mining corporation; or (b) that a
non-agricultural or non-mining corporation shall be restricted to own not more than
15% of the voting stock of any agricultural or mining corporation; and (c) that such
holdings shall be solely for investment and not for the purpose of bringing about a
monopoly in any line of commerce or combination in restraint of trade."
Power to invest corporate funds - A private corporation has the power to invest
its corporate funds 'in any other corporation or business, or for any purpose other
than the main purpose for which it was organized,' provided that 'its board of directors
has been so authorized in a resolution by the affirmative vote of stockholders holding
shares in the corporation entitling them to exercise at least two-thirds of the voting
power on such a proposal at a stockholders' meeting called for that purpose,' and
provided further, that no agricultural or mining corporation shall in anywise be
interested in any other agricultural or mining corporation. When the investment is
necessary to accomplish its purpose or purposes as stated in its articles of
incorporation, the approval of the stockholders is not necessary."

225 | P a g e
Law 321_Corporation LAW_ Case Digest

To Acquire Own Shares

BOMAN ENVIRONMENTAL DEVELOPMENT CORPORATION


vs.
HON. COURT OF APPEALS and NILCAR Y. FAJILAN
G.R. No. 77860. November 22, 1988

FACTS:

On May 7, 1984, respondent Nilcar Y. Fajilan offered in writing to resign as


President and Member of the Board of Directors of petitioner, Boman Environmental
Development Corporation (BEDECO), and to sell to the company all his shares, rights,
and interests therein for P 300,000 plus the transfer to him of the company's Isuzu
pick-up truck which he had been using.
At a meeting of the Board of Directors of BEDECO, Fajilan's resignation as
president was accepted and new officers were elected. Fajilan's offer to sell his shares
back to the corporation was approved, the Board promising to pay for them on a
staggered basis from July 15, 1984 to December 15, 1984.
A promissory note was signed by BEDECO'S new president, Alfredo Pangilinan,
in the presence of two directors, committing BEDECO to pay him P300,000 over a six-
month period from July 15, 1984 to December 15, 1984. However, BEDECO paid only
P50,000 on July 15, 1984 and another P50,000 on August 31, 1984 and defaulted in
paying the balance of P200,000. On April 30, 1985, Fajilan filed a complaint in the
Regional Trial Court of Makati for collection of that balance from BEDECO.

ISSUES:

Whether or not Petitioner Corporation can acquire its own shares.

RULING:

YES.

The provisions of the Corporation Code should be deemed written into the
agreement between the corporation and the stockholders even if there is no express
reference to them in the promissory note. The principle is well settled that an existing
law enters into and forms part of a valid contract without need for the parties'
expressly making reference to it.
The requirement of unrestricted retained earnings to cover the shares is based
on the trust fund doctrine which means that the capital stock, property and other
assets of a corporation are regarded as equity in trust for the payment of corporate
creditors. The reason is that creditors of a corporation are preferred over the
stockholders in the distribution of corporate assets. There can be no distribution of
assets among the stockholders without first paying corporate creditors. Hence, any
disposition of corporate funds to the prejudice of creditors is null and void.

226 | P a g e
Law 321_Corporation LAW_ Case Digest

C. H. STEINBERG, as Receiver of the Sibuguey Trading Company, Incorporated


vs.
GREGORIO VELASCO, ET AL.
G.R. No. L-30460. March 12, 1929

FACTS:

Plaintiff is the receiver of the Sibuguey Trading Company, a domestic


corporation. The defendants are residents of the Philippine Islands. It is alleged that
the defendants, Gregorio Velasco, as president, Felix del Castillo, as vice-president,
Andres L. Navallo, as secretary-treasurer, and Rufino Manuel, as director of Trading
Company, at a meeting of the board of directors, approved and authorized various
lawful purchases already made of a large portion of the capital stock of the company
from its various stockholders with total amount of the capital stock unlawfully
purchased was P3,300. At the time of such purchase, the corporation had accounts
payable amounting to P13,807.50, most of which were unpaid at the time petition for
the dissolution of the corporation was its financial condition, in contemplation of an
insolvency and dissolution. That on September 11, 1923, when the petition was filed
for its dissolution upon the ground that it was insolvent, its accounts payable
amounted to P9,241.19, and its accounts receivable P12,512.47, or an apparent asset
of P3,271.28 over and above its liabilities.

ISSUE:

Whether or not the Petition Corporation can acquire its own shares.

RULING:

NO.

It is, indeed, peculiar that the action of the board in purchasing the stock from
the corporation and in declaring the dividends on the stock was all done at the same
meeting of the board of directors, and it appears in those minutes that the both
Ganzon and Mendaros were formerly directors and resigned before the board approved
the purchase and declared the dividends, and that out of the whole 330 shares
purchased, Ganzon, sold 100 and Mendaros 200, or a total of 300 shares out of the
330, which were purchased by the corporation, and for which it paid P3,300.
In other words, the directors were permitted to resign so that they could sell
their stock to the corporation. As stated, the authorized capital stock was P20,000
divided into 2,000 shares of the par value of P10 each, which only P10,030 was
subscribed and paid. Deducting the P3,300 paid for the purchase of the stock, there
would be left P7,000 of paid up stock, from which deduct P3,000 paid in dividends,
there would be left P4,000 only. In this situation and upon this state of facts, it is very
apparent that the directors did not act in good faith or that they were grossly ignorant
of their duties.
Creditors of a corporation have the right to assume that so long as there are
outstanding debts and liabilities, the board of directors will not use the assets of the
corporation to purchase its own stock, and that it will not declare dividends to
stockholders when the corporation is insolvent.

227 | P a g e
Law 321_Corporation LAW_ Case Digest

Dividend: Kinds: Cash, Stock, Property, Scrip


Declaration, Payment and Record Dates

IMELDA O. COJUANGCO, PRIME HOLDINGS, INC., AND THE ESTATE OF RAMON


U. COJUANGCO
vs.
SANDIGANBAYAN, REPUBLIC OF THE PHILIPPINES, AND THE SHERIFF OF
SANDIGANBAYAN
G.R. No. 183278. April 24, 2009

FACTS:

On July 16, 1987, respondent Republic of the Philippines filed before the
Sandiganbayan a "Complaint for Reconveyance, Reversion, Accounting, Restitution
and Damages," praying for the recovery of alleged ill-gotten wealth from the late
President Marcos and former First Lady Imelda Marcos and their cronies, including
some 2.4 million shares of stock in the Philippine Long Distance Telephone Company
(PLDT) allegedly registered in the name of Prime Holdings, Inc. (Prime Holdings).
The Sandiganbayan dismissed the complaint with respect to the recovery of the
PLDT shares, hence, the Republic appealed to this Court, docketed as G.R. No.
153459, which appeal was later consolidated with pending cases of similar import –
G.R. Nos. 149802, 150320, and 150367.
The Decision became final and executory on October 26, 2006, hence, the
Republic filed on November 20, 2006 with the Sandiganbayan a Motion for the
Issuance of a Writ of Execution, praying for the cancellation of the 111,415
shares/certificates of stock registered in the name of Prime Holdings and the
annotation of the change of ownership on PTIC‘s Stock and Transfer Book. The
Republic further prayed for the issuance of an order for PTIC to account for all cash
and stock dividends declared and/or issued by PLDT in favor of PTIC from 1986 up to
the present including compounded interests appurtenant thereto.
The Sandiganbayan granted the Motion for the Issuance of a Writ of Execution
with respect to the reconveyance of the shares, but denied the prayer for accounting of
dividends. On Motion for Reconsideration of the Republic, the Sandiganbayan, by the
first assailed Resolution dated November 7, 2007, directed PTIC to deliver the cash
and stock dividends pertaining to the 111,415 shares, including compounded
interests, ratiocinating that the same were covered by this Court‘s Decision in G.R. No.
153459, since the Republic was therein adjudged the owner of the shares and,
therefore, entitled to the fruits thereof.

ISSUES:

Whether or not the Sandiganbayan gravely abused its discretion in ordering the
accounting, delivery, and remittance to the Republic of the stock, cash, and property
dividends pertaining to the 111,415 PTIC shares of Prime Holdings.

Whether or not the Republic, having transferred the shares to a third party, is
entitled to the dividends, interests, and earnings thereof.

RULING:

NO.

The term "dividend" in its technical sense and ordinary acceptation is that part
or portion of the profits of the enterprise which the corporation, by its governing
agents, sets apart for ratable division among the holders of the capital stock. It is a
payment to the stockholders of a corporation as a return upon their investment and
the right thereto is an incident of ownership of stock.
In directing the reconveyance to the Republic of the 111,415 shares of PLDT
stock owned by PTIC in the name of Prime Holdings, the Court declared the Republic
as the owner of said shares and, necessarily, the dividends and interests accruing
thereto.

228 | P a g e
Law 321_Corporation LAW_ Case Digest

Contrary to petitioners‘ contention, while the general rule is that the portion of
a decision that becomes the subject of execution is that ordained or decreed in the
dispositive part thereof, there are recognized exceptions to this rule, viz: (a).where
there is ambiguity or uncertainty, the body of the opinion may be referred to for
purposes of construing the judgment, because the dispositive part of a decision must
find support from the decision‘s ratio decidendi; and (b).where extensive and explicit
discussion and settlement of the issue is found in the body of the decision.
In G.R. No. 153459, although the inclusion of the dividends, interests, and
earnings of the 111,415 PTIC shares as belonging to the Republic was not mentioned
in the dispositive portion of the Court‘s Decision, it is clear from its body that what
was being adjudicated in favor of the Republic was the whole block of shares and the
fruits thereof, said shares having been found to be part of the Marcoses‘ ill-gotten
wealth, and therefore, public money.
It would be absurd to award the shares to the Republic as their owner and not
include the dividends and interests accruing thereto. An owner who cannot exercise
the "juses" or attributes of ownership -- the right to possess, to use and enjoy, to
abuse or consume, to accessories, to dispose or alienate, to recover or vindicate, and
to the fruits - is a crippled owner.

229 | P a g e
Law 321_Corporation LAW_ Case Digest

Limitation on Retention of Surplus Profits

C. H. STEINBERG, as Receiver of the Sibuguey Trading Company, Incorporated


vs.
GREGORIO VELASCO, ET AL.
G.R. No. L-30460. March 12, 1929

FACTS:

Plaintiff is the receiver of the Sibuguey Trading Company, a domestic


corporation. The defendants are residents of the Philippine Islands. It is alleged that
the defendants, Gregorio Velasco, as president, Felix del Castillo, as vice-president,
Andres L. Navallo, as secretary-treasurer, and Rufino Manuel, as director of Trading
Company, at a meeting of the board of directors, approved and authorized various
lawful purchases already made of a large portion of the capital stock of the company
from its various stockholders with total amount of the capital stock unlawfully
purchased was P3,300. At the time of such purchase, the corporation had accounts
payable amounting to P13,807.50, most of which were unpaid at the time petition for
the dissolution of the corporation was its financial condition, in contemplation of an
insolvency and dissolution. That on September 11, 1923, when the petition was filed
for its dissolution upon the ground that it was insolvent, its accounts payable
amounted to P9,241.19, and its accounts receivable P12,512.47, or an apparent asset
of P3,271.28 over and above its liabilities.

ISSUE:

Whether or not the Corporation acted in bad faith in acquiring its own shares of
stocks.

RULING:

YES.

There is no stipulation or finding of facts as to what was the actual cash value
of its accounts receivable. Neither is there any stipulation that those accounts or any
part of them ever have been or will be collected, and it does appear that after his
appointment on February 28, 1924, the receiver made a diligent effort to collect them,
and that he was unable to do so, and it also appears from the minutes of the board of
directors that the president and manager "recommended that P3,000 — out of the
surplus account to be set aside for dividends payable, and that payments be made in
installments so as not to effect the financial condition of the corporation."
It is very apparent that on June 24, 1922, the board of directors acted on
assumption that, because it appeared from the books of the corporation that it had
accounts receivable of the face value of P19,126.02, therefore it had a surplus over
and above its debts and liabilities. Thus, in the purchase of its own stock to the
amount of P3,300 and in declaring the dividends to the amount of P3,000, the real
assets of the corporation were diminished P6,300. The corporation did not then have
an actual bona fide surplus from which the dividends could be paid, and that the
payment of them in full at the time would "affect the financial condition of the
corporation."
Creditors of a corporation have the right to assume so long as there are
outstanding debts and liabilities, the board of directors will not use the assets of the
corporation to purchase its own stock, and that it will not declare dividends to
stockholders when the corporation is insolvent.

230 | P a g e
Law 321_Corporation LAW_ Case Digest

NIELSON & COMPANY, INC.


vs.
LEPANTO CONSOLIDATED MINING COMPANY
G.R. NO. L-21601 DECEMBER 28, 1968

FACTS:

Lepanto entered into a contract with Nielson wherein the latter was to manage
and operate the mining properties of the former claiming to be a contract of agency.
However, Nielson claims that the agreement is not one of agency.The phrase "Both
parties to this agreement fully recognize that the terms of this agreement are made
possible only because of the faith and confidence of the officials of each company have
in the other" in paragraph XI of the management contract does not qualify the relation
between Lepanto and Nielson as that of principal and agent based on trust and
confidence, such that the contractual relation may be terminated by the principal at
any time that the principal loses trust and confidence in the agent. Rather, that
phrase simply implies the circumstance that brought about the execution of the
management contract. In the annual report for 1936, submitted by Mr. C. A. Dewit,
President of Lepanto, to its stockholders, under date of March 15, 1937, it states that
instead of giving a monthly compensation to Nielson such was modified by giving the
amount of P2,500 plus 10% of cash value of the dividends declared and paid by
Lepanto.
The Court ruled that from the foregoing statements in the annual report for
1936, and from the provision of paragraph XI of the Management contract, that the
employment by Lepanto of Nielson to operate and manage its mines was principally in
consideration of the know-how and technical services that Nielson offered Lepanto.
The contract thus entered into pursuant to the offer made by Nielson and accepted by
Lepanto was a "detailed operating contract".

ISSUE:

Whether or not Nielson is entitled to receive shares of stock forming part of the
stock dividend of Lepanto in lieu of the cash value of the dividends declared by
Lepanto during the Japanese occupation.

RULING:

NO.

Shares of stock are given the special name "stock dividends" only if they are
issued in lieu of undistributed profits. If shares of stocks are issued in exchange of
cash or property then those shares do not fall under the category of "stock dividends".
A corporation may legally issue shares of stock in consideration of services rendered to
it by a person not a stockholder, or in payment of its indebtedness. A share of stock
issued to pay for services rendered is equivalent to a stock issued in exchange of
property, because services are equivalent to property.Likewise a share of stock issued
in payment of indebtedness is equivalent to issuing a stock in exchange for cash.
In other words, it is the shares of stock that are originally issued by the
corporation and forming part of the capital that can be exchanged for cash or services
rendered, or property; that is, if the corporation has original shares of stock unsold or
unsubscribed, either coming from the original capitalization or from the increased
capitalization. Those shares of stock may be issued to a person who is not a
stockholder, or to a person already a stockholder in exchange for services rendered or
for cash or property. But a share of stock coming from stock dividends declared
cannot be issued to one who is not a stockholder of a corporation.
Thus, stock dividends cannot be issued to a person who is not a stockholder in
payment of services rendered. And so, in the case at bar, Nielson cannot be paid in
shares of stock which form part of the stock dividends of Lepanto for services it
rendered under the management contract.

231 | P a g e
Law 321_Corporation LAW_ Case Digest

COMMISSIONER OF INTERNAL REVENUE


vs.
MANNING, MCDONALD, SIMMONS
AUGUST 06, 1975

FACTS:

In 1952 the MANTRASCO had an authorized capital stock of P2,500,000


divided into 25,000 common shares; 24,700 of these were owned by Julius S. Reese,
and the rest, at 100 shares each, by the three respondents.
In view of Reese's desire that upon his death MANTRASCO and its two
subsidiaries, MANTRASCO (Guam), Inc. and the Port Motors, Inc., would continue
under the management of the respondents, a trust agreement was executed by and
among Reese, MANTRASCO ,the law firm of Ross, Selph, Carrascoso and Jand ,and
the respondents .
On October 19, 1954 Reese died. In 1955, after MANTRASCO made a partial
payment of Reese's shares, the certificate for the 24,700 shares in Reese's name was
cancelled and a new certificate was issued in the name of MANTRASCO, which was
endorsed to the law firm of Ross, Selph, Carrascoso and Janda, as trustees for and in
behalf of MANTRASCO.
In 1963 the entire purchase price of Reese's interest in MANTRASCO was
finally paid in full by the latter, In 1964 the trust agreement was terminated and the
trustees delivered to MANTRASCO all the shares which they were holding in trust.

ISSUE:

Whether or not the issuance of the notices of assessment for deficiency income
taxes to the respondents for the year 1958 was proper.

RULING:

YES.

―A stock dividend always involves a transfer of surplus (or profit) to capital


stock. A stock dividend is a conversion of surplus or undivided profits into capital
stock, which is distributed to stockholders in lieu of a cash dividend.' Congress itself
has defined the term 'dividend' in No. 115(a) of the Act as meaning any distribution
made by a corporation to its shareholders, whether in money or in other property, out
of its earnings or profits.
The declaration by the respondents and Reese's trustees of MANTRASCO's
alleged treasury stock dividends in favor of the former, brings the ultimate purpose
which the parties to the trust instrument aimed to realize: to make the respondents
the sole owners of Reese's interest in MANTRASCO by utilizing the periodic earnings of
that company and its subsidiaries to directly subsidize their purchase of the said
interests, and by making it appear outwardly, through the formal declaration of non-
existent stock dividends in the treasury, that they have not received any income from
those firms when, in fact, by that declaration they secured to themselves the means to
turn around as full owners of Reese's shares. In other words, the respondents, using
the trust instrument as a convenient technical device, bestowed unto themselves the
full worth and value of Reese's corporate holdings with the use of the very earnings of
the companies. Such package device, obviously not designed to carry out the usual
stock dividend purpose of corporate expansion reinvestment, e.g. the acquisition of
additional facilities and other capital budget items, but exclusively for expanding the
capital base of the respondents in MANTRASCO, cannot be allowed to deflect the
respondents' responsibilities toward our income tax laws. The conclusion is thus
ineluctable that whenever the companies involved herein parted with a portion of their
earnings "to buy" the corporate holdings of Reese, they were in ultimate effect and
result making a distribution of such earnings to the respondents.
All these amounts are consequently subject to income tax as being, in truth and
in fact, a flow of cash benefits to the respondents.

232 | P a g e
Law 321_Corporation LAW_ Case Digest

MADRIGAL & COMPANY, INC.


vs.
HON. RONALDO B. ZAMORA, PRESIDENTIAL ASSISTANT FOR LEGAL AFFAIRS,
THE HON. SECRETARY OF LABOR, and MADRIGAL CENTRAL OFFICE
EMPLOYEES UNION
G.R. No. L-48237. June 30, 1987

MADRIGAL & COMPANY, INC.


vs.
HON. MINISTER OF LABOR and MADRIGAL CENTRAL OFFICE EMPLOYEES
UNION
No. L-49023. June 30, 1987

FACTS:

The petitioner was engaged, among several other corporate objectives, in the
management of Rizal Cement Co., Inc.Admittedly, the petitioner and Rizal Cement Co.,
Inc. are sister companies.Both are owned by the same or practically the same
stockholders.On December 28, 1973, the respondent, the Madrigal Central Office
Employees Union, sought for the renewal of its collective bargaining agreement with
the petitioner, which was due to expire on February 28, 1974.Specifically, it proposed
a wage increase of P200.00 a month, an allowance of P100.00 a month, and other
economic benefits.The petitioner, however, requested for a deferment in the
negotiations.
After the petitioner's failure to sit down with the respondent union, the latter,
commenced a complaint for unfair labor practice.Pending the resolution of the case,
the petitioner, in a letter informed the Secretary of Labor that Rizal Cement Co., Inc.,
"from which it derives income" had "ceased operating temporarily."In addition, because
of the desire of the stockholders to phase out the operations of the Madrigal & Co.,
Inc. due to lack of business incentives and prospects, and in order to prevent further
losses,it had to reduce its capital stock on two occasions, the Madrigal & Co., Inc. is
without substantial income to speak of, necessitating a reorganization, by way of
retrenchment, of its employees and operations.

ISSUE:

Whether or not the profits earned by the Corporation were in the nature of
dividends declared on its shareholdings in other companies in the earning of which
the employees had no participation whatsoever.

RULING:

NO.

The Court agreed with the National Labor Relations Commission that "the
dividends received by the company are corporate earnings arising from corporate
investment."Indeed, as found by the Commission, the petitioner had entered such
earnings in its financial statements as profits, which it would not have done if they
were not in fact profits.
Moreover, it is incorrect to say that such profits — in the form of dividends —
are beyond the reach of the petitioner's creditors since the petitioner had received
them as compensation for its management services in favor of the companies it
managed as a shareholder thereof. As such shareholder, the dividends paid to it were
its own money, which may then be available for wage increments. It is not a case of a
corporation distributing dividends in favor of its stockholders, in which case, such
dividends would be the absolute property of the stockholders and hence, out of reach
by creditors of the corporation. Here, the petitioner was acting as stockholder itself,
and in that case, the right to a share in such dividends, by way of salary increases,
may not be denied its employees.

233 | P a g e
Law 321_Corporation LAW_ Case Digest

REPUBLIC PLANTERS BANK


vs.
HON. ENRIQUE A. AGANA, SR., as Presiding Judge, Court of First Instance of
Rizal, Branch XXVIII, Pasay City, ROBES-FRANCISCO REALTY & DEVELOPMENT
CORPORATION and ADALIA F. ROBES
G.R. No. 51765. March 3, 1997

FACTS:

On September 18, 1961, private respondent Corporation secured a loan from


petitioner in the amount of P120,000.00. Instead of giving the legal tender totaling to
the full amount of the loan, which is P120,000.00, petitioner lent such amount
partially in the form of money and partially in the form of stock certificates numbered
3204 and 3205, each for 400 shares with a par value of P10.00 per share, or for
P4,000.00 each, for a total of P8,000.00. Said stock certificates were in the name of
private respondent Adalia F. Robes and Carlos F. Robes, who subsequently, however,
endorsed his shares in favor of Adalia F. Robes.
On January 31, 1979, private respondents proceeded against petitioner and
filed a Complaint anchored on private respondents' alleged rights to collect dividends
under the preferred shares in question and to have petitioner redeem the same under
the terms and conditions of the stock certificates.
The trial court rendered the herein assailed decision in favor of private
respondents ordering petitioner to pay private respondents the face value of the stock
certificates as redemption price, plus 1% quarterly interest thereon until full payment.

ISSUES:

Whether or not the corporation can declare dividends.

RULING:

YES.

Under the old Corporation Law in force at the time the contract between the
petitioner and the private respondents was entered into, it was provided that "no
corporation shall make or declare any dividend except from the surplus profits arising
from its business, or distribute its capital stock or property other than actual profits
among its members or stockholders until after the payment of its debts and the
termination of its existence by limitation or lawful dissolution."Similarly, the present
Corporation Codeprovides that the board of directors of a stock corporation may
declare dividends only out of unrestricted retained earnings.
Thus, the declaration of dividends is dependent upon the availability of surplus
profit or unrestricted retained earnings, as the case may be. Dividends are thus
payable only when there are profits earned by the corporation and as a general rule,
even if there are existing profits, the board of directors has the discretion to determine
whether or not dividends are to be declared.
Redeemable shares, on the other hand, are shares usually preferred, which by
their terms are redeemable at a fixed date, or at the option of either issuing
corporation, or the stockholder, or both at a certain redemption price.A redemption by
the corporation of its stock is, in a sense, a repurchase of it for cancellation.The
present Code allows redemption of shares even if there are no unrestricted retained
earnings on the books of the corporation.
However, while redeemable shares may be redeemed regardless of the existence
of unrestricted retained earnings, this is subject to the condition that the corporation
has, after such redemption, assets in its books to cover debts and liabilities inclusive
of capital stock. Redemption, therefore, may not be made where the corporation is
insolvent or if such redemption will cause insolvency or inability of the corporation to
meet its debts as they mature.

234 | P a g e
Law 321_Corporation LAW_ Case Digest

NORA A. BITONG
vs.
COURT OF APPEALS (FIFTH DIVISION), EUGENIA D. APOSTOL, JOSE A.
APOSTOL, MR. & MS. PUBLISHING CO., LETTY J. MAGSANOC, AND ADORACION
G. NUYDA
G.R. No. 123553. July 13, 1998
NORA A. BITONG
vs.
COURT OF APPEALS (FIFTH DIVISION) and EDGARDO B. ESPIRITU
(CA-G.R. No. 33873) July 13, 1998

FACTS:

Bitong alleged that she was the treasurer and member of the BoD of Mr. & Mrs.
Corporation. She filed a complaint with the SEC to hold respondent spouses Apostol
liable for fraud, misrepresentation, disloyalty, evident bad faith, conflict of interest and
mismanagement in directing the affairs of the corporation to the prejudice of the
stockholders. She alleges that certain transactions entered into by the corporation
were not supported by any stockholder‘s resolution. The complaint sought to enjoin
Apostol from further acting as president-director of the corporation and from
disbursing any money or funds.
Apostol contends that Bitong was merely a holder-in-trust of the JAKA shares
of the corporation, hence, not entitled to the relief she prays for. SEC Hearing Panel
issued a writ enjoining Apostol. After hearing the evidence, SEC Hearing Panel
dissolved the writ and dismissed the complaint filed by Bitong. Bitong appealed to the
SEC en banc which reversed SEC Hearing Panel decision. Apostol filed petition for
review with the CA. CA reversed SEC en banc ruling holding that Bitong was not the
owner of any share of stock in the corporation and therefore, not a real party in
interest to prosecute the complaint.

ISSUE:

Whether or not petitioner validly declared dividends.

RULING:

YES.

The records show that the original stock and transfer book and the stock
certificate book of Mr. & Ms. were in the possession of petitioner before their custody
was transferred to the Corporate Secretary, Atty. Augusto San Pedro. On 25 May
1988, Assistant Corporate Secretary Renato Jose Unson wrote Mr.& Ms. about the lost
stock and transfer book which was also noted by the corporation's external auditors,
Punongbayan and Araullo, in their audit. Atty. Unson even informed respondent
Eugenia D. Apostol as President of Mr. & Ms. that steps would be undertaken to
prepare and register a new Stock and Transfer Book with the SEC. Incidentally,
perhaps strangely, upon verification with the SEC, it was discovered that the general
file of the corporation with the SEC was missing. Hence, it was even possible that the
original Stock and Transfer Book might not have been registered at all.
This simply shows that as of 1988 there still existed certain issues affecting the
ownership of the JAKA shares, thus raising doubts whether the alleged transactions
recorded in the Stock and Transfer Book were proper, regular and authorized. JAKA
retained its ownership of Mr.& Ms. shares as clearly shown by its receipt of the
dividends issued in December 1986. This only means, very obviously, that Mr.& Ms.
shares in question still belonged to JAKA and not to petitioner. For, dividends are
distributed to stockholders pursuant to their right to share in corporate profits. When
a dividend is declared, it belongs to the person who is the substantial and beneficial
owner of the stock at the time regardless of when the distribution profit was earned.

235 | P a g e
Law 321_Corporation LAW_ Case Digest

COMMISSIONER OF INTERNAL REVENUE,


vs.
THE COURT OF APPEALS, COURT OF TAX APPEALS AND A. SORIANO
CORPORATION
G.R. NO. 108576; JANUARY 20, 1999

FACTS:

Sometime in the 1930s, Don Andres Soriano, a citizen and resident of the
United States, formed the corporation "A. Soriano Y Cia", predecessor of ANSCOR,
with a P1,000,000.00 capitalization divided into 10,000 common shares at a par value
of P100/share. In 1937, Don Andres subscribed to 4,963 shares of the 5,000 shares
originally issued. On September 12, 1945, ANSCOR's authorized capital stock was
increased to P2,500,000.00 divided into 25,000 common shares with the same par
value of the additional 15,000 shares, only 10,000 was issued which were all
subscribed by Don Andres. A month later, Don Andres transferred 1,250 shares each
to his two sons, Jose and Andres, Jr., as their initial investments in ANSCOR. Both
sons are foreigners. By 1947, ANSCOR declared stock dividends.
As of that date, the records revealed that he has total shareholdings of 185,154
shares, 50,495 of which are original issues and the balance of 134.659 shares as
stock dividend declarations. Correspondingly, one-half of that shareholdings or 92,577
shares were transferred to his wife, Doña Carmen Soriano, as her conjugal share. The
other half formed part of his estate.
A day after Don Andres died, ANSCOR increased its capital stock to P20M and
in 1966 further increased it to P30M. In the same year (December 1966), stock
dividends worth 46,290 and 46,287 shares were respectively received by the Don
Andres estate and Doña Carmen from ANSCOR. Hence, increasing their accumulated
shareholdings to 138,867 and 138,864 common shares each.

ISSUE:

Whether or not ANSCOR's redemption of stocks from its stockholder as well as


the exchange of common with preferred shares can be considered as "essentially
equivalent to the distribution of taxable dividend" making the proceeds thereof
taxable.

RULING:

YES.

Stock dividends, strictly speaking, represent capital and do not constitute


income to its recipient. So that the mere issuance thereof is not yet subject to income
tax as they are nothing but enrichment through increase in value of capital
investment." As capital, the stock dividends postpone the realization of profits because
the "fund represented by the new stock has been transferred from surplus to capital
and no longer available for actual distribution." In a loose sense, stock dividends
issued by the corporation, are considered unrealized gain, and cannot be subjected to
income tax until that gain has been realized. Before the realization, stock dividends
are nothing but a representation of an interest in the corporate properties. As capital,
it is not yet subject to income tax.
However, if a corporation cancels or redeems stock issued as a dividend at such
time and in such manner as to make the distribution and cancellation or redemption,
in whole or in part, essentially equivalent to the distribution of a taxable dividend, the
amount so distributed in redemption or cancellation of the stock shall be considered
as taxable income to the extent it represents a distribution of earnings or profits
accumulated after March first, nineteen hundred and thirteen.
It is not the stock dividends but the proceeds of its redemption that may be
deemed as taxable dividends.

236 | P a g e
Law 321_Corporation LAW_ Case Digest

To Enter into a Management Contract

WOLRGANG AURBACH, JOHN GRIFFIN, DAVID P. WHITTINGHAM and CHARLES


CHAMSAY
vs.
SANITARY WARES MANUFACTURING CORPORATOIN, ERNESTO V. LAGDAMEO,
ERNESTO R. LAGDAMEO, JR., ENRIQUE R. LAGDAMEO, GEORGE F. LEE, RAUL
A. BONCAN, BALDWIN YOUNG and AVELINO V. CRUZ
G.R. No. 75875. December 15, 1989

SANITARY WARES MANUFACTURING CORPORATION, et.al


vs.
THE COURT OF APPEALS, WOLFGANG AURBACH, JOHN GRIFFIN, DAVID P.
WHITTINGHAM, CHARLES CHAMSAY and LUCIANO SALAZAR
G.R. No. 75951 December 15, 1989

FACTS:

In 1961, Saniwares, a domestic corporation was incorporated for the primary


purpose of manufacturing and marketing sanitary wares. One of the incorporators,
Mr. Baldwin Young went abroad to look for foreign partners, European or American
who could help in its expansion plans.
On August 15, 1962, ASI, a foreign corporation domiciled in Delaware, United
States entered into an Agreement with Saniwares and some Filipino investors whereby
ASI and the Filipino investors agreed to participate in the ownership of an enterprise
which would engage primarily in the business of manufacturing in the Philippines and
selling here and abroad vitreous china and sanitary wares. The parties agreed that the
business operations in the Philippines shall be carried on by an incorporated
enterprise in the name of "Sanitary Wares Manufacturing Corporation."
Later, the 30% capital stock of ASI was increased to 40%. The corporation was
also registered with the Board of Investments for availment of incentives with the
condition that at least 60% of the capital stock of the corporation shall be owned by
Philippine nationals.

ISSUES:

Whether or not the ASI group may vote their additional equity during elections
of Saniwares' board of directors.

RULING:

YES.

In regard to the question as to whether or not the ASI group may vote their
additional equity during elections of Saniwares' board of directors. As in other joint
venture companies, the extent of ASI's participation in the management of the
corporation is spelled out in the Agreement. Section 5(a) hereof says that three of the
nine directors shall be designated by ASI and the remaining six by the other
stockholders, i.e., the Filipino stockholders. This allocation of board seats is obviously
in consonance with the minority position of ASI.
Having entered into a well-defined contractual relationship, it is imperative that
the parties should honor and adhere to their respective rights and obligations
thereunder. Appellants seem to contend that any allocation of board seats, even in
joint venture corporations, are null and void to the extent that such may interfere with
the stockholder's rights to cumulative voting as provided in Section 24 of the
Corporation Code.
On the one hand, the clearly established minority position of ASI and the
contractual allocation of board seats cannot be disregarded. On the other hand, the
rights of the stockholders to cumulative voting should also be protected.

237 | P a g e
Law 321_Corporation LAW_ Case Digest

PHILIPPINE NATIONAL BANK


vs.
PRODUCERS' WAREHOUSE ASSOCIATION
G.R. No. L-16510. January 9, 1922

FACTS:

The plaintiff is a corporation organized under the banking laws of the Philippine
while defendant is a domestic corporation doing a general warehouse business and the
Philippine Fiber and Produce Company, to which hereafter refer as the Produce
Company, is another domestic corporation.
In May, 1916, the defendant, as party of the first part, entered into a written
contract with the Produce Company, as party of the second part, in and by which "the
above-named party of the second part is hereby named, constituted, and appointed as
the general manager of the business of the party of the first part, in all of the branches
thereof, with the duties, powers, authority and compensation hereinafter provided." It
shall exercise a general and complete supervision over and management of the
business of the party of the first part," and "shall direct, manage, promote and
advance the said business, subject only to the control and instructions of the board of
directors of the party of the first part."
It is also alleged that in January, 1919, with the consent of the plaintiff, the
Produce Company removed from the warehouse of the defendant 1,112.15 piculs of
copra described in receipt No. 1255, of the declared value of P18,350.

ISSUE:

Whether or not the corporation has the power to enter into management
contract.

RULING:

YES.

Under the written contract between them, the Produce Company was the
general manager of the defendant's warehouse business, and that it had authority to
issue quedans in its name, and as its corporate act and deed. That the quedans in
question are duly authenticated, and were duly issued by the defendant to, and in the
name of, the Produce Company, and when issued were duly endorsed, and delivered to
the plaintiff for value. For aught that appears in the record, the bank was acting in
good faith, and the quedans were duly issued, endorsed and delivered to it as
collateral in the ordinary course of business. Although there may have been fraud,
there is no allegation or proof that the bank was a party to it, or had any knowledge of
it, and this court has no right to assume that the bank was a party to a fraud. Giving
to the quedans their legal force and effect, it must follow that at the time the demand
was made; the bank was the owner and entitled to the possession of the copra therein
described. The receipts call for 15,699.34 piculs of copra, but plaintiff admits that,
with its consent, 1,112.15 piculs of copra, of the declared value of P18,350, were
delivered to the Produce Company from and out of receipt No. 1255. This would leave
14,587.19 piculs of copra evidenced by the quedans.

238 | P a g e
Law 321_Corporation LAW_ Case Digest

NIELSON & COMPANY, INC.


vs.
LEPANTO CONSOLIDATED MINING COMPANY
G.R. NO. L-21601 DECEMBER 28, 1968

FACTS:

Lepanto entered into a contract with Nielson wherein the latter was to manage
and operate the mining properties of the former claiming to be a contract of agency.
However, Nielson claims that the agreement is not one of agency.The phrase "Both
parties to this agreement fully recognize that the terms of this agreement are made
possible only because of the faith and confidence of the officials of each company have
in the other" in paragraph XI of the management contract does not qualify the relation
between Lepanto and Nielson as that of principal and agent based on trust and
confidence, such that the contractual relation may be terminated by the principal at
any time that the principal loses trust and confidence in the agent. Rather, that
phrase simply implies the circumstance that brought about the execution of the
management contract. In the annual report for 1936, submitted by Mr. C. A. Dewit,
President of Lepanto, to its stockholders, under date of March 15, 1937, it states that
instead of giving a monthly compensation to Nielson such was modified by giving the
amount of P2,500 plus 10% of cash value of the dividends declared and paid by
Lepanto.
Thus, the contention of Lepanto that it had terminated the management
contract in 1945, following the liberation of the mines from Japanese control, because
the relation between it and Nielson was one of agency and as such it could terminate
the agency at will, is, therefore, untenable.

ISSUE:

Whether or not the nature of the management contracta contract of agency.

RULING:

NO.

By the contract of agency a person binds himself to render some service or to do


something in representation or on behalf of another, with the consent or authority of
the latter.
Under the contract, Nielson had agreed, for a period of five years, with the right
to renew for a like period, to explore, develop and operate the mining claims of
Lepanto, and to mine, or mine and mill, such pay ore as may be found therein and to
market the metallic products recovered therefrom which may prove to be marketable,
as well as to render for Lepanto other services specified in the contract.
Moreover, the contract thus entered into pursuant to the offer made by Nielson
and accepted by Lepanto was a "detailed operating contract". It was not a contract of
agency. Nowhere in the record is it shown that Lepanto considered Nielson as its agent
and that Lepanto terminated the management contract because it had lost its trust
and confidence in Nielson.
In the construction of an instrument where there are several provisions or
particulars, such a construction is, if possible, to be adopted as will give effect to all,
and if some stipulation of any contract should admit of several meanings, it shall be
understood as bearing that import which is most adequate to render it effectual.
Thus, by express stipulation of the parties, the management contract in
question is not revocable at the will of Lepanto. The Court ruled that this management
contract is not a contract of agency as defined in Article 1709 of the old Civil Code, but
a contract of lease of services as defined in Article 1544 of the same Code.

239 | P a g e
Law 321_Corporation LAW_ Case Digest

J. M. TUASON & CO., INC., represented by it Managing PARTNER, GREGORIA


ARANETA, INC.
vs.
QUIRINO BOLAÑOS
G.R. No. L-4935. May 28, 1954

FACTS:

The complaint described the land as a portion of a lot registered in the name of
the plaintiff and as containing an area of 13 hectares more or less. But the complaint
was amended by reducing the area of 6 hectares after the defendant had indicated the
plaintiff‘s surveyors the portion of the land claimed and occupied by him. The
defendant set up the defense of prescription.
After trial, the lower court rendered judgment for plaintiff, declaring defendant
to be without any right to the land in question and ordering him to restore possession
thereof to plaintiff and to pay the latter a monthly rent.

ISSUE:

Whether or not the trial court erred in not dismissing the case on the ground
that the case was not brought by the real party in interest.

RULING:

NO.

There is nothing to the contention that the persent action is not brought by the
real party in interest, that is, by J. Tuason & Co., Inc. The Rules of Court requires that
action must be brought in the name of the real party in interest.
The practice is for an attorney-at-law to bring the action that is to file the
complaint, in the name of the plaintiff. That practice appears to have been followed in
this case, since the case is signed by the law firm of Araneta and Araneta, ―counsel for
plaintiff‖ and commences with the statement ―comes plaintiff, through its undersigned
counsel‖. It is true that the complaint also states that the plaintiff is ―represented
herein by its Managing Partner Gregorio Araneta, Inc. ― another corporation, but there
is nothing against one corporation being represented by another person, natural or
juridical, in suit in court.
The contention that Gregorio Araneta, Inc. cannot act as managing partner for
plaintiff on the theory that it is illegal for two corporations to enter into partnership is
without merit, for the true rule is that ―through a corporation has no power to enter
into a partnership, it nevertheless enter into a joint venture with the another where
the nature of that venture is in line with the business authorized by its charter.‖

240 | P a g e
Law 321_Corporation LAW_ Case Digest

Ultra Vires Acts

HEIRS OF ANTONIO PAEL and ANDREA ALCANTARA and CRISANTO PAEL


vs.
COURT OF APPEALS, JORGE H. CHIN and RENATO B. MALLARI
G.R. No. 133547. December 7, 2001
MARIA DESTURA
vs.
COURT OF APPEALS, JORGE H. CHIN and RENATO B. MALLARI
G.R. No. 133843. December 7, 2001

FACTS:

PFINA acquired the properties from the Heirs of Pael by virtue of a deed of
assignment dated January 25, 1983. It filed a motion to intervene before the Court of
Appeals; however, before it filed its motion for intervention, or for a long period of
fifteen (15) years, PFINA and the Heirs of Pael were totally silent about the alleged deed
of assignment. No steps were taken by either of them to register the deed or secure
transfer certificate of title evidencing the change of ownership during this long period
of time.
At the time PFINA acquired the disputed properties in 1983, its corporate name
was PFINA Mining and Exploration, Inc., a mining company which had no valid
grounds to engage in the highly speculative business of urban real estate
development.

ISSUE:

Whether or not the title could produce legal effect.

RULING:

YES.

Notwithstanding its belated filing, the motion for intervention of U.P. is granted,
albeit the adjudication thereof shall be limited to a determination of the alleged
overlapping or encroachment between U.P.‘s title, on the one hand, and respondents‘
TCT Nos. 52928 and 52929, on the other hand.
The Court highlighted the citation in the comment of Intervenor U.P.,
specifically citing the decision in Roberto A. Pael et al. v. Court of Appeals, et al.,
supra, wherein the title of the Paels was declared to be of dubious origin and a
fabrication. Hence, since respondents derive their titles from a defective title, their
titles should also be null and void.
Considering the conflicting claims by U.P. and respondents, the ascertainment
of boundaries of the lands they respectively claim becomes imperative. The instant
cases have altogether taken more than eight (8) years. The boundaries of the
properties covered by the disputed titles of respondents and the boundaries of the
lands covered by the title of U.P. are not discussed therein. Thus, in order to avoid the
institution of new cases and thus obviate further litigation, the case should be
remanded to the Court of Appeals for reception of evidence relevant to determining the
boundaries of the conflicting claims between U.P. and respondents Chin and Mallari
over the property in dispute.

241 | P a g e
Law 321_Corporation LAW_ Case Digest

PILIPINAS LOAN COMPANY, INC.


vs.
HON. SECURITES AND EXCHANGE COMMISSION AND FILIPINAS PAWNSHOP,
INC.
G.R. No. 104720. April 4, 2001

FACTS:

Private respondent Filipinas Pawnshop, Inc. (private respondent) is a duly


organized corporation registered with the Securities and Exchange Commission (SEC)
on February 9, 1959. The articles of incorporation of private respondent states that its
primary purpose is to extend loans at legal interest on the security of either personal
properties or on the security of real properties, and to finance installment sales of
motor vehicles, home appliances and other chattels.
On September 11, 1990, private respondent filed a complaint against petitioner
with the Prosecution and Enforcement Department (PED) of the SEC and alleged that
petitioner, contrary to the restriction set by the Commission, has been operating and
doing business as a pawnbroker, pawnshop or "sanglaan" in the same neighborhood
where private respondent has had its own pawnshop for 30 years in violation of its
primary purpose and without the imprimatur of the Central Bank to engage in the
pawnshop business thereby causing unjust and unfair competition with private
respondent.
On October 18, 1990, petitioner filed its Comment/Answer questioning the
power of the SEC to take cognizance of the complaint involving (1) a supposed
violation of the Pawnshop Regulations Act which is more properly within the
jurisdiction of the Central Bank; and (2) the determination of whether a corporate
name is confusingly similar to another which is within the jurisdiction of the regular
courts. Petitioner denied that it is engaged in the pawnshop business, alleging that it
is a lending investor duly registered with the Central Bank.

ISSUE:

Whether or not Pilipinas Loan was acting beyond its authority.

RULING:

YES.

A corporation, under the Corporation Code, has only such powers as are
expressly granted to it by law and by its articles of incorporation,those which may be
incidental to such conferred powers, those reasonably necessary to accomplish its
purposes and those which may be incident to its existence. In the case at bar, the limit
of the powers of petitioner as a corporation is very clear, it is categorically prohibited
from "engaging in pawnbroking as defined under PD 114". Hence, in determining what
constitutes pawnbrokerage, the relevant law to consider is PD 114. This reference to
PD 114 is also in line with Article 2123 of the Civil.
Clearly, the recital in the complaint of Filipinas Pawnshop that Pilipinas Loan is
engaged in the pawnshop business when it is not authorized to do so by its articles of
incorporation amounts to fraud, detrimental not only to the corporation but also to the
stockholders and the public. The billboards of Pilipinas loan uses the word
―SANGLAAN‖ which cannot but give the impression to the public that its
establishment is more of a pawnshop than a lending institution servicing different
kinds of loans. The use of such word by petitioner was more calculated to attract
customers who will acquire loans on the security of personal properties alone.

242 | P a g e
Law 321_Corporation LAW_ Case Digest

ERNESTINA CRISOLOGO-JOSE
vs.
COURT OF APPEALS and RICARDO S. SANTOS, JR. in his own behalf and as Vice-
President for Sales of Mover Enterprises, Inc
G.R. No. 80599. September 15, 1989

FACTS:

In 1980, plaintiff Ricardo S. Santos, Jr. was the vice-president of Mover


Enterprises, Inc. in-charge of marketing and sales; and the president of the said
corporation was Atty. Oscar Z. Benares. On April 30, 1980, Atty. Benares, in
accommodation of his clients, the spouses Jaime and Clarita Ong, issued Check No.
093553 drawn against Traders Royal Bank, dated June 14, 1980, in the amount of
P45,000.00 payable to defendant Ernestina Crisologo-Jose. Since the check was under
the account of Mover Enterprises, Inc., the same was to be signed by its president,
Atty. Oscar Z. Benares, and the treasurer of the said corporation. However, since at
that time, the treasurer of Mover Enterprises was not available, Atty. Benares
prevailed upon the plaintiff, Ricardo S. Santos, Jr., to sign the aforesaid check as an
alternate story. Plaintiff Ricardo S. Santos, Jr. did sign the check.
It appears that the check was issued to defendant Ernestina Crisologo-Jose in
consideration of the waiver or quitclaim by said defendant over a certain property
which the Government Service Insurance System (GSIS) agreed to sell to the clients of
Atty. Oscar Benares, the spouses Jaime and Clarita Ong. However, since the
compromise agreement was not approved within the expected period of time, the
aforesaid check for P45,000.00 was replaced by Atty. Benares with another Traders
Royal Bank check dated August 10, 1980, in the same amount.

ISSUE:

Whether or not the accommodation party is Mover Enterprises, Inc. and hence
should be made liable.

RULING:

NO.

The provision of the Negotiable Instruments Law which holds an


accommodation party liable on the instrument to a holder for value, although such
holder at the time of taking the instrument knew him to be only an accommodation
party, does not include nor apply to corporations which are accommodation parties.
This is because the issue or indorsement of negotiable paper by a corporation
without consideration and for the accommodation of another is ultra vires. Hence, one
who has taken the instrument with knowledge of the accommodation nature thereof
cannot recover against a corporation where it is only an accommodation party. If the
form of the instrument, or the nature of the transaction, is such as to charge the
indorsee with knowledge that the issue or indorsement of the instrument by the
corporation is for the accommodation of another, he cannot recover against the
corporation thereon.
By way of exception, an officer or agent of a corporation shall have the power to
execute or indorse a negotiable paper in the name of the corporation for the
accommodation of a third person only if specifically authorized to do so. Corollarily,
corporate officers, such as the president and vice-president, have no power to execute
for mere accommodation a negotiable instrument of the corporation for their
individual debts or transactions arising from or in relation to matters in which the
corporation has no legitimate concern. Since such accommodation paper cannot thus
be enforced against the corporation, especially since it is not involved in any aspect of
the corporate business or operations, the inescapable conclusion in law and in logic is
that the signatories thereof shall be personally liable therefor, as well as the
consequences arising from their acts in connection therewith.

243 | P a g e
Law 321_Corporation LAW_ Case Digest

IRINEO G. CARLOS
vs.
MINDORO SUGAR CO., ET AL.
G.R. No. L-36207. October 26, 1932

FACTS:

The Mindoro Sugar Company is a corporation constituted in accordance with


the laws of the country and registered on July 30, 1917. According to its articles of
incorporation, one of its principal purposes was to acquire and exercise the franchise
granted by Act No. 2720 to George H. Fairchild, to substitute the organized
corporation, the Mindoro Company, and to acquire all the rights and obligations of the
latter and of Horace Havemeyer and Charles J. Welch in the so-called San Jose Estate
in the Province of Mindoro.
The Philippine Trust Company is another domestic corporation, registered on
October 21, 1917. In its articles of incorporation, some of its purposes are expressed
thus: "To acquire by purchase, subscription, or otherwise, and to invest in, hold, sell,
or otherwise dispose of stocks, bonds, mortgages, and other securities, or any interest
in either, or any obligations or evidences of indebtedness, of any other corporation or
corporations, domestic or foreign.
In pursuance of this resolution, the Mindoro Sugar Company executed in favor
of the Philippine Trust Company the deed of trust transferring all of its property to it
in consideration of the bonds it had issued to the value of P3,000,000.
The Philippine Trust Company sold thirteen bonds, Nos. 1219 to 1231, to
Ramon Diaz for P27,300, at a net profit of P100 per bond. The four bonds Nos. 1219,
1220, 1221, and 1222, here in litigation, are included in the thirteen sold to Diaz. The
Philippine Trust Company paid the appellant, upon presentation of the coupons, the
stipulated interest from the date of their maturity until the 1st of July, 1928, when it
stopped payments; and thenceforth it alleged that it did not deem itself bound to pay
such interest or to redeem the obligation because the guarantee given for the bonds
was illegal and void.

ISSUE:

Whether or not Philippine Trust Company bound itself legally and acted within
its corporate powers in guaranteeing the four bonds in question.

RULING:

YES.

The Philippine Trust Company has full powers to acquire personal property
such as the bonds in question. Being authorized to acquire the bonds, it was given
implied power to guarantee them in order to place them upon the market under better,
more advantageous conditions, and thereby secure the profit derived from their sale. It
is not, however, ultra vires for a corporation to enter into contracts of guaranty or
suretyship where it does so in the legitimate furtherance of its purposes and business.
And it is well settled that where a corporation acquires commercial paper or bonds in
the legitimate transaction of its business it may sell them, and in furtherance of such
a sale it may, in order to make them the more readily marketable, indorse or
guarantee their payment.
Guaranties of payment of bonds taken by a loan and trust company in the
ordinary course of its business, made in connection with their sale, are not ultra vires,
and are binding.

244 | P a g e
Law 321_Corporation LAW_ Case Digest

MARIA CLARA PIROVANO ET AL.


vs.
THE DE LA RAMA STEAMSHIP CO.
G.R. No. L-5377. December 29, 1954

FACTS:

Plaintiffs herein are the minor children of the late Enrico Pirovano represented
by their mother and judicial guardian Estefania R. Pirovano. They seek to enforce
certain resolutions adopted by the Board of Directors and stockholders of the
defendant company giving to said minor children of the proceeds of the insurance
policies taken on the life of their deceased father Enrico Pirovano with the company as
beneficiary. Defendant's main defense is: that said resolutions and the contract
executed pursuant thereto are ultra vires, and, if valid, the obligation to pay the
amount given is not yet due and demandable.
Plaintiff-appellant Pirovano is the owner of 3424 shares of stocks in defendant-
appellee Corporation which declared a dividend of P100 per share. Appellant wants to
recover from appellee the sum of P221, 975 after deducting the sum of P120, 424
which she had withdrawn or received from appellee for advances she received after the
death of her father, the late Esteban de la Rama.
Appellant‘s theory is that the cash advances to her for her personal use and
that of her children were assumed by Esteban de la Rama. She claims that the
advances made to her by appellees were debited from the account of Hijos de I. de la
Rama, another corporation practically owned by Esteban de la Rama. She further
claims that the appellee can only deduct from the amount of dividend she is entitled
to, the amount of cash advances which was not assumed by her father. The
withdrawals by the appellant were made during the period 1940 to 1949 during which
the appellee made a deed of trust with Hijos. The deed of trust was made to
circumvent the prohibition of declaring dividends during the period.

ISSUE:

Whether or not the donation made by the corporation of the proceeds of the
insurance is a valid act.

RULING:

YES.

Even assuming that the donation was ultra vires, still it cannot be invalidated
or declared legally ineffective for that reason alone, it appearing that the donation
represents not only the act of the Board but also that of the stockholders themselves
since they expressly ratified the resolution. By this ratification, the infirmity of the
corporate act, if any, has been obliterated thereby making the act perfectly valid and
enforceable, especially so if the donation is not merely executory but consummated.
The defense of ultra vires cannot be set up against completed or consummated
transactions.
An ultra vires act may either be an act performed merely outside the scope of
the powers granted to the corporation by its AOI or one which is contrary to law or
violative of any principle which would void any contract. A distinction has to be made
with respect to corporate acts which are illegal and those merely ultra vires. The
former are contrary to law, morals, public order or policy, while the latter are not void
ab initio, but merely go beyond the scope of the powers in the AOI, and which renders
the act merely voidable and thus can be ratified by the stockholders.
The defendant corporation, therefore, is now prevented or estopped from
contesting the validity of the donation. This is especially so in this case when the very
directors who conceived the idea of granting said donation are practically the
stockholders themselves, with few nominal exceptions.

245 | P a g e
Law 321_Corporation LAW_ Case Digest

REPUBLIC OF THE PHILIPPINES


vs.
ACOJE MINING COMPANY, INC.
G.R. No. L-18062. February 28, 1963

FACTS:

On May 17, 1948, the Acoje Mining Company, Inc. wrote the Director of Posts
requesting the opening of a post, telegraph and money order offices at its mining camp
at Sta. Cruz, Zambales, to service its employees and their families that were living in
said camp. Acting on the request, the Director of Posts wrote in reply stating that if
aside from free quarters the company would provide for all essential equipment and
assign a responsible employee to perform the duties of a postmaster without
compensation from his office until such time as funds therefor may be available he
would agree to put up the offices requested. The company in turn replied signifying its
willingness to comply with all the requirements.
On April 11, 1949, the Director of Posts again wrote a letter to the company
stating among other things that "In cases where a post office will be opened under
circumstances similar to the present, it is the policy of this office to have the company
assume direct responsibility for whatever pecuniary loss may be suffered by the
Bureau of Posts by reason of any act of dishonesty, carelessness or negligence on the
part of the employee of the company who is assigned to take charge of the post office,"
thereby suggesting that a resolution be adopted by the board of directors of the
company expressing conformity to the above condition relative to the responsibility to
be assumed buy it in the event a post office branch is opened as requested.
On September 2, 1949, the company informed the Director of Posts of the
passage by its board of directors of a resolution The letter further states that the
company feels that that resolution fulfills the last condition imposed by the Director of
Posts and that, therefore, it would request that an inspector be sent to the camp for
the purpose of acquainting the postmaster with the details of the operation of the
branch office.

ISSUE:

Whether or not the act of the Board in issuing the said resolution of conformity
was ultra vires.

RULING:

NO.

The corporate act was a necessary corollary to promote the interest and welfare
of the corporation. This is further bolstered by the fact that the opening of the post
was upon the request of the company for the convenience and benefit of its employees,
and not an idea of the Director of Posts. Thus, having benefited from the agreement,
the corporation is estopped from raising the defense that the said corporate act by its
board in conforming to the condition imposed by the Director of Posts is ultra vires.
Neither can the corporation interpose the defense that its liability is only that of
a guarantor. A mere reading of the resolution of the Board of Directors dated August
31, 1949, upon which the plaintiff based its claim, would show that the responsibility
of the defendant company is not just that of a guarantor. The phraseology and the
terms employed are so clear and sweeping and that the defendant assumed 'full
responsibility for all cash received by the Postmaster.' Here the responsibility of the
defendant is not just that of a guarantor. It is clearly that of a principal."

246 | P a g e
Law 321_Corporation LAW_ Case Digest

REPUBLIC OF THE PHILIPPINES


vs.
SECURITY CREDIT AND ACCEPTANCE CORPORATION, ROSENDO T. RESUELLO,
PABLO TANJUTCO, ARTURO SORIANO, RUBEN BELTRAN, BIENVENIDO V. ZAPA,
PILAR G. RESUELLO, RICARDO D. BALATBAT, JOSE SEBASTIAN and VITO
TANJUTCO JR.
G.R. No. L-20583. January 23, 1967

FACTS:

The Articles of Incorporation of defendant corporation were registered with the


Securities and Exchange Commission on March 27, 1961. Thereafter, the Board of
Directors of the corporation adopted a set of by-laws which were filed with said
Commission on April 5, 1961
On September 19, 1961, the Superintendent of Banks of the Central Bank of
the Philippines asked its legal counsel an opinion on whether or not said corporation
is a banking institution, within the purview of Republic Act No. 337; that, acting upon
this request, on October 11, 1961, said legal counsel rendered an opinion resolving
the query in the affirmative
On March 9, 1961, the corporation had applied with the Securities and
Exchange Commission for the registration and licensing of its securities under the
Securities Act. However, SCAC‘s registration of its Articles of Incorporation was denied
on the ground that it has not complied with the requirements under the General
Banking Act (RA No. 337). Later, a Search Warrant was issued against SCAC where
documents and records relative to its business operation were seized.
Even when SCAC was duly advised of the findings, SCAC and its BOD and
Officers still continued operations prompting the Solicitor General to file a quo
warranto proceedings for the dissolution of SCAC.

ISSUE:

Whether or not SCAC was illegally engaged in the business of banking.

RULING:

YES.

In dissolving SCAC, the Court held that the corporation was indeed engaged in
the business of banking without first securing the administrative authority required by
RA No. 337.
Although, admittedly, SCAC has not secured the requisite authority to engage
in banking, defendants deny that its transactions partake of the nature of banking
operations. Note however that, in consequence of their propaganda campaign, a total
of 59,463 savings account deposits have been made by the public with SCAC and its
74 branches, with an aggregate deposit of P1,689,136.74, which has been lent out to
such persons as SCAC deemed suitable. It is clear that these transactions partake of
the nature of banking, as the term is used in Section 2 of RA No. 337. Indeed, a bank
has been defined as: A moneyed institute founded to facilitate the borrowing, lending,
and safe-keeping of money and to deal in notes, bills of exchange, and credits; an
investment company which loans out the money of its customers, collects the
interests, and charges a commission to both lender and borrower is a bank; any
person engaged in the business carried on by banks of deposit, of discount, or of
circulation is doing a banking business, although but one of these functions is
exercised.
The illegal transactions thus undertaken by SCAC to warrant its dissolution is
apparent from the fact that the foregoing misuser of the corporate funds and franchise
affects the essence of its business, that it is willful and has been repeated 59,643
times, and that its continuance inflicts injury upon the public, owing to the number of
persons affected thereby.

247 | P a g e
Law 321_Corporation LAW_ Case Digest

BY-LAWS
Function

DILY DANY NACPIL


vs.
INTERNATIONAL BROADCASTING CORPORATION
G.R. No. 144767. March 21, 2002

FACTS:

Petitioner was Assistant General Manager for Finance/Administration and


Comptroller of private respondent Intercontinental Broadcasting Corporation (IBC)
from 1996 until April 1997. According to petitioner, when Emiliano Templo was
appointed to replace IBC President Tomas Gomez III sometime in March 1997, the
former told the Board of Directors that as soon as he assumes the IBC presidency, he
would terminate the services of petitioner. Apparently, Templo blamed petitioner,
along with a certain Mr. Basilio and Mr. Gomez, for the prior mismanagement of IBC.
Upon his assumption of the IBC presidency, Templo allegedly harassed, insulted,
humiliated and pressured petitioner into resigning until the latter was forced to retire.
However, Templo refused to pay him his retirement benefits, allegedly because he had
not yet secured the clearances from the Presidential Commission on Good Government
and the Commission on Audit.
IBC filed a motion to dismiss contending that petitioner was a corporate officer
who was duly elected by the Board of Directors of IBC; hence, the case qualifies as an
intra-corporate dispute falling within the jurisdiction of the SEC.
On the other hand, petitioner argues that he is not a corporate officer of IBC
but an employee thereof since he had not been elected nor appointed as Comptroller
and Assistant Manager by the IBC's Board of Directors but by an IBC General
Manager. This is also because the IBC's By-Laws do not even include the position of
comptroller in its roster of corporate officers.He therefore contends that his dismissal
is a controversy falling within the jurisdiction of the labor courts.

ISSUE:

Whether or not petitioner is a corporate officer although the position of


comptroller is not expressly mentioned in the by-laws.

RULING:

NO.

The fact that the position of Comptroller is not expressly mentioned among the
officers of the IBC in the By-Laws is of no moment, because the IBC's Board of
Directors is empowered under Section 25 of the Corporation Code and under the
corporation's By-Laws to appoint such other officers as it may deem necessary.
The by-laws may and usually do provide for such other officers," and that where
a corporate office is not specifically indicated in the roster of corporate offices in the
by-laws of a corporation, the board of directors may also be empowered under the by-
lawsto create additional officers as may be necessary. Furthermore, as petitioner's
appointment as comptroller required the approval and formal action of the IBC's
Board of Directors to become valid, it is clear therefore holds that petitioner is a
corporate officer whose dismissal may be the subject of a controversy cognizable by
the SEC under Section 5(c) of P.D. 902-A which includes controversies involving both
election and appointmentof corporate directors, trustees, officers, and managers. Had
petitioner been an ordinary employee, such board action would not have been
required.

248 | P a g e
Law 321_Corporation LAW_ Case Digest

PMI COLLEGES
vs.
THE NATIONAL LABOR RELATIONS COMMISSION and ALEJANDRO GA LVA N
G.R. No. 121466. August 15, 1997

FACTS:

On July 7, 1991, petitioner, an educational institution offering courses on basic


seaman's training and other marine-related courses, hired private respondent as
contractual instructor with an agreement that the latter shall be paid at an hourly rate
of P30.00 to P50.00, depending on the description of load subjects and on the
schedule for teaching the same. Pursuant to this engagement, private respondent then
organized classes in marine engineering.
Initially, private respondent and other instructors were compensated for
services rendered during the first three periods of the abovementioned contract.
However, for reasons unknown to private respondent, he stopped receiving payment
for the succeeding rendition of services. This claim of non-payment was embodied in a
letter dated March 3, 1992, written by petitioner's Acting Director, Casimiro A.
Aguinaldo, addressed to its President, Atty. Santiago Pastor, calling attention to and
appealing for the early approval and release of the salaries of its instructors including
that of private respondent.
Private respondent's claims, were resisted by petitioner. Later in the
proceedings, PMI Colleges manifested that Mr. Tomas Cloma Jr., a member of the
board of trustees write a letter to the Chairman of the Board, clarifying the case of
Galvan and stating therein, inter alia, that under PMI‘s by-laws only the Chairman is
authorized to sign any contract and that Galvan, in any event, failed to submit
documents on the alleged shipyard and plant visits in Cavite Naval Base.

ISSUE:

Whether or not the contract of employment of Galvan valid even if the signatory
therein was not the Chairman of the Board.

RULING:

YES.

The contract of employment is valid. The contract remained valid even if the
signatory thereon was not the chairman of the board which allegedly violated
petitioner‘s by-laws. Since by-laws operate merely as internal rules among the
stockholders, they cannot affect or prejudice third persons who deal with the
corporation, unless they have knowledge of the same. No proof appears on record that
private respondent ever knew anything about the provisions of the said by-laws. In
fact, petitioner itself merely asserts the same without even bothering to attach a copy
or excerpt thereof to show that there is such provision. That this allegation has never
been denied to private respondent nor necessarily signify admission of its existence
because technicalities of law and procedure and the rules obtaining in the courts of
law do not strictly apply to proceeding of this nature.

249 | P a g e
Law 321_Corporation LAW_ Case Digest

LOYOLA GRAND VILLAS HOMEOWNERS (SOUTH) ASSOCIATION, INC.


vs.
CA, HOME INSURANCE AND GUARANTY CORPORATION, EMDEN ENCARNACION
and HORATIO AYCARDO
G.R. No. 117188 August 7, 1997

FACTS:

Loyola Grand Villas Homeowners Association (LGVHA) was organized on


February 8, 1983 as the association of homeowners and residents of the Loyola Grand
Villas. It was registered with the Home Financing Corporation, the predecessor of
herein respondent Home Insurance and Guaranty Corporation (HIGC), as the sole
homeowners' organization in the said subdivision. It was organized by the developer of
the subdivision and its first president was Victorio V. Soliven, himself the owner of the
developer. For unknown reasons, however, LGVHAI did not file its corporate by-laws.
Sometime in 1988, the officers of the LGVHAI tried to register its by-laws but failed to
do so. Then the officers that there were two other organizations within the subdivision
the Loyola Grand Villas homeowners North Association Incorporated (North
Association) and the Loyola Grand Villas homeowners South Association Incorporated
(South Association). According to private respondents, a non-resident and Soliven
himself, respectively headed these associations. They also discovered that these
associations had five (5) registered homeowners each who were also the incorporators,
directors and officers thereof. None of the members of the LGVHAI was listed as
member of the North Association while three (3) members of LGVHAI were listed as
members of the South Association. When Soliven inquired about the status of
LGVHAI, Atty. Joaquin A. Bautista, the head of the legal department of the HIGC,
informed him that LGVHAI had been automatically dissolved because it did not submit
its by-laws within the period required by the Corporation Code and there was non-
user of corporate charter because HIGC had not received any report on the
association's activities. Apparently, this information resulted in the registration of the
North and South Association.

ISSUE:

Whether or not failure of LGVHAI to file its by-laws within one month from the
date of its incorporation result in its automatic dissolution.

RULING:

NO.

The Supreme Court ruled that the non-filing of the by-laws within the period of
1 month from the issuance by SEC of the Certificate of Incorporation will not result to
the automatic dissolution of the corporation because the word ―MUST‖ in Sec 46 of the
Corporation Code is merely directory not mandatory in meaning. In fact the second
paragraph allows the filing of by-laws even prior to incorporation.
This provision of the Code rules out mandatory compliance with the
requirement of filing the by-laws "within one (1) month after receipt of official notice of
the issuance of its certificate of incorporation by the Securities and Exchange
Commission." It necessarily follows that failure to file the by-laws within that period
does not imply the "demise" of the corporation. By-laws may be necessary for the
"government" of the corporation but these are subordinate to the articles of
incorporation as well as to the Corporation Code and related statutes.

250 | P a g e
Law 321_Corporation LAW_ Case Digest

CITIBANK, N.A.
vs.
HON. SEGUNDINO G. CHUA, SANTIAGO M. KAPUNAN and LUIS L. VICTOR,
ASSOCIATE JUSTICES OF THE HON. COURT OF APPEALS, THIRD DIVISION,
MANILA, HON. LEONARDO B. CANARES, Judge of Regional, Trial Court of Cebu,
Branch 10, and SPOUSES CRESENCIO AND ZENAIDA VELEZ
G.R. No. 102300. March 17, 1993

FACTS:

Citibank is a foreign commercial banking corporation duly licensed to do


business in the Philippines. Private respondents, spouses Cresencio and Zenaida
Velez, who were good clients alleged that the petitioner bank extended to them credit
lines sufficiently secured with real estate and chattel mortgages on equipment. They
claim that a restructuring agreement has been entered into between them and the
bank. However, the bank failed to comply thereto thus spouses Velez sued for specific
performance and damages.
On March 30, 1990, the date of the pre-trial conference, counsel for petitioner
bank appeared, presenting a special power of attorney executed by Citibank officer
Florencia Tarriela in favor of petitioner bank's counsel, the J.P. Garcia & Associates, to
represent and bind petitioner bank at the pre-trial conference of the case at bar.
Inspite of this special power of attorney, counsel for spouses Velez orally moved to
declare petitioner bank as in default on the ground that the special power of attorney
was not executed by the Board of Directors of Citibank. Thus petitioner bank executed
another special power of attorney made by William W. Ferguson, Vice President and
highest ranking officer of Citibank, Philippines, constituting and appointing the J.P.
Garcia & Associates to represent and bind the BANK. Unsatisfied, private respondents
moved again for declaration of default. Though the bank again executed
anotherspecial power of attorney through William W. Ferguson in favor of Citibank
employees, the court issued an order declaring petitioner bank as in default. The CA
dismissed the petition filed by the bank. The CA relied on Section 46 of the
Corporation Code to support its conclusion that the by-laws in question are without
effect because they were not approved by the SEC.

ISSUE:

Whether or not petitioner bank's by-laws, which constitute the basis for
Ferguson's special power of attorney in favor of petitioner bank's legal counsel are
effective, considering that petitioner bank has been previously granted a license to do
business in the Philippines.

RULING:

YES.

Section 46 (which was relied upon by the CA) starts with the phrase "Every
corporation formed under this Code", which can only refer to corporations
incorporated in the Philippines. Hence, Section 46, in so far as it refers to the
effectivity of corporate by-laws, applies only to domestic corporations and not to
foreign corporations. On the other hand, Section 125 of the same Code requires that a
foreign corporation applying for a license to transact business in the Philippines must
submit, among other documents, to the SEC, a copy of its articles of incorporation and
by-laws, certified in accordance with law. Unless these documents are submitted, the
application cannot be acted upon by the SEC.
Since the SEC will grant a license only when the foreign corporation has
complied with all the requirements of law, it follows that when it decides to issue such
license, it is satisfied that the applicant's by-laws, among the other documents, meet
the legal requirements. This, in effect, is an approval of the foreign corporations‘ by-
laws. It may not have been made in express terms; still it is clearly an approval.
Therefore, petitioner bank's by-laws, though originating from a foreign jurisdiction, are
valid and effective in the Philippines.

251 | P a g e
Law 321_Corporation LAW_ Case Digest

When to Adopt and File

LOYOLA GRAND VILLAS HOMEOWNERS (SOUTH) ASSOCIATION, INC.


vs.
CA, HOME INSURANCE AND GUARANTY CORPORATION, EMDEN ENCARNACION
and HORATIO AYCARDO
G.R. No. 117188 August 7, 1997

FACTS:

Loyola Grand Villas Homeowners Association (LGVHA) was organized on


February 8, 1983 as the association of homeowners and residents of the Loyola Grand
Villas. It was registered with the Home Financing Corporation, the predecessor of
herein respondent Home Insurance and Guaranty Corporation (HIGC), as the sole
homeowners' organization in the said subdivision. It was organized by the developer of
the subdivision and its first president was Victorio V. Soliven, himself the owner of the
developer. For unknown reasons, however, LGVHAI did not file its corporate by-laws.
Sometime in 1988, the officers of the LGVHAI tried to register its by-laws but
failed to do so. Then the officers that there were two other organizations within the
subdivision the Loyola Grand Villas homeowners North Association Incorporated
(North Association) and the Loyola Grand Villas homeowners South Association
Incorporated (South Association). According to private respondents, a non-resident
and Soliven himself, respectively headed these associations. They also discovered that
these associations had five (5) registered homeowners each who were also the
incorporators, directors and officers thereof. None of the members of the LGVHAI was
listed as member of the North Association while three (3) members of LGVHAI were
listed as members of the South Association. When Soliven inquired about the status of
LGVHAI, Atty. Joaquin A. Bautista, the head of the legal department of the HIGC,
informed him that LGVHAI had been automatically dissolved because it did not submit
its by-laws within the period required by the Corporation Code and there was non-
user of corporate charter because HIGC had not received any report on the
association's activities. Apparently, this information resulted in the registration of the
North and South Association.

ISSUE:

Whether or not failure of LGVHAI to file its by-laws within one month from the
date of its incorporation result in its automatic dissolution.

RULING:

NO.

The Supreme Court ruled that the non-filing of the by-laws within the period of
1 month from the issuance by SEC of the Certificate of Incorporation will not result to
the automatic dissolution of the corporation because the word ―MUST‖ in Sec 46 of the
Corporation Code is merely directory not mandatory in meaning. In fact the second
paragraph allows the filing of by-laws even prior to incorporation.
This provision of the Code rules out mandatory compliance with the
requirement of filing the by-laws "within one (1) month after receipt of official notice of
the issuance of its certificate of incorporation by the Securities and Exchange
Commission." It necessarily follows that failure to file the by-laws within that period
does not imply the "demise" of the corporation. By-laws may be necessary for the
"government" of the corporation but these are subordinate to the articles of
incorporation as well as to the Corporation Code and related statutes.

252 | P a g e
Law 321_Corporation LAW_ Case Digest

Authority to Elect Additional By-Laws Officers

HENRY FLEISCHER
vs.
BOTICA NOLASCO CO., INC.
G.R. No. L-23241. March 14, 1925

FACTS:

On November 15, 1923, the plaintiff filed an amended complaint against the
Botica Nolasco, Inc., alleging that he became the owner of five shares of stock of said
corporation, by purchase from their original owner, one Manuel Gonzalez; that the
said shares were fully paid; and that the defendant refused to register said shares in
his name in the books of the corporation in spite of repeated demands to that effect
made by him upon said corporation, which refusal caused him damages amounting to
P500. The defendant filed a demurrer on the ground that the amended complaint did
not state facts sufficient to constitute a cause of action, and that said amended
complaint was ambiguous, unintelligible, uncertain, which demurrer was overruled by
the court.
The defendant answered the amended complaint denying generally and
specifically each and every one of the material allegations thereof, and, as a special
defense, alleged that the defendant, pursuant to article 12 of its by-laws, had
preferential right to buy from the plaintiff said shares at the par value of P100 a share,
plus P90 as dividends corresponding to the year 1922, and that said offer was refused
by the plaintiff. The defendant prayed for a judgment absolving it from all liability
under the complaint and directing the plaintiff to deliver to the defendant the five
shares of stock in question, and to pay damages.

ISSUE:

Whether or not article 12 of the by-laws of the corporation is in conflict with the
provisions of the Corporation Law (Act No. 1459).

RULING:

YES.

The holder of shares, as owner of personal property, is at liberty, under said


section, to dispose of them in favor of whomsoever he pleases, without any other
limitation in this respect, than the general provisions of law.
Therefore, a stock corporation in adopting a by-law governing transfer of shares
of stock should take into consideration the specific provisions of section 35 of Act No.
1459, and said by-law should be made to harmonize with said provisions. It should
not be inconsistent therewith.
The by-law now in question was adopted under the power conferred upon the
corporation by section 13, paragraph 7, above quoted; but in adopting said by-law the
corporation has transcended the limits fixed by law in the same section, and has not
taken into consideration the provisions of section 35 of Act No. 1459.
As a general rule, the by-laws of a corporation are valid if they are reasonable
and calculated to carry into effect the objects of the corporation, and are not
contradictory to the general policy of the laws of the land
The only restraint imposed by the Corporation Law upon transfer of shares is
found in section 35 of Act No. 1459, quoted above, as follows: "No transfer, however,
shall be valid, except as between the parties, until the transfer is entered and noted
upon the books of the corporation so as to show the names of the parties to the
transaction, the date of the transfer, the number of the certificate, and the number of
shares transferred." This restriction is necessary in order that the officers of the
corporation may know who are the stockholders, which is essential in conducting
elections of officers, in calling meeting of stockholders, and for other purposes. but
any restriction of the nature of that imposed in the by-law now in question, is ultra
vires, violative of the property rights of shareholders, and in restraint of trade.

253 | P a g e
Law 321_Corporation LAW_ Case Digest

JOHN GOKONGWEI, JR., petitioner


vs.
SECURITIES AND EXCHANGE COMMISSION, ANDRES M. SORIANO et.al.
respondents.
G.R. No. L-45911 April 11, 1979.

FACTS:

Petitioner alleged that on September 18, 1976, individual respondents amended


the by-laws of the corporation, basing their authority to do so on a resolution of the
stockholders adopted on March 13, 1961. It was contended that according to section
22 of the Corporation Law and Article VIII of the by-laws of the corporation, the power
to amend, modify, repeal or adopt new by-laws may be delegated to the Board of
Directors only by the affirmative vote of stockholders representing not less than 2/3 of
the subscribed and paid up capital stock of the corporation, which 2/3 should have
been computed on the basis of the capitalization at the time of the amendment. Since
the amendment was based on the 1961 authorization, petitioner contended that the
Board acted without authority and in usurpation of the power of the stockholders.
Petitioner averred that the membership of the Board of Directors had changed since
the authority was given in 1961, there being six (6) new directors.
It was claimed that prior to the questioned March 13, 1961 amendment,
petitioner had all the qualifications to be a director of respondent corporation, being a
substantial stockholder thereof; that as a stockholder, petitioner had acquired rights
inherent in stock ownership, such as the rights to vote and to be voted upon in the
election of directors; and that in amending the by-laws, respondents purposely
provided for petitioner's disqualification and deprived him of his vested right as afore-
mentioned, hence the amended by-laws are null and void.

ISSUE:

Whether or not the disqualification of Gokongwei Jr. to run for directorship of


the corporation valid, as such was only provided in the amended by-laws of the
corporation.

RULING:

YES.

It is recognized by all authorities that 'every corporation has the inherent power
to adopt by-laws 'for its internal government, and to regulate the conduct and
prescribe the rights and duties of its members towards itself and among themselves in
reference to the management of its affairs.'" At common law, the rule was "that the
power to make and adopt by-laws was inherent in every corporation as one of its
necessary and inseparable legal incidents.
Any person "who buys stock in a corporation does so with the knowledge that
its affairs are dominated by a majority of the stockholders and that he impliedly
contracts that the will of the majority shall govern in all matters within the limits of
the act of incorporation and lawfully enacted by-laws and not forbidden by law."
Under section 22 of the same law, the owners of the majority of the subscribed
capital stock may amend or repeal any by-law or adopt new by-laws. It cannot be said,
therefore, that petitioner has a vested right to be elected director, in the face of the fact
that the law at the time such right as stockholder was acquired contained the
prescription that the corporate charter and the by-law shall be subject to amendment,
alteration and modification.
It is a settled that corporations have the power to make by-laws declaring a
person employed in the service of a rival company to be ineligible for the corporation's
Board of Directors. ".An amendment which renders ineligible, or if elected, subjects to
removal, a director if he be also a director in a corporation whose business is in
competition with or is antagonistic to the other corporation is valid."

254 | P a g e
Law 321_Corporation LAW_ Case Digest

THE GOVERNMENT OF THE PHILIPPINE ISLANDS (on relation of the Attorney-


General)
vs.
EL HOGAR FILIPINO
G.R. No. L-26649 July 13, 1927

FACTS:

This case has 17 causes of action proceeded by the Government of the


Philippines through Quo Warranto alleging that El Hogar Filipino, a corporation
organized as a mutual building and loan association under the provisions of the
Corporation Law, has violated or went beyond its stated primary purposes for mutual
building and loan associations. Under Corporation Law Section 171 to 190, inclusive,
of this Act are devoted to the subject of building and loan associations, defining their
objects making various provisions governing their organization and administration,
and providing for the supervision to be exercised over them.. The respondent, El Hogar
Filipino, was apparently the first corporation organized in the Philippine Islands under
the provisions cited, and the association has been favored with extraordinary success.
The articles of incorporation bear the date of December 28, 1910, at which time capital
stock in the association had been subscribed to the amount of P150,000 of which the
sum of P10,620 had been paid in. Under the law as it then stood, the capital of the
Association was not permitted to exceed P3,000,000, but by Act No. 2092, passed
December 23, 1911, the statute was so amended as to permit the capitalization of
building and loan associations to the amount of ten millions. Soon thereafter the
association took advantage of this enactment by amending its articles so as to provide
that the capital should be in an amount not exceeding the then lawful limit. From the
time of its first organization the number of shareholders has constantly increased,
with the result that on December 31, 1925, the association had 5,826 shareholders
holding 125,750 shares, with a total paid-up value of P8,703,602.25. During the
period of its existence prior to the date last above-mentioned the association paid to
withdrawing stockholders the amount of P7,618,257,.72; and in the same period it
distributed in the form of dividends among its stockholders the sum of P7,621,565.81.
As one of the causes of action, the respondent is charged with having a
provision in its by-laws stating that ―The board of directors of the association, by the
vote of an absolute majority of its members, is empowered to cancel shares and to
return to the owner thereof the balance resulting from the liquidation thereof
whenever, by reason of their conduct, or for any other motive, the continuation as
members of the owners of such shares is not desirable‖.

ISSUE:

Whether or not the provision of the by-laws valid.

RULING:

YES.

The by-law is of course a patent nullity, since it is in direct conflict with the
latter part of section 187 of the Corporation Law, which expressly declares that the
board of directors shall not have the power to force the surrender and withdrawal of
unmatured stock except in case of liquidation of the corporation or of forfeiture of the
stock for delinquency. It is agreed that this provision of the by-laws has never been
enforced, and in fact no attempt has ever been made by the board of directors to make
use of the power therein conferred. It appears, however, that no annual meeting of the
shareholders called since that date has been attended by a sufficient number of
shareholders to constitute a quorum, with the result that the provision referred to has
no been eliminated from the by-laws, and it still stands among the by-laws of the
association, notwithstanding its patent conflict with the law.

255 | P a g e
Law 321_Corporation LAW_ Case Digest

Amendment and/or Rejection of By Laws

ENRIQUE SALAFRANCA
vs.
PHILAMLIFE (PVHA VILLAGE, HOMEOWNERS ASSOCIATION, INC.,et al.,
respondents.
G.R. No. 121791 December 23, 1998

FACTS:

Salafranca was hired as Administrative Officer by PVHA on May 1, 1981 and


was extended successive appointments. Sometime in 1987, PVHA decided to amend
its by-laws. Included therein was a provision regarding officers, specifically, the
position of administrative officer under which said officer shall hold office at the
pleasure of the Board of Directors. In a letter dated December 7, 1992, PVHA and
Dazo informed Salafranca that they had decided to discontinue his services. Claiming
that his services had been unlawfully and unceremoniously dispensed with,
Salafranca filed a complaint for illegal dismissal with money claims and for damages.
The LA held that respondents‘ contention that complainant‘s term of
employment was co-terminous with the term of Office of the Board of Directors, is
wanting in merit. The 1987 Amendment would not be applicable to the case of
complainant who had become a regular employee long time before the Amendment
took place. Moreover, the Amendment should be applied prospectively and not
retroactively. On appeal, the NLRC reversed the decision of the LA.

ISSUE:

Whether or not Salafranca was legally dismissed by private respondents


pursuant to the 1987 amendment in the By-laws.

RULING:

NO.

Salafranca had already attained the status of a regular employee, as evidenced


by his eleven years of service with PVHA. Accordingly, petitioner enjoys the right to
security of tenure and his services may be terminated only for causes provided by law.
While PVHA has the right to terminate the services of Salafranca, this is subject to
both substantive and procedural grounds. PVHA failed to substantiate petitioner‘s
dismissal, rendering the latter‘s termination illegal.
In an effort to validate the dismissal of Salafranca, respondents posit the theory
that the latter‘s position is co-terminous with that of the Board of Directors, as
provided for in its amended by-laws. Admittedly, the right to amend the by-laws lies
solely in the discretion of the employer, this being in the exercise of management
prerogative or business judgment. However this right, extensive as it may be, cannot
impair the obligation of existing contracts or rights.
PVHA‘s insistence that it can legally dismiss Salafranca on the ground that his
tenure has expired is untenable. Salaranca, being a regular employee, is entitled to
security of tenure; hence, his services may only be terminated for causes provided by
law. A contrary interpretation would not find justification in the laws or the
Constitution. If the Court was to rule otherwise, it would enable an employer to
remove any employee from his employment by the simple expediency of amending its
by-laws and providing that his/her position shall cease to exist upon the occurrence of
a specified event.
If PVHA wanted to make the Salafranca‘s position co-terminous with that of the
Board of Directors, then the amendment must be effective after Salafranca‘s stay with
PVHA, not during his term. Obviously, the measure taken by the private respondent
in amending its by-laws is nothing but a devious, but crude, attempt to circumvent
Salafranca‘s right to security of tenure as a regular employee guaranteed under the
Labor Code.

256 | P a g e
Law 321_Corporation LAW_ Case Digest

MEETINGS OF STOCKHOLDERS AND THE BOARD OF DIRECTORS


Notice Required

ROSITA PEÑA
vs.
COURT OF APPEALS
G.R. No. 91478 February 7, 1991

FACTS:

Pampanga Bus Co. (PAMBUSCO), original owners of the lots in question,


mortgaged the same to the Development Bank of the Philippines (DBP) in
consideration of P935,000.00. This mortgage was foreclosed. In the foreclosure, the
said properties were awarded to Peña as highest bidder. Thereafter, the board of
directors of PAMBUSCO, through (3) out of its (5) directors, resolved to assign its right
of redemption over the aforesaid lots and authorized one of its members, Atty. Joaquin
Briones "to execute and sign a Deed of Assignment for and in behalf of PAMBUSCO in
favor of any interested party. Consequently, Briones executed a Deed of Assignment of
PAMBUSCO's redemption right over the subject lots in favor of Enriquez. Thereafter,
Enriquez executed a deed of absolute sale of the subject properties in favor of
plaintiffs-appellants, the spouses Rising T. Yap and Catalina Lugue, for the sum of
P140,000.00.
Plaintiffs-appellants, the spouses Rising T. Yap and Catalina Lugue, are the
registered owners of the lots in question. In the complaint filed, appellants sought to
recover possession over the subject lands from defendants Rosita Peña and
Washington Distillery on the ground that being registered owners, they have to enforce
their right to possession against defendants who have been allegedly in unlawful
possession thereof since October 1974 "when the previous owners assigned (their)
right to collect rentals in favor of plaintiffs. After trial, a decision was rendered by the
court in favor of the defendants.

ISSUE:

Whether or not the board resolution of PAMBUSCO is valid.

RULING:

NO.

Under Section 25 of the Corporation Code of the Philippines, the articles of


incorporation or by-laws of the corporation may fix a greater number than the majority
of the number of board members to constitute the quorum necessary for the valid
transaction of business. Any number less than the number provided cannot constitute
a quorum and any act therein would not bind the corporation; all that the attending
directors could do is to adjourn.
Records show that PAMBUSCO ceased to operate as of November 15, 1949.
Being a dormant corporation for several years, it was highly irregular, if not
anomalous, for a group of three (3) individuals representing themselves to be the
directors of PAMBUSCO to pass a resolution disposing of the only remaining asset of
the corporation in favor of a former corporate officer. As a matter of fact, the three (3)
alleged directors who attended the said meeting were not listed as directors of
respondent PAMBUSCO. Furthermore, PAMBUSCO was insolvent and its only
remaining asset was its right of redemption over the subject properties. Since the
disposition of said redemption right of respondent PAMBUSCO by virtue of the
questioned resolution was not approved by the required number of stockholders under
the law, the said resolution, as well as the subsequent assignment to respondent
Enriquez should be struck down as null and void.

257 | P a g e
Law 321_Corporation LAW_ Case Digest

Quorum Required

THE BOARD OF DIRECTORS And ELECTION COMMITTEE OF THE SMB


WORKERS SAVINGS AND LOAN ASSOCIATION, INC., ET AL.
vs.
HON BIENVENIDO A. TAN, ETC., ET AL.
G.R. No. L-12282 1959 March 31

FACTS:

On January 17, 1957 John de Castillo et al., commenced a suit in the Court of
First Instance of Manila to declare null and void the election of the members of the
board of directors of the SMB Workers Savings and Loan Association, Inc. and of the
members of the Election Committee for the year 1957 held on January 11 and 12 and
to compel the board of directors of the association to call for and hold another election
in accordance with its constitution and by-laws and the Corporation Law. Such was
granted by the court, however, another suit was filed alleging that the subsequent
meeting for the elections would not be in accordance with the constitution and by-laws
regarding notice to the stockholders.

ISSUE:

Whether or not proper notice was given as regards the new meeting for the
elections of the board of directors.

RULING:

NO.

There was no proper notice. Notice of a special meeting of members should be


given at least five days before the date of the meeting. It appears that the notice was
posted on 26 March and the election was set for 28 March. Therefore, the five days
previous notice required would not be complied with.

258 | P a g e
Law 321_Corporation LAW_ Case Digest

Who Presides

ABELARDO JAVELLANA, TOMAS JONCO, et al., in their capacities as Councilors


of the Municipal Municipality of Buenavista, Province of Iloilo
vs.
SUSANO TAYO, as Mayor of the Municipal Municipality of Buenavista, Iloilo
G.R. No. L-18919 December 29, 1962

FACTS:

Petitioners were members of the municipal council. On several sessions, the


mayor, herein defendant, was absent prompting the council to decide emong
themselves as to who to appoint as presiding officers. The mayor refused to act on the
resulting minutes also refused to sign the payrolls of the council covering the per
diems of the petitioners, alleging that the proceedings were illegal due to his absence.
Despite the Provincial Fiscal and the Provincial Board upholding the
controverted sessions of the Municipal Council, the Mayor refused and still refuses to
recognize the validity of the acts of the Municipal Council and the legality of its regular
session held in his absence.
The trial court ruled that attendance of the Mayor is not essential to the validity
of the session as long as there is quorum constituted in accordance with law. To
declare that the proceedings of the petitioners were null and void, is to encourage
recalcitrant public officials who would frustrate valid sessions for political end or
consideration.

ISSUE:

Whether or not the sessions held by petitioners were valid and legal, having
constituted a quorum, and despite the absence of the defendant.

RULING:

YES.

The term "quorum" has been defined as "that number of members of the body
which, when legally assembled in their proper places, will enable the body to transact
its proper business, or, in other words, that number that makes a lawful body and
gives it power to pass a law or ordinance or do any other valid corporate act.
The Revised Administrative Code states that for the majority of the members of
the council to constitute a quorum to do business, the council "shall be presided by
the Mayor and no one else.
The procedure, as provided in the Administrative Code, provides that in case of
temporary incapacity of the mayor, the council member having the highest number of
votes can sit as presiding officer. This rule on incapacity was declared as valid by the
court in the case. Thus, the quorum requirement was satisfied despite the continuous
absence of the mayor on those scheduled sessions.
Thus, the questioned sessions and the resulting resolutions were declared valid.

259 | P a g e
Law 321_Corporation LAW_ Case Digest

Who Could Attend and Vote

JULIO E. T. SALES and GEORGE V. AGONIAS and SMEC


vs.
SEC, SIHI and ATCO, represented by its President, ANSELMO TRINIDAD; VIMC,
represented by its President, et al.
G.R. No. 54330 January 13, 1989

FACTS:

SMEC sold 200M common shares of its capital stock in the amount of P2.6M to
SIHI under a Sales Agreement providing that the sale shall be only up to 5m shares
per buyer. SIHI requested for the transfer of the 200M shares to ATCO to which SMEC
complied. During the time that ATCO held the shares, it voted them in the SHs'
meetings of SMEC. ATCO in turn sold 198,500,000 of the shares to respondent VIMC.
Upon request, SMEC BOD issued a resolution directing its President to sign the
certificate of stock that would effect the transfer. Before the 1979 annual SH meeting
of SMEC, petitioners sought to nullify the sales of the shares to VIMC with the SEC
and to enjoin VIMC from voting the said shares. VIMC was temporarily restrained and
the meeting was held without the participation of VIMC‘s shares and BODs were
elected only from the group of petitioners. In VIMC‘s answer, it questioned the said
election. SEC denied the petition as well as motion to dismiss and lifted the
Restraining it issued earlier and allowed the shares of VIMC to be counted in
determining the quorum of the 1980 annual SHs meeting, which was already near,
and the same shares were allowed to vote and be voted for. Before the SC, petitioners
contended that the SEC gravely abused its discretion in not enjoining the participation
of VIMC in the 1980 election considering that the sale of the shares to VIMC was null
and void as it was done in violation of the Sales Agreement on the limit of shares to be
sold to each buyer and that VIMC‘s ownership of the shares is contrary to Sec. 13 (5-
A) of the old corporation law.

ISSUE:

Whether or not SEC acted with grave abuse of discretion in not permanently
enjoining VIMC in voting.

RULING:

NO.

SC found no grave abuse of discretion on the part of the SEC in not restraining
VIMC. It adopted the SEC resolution stating that the sale of the shares of stock had
long been perfected and is presumed valid until declared otherwise. As against this
presumption, petitioners' prayer for injunction cannot prevail as the issue of the
validity of the sale is still to be resolved by the SEC.
Considering that the shares constitute the majority, it is more equitable that
the same be allowed to vote rather than be enjoined. As it has been ruled the removal
of a majority SH from the management of the corporation and/or the dissolution of a
corporation in a suit filed by a minority SH is a drastic measure. It should be resorted
to only when the necessity is clear. With more reason, the Court will not deprive a SH
of his right to vote his shares in the annual SHs' meeting, except upon a clear showing
of its lawful denial under the articles of incorporation or by-laws of the corporation, as
it is a right inherent in stock ownership.

260 | P a g e
Law 321_Corporation LAW_ Case Digest

DOMINGO PONCE AND BUHAY PONCE


vs.
DEMETRIO B. ENCARNACION AND POTENCIANO GAPOL
G.R. NO. L-5883 NOVEMBER 28, 1953

FACTS:

Daguhoy Enterprises, Inc., was duly registered as such on 24 June 1948. On


16 April 1951 at a meeting duly called, the voluntary dissolution of the corporation
and the appointment of Gapol as receiver were agreed upon and to that end a petition
for voluntary dissolution was drafted which was sent to, and signed by, the petitioner
Domingo Ponce. Instead of filing the petition for voluntary dissolution of the
corporation as agreed upon, Gapol, who is the largest stockholder, changed his mind
and filed a complaint in the CFI of Manila to compel the petitioners to render an
accounting of the funds and assets of the corporation, to reimburse it, jointly and
severally, a total sum of P18,690, plus interest, which have been converted by the
petitioner Domingo Ponce to his own use and benefit.
On 18 May 1951 Gapol filed a motion praying that the petitioners be removed
as members of the board of directors which was denied by the court. On 3 January
1952 Gapol filed a petition praying for an order directing him to call a meeting of the
stockholders of the corporation and to preside at such meeting in accordance with
section 26 of the Corporation Law. Two-days later, without notice to the petitioners
and to the other members of the board of directors and in violation of the Rules of
Court which require that the adverse parties be notified of the hearing of the motion
three days in advance, the respondent court issued the order as prayed for.

ISSUE:

Whether or not under and pursuant to section 26 of the Corporation Law, the
respondent court may issue the order complained of.

RULING:

NO.

Article 9 of the by-laws of the Daguhoy Enterprises, Inc., provides: The Board of
Directors shall compose of five (5) members who shall be elected by the stockholders
in a general meeting called for that purpose which shall be held every even year during
the month of January. Article 22 of the by-laws provides: The Chairman shall have the
right to fix the date, the time and the place where the general meeting shall be held,
either special or general.
Section 26 of the Corporation Code provides: - Whenever, from any cause, there
is no person authorized to call a meeting, or when the officer authorized to do so
refuses, fails, or neglects to call a meeting, any judge of a Court of First Instance, on
the showing of good cause therefor, may issue an order to any stockholder or member
of a corporation, directing him to call a meeting of the corporation by giving the proper
notice required by this Act or the by-laws; and if there be no person legally authorized
to preside at such meeting, the judge of the Court of First Instance may direct the
person calling the meeting to preside at the same until a majority of the members or
stockholders representing a majority of the stock present and permitted by law to be
voted have chosen one of their number to act as presiding officer for the purposes of
the meeting.
Petitioners were not deprived of their right without due process of law. They had
no right to continue as directors of the corporation unless reelected by the
stockholders in a meeting called for that purpose every even year.

261 | P a g e
Law 321_Corporation LAW_ Case Digest

SALVADOR P. LOPEZ
vs.
ERICTA
G.R. No. L-32991 June 29, 1972

FACTS:

The first such appointment was extended on April 27, 1970, "effective May 1,
1970 until April 30, 1971, unless sooner terminated and subject to the appproval of
the Board of Regents and to pertinent University regulations." Pursuant thereto Dr.
Blanco assumed office as ad interim Dean on May 1, 1970.
The Board of Regents met on May 26, 1970, and President Lopez submitted to it
the ad interim appointment of Dr. Blanco for reconsideration. The minutes of that
meeting disclose that "the Board voted to defer action on the matter in view of the
objections cited by Regent Kalaw based on the petition against the appointment,
addressed to the Board, from a majority of the faculty and from a number of alumni
Dr. Blanco's appointment had lapsed.
On May 26, 1970, President Lopez extended another ad interim appointment to
her, effective from May 26, 1970 to April 30, 1971, with the same conditions as the
first.However, such ad interim appointment had not been confirmed by the Board of
Regents. Due to the following votes: 5-yes, 3-no and 4-abstain.
On August 18, 1970 Dr. Blanco wrote the President of the University, protesting
the appointment of Oseas A. del Rosario as Officer-in-Charge of the College of
Education. Neither communication having elicited any official reply, Dr. Blanco went
to the Court of First Instance of Quezon City on a petition for certiorari and prohibition
with preliminary injunction.

ISSUE:

Whether or not respondent Dr. Consuelo S. Blanco was duly elected Dean of the
College of Education, University of the Philippines, in the meeting of the Board of
Regents on July 9, 1970.

RULING:

NO.

The votes of abstention, viewed in their setting, can in no way be construed as


votes for confirmation of the appointment. There can be no doubt whatsoever as to the
decision and recommendation of the three members of the Personnel Committee: it
was for rejection of the appointment. No inference can be drawn from this that the
members of the Personnel Committee, by their abstention, intended to acquiesce in
the action taken by those who voted affirmatively. Neither, for that matter, can such
inference be drawn from the abstention that he was abstaining because he was not
then ready to make a decision.
Dr. Blanco was clearly not the choice of a majority of the members of the Board
of Regents, as unequivocally demonstrated by the transcript of the proceedings. This
fact cannot be ignored simply because the Chairman, in submitting the question to
the actual vote, did not frame it as accurately as the preceding discussion called for,
such that two of the Regents present (Silva and Kalaw) had to make some kind of
clarification.

262 | P a g e
Law 321_Corporation LAW_ Case Digest

VOTING
Who May Exercise

Wilson P. Gamboa
vs.
Finance Secretary Margarito Teves, et al.,
G.R. No. 176579, June 28, 2011

FACTS:

This is a petition to nullify the sale of shares of stock of Philippine


Telecommunications Investment Corporation (PTIC) by the government of the Republic
of the Philippines, acting through the Inter-Agency Privatization Council (IPC), to
Metro Pacific Assets Holdings, Inc. (MPAH), an affiliate of First Pacific Company
Limited (First Pacific), a Hong Kong-based investment management and holding
company and a shareholder of the Philippine Long Distance Telephone Company
(PLDT). The petitioner questioned the sale on the ground that it also involved an
indirect sale of 12 million shares (or about 6.3 percent of the outstanding common
shares) of PLDT owned by PTIC to First Pacific. With the this sale, First Pacific‘s
common shareholdings in PLDT increased from 30.7 percent to 37 percent, thereby
increasing the total common shareholdings of foreigners in PLDT to about
81.47%. This, according to the petitioner, violates Section 11, Article XII of the 1987
Philippine Constitution which limits foreign ownership of the capital of a public utility
to not more than 40%, thus: Section 11. No franchise, certificate, or any other form of
authorization for the operation of a public utility shall be granted except to citizens of
the Philippines or to corporations or associations organized under the laws of the
Philippines, at least sixty per centum of whose capital is owned by such citizens; nor
shall such franchise, certificate, or authorization be exclusive in character or for a
longer period than fifty years. Neither shall any such franchise or right be granted
except under the condition that it shall be subject to amendment, alteration, or repeal
by the Congress when the common good so requires. The State shall encourage equity
participation in public utilities by the general public. The participation of foreign
investors in the governing body of any public utility enterprise shall be limited to their
proportionate share in its capital, and all the executive and managing officers of such
corporation or association must be citizens of the Philippines. (Emphasis supplied)

ISSUE:

Whether or not the term ―capital‖ in Section 11, Article XII of the Constitution
refer to the total common shares only, or to the total outstanding capital stock
(combined total of common and non-voting preferred shares) of PLDT, a public utility.

HELD:

YES.

Considering that common shares have voting rights which translate to control,
as opposed to preferred shares which usually have no voting rights, the term ―capital‖
in Section 11, Article XII of the Constitution refers only to common shares. However, if
the preferred shares also have the right to vote in the election of directors, then the
term ―capital‖ shall include such preferred shares because the right to participate in
the control or management of the corporation is exercised through the right to vote in
the election of directors. In short, the term ―capital‖ in Section 11, Article XII of the
Constitution refers only to shares of stock that can vote in the election of directors.
To construe broadly the term ―capital‖ as the total outstanding capital stock,
including both common and non-voting preferred shares, grossly contravenes the
intent and letter of the Constitution that the ―State shall develop a self-reliant and
independent national economy effectively controlled by Filipinos.‖ A broad definition
unjustifiably disregards who owns the all-important voting stock, which necessarily
equates to control of the public utility.

263 | P a g e
Law 321_Corporation LAW_ Case Digest

Holders of PLDT preferred shares are explicitly denied of the right to vote in the
election of directors. PLDT‘s Articles of Incorporation expressly state that ―the holders
of Serial Preferred Stock shall not be entitled to vote at any meeting of the
stockholders for the election of directors or for any other purpose or otherwise
participate in any action taken by the corporation or its stockholders, or to receive
notice of any meeting of stockholders.‖ On the other hand, holders of common shares
are granted the exclusive right to vote in the election of directors. PLDT‘s Articles of
Incorporation state that ―each holder of Common Capital Stock shall have one vote in
respect of each share of such stock held by him on all matters voted upon by the
stockholders, and the holders of Common Capital Stock shall have the exclusive right
to vote for the election of directors and for all other purposes.‖
It must be stressed, and respondents do not dispute, that foreigners hold a
majority of the common shares of PLDT. In fact, based on PLDT‘s 2010 General
Information Sheet (GIS), which is a document required to be submitted annually to the
Securities and Exchange Commission, foreigners hold 120,046,690 common shares of
PLDT whereas Filipinos hold only 66,750,622 common shares. In other words,
foreigners hold 64.27% of the total number of PLDT‘s common shares, while Filipinos
hold only 35.73%. Since holding a majority of the common shares equates to control,
it is clear that foreigners exercise control over PLDT. Such amount of control
unmistakably exceeds the allowable 40 percent limit on foreign ownership of public
utilities expressly mandated in Section 11, Article XII of the Constitution.
As shown in PLDT‘s 2010 GIS, as submitted to the SEC, the par value of PLDT
common shares is P5.00 per share, whereas the par value of preferred shares
is P10.00 per share. In other words, preferred shares have twice the par value of
common shares but cannot elect directors and have only 1/70 of the dividends of
common shares. Moreover, 99.44% of the preferred shares are owned by Filipinos
while foreigners own only a minuscule 0.56% of the preferred shares. Worse, preferred
shares constitute 77.85% of the authorized capital stock of PLDT while common
shares constitute only 22.15%. This undeniably shows that beneficial interest in PLDT
is not with the non-voting preferred shares but with the common shares, blatantly
violating the constitutional requirement of 60 percent Filipino control and Filipino
beneficial ownership in a public utility.

264 | P a g e
Law 321_Corporation LAW_ Case Digest

PHILIPPINE COCONUT PRODUCERS FEDERATION, INC. (COCOFED), et al.


vs.
REPUBLIC OF THE PHILIPPINES
G.R. Nos. 177857-58 February 11, 2010

FACTS:

The Court, in its earlier resolution adverted to, approved, upon motion of
petitioner Philippine Coconut Producers Federation, Inc. (COCOFED), the conversion
of the sequestered 753,848,312 Class "A" and "B" common shares of San Miguel
Corporation (SMC), registered in the name of Coconut Industry Investment Fund (CIIF)
Holding Companies (hereunder referred to as SMC Common Shares), into 753,848,312
SMC Series 1 Preferred Shares. The oppositors herein made the following arguments:
(1) economic disadvantage and harm that government might suffer by such proposed
conversion; (2) they question the wisdom of PCGG in converting those sequestered
shares; (3) that the conversion is invalid in view of the Commission on Audit Circular
No. 89- 296 which provides that disposal of government property must be undertaken
via public Auction; (4) that the conversion thereof needs the acquiescence of the 14
CIIF companies; (5) As to the Motion to Intervene by UCPB, that it should be the sole
depositary of the proceeds of the dividends.

ISSUE:

Whether or not the arguments of the Oppositors herein have merits.

RULING:

NO.

Anent the 1st contention, it is not tenable because in fact this conversion is a
business strategy to preserve and conserve the value of the Government‘s interest in
the CIIF SMC shares. As to the 2nd argument, it is also untenable because it is not
within the Courts to determine wisdom of other agencies of the government. As to the
3rd argument, likewise untenable because FIRST, there is really no disposal of SMC
shares and SECOND, there is no yet government assets to talk about because the
ownership thereto is still to be determined, hence, those shares are akin to properties
subject of attachment. As to the 4th contention, PCGG need not obtain the
acquiescence of the owners of those sequestered shares with respect to any of its acts
intended to preserve such assets. Otherwise, it would be impossible for it to perform
its function as provided by law. And as to the 5th argument, it is also of no merit
because the Court has the discretion where to deposit those net dividends, whether it
be on Development Bank of the Philippines/ Land Bank of the Philippines or the
UCPB.

265 | P a g e
Law 321_Corporation LAW_ Case Digest

REPUBLIC OF THE PHILIPPINES, represented by the PRESIDENTIAL


COMMISSION ON GOOD GOVERNMENT (PCGG)
vs.
COCOFED, ET AL. and BALLARES, ET AL., EDUARDO M. COJUANGCO JR. and
the SANDIGANBAYAN (First Division)
G.R. No. 147062-64 December 14, 2001

FACTS:

On the explicit premise that 'vast resources of the government have been
amassed by former President Ferdinand E. Marcos, his immediate family, relatives,
and close associates both here and abroad,' the Presidential Commission on Good
Government (PCGG) was created by Executive Order No. 1 to assist the President in
the recovery of the ill-gotten wealth thus accumulated whether located in the
Philippines or abroad. Several executive orders were then issued describing the
properties to be recovered.Among the properties sequestered by the Commission were
shares of stock in the United Coconut Planters Bank (UCPB) registered in the names
of the alleged "one million coconut farmers," the so-called Coconut Industry
Investment Fund companies (CIIF companies) and Private Respondent Eduardo
Cojuangco Jr.
Six years later, on February 13, 2001, the Board of Directors of UCPB received
from the ACCRA Law Office a letter written on behalf of the COCOFED and the alleged
nameless one million coconut farmers, demanding the holding of a stockholders'
meeting for the purpose of, among others, electing the board of directors. In response,
the board approved a Resolution calling for a stockholders' meeting on March 6, 2001
at three o'clock in the afternoon. However, the same was meted by a Class Action
Omnibus Motion seeking to enjoin PCGG from voting the UCPB shares of stock
registered in the respective names of the more than one million coconut farmers; and
to enjoin the PCGG from voting the SMC shares registered in the names of the 14 CIF
holding companies including those registered in the name of the PCGG.

ISSUE:

Whether or not PCGG may vote the sequestered UCPB shares while the main
case for their reversion to the State is pending in the Sandiganbayan.

RULING:

YES.

The SC holds that the government should be allowed to continue voting those
shares inasmuch as they were purchased with coconut levy funds since those are
prima facie public in character or, at the very least, are "clearly affected with public
interest."
The general rule is that the registered owner of the shares of a corporation
exercises the right and the privilege of voting. This principle applies even to shares
that are sequestered by the government, over which the PCGG as a mere conservator
cannot, as a general rule, exercise acts of dominion. On the other hand, it is
authorized to vote these sequestered shares registered in the names of private persons
and acquired with allegedly ill-gotten wealth, if it is able to satisfy the two-tiered test.
Unfortunately, this test is not applicable under the circumstances of this case.
Hence, the Court granted PCGG the right to vote the sequestered shares because they
appeared to be assets belonging to the government itself.

266 | P a g e
Law 321_Corporation LAW_ Case Digest

RAMON C. LEE and ANTONIO DM. LACDAO


vs.
THE HON. COURT OF APPEALS, SACOBA MANUFACTURING CORP., PABLO
GONZALES, JR. and THOMAS GONZALES
G.R. No. 93695 February 4, 1992

FACTS:

In 1985, a complaint for sum of money was filed by the International Corporate
Bank, Inc. against the private respondents who, in turn, filed a third party complaint
against Alfa Integrated Textile Mills (ALFA) and the petitionersRamon C. Lee and
Antonio Dm. Lacdao who were officers of ALFA. Meanwhile, in 1988, the trial court
issued an order requiring the issuance of an alias summons upon ALFA through the
DBP as a consequence of the petitioners' letter informing the court that the summons
for ALFA was erroneously served upon them considering that the management of ALFA
had been transferred to the Development Bank of the Philippines (DBP).
In a manifestation, the DBP claimed that it was not authorized to receive
summons on behalf of ALFA since the DBP had not taken over the company which has
a separate and distinct corporate personality and existence.

ISSUE:

Whether or not despite the execution of the Voting Trust Agreement, the
summons be served upon the petitioners who were officers and directors of ALFA (the
trustor).

RULING:

NO.

There is no dispute as to the most immediate effect of a voting trust agreement


on the status of a stockholder who is a party to its execution from legal titleholder or
owner of the shares subject of the voting trust agreement, he becomes the equitable or
beneficial owner.
Note that in order to be eligible as a director, what is material is the legal title
to, not beneficial ownership of, the stock as appearing on the books of the corporation
Considering that the voting trust agreement between ALFA and the DBP transferred
legal ownership of the stocks covered by the agreement to the DBP as trustee, the
latter became the stockholder of record with respect to the said shares of stocks. In
the absence of a showing that the DBP had caused to be transferred in their names
one share of stock for the purpose of qualifying as directors of ALFA, the petitioners
can no longer be deemed to have retained their status as officers of ALFA which was
the case before the execution of the subject voting trust agreement. There appears to
be no dispute from the records that DBP has taken over full control and management
of the firm.

267 | P a g e
Law 321_Corporation LAW_ Case Digest

Republic of the Philippines (Presidential Commission on Good Government),


vs.
Sandiganbayan
GR 107789 30 April 2003

FACTS:

On 7 August 1991, the PCGG conducted an Eastern Telecommunications,


Philippines, Inc. (ETPI) stockholders meeting during which a PCGG controlled board of
directors was elected. A special stockholders meeting was later convened by the
registered ETPI stockholders wherein another set of board of directors was elected, as
a result of which two sets of such board and officers were elected. Victor Africa, a
stockholder of ETPI, alleging that the PCGG had since been "illegally 'exercising' the
rights of stockholders of ETPI," especially in the election of the members of the board
of directors, filed a motion before the Sandiganbayan, prayed that said court order the
"calling and holding of the ETPI annual stockholders meeting for 1992 under the
court's control and supervision and prescribed guidelines." The PCGG did not object to
Africa's motion provided that "(1) An Order be issued upholding the right of PCGG to
vote all the Class "A" shares of ETPI; (2) In the alternative, in the remote event that
PCGG's right to vote the sequestered shares be not upheld, an Order be issued (a)
disregarding the Stock and Transfer Book and Booklet of Stock Certificates of ETPI in
determining who can vote the shares in an Annual Stockholders Meeting of ETPI, (b)
allowing PCGG to vote 23.9% of the total subscription in ETPI, and (c) directing the
amendment of the Articles of Incorporation and By-laws of ETPI providing for the
minimum safeguards for the conservation of assets prior to the calling of a
stockholders meeting. The Sandiganbayan resolved Africa's motion, ordering the
conduct of an annual stockholders meeting of ETPI, for 1992. Assailing the foregoing
resolution, the PCGG filed before the Supreme Court a petition for Certiorari,
Mandamus and Prohibition.

ISSUE:

Whether or notthe PCGG can vote the sequestered ETPI Class "A" shares in the
stockholders meeting for the election of the board of directors.

RULING:

YES.

The PCGG cannot vote sequestered shares to elect the ETPI Board of Directors
or to amend the Articles of Incorporation for the purpose of increasing the authorized
capital stock unless there is a prima facie evidence showing that said shares are ill-
gotten and there is an imminent danger of dissipation. (2)The ETPI Stock and Transfer
Book should be the basis for determining which persons have the right to vote in the
stockholders meeting for the election of the ETPI Board of Directors. (3) The PCGG is
entitled to vote the shares ceded to it by Roberto S. Benedicto and his controlled
corporations under the Compromise Agreement, provided that the shares are first
registered in the name of the PCGG. The PCGG may not register the transfer of the
Malacañang and the Nieto shares in the ETPI Stock and Transfer Book; however, it
may vote the same as conservator provided that the PCGG satisfies the two-tiered test
devised by the Court in Cojuangco v. Calpo. (4) The safeguards laid down in the case
of Cojuangco v. Roxas shall be incorporated in the ETPI Articles of Incorporation
substantially contemporaneous to, but not before, the election of the ETPI Board of
Directors. (5) Members of the Sandiganbayan shall not participate in the stockholders
meeting for the election of the ETPI Board of Directors.

268 | P a g e
Law 321_Corporation LAW_ Case Digest

Voting Trust Agreement

ROSAURA P. CORDON
vs.
JESUS BALICANTA
A.C. No. 2797 October 4, 2002

FACTS:

Sometime in the early part of 1981, respondent enticed complainant and her
daughter to organize a corporation that would develop the said real properties into a
high-scale commercial complex with a beautiful penthouse for complainant. Relying
on these apparently sincere proposals, complainant and her daughter assigned 19
parcels of land to Rosaura Enterprises, Incorporated, and a newly-formed and duly
registered corporation in which they assumed majority ownership. The subject
parcels of land were then registered in the name of the corporation.
Thereafter, respondent single-handedly ran the affairs of the corporation in his
capacity as Chairman of the Board, President, General Manager and Treasurer. The
respondent also made complainant sign a document which turned out to be a voting
trust agreement. Respondent likewise succeeded in making complainant sign a
special power of attorney to sell and mortgage some of the parcels of land she
inherited from her deceased husband. She later discovered that respondent
transferred the titles of the properties to a certain Tion Suy Ong who became the new
registered owner thereof. Respondent never accounted for the proceeds of said
transfers. Other spurious transactions not approved by the Board were entered into by
the defendant through spurious board resolutions.

ISSUE:

Whether or not there was really a voting trust agreement made by the
complainant in favor of the defendant.

RULING:

NO.

The claim is baseless. The voting trust referred to by respondent, even if it were
assumed to be valid, covered only 266 shares of complainants yet she owned a total of
1,039 shares after she and her daughter ceded in favor of the corporation 19 parcels of
land. Being a former lawyer to complainant, respondent should have ensured that her
interest was safeguarded. Yet, complainant was apparently and deliberately left it on
the pretext that, she had executed a voting trust agreement in favor of respondent. It
is suspicious that complainant was made to sign a voting trust agreement on 21
August 1981 and immediately thereafter, the resolutions authorizing respondent to
obtain a loan and to mortgage the 9 parcels of land were passed and approved. It is
further worth noting that complainant‘s voting trust where she allegedly entrusted 266
shares to respondent on August 21, 1981 had only a validity of 5 years. Thus, she
should have had her entire holdings of 1,283 shares back in her name in August
1986.―Respondent‘s purported minutes of stockholders‘ meeting do not reflect this.
―There was no explanation whatsoever from respondent on how complainant and her
daughter lost their 97% control holding in the corporation. Respondent cannot take
refuge in the contested voting trust agreement supposedly executed by complainant
and her daughter for the reason that it authorized respondent to represent
complainant for such matters.
Moreover the factual findings of the investigating commission, affirmed by the
IBP Board, disclosed that complainant and her daughter own 1,711 out of 1,750
shares of the outstanding capital stock of the corporation, based on the Articles of
Incorporation and deeds of transfer of the properties. But respondent‘s evidence
showed that complainant had only 266 shares of stock in the corporation while her
daughter had none, notwithstanding the fact that there was nothing to indicate that
complainant and her daughter ever conveyed their shares to others.

269 | P a g e
Law 321_Corporation LAW_ Case Digest

NATIONAL INVESTMENT AND DEVELOPMENT CORPORATION, EUSEBIO


VILLATUYA MARIO Y. CONSING and ROBERTO S. BENEDICTO
vs.
HON. BENJAMIN AQUINO, et al.
G.R. No. L-34192 June 30, 1988

FACTS:

Batjak, is a Filipino-American corporation which has indebtedness to Philippine


National Bank (PNB) amounted to P11,915,000.00, As security for the payment of its
obligations and advances against shipments, Batjak mortgaged its three (3) coco-
processing oil mills to Manila Bank, Republic Bank , and PCIB, respectively. In need
for additional operating capital to place the three (3) coco-processing mills at their
optimum capacity and maximum efficiency and to settle, pay or otherwise liquidate
pending financial obligations with the different private banks, Batjak applied to PNB
for additional financial assistance. A Financial Agreement was submitted by PNB to
Batjak for acceptance which was duly accepted by Batjak. Upon receiving payment,
RB, PCIB, and MBTC released in favor of PNB the first and any mortgages they held on
the properties of Batjak. Batjak executed a first mortgage in favor of PNB on all its
properties A Voting Trust Agreement was executed in favor of NIDC by the
stockholders representing 60% of the outstanding paid-up and subscribed shares of
Batjak. This agreement was for a period of five (5) years and, upon its expiration, was
to be subject to negotiation between the parties. Forced by the insolvency of Batjak,
PNB instituted extrajudicial foreclosure proceedings against the oil mills of Batjak. The
properties were sold to PNB as the highest bidder. Three years thereafter, Batjak wrote
a letter to NIDC inquiring if the latter was still interested in negotiating the renewal of
the Voting Trust Agreement. Batjak wrote another letter to NIDC informing the latter
that Batjak would now safely assume that NIDC was no longer interested in the
renewal of said Voting Trust Agreement.

ISSUE:

Whether or not the NIDC and PNB acquired ownership over the assets of Batjak
despite a voting trust agreement between Batjak‘s stockholders and NIDC.

RULING:

YES.

What was assigned to NIDC was the power to vote the shares of stock of the
stockholders of Batjak, representing 60% of Batjak's outstanding shares, and who are
the signatories to the agreement. The power entrusted to NIDC also included the
authority to execute any agreement or document that may be necessary to express the
consent or assent to any matter, by the stockholders. Nowhere in the said provisions
or in any other part of the Voting Trust Agreement is mention made of any transfer or
assignment to NIDC of Batjak's assets, operations, and management. NIDC was
constituted as trustee only of the voting rights of 60% of the paid-up and outstanding
shares of stock in Batjak. Under the provision on termination what was to be returned
by NIDC as trustee to Batjak's stockholders, upon the termination of the agreement,
are the certificates of shares of stock belonging to Batjak's stockholders, not the
properties or assets of Batjak itself which were never delivered, in the first place to
NIDC, under the terms of said Voting Trust Agreement. A voting trust transfers only
voting or other rights pertaining to the shares subject of the agreement or control over
the stock hence the acquisition by PNB-NIDC of the properties in question was not
made or effected under the capacity of a trustee but as a foreclosing creditor for the
purpose of recovering on a just and valid obligation of Batjak.

270 | P a g e
Law 321_Corporation LAW_ Case Digest

LEON J. LAMBERT
vs.
T. J. FOX
G.R. No. L-7991 January 29, 1914

FACTS:

Early in 1911 the firm known as John R. Edgar & Co., engaged in the retail
book and stationery business, found itself in such condition financially that its
creditors, including the plaintiff and the defendant, together with many others, agreed
to take over the business, incorporate it and accept stock therein in payment of their
respective credits. This was done, the plaintiff and the defendant becoming the two
largest stockholders in the new corporation called John R. Edgar & Co., Incorporated.
A few days after the incorporation was completed plaintiff and defendant entered into
an agreement whereby the shockholders mutually and reciprocally agree not to sell,
transfer, or otherwise dispose of any part of their present holdings of stock in said
John R. Edgar & Co. Inc., till after one year from the date hereof and that Either party
violating this agreement shall pay to the other the sum of one thousand (P1,000) pesos
as liquidated damages, unless previous consent in writing to such sale, transfer, or
other disposition be obtained.
Notwithstanding this contract the defendant Fox on October 19, 1911, sold his
stock in the said corporation to E. C. McCullough of the firm of E. C. McCullough &
Co. of Manila, a strong competitor of the said John R. Edgar & Co., Inc. This sale was
made by the defendant against the protest of the plaintiff and with the warning that he
would be held liable under the contract hereinabove set forth and in accordance with
its terms. In fact, the defendant Foz offered to sell his shares of stock to the plaintiff
for the same sum that McCullough was paying them less P1,000, the penalty specified
in the contract.

ISSUE:

Whether or not the suspension of the power to sell the stock is valid and legal.

RULING:

YES.

The suspension of the power to sell has a beneficial purpose, results in the
protection of the corporation as well as of the individual parties to the contract, and is
reasonable as to the length of time of the suspension. We do not here undertake to
discuss the limitations to the power to suspend the right of alienation of stock,
limiting ourselves to the statement that the suspension in this particular case is legal
and valid.

271 | P a g e
Law 321_Corporation LAW_ Case Digest

CAPITAL STRUCTURE STOCKS AND STOCKHOLDERS


As Legal/Stated Capital: Trust Fund Doctrine

PHILIPPINE LONG DISTANCE TELEPHONE COMPANY


vs.
NATIONAL TELECOMMUNICATIONS COMMISSION, JOSEPH A.SANTIAGO, in his
capacity as NTC Commissioner, and EDGARDO CABARRIOS
G.R. No. 152685 4 December 2007

FACTS:

Case pertains to Section 40 (e) the Public Service Act (PSA), as amended on
March 15, 1984, pursuant to Batas Pambansa Blg This. 325, which authorized the
NTC to collect from public telecommunications companies Supervision and Regulation
Fees (SRF) of PhP 0.50 for every PhP 100 or a fraction of the capital and stock
subscribed or paid for of a stock corporation, partnership or single proprietorship of
the capital invested, or of the property and equipment, whichever is higher. Under
Section 40 (e) of the PSA, the NTC sent SRF assessments to petitioner Philippine Long
Distance Telephone Company (PLDT) starting sometime in 1988. The SRF
assessments were based on the market value of the outstanding capital stock,
including stock dividends, of PLDT. PLDT protested the assessments contending that
the SRF ought to be based on the par value of its outstanding capital stock. Its protest
was denied by the NTC and likewise, its motion for reconsideration. PLDT appealed
before the CA. The CA modified the disposition of the NTC by holding that the SRF
should be assessed at par value of the outstanding capital stock of PLDT, excluding
stock dividends.

ISSUE:

Whether or not the value transferred from the unrestricted retained earnings of
PLDT to the capital stock account pursuant to the issuance of stock dividends is the
proper basis for the assessment of the SRF.

RULING:

NO.

In the case of stock dividends, it is the amount that the corporation transfers
from its surplus profit account to its capital account. It is the same amount that can
be loosely termed as the "trust fund" of the corporation. The "Trust Fund" doctrine
considers this subscribed capital as a trust fund for the payment of the debts of the
corporation, to which the creditors may look for satisfaction. Until the liquidation of
the corporation, no part of the subscribed capital may be returned or released to the
stockholder (except in the redemption of redeemable shares) without violating this
principle. Thus, dividends must never impair the subscribed capital; subscription
commitments cannot be condoned or remitted; nor can the corporation buy its own
shares using the subscribed capital as the considerations therefore.
When stock dividends are distributed, the amount declared ceases to belong to
the corporation but is distributed among the shareholders. Consequently, the
unrestricted retained earnings of the corporation are diminished by the amount of the
declared dividend while the stockholders equity is increased. Furthermore, the actual
payment is the cash value from the unrestricted retained earnings that each
shareholder foregoes for additional stocks/shares which he would otherwise receive as
required by the Corporation Code to be given to the stockholders subject to the
availability and conditioned on a certain level of retained earnings.
In essence, therefore, the stockholders by receiving stock dividends are forced
to exchange the monetary value of their dividend for capital stock, and the monetary
value they forego is considered the actual payment for the original issuance of the
stocks given as dividends. Therefore, stock dividends acquired by shareholders for the
monetary value they forego are under the coverage of the SRF and the basis for the
latter is such monetary value as declared by the board of directors.

272 | P a g e
Law 321_Corporation LAW_ Case Digest

NATIONAL TELECOMMUNICATIONS COMMISSION


vs.
HONORABLE COURT OF APPEALS and PHILIPPINE LONG DISTANCE TELEPHONE
COMPANY
G.R. No. 127937 July 28, 1999

FACTS:

Sometime in 1988, the NTC served on the PLDT the following assessment
notices and demands for payment: 1. the amount of P7,495,161.00 as supervision and
regulation fee under Section 40 (e) of the PSA for the said year, 1988, computed at
P0.50 per P100.00 of the Protestant's (PLDT) outstanding capital stock as at December
31, 1987 which then consisted of Serial Preferred Stock amounting to
P1,277,934,390.00 and Common Stock of P221,097,785 (Million) or a total of
P1,499,032,175.00; 2. the amount of P9.0 Million as permit fee under Section 40 (f) of
the PSA for the approval of the protestant's increase of its authorized capital stock
from P2.7 Billion to P4.5 Billion; and the amounts of P12,261,600.00 and
P33,472,030.00 as permit fees under Section 40(g) of the PSA in connection with the
Commission's decisions in NTC Cases Nos. 86-13 and 87-008 respectively, approving
the Protestant's equity participation in the Fiber Optic Interpacific Cable systems and
X-5 Service Improvement and Expansion Program.

ISSUE:

Whether or not the Court of Appeals erred in holding that the computation of
supervision and regulation fees under section 40 (f) of the public service act should be
based on the par value of the subscribed capital stock.

RULING:

NO.

The basis for computation of the fee to be charged by NTC on PLDT, is the
capital stock subscribed or paid and not, alternatively, the property and equipment.
The term "capital" and other terms used to describe the capital structure of a
corporation are of universal acceptance, and their usages have long been established
in jurisprudence. Briefly, capital refers to the value of the property or assets of a
corporation. The capital subscribed is the total amount of the capital that persons
(subscribers or shareholders) have agreed to take and pay for, which need not
necessarily be, and can be more than, the par value of the shares. In fine, it is the
amount that the corporation receives, inclusive of the premiums if any, in
consideration of the original issuance of the shares. In the case of stock dividends, it is
the amount that the corporation transfers from its surplus profit account to its capital
account. It is the same amount that can loosely be termed as the "trust fund" of the
corporation. The "Trust Fund" doctrine considers this subscribed capital as a trust
fund for the payment of the debts of the corporation, to which the creditors may look
for satisfaction. Until the liquidation of the corporation, no part of the subscribed
capital may be returned or released to the stockholder (except in the redemption of
redeemable shares) without violating this principle. Thus, dividends must never impair
the subscribed capital; subscription commitments cannot be condoned or remitted;
nor can the corporation buy its own shares using the subscribed capital as the
consideration therefor.

273 | P a g e
Law 321_Corporation LAW_ Case Digest

Voting Control Test v. Beneficial Control Test

Wilson P. Gamboa
vs.
Finance Secretary Margarito Teves, et al.,
G.R. No. 176579, June 28, 2011

FACTS:

This is a petition to nullify the sale of shares of stock of Philippine


Telecommunications Investment Corporation (PTIC) by the government of the Republic
of the Philippines, acting through the Inter-Agency Privatization Council (IPC), to
Metro Pacific Assets Holdings, Inc. (MPAH), an affiliate of First Pacific Company
Limited (First Pacific), a Hong Kong-based investment management and holding
company and a shareholder of the Philippine Long Distance Telephone Company
(PLDT). The petitioner questioned the sale on the ground that it also involved an
indirect sale of 12 million shares (or about 6.3 percent of the outstanding common
shares) of PLDT owned by PTIC to First Pacific. With the this sale, First Pacific‘s
common shareholdings in PLDT increased from 30.7 percent to 37 percent, thereby
increasing the total common shareholdings of foreigners in PLDT to about
81.47%. This, according to the petitioner, violates Section 11, Article XII of the 1987
Philippine Constitution which limits foreign ownership of the capital of a public utility
to not more than 40%.

ISSUE:

Whether or not the term ―capital‖ in Section 11, Article XII of the Constitution
refer to the total common shares only, or to the total outstanding capital stock
(combined total of common and non-voting preferred shares) of PLDT, a public utility.

RULING:

NO.

The Court partly granted the petition and held that the term ―capital‖ in Section
11, Article XII of the Constitution refers only to shares of stock entitled to vote in the
election of directors of a public utility, i.e., to the total common shares in PLDT.
It must be stressed, and respondents do not dispute, that foreigners hold a
majority of the common shares of PLDT. In fact, based on PLDT‘s 2010 General
Information Sheet (GIS), which is a document required to be submitted annually to the
Securities and Exchange Commission, foreigners hold 120,046,690 common shares of
PLDT whereas Filipinos hold only 66,750,622 common shares. In other words,
foreigners hold 64.27% of the total number of PLDT‘s common shares, while Filipinos
hold only 35.73%. Since holding a majority of the common shares equates to control,
it is clear that foreigners exercise control over PLDT. Such amount of control
unmistakably exceeds the allowable 40 percent limit on foreign ownership of public
utilities expressly mandated in Section 11, Article XII of the Constitution.
As shown in PLDT‘s 2010 GIS, as submitted to the SEC, the par value of PLDT
common shares is P5.00 per share, whereas the par value of preferred shares
is P10.00 per share. In other words, preferred shares have twice the par value of
common shares but cannot elect directors and have only 1/70 of the dividends of
common shares. Moreover, 99.44% of the preferred shares are owned by Filipinos
while foreigners own only a minuscule 0.56% of the preferred shares. Worse, preferred
shares constitute 77.85% of the authorized capital stock of PLDT while common
shares constitute only 22.15%. This undeniably shows that beneficial interest in PLDT
is not with the non-voting preferred shares but with the common shares, blatantly
violating the constitutional requirement of 60 percent Filipino control and Filipino
beneficial ownership in a public utility.

274 | P a g e
Law 321_Corporation LAW_ Case Digest

HEIRS OF WILSON P. GAMBOA


vs.
FINANCE SECRETARYMARGARITO B. TEVES, FINANCE UNDERSECRETARYJOHN
P. SEVILLA, AND COMMISSIONER RICARDO ABCEDE OF THE PRESIDENTIAL
COMMISSION ON GOOD GOVERNMENT(PCGG), et.al.
G.R. No. 176579 October 9, 2012

FACTS:

The Office of the Solicitor General (OSG) initially filed a motion for
reconsideration on behalfofthe SEC, assailing the 28 June 2011 Decision. However, it
subsequently filed a Consolidated Comment on behalf of the State, declaring expressly
that it agrees with the Court's definition of the term "capital" in Section 11, Article XII
of the Constitution. During the Oral Arguments on 26 June 2012, the OSG reiterated
its position consistent with the Court's 28 June 2011 Decision.

ISSUE:

Whether or not the term "capital" in Section 11, Article XII of the Constitution
has long been settled and defined to refer to the total outstanding shares of stock,
whether voting or non-voting.

RULING:

NO.

Since a specific class of shares may have rights and privileges or restrictions
different from the rest of the shares in a corporation, the 60-40 ownership
requirement in favor of Filipino citizens in Section 11, Article XII of the Constitution
must apply not only to shares with voting rights but also to shares without voting
rights. Preferred shares, denied the right to vote in the election of directors, are
anyway still entitled to vote on the eight specific corporate matters mentioned
above. Thus, if a corporation, engaged in a partially nationalized industry, issues a
mixture of common and preferred non-voting shares, at least 60 percent of the
common shares and at least 60 percent of the preferred non-voting shares must be
owned by Filipinos. Of course, if a corporation issues only a single class of shares, at
least 60 percent of such shares must necessarily be owned by Filipinos. In short, the
60-40 ownership requirement in favor of Filipino citizens must apply separately to
each class of shares, whether common, preferred non-voting, preferred voting or any
other class of shares. This uniform application of the 60-40 ownership requirement in
favor of Filipino citizens clearly breathes life to the constitutional command that the
ownership and operation of public utilities shall be reserved exclusively to
corporations at least 60 percent of whose capital is Filipino-owned. Applying uniformly
the 60-40 ownership requirement in favor of Filipino citizens to each class of shares,
regardless of differences in voting rights, privileges and restrictions, guarantees
effective Filipino control of public utilities, as mandated by the Constitution.

275 | P a g e
Law 321_Corporation LAW_ Case Digest

G.R. NOS. 174457-59


EXPRESS INVESTMENTS III PRIVATE LTD. AND EXPORT DEVELOPMENT
CANADA
vs.
DAYAN TELECOMMUNICATIONS, INC., THE BANK OF NEW YORK (AS TRUSTEE
FOR THE HOLDERS OF THE US$200,000,000 13.5% SENIOR NOTES OF DAYAN
TELECOMMUNICATIONS, INC.) AND ATTY. REMIGIO A. NOVAL (AS THE COURT-
APPOINTED REHABILITATION RECEIVER OF BAYANTEL)
x---------------x

G.R. Nos. 175418-20


IN THE MATTER OF:
THE CORPORATE REHABILITATION OF DAYAN TELECOMMUNICATIONS, INC.
PURSUANT TO THE INTERIM RULES OF PROCEDURE ON CORPORATE
REHABILITATION (A.M. NO. 00-8-10-SC)
THE BANK OF NEW YORK AS TRUSTEE FOR THE HOLDERS OF THE
US$200,000,000 13.5% SENIOR NOTES OF DAYAN TELECOMMUNICATIONS,
INC.
DUE 2006 ACTING ON THE INSTRUCTIONS OF THE INFORMAL STEERING
COMMITTEE: AVENUE ASIA INVESTMENTS, L.P., AVENUE ASIA
INTERNATIONAL, LTD., AVENUE ASIA SPECIAL SITUATIONS FUND II, L.P. AND
AVENUE ASIA CAPITAL PARTNERS, L.P.
vs.
DAYAN TELECOMMUNICATIONS, INC.

x---------------x
IN THE MATTER OF:
THE CORPORATE REHABILITATION OF BAY AN TELECOMMUNICATIONS, INC.
PURSUANT TO THE INTERIM RULES OF PROCEDURE ON CORPORATE
REHABILITATION (A.M. NO. 00-8-10-SC)
AVENUE ASIA INVESTMENTS, L.P., AVENUE ASIA INTERNATIONAL, LTD.,
AVENUE ASIA SPECIAL SITUATIONS FUND II, L.P., AVENUE ASIA CAPITAL
PARTNERS, L.P. AND AVENUE ASIA SPECIAL SITUATIONS FUND III, L.P.
vs.
DAYAN TELECOMMUNICATIONS, INC.

x---------------x
G.R. No. 177270 December 5, 2012
THE BANK OF NEW YORK AS TRUSTEE FOR THE HOLDERS OF THE
US$200,000,000 13.5% SENIOR NOTES OF BAY AN TELECOMMUNICATIONS,
INC.
vs.
BAY AN TELECOMMUNICATIONS, INC.

FACTS:

Respondent Bayantel is a duly organized domestic corporation engaged in the


business of providing telecommunication services. It is 98.6% owned by Bayan
Telecommunications Holdings Corporation (BTHC), which in turn is 85.4% owned by
the Lopez Group of Companies and Benpres Holdings Corporation.
On various dates between the years 1995 and 2001, Bayantel entered into
several credit agreements. In July 1999, Bayantel issued US$200 million worth of
13.5% Senior Notes pursuant to an Indenture dated July 22, 1999 that it entered into
with The Bank of New York as trustee for the holders of said notes.

ISSUES:

Whether or not the claims of secured and unsecured creditors should be


treated pari passu during rehabilitation.
Whether or not the debt-equity complies with the citizenship requirement under
the Constitution.

276 | P a g e
Law 321_Corporation LAW_ Case Digest

RULING:

YES.

As between the creditors, the key phrase is "equality is equity." When a


corporation threatened by bankruptcy is taken over by a receiver, all the creditors
should stand on equal footing. Not anyone of them should be given any preference by
paying one or some of them ahead of the others. This is precisely the reason for the
suspension of all pending claims against the corporation under receivership. Instead
of creditors vexing the courts with suits against the distressed firm, they are directed
to file their claims with the receiver who is a duly appointed officer of the SEC.
Since then, the principle of equality in equity has been cited as the basis for
placing secured and unsecured creditors in equal footing or in pari passu with each
other during rehabilitation. In legal parlance, pari passu is used especially of creditors
who, in marshaling assets, are entitled to receive out of the same fund without any
precedence over each other.

YES.

Applying this, two steps must be followed in order to determine whether the
conversion of debt to equity in excess of 40% of the outstanding capital stock violates
the constitutional limit on foreign ownership of a public utility: First, identify into
which class of shares the debt shall be converted, whether common shares, preferred
shares that have the right to vote in the election of directors or non-voting preferred
shares; Second, determine the number of shares with voting right held by foreign
entities prior to conversion. If upon conversion, the total number of shares held by
foreign entities exceeds 40% of the capital stock with voting rights, the constitutional
limit on foreign ownership is violated. Otherwise, the conversion shall be respected.
In its Rehabilitation Plan, among the material financial commitments made by
respondent Bayantel is that its shareholders shall "relinquish the agreed-upon
amount of common stock[s] as payment to Unsecured Creditors as per the Term
Sheet." Evidently, the parties intend to convert the unsustainable portion of
respondent's debt into common stocks, which have voting rights. If we indulge
petitioners on their proposal, the Omnibus Creditors which are foreign corporations,
shall have control over 77.7% of Bayantel, a public utility company. This is precisely
the scenario proscribed by the Filipinization provision of the Constitution. Therefore,
the Court of Appeals acted correctly in sustaining the 40% debt-to-equity ceiling on
conversion.

277 | P a g e
Law 321_Corporation LAW_ Case Digest

REDMONT CONSOLIDATED MINING CORPORATION


vs.
MCARTHUR MINING, INC. ET.AL.
SEC En Banc Case No. 09-09-177, March 25. 2010

Significantly, the SEC en banc, which is the collegial body statutorily


empowered to issue rules and opinions on behalf of the SEC, has adopted the 60-40
ownership requirement in favor of Filipino citizens mandated by the Constitution for
certain economic activities.
The avowed purpose of the Constitution is to place in the hands of Filipinos the
exploitation of our natural resources. Necessarily, therefore, the Rule interpreting the
constitutional provision should not diminish that right through the legal fiction of
corporate ownership and control. But the constitutional provision, as interpreted and
practiced via the 1967 SEC Rules, has favored foreigners contrary to the command of
the Constitution. Hence, the Grandfather Rule must be applied to accurately determine
the actual participation, both direct and indirect, of foreigners in a corporation engaged
in a nationalized activity or business.

278 | P a g e
Law 321_Corporation LAW_ Case Digest

DEMOSTHENES P. AGAN, et al., petitioners,,


vs.
PHILIPPINE INTERNATIONAL AIR TERMINALS CO., INC., MANILA
INTERNATIONAL AIRPORT AUTHORITY, et al., Respondents.
G.R. No. 155001 January 21, 2004

FACTS:

Sometime in 1993, six business leaders, explored the possibility of investing in


the new NAIA airport terminal, so they formed Asians Emerging Dragon Corp. They
submitted proposals to the government for the development of NAIA Intl. Passenger
Terminal III (NAIA IPT III). The NEDA approved the NAIA IPT III project. Bidders were
invited, and among the proposal Peoples Air Cargo (Paircargo) was chosen. AEDC
protested alleging that preference was given to Paircargo, but still the project was
awarded to Paircargo. Because of that, it incorporated into, Phil. Intl. Airport
Terminals Co. (PIATCO). The DOTC and PIATCO entered into a concession agreement
in 1997 to franchise and operate the said terminal for 21years. In Nov. 1998 it was
amended in the matters of pertaining to the definition of the obligations given to the
concessionaire, development of facilities and proceeds, fees and charges, and the
termination of contract. Since MIAA is charged with the maintenance and operations
of NAIA terminals I and II, it has a contract with several service providers. The workers
filed the petition for prohibition claiming that they would lose their job, and the service
providers joined them, filed a motion for intervention. Likewise several employees of
the MIAA filed a petition assailing the legality of arrangements. A group of
congressmen filed similar petitions. Pres. Arroyo declared in her speech that she will
not honor PIATCO contracts which the Exec. Branch's legal office concluded null and
void.

ISSUE:
Whether or Not the 1997 concession agreement is void, together with its
amendments for being contrary to the constitution.

RULING:

YES.

The 1997 concession agreement is void for being contrary to public policy. The
amendments have the effect of changing it into and entirely different agreement from
the contract bidded upon. The amendments present new terms and conditions which
provide financial benefit to PIATCO which may have the altered the technical and
financial parameters of other bidders had they know that such terms were available.
The 1997 concession agreement, the amendments and supplements thereto are set
aside for being null and void.
The petitioners have local standi. They are prejudiced by the concession
agreement as their livelihood is to be taken away from them.

279 | P a g e
Law 321_Corporation LAW_ Case Digest

Voting v. Non-Voting

CECILIA CASTILLO, et al., and MEDICAL CENTER PARAÑAQUE, INC.


vs.
ANGELES BALINGHASAY, et al.
G.R. No. 150976 October 18, 2004

FACTS:

Petitioners are stockholders of MCPI holding Class ―B‖ shares while the
respondents are also stockholders owning Class ―A‖ shares. In a 1992 amendment of
the Articles of Incorporation of MCPI, the Articles of incorporation of the MCPI provides
that, ―except when otherwise provided by law, only holders of Class ―A‖ shares are
entitled to vote and to have the right to be elected as directors and corporate officers.
During the 2001meeting, petitioners raised an objection to the fact that only Class ―A‖
shares are allowed to vote and to be elected. They contended that the Class ―B‖ share
holders right to vote is violated in violation of law.

ISSUE:

Whether or not holders of Class ―B‖ shares of MCPI may be deprives of the right
to vote and be voted for as directors.

RULING:

NO.

The 1992 amendment contains a proviso ―except as otherwise provided for by


law‖ the law being referred to by the proviso is that which is in force at the time of the
amendment, in this case, was the Corporation Code.
Under Sec. 6 of the Corporation Code, it provides that no share may be deprived
of voting rights except those classified and issued as ―preffered‖ or ―redeemable‖
shares unless otherwise provided in this code.there is nothing in the articles of
incorporation or an iota of evidence on record that shows that Class ―B‖ shares were
categorized as either preffered or redeemable shares.

280 | P a g e
Law 321_Corporation LAW_ Case Digest

JULIO E. T. SALES and GEORGE V. AGONIAS and SMEC, petitioners,


vs.
SEC, SIHI and ATCO, represented by its President, ANSELMO TRINIDAD; VIMC,
represented by its President, et al., respondents;
G.R. No. 54330 January 13, 1989

FACTS:

SMEC sold 200M common shares of its capital stock in the amount of P2.6M to
SIHI under a Sales Agreement providing that the sale shall be only up to 5m shares
per buyer. SIHI requested for the transfer of the 200M shares to ATCO to which SMEC
complied. During the time that ATCO held the shares, it voted them in the SHs'
meetings of SMEC. ATCO in turn sold 198,500,000 of the shares to respondent VIMC.
Upon request, SMEC BOD issued a resolution directing its President to sign the
certificate of stock that would effect the transfer. Before the 1979 annual SH meeting
of SMEC, petitioners sought to nullify the sales of the shares to VIMC with the SEC
and to enjoin VIMC from voting the said shares. VIMC was temporarily restrained and
the meeting was held without the participation of VIMC‘s shares and BODs were
elected only from the group of petitioners. In VIMC‘s answer, it questioned the said
election. SEC denied the petition as well as motion to dismiss and lifted the
Restraining it issued earlier and allowed the shares of VIMC to be counted in
determining the quorum of the 1980 annual SHs meeting, which was already near,
and the same shares were allowed to vote and be voted for. Before the SC, petitioners
contended that the SEC gravely abused its discretion in not enjoining the participation
of VIMC in the 1980 election considering that the sale of the shares to VIMC was null
and void as it was done in violation of the Sales Agreement on the limit of shares to be
sold to each buyer and that VIMC‘s ownership of the shares is contrary to Sec. 13 (5-
A) of the old corporation law.

ISSUE:

Whether or not SEC acted with grave abuse of discretion in not permanently
enjoining VIMC in voting.

RULING:

YES.

SC found no grave abuse of discretion on the part of the SEC in not restraining
VIMC. It adopted the SEC resolution stating that the sale of the shares of stock had
long been perfected and is presumed valid until declared otherwise. As against this
presumption, petitioners' prayer for injunction cannot prevail as the issue of the
validity of the sale is still to be resolved by the SEC.
Considering that the shares constitute the majority, it is more equitable that
the same be allowed to vote rather than be enjoined. As it has been ruled the removal
of a majority SH from the management of the corporation and/or the dissolution of a
corporation in a suit filed by a minority SH is a drastic measure. It should be resorted
to only when the necessity is clear. With more reason, the Court will not deprive a SH
of his right to vote his shares in the annual SHs' meeting, except upon a clear showing
of its lawful denial under the articles of incorporation or by-laws of the corporation, as
it is a right inherent in stock ownership.

281 | P a g e
Law 321_Corporation LAW_ Case Digest

Redeemable Preferred

REPUBLIC PLANTERS BANK


vs.
HON. ENRIQUE A. AGANA, SR., as Presiding Judge, Court of First Instance of
Rizal, Branch XXVIII, Pasay City, ROBES-FRANCISCO REALTY & DEVELOPMENT
CORPORATION and ADALIA F. ROBES
G.R. No. 51765. March 3, 1997

FACTS:

On September 18, 1961, private respondent Corporation secured a loan from


petitioner in the amount of P120,000.00. Instead of giving the legal tender totaling to
the full amount of the loan, which is P120,000.00, petitioner lent such amount
partially in the form of money and partially in the form of stock certificates numbered
3204 and 3205, each for 400 shares with a par value of P10.00 per share, or for
P4,000.00 each, for a total of P8,000.00. Said stock certificates were in the name of
private respondent Adalia F. Robes and Carlos F. Robes, who subsequently, however,
endorsed his shares in favor of Adalia F. Robes.
On January 31, 1979, private respondents proceeded against petitioner and
filed a Complaint anchored on private respondents' alleged rights to collect dividends
under the preferred shares in question and to have petitioner redeem the same under
the terms and conditions of the stock certificates.
The trial court rendered the herein assailed decision in favor of private
respondents ordering petitioner to pay private respondents the face value of the stock
certificates as redemption price, plus 1% quarterly interest thereon until full payment.

ISSUES:

Whether or not the respondent court was correct in ordering petitioner to pay
private respondents the face value of the stock certificates as redemption price.

RULING:

NO.

A preferred share of stock is one which entitles the holder thereof to certain
preferences over the holders of common stock. The preferences are designed to induce
persons to subscribe for shares of a corporation.
Preferred shares take a multiplicity of forms. The most common forms may be
classified into two: (1) preferred shares as to assets; and (2) preferred shares as to
dividends. The former is a share which gives the holder thereof preference in the
distribution of the assets of the corporation in case of liquidation;the latter is a share
the holder of which is entitled to receive dividends on said share to the extent agreed
upon before any dividends at all are paid to the holders of common stock.There is no
guaranty, however, that the share will receive any dividends.
The redemption of said shares cannot be allowed. As pointed out by the
petitioner, the Central Bank made a finding that said petitioner has been suffering
from chronic reserve deficiency, and that such finding resulted in a directive, issued
on January 31, 1973 by then Gov. G.S. Licaros of the Central Bank, to the President
and Acting Chairman of the Board of the petitioner bank prohibiting the latter from
redeeming any preferred share, on the ground that said redemption would reduce the
assets of the Bank to the prejudice of its depositors and creditors.Redemption of
preferred shares was prohibited for a just and valid reason. The directive issued by the
Central Bank Governor was obviously meant to preserve the status quo, and to
prevent the financial ruin of a banking institution that would have resulted in adverse
repercussions, not only to its depositors and creditors, but also to the banking
industry as a whole.

282 | P a g e
Law 321_Corporation LAW_ Case Digest

Treasury

COMMISSIONER OF INTERNAL REVENUE


vs.
MANNING, MCDONALD, SIMMONS
No. L-28398 6 August 1975

FACTS:

In 1952 the MANTRASCO had an authorized capital stock of P2,500,000


divided into 25,000 common shares; 24,700 of these were owned by Julius S. Reese,
and the rest, at 100 shares each, by the three respondents.
On October 19, 1954 Reese died. In 1955, after MANTRASCO made a partial
payment of Reese's shares, the certificate for the 24,700 shares in Reese's name was
cancelled and a new certificate was issued in the name of MANTRASCO, which was
endorsed to the law firm of Ross, Selph, Carrascoso and Janda, as trustees for and in
behalf of MANTRASCO. In 1958, at a special meeting of MANTRASCO stockholders,
the following resolution was passed:"RESOLVED, that the 24,700 shares in the
Treasury be reverted back to the capital account of the company as a stock dividend to
be distributed to shareholders of record."
In 1963 the entire purchase price of Reese's interest in MANTRASCO was
finally paid in full by the latter, In 1964 the trust agreement was terminated and the
trustees delivered to MANTRASCO all the shares which they were holding in trust. On
the basis of their examination, the BIR examiners concluded that the distribution of
Reese's shares as stock dividends was in effect a distribution of the "asset or property
of the corporation as may be gleaned from the payment of cash for the redemption of
said stock and distributing the same as stock dividend." On April 14, 1965 the CIR
issued notices of assessment for deficiency income taxes to the respondents for the
year 1958.

ISSUE:

Whether or not the issuance of the notices of assessment for deficiency income
taxes to the respondents for the year 1958 was proper.

RULING:

YES.

The declaration by the respondents and Reese's trustees of MANTRASCO's


alleged treasury stock dividends in favor of the former, brings, however, into clear
focus the ultimate purpose which the parties to the trust instrument aimed to realize:
to make the respondents the sole owners of Reese's interest in MANTRASCO by
utilizing the periodic earnings of that company and its subsidiaries to directly
subsidize their purchase of the said interests, and by making it appear outwardly,
through the formal declaration of non-existent stock dividends in the treasury, that
they have not received any income from those firms when, in fact, by that declaration
they secured to themselves the means to turn around as full owners of Reese's shares.
In other words, the respondents, using the trust instrument as a convenient technical
device, bestowed unto themselves the full worth and value of Reese's corporate
holdings with the use of the very earnings of the companies. Such package device,
obviously not designed to carry out the usual stock dividend purpose of corporate
expansion reinvestment, e.g. the acquisition of additional facilities and other capital
budget items, but exclusively for expanding the capital base of the respondents in
MANTRASCO, cannot be allowed to deflect the respondents' responsibilities toward
our income tax laws. The conclusion is thus ineluctable that whenever the companies
involved herein parted with a portion of their earnings "to buy" the corporate holdings
of Reese, they were in ultimate effect and result making a distribution of such
earnings to the respondents. All these amounts are consequently subject to income
tax as being, in truth and in fact, a flow of cash benefits to the respondents.

283 | P a g e
Law 321_Corporation LAW_ Case Digest

SAN MIGUEL CORPORATION


vs.
SANDIGANBAYAN (First Division), EDUARDO M. COJUANGCO, JR., et al.,
G.R. No. 118661 SEPTEMBER 14, 2000

FACTS:

CIIF sold 33,133,266 shares of the outstanding capital stock of SMC to Andres
Soriano III of the SMC Group payable in 4 installments. April 1, 1986, Soriano paid
the initial 500M to the UCPB as administrator of CIIF. The sale was transacted
through the stock exchange and the shares were registered in the name of AHSI. On
April 7, 1986, PCGG sequestered the shares subject of the sale, thus SMC suspended
payment of the balance. UCPB, filed a complaint for rescission and damages with the
RTC Makati. SMC assailed the RTC jurisdiction on the ground that primary
jurisdiction was vested with the PCGG since the SMC shares were sequestered shares.
SC dismissed the complaint for rescission without prejudice to the ventilation of the
parties‘ claims before the Sandiganbayan (SBN).
The Republic, thru the OSG opposed the Agreement contending that the
involved coco-levy funds, whether in the form of earnings or dividends therefrom, or in
the form of the value of liquidated corporate assets represented by all sequestered
share (like the value of assets sold/mortgaged to finance the 500M 1st installment), or
in the form of cash, or, as in the case of subject Settlement, in the form of proceeds of
sale or of payments of certain alleged obligations are public funds and are beyond or
outside the commerce and not within the private disposition of private individuals.

ISSUE:

Whether or not there is a valid delivery of certificates of stock of smc shares and
the dividends thereon to the PCGG.

RULING:

YES.

No grave abuse of discretion on the part of SBN when it ordered petitioners to


deliver the treasury shares to the PCGG and pay their corresponding dividends for the
following reasons: Under the Corporation Code, Treasury shares are shares of stocks
which have been issued and fully paid for, but subsequently reacquired by, the
issuing corporation by purchase, redemption, donation or through some lawful means.
These 26.45M shares or any portion thereof can, therefore, become treasury shares,
i.e., property of SMC, only if the sale between the UCPB Group and the SMC Group is
allowed; otherwise these shares cannot even begin to be deemed to have been re-
acquired by the issuing corporation, the SMC.

284 | P a g e
Law 321_Corporation LAW_ Case Digest

Trust Fund Doctrine

NATIONAL TELECOMMUNICATIONS COMMISSION


vs.
COURT OF APPEALS AND PHILIPPINE LONG DISTANCE TELEPHONE COMPANY
G.R. NO. 127937 1999 JULY 28

FACTS:

Sometime in 1988, the National Telecommunications Commission (NTC) served


on the Philippine Long Distance Telephone Company (PLDT) assessment notices and
demands for payment. PLDT challenged the aforesaid assessments. On September 29,
1993, the NTC rendered a Decision, denying the protest of PLDT. On October 22,
1993, PLDT interposed a Motion for Reconsideration, which was denied by NTC in an
Order issued on May 3, 1994. On May 12, 1994, PLDT appealed the aforesaid Decision
to the Court of Appeals, which came out with its questioned Decision of October 30,
1996, modifying the disposition of NTC. On November 20, 1996, NTC moved for partial
reconsideration of the Decision, with respect to the basis of the assessment under
Section 40(e), i.e., par value of the subscribed capital stock. It also sought a partial
reconsideration of the fee of fifty (P0.50) centavos for the issuance or increasing of the
capital stock under Section 40 (f). With the denial of its motions for reconsideration by
the Resolution of the Court of Appeals dated January 27, 1997, petitioner found its
way to the Court via the Petition for Certiorari.

ISSUE:

Whether or not the Court of Appeals erred in holding that the computation of
Supervision and regulation Fees under Section 40(f) of the Public Service Act should
be based on the par value of the subscribed capital stock.

RULING:

NO.

Succinct and clear is the ruling of this Court in the case of Philippine Long
Distance Telephone Company vs. Public Service Commission, 66 SCRA 341, that the
basis for computation of the fee to be charged by NTC on PLDT, is ―the capital stock
subscribed or paid and not, alternatively, the property and equipment.‖ The fee in
question is based on the capital stock subscribed or paid, nothing less nothing more.
The ―Trust Fund‖ doctrine considers this subscribed capital as a trust fund for the
payment of the debts of the corporation, to which the creditors may look for
satisfaction. Until the liquidation of the corporation, no part of the subscribed capital
may be returned or released to the stockholder (except in the redemption of
redeemable shares) without violating this principle. Thus, dividends must never
impair the subscribed capital; subscription commitments cannot be condoned or
remitted; nor can the corporation buy its own shares using the subscribed capital as
the consideration therefore.
In the same way that the Court in PLDT vs. PSC has rejected the ―value of the
property and equipment‖ as being the proper basis for the fee imposed by Section 40(e)
of the Public Service Act, as amended by Republic Act No. 3792, so also must the
Court disallow the idea of computing the fee on ―the par value of [PLDT‘s] capital stock
subscribed or paid excluding stock dividends, premiums, or capital in excess of par.‖
Neither is the assessment made by the National Telecommunications Commission on
the basis of the market value of the subscribed or paid-in capital stock acceptable
since it is itself a deviation from the explicit language of the law.

285 | P a g e
Law 321_Corporation LAW_ Case Digest

ONG YONG, et al., petitioner


vs.
TIU, et al., respondent
G.R. No. 144476 8 April 2003

FACTS:

In 1994, the construction of the Masagana Citimall in Pasay City was


threatened with stoppage and incompletion when its owner, the First Landlink Asia
Development Corporation (FLADC), which was owned by David S. Tiu, Cely Y. Tiu,
Moly Yu Gow, Belen See Yu, D. Terence Y. Tiu, John Yu and Lourdes C. Tiu (the Tius),
encountered dire financial difficulties. It was heavily indebted to the Philippine
National Bank (PNB) for P190 million. To stave off foreclosure of the mortgage on the
two lots where the mall was being built, the Tius invited Ong Yong, Juanita Tan Ong,
Wilson T. Ong, Anna L. Ong, William T. Ong and Julia Ong Alonzo (the Ongs), to invest
in FLADC. Under the Pre-Subscription Agreement they entered into, the Ongs and the
Tius agreed to maintain equal shareholdings in FLADC: the Ongs were to subscribe to
1,000,000 shares at a par value of P100.00 each while the Tius were to subscribe to
an additional 549,800 shares at P100.00 each in addition to their already existing
subscription of 450,200 shares. Furthermore, they agreed that the Tius were entitled
to nominate the Vice-President and the Treasurer plus 5 directors while the Ongs were
entitled to nominate the President, the Secretary and 6 directors (including the
chairman) to the board of directors of FLADC. Moreover, the Ongs were given the right
to manage and operate the mall. Accordingly, the Ongs paid P100 million in cash for
their subscription to 1,000,000 shares of stock while the Tius committed to contribute
to FLADC a four-storey building and two parcels of land respectively valued at P20
million (for 200,000 shares), P30 million (for 300,000 shares) and P49.8 million (for
49,800 shares) to cover their additional 549,800 stock subscription therein. The Ongs
paid in another P70 million 3 to FLADC and P20 million to the Tius over and above
their P100 million investment, the total sum of which (P190 million) was used to settle
the P190 million mortgage indebtedness of FLADC to PNB. The business harmony
between the Ongs and the Tius in FLADC, however, was shortlived because the Tius,
on 23 February 1996, rescinded the Pre-Subscription Agreement. The Tius accused
the Ongs of (1) refusing to credit to them the FLADC shares covering their real
property contributions; (2) preventing David S. Tiu and Cely Y. Tiu from assuming the
positions of and performing their duties as Vice-President and Treasurer, respectively,
and (3) refusing to give them the office spaces agreed upon. The controversy finally
came to a head when the case was commenced by the Tius on 27 February 1996 at
the Securities and Exchange Commission (SEC), seeking confirmation of their
rescission of the Pre-Subscription Agreement.

ISSUE:

Whether or not the rescission of Pre-Subscription Agreement would result in


unauthorized liquidation.

RULING:

YES.

The rescission of the Pre-Subscription Agreement will effectively result in the


unauthorized distribution of the capital assets and property of the corporation,
thereby violating the Trust Fund Doctrine and the Corporation Code, since rescission
of a subscription agreement is not one of the instances when distribution of capital
assets and property of the corporation is allowed. Rescission will, in the final analysis,
result in the premature liquidation of the corporation without the benefit of prior
dissolution in accordance with Sections 117, 118, 119 and 120 of the Corporation
Code.

286 | P a g e
Law 321_Corporation LAW_ Case Digest

What is a “Subscription”

ONG YONG, et al., petitioner


vs.
TIU, et al., respondent
G.R. No. 144476 8 April 2003

FACTS:

In 1994, the construction of the Masagana Citimall in Pasay City was


threatened with stoppage and incompletion when its owner, the First Landlink Asia
Development Corporation (FLADC), which was owned by David S. Tiu, Cely Y. Tiu,
Moly Yu Gow, Belen See Yu, D. Terence Y. Tiu, John Yu and Lourdes C. Tiu (the Tius),
encountered dire financial difficulties. It was heavily indebted to the Philippine
National Bank (PNB) for P190 million. To stave off foreclosure of the mortgage on the
two lots where the mall was being built, the Tius invited Ong Yong, Juanita Tan Ong,
Wilson T. Ong, Anna L. Ong, William T. Ong and Julia Ong Alonzo (the Ongs), to invest
in FLADC. Under the Pre-Subscription Agreement they entered into, the Ongs and the
Tius agreed to maintain equal shareholdings in FLADC: the Ongs were to subscribe to
1,000,000 shares at a par value of P100.00 each while the Tius were to subscribe to
an additional 549,800 shares at P100.00 each in addition to their already existing
subscription of 450,200 shares. Furthermore, they agreed that the Tius were entitled
to nominate the Vice-President and the Treasurer plus 5 directors while the Ongs were
entitled to nominate the President, the Secretary and 6 directors (including the
chairman) to the board of directors of FLADC. Moreover, the Ongs were given the right
to manage and operate the mall. Accordingly, the Ongs paid P100 million in cash for
their subscription to 1,000,000 shares of stock while the Tius committed to contribute
to FLADC a four-storey building and two parcels of land respectively valued at P20
million (for 200,000 shares), P30 million (for 300,000 shares) and P49.8 million (for
49,800 shares) to cover their additional 549,800 stock subscription therein. The Ongs
paid in another P70 million 3 to FLADC and P20 million to the Tius over and above
their P100 million investment, the total sum of which (P190 million) was used to settle
the P190 million mortgage indebtedness of FLADC to PNB.

ISSUE:

Whether or not the pre-Subscription Agreement executed by the Ongs is


actually a subscription contract.

RULING:

YES.

FLADC was originally incorporated with an authorized capital stock of 500,000


shares with the Tius owning 450,200 shares representing the paid-up capital. When
the Tius invited the Ongs to invest in FLADC as stockholders, an increase of the
authorized capital stock became necessary to give each group equal (50-50)
shareholdings as agreed upon in the Pre-Subscription Agreement. The authorized
capital stock was thus increased from 500,000 shares to 2,000,000 shares with a par
value of P100 each, with the Ongs subscribing to 1,000,000 shares and the Tius to
549,800 more shares in addition to their 450,200 shares to complete 1,000,000
shares. Thus, the subject matter of the contract was the 1,000,000 unissued shares of
FLADC stock allocated to the Ongs. Since these were unissued shares, the parties'
Pre-Subscription Agreement was in fact a subscription contract as defined under
Section 60, Title VII of the Corporation Code. A subscription contract necessarily
involves the corporation as one of the contracting parties since the subject matter of
the transaction is property owned by the corporation — its shares of stock. Thus, the
subscription contract (denominated by the parties as a Pre-Subscription Agreement)
whereby the Ongs invested P100 million for 1,000,000 shares of stock was, from the
viewpoint of the law, one between the Ongs and FLADC, not between the Ongs and the
Tius.

287 | P a g e
Law 321_Corporation LAW_ Case Digest

BAYLA, et al., petitioner


vs.
SILANG TRAFFIC CO., INC., respondent
G.R. Nos. L-48195 and 48196 May 1, 1942

FACTS:

Petitioners in G.R. No. 48195 instituted this action in the Court of First
Instance of Cavite against the respondent Silang Traffic Co., Inc. (cross-petitioner in
G.R. No. 48196), to recover certain sums of money which they had paid severally to
the corporation on account of shares of stock they individually agreed to take and pay
for under certain specified terms and conditions. The agreements signed by the other
petitioners were of the same date (March 30, 1935) and in identical terms as the
foregoing except as to the number of shares and the corresponding purchase price.
The petitioners agreed to purchase a total of 46 shares and, up to April 30, 1937, had
paid the corresponding amount on account thereof.Petitioners' action for the recovery
of the sums above mentioned is based on a resolution by the board of directors of the
respondent corporation on August 1, 1937.
The respondent corporation set up the following defenses: (1) That the above-
quoted resolution is not applicable to the petitioners Sofronio T. Bayla, Josefa Naval,
and Paz Toledo because on the date thereof "their subscribed shares of stock had
already automatically reverted to the defendant, and the installments paid by them
had already been forfeited"; and (2) that said resolution of August 1, 1937, was
revoked and cancelled by a subsequent resolution of the board of directors of the
defendant corporation dated August 22, 1937.

ISSUE:

Whether or not the agreement was a contract of subscription to the capital


stock of the respondent corporation.

RULING:

NO.

Whether a particular contract is a subscription or a sale of stock is a matter of


construction and depends upon its terms and the intention of the parties. In the
Unson case just cited, this Court held that a subscription to stock in an existing
corporation is, as between the subscriber and the corporation, simply a contract of
purchase and sale.
It seems clear from the terms of the contracts in question that they are
contracts of sale and not of subscription. The lower courts erred in overlooking the
distinction between subscription and purchase "A subscription, properly speaking, is
the mutual agreement of the subscribers to take and pay for the stock of a
corporation, while a purchase is an independent agreement between the individual
and the corporation to buy shares of stock from it at stipulated price." In some
particulars the rules governing subscriptions and sales of shares are different. For
instance, the provisions of our Corporation Law regarding calls for unpaid
subscription and assessment of stock do not apply to a purchase of stock. Likewise
the rule that corporation has no legal capacity to release an original subscriber to its
capital stock from the obligation to pay for his shares, is inapplicable to a contract of
purchase of shares.

288 | P a g e
Law 321_Corporation LAW_ Case Digest

SALMON, DEXTER & Co., plaintiff


vs.
TIMOTEO UNSON, defendant
G.R. No. L-23608 March 17, 1925

FACTS:

The plaintiff seeks to recover of the defendant the sum of P1,000 with legal
interest on a subscription for capital stock contract. The defense is that the defendant
is released from his obligation on the subscription agreement by virtue of the increase
of the capital stock of the plaintiff from P250,000, the amount mentioned in the
agreement, to P500,000, the amount agreed upon the stockholders prior to the
defendant's signing the agreement. On this issue, judgment in the lower court was
with the plaintiff.
The plaintiff is Salmon, Dexter and Company, a domestic corporation. It was
organized under the name of C.S. Salmon and Company on May 28, 1918, with a
capital stock of P250,000. Thereafter, pursuant to a resolution of the board of
directors of the corporation of June 24, 1920, a meeting of the stockholders was had
on July 14, 1920, at which the capital stock of C.S. Salmon and Company was
increased to P500,000. The certificate of increase of capital stock from P250,000 to
P500,000, and articles of incorporation, as amended, of Salmon, Dexter and Company
were filed with the Mercantile Registry of the Bureau of Commerce and Industry on
September 16, 1920.
On July 28, 1920, Timoteo Unson, the defendant, to follow the allegation in the
third paragraph of the complaint, "became a subscriber of C.S. Salmon and Company,
by signing an agreement in writing and delivering the same to C.S. Salmon and
Company, the name of which company was later changed to Salmon, Dexter and
Company."

ISSUE:

Whether or not the contract entered into by the parties is a Subscription


contract.

RULING:

YES.

After incorporation, one may become a shareholder by subscription, or by


purchasing stock directly from the corporation, or from individual owners thereof. A
distinction is drawn by the authorities between a subscription to the capital stock of
the corporation after its organization and a sale of shares by it. Whether a particular
contract is a subscription or a sale of stock is a matter of construction, and depends
upon its terms and the intention of the parties. It has been held that a subscription to
stock in an existing corporation is, as between the subscriber and the corporation,
simply a contract of purchase and sale.
Admitting that the terminology of the agreement is not conclusive, and
admitting that it is a contract between a subscriber and the corporation, and thus
simply a contract of purchase and sale, then under the last hypothesis we have to
determine if the contract is avoided by misrepresentation.
In our opinion, a contract different from that which was entered into cannot be
made for the parties and imposed upon Unson. Unson has the right to stand upon the
contract he has made. In our opinion also, there was such a non-disclosure of a
material fact as was equivalent to false representation. This representation was of a
character that the party to whom it was made had a right to rely upon it.

289 | P a g e
Law 321_Corporation LAW_ Case Digest

SUNSET VIEW CONDOMINIUM CORPORATION, petitioner


vs.
CAMPOS, respondent
G.R. No. L-52361 April 27, 1981

FACTS:

The private respondent, Aguilar-Bernares Realty, a sole proprietorship with


business name registered with the Bureau of Commerce, owned and operated by the
spouses Emmanuel G. Aguilar and Zenaida B. Aguilar, is the assignee of a unit,
"Solana", in the Sunset View Condominium Project with La Perla Commercial,
Incorporated, as assignor. 3 The La Perla Commercial, Incorporated bought the
"Solana" unit on installment from the Tower Builders, Inc. 4 The petitioner, Sunset
View Condominium Corporation, filed for the collection of assessments levied on the
unit against Aguilar-Bernares Realty, private respondent herein, a complaint dated
June 22, 1979 docketed as Civil Case No. 7303-P of the Court of First Instance of
Pasay City, Branch XXX. The private respondent filed a Motion to Dismiss the
complaint on the grounds (1) that the complaint does not state a cause of action: (2)
that the court has no jurisdiction over the subject or nature other action; and (3) that
there is another action pending between the same parties for the same cause. The
petitioner filed its opposition thereto. The motion to dismiss was granted on December
11, 1979 by the respondent Judge who opined that the private respondent is,
pursuant to Section 2 of Republic Act No. 4726, a "holder of a separate interest" and
consequently, a shareholder of the plaintiff condominium corporation; and that "the
case should be properly filed with the Securities & Exchange Commission which has
exclusive original jurisdiction on controversies arising between shareholders of the
corporation." the motion for reconsideration thereof having been denied, the petitioner,
alleging grave abuse of discretion on the part of respondent Judge, filed the instant
petition for certiorari praying that the said orders be set aside.

ISSUE:

Whether or not a purchaser of a condominium unit in the condominium project


managed by the petitioner, who has not yet fully paid the purchase price thereof,
automatically a stockholder of the petitioner Condominium Corporation.

RULING:

NO.

The share of stock appurtenant to the unit win be transferred accordingly to the
purchaser of the unit only upon full payment of the purchase price at which time he
will also become the owner of the unit. Consequently, even under the contract, it is
only the owner of a unit who is a shareholder of the Condominium Corporation.
Inasmuch as owners is conveyed only upon full payment of the purchase price, it
necessarily follows that a purchaser of a unit who has not paid the full purchase price
thereof is not The owner of the unit and consequently is not a shareholder of the
Condominium Corporation.
That only the owner of a unit is a stockholder of the Condominium Corporation
is inferred from Section 10 of the Condominium Act. Pursuant to such statutory
provision, ownership of a unit is a condition sine qua non to being a shareholder in the
condominium corporation. It follows that a purchaser of a unit who is not yet the
owner thereof for not having fully paid the full purchase price, is not a shareholder By
necessary implication, the "separate interest" in a condominium, which entitles the
holder to become automatically a share holder in the condominium corporation, as
provided in Section 2 of the Condominium Act, can be no other than ownership of a
unit. This is so because nobody can be a shareholder unless he is the owner of a unit
and when he ceases to be the owner, he also ceases automatically to be a shareholder.
The private respondents, therefore, who have not fully paid the purchase price of their
units and are consequently not owners of their units are not members or shareholders
of the petitioner condominium corporation.

290 | P a g e
Law 321_Corporation LAW_ Case Digest

VELASCO, petitioner
vs.
POIZAT, respondent
G.R. No. L-11528 March 15, 1918

FACTS:

From the amended complaint filed in this cause upon February 5, 1915, it
appears that the plaintiff, as assignee in insolvency of "The Philippine Chemical
Product Company" (Ltd.) is seeking to recover of the defendant, Jean M. Poizat, the
sum of P1,500, upon a subscription made by him to the corporate stock of said
company. It appears that the corporation in question was originally organized by
several residents of the city of Manila, where the company had its principal place of
business, with a capital of P50,000, divided into 500 shares. The defendant
subscribed for 20 shares of the stock of the company, an paid in upon his
subscription the sum of P500, the par value of 5 shares . The action was brought to
recover the amount subscribed upon the remaining shares.
It appears that the defendant was a stock holder in the company from the
inception of the enterprise, and for sometime acted as its treasurer and manager.
While serving in this capacity he called in and collected all subscriptions to the capital
stock of the company, except the aforesaid 15 shares subscribed by himself and
another 15 shares owned by Jose R. Infante.
Upon July 13, 1914, a meeting of the board of directors of the company was
held at which a majority of the stock was presented. Upon this occasion two
resolutions, important to be here noted, were adopted. The first was a proposal that
the directors, or shareholders, of the company should make good by new
subscriptions, in proportion to their respective holdings, 15 shares which had been
surrendered by Infante.

ISSUE:

Whether or not Poizat is liable for his unpaid subscription.

RULING:

YES.

A stock subscription is a contract between the corporation on one side, and the
subscriber on the other, and courts will enforce it for or against either. It is a rule,
accepted by the Supreme Court of the United States that a subscription for shares of
stock does not require an express promise to pay the amount subscribed, as the law
implies a promise to pay on the part of the subscriber. Section 36 of the Corporation
Law clearly recognizes that a stock subscription is subsisting liability from the time
the subscription is made, since it requires the subscriber to pay interest quarterly
from that date unless he is relieved from such liability by the by-laws of the
corporation. The subscriber is as much bound to pay the amount of the share
subscribed by him as he would be to pay any other debt, and the right of the company
to demand payment is no less incontestable.
The provisions of the Corporation Law (Act No. 1459) give recognition of two
remedies for the enforcement of stock subscriptions. The first and most special remedy
given by the statute consists in permitting the corporation to put up the unpaid stock
for sale and dispose of it for the account of the delinquent subscriber. In this case the
provisions of section 38 to 48, inclusive, of the Corporation Law are applicable and
must be followed.
It is generally accepted doctrine that the statutory right to sell the subscriber's
stock is merely a remedy in addition to that which proceeds by action in court; and it
has been held that the ordinary legal remedy by action exists even though no express
mention thereof is made in the statute.

291 | P a g e
Law 321_Corporation LAW_ Case Digest

Acquisition and Ownership of Shares in a Corporation; Extent of Proprietary


Right/Doctrine of Limited Liability

Conjuangco
vs
Republic
12 April 2011 GR NO. 166859

FACTS:

Last April 12, 2011, the Supreme Court en banc rendered its ruling on one of
the most crucial case against the ill-gotten wealth of the Marcoses and their associates
dating back from the Martial Law era involving the shares in San Miguel Corporation
(SMC) allegedly bought with coconut levy funds exacted from the poor marginal
coconut farmers all over the country. This block of shares was purportedly owned by
businessman Eduardo ―Danding‖ Cojuangco and is one of many cases filed way back
in 1987 by the Presidential Commission on Good Government, as part of its mandate
to recover illgotten wealth.
In a nutshell, four justices of the Supreme Court ruled that the Government of
the Philippines (dubbed as the Republic of the Philippines or Republic, for short, in
this case) failed to submit further evidence to prove that the loans from United
Coconut Planters Bank (UCPB) and the Coconut Industry Investment Fund (CIIF)
secured by Eduardo ―Danding‖ Cojuangco to purchase the shares of SMC, were public
in character. A dissenting opinion by Justice Conchita CarpioMorales held the view
that it was Dganding who has failed in his burden of showing that such funds were
not taken from public funds, while another dissenting opinion, by Justice Brion, takes
the view of the majority handing a loss to the Government but urging instead a
prosecution of the government lawyers handling the case of Danding, for their
mishandling of the case, citing several instances when they could have presented
stronger evidence and have taken other steps to bolster their case, but didn't.

ISSUE :

Whether or not Cojuangco breach his ―fiduciary duties‖ as an officer and


member of the Board of Directors of the UCPB? Did his acquisition and holding of the
contested SMC shares come under a constructive trust in favor of the Republic.

RULING:

NO.

The thrust of the Republic that the funds were borrowed or lent might even
preclude any consequent trust implication. In a contract of loan, one of the parties
(creditor) delivers money or other consumable thing to another (debtor) on the
condition that the same amount of the same kind and quality shall be paid.
To say that a relationship is fiduciary when existing laws do not provide for
such requires evidence that confidence is reposed by one party in another who
exercises dominion and influence. Absent any special facts and circumstances proving
a higher degree of responsibility, any dealings between a lender and borrower are not
fiduciary in nature. This explains why, for example, a trust receipt transaction is not
classified as a simple loan and is characterized as fiduciary, because the Trust
Receipts Law punishes the dishonesty and abuse of confidence in the handling of
money or goods to the prejudice of another regardless of whether the latter is the
owner.
Based on the foregoing, a debtor can appropriate the thing loaned without any
responsibility or duty to his creditor to return the very thing that was loaned or to
report how the proceeds were used. Nor can he be compelled to return the proceeds
and fruits of the loan, for there is nothing under our laws that compel a debtor in a
contract of loan to do so. As owner, the debtor can dispose of the thing borrowed and
his act will not be considered misappropriation of the thing.

292 | P a g e
Law 321_Corporation LAW_ Case Digest

MANUEL C. ESPIRITU, JR., AUDIE LLONA, FREIDA F. ESPIRITU, CARLO F.


ESPIRITU, RAFAEL F. ESPIRITU, ROLANDO M. MIRABUNA, HERMILYN A.
MIRABUNA, KIM ROLAND A. MIRABUNA, KAYE ANN A. MIRABUNA, KEN RYAN A.
MIRABUNA, JUANITO P. DE CASTRO, GERONIMA A. ALMONITE and MANUEL C.
DEE, who are the officers and directors of BICOL GAS REFILLING PLANT
CORPORATION
vs.
PETRON CORPORATION and CARMEN J. DOLOIRAS, doing business under the
name "KRISTINA PATRICIA ENTERPRISES,
G.R. No. 170891 November 24, 2009

FACTS:

Petron sold and distributed LPG in cylinder tanks that carried its trademark
"Gasul."1 Respondent Carmen J. Doloiras owned and operated Kristina Patricia
Enterprises, the exclusive distributor of Gasul LPGs in the whole of Sorsogon. Jose
Nelson Doloiras served as KPE‘s manager. Bicol Gas Refilling Plant Corporation was
also in the business of selling and distributing LPGs in Sorsogon but theirs carried the
trademark "Bicol Savers Gas." Petitioner Audie Llona managed Bicol Gas.
On August 4, 2001 KPE‘s Jose saw a particular Bicol Gas truck on the
Maharlika Highway. While the truck carried mostly Bicol Savers LPG tanks, it had on
it one unsealed 50-kg Gasul tank and one 50-kg Shellane tank. He offered to make a
swap for these but Llona declined, saying the Bicol Gas owners wanted to send those
tanks to Batangas. Later Bicol Gas told Jose that it had no more Gasul tanks left in its
possession.

ISSUE:

Whether or not all the Petitioners are liable.

RULING:

NO.

The "owners" of a corporate organization are its stockholders and they are to be
distinguished from its directors and officers. The petitioners here, with the exception
of Audie Llona, are being charged in their capacities as stockholders of Bicol Gas. But
the Court of Appeals forgets that in a corporation, the management of its business is
generally vested in its board of directors, not its stockholders. Stockholders are
basically investors in a corporation. They do not have a hand in running the day-to-
day business operations of the corporation unless they are at the same time directors
or officers of the corporation. Before a stockholder may be held criminally liable for
acts committed by the corporation, therefore, it must be shown that he had knowledge
of the criminal act committed in the name of the corporation and that he took part in
the same or gave his consent to its commission, whether by action or inaction.
The finding of the Court of Appeals that the employees "could not have
committed the crimes without the consent, [abetment], permission, or participation of
the owners of Bicol Gas" is a sweeping speculation especially since, as demonstrated
above, what was involved was just one Petron Gasul tank found in a truck filled with
Bicol Gas tanks. Although the KPE manager heard petitioner Llona say that he was
going to consult the owners of Bicol Gas regarding the offer to swap additional
captured cylinders, no indication was given as to which Bicol Gas stockholders Llona
consulted. It would be unfair to charge all the stockholders involved, some of whom
were proved to be minors. No evidence was presented establishing the names of the
stockholders who were charged with running the operations of Bicol Gas. The
complaint even failed to allege who among the stockholders sat in the board of
directors of the company or served as its officers.

293 | P a g e
Law 321_Corporation LAW_ Case Digest

CRISOSTOMO, petitioner
vs.
S.E.C, respondent
G.R. Nos. 89095 & 89555 November 6, 1989

FACTS:

Sixto Crisostomo, Felipe Crisostomo (deceased), Veronica Palanca, Juanito


Crisostomo, Carlos Crisostomo, Ricardo Alfonso, Regino Crisostomo and Ernesto
Crisostomo (known as the Crisostomo group) were the original stockholders of the
United Doctors Medical Center (UDMC) which was organized in 1968 with an
authorized capital stock of P1,000,000 (later increased to P15,000,000 in 1972). They
owned approximately 40% of UDMC's outstanding capital stock, while the 60%
majority belonged to the members of the United Medical Staff Association (UMSA),
numbering approximately 150 doctors and medical personnel of UDMC.
In 1988, UDMC defaulted in paying its loan obligation of approximately P55
million to the DBP. In the last quarter of 1987, UDMC's assets (principally its hospital)
and those of the Crisostomos which had been given as collateral to the DBP, faced
foreclosure by the Asset Privatization' rust (APT), which had taken over UDMC's loan
obligation to the DBP.
To stave off the threatened foreclosure, UDMC, through its principal officers,
Ricardo Alfonso and Juanito Crisostomo, persuaded the Yamadas and Enatsu (Shoji
Yamada and Tomotada Enatsu are Japanese doctors) to invest fresh capital in UDMC.
The wife of Tomotada Enatsu, Edita Enatsu, is a Filipina. They invested approximately
P57 million in UDMC.
The investment was effected by means of: (1) a Stock Purchase Agreement; and
(2) an Amended Memorandum of Agreement whereby the group subscribed
to 82.09% of the outstanding shares of UDMC. Upon the completion of the
governmental approval process, shares of stock, duly signed by UDMC's authorized
officers, were issued to the Yamadas and Enatsus.
As it had been agreed in the Amended Memorandum of Agreement between
UDMC and the Japanese group that upon the latter's acquisition of the controlling
interest in UDMC, the corporation would be reorganized, a special stockholders'
meeting and board of directors' meeting were scheduled to be held on August 20,
1988.

ISSUE:

Whether or not the investment of the Japanese group in UDMC is


unconstitutional.

RULING:

NO.

While 82% of UDMC's capital stock is indeed subscribed by the Japanese


group, only 30% (equivalent to 171,721 shares or P17,172.00) is owned by the
Japanese citizens, namely, the Yamada spouses and Tomotada Enatsu. 52% is owned
by Edita Enatsu, who is a Filipino. Accordingly, in its application for
approval/registration of the foreign equity investments of these investors, UDMC
declared that 70% of its capital stock is owned by Filipino citizens, including Edita
Enatsu. That application was approved by the Central Bank on August 3, 1988.
The investments in UDMC of Doctors Yamada and Enatsu do not violate the
Constitutional prohibition against foreigners practising a profession in the Philippines
(Section 14, Article XII, 1987 Constitution) for they do not practice their profession
(medicine) in the Philippines, neither have they applied for a license to do so. They
only own shares of stock in a corporation that operates a hospital. No law limits the
sale of hospital shares of stock to doctors only. The ownership of such shares does not
amount to engaging (illegally,) in the practice of medicine, or, nursing. If it were
otherwise, the petitioner's stockholding in UDMC would also be illegal.

294 | P a g e
Law 321_Corporation LAW_ Case Digest

GARCIA, petitioner
vs.
LIM CHU SING, respondent
G.R. No. L-39427 February 24, 1934

FACTS:

On June 20, 1930, the defendant-appellant Lim Chu Sing executed and
delivered to the Mercantile Bank of China promissory note for the sum of P19,605.17
with interest thereon at 6 per cent per annum, payable monthly as follows: P1,000 on
July 1, 1930; P500 on August 1, 1930; and P500 on the first of every month thereafter
until the amount of the promissory note together with the interest thereon is fully paid
(Exhibit A). One of the conditions stipulated in said promissory note is that in case of
defendant's default in the payment of any of the monthly installments, as they become
due, the entire amount or the unpaid balance thereof together with interest thereon at
6 per cent per annum, shall become due and payable on demand. The defendant had
been, making several partial payments thereon, leaving an unpaid balance of
P9,105.17. However, he defaulted in the payment of several installments by reason of
which the unpaid balance of P9,105.17 on the promissory note has ipso facto become
due and demandable.

ISSUE:

Whether or not it is proper to compensate the defendant-appellant's


indebtedness of P9,105.17, which is claimed in the complaint, with the sum of
P10,000 representing the value of his shares of stock with the plaintiff entity, the
Mercantile Bank of China.

RULING:

NO.

According to the weight of authority, a share of stock or the certificate thereof is


not indebtedness to the owner or evidence of indebtedness and, therefore, it is not a
credit. Stockholders, as such, are not creditors of the corporation. It is the prevailing
doctrine of the American courts, repeatedly asserted in the broadest terms, that the
capital stock of a corporation is a trust fund to be used more particularly for the
security of creditors of the corporation, who presumably deal with it on the credit of its
capital stock. Therefore, the defendant-appellant Lim Chu Sing not being a creditor of
the Mercantile Bank of China, although the latter is a creditor of the former, there is
no sufficient ground to justify compensation.

295 | P a g e
Law 321_Corporation LAW_ Case Digest

MAGSAYSAY-LABRADOR, petitioner
vs.
COURT OF APPEALS, respondent
G.R. No. 58168 December 19, 1989

FACTS:

On February 9, 1979, Adelaida Rodriguez-Magsaysay, widow and special


administratix of the estate of the late Senator Genaro Magsaysay, brought before the
then Court of First Instance of Olongapo an action against Artemio Panganiban, Subic
Land Corporation (SUBIC), Filipinas Manufacturer's Bank (FILMANBANK) and the
Register of Deeds of Zambales. In her complaint, she alleged that in 1958, she and her
husband acquired, thru conjugal funds, a parcel of land with improvements, known as
"Pequena Island", covered by TCT No. 3258; that after the death of her husband, she
discovered [a] an annotation at the back of TCT No. 3258 that "the land was acquired
by her husband from his separate capital;" [b] the registration of a Deed of Assignment
dated June 25, 1976 purportedly executed by the late Senator in favor of SUBIC, as a
result of which TCT No. 3258 was cancelled and TCT No. 22431 issued in the name of
SUBIC; and [c] the registration of Deed of Mortgage dated April 28, 1977 in the
amount of P 2,700,000.00 executed by SUBIC in favor of FILMANBANK; that the
foregoing acts were void and done in an attempt to defraud the conjugal partnership
considering that the land is conjugal, her marital consent to the annotation on TCT
No. 3258 was not obtained, the change made by the Register of Deeds of the
titleholders was effected without the approval of the Commissioner of Land
Registration and that the late Senator did not execute the purported Deed of
Assignment or his consent thereto, if obtained, was secured by mistake, violence and
intimidation. She further alleged that the assignment in favor of SUBIC was without
consideration and consequently null and void. She prayed that the Deed of
Assignment and the Deed of Mortgage be annulled and that the Register of Deeds be
ordered to cancel TCT No. 22431 and to issue a new title in her favor.

ISSUE:

Whether or not petitioner‘s ownership in the outstanding capital stock of SUBIC


entitles them to a significant vote in the corporate affairs.

RULING:

NO.

The words "an interest in the subject" mean a direct interest in the cause of
action as pleaded, and which would put the intervenor in a legal position to litigate a
fact alleged in the complaint, without the establishment of which plaintiff could not
recover.
Here, the interest, if it exists at all, of petitioners-movants is indirect,
contingent, remote, conjectural, consequential and collateral. At the very least, their
interest is purely inchoate, or in sheer expectancy of a right in the management of the
corporation and to share in the profits thereof and in the properties and assets thereof
on dissolution, after payment of the corporate debts and obligations.
While a share of stock represents a proportionate or aliquot interest in the
property of the corporation, it does not vest the owner thereof with any legal right or
title to any of the property, his interest in the corporate property being equitable or
beneficial in nature. Shareholders are in no legal sense the owners of corporate
property, which is owned by the corporation as a distinct legal person.

296 | P a g e
Law 321_Corporation LAW_ Case Digest

NICOLAS, petitioner
vs.
COURT OF APPEALS, respondent
G.R. No. 122857 March 27, 1998

FACTS:

On February 19, 1987, petitioner Roy Nicolas and private respondent Blesito
Buan entered into a Portfolio Management Agreement, wherein the former was to
manage the stock transactions of the latter for a period of three months with an
automatic renewal clause. However, upon the initiative of the private respondent the
agreement was terminated on August 19, 1987, and thereafter he requested for an
accounting of all transactions made by the petitioner.
Three weeks after the termination of the agreement, petitioner demanded from
the private respondent the amount of P68,263.67 representing his alleged
management fees covering the periods of June 30, July 31 and August 19, 1987 as
provided for in the Portfolio Management Agreement. But the demands went
unheeded, much to the chagrin of the petitioner.
Rebuffed, petitioner filed a complaint for collection of sum of money against the
private respondent before the trial court. In his answer, private respondent contended
that petitioner mismanaged his transactions resulting in losses, thus, he was not
entitled to any management fees.

ISSUE:

Whether or not petitioner is entitled to management fees.

RULING:

NO.

To begin with, petitioner has the burden to prove that the transaction realized
gains or profits to entitle him to said management fees, as provided in the
Agreement. Accordingly, petitioner submitted the profit and loss statements for the
period of June 30, July 31 and August 19, 1987, showing a total profit
of P341,318.34, of which 20% would represent his management fees amounting
to P68,263.70.
Unfortunately, the profit and loss statements presented by the petitioner are
nothing but bare assertions, devoid of any concrete basis or specifics as to the method
of arriving at the amounts indicated in the documents. In fact, it did not even state
when the stocks were purchased, the type of stocks (whether Class ―A‖ or ―B‖ or
common or preferred) bought, when the stocks were sold, the acquisition and selling
price of each stock, when the profits, if any, were delivered to the private respondent,
the cost of safekeeping or custody of the stocks, as well as the taxes paid for each
transaction. With respect to the alleged losses, it has been held that where a profit or
loss statement shows a loss, the statement must show income and items of expense to
explain the method of determining such loss. However, in the instant petition,
petitioner hardly elucidated the reasons and the factors behind the losses incurred in
the course of the transactions.
In short, no evidentiary value can be attributed to the profit and loss
statements submitted by the petitioner. These documents can hardly be considered a
credible or true reflection of the transactions. It is an incomplete record yielding easily
to the inclusion or deletion of certain matters. The contents are subject to suspicion
since they are not reflective of all pertinent and relevant data. Thus, even assuming
the admissibility of these alleged profit and loss statements, they are devoid of any
evidentiary weight, for the amounts are conclusions without premises, its bases left to
speculation, conjectures, assertions and guesswork.

297 | P a g e
Law 321_Corporation LAW_ Case Digest

RAMOS, petitioner
vs.
COURT OF APPEALS, respondent
G.R. No. L-41295 December 4, 1989

FACTS:

On August 14 and 26, 1969, CMS Stock Brokerage, Inc. (or CMS) sold to Lopez,
Locsin, Ledesma & Co., Inc. (or LLL) on the floor of the Makati Stock Exchange (or
MSE) 2,650 shares of Benguet Consolidated Corporation for P297,650 on a delayed
delivery basis of 10 to 20 days, evidenced by Exchange Contracts Nos. B-11807 and B-
11814 both dated August 14, 1969 and B-13084 dated August 26, 1969. LLL bought
the shares for the account of its clients, the third-party defendants, Rene Ledesma,
Jose Maria Lopez, Cesar A. Lopez, Jr. and Alfredo Ramos. CMS failed to deliver the
shares of stocks within the agreed period, but LLL did not demand delivery.
On January 6, 1970, CMS informed LLL that it would deliver the shares the
next day. LLL wrote CMS that it would not accept the shares because its principals
had cancelled their orders. In its reply, CMS insisted that LLL take delivery of the
Benguet shares.
In CMS's Clearing House Report of January 9, 1970, the disposition of the
shares in favor of LLL appeared, but the latter refused to acknowledge receipt of the
covering disposal letter. CMS then deposited the letter in the Office of the Exchange
Executive, Secretary with the notation "Refused acceptance pending decision of the
Exchange".
When the controversy was submitted to the Board of Governors of the Exchange
for determination, the Board issued Resolution No. 523 on August 10, 1970 advising
the parties to litigate the matter in court.
Accordingly, CMS filed in the Court of First Instance of Rizal a complaint to
compel LLL to accept the Benguet shares, to pay the price of P297,650, as well as
P25,000 as attorney's fees and costs. LLL's motion to dismiss the complaint was
denied.

ISSUE:

Whether or not appellate court erred in rendering a summary judgment, in


failing to find that CMS has no right to damages against the petitioner there being no
privity of contract between them, and in refusing to exonerate the petitioner from
personal liability for the orders he placed with LLL for the account of the Alakor
Corporation.

RULING:

NO.

In the case at bar, the stock purchases of LLL were on a 10-20 day delayed
delivery basis. Accordingly, after that period lapsed, the Buying Member (LLL) should
have advised the Selling Member CMS in writing.
As observed by the trial court, Section 1, Article V of the Exchange Rules does
not vest on the buyer, respondent LLL, a right to rescind its contract with CMS upon
the latter's default. The Exchange Rules obligate the buyer to make a demand, and if
the selling member fails to deliver the ordered shares despite the demand, the buyer is
further obligated to deliver a copy of his demand letter to the Chairman of the Floor
Trading and Arbitration Committee so that the latter may purchase the shares for the
selling member's account. Said rules were held binding on members of the Exchange.
Inasmuch as petitioner placed his order for Benguet shares through a member of the
Exchange (LLL), he is indirectly bound by the rules of the Exchange.
The defendants' lack of knowledge regarding the truth of the allegation in the
complaint, that the failure of CMS to deliver the Benguet shares on time was due to
oversight, did not constitute an obstacle to the rendition of a summary judgment by
the trial court, for although an averment of lack of knowledge has the effect of a
denial, it does not raise a genuine issue.

298 | P a g e
Law 321_Corporation LAW_ Case Digest

SAW, petitioner
vs.
COURT OF APPEALS, respondent
G.R. No. 90580 April 8, 1991

FACTS:

A collection suit with preliminary attachment was filed by Equitable Banking


Corporation against Freeman, Inc. and Saw Chiao Lian, its President and General
Manager. The petitioners moved to intervene, alleging that (1) the loan transactions
between Saw Chiao Lian and Equitable Banking Corp. were not approved by the
stockholders representing at least 2/3 of corporate capital; (2) Saw Chiao Lian had no
authority to contract such loans; and (3) there was collusion between the officials of
Freeman, Inc. and Equitable Banking Corp. in securing the loans. The motion to
intervene was denied, and the petitioners appealed to the Court of Appeals.
Meanwhile, Equitable and Saw Chiao Lian entered into a compromise
agreement which they submitted to and was approved by the lower court. But because
it was not complied with, Equitable secured a writ of execution, and two lots owned by
Freeman, Inc. were levied upon and sold at public auction to Freeman Management
and Development Corp.
The Court of Appeals sustained the denial of the petitioners' motion for
intervention, holding that "the compromise agreement between Freeman, Inc., through
its President, and Equitable Banking Corp. will not necessarily prejudice petitioners
whose rights to corporate assets are at most inchoate, prior to the dissolution of
Freeman, Inc. And intervention under Sec. 2, Rule 12 of the Revised Rules of Court is
proper only when one's right is actual, material, direct and immediate and not simply
contingent or expectant."
It also ruled against the petitioners' argument that because they had already
filed a notice of appeal, the trial judge had lost jurisdiction over the case and could no
longer issue the writ of execution.

ISSUE:

Whether or not the Honorable Court of Appeals erred in holding that the
petitioners cannot intervene in Civil Case No. 88-44404 because their rights as
stockholders of Freeman are merely inchoate and not actual, material, direct and
immediate prior to the dissolution of the corporation.

RULING:

NO.

The petitioners base their right to intervene for the protection of their interests
as stockholders on Everett v. Asia Banking Corp. where it was held: The well-known
rule that shareholders cannot ordinarily sue in equity to redress wrongs done to the
corporation, but that the action must be brought by the Board of Directors, has its
exceptions.
Equitable demurs, contending that the collection suit against Freeman, Inc,
and Saw Chiao Lian is essentially in personam and, as an action against defendants in
their personal capacities, will not prejudice the petitioners as stockholders of the
corporation. The Everett case is not applicable because it involved an action filed by
the minority stockholders where the board of directors refused to bring an action in
behalf of the corporation. In the case at bar, it was Freeman, Inc. that was being sued
by the creditor bank.
On the second assignment of error, Equitable maintains that the petitioners'
appeal could only apply to the denial of their motion for intervention and not to the
main case because their personality as party litigants had not been recognized by the
trial court.
After examining the issues and arguments of the parties, the Court finds that
the respondent court committed no reversible error in sustaining the denial by the
trial court of the petitioners' motion for intervention.

299 | P a g e
Law 321_Corporation LAW_ Case Digest

Consideration for Stocks

APODACA, petitioner
vs.
NATIONAL LABOR RELATIONS COMMISSION, respondent
G.R. No. 80039 April 18, 1989

FACTS:

Petitioner was employed in respondent corporation. On August 28, 1985,


respondent Jose M. Mirasol persuaded petitioner to subscribe to 1,500 shares of
respondent corporation at P100.00 per share or a total of P150,000.00. He made an
initial payment of P37,500.00. On September 1, 1975, petitioner was appointed
President and General Manager of the respondent corporation. However, on January
2, 1986, he resigned.
On December 19, 1986, petitioner instituted with the NLRC a complaint against
private respondents for the payment of his unpaid wages, his cost of living allowance,
the balance of his gasoline and representation expenses and his bonus compensation
for 1986. Petitioner and private respondents submitted their position papers to the
labor arbiter. Private respondents admitted that there is due to petitioner the amount
of P17,060.07 but this was applied to the unpaid balance of his subscription in the
amount of P95,439.93. Petitioner questioned the set-off alleging that there was no call
or notice for the payment of the unpaid subscription and that, accordingly, the alleged
obligation is not enforceable.
In a decision dated April 28, 1987, the labor arbiter sustained the claim of
petitioner for P17,060.07 on the ground that the employer has no right to withhold
payment of wages already earned under Article 103 of the Labor Code. Upon the
appeal of the private respondents to public respondent NLRC, the decision of the labor
arbiter was reversed in a decision dated September 18, 1987. The NLRC held that a
stockholder who fails to pay his unpaid subscription on call becomes a debtor of the
corporation and that the set-off of said obligation against the wages and others due to
petitioner is not contrary to law, morals and public policy.

ISSUE:

Whether or not an obligation arising from non-payment of stock subscriptions


to a corporation can be offset against a money claim of an employee against the
employer.

RULING:

NO.

The unpaid subscriptions are not due and payable until a call is made by the
corporation for payment. Private respondents have not presented a resolution of the
board of directors of respondent corporation calling for the payment of the unpaid
subscriptions. It does not even appear that a notice of such call has been sent to
petitioner by the respondent corporation.
What the records show is that the respondent corporation deducted the amount
due to petitioner from the amount receivable from him for the unpaid
subscriptions. No doubt such set-off was without lawful basis, if not premature. As
there was no notice or call for the payment of unpaid subscriptions, the same is not
yet due and payable.
Lastly, assuming further that there was a call for payment of the unpaid
subscription, the NLRC cannot validly set it off against the wages and other benefits
due petitioner. Article 113 of the Labor Code allows such a deduction from the wages
of the employees by the employer, only in three instances, to wit: No employer, in his
own behalf or in behalf of any person, shall make any deduction from the wages of his
employees, except in certain cases.

300 | P a g e
Law 321_Corporation LAW_ Case Digest

FUA CUN, petitioner


vs.
SUMMERS, respondent
G.R. No. L-19441 March 27, 1923

FACTS:

It appears from the evidence that on August 26, 1920, one Chua Soco
subscribed for five hundred shares of stock of the defendant Banking Corporation at a
par value of P100 per share, paying the sum of P25,000, one-half of the subscription
price, in cash, for which a receipt was issued.
On May 18, 1921, Chua Soco executed a promissory note in favor of the
plaintiff Fua Cun for the sum of P25,000 payable in ninety days and drawing interest
at the rate of 1 per cent per month, securing the note with a chattel mortgage on the
shares of stock subscribed for by Chua Soco, who also endorsed the receipt above
mentioned and delivered it to the mortgagee. The plaintiff thereupon took the receipt
to the manager of the defendant Bank and informed him of the transaction with Chua
Soco, but was told to await action upon the matter by the Board of Directors.
In the meantime Chua Soco appears to have become indebted to the China
Banking Corporation in the sum of P37,731.68 for dishonored acceptances of
commercial paper and in an action brought against him to recover this amount, Chua
Soco's interest in the five hundred shares subscribed for was attached and the receipt
seized by the sheriff. The attachment was levied after the defendant bank had received
notice of the facts that the receipt had been endorsed over to the plaintiff. Fua Cun
thereupon brought the present action maintaining that by virtue of the payment of the
one-half of the subscription price of five hundred shares Chua Soco in effect became
the owner of two hundred and fifty shares and praying that his, the plaintiff's, lien on
said shares, by virtue of the chattel mortgage, be declared to hold priority over the
claim of the defendant Banking Corporation; that the defendants be ordered to deliver
the receipt in question to him; and that he be awarded the sum of P5,000 in damages
for wrongful attachment.

ISSUE:

Whether or not the trial court erred in declaring that Chua Soco, through the
payment of the P25,000, acquired the right to two hundred and fifty shares fully paid
up, upon which shares the plaintiff holds a lien superior to that of the defendant
Banking Corporation and ordering that the receipt be returned to said plaintiff.

RULING:

YES.

The claim of the defendant Banking Corporation upon which it brought the
action in which the writ of attachment was issued, was for the non-payment of drafts
accepted by Chua Soco and had no direct connection with the shares of stock in
question. At common law a corporation has no lien upon the shares of stockholders
for any indebtedness to the corporation and our attention has not been called to any
statute creating such lien here. On the contrary, section 120 of the Corporation Act
provides that "no bank organized under this Act shall make any loan or discount on
the security of the shares of its own capital stock, nor be the purchaser or holder of
any such shares, unless such security or purchase shall be necessary to prevent loss
upon a debt previously contracted in good faith, and stock so purchased or acquired
shall, within six months from the time of its purchase, be sold or disposed of at public
or private sale, or, in default thereof, a receiver may be appointed to close up the
business of the bank in accordance with law."
The reasons for this doctrine are obvious; if banking corporations were given a
lien on their own stock for the indebtedness of the stockholders, the prohibition
against granting loans or discounts upon the security of the stock would become
largely ineffective.

301 | P a g e
Law 321_Corporation LAW_ Case Digest

NATIONAL EXCHANGE CO., INC., petitioner


vs.
I.B. DEXTER, respondent
G.R. No. L-27872 February 25, 1928

FACTS:

This action was instituted in the Court of First Instance of Manila by the
National Exchange Co., Inc., as assignee (through the Philippine National Bank) of C.
S. Salmon & Co., for the purpose of recovering from I. B. Dexter a balance of P15,000,
the par value of one hundred fifty shares of the capital stock of C. S. Salmon & co.,
with interest and costs. Upon hearing the cause the trial judge gave judgment for the
plaintiff to recover the amount claimed, with lawful interest from January 1, 1920,
and with costs. From this judgment the defendant appealed.
It appears that on August 10, 1919, the defendant, I. B. Dexter, signed a
written subscription to the corporate stock of C. S. Salmon & Co. in the following
form: I hereby subscribe for three hundred (300) shares of the capital stock of C. S.
Salmon and Company, payable from the first dividends declared on any and all shares
of said company owned by me at the time dividends are declared, until the full amount
of this subscription has been paid.
Upon this subscription the sum of P15,000 was paid in January, 1920, from a
dividend declared at about that time by the company, supplemented by money
supplied personally by the subscriber. Beyond this nothing has been paid on the
shares and no further dividend has been declared by the corporation. There is
therefore a balance of P15,000 still paid upon the subscription.

ISSUE:

Whether or not the stipulation contained in the subscription to the effect that
the subscription is payable from the first dividends declared on the shares has the
effect of relieving the subscriber from personal liability in an action to recover the
value of the shares.

RULING:

NO.

In discussing this problem we accept as sound law the proposition propounded


by the appellant's attorneys and taken from Fletcher's Cyclopedia as follows: In the
absence of restrictions in its character, a corporation, under its general power to
contract, has the power to accept subscriptions upon any special terms not prohibited by
positive law or contrary to public policy, provided they are not such as to require the
performance of acts which are beyond the powers conferred upon the corporation by its
character, and provided they do not constitute a fraud upon other subscribers or
stockholders, or upon persons who are or may become creditors of the corporation.
Pursuant to such, we find that the Philippine Commission inserted in the
Corporation Law, enacted March 1, 1906, the following provision: "no corporation
shall issue stock or bonds except in exchange for actual cash paid to the corporation
or for property actually received by it at a fair valuation equal to the par value of the
stock or bonds so issued."
The prohibition against the issuance of shares by corporations except for actual
cash to the par value of the stock to its full equivalent in property is thus enshrined in
both the organic and statutory law of the Philippine; Islands; and it would seem that
our lawmakers could scarely have chosen language more directly suited to secure
absolute equality stockholders with respect to their liability upon stock subscriptions.
Now, if it is unlawful to issue stock otherwise than as stated it is self-evident that a
stipulation such as that now under consideration, in a stock subcription, is illegal, for
this stipulation obligates the subcriber to pay nothing for the shares except as
dividends may accrue upon the stock. In the contingency that dividends are not paid,
there is no liability at all. This is discrimination in favor of the particular subcriber,
and hence the stipulation is unlawful.

302 | P a g e
Law 321_Corporation LAW_ Case Digest

NIELSON & CO., INC., plaintiff


vs.
LEPANTO CONSOLIDATED MINING CO., defendant
G.R. No. L-21601 December 17, 1966

FACTS:

An operating agreement was executed before World War II (on 30 January 1937)
between Nielson & Co. Inc. and the Lepanto Consolidated Mining Co. whereby the
former operated and managed the mining properties owned by the latter for a
management fee of P2,500.00 a month and a 10% participation in the net profits
resulting from the operation of the mining properties, for a period of 5 years. In 1940,
a dispute arose regarding the computation of the 10% share of Nielson in the profits.
The Board of Directors of Lepanto, realizing that the mechanics of the contract was
unfair to Nielson, authorized its President to enter into an agreement with Nielson
modifying the pertinent provision of the contract effective 1 January 1940 in such a
way that Nielson shall receive (1) 10% of the dividends declared and paid, when and as
paid, during the period of the contract and at the end of each year, (2) 10% of any
depletion reserve that may be set up, and (3) 10% of any amount expended during the
year out of surplus earnings for capital account. In the latter part of 1941, the parties
agreed to renew the contract for another period of 5 years, but in the meantime, the
Pacific War broke out in December 1941. In January 1942 operation of the mining
properties was disrupted on account of the war. In February 1942, the mill, power
plant, supplies on hand, equipment, concentrates on hand and mines, were destroyed
upon orders of the United States Army, to prevent their utilization by the invading
Japanese Army. The Japanese forces thereafter occupied the mining properties,
operated the mines during the continuance of the war, and who were ousted from the
mining properties only in August 1945. Shortly after the mines were liberated from the
Japanese invaders in 1945, a disagreement arose between NIELSON and LEPANTO
over the status of the operating contract which as renewed expired in 1947.

ISSUE:

Whether or not the management contract is a contract of agency.

RULING:

NO.

In the performance of this principal undertaking Nielson was not in any way
executing juridical acts for Lepanto, destined to create, modify or extinguish business
relations between Lepanto and third persons. In other words, in performing its
principal undertaking Nielson was not acting as an agent of Lepanto, in the sense that
the term agent is interpreted under the law of agency, but as one who was performing
material acts for an employer, for a compensation. It is true that the management
contract provides that Nielson would also act as purchasing agent of supplies and
enter into contracts regarding the sale of mineral, but the contract also provides that
Nielson could not make any purchase, or sell the minerals, without the prior approval
of Lepanto. It is clear, therefore, that even in these cases Nielson could not execute
juridical acts which would bind Lepanto without first securing the approval of
Lepanto. Nielson, then, was to act only as an intermediary, not as an agent. Further,
from the statements in the annual report for 1936, and from the provision of
paragraph XI of the Management contract, that the employment by Lepanto of Nielson
to operate and manage its mines was principally in consideration of the know-how and
technical services that Nielson offered Lepanto. The contract thus entered into
pursuant to the offer made by Nielson and accepted by Lepanto was a "detailed
operating contract". It was not a contract of agency. Nowhere in the record is it shown
that Lepanto considered Nielson as its agent and that Lepanto terminated the
management contract because it had lost its trust and confidence in Nielson.

303 | P a g e
Law 321_Corporation LAW_ Case Digest

TRILLANA, petitioner
vs.
QUEZON COLLEGE, INC., respondent
G.R. No. L-5003 June 27, 1953

FACTS:

Damasa Crisostomo sent a letter to the Board of Trustees of the Quezon College
subscribing to 200 shares of its capital stock at par value of Php100 each. Damasa
Crisostomo died on October 26, 1948. As no payment appears to have been made on
the subscription mentioned in her letter, the Quezon College, Inc. presented a claim
before the Court of First Instance of Bulacan in her testate proceeding, for the
collection of the sum of P20,000, representing the value of the subscription to the
capital stock of the Quezon College, Inc. This claim was opposed by the administrator
of the estate, and the Court of First Instance of Bulacan, after hearing issued an order
dismissing the claim of the Quezon College, Inc. on the ground that the subscription
in question was neither registered in nor authorized by the Securities and Exchange
Commission. From this order the Quezon College, Inc. has appealed.

ISSUE:

Whether or not the subscription applied for by Damasa Crisostomo is an


enforceable contract.

RULING:

NO.

It appears that the application sent by Damasa Crisostomo to the Quezon


College, Inc. was written on a general form indicating that an applicant will enclose an
amount as initial payment and will pay the balance in accordance with law and the
regulations of the College. On the other hand, in the letter actually sent by Damasa
Crisostomo, the latter (who requested that her subscription for 200 shares be entered)
not only did not enclose any initial payment but stated that "babayaran kong lahat
pagkatapos na ako ay makapagpahuli ng isda." There is nothing in the record to show
that the Quezon College, Inc. accepted the term of payment suggested by Damasa
Crisostomo, or that if there was any acceptance the same came to her knowledge
during her lifetime. As the application of Damasa Crisostomo is obviously at variance
with the terms evidenced in the form letter issued by the Quezon College, Inc., there
was absolute necessity on the part of the College to express its agreement to Damasa's
offer in order to bind the latter. Conversely, said acceptance was essential, because it
would be unfair to immediately obligate the Quezon College, Inc. under Damasa's
promise to pay the price of the subscription after she had caused fish to be caught. In
other words, the relation between Damasa Crisostomo and the Quezon College, Inc.
had only thus reached the preliminary stage whereby the latter offered its stock for
subscription on the terms stated in the form letter, and Damasa applied for
subscription fixing her own plan of payment, — a relation, in the absence as in the
present case of acceptance by the Quezon College, Inc. of the counter offer of Damasa
Crisostomo, that had not ripened into an enforceable contract.
Indeed, the need for express acceptance on the part of the Quezon College, Inc.
becomes the more imperative, in view of the proposal of Damasa Crisostomo to pay the
value of the subscription after she has harvested fish, a condition obviously dependent
upon her sole will and, therefore, facultative in nature, rendering the obligation void,
under article 1115 of the old Civil Code which provides as follows: "If the fulfillment of
the condition should depend upon the exclusive will of the debtor, the conditional
obligation shall be void. If it should depend upon chance, or upon the will of a third
person, the obligation shall produce all its effects in accordance with the provisions of
this code." It cannot be argued that the condition solely is void, because it would have
served to create the obligation to pay, wherein only the potestative condition was held
void because it referred merely to the fulfillment of an already existing indebtedness.

304 | P a g e
Law 321_Corporation LAW_ Case Digest

Unpaid Subscriptions: Call: When necessary

GARCIA, plaintiff
vs.
SUAREZ, defendant
G.R. No. L-45493 April 21, 1939

FACTS:

On October 4, 1924, the appellant subscribed to sixteen shares of the capital


stock of the Compañia Hispano-Filipina, Inc., a corporation which is duly formed and
organized. Of the sixteen subscribed shares, at the par value of P100 each, the
appellant only paid P400, the value of four shares. On June 5, 1931, the plaintiff-
appellee was appointed by the court receiver of the Compañia Hispano-Filipina, Inc.,
to collect all the credits of said corporation, pay its debts and dispose of the remainder
of its assets and of its properties. On June 18, 1931, the plaintiff-appellee in vain
made demand upon the defendant-appellant to pay the balance of his subscription.
On July 10, 1933, the plaintiff, as receiver, brought an action in the Court of First
Instance of Manila to recover from the defendant-appellant and other shareholders the
balance of their subscriptions, but the complaint was dismissed for lack of
prosecution. On October 10, 1935, a similar complaint was filed against the appellant,
and after trial, judgment was rendered therein ordering the said defendant to pay to
the plaintiff, as receiver of Compañia Hispano-Filipina, Inc., the sum of P1,200, with
legal interest thereon from October 4, 1924, and the costs. The defendant appealed
and in this instance contends that the trial court erred in holding that the action of
the plaintiff-appellee has not prescribed, and that the appellant has not been released
from his obligation to pay the balance of his subscription.

ISSUE:

Whether or not the obligation contracted by the appellant to pay the value of his
subscription was demandable from the date of subscription in the absence of any
stipulation to the contrary.

RULING:

NO.

Section 37 of the Corporation Law provides when the obligation to pay interest
arises and when payment should be made, but it is absolutely silent as to when the
subscription to a stock should be paid. Of course, the obligation to pay arises from the
date of the subscription, but the coming into being of an obligation should not be
confused with the time when it becomes demandable. In a loan for example, the
obligation to pay arises from the time the loan is taken; but the maturity of that
obligation, the date when the debtor can be compelled to pay, is not the date itself of
the loan, because this would be absurd. The date when payment can be demanded is
necessarily distinct from and subsequent to that the obligation is contracted.
By the same token, the subscription to the capital stock of the corporation,
unless otherwise stipulation, is not payable at the moment of the subscription but on
a subsequent date which may be fixed by the corporation. Hence, section 38 of the
Corporation Law, amended by Act No. 3518, provides that: The board of directors or
trustees of any stock corporation formed, organized, or existing under this Act may at
any time declare due and payable to the corporation unpaid subscriptions to the capital
stock.
The board of directors of the Compañia Hispano-Filipino, Inc., not having
declared due and payable the stock subscribed by the appellant, the prescriptive
period of the action for the collection thereof only commenced to run from June 18,
1931 when the plaintiff, in his capacity as receiver and in the exercise of the power
conferred upon him by the said section 38 of the Corporation Law, demanded of the
appellant to pay the balance of his subscription. The present action having been filed
on October 10, 1935, the defense of prescription is entirely without basis.

305 | P a g e
Law 321_Corporation LAW_ Case Digest

PHILIPPINE NATIONAL BANK, plaintiff


vs.
BITULOK SAWMILL INC., defendant
G.R. Nos. L-24177-85 June 29, 1968

FACTS:

The Philippine Lumber Distributing Agency, Inc., according to the lower court,
"was organized sometime in the early part of 1947 upon the initiative and insistence of
the late President Manuel Roxas of the Republic of the Philippines who for the
purpose, had called several conferences between him and the subscribers and
organizers of the Philippine Lumber Distributing Agency, Inc." The purpose was
praiseworthy, to insure a steady supply of lumber, which could be sold at reasonable
prices to enable the war sufferers to rehabilitate their devastated homes. At the
beginning, the lumber producers were reluctant to organize the cooperative agency as
they believed that it would not be easy to eliminate from the retail trade the alien
middlemen who had been in this business from time immemorial, but because the late
President Roxas made it clear that such a cooperative agency would not be successful
without a substantial working capital which the lumber producers could not entirely
shoulder, and as an inducement he promised and agreed to finance the agency by
making the Government invest P9.00 by way of counterpart for every peso that the
members would invest therein."
Accordingly, "the late President Roxas instructed the Hon. Emilio Abello, then
Executive Secretary and Chairman of the Board of Directors of the Philippine National
Bank, for the latter to grant said agency an overdraft in the original sum of
P250,000.00 which was later increased to P350,000.00, which was approved by said
Board of Directors of the Philippine National Bank on July 28, 1947, payable on or
before April 30, 1958, with interest at the rate of 6% per annum, and secured by the
chattel mortgages on the stock of lumber of said agency." The Philippine Government
did not invest the P9.00 for every peso coming from defendant lumber producers. The
loan extended to the Philippine Lumber Distributing Agency by the Philippine National
Bank was not paid.

ISSUE:

Whether or not the non-compliance with a plain statutory command,


considering the persuasiveness of the plea that defendants-appellees would "not have
subscribed to the capital stock" of the Philippine Lumber Distributing Agency "were it
not for the assurance of the then President of the Republic that the Government would
back it up by investing P9.00 for every peso" subscribed, a condition which was not
fulfilled, such commitment not having been complied with, be justified.

RULING:

NO.

It would be unwarranted to ascribe to the late President Roxas the view that the
payment of the stock subscriptions, as thus required by law, could be condoned in the
event that the counterpart fund to be invested by the Government would not be
available. Even if such were the case, however, and such a promise were in fact made,
to further the laudable purpose to which the proposed corporation would be devoted
and the possibility that the lumber producers would lose money in the process, still
the plain and specific wording of the applicable legal provision as interpreted by this
Court must be controlling. It is a well-settled principle that with all the vast powers
lodged in the Executive, he is still devoid of the prerogative of suspending the
operation of any statute or any of its terms.

306 | P a g e
Law 321_Corporation LAW_ Case Digest

VELASCO, plaintiff
vs.
POIZAT, defendant
G.R. No. L-11528 March 15, 1918

FACTS:

From the amended complaint filed in this cause upon February 5, 1915, it
appears that the plaintiff, as assignee in insolvency of "The Philippine Chemical
Product Company" (Ltd.) is seeking to recover of the defendant, Jean M. Poizat, the
sum of P1,500, upon a subscription made by him to the corporate stock of said
company. It appears that the corporation in question was originally organized by
several residents of the city of Manila, where the company had its principal place of
business, with a capital of P50,000, divided into 500 shares. The defendant
subscribed for 20 shares of the stock of the company, an paid in upon his
subscription the sum of P500, the par value of 5 shares . The action was brought to
recover the amount subscribed upon the remaining shares.
It appears that the defendant was a stock holder in the company from the
inception of the enterprise, and for sometime acted as its treasurer and manager.
While serving in this capacity he called in and collected all subscriptions to the capital
stock of the company, except the aforesaid 15 shares subscribed by himself and
another 15 shares owned by Jose R. Infante.
Upon July 13, 1914, a meeting of the board of directors of the company was
held at which a majority of the stock was presented. Upon this occasion two
resolutions, important to be here noted, were adopted. The first was a proposal that
the directors, or shareholders, of the company should make good by new
subscriptions, in proportion to their respective holdings, 15 shares which had been
surrendered by Infante.

ISSUE:

Whether or not Poizat is liable for his unpaid subscription.

RULING:

YES.

A stock subscription is a contract between the corporation on one side, and the
subscriber on the other, and courts will enforce it for or against either. It is a rule,
accepted by the Supreme Court of the United States that a subscription for shares of
stock does not require an express promise to pay the amount subscribed, as the law
implies a promise to pay on the part of the subscriber. Section 36 of the Corporation
Law clearly recognizes that a stock subscription is subsisting liability from the time
the subscription is made, since it requires the subscriber to pay interest quarterly
from that date unless he is relieved from such liability by the by-laws of the
corporation. The subscriber is as much bound to pay the amount of the share
subscribed by him as he would be to pay any other debt, and the right of the company
to demand payment is no less incontestable.
The provisions of the Corporation Law (Act No. 1459) given recognition of two
remedies for the enforcement of stock subscriptions. The first and most special remedy
given by the statute consists in permitting the corporation to put up the unpaid stock
for sale and dispose of it for the account of the delinquent subscriber. In this case the
provisions of section 38 to 48, inclusive, of the Corporation Law are applicable and
must be followed. The other remedy is by action in court.
It is generally accepted doctrine that the statutory right to sell the subscriber's
stock is merely a remedy in addition to that which proceeds by action in court; and it
has been held that the ordinary legal remedy by action exists even though no express
mention thereof is made in the statute.

307 | P a g e
Law 321_Corporation LAW_ Case Digest

Court Action

LUMANLAN, plaintiff
vs.
CURA, et al., defendants
G.R. No. L-39861 March 21, 1934

FACTS:

The appellant is a corporation duly organized under the laws of the Philippine
Islands with its central office in the City of Manila. The plaintiff-appellee Bonifacio
Lumanlan, on July 31, 1922, subscribed for 300 shares of stock of said corporation at
a par value of P50 or a total of P15,000. Julio Valenzuela, Pedro Santos and Francisco
Escoto, creditors of this corporation, filed suit against it in the Court of First Instance
of Manila, case No. 37007, praying that a receiver be appointed, as it appeared that
the corporation at that time had no assets except credits against those who had
subscribed for shares of stock. The court named Tayag as receiver for the purpose of
collecting, said subscriptions. As Bonifacio Lumanlan had only paid P1,500 of the
P15,000, par value of the stock for which he subscribed, the receiver on August 30,
1930, filed a suit against him in the Court of First Instance of Manila, civil case No.
37492, for the collection of P15,109, P13,500 of which was the amount he owed for
unpaid stock and P1,609 for loans and advances by the corporation to Lumanlan. In
that case Lumanlan was sentenced to pay the corporation the above-mentioned sum
of P15,109 with legal interest thereon from August 30, 1930, and costs. Lumanlan
appealed from this decision.

ISSUE:

Whether or not Bonifacio Lumanlan is entitled to a credit against the judgment


in case No. 37492 for P11,840 and an additional sum of P2,000, which is 25 per cent
on the principal debt, as he had to file this suit to collect, or receive credit for the sum
which he had paid Valenzuela for and in place of the corporation, or a total of
P13,840.

RULING:

YES.

It appears from the record that during the trial of the case now under
consideration, the Bank of the Philippine Islands appeared in this case as assignee in
the "Involuntary Insolvency of Dizon & Co., Inc. That bank was appointed assignee in
case No. 43065 of the Court of First Instance of the City of Manila on November 28,
1932. It is therefore evident that there are still other creditors of Dizon & Co., Inc. This
being the case that corporation has a right to collect all unpaid stock subscriptions
and any other amounts which may be due it.
It is established doctrine that subscriptions to the capital of a corporation
constitute a fund to which the creditors have a right to look for satisfaction of their
claims and that the assignee in insolvency can maintain an action upon any unpaid
stock subscription in order to realize assets for the payment of its debts. The
Corporation Law clearly recognizes that a stock subscription is a subsisting liability
from the time the subscription is made, since it requires the subscriber to pay interest
quarterly from that date unless he is relieved from such liability by the by-laws of the
corporation. The subscriber is as much bound to pay the amount of the share
subscribed by him as he would be to pay any other debt, and the right of the company
to demand payment is no less incontestable.

308 | P a g e
Law 321_Corporation LAW_ Case Digest

EDWARD KELLER & Co., Ltd., petitioner


vs.
COB GROUP MARKETING, respondent
G.R. No. L-68097 January 16, 1986

FACTS:

Edward A. Keller & Co., Ltd. appointed COB Group Marketing, Inc. as exclusive
distributor of its household products, Brite and Nuvan in Panay and Negros, as shown
in the sales agreement dated March 14, 1970 . Under that agreement Keller sold on
credit its products to COB Group Marketing.
As security for COB Group Marketing's credit purchases up to the amount of
P35,000, one Asuncion Manahan mortgaged her land to Keller. Manahan assumed
solidarily with COB Group Marketing the faithful performance of all the terms and
conditions of the sales agreement.
In July, 1970 the parties executed a second sales agreement whereby COB
Group Marketing's territory was extended to Northern and Southern Luzon. As
security for the credit purchases up to P25,000 of COB Group Marketing for that area,
Tomas C. Lorenzo, Jr. and his father Tomas, Sr. (now deceased) executed a mortgage
on their land in Nueva Ecija. Like Manahan, the Lorenzos were solidarily liable with
COB Group Marketing for its obligations under the sales agreement.
The credit purchases of COB Group Marketing, which started on October 15,
1969, limited up to January 22, 1971. On May 8, the board of directors of COB Group
Marketing were apprised by Jose E. Bax the firm's president and general
manager, that the firm owed Keller about P179,000. Bax was authorized to negotiate
with Keller for the settlement of his firm's liability. On the same day, May 8, Bax and
R. Oefeli of Keller signed the conditions for the settlement of COB Group Marketing's
liability. Twelve days later, or on May 20, COB Group Marketing, through Bax
executed two second chattel mortgages over its 12 trucks (already mortgaged to
Northern Motors, Inc.) as security for its obligation to Keller amounting to
P179,185.16 as of April 30, 1971.

ISSUE:

Whether or not the lower courts erred in nullifying the admissions of liability
made in 1971 by Bax as president and general manager of COB Group Marketing and
in giving credence to the alleged overpayment computed by Bax.

RULING:

YES.

The lower courts not only allowed Bax to nullify his admissions as to the
liability of COB Group Marketing but they also erroneously rendered judgment in its
favor in the amount of its supposed overpayment in the sum of P100,596.72, in spite
of the fact that COB Group Marketing was declared in default and did not file any
counterclaim for the supposed overpayment. The lower courts harped on Keller's alleged
failure to thresh out with representatives of COB Group Marketing their "diverse
statements of credits and payments". This contention has no factual basis. That
means that there was a conference on the COB Group Marketing's liability. Bax in
that discussion did not present his reconciliation statements to show overpayment.
Bax admitted that Keller sent his company monthly statements of accounts but
he could not produce any formal protest against the supposed inaccuracy of the said
statements. He lamely explained that he would have to dig up his company's records
for the formal protest. He did not make any written demand for reconciliation of
accounts.
As to the liability of the stockholders, it is settled that a stockholder is
personally liable for the financial obligations of a corporation to the extent of his
unpaid subscription.

309 | P a g e
Law 321_Corporation LAW_ Case Digest

Effect of Delinquency

VALLEY GOLF & COUNTRY CLUB, Inc., petitioner


vs.
VDA. DE CARAM, respondent
G.R. No. 158805 April 16, 2009

FACTS:

Valley Golf & Country Club (Valley Golf) is a duly constituted non-stock, non-
profit corporation which operates a golf course. The members and their guests are
entitled to play golf on the said course and otherwise avail of the facilities and
privileges provided by Valley Golf. The shareholders are likewise assessed monthly
membership dues.
In 1961, the late Congressman Fermin Z. Caram, Jr. (Caram), the husband of
the present respondent, subscribed to purchased and paid for in full one share (Golf
Share) in the capital stock of Valley Golf. He was issued Stock Certificate No. 389
dated 26 January 1961 for the Golf Share. The Stock Certificate likewise indicates a
par value of P9,000.00.
Valley Golf would subsequently allege that beginning 25 January 1980, Caram
stopped paying his monthly dues, which were continually assessed until 31 June
1987. Valley Golf claims to have sent five (5) letters to Caram concerning his
delinquent account within the period from 27 January 1986 until 3 May 1987, all
forwarded to P.O. Box No. 1566, Makati Commercial Center Post Office, the mailing
address which Caram allegedly furnished Valley Golf.
The Golf Share was sold at public auction on 11 June 1987 for P25,000.00 after
the Board of Directors had authorized the sale in a meeting on 11 April 1987, and the
Notice of Auction Sale was published in the 6 June 1987 edition of the Philippine Daily
Inquirer.

ISSUE:

Whether or not a non-stock corporation seize and dispose of the membership


share of a fully-paid member on account of its unpaid debts to the corporation when it
is authorized to do so under the corporate by-laws but not by the Articles of
Incorporation.

RULING:

NO.

It may be conceded that the actions of Valley Golf were, technically speaking, in
accord with the provisions of its by-laws on termination of membership, vaguely
defined as these are. Yet especially since the termination of membership in Valley Golf
is inextricably linked to the deprivation of property rights over the Golf Share, the
emergence of such adverse consequences make legal and equitable standards come to
fore.
The commentaries of Lopez advert to an SEC Opinion dated 29 September 1987
which we can cite with approval. Lopez cites: In order that the action of a corporation
in expelling a member for cause may be valid, it is essential, in the absence of a
waiver, that there shall be a hearing or trial of the charge against him, with reasonable
notice to him and a fair opportunity to be heard in his defense. If the method of trial is
not regulated by the by-laws of the association, it should at least permit substantial
justice. The hearing must be conducted fairly and openly and the body of persons
before whom it is heard or who are to decide the case must be unprejudiced.
It is unmistakably wise public policy to require that the termination of
membership in a non-stock corporation be done in accordance with substantial
justice. No matter how one may precisely define such term, it is evident in this case
that the termination of Caram‘s membership betrayed the dictates of substantial
justice.

310 | P a g e
Law 321_Corporation LAW_ Case Digest

CALATAGAN GOLF CLUB, INC., petitioner


vs.
CLEMENTE Jr., respondent
G.R. No. 165443 April 16, 2009

FACTS:

Clemente applied to purchase one share of stock of Calatagan, indicating in his


application for membership his mailing address at "Phimco Industries, Inc. – P.O. Box
240, MCC," complete residential address, office and residence telephone numbers, as
well as the company (Phimco) with which he was connected, Calatagan issued to him
Certificate of Stock No. A-01295 on 2 May 1990 after paying P120,000.00 for the
share.
Calatagan charges monthly dues on its members to meet expenses for general
operations, as well as costs for upkeep and improvement of the grounds and facilities.
The provision on monthly dues is incorporated in Calatagan‘s Articles of Incorporation
and By-Laws. It is also reproduced at the back of each certificate of stock.
Calatagan declared Clemente delinquent for having failed to pay his monthly
dues for more than sixty (60) days, specifically P5,600.00 as of 31 October 1992.
Calatagan also included Clemente‘s name in the list of delinquent members posted on
the club‘s bulletin board. On 1 December 1992, Calatagan‘s board of directors adopted
a resolution authorizing the foreclosure of shares of delinquent members, including
Clemente‘s; and the public auction of these shares.

ISSUE:

Whether or not the action of Clemente had prescribed pursuant to Section 69 of


the Corporation Code, and that the requisite notices under both the law and the by-
laws had been rendered to Clemente.

RULING:

YES.

There are fundamental differences that defy equivalence or even analogy


between the sale of delinquent stock under Section 68 and the sale that occurred in
this case. At the root of the sale of delinquent stock is the non-payment of the
subscription price for the share of stock itself. The stockholder or subscriber has yet
to fully pay for the value of the share or shares subscribed. In this case, Clemente had
already fully paid for the share in Calatagan and no longer had any outstanding
obligation to deprive him of full title to his share. Perhaps the analogy could have been
made if Clemente had not yet fully paid for his share and the non-stock corporation,
pursuant to an article or by-law provision designed to address that situation, decided
to sell such share as a consequence. But that is not the case here, and there is no
purpose for us to apply Section 69 to the case at bar.
It is plain that Calatagan had endeavored to install a clear and comprehensive
procedure to govern the payment of monthly dues, the declaration of a member as
delinquent, and the constitution of a lien on the shares and its eventual public sale to
answer for the member‘s debts. Under Section 91 of the Corporation Code,
membership in a non-stock corporation "shall be terminated in the manner and for
the causes provided in the articles of incorporation or the by-laws." The By-law
provisions are elaborate in explaining the manner and the causes for the termination
of membership in Calatagan, through the execution on the lien of the share. The Court
is satisfied that the By-Laws, as written, affords due protection to the member by
assuring that the member should be notified by the Secretary of the looming execution
sale that would terminate membership in the club. In addition, the By-Laws
guarantees that after the execution sale, the proceeds of the sale would be returned to
the former member after deducting the outstanding obligations. If followed to the
letter, the termination of membership under this procedure outlined in the By-Laws
would accord with substantial justice.

311 | P a g e
Law 321_Corporation LAW_ Case Digest

Issuance of Certificates of Stock

FUA CUN, petitioner


vs.
SUMMERS, respondent
G.R. No. L-19441 March 27, 1923

FACTS:

It appears from the evidence that on August 26, 1920, one Chua Soco
subscribed for five hundred shares of stock of the defendant Banking Corporation.
On May 18, 1921, Chua Soco executed a promissory note in favor of the
plaintiff Fua Cun for the sum of P25,000 payable in ninety days and drawing interest
at the rate of 1 per cent per month, securing the note with a chattel mortgage on the
shares of stock subscribed for by Chua Soco, who also endorsed the receipt above
mentioned and delivered it to the mortgagee. The plaintiff thereupon took the receipt
to the manager of the defendant Bank and informed him of the transaction with Chua
Soco, but was told to await action upon the matter by the Board of Directors.
In the meantime Chua Soco appears to have become indebted to the China
Banking Corporation in the sum of P37,731.68 for dishonored acceptances of
commercial paper and in an action brought against him to recover this amount, Chua
Soco's interest in the five hundred shares subscribed for was attached and the receipt
seized by the sheriff. The attachment was levied after the defendant bank had received
notice of the facts that the receipt had been endorsed over to the plaintiff. Fua Cun
thereupon brought the present action maintaining that by virtue of the payment of the
one-half of the subscription price of five hundred shares Chua Soco in effect became
the owner of two hundred and fifty shares and praying that his, the plaintiff's, lien on
said shares, by virtue of the chattel mortgage, be declared to hold priority over the
claim of the defendant Banking Corporation; that the defendants be ordered to deliver
the receipt in question to him; and that he be awarded the sum of P5,000 in damages
for wrongful attachment.

ISSUE:

Whether or not the interest held by Chua Soco was merely an equity which
could not be made the subject of a chattel mortgage.

RULING:

NO.

Though the courts have uniformly held that chattel mortgages on shares of
stock and other choses in action are valid as between the parties, there is still much to
be said in favor of the defendants' contention that the chattel mortgage here in
question would not prevail over liens of third parties without notice; an equity in
shares of stock is of such an intangible character that it is somewhat difficult to see
how it can be treated as a chattel and mortgaged in such a manner that the recording
of the mortgage will furnish constructive notice to third parties.
In regard to a chattel mortgage of shares of stock: These certificates of stock are
in the pockets of the owner, and go with him where he may happen to locate, as choses
in action, or evidence of his right, without any means on the part of those with whom he
proposes to deal on the faith of such a security of ascertaining whether or not this stock
is in pledge or mortgaged to others. The chief office of the company may be at one place
to-day and at another tomorrow. The owner may have no fixed or permanent abode, and
with his notes in one pocket and his certificates of stock in the other.
But a determination of this question is not essential in the present case. There
can be no doubt that an equity in shares of stock may be assigned and that the
assignment is valid as between the parties and as to persons to whom notice is
brought home. Such an assignment exists here, though it was made for the purpose of
securing a debt.

312 | P a g e
Law 321_Corporation LAW_ Case Digest

BALTAZAR, plaintiff
vs.
LINGAYEN GULF ELECTRIC POWER CO., INC., respondent
G.R. No. L-16236 June 30, 1965

FACTS:

The Lingayen Gulf Electric Power Co., Inc., hereinafter referred to as


Corporation, was doing business in the Philippines, with principal offices at Lingayen,
Pangasinan, and with an authorized capital stock of P300.000.00 divided into 3,000
shares of voting stock at P100.00 par value, per share. Plaintiffs Baltazar and Rose
were among the incorporators, having subscribed to 600 and 400 shares of the capital
stock, or a total par value of P60,000.00 and P40.000.00, respectively. It is alleged
that it has always been the practice and procedure of the Corporation to issue
certificates of stock to its individual subscribers for unpaid shares of stock. Of the 600
shares of capital stock subscribed by Baltazar, he had fully paid 535 shares of stock,
and the Corporation issued to him several fully paid up and non-assessable
certificates of stock, corresponding to the 535 shares. After having made transfers to
third persons and acquired new ones, Baltazar had to his credit, on the filing of the
complaint 341 shares fully paid and non-assessable.
The respondents Ungson, Estrada, Fernandez and Yuson were small
stockholders of the Corporation, all holding a total number of fully paid-up shares of
stock, of not more than 100 shares, with a par value of P10,000.00 and the defendant
Acena, was likewise an incorporator and stockholder, holding 600 shares of stock, for
which certificate of stock were issued to him and as such, was the largest individual
stockholder thereof. Defendants Ungson, Estrada, Fernandez and Yuzon, constituted
the majority of the holdover seven-member Board of Directors of the Corporation, in
1955, two (2) of said defendants having been elected as members of the Board in the
annual stockholders' meeting held in May 1954, largely on the vote of their co-
defendant Acena, while the other two (2) were elected mainly on the vote of the
plaintiffs and their group of stockholders. Let the first group be called theUngson
group and the second, the Baltazar group.

ISSUE:

Whether or not a stockholder, in a stock corporation, subscribes to a certain


number of shares of stock, and he pays only partially, for which he is issued
certificates of stock, is he entitled to vote the latter, notwithstanding the fact that he
has not paid the balance of his subscription, which has been called for payment or
declared delinquent.

RULING:

YES.

The cases at bar do not come under the aegis of the principle enunciated in
the Fua Cun v. Summers case, because it was the practice and procedure, since the
inception of the corporation, to issue certificates of stock to its individual subscribers
for unpaid shares of stock and gave voting power to shares of stock fully paid. And
even though no agreement existed, the ruling in said case does not now reflect the
correct view on the matter, for better than an agreement or practice, there is the law,
which renders the said case of Fua Cun-Summers, obsolescent.
In the cases at bar, the defendant-corporation had chosen to apply payments by
its stockholders to definite shares of the capital stock of the corporation and had fully
paid capital stock shares certificates for said payments; its call for payment of unpaid
subscription and its declaration of delinquency for non-payment of said call affecting
only the remaining number of shares of its capital stock for which no fully paid capital
stock shares certificates have been issued, "and only these have been legally shorn of
their voting rights by said declaration of delinquency" (amended decision).

313 | P a g e
Law 321_Corporation LAW_ Case Digest

TAN, petitioner
vs.
SECURITIES AND EXCHANGE COMMISSION, respondent
G.R. No. 95696 March 3, 1992

FACTS:

Respondent corporation was registered on October 1, 1979. As incorporator,


petitioner had four hundred (400) shares of the capital stock standing in his name at
the par value of P100.00 per share, evidenced by Certificate of Stock No. 2. He was
elected as President and subsequently reelected, holding the position as such until
1982 but remained in the Board of Directors until April 19, 1983 as director.
On January 31, 1981, while petitioner was still the president of the respondent
corporation, two other incorporators, namely, Antonia Y. Young and Teresita Y. Ong,
assigned to the corporation their shares, represented by certificate of stock No. 4 and
5 after which, they were paid the corresponding 40% corporate stock-in-trade.
Petitioner's certificate of stock No. 2 was cancelled by the corporate secretary
and respondent Patricia Aguilar by virtue of Resolution No. 1981 (b), which was
passed and approved while petitioner was still a member of the Board of Directors of
the respondent corporation.
Due to the withdrawal of the aforesaid incorporators and in order to complete
the membership of the five (5) directors of the board, petitioner sold fifty (50) shares
out of his 400 shares of capital stock to his brother Angel S. Tan. Another
incorporator, Alfredo B. Uy, also sold fifty (50) of his 400 shares of capital stock to
Teodora S. Tan and both new stockholders attended the special meeting, Angel Tan
was elected director and on March 27, 1981, the minutes of said meeting was filed
with the SEC. These facts stand unchallenged.

ISSUE:

Whether or not the cancellation and transfer of petitioner's shares and


Certificate of Stock No. 2 as well as the issuance and cancellation of Certificate of
Stock No. 8 was patently and palpably unlawful, null and void, invalid and fraudulent.

RULING:

YES.

Under the instant case, the fact of the matter is, the new holder, Angel S. Tan
has already exercised his rights and prerogatives as stockholder and was even elected
as member of the board of directors in the respondent corporation with the full
knowledge and acquiescence of petitioner. Due to the transfer of fifty (50) shares,
Angel S. Tan was clothed with rights and responsibilities in the board of the
respondent corporation when he was elected as officer thereof.
Besides, in Philippine jurisprudence, a certificate of stock is not a negotiable
instrument. "Although it is sometime regarded as quasi-negotiable, in the sense that it
may be transferred by endorsement, coupled with delivery, it is well-settled that it is
non-negotiable, because the holder thereof takes it without prejudice to such rights or
defenses as the registered owner/s or transferor's creditor may have under the law,
except insofar as such rights or defenses are subject to the limitations imposed by the
principles governing estoppel."
To follow the argument put up by petitioner which was upheld by the Cebu SEC
Extension Office Hearing Officer, Felix Chan, that the cancellation of Stock Certificate
Nos. 2 and 8 was null and void for lack of delivery of the cancelled "mother" Certificate
No. 2 whose endorsement was deliberately withheld by petitioner, is to prescribe
certain restrictions on the transfer of stock in violation of the corporation law itself as
the only law governing transfer of stocks. While Section 47(s) grants stock
corporations the authority to determine in the by-laws "the manner of issuing
certificates" of shares of stock, however, the power to regulate is not the power to
prohibit, or to impose unreasonable restrictions of the right of stockholders to transfer
their shares.

314 | P a g e
Law 321_Corporation LAW_ Case Digest

EMBASSY FARMS, INC., petitioner


vs.
COURT OF APPEALS, respondent
G.R. No. 80682 August 13, 1990

FACTS:

It appears on record that sometime on August 2, 1984, Alexander G. Asuncion


(AGA for short) and Eduardo B. Evangelists (EBE for short) entered into a
Memorandum of Agreement (Annex "A" of the petition). Under said agreement EBE
obligated himself to transfer to AGA 19 parcels of agricultural land registered in his
name with an aggregate area of 104,447 square meters located in Loma de Gato,
Marilao, Bulacan, together with the stocks, equipment and facilities of a piggery farm
owned by Embassy Farms, Inc., a registered corporation wherein ninety (90) per cent
of its shares of stock is owned by EBE. EBE also obligated himself to cede, transfer
and convey "in a manner absolute and irrevocable any and all of his shares of stocks"
in Embassy Farins Inc. to AGA or his nominees "until the total of said shares of stock
so transferred shall constitute 90% of the paid-in-equity of said corporation" within a
reasonable time from signing of the document. Likewise, EBE obligated to turnover to
AGA the effective control and management of the piggery upon the signing of the
agreement.
On the other hand, AGA obligated himself, upon signing of the agreement to
pay to EBE the total sum of close to P8,630,000.00. Within reasonable time from
signing of the agreement AGA obligated himself to organize and register a new
corporation with an authorized capital stock of P10,000,000.00 which upon
registration will take over all the rights and liabilities of AGA.

ISSUE:

Whether or not there has been an effective transfer of shares of stock from AGA
to other persons.

RULING:

NO.

There being no delivery of the indorsed shares of stock AGA cannot therefore
effectively transfer to other person or his nominees the undelivered shares of stock.
For an effective transfer of shares of stock the mode and manner of transfer as
prescribed by law must be followed (Navea v. Peers Marketing Corp., 74 SCRA 65). As
provided under Section 3 of Batas Pambansa Bilang 68, otherwise known as the
Corporation Code of the Philippines, shares of stock may be transferred by delivery to
the transferree of the certificate properly indorsed. Title may be vested in the
transferree by the delivery of the duly indorsed certificate of stock (18 C.J.S. 928, cited
in Rivera v. Florendo, 144 SCRA 643). However, no transfer shall be valid, except as
between the parties until the transfer is properly recorded in the books of the
corporation.
In the case at bar the indorsed certificate of stock was not actually delivered to
AGA so that EBE is still the controlling stockholder of Embassy Farms despite the
execution of the memorandum of agreement and the turn over of control and
management of the Embassy Farms to AGA on August 2, 1984.
When AGA filed on April 10, 1986 an action for the rescission of contracts with
damages the Pasig Court merely restored and established the status quo prior to the
execution of the memorandum of agreement by the issuance of a restraining order on
July 10, 1987 and the writ of preliminary injunction on July 30, 1987. It would be
unjust and unfair to allow AGA and his nominees to control and manage the Embassy
Farms despite the fact that AGA who is the source of their supposed shares of stock in
the corporation is not asking for the delivery of the indorsed certificate of stock but for
the rescission of the memorandum of agreement. Rescission would result in mutual
restitution (Magdalena Estate v. Myrick, 71 Phil. 344) so it is but proper to allow EBE
to manage the farm.

315 | P a g e
Law 321_Corporation LAW_ Case Digest

Right to Transfer of Shares/Validity of Restrictions on Right

Makati Sports Club Inc


vs.
Cecile Cheng
G.R. No. 178523 June 16, 2010

FACTS:

October 20, 1994: Makati Sports Club Inc (MSCI) BOD adopted a
resolution authorizing the sale of 19 unissued shares at a floor price of P400,000
and P450,000 per share for Class A and B, respectively. Cheng was a Treasurer and
Director of Makati Sports Club in 1995
On July 7, 1995, Hodreal expressed his interest to buy a share, for this purpose
he sent the letter requesting to be wait listed. On November 1995, McFoods acquiried
shares of Makati Sports Club at P1,800,000 through Urban Bank. Thereafter,
Cheng advised sale by McFoods to Hodreal of the share evidenced by a certificate
new certificate was issued. Investigation showed that Cheng profited from the
transaction because of her knowledge MSCI sought judgment that would order
respondents to pay the sum of P1,000,000.00, representing the amount allegedly
defrauded, together with interest and damages.

ISSUE:

Whether or not MSCI was defrauded by Cheng's collaboration with Mc Foods.

RULING:

NO.

No evidence on record that the Membership Committee acted


on Hodreal's letter. SEC. 29. (a) The Membership Committee shall process applications
for membership; ascertain that the requirements for stock ownership, including
citizenship, are complied with; submit to the Board its recommended on applicants for
inclusion in the Waiting List; take charge of auction sales of shares of stock; and
exercise such other powers and perform such other functions as may be authorized by
the Board. Membership Committee failed to question the alleged irregularities
attending Mc Foods‘ purchase price of P1,800,000.00 is P1,400,000.00 more than the
floor price – it is not detrimental.
Upon payment and the execution of the Deed of Absolute Sale, it had the right
to demand the delivery of the stock certificate in its name. The right of a transferee to
have stocks transferred to its name is an inherent right flowing from its ownership of
the stocks certificate of stock paper representative or tangible evidence of the stock
itself and of the various interests therein not a stock in the corporation but is merely
evidence of the holder‘s interest and status in the corporation, his ownership of the
share represented thereby MSCI failed to repurchase Mc Foods‘ Class "A" share within
the 30 day pre-emptive period and no proof that Cheng personally profited.

316 | P a g e
Law 321_Corporation LAW_ Case Digest

FLEISCHER, plaintiff
vs.
BOTICA NOLASCO CO., INC., defendant
G.R. No. L-23241 March 14, 1925

FACTS:

Manuel Gonzalez was the original owner of the five shares of stock in question,
Nos. 16, 17, 18, 19 and 20 of the Botica Nolasco, Inc.; that on March 11, 1923, he
assigned and delivered said five shares to the plaintiff, Henry Fleischer, by
accomplishing the form of endorsement provided on the back thereof, together with
other credits, in consideration of a large sum of money owed by Gonzalez to Fleischer;
that on March 13, 1923, Dr. Eduardo Miciano, who was the secretary-treasurer of said
corporation, offered to buy from Henry Fleischer, on behalf of the corporation, said
shares of stock, at their par value of P100 a share, for P500; that by virtue of article
12 of the by-laws of Botica Nolasco, Inc., said corporation had the preferential right to
buy from Manuel Gonzalez said shares; that the plaintiff refused to sell them to the
defendant; that the plaintiff requested Doctor Miciano to register said shares in his
name; that Doctor Miciano refused to do so, saying that it would be in contravention of
the by-laws of the corporation.
It also appears from the record that on the 13th day of March, 1923, two days
after the assignment of the shares to the plaintiff, Manuel Gonzales made a written
statement to the Botica Nolasco, Inc., requesting that the five shares of stock sold by
him to Henry Fleischer be noted transferred to Fleischer's name. He also
acknowledged in said written statement the preferential right of the corporation to buy
said five shares.

ISSUE:

Whether or not article 12 of the by-laws of the Botica Nolasco, Inc., is in conflict
with the provisions of the Corporation Law (Act No. 1459).

RULING:

NO.

It does not suggest that any discrimination may be created by the corporation
in favor or against a certain purchaser. The holder of shares, as owner of personal
property, is at liberty, under said section, to dispose of them in favor of whomsoever
he pleases, without any other limitation in this respect, than the general provisions of
law. Therefore, a stock corporation in adopting a by-law governing transfer of shares of
stock should take into consideration the specific provisions of section 35 of Act No.
1459, and said by-law should be made to harmonize with said provisions. It should
not be inconsistent therewith.
The only restraint imposed by the Corporation Law upon transfer of shares is
found in section 35 of Act No. 1459, quoted above, as follows: "No transfer, however,
shall be valid, except as between the parties, until the transfer is entered and noted
upon the books of the corporation so as to show the names of the parties to the
transaction, the date of the transfer, the number of the certificate, and the number of
shares transferred." This restriction is necessary in order that the officers of the
corporation may know who are the stockholders, which is essential in conducting
elections of officers, in calling meeting of stockholders, and for other purposes. but
any restriction of the nature of that imposed in the by-law now in question, is ultra
vires, violative of the property rights of shareholders, and in restraint of trade.
And moreover, the by-laws now in question cannot have any effect on the
appellee. He had no knowledge of such by-law when the shares were assigned to him.
He obtained them in good faith and for a valuable consideration. He was not a privy to
the contract created by said by-law between the shareholder Manuel Gonzalez and the
Botica Nolasco, Inc. Said by-law cannot operate to defeat his rights as a purchaser.

317 | P a g e
Law 321_Corporation LAW_ Case Digest

CYRUS PADGETT
vs.
BABCOCK & TEMPLETON, INC., and W. R. BABCOCK
G.R. No. L-38684, December 21, 1933

FACTS:

The appellee was an employee of the Appellant Corporation and rendered


services as such from January 1, 1923, to April 15, 1929. During that period he
bought 35 shares thereof at P100 a share at the suggestion of the president of said
corporation. He was also the recipient of 9 shares by way of bonus during Christmas
seasons. In this way the said appellee became the owner of 44 shares for which the 12
certificates, Exhibits F to F-11, were issued in his favor. The word "nontransferable"
appears on each and every one of these certificates. Before severing his connections
with the said corporation, the appellee proposed to the president that the said
corporation buy his 44 shares at par value plus the interest thereon, or that he be
authorized to sell them to other persons. The corporation bought similar shares
belonging to other employees, at par value. Sometime later, the said president offered
to buy the appellee's shares first at P85 each and then at P80. The appellee did not
agree thereto.

ISSUE:

Whether or not the defendant obliged to buy his shares of stock at par value.

RULING:

NO.

A restriction imposed upon a certificate of shares, similar to the ones under


consideration, is null and void on the ground that it constitutes and unreasonable
limitation of the right of ownership and is in restraint of trade.
Shares of corporate stock being regarded as property, the owner of such shares
may, as a general rule, dispose of them as he sees fit, unless the corporation has been
dissolved, or unless the right to do so is properly restricted, or the owner's privilege of
disposing of his shares has been hampered by his own action.
Any restriction on a stockholder's right to dispose of his shares must be
construed strictly; and any attempt to restrain a transfer of shares is regarded as
being in restraint of trade, in the absence of a valid lien upon its shares, and except to
the extent that valid restrictive regulations and agreements exist and are applicable.
Subject only to such restrictions, a stockholder cannot be controlled in or restrained
from exercising his right to transfer by the corporation or its officers or by other
stockholders, even though the sale is to a competitor of the company, or to an
insolvent person, or even though a controlling interest is sold to one purchaser.

318 | P a g e
Law 321_Corporation LAW_ Case Digest

RURAL BANK OF SALINAS, INC., MANUEL SALUD, LUZVIMINDA TRIAS and


FRANCISCO TRIAS
vs.
COURT OF APPEALS, SECURITIES AND EXCHANGE COMMISSION, MELANIA A.
GUERRERO, LUZ ANDICO, WILHEMINA G. ROSALES, FRANCISCO M. GUERRERO,
JR., and FRANCISCO GUERRERO , SR.
G.R. No. 96674, June 26, 1992

FACTS:

Clemente G. Guerrero, President of the Rural Bank of Salinas, Inc., executed


a Special Power of Attorney in favor of his wife, private respondent Melania Guerrero,
giving and granting the latter full power and authority to sell or otherwise dispose of
and/or mortgage 473 shares of stock of the Bank registered in his name (represented
by the Bank's stock certificates nos. 26, 49 and 65), to execute the proper documents
therefor, and to receive and sign receipts for the dispositions. On February 27, 1980,
and pursuant to said Special Power of Attorney, private respondent Melania Guerrero,
as Attorney-in-Fact, executed a Deed of Assignment for 472 shares out of the 473
shares, in favor of private respondents Luz Andico (457 shares), Wilhelmina Rosales
(10 shares) and Francisco Guerrero, Jr. (5 shares).Almost four months later, or two (2)
days before the death of Clemente Guerrero on June 24, 1980, private respondent
Melania Guerrero, pursuant to the same Special Power of Attorney, executed a Deed of
Assignmentfor the remaining one (1) share of stock in favor of private respondent
Francisco Guerrero, Sr.
Subsequently, private respondent Melania Guerrero presented to petitioner
Rural Bank of Salinas the two (2) Deeds of Assignment for registration with a request
for the transfer in the Bank's stock and transfer book of the 473 shares of stock so
assigned, the cancellation of stock certificates in the name of Clemente G. Guerrero,
and the issuance of new stock certificates covering the transferred shares of stocks in
the name of the new owners thereof. However, petitioner Bank denied the request of
respondent Melania Guerrero.

ISSUE:

Whether or not a Mandamus lie against the Rural Bank of Salinas to register in
its stock and transfer book the transfer of 473 shares of stock to private respondents.

RULING:

YES.

Section 5 (b) of P.D. No. 902-A grants to the SEC the original and exclusive
jurisdiction to hear and decide cases involving intracorporate controversies. An intra-
corporate controversy has been defined as one which arises between a stockholder and
the corporation. There is neither distinction, qualification, nor any exception
whatsoever. The case at bar involves shares of stock, their registration, cancellation
and issuances thereof by petitioner Rural Bank of Salinas. It is therefore within the
power of respondent SEC to adjudicate.
A corporation, either by its board, its by-laws, or the act of its officers, cannot
create restrictions in stock transfers, because: Restrictions in the traffic of stock must
have their source in legislative enactment, as the corporation itself cannot create such
impediment. By-laws are intended merely for the protection of the corporation, and
prescribe regulation, not restriction; they are always subject to the charter of the
corporation. The corporation, in the absence of such power, cannot ordinarily inquire
into or pass upon the legality of the transactions by which its stock passes from one
person to another, nor can it question the consideration upon which a sale is based.
Whenever a corporation refuses to transfer and register stock in cases like the
present, mandamus will lie to compel the officers of the corporation to transfer said
stock in the books of the corporation.

319 | P a g e
Law 321_Corporation LAW_ Case Digest

MARSH THOMSON
vs.
COURT OF APPEALS and THE AMERICAN CHAMPER OF COMMERCE OF THE
PHILIPPINES, INC.
G.R. No. 116631, October 28, 1998

FACTS:

A. Lewis Burridge, retired as AmCham's President while petitioner was still


working with private respondent, his superior,. Before Burridge decided to return to
his home country, he wanted to transfer his proprietary share in the Manila Polo Club
(MPC) to petitioner. However, through the intercession of Burridge, private respondent
paid for the share but had it listed in petitioner's name. This was made clear in an
employment advice dated January 13, 1986, wherein petitioner was informed by
private respondent.
Burridge transferred said proprietary share to petitioner, as confirmed in a
letter of notification to the Manila Polo Club. Upon his admission as a new member of
the MPC, petitioner paid the transfer fee of P40,000.00 from his own funds; but
private respondent subsequently reimbursed this amount.
MPC issued Proprietary Membership Certificate Number 3398 in favor of
petitioner. But petitioner, however, failed to execute a document recognizing private
respondent's beneficial ownership over said share.
When petitioner's contract of employment was up for renewal in 1989, he
notified private respondent that he would no longer be available as Executive Vice
President after September 30, 1989. Still, the private respondent asked the petitioner
to stay on for another six (6) months.

ISSUE:

Whether or not private respondent the beneficial owner of the disputed share.

RULING:

YES.

In the present case, as the Executive Vice-President of AMCHAM, petitioner


occupied a fiduciary position in the business of AMCHAM. It released the funds to
acquire a share in the Club for the use of petitioner but obliged him to "execute such
document as necessary to acknowledge beneficial ownership thereof by the
Chamber". A trust relationship is, therefore, manifestly indicated.
The beneficiary of a trust has beneficial interest in the trust property, while a
creditor has merely a personal claim against the debtor. In trust, there is a fiduciary
relation between a trustee and a beneficiary, but there is no such relation between a
debtor and creditor. While a debt implies merely an obligation to pay a certain sum of
money, a trust refers to a duty to deal with a specific property for the benefit of
another. If a creditor-debtor relationship exists, but not a fiduciary relationship
between the parties, there is no express trust. However, it is understood that when the
purported trustee of funds is entitled to use them as his or her own (and commingle
them with his or her own money), a debtor-creditor relationship exists, not a trust.
Moreover, petitioner failed to present evidence to support his allegation of being
merely a debtor when the private respondent paid the purchase price of the MPC
share. Applicable here is the rule that a trust arises in favor of one who pays the
purchase money of property in the name of another, because of the presumption that
he who pays for a thing intends a beneficial interest therein for himself.

320 | P a g e
Law 321_Corporation LAW_ Case Digest

ENRIQUE T. YUCHENGCO, INC., A. T. YUCHENGCO, INC., ANNABELLE Y. PUEY


and MONA LISA Y. ABAYA
vs.
CONRADO M. VELAYO
G.R. No. L-50439, July 20, 1982

FACTS:

Conrado M. Velayo offered to sell to the plaintiffs-appellees 2,265 shares of


common stock of the RIC Tours Philippines, Inc. ("Ric Tours Phil., for short) a
Philippine Corporation then duly licensed as a tourist operator, constituting 70% of
the subscribed and outstanding capital stock of the said corporation. Appellees paid
the entire purchase price of P367,500.00 to appellant Velayo, and the latter, on his
part, delivered to the former all the 2,265 shares of stock of Ric Tours Phil.
Appellant claims that the shares of stock of Ric Tours Phil. were sold to another
group without previous clearance from the Department of Tourism because he really
was not aware of the rule requiring prior approval by the Department of Tourism for
the validity of transfers of shares of local tour operators.
On September 3, 1974, appellees wrote a letter to appellant demanding
rescission of the contract, the restitution of a sum of money.

ISSUE:

Whether or not the "Stock Purchase Agreement" entered into by the appellees
and appellant Velayo annulled, or in the alternative, declared void ab initio.

RULING:

NO.

The provision governing the Agreement sought to be annuled is Sec 4, Part IV of


the Rules and Regulations Governing the Business of Tour Operators and Tour
Guides, which recites as follows: Sec. 4. No transfer of rights to a license of a tour
operator or ownership of shares or interests in the agency shall be valid unless made
with the prior approval of the Department.
The above-quoted rule is clear and mandatory. It requires the prior approval of
the Department of Tourism for the validity of any transfer of rights to a license of a
tour operator or ownership of shares or interests in any tour agency. In the case at
bar, it was admitted by both parties, that the Stock Purchase Agreement was made
without the prior approval of the Department of Tourism. Pursuant to paragraph 7,
article 1409 of the New Civil Code, such agreement would be inexistent and null and
void from the beginning. For it is well-settled that any contract entered into must be in
accordance with, and not repugnant to, an applicable statute whose terms are deemed
embodied therein and without the need for the parties of expressly making reference
to it. Inasmuch as the agreement between the parties is null and void from the
beginning, it produces no legal effect. No valid transfer of ownership of Ric Tours Phil.,
to the appellees, therefore, took place upon delivery to them by the appellant of the
shares of stock of said corporation as to make them suffer the consequence of the
subsequent revocation by the Department of Tourism of the license of Ric Tours Phil.,
as they would indeed suffer much loss after parting with their money for which they
would receive nothing. The doctrine of res suo domino perit advanced by the defendant
cannot, therefore, be applied.

321 | P a g e
Law 321_Corporation LAW_ Case Digest

LIM TAY
vs.
COURT OF APPEALS, GO FAY AND CO. INC., SY GUIOK, and THE ESTATE OF
ALFONSO LIM
G.R. No. 126891, August 5, 1998

FACTS:

On January 8, 1980, Respondent-Appellee Sy Guiok secured a loan from the


petitioner in the amount of P40,000 payable within six (6) months. To secure the
payment of the aforesaid loan and interest thereon, Respondent Guiok executed a
Contract of Pledge in favor of the [p]etitioner whereby he pledged his three hundred
(300) shares of stock in the Go Fay & Company Inc., Respondent Corporation, for
brevity's sake. Respondent Guiok obliged himself to pay interest on said loan at the
rate of 10% per annum from the date of said contract of pledge. On the same date,
Alfonso Sy Lim secured a loan from the [p]etitioner in the amount of P40,000 payable
in six (6) months. To secure the payment of his loan, Sy Lim executed a "Contract of
Pledge" covering his three hundred (300) shares of stock in Respondent Corporation.
Under said contract, Sy Lim obliged himself to pay interest on his loan at the rate of
10% per annum from the date of the execution of said contract.
However, Respondent Guiok and Sy Lim failed to pay their respective loans and
the accrued interests thereon to the [p]etitioner. In October, 1990, the petitioner filed a
"Petition for Mandamus" against Respondent Corporation, with the SEC entitled "Lim
Tay versus Go Fay & Company. Inc., SEC Case No. 03894".

ISSUE:

Whether or not there is there dacion en pago.

RULING:

NO.

At the outset, it must be underscored that petitioner did not acquire ownership
of the shares by virtue of the contracts of pledge. Article 2112 of the Civil Code states:
The creditor to whom the credit has not been satisfied in due time, may proceed before
a Notary Public to the sale of the thing pledged. This sale shall be made at a public
auction and with notification to the debtor and the owner of the thing pledged in a
proper case, stating the amount for which the public sale is to be held. If at the first
auction the thing is not sold, a second one with the same formalities shall be held; and
if at the second auction there is no sale either, the creditor may appropriate the thing
pledged. In this case he shall be obliged to give an acquaintance for his entire claim.
There is no showing that petitioner made any attempt to foreclose or sell the
shares through public or private auction, as stipulated in the contracts of pledge and
as required by Article 2112 of the Civil Code. Therefore, ownership of the shares could
not have passed to him. The pledgor remains the owner during the pendency of the
pledge and prior to foreclosure and sale, as explicitly provided by Article 2103 of the
same Code: Unless the thing pledged is expropriated, the debtor continues to be the
owner thereof.
Neither did petitioner acquire the shares by virtue of a novation of the contract
of pledge. Novation is defined as "the extinguishment of an obligation by a subsequent
one which terminates it, either by changing its object or principal conditions, by
substituting a new debtor in place of the old one, or by subrogating a third person to
the rights of the creditor." Novation of a contract must not be presumed. "In the
absence of an express agreement, novation takes place only when the old and the new
obligations are incompatible on every point.

322 | P a g e
Law 321_Corporation LAW_ Case Digest

Transfer of Shares of Stock and Registration

JOSELITO MUSNI PUNO (as heir of the late Carlos Puno)


vs.
PUNO ENTERPRISES, INC., represented by JESUSA PUNO
G.R. No. 177066 September 11, 2009

FACTS:

Carlos L. Puno, who died on June 25, 1963, was an incorporator of respondent
Puno Enterprises, Inc. On March 14, 2003, petitioner Joselito Musni Puno, claiming to
be an heir of Carlos L. Puno, initiated a complaint for specific performance against
respondent. Petitioner averred that he is the son of the deceased with the latter‘s
common-law wife, Amelia Puno. As surviving heir, he claimed entitlement to the rights
and privileges of his late father as stockholder of respondent. The complaint thus
prayed that respondent allow petitioner to inspect its corporate book, render an
accounting of all the transactions it entered into from 1962, and give petitioner all the
profits, earnings, dividends, or income pertaining to the shares of Carlos L. Puno.
Respondent filed a motion to dismiss on the ground that petitioner did not have
the legal personality to sue because his birth certificate names him as "Joselito Musni
Muno." Apropos, there was yet a need for a judicial declaration that "Joselito Musni
Puno" and "Joselito Musni Muno" were one and the same.

ISSUE:

Whether or not there was an automatic transfer of shares of stovk.

RULING:

NO.

Upon the death of a shareholder, the heirs do not automatically become


stockholders of the corporation and acquire the rights and privileges of the deceased
as shareholder of the corporation. The stocks must be distributed first to the heirs in
estate proceedings, and the transfer of the stocks must be recorded in the books of the
corporation. Section 63 of the Corporation Code provides that no transfer shall be
valid, except as between the parties, until the transfer is recorded in the books of the
corporation.16 During such interim period, the heirs stand as the equitable owners of
the stocks, the executor or administrator duly appointed by the court being vested
with the legal title to the stock.17 Until a settlement and division of the estate is
effected, the stocks of the decedent are held by the administrator or executor.
Consequently, during such time, it is the administrator or executor who is entitled to
exercise the rights of the deceased as stockholder.
Thus, even if petitioner presents sufficient evidence in this case to establish
that he is the son of Carlos L. Puno, he would still not be allowed to inspect
respondent‘s books and be entitled to receive dividends from respondent, absent any
showing in its transfer book that some of the shares owned by Carlos L. Puno were
transferred to him. This would only be possible if petitioner has been recognized as an
heir and has participated in the settlement of the estate of the deceased.
Corollary to this is the doctrine that a determination of whether a person,
claiming proprietary rights over the estate of a deceased person, is an heir of the
deceased must be ventilated in a special proceeding instituted precisely for the
purpose of settling the estate of the latter. The status of an illegitimate child who
claims to be an heir to a decedent‘s estate cannot be adjudicated in an ordinary civil
action, as in a case for the recovery of property. The doctrine applies to the instant
case, which is one for specific performance — to direct Respondent Corporation to
allow petitioner to exercise rights that pertain only to the deceased and his
representatives.

323 | P a g e
Law 321_Corporation LAW_ Case Digest

IMELDA O. COJUANGCO, PRIME HOLDINGS, INC., AND THE ESTATE OF RAMON


U. COJUANGCO
vs.
SANDIGANBAYAN, REPUBLIC OF THE PHILIPPINES, AND THE SHERIFF OF
SANDIGANBAYAN
G.R. No. 183278 April 24, 2009

FACTS:

The Republic of the Philippines (Republic) filed before the Sandiganbayan a


"Complaint for Reconveyance, Reversion, Accounting, Restitution and Damages,"
docketed as Civil Case 0002, praying for the recovery of alleged ill-gotten wealth from
the late President Marcos and former First Lady Imelda Marcos and their cronies,
including some 2.4 million shares of stock in the Philippine Long Distance Telephone
Company (PLDT).
The complaint, which was later amended to implead herein petitioners Ramon
and Imelda Cojuangco (the Cojuangcos), alleged that the Marcoses‘ ill-gotten wealth
included shares in the PLDT covered by shares of stock in the Philippine
Telecommunications Investment Corporation (PTIC), registered in the name of Prime
Holdings, Inc. (Prime Holdings).

ISSUE:

Whether or not the Republic entitled to the dividends, interests, and earnings
thereof.

RULING:

YES.

It would be absurd to award the shares to the Republic as their owner and not
include the dividends and interests accruing thereto. An owner who cannot exercise
the "juses" or attributes of ownership -- the right to possess, to use and enjoy, to
abuse or consume, to accessories, to dispose or alienate, to recover or vindicate, and
to the fruits - is a crippled owner.
This Court, in directing the reconveyance to the Republic of the 111,415 shares
of PLDT stock owned by PTIC in the name of Prime Holdings, declared the Republic
as the owner of said shares and, necessarily, the dividends and interests accruing
thereto.
Ownership is a relation in law by virtue of which a thing pertaining to one
person is completely subjected to his will in everything not prohibited by law or the
concurrence with the rights of another. Its traditional elements or attributes
include jus utendi or the right to receive from the thing what it produces.

324 | P a g e
Law 321_Corporation LAW_ Case Digest

REPUBLIC OF THE PHILIPPINES represented by the PRESIDENTIAL


COMMISSION ON GOOD GOVERNMENT (PCGG)
vs.
SANDIGANBAYAN (SECOND DIVISION) and ROBERTO S. BENEDICTO
G.R. No. 129406 March 6, 2006

FACTS:

The case is one of several suits involving ill-gotten or unexplained wealth that
petitioner Republic, through the PCGG, filed with the Sandiganbayan against private
respondent Roberto S. Benedicto and others pursuant to Executive Order (EO) No. 14,
series of 1986. Pursuant to its mandate under EO No. 1, series of 1986, the PCGG
issued writs placing under sequestration all business enterprises, entities and other
properties, real and personal, owned or registered in the name of private respondent
Benedicto, or of corporations in which he appeared to have controlling or majority
interest. Among the properties thus sequestered and taken over by PCGG fiscal agents
were the 227 shares in NOGCCI owned by private respondent Benedicto and registered
in his name or under the names of corporations he owned or controlled.Following the
sequestration process, PCGG representatives sat as members of the Board of Directors
of NOGCCI, which passed, sometime in October 1986, a resolution effecting a
corporate policy change. The change consisted of assessing a monthly membership
due of P150.00 for each NOGCCI share. Prior to this resolution, an investor
purchasing more than one NOGCCI share was exempt from paying monthly
membership due for the second and subsequent shares that he/she owned.
As sequestrator of the 227 shares of stock in question, PCGG did not pay the
corresponding monthly membership due thereon totalingP2,959,471.00. On account
thereof, the 227 sequestered shares were declared delinquent to be disposed of in an
auction sale.

ISSUE:

Whether or not the Sandiganbayan, Second Division, gravely abuse its


discretion in holding that the PCGG is at fault for not paying the membership dues on
the 227 sequestered NOGCCI shares of stock, a failing which eventually led to the
foreclosure sale thereof.

RULING:

NO.

The PCGG‘s posture that to the owner of the sequestered shares rests the
burden of paying the membership dues is untenable. For one, it lost sight of the reality
that such dues are basically obligations attached to the shares, which, in the final
analysis, shall be made liable, thru delinquency sale in case of default in payment of
the dues. For another, the PCGG as sequestrator-receiver of such shares is, as
stressed earlier, duty bound to preserve the value of such shares. Needless to state,
adopting timely measures to obviate the loss of those shares forms part of such duty
and due diligence.
Given the circumstances leading to the auction sale of the subject NOGCCI
shares, PCGG‘s lament about public respondent Sandiganbayan having erred or,
worse still, having gravely abused its discretion in its determination as to who is at
fault for the loss of the shares in question can hardly be given cogency.

325 | P a g e
Law 321_Corporation LAW_ Case Digest

THE RURAL BANK OF LIPA CITY, INC., THE OFFICERS AND DIRECTORS,
BERNARDO BAUTISTA, JAIME CUSTODIO, OCTAVIO KATIGBAK, FRANCISCO
CUSTODIO, and JUANITA BAUTISTA OF THE RURAL BANK OF LIPA CITY, INC.,
vs.
HONORABLE COURT OF APPEALS, HONORABLE COMMISSION EN BANC,
SECURITIES AND EXCHANGE COMMISSION, HONORABLE ENRIQUE L. FLORES,
JR., in his capacity as Hearing Officer, REYNALDO VILLANUEVA, SR, AVELINA
M. VILLANUEVA, CATALINO VILLANUEVA, ANDRES GONZALES, AURORA
LACERNA, CELSO LAYGO, EDGARDO REYES, ALEJANDRA TONOGAN
G.R. No. 124535 September 28, 2001

FACTS:

Private respondent Reynaldo Villanueva, Sr., a stockholder of the Rural Bank of


Lipa City, executed a Deed of Assignment, wherein he assigned his shares, as well as
those of eight (8) other shareholders under his control with a total of 10,467 shares, in
favor of the stockholders of the Bank represented by its directors Bernardo Bautista,
Jaime Custodio and Octavio Katigbak. Sometime thereafter, Reynaldo Villanueva, Sr.
and his wife, Avelina, executed an Agreement wherein they acknowledged their
indebtedness to the Bank in the amount of Four Million Pesos (P4,000,000.00), and
stipulated that said debt will be paid out of the proceeds of the sale of their real
property described in the Agreement. At a meeting of the Board of Directors of the
Bank on November 15, 1993, the Villanueva spouses assured the Board that their
debt would be paid on or before December 31 of that same year; otherwise, the Bank
would be entitled to liquidate their shareholdings, including those under their control.
When the Villanueva spouses failed to settle their obligation to the Bank on the
due date, the Board sent them a letter3 demanding: (1) the surrender of all the stock
certificates issued to them; and (2) the delivery of sufficient collateral to secure the
balance of their debt amounting to P3,346,898.54.

ISSUE:

Whether or not the transfer of title to such shares is ineffective until and unless
the duly indorsed certificate of stock is delivered to them notwithstanding the
execution of the deed of assignment in favor of the petitioners.

RULING:

NO.

The rule is that the delivery of the stock certificate duly endorsed by the owner
is the operative act of transfer of shares from the lawful owner to the transferee. Thus,
title may be vested in the transferee only by delivery of the duly indorsed certificate of
stock.
We have uniformly held that for a valid transfer of stocks, there must be strict
compliance with the mode of transfer prescribed by law. The requirements are: (a)
There must be delivery of the stock certificate: (b) The certificate must be endorsed by
the owner or his attorney-in-fact or other persons legally authorized to make the
transfer; and (c) To be valid against third parties, the transfer must be recorded in the
books of the corporation. As it is, compliance with any of these requisites has not been
clearly and sufficiently shown.
It may be argued that despite non-compliance with the requisite endorsement
and delivery, the assignment was valid between the parties, meaning the private
respondents as assignors and the petitioners as assignees. While the assignment may
be valid and binding on the petitioners and private respondents, it does not
necessarily make the transfer effective. Consequently, the petitioners, as mere
assignees, cannot enjoy the status of a stockholder, cannot vote nor be voted for, and
will not be entitled to dividends, insofar as the assigned shares are concerned.
Parenthetically, the private respondents cannot, as yet, be deprived of their rights as
stockholders, until and unless the issue of ownership and transfer of the shares in
question is resolved with finality.

326 | P a g e
Law 321_Corporation LAW_ Case Digest

BATANGAS LAGUNA TAYABAS BUS COMPANY, INC., DOLORES A. POTENCIANO,


MAX JOSEPH A. POTENCIANO, MERCEDELIN A. POTENCIANO, and DELFIN C.
YORRO
vs.
BENJAMIN M. BITANGA, RENATO L. LEVERIZA, LAUREANO A. SIY, JAMES A.
OLAYVAR, EDUARDO A. AZUCENA, MONINA GRACE S. LIM, and GEMMA M.
SANTOS
G.R. No. 137934 August 10, 2001

FACTS:

Dolores A. Potenciano, Max Joseph A. Potenciano, Mercedelin A. Potenciano,


Delfin C. Yorro, and Maya Industries, Inc., entered into a Sale and Purchase
Agreement, whereby they sold to BMB Property Holdings, Inc., represented by its
President, Benjamin Bitanga, their 21,071,114 shares of stock in BLTB. The said
shares represented 47.98% of the total outstanding capital stock of BLTB. The
contracting parties stipulated that the downpayment was conditioned upon receipt by
the buyer of certain documents upon signing of the Agreement, namely, the
Secretary's Certificate stating that the Board of Directors of Maya Industries, Inc.
authorized the sale of its shares in BLTB and the execution of the Agreement, and
designating Dolores A. Potenciano as its Attorney-in-Fact; the Special Power of
Attorney executed by each of the sellers in favor of Dolores A. Potenciano for purposes
of the Agreement; the undated written resignation letters of the Directors of BLTB,
except Henry John A. Potenciano, Michael A. Potericiano and Candido A. Potenciano);
a revocable proxy to vote the subject shares made by the sellers in favor of the buyer;
a Declaration of Trust made by the sellers in favor of the buyer acknowledging that the
subject shares shall be held in trust by the sellers for the buyer pending their transfer
to the latter's name; and the duly executed capital gains tax return forms covering the
sale, indicating no taxable gain on the same.

ISSUE:

Whether or not the Bitanga group vote or be voted upon.

RULING:

NO.

We are in full accord with the SEC En Banc on this matter. Indeed, until
registration is accomplished, the transfer, though valid between the parties, cannot be
effective as against the corporation. Thus, the unrecorded transferee, the Bitanga
group in this case, cannot vote nor be voted for. The purpose of registration, therefore,
is two-fold: to enable the transferee to exercise all the rights of a stockholder,
including the right to vote and to be voted for, and to inform the corporation of any
change in share ownership so that it can ascertain the persons entitled to the rights
and subject to the liabilities of a stockholder. Until challenged in a proper proceeding,
a stockholder of record has a right to participate in any meeting; his vote can be
properly counted to determine whether a stockholders' resolution was approved,
despite the claim of the alleged transferee. On the other hand, a person who has
purchased stock, and who desires to be recognized as a stockholder for the purpose of
voting, must secure such a standing by having the transfer recorded on the corporate
books. Until the transfer is registered, the transferee is not a stockholder but an
outsider.
The Court finds no error either in jurisdiction or judgment on the part of the
SEC En Banc, since its conclusions of law were anchored on established principles
and jurisprudence. The petition is denied.

327 | P a g e
Law 321_Corporation LAW_ Case Digest

SPOUSES JOSE ABEJO AND AURORA ABEJO, TELEC. TRONIC SYSTEMS, INC.
vs.
HON. RAFAEL DE LA CRUZ, JUDGE OF THE REGIONAL TRIAL COURT (NATIONAL
CAPITAL JUDICIAL REGION, BRANCH CLX-PASIG), SPOUSES AGAPITO BRAGA
AND VIRGINIA BRAGA, VIRGILIO BRAGA AND NORBERTO BRAGA
G.R. No. L-63558 May 19, 1987

FACTS:

This case involves a question of who, between the Regional Trial Court and the
Securities and Exchange Commission (SEC), has original and exclusive jurisdiction
over the dispute between the principal stockholders of the corporation Pocket Bell
Philippines, Inc. (Pocket Bell), a "tone and voice paging corporation," namely, the
spouses Jose Abejo and Aurora Abejo (hereinafter referred to as the Abejos) and the
purchaser, Telectronic Systems, Inc. (hereinafter referred to as Telectronics) of their
133,000 minority shareholdings (for P5 million) and of 63,000 shares registered in the
name of Virginia Braga and covered by five stock certificates endorsed in blank by her
(for P1,674,450.00), and the spouses Agapito Braga and Virginia Braga (hereinafter
referred to as the Bragas), erstwhile majority stockholders. With the said purchases,
Telectronics would become the majority stockholder, holding 56% of the outstanding
stock and voting power of the corporation Pocket Bell.
With the said purchases in 1982, Telectronics requested the corporate secretary
of the corporation, Norberto Braga, to register and transfer to its name, and those of
its nominees the total 196,000 Pocket Bell shares in the corporation's transfer book,
cancel the surrendered certificates of stock and issue the corresponding new
certificates of stock in its name and those of its nominees. Norberto Braga, the
corporate secretary and son of the Bragas, refused to register the aforesaid transfer of
shares in t e corporate oo s, asserting that the Bragas claim preemptive rights over the
133,000 Abejo shares and that Virginia Braga never transferred her 63,000 shares to
Telectronics but had lost the five stock certificates representing those shares. This
triggered off the series of intertwined actions between the protagonists, all centered on
the question of jurisdiction over the dispute, which were to culminate in the filing of
the two cases at bar.

ISSUE:

Whether or not the trial court have jurisdiction over aforesaid case.

RULING:

YES.

The very complaint of the Bragas for annulment of the sales and transfers as
filed by them in the regular court questions the validity of the transfer and
endorsement of the certificates of stock, claiming alleged pre-emptive rights in the case
of the Abejos' shares and alleged loss of thio certificates and lack of consent and
consideration in the case of Virginia Braga's shares. Such dispute c learly involve's
controversies "between and among stockholders, " as to the Abej os' right to sell and
dispose of their shares to Telectronics, the validity of the latter's acquisition of Virginia
Braga's shares, who between the Bragas and the Abejos' transferee should be
recognized as the controlling shareholders of the corporation, with the right to elect
the corporate officers and the management and control of its operations. Such a
dispute and case clearly fag within the original and exclusive jurisdiction of the SEC to
decide, under Section 5 of P.D. 902-A, above-quoted. The restraining order issued by
the Regional Trial Court restraining Telectronics agents and representatives from
enforcing their resolution constituting themselves as the new set of officers of Pocket
Bell and from assuming control of the corporation and discharging their functions
patently encroached upon the SEC's exclusive jurisdiction over such specialized
corporate controversies calling for its special competence.

328 | P a g e
Law 321_Corporation LAW_ Case Digest

BATONG BUHAY GOLD MINES, INC.


vs.
THE COURT OF APPEALS and INC. MINING CORPORATION
G.R. No. L-45048 January 7, 1987

FACTS:

The defendant Batong Buhay Gold Mines, Inc. issued Stock Certificate No.
16807 covering 62,495 shares with a par value of P0.01 per share to Francisco Aguac
who was then legally married to Paula G. Aguac, but the said spouses had lived
separately for more than fourteen (14) years prior to the said date.
Later, Francisco Aguac sold his shares covered by Stock Certificate No. 16807
for the sum of P9,374.70 in favor of the plaintiff, the said transaction being evidenced
by a deed of sale (Exhibit D). The said sale was made by Francisco Aguac without the
knowledge or consent of his wife Paula G. Aguac.
On the same date of the sale, Paula G. Aguac wrote a letter to the president of
defendant Batong Buhay Gold Mines, Inc. asking that the transfer of the shares sold
by her husband be withheld, inasmuch as the same constituted conjugal property and
her share of proceeds of the sale was not given to her (Exhibit 1).

ISSUE:

Whether or not the Court of Appeals may award damages by way of unrealized profits
despite the absence of supporting evidence, or merely on the basis of pure
assumption, speculation or conjecture; or can the respondent recover damages by way
of unrealized profits when it has not shown that it was damaged in any manner by the
act of petitioner.

RULING:

NO.

The stipulation of facts of the parties does not at all show that private
respondent intended to sell, or would sell or would have sold the stocks in question on
specified dates. While it is true that shares of stock may go up or down in value (as in
fact the concerned shares here really rose from fifteen (15) centavos to twenty three or
twenty four (23/24) centavos per share and then fell to about two (2) centavos per
share, still whatever profits could have been made are purely SPECULATIVE, for it was
difficult to predict with any decree of certainty the rise and fall in the value of the
shares. Thus this Court has ruled that speculative damages cannot be recovered.
It is easy to say now that had private respondent gained legal title to the shares,
it could have sold the same and reaped a profit of P5,624.95 but it could not do so
because of petitioner's refusal to transfer the stocks in the former's name at the time
demand was made, but then it is also true that human nature, being what it is,
private respondent's officials could also have refused to sell and instead wait for
expected further increases in value.

329 | P a g e
Law 321_Corporation LAW_ Case Digest

CHEMPHIL EXPORT AND IMPORT CORPORATION


vs.
THE HON. COURT OF APPEALS (Former Twelfth Division), PHILIPPINE
INVESTMENTS SYSTEMS ORGANIZATION (PISO), BANK OF THE PHILIPPINE
ISLANDS (BPI), PHILIPPINE COMMERCIAL INDUSTRIAL BANK (PCIB),
RIZALCOMMERCIAL BANKING CORPORATION (RCBC) and LAND BANK OF THE
PHILIPPINES (LBP)
G.R. No. 97217 April 10, 1992

FACTS

Dynetics, Inc. and Antonio M. Garcia filed a complaint for declaratory relief
and/or injunction against the PISO, BPI, LBP, PCIB and RCBC or the consortium with
the Regional Trial Court of Makati, Branch 45 (Civil Case No. 8527), seeking judicial
declaration, construction and interpretation of the validity of the surety agreement
that Dynetics and Garcia had entered into with the consortium and to perpetually
enjoin the latter from claiming, collecting and enforcing any purported obligations
which Dynetics and Garcia might have undertaken in said agreement.
The consortium filed their respective answers with counterclaims alleging that
the surety agreement in question was valid and binding and that Dynetics and Garcia
were liable under the terms of the said agreement. It likewise applied for the issuance
of a writ of preliminary attachment against Dynetics and Garcia.

ISSUE:

Whether or not the attachment of shares of stock, in order to bind third


persons, must be recorded in the stock and transfer book of the corporation.

RULING:

NO.

Section 7(d), Rule 57 of the Rules of Court was complied with by the consortium
(through the Sheriff of the trial court) when the notice of garnishment over the
Chemphil shares of Garcia was served on the president of Chemphil on July 19, 1985.
Indeed, to bind third persons, no law requires that an attachment of shares of stock
be recorded in the stock and transfer book of a corporation. The statement attributed
by the Regional Trial Court to the Supreme Court in Samahang Magsasaka, Inc. vs.
Gonzalo Chua Guan, G.R. No. L-7252, February 25, 1955 (unreported), to the effect
that "as between two attaching creditors, the one whose claim was registered first on
the books of the corporation enjoys priority," is an obiter dictum that does not modify
the procedure laid down in Section 7(d), Rule 57 of the Rules of Court.
Therefore, ruled the Court of Appeals, the attachment made over the Chemphil
shares in the name of Garcia on July 19, 1985 was made in accordance with law and
the lien created thereby remained valid and subsisting at the time Garcia sold those
shares to FCI (predecessor-in-interest of appellee CEIC) in 1988.

330 | P a g e
Law 321_Corporation LAW_ Case Digest

GONZALO CHUA GUAN


vs.
SAMAHANG MAGSASAKA, INC., and SIMPLICIO OCAMPO, ADRIANO G. SOTTO,
and EMILIO VERGARA, as president, secretary and treasurer respectively of the
same
G.R. No. L-42091 November 2, 1935

FACTS:

Samahang Magsasaka, Inc., is a corporation duly organized under the laws of


the Philippine Islands with principal office in Cabanatuan, Nueva Ecija, and that the
individual defendants are the president, secretary and treasurer respectively of the
same; that on June 18, 1931, Gonzalo H. Co Toco was the owner of 5,894 shares of
the capital stock of the said corporation represented by nine certificates having a par
value of P5 per share; that on said date Gonzalo H. Co Toco, a resident of Manila,
mortgaged said 5,894 shares to Chua Chiu to guarantee the payment of a debt of
P20,000 due on or before June 19, 1932. The said certificates of stock were delivered
with the mortgage to the mortgagee, Chua Chiu. The said mortgage was duly
registered in the office of the register of deeds of Manila on June 23, 1931, and in the
office of the said corporation on September 30, 1931. On November 28, 1931, Chua
Chiu assigned all his right and interest in the said mortgage to the plaintiff and the
assignment was registered in the office of the register of deeds in the City of Manila on
December 28, 1931, and in the office of the said corporation on January 4, 1932.
The debtor, Gonzalo H. Co Toco, having defaulted in the payment of said debt at
maturity, the plaintiff foreclosed said mortgage and delivered the certificates of stock
and copies of the mortgage and assignment to the sheriff of the City of Manila in order
to sell the said shares at public auction. The sheriff auctioned said 5,894 shares of
stock on December 22, 1932, and the plaintiff having been the highest bidder for the
sum of P14,390, the sheriff executed in his favor a certificate of sale of said shares.
The plaintiff tendered the certificates of stock standing in the name of Gonzalo H. Co
Toco to the proper officers of the corporation for cancellation and demanded that they
issue new certificates in the name of the plaintiff. The said officers (the individual
defendants) refused and still refuse to issue said new shares in the name of the
plaintiff.

ISSUE:

Whether or not the mortgage registered considered the certificate of registration


in the corporation.

RULING:

YES.

By analogy with the foregoing and considering the ownership of shares in a


corporation as property distinct from the certificates which are merely the evidence of
such ownership, it seems to us a reasonable construction of section 4 of Act No. 1508
to hold that the property in the shares may be deemed to be situated in the province
in which the corporation has its principal office or place of business. If this province is
also the province of the owner's domicile, a single registration sufficient. If not, the
chattel mortgage should be registered both at the owner's domicile and in the province
where the corporation has its principal office or place of business. In this sense the
property mortgaged is not the certificate but the participation and share of the owner
in the assets of the corporation.

331 | P a g e
Law 321_Corporation LAW_ Case Digest

COLLECTOR OF INTERNAL REVENUE


vs.
ANGLO CALIFORNIA NATIONAL BANK (CROCKER-ANGLO NATIONAL BANK), as
Treasurer for CALAMBA SUGAR ESTATE, INC
G .R. No. L-12476 January 29, 1960

FACTS:

Calamba Sugar Estate, Inc., herein represented by its trustee, the Anglo
California National Bank, is a foreign corporation organized and existing under the
laws of the State of California, U.S.A., duly licensed (on May 8, 1946) to do business in
the Philippines. It has consistently filed its income tax returns here through its
resident attorney-in-fact. On May 14, 1956, the petitioners Collector of Internal
Revenue the corporation of an assessment for alleged deficiency income taxes for the
years 1953, 1954 and 1955 in the respective amounts of P138,855.00, P131,759.00
and P393,459.00, supposedly based upon capital again derived from the respondent's
sale to the Pasumil Planters, Inc., of P250,000 shares of the capital stock of the
Pampanga Sugar Mills (a domestic corporation) and of a promissory note, dated
January 1, 1950, executed by the Pampanga Sugar Mills in the sum of $500,000.00.
In an appeal by the respondent from the ruling of the Collector, the Court of Tax
Appeals reversed said ruling and absolved the respondent form liability.

ISSUE:

Whether or not respondent is liable for tax regarding the transfer of share of
stock.

RULING:

YES.

It is hardly disputable that although shares of stock of a corporation represent


equities may consist of real as well as personal properties therein, they are considered
under applicable law and jurisprudence as intangible personal properties.
Section 24 of the National Internal Revenue Codes levies income taxes on
foreign corporations only on income derived from sources within the Philippines; and
with respect to capital gains on the sale of personal properties, section 37 (e) of the
same Tax Code deems the place of sale as also that place or source of the capital gain:
In this case, it is admitted that the negotiation, perfection and consummation of
the contract of sale were all done in California, U.S.A. It follows that title to the shares
of stock passed from the vendor to the vendee at said place, from which time the
incidents of ownership vested on the buyer. The petition is denied.

332 | P a g e
Law 321_Corporation LAW_ Case Digest

APOLINARIO G. DE LOS SANTOS and ISABELO ASTRAQUILLO


vs.
J. HOWARD MCGRATH ATTORNEY GENERAL OF THE UNITED STATES,
SUCCESSOR TO THE PHILIPPINE ALIEN PROPERTY ADMINISTRATION OF THE
UNITED STATES, REPUBLIC OF THE PHILIPPINES
G.R. No. L-4818 February 28, 1955

FACTS:

This action involves the title to 1,600,000 shares of stock of the Lepanto
Consolidated Mining Co., Inc., a corporation duly organized and existing under the
laws of the Philippines, hereinafter referred to, for the sake of brevity, as the Lepanto.
Originally, one-half of said shares of stock were claimed by plaintiff, Apolinario de los
Santos, and the other half, by his co-plaintiff Isabelo Astraquillo. During the pendency
of this case, the latter has allegedly conveyed and assigned his interest in and to said
half claimed by him to the former. The shares of stock in question are covered by
several stock certificates issued in favor of Vicente Madrigal, who is registered in the
books of the Lepanto as owner of said stocks and whose indorsement in blank appears
on the back of said certificates, all of which, except certificates No. 2279 — marked
Exhibit 2 — covering 55,000 shares, are in plaintiffs' possession. So was said Exhibit
2, up to sometime in 1945 or 1946 when said possession was lost under the
conditions set forth in subsequent pages.

ISSUE:

Whether or not the plaintiffs had the owners of the shares of stock in question.

RULING:

NO.

In the case at bar, neither madrigal nor the Mitsuis had alienated shares of
stock in question. It is not even claimed that either had, through negligence, given —
occasion for an improper or irregular disposition of the corresponding stock
certificates. Plaintiffs merely argue without any evidence whatsoever thereon — that
Kitajimamight have, or must have, assigned the certificates on or before December
1942, although, as above stated, this is, not only, improbable, under the conditions,
then obtaining, but, also., impossible, considering that, in April 1943, Kitajima
delivered the instruments to Miwa, who kept them in its possession until 1945. At any
rate, such assignment by Miwa — granting for the sake of argument the accuracy of
the surmise of plaintiffs herein — was unauthorized by the mitsuis, who, in the light
of the precedents cited above, are not chargeable with negligence. In other words,
assuming that Kitajima had been guilty of embezzlement, by negotiating the stock
certificates in question for his personal benefit, as claimed by the plaintiffs, the title of
his assignees and successors in interest would still be subject to the rights of the
registered owner, namely, Madrigal, and consequently, of the party for whose benefit
and account the latter held the corresponding shares of stock, that is to say, the
Mitsuis.
In conclusion, when the Property Custodian issued the Vesting Order
complained of, the shares of stock in question belonged to the Mitsuis, admittedly an
enemy corporation, so that Vesting Order is in conformity with law and should be
upheld. Wherefore, the decision appealed from is hereby reversed, and the complaint,
accordingly, dismissed, with costs against the plaintiffs-appellees. It is so ordered.

333 | P a g e
Law 321_Corporation LAW_ Case Digest

GLORIA M. DE ERQUIAGA, administratrix of the estate of the late SANTIAGO DE


ERQUIAGA & HON. FELICIANO S. GONZALES
vs.
HON. COURT OF APPEALS, AFRICA VALDEZ VDA. DE REYNOSO, JOSES V.
REYNOSO, JR., EERNESTO , SYLVIA REYNOSO, LOURDES REYNOSO, CECILE
REYNOSO, EDNA REYNOSO, ERLINDA REYNOSO & EMILY REYNOSO
G.R. No. 47206 September 27, 1989

FACTS:

Santiago de Erquiaga was the owner of 100% or 3,100 paid-up shares of stock
of the Erquiaga Development Corporation which owns the Hacienda San Jose in
Irosin, Sorsogon (p. 212, Rollo). He entered into an Agreement with Jose L. Reynoso to
sell to the latter his 3,100 shares (or 100%) of Erquiaga Development Corporation for
P900,000 payable in installments on definite dates fixed in the contract but not later
than November 30, 1968.
Because Reynoso failed to pay the second and third installments on time, the
total price of the sale was later increased to P971,371.70 payable on or before
December 17, 1969. The difference of P71,371.70 represented brokers' commission
and interest (CFI Decision, pp. 75, 81, 90, 99,Rollo). As of December 17, 1968,
Reynoso was able to pay the total sum of P410,000 to Erquiaga who thereupon
transferred all his shares (3,100 paid-up shares) in Erquiaga Development
Corporation to Reynoso, as well as the possession of the Hacienda San Jose, the only
asset of the corporation However, as provided in paragraph 3, subparagraph (c) of the
contract to sell, Reynoso pledged 1,500 shares in favor of Erquiaga as security for the
balance of his obligation. Reynoso failed to pay the balance of P561,321.70 on or
before December 17, 1969, as provided in the promissory notes he delivered to
Erquiaga.

ISSUE:

Whether or not the Corporation Code Applicable.

RULING:

YES.

We find no reversible error in the Court of Appeals' decision directing the clerk
of court of the trial court to execute a deed of conveyance to Erquiaga of the 1,600
shares of stock of the Erquiaga Development Corporation still in Reynoso's name
and/or possession, in accordance with the procedure in Section 10, Rule 39 of the
Rules of Court. Neither did it err in annulling the trial court's order: (1) allowing
Erquiaga to vote the 3,100 shares of Erquiaga Development Corporation without
having effected the transfer of those shares in his name in the corporate books; and (2)
authorizing Erquiaga to call a special meeting of the stockholders of the Erquiaga
Development Corporation and to vote the 3,100 shares, without the pre-requisite
registration of the shares in his name. It is a fundamental rule in Corporation Law
(Section 35) that a stockholder acquires voting rights only when the shares of stock to
be voted are registered in his name in the corporate books.
The order of respondent Court directing Erquiaga to return the sum of
P410,000 (or net P348,000 after deducting P62,000 due from Reynoso under the
decision) as the price paid by Reynoso for the shares of stock, with legal rate of
interest, and the return by Reynoso of Erquiaga's 3,100 shares with the
fruits(construed to mean not only dividends but also fruits of the corporation's
Hacienda San Jose) is in full accord with Art. 1385 of the Civil Code.
The Hacienda San Jose and 1,500 shares of stock have already been returned
to Erquiaga. Therefore, upon the conveyance to him of the remaining 1,600 shares,
Erquiaga (or his heirs) should return to Reynoso the price of P410,000 which the latter
paid for those shares. Pursuant to the rescission decreed in the final judgment, there
should be simultaneous mutual restitution of the principal object of the contract to
sell (3,100 shares) and of the consideration paid (P410,000).

334 | P a g e
Law 321_Corporation LAW_ Case Digest

NEMESIO GARCIA
vs.
NICOLAS JOMOUAD, Ex-officio Provincial Sheriff of Cebu and SPOUSES JOSE
ATINON & SALLY ATINON
G.R. No. 133969 January 26, 2000

FACTS:

Petitioner filed an action for injunction with prayer for preliminary injunction
against respondents spouses Jose and Sally Atinon and Nicolas Jomouad, ex-
officio sheriff of Cebu.
Said action stemmed from an earlier case for collection of sum of money,
docketed as Civil Case No. CEB-10433, before the RTC, Branch 10 of Cebu, filed by
the spouses Atinon against Jaime Dico. In that case (collection of sum of money), the
trial court rendered judgment ordering Dico to pay the spouses Atinon the sum of
P900,000.00 plus interests.
After said judgment became final and executory, respondent sheriff proceeded
with its execution. In the course thereof, the Proprietary Ownership Certificate (POC)
No. 0668 in the Cebu Country Club, which was in the name of Dico, was levied on and
scheduled for public auction. Claiming ownership over the subject certificate,
petitioner filed the aforesaid action for injunction with prayer for preliminary
injunction to enjoin respondents from proceeding with the auction.

ISSUE:

Whether or not the appellate court erroneously rely on Section 63 of the


Corporation Code in upholding the levy on the subject certificate to satisfy the
judgment debt of Dico in Civil Case No. CEB-14033.

RULING:

NO.

The transfer of the subject certificate made by Dico to petitioner was not valid
as to the spouses Atinon, the judgment creditors, as the same still stood in the name
of Dico, the judgment debtor, at the time of the levy on execution. In addition, as
correctly ruled by the CA, the entry in the minutes of the meeting of the Club's board
of directors noting the resignation of Dico as proprietary member thereof does not
constitute compliance with Section 63 of the Corporation Code. Said provision of law
strictly requires the recording of the transfer in the books of the corporation, and not
elsewhere, to be valid as against third parties. Accordingly, the CA committed no
reversible error in rendering the assailed decision.

335 | P a g e
Law 321_Corporation LAW_ Case Digest

BENITO H. LOPEZ
vs.
THE COURT OF APPEALS and THE PHILIPPINE AMERICAN GENERAL
INSURANCE CO., INC.
G.R. No. L-33157 June 29, 1982

FACTS:

Petitioner Benito H. Lopez obtained a loan in the amount of P20,000.00 from


the Prudential Bank and Trust Company. Also, he executed a promissory note for the
same amount, in favor of the said Bank, binding himself to repay the said sum one (1)
year after the said date, with interest at the rate of 10% per annum. In addition to said
promissory note, he executed Surety Bond No. 14164 in which he, as principal, and
Philippine American General Insurance Co., Inc. (PHILAMGEN) as surety, bound
themselves jointly and severally in favor of Prudential Bank for the payment of the
sum of P20,000.00. On the same occasion, Lopez also executed in favor of Philamgen
an indemnity agreement whereby he agreed "to indemnify the Company and keep it
indemnified and hold the same harmless from and against any and all damages,
losses, costs, stamps, taxes, penalties, charges and expenses of whatever kind and
nature which the Company shall or may at any time sustain or incur in consequence
of having become surety upon the bond." 1 At the same time, Lopez executed a deed of
assignment of 4,000 shares of the Baguio Military Institution entitled "Stock
Assignment Separate from Certificate". With the execution of this deed of assignment,
Lopez endorsed the stock certificate and delivered it to Philamgen.

ISSUE:

Whether or not where, as in this case, a party "sells, assigns and transfers" and
delivers shares of stock to another, duly endorsed in blank, in consideration of a
contingent obligation of the former to the latter, and, the obligations having arisen, the
latter causes the shares of stock to be transferred in its name, hass the juridical
nature of the transaction-a dation in payment or a pledge.

RULING:

YES.

Considering the explicit terms of the deed denominated "Stock Assignment


Separate from Certificate", hereinbefore copied verbatim, Lopez sold, assigned and
transferred unto Philamgen the stocks involved "for and in consideration of the
obligations undertaken" by Philamgen "under the terms and conditions of the surety
bond executed by it in favor of the Prudential Bank" and "for value received". On its
face, it is neither pledge nor dation in payment. The document speaks of an outright
sale as there is a complete and unconditional divestiture of the incorporeal property
consisting of stocks from Lopez to Philamgen. The transfer appears to have been an
absolute conveyance of the stocks to Philamgen whether or not Lopez defaults in the
payment of P20,000.00 to Prudential Bank. While it is a conveyance in consideration
of a contingent obligation, it is not itself a conditional conveyance.
It is true that if Lopez should "well and truly perform and fulfill all the
undertakings, covenants, terms, conditions, and agreements stipulated" in his
promissory note to Prudential Bank, the obligation of Philamgen under the surety
bond would become null and void. Corollarily, the stock assignment, which is
predicated on the obligation of Philamgen under the surety bond, would necessarily
become null and void likewise, for want of cause or consideration under Article 1352
of the New Civil Code. But this is not the case here because aside from the obligations
undertaken by Philamgen under the surety bond, the stock assignment had other
considerations referred to therein as "value received". Hence, based on the manifest
terms thereof, it is an absolute transfer.

336 | P a g e
Law 321_Corporation LAW_ Case Digest

ENRIQUE MONSERRAT
vs.
CARLOS G. CERON, ET AL. ERMA, INC., and, THE SHERIFF OF MANILA
G.R. No. 37078 September 27, 1933

FACTS:

Enrique Monserrat, was the president and manager of the Manila Yellow
Taxicab Co., Inc., and the owner of P1,200 common shares of stock thereof. On March
25, 1930, in consideration of the interest shown and the financial aid extended him in
the organization of the corporation by Carlos G. Ceron, one of the defendants herein,
Enrique Monserrat assigned to the former the usufruct of half of the aforesaid
common shares of stock, the corresponding certificate of stock No. 7, having been
issued in the name of said Carlos G, Ceron to that effect on March 24, 1930. (Exhibit
1.) Said assignment or transfer only gave the transferee the right to enjoy, during his
lifetime, the profits which might be derived from the shares assigned him, prohibiting
him from selling, mortgaging, encumbering, alienating or otherwise exercising any act
implying absolute ownership of all or any of the shares in question, the transferor
having reserved for himself and his heirs the right to vote derived from said shares of
stock and to recover the ownership thereof at the termination of the usufruct (Exhibit
A). Stock certificate No. 7 was recorded in the name of Carlos G. Ceron and the
aforesaid deed of transfer Exhibit A, was noted by himself as secretary, on page 22 of
the Stock and Transfer Book of the Manila Yellow Taxicab Co., Inc.

ISSUE:

Whether or not it is necessary to enter upon the books of the corporation a


mortgage constituted on common shares of stock in order that such mortgage may be
valid and may have force and effect as against third persons.

RULING:

YES.

A "transfer" is the act by which owner of a thing delivers it to another with the
intent of passing the rights which he has in it to the latter, and a chattel mortgage is
not within the meaning of such term.
Therefore, the chattel mortgage is not the transfer referred to in section 35 of
Act No. 1459 commonly known as the Corporation law, which transfer should be
entered and noted upon the books of a corporation in order to be valid, and which, as
has already been said, means the absolute and unconditional conveyance of the title
and ownership of a share of stock.
If, in accordance with said section 35 of the Corporation Law, only the transfer
or absolute conveyance of the ownership of the title to a share need be entered and
noted upon the books of the corporation in order that such transfer may ba valid,
therefore, inasmuch as a chattel mortgage of the aforesaid title is not a complete and
absolute alienation of the dominion and ownership thereof, its entry and notation
upon the books of the corporation is not necessary requisite to its validity.

337 | P a g e
Law 321_Corporation LAW_ Case Digest

EUGENIO J. PUYAT, ERWIN L. CHIONGBIAN, EDGARDO P. REYES, ANTONIO G.


PUYAT, JAIME R. BLANCO, RAFAEL R. RECTO and REYNALDO L. LARDIZABAL
vs.
HON. SIXTO T. J. DE GUZMAN, JR., as Associate Commissioner of the Securities
& Exchange Commission, EUSTAQUIO T. C. ACERO, R. G. VILDZIUS, ENRIQUE M.
BELO, MANUEL G. ABELLO, SERVILLANO DOLINA, JUANITO MERCADO and
ESTANISLAO A. FERNANDEZ
G.R. No. L-51122 March 25, 1982

FACTS:

On May 14, 2979, an election for the eleven Directors of the International Pipe
Industries Corporation (IPI) a private corporation, was held. b) May 25, 1979. The
Acero Group instituted at the Securities and Exchange Commission (SEC) quo
warrantoproceedings, docketed as Case No. 1747 (the SEC Case), questioning the
election of May 14, 1979. The Acero Group claimed that the stockholders' votes were
not properly counted. The Puyat Group claims that at conferences of the parties with
respondent SEC Commissioner de Guzman, Justice Estanislao A. Fernandez, then a
member of the Interim Batasang Pambansa, orally entered his appearance as counsel
for respondent Acero to which the Puyat Group objected on Constitutional grounds.
Section 11, Article VIII, of the 1973 Constitution, then in force, provided that no
Assemblyman could "appear as counsel before ... any administrative body", and SEC
was an administrative body. Incidentally, the same prohibition was maintained by the
April 7, 1981 plebiscite. The cited Constitutional prohibition being clear,
Assemblyman Fernandez did not continue his appearance for respondent Acero.

ISSUE:

Whether or not in intervening in the SEC Case, Assemblyman Fernandez is, in


effect, appearing as counsel, albeit indirectly, before an administrative body in
contravention of the Constitutional provision.

RULING:

YES.

Ordinarily, by virtue of the Motion for Intervention, Assemblyman Fernandez


cannot be said to be appearing as counsel. Ostensibly, he is not appearing on behalf of
another, although he is joining the cause of the private respondents. His appearance
could theoretically be for the protection of his ownership of ten (10) shares of IPI in
respect of the matter in litigation and not for the protection of the petitioners nor
respondents who have their respective capable and respected counsel.
However, certain salient circumstances militate against the intervention of
Assemblyman Fernandez in the SEC Case. He had acquired a mere P200.00 worth of
stock in IPI, representing ten shares out of 262,843 outstanding shares. Realizing,
perhaps, the validity of the objection, he decided, instead, to "intervene" on the ground
of legal interest in the matter under litigation. And it may be noted that in the case
filed before the Rizal Court of First Instance (L-51928), he appeared as counsel for
defendant Excelsior, co-defendant of respondent Acero therein.
Under those facts and circumstances, we are constrained to find that there has
been an indirect "appearance as counsel before ... an administrative body" and, in our
opinion, that is a circumvention of the Constitutional prohibition. The "intervention"
was an afterthought to enable him to appear actively in the proceedings in some other
capacity. To believe the avowed purpose, that is, to enable him eventually to vote and
to be elected as Director in the event of an unfavorable outcome of the SEC Case
would be pure naivete. He would still appear as counsel indirectly.
In brief, we hold that the intervention of Assemblyman Fernandez in SEC. No.
1747 falls within the ambit of the prohibition contained in Section 11, Article VIII of
the Constitution.

338 | P a g e
Law 321_Corporation LAW_ Case Digest

ENRIQUE RAZON
vs.
INTERMEDIATE APPELLATE COURT and VICENTE B. CHUIDIAN, in his capacity
as Administrator of the Estate of the Deceased JUAN T. CHUIDIAN
G .R. No. 74306 March 16, 1992

FACTS:

In his complaint filed on June 29, 1971, and amended on November 16, 1971,
Vicente B. Chuidian prayed that defendants Enrique B. Razon, E. Razon, Inc.,
Geronimo Velasco, Francisco de Borja, Jose Francisco, Alfredo B. de Leon, Jr., Gabriel
Llamas and Luis M. de Razon be ordered to deliver certificates of stocks representing
the shareholdings of the deceased Juan T. Chuidian in the E. Razon, Inc. with a
prayer for an order to restrain the defendants from disposing of the said shares of
stock, for a writ of preliminary attachment v. properties of defendants having
possession of shares of stock and for receivership of the properties of defendant
corporation.
In their answer filed on June 18, 1973, defendants alleged that all the shares of
stock in the name of stockholders of record of the corporation were fully paid for by
defendant, Razon; that said shares are subject to the agreement between defendants
and incorporators; that the shares of stock were actually owned and remained in the
possession of Razon. Appellees also alleged . . . that neither the late Juan T. Chuidian
nor the appellant had paid any amount whatsoever for the 1,500 shares of stock in
question

ISSUE:

Whether or not petitioner have right over the ownership of the 1,500 shares of
stock in E. Razon, Inc.

RULING:

NO.

In the instant case, there is no dispute that the questioned 1,500 shares of
stock of E. Razon, Inc. are in the name of the late Juan Chuidian in the books of the
corporation. Moreover, the records show that during his lifetime Chuidian was ellected
member of the Board of Directors of the corporation which clearly shows that he was a
stockholder of the corporation. From the point of view of the corporation, therefore,
Chuidian was the owner of the 1,500 shares of stock. In such a case, the petitioner
who claims ownership over the questioned shares of stock must show that the same
were transferred to him by proving that all the requirements for the effective transfer
of shares of stock in accordance with the corporation's by laws, if any, were followed or
in accordance with the provisions of law.
The petitioner failed in both instances. The petitioner did not present any by-
laws which could show that the 1,500 shares of stock were effectively transferred to
him. In the absence of the corporation's by-laws or rules governing effective transfer of
shares of stock, the provisions of the Corporation Law are made applicable to the
instant case.
The law is clear that in order for a transfer of stock certificate to be effective, the
certificate must be properly indorsed and that title to such certificate of stock is vested
in the transferee by the delivery of the duly indorsed certificate of stock. Since the
certificate of stock covering the questioned 1,500 shares of stock registered in the
name of the late Juan Chuidian was never indorsed to the petitioner, the inevitable
conclusion is that the questioned shares of stock belong to Chuidian. The petitioner's
asseveration that he did not require an indorsement of the certificate of stock in view
of his intimate friendship with the late Juan Chuidian can not overcome the failure to
follow the procedure required by law or the proper conduct of business even among
friends. To reiterate, indorsement of the certificate of stock is a mandatory.

339 | P a g e
Law 321_Corporation LAW_ Case Digest

AQUILINO RIVERA, ISAMU AKASAKO, FUJIYAMA HOTEL & RESTAURANT, INC.


vs.
THE HON. ALFREDO C. FLORENDO, as Judge of the Court of First Instance of
Manila (Branch XXXVI), LOURDES JUREIDINI and MILAGROS TSUCHIYA
G.R. No. L-57586. October 8, 1986

FACT:

Petitioner corporation was organized and register under Philippine laws with a
capital stock of P1,000,000.00 divided into 10,000 shares of P100.00 par value each
by the herein petitioner Rivera and four (4) other incorporators. Sometime thereafter
petitioner Rivera increased his subscription from the original 1,250 to a total of 4899
shares.
Subsequently, Isamu Akasako, a Japanese national and co-petitioner who is
allegedly the real owner of the shares of stock in the name of petitioner Aquilino
Rivera, sold 2550 shares of the same to private respondent Milagros Tsuchiya for a
consideration of P440,000.00 with the assurance that Milagros Tsuchiya will be made
the President and Lourdes Jureidini a director after the purchase. Aquilino Rivera who
was in Japan also assured private respondents by overseas call that he will sign the
stock certificates because Isamu Akasako is the real owner. However, after the sale
was consummated and the consideration was paid with a receipt of payment therefor
shown, Aquilino Rivera refused to make the indorsement unless he is also paid.

ISSUE:

Whether or not the respondent court of first instance have no jurisdiction over
the petition for mandamus and receivership "as well as in placing the corporate assets
under provisional receivership in the guise of a writ of preliminary mandatory
injunction.

RULING:

YES.

It has already been settled that an intracorporate controversy would call for the
jurisdiction of the Securities and Exchange Commission. On the other hand, an intra-
corporate controversy has been defined as "one which arises between a stockholder
and the corporate. There is no distinction, qualification, nor any exemption
whatsoever."
This Court has also ruled that cases of private respondents who are not
shareholders of the corporation, cannot be a "controversy arising out of intracorporate
or partnership relations between and among stockholders, members or associates;
between any or all of them and the corporation, partnership or association, of which
they are stockholders, members or associates, respectively."

340 | P a g e
Law 321_Corporation LAW_ Case Digest

JOSEFA SANTAMARIA, assisted by her husband, FRANCISCO SANTAMARIA, Jr.


vs.
THE HONGKONG AND SHANGHAI BANKING CORPORATION and R. W. TAPLIN.
G.R. No. L-2808 August 31, 1951

FACTS:

Mrs. Josefa T. Santamaria bought 10,000 shares of the Batangas Minerals,


Inc., through the offices of Woo, Uy-Tioco & Naftaly, a stock brokerage firm and pay
therefore the sum of P8,041.20 as shown by receipt Exh. B. The buyer received Stock
Certificate No. 517 issued in the name of Woo, Uy-Tioco & Naftaly and indorsed in
bank by this firm.
On March 9, 1937, Mrs. Santamaria placed an order for the purchase of 10,000
shares of the Crown Mines, Inc. with R.J. Campos & Co., a brokerage firm, and
delivered Certificate No. 517 to the latter as security therefor with the understanding
that said certificate would be returned to her upon payment of the 10,000 Crown
Mines, Inc. shares. Exh. D. is the receipt of the certificate in question signed by one
Mr. Cosculluela, Manager of the R.J. Campos & Co., Inc. According to certificate Exh.
E, R. J. Campos & Co., Inc. bought for Mrs. Josefa Santamaria 10,000 shares of the
Crown Mines, Inc. at .225 a share, or the total amount of P2,250. Two days later, on
March 11, Mrs. Santamaria went to R.J. Campos & Co., Inc. to pay for her order of
10,000 Crown Mines shares and to get back Certificate No. 517. Cosculluela then
informed her that R.J. Campos & Co., Inc. was no longer allowed to transact business
due to a prohibition order from Securities and Exchange Commission. She was also
inform that her Stock certificate was in the possession of the Hongkong and Shanghai
Banking Corporation.

ISSUE:

Whether or not the obligation of the defendant Bank to have inquired into the
ownership of the certificate when it received it from R.J. Campos & Co., Inc. and not
conclude that the Bank was negligent for not having done so, contrary to the claim of
the plaintiff that defendant Bank acted negligently, if not in bad faith, in accepting
delivery of said certificate from RJ. Campos & Co., Inc.

RULING:

YES.

Certificate No. 517 came into the possession of the defendant Bank because
R.J. Campos & Co., Inc. had opened an overdraft account with said Bank and to this
effect it had executed on April 16, 1946, a letter of hypothecation by the terms of
which R.J. Campos & Co., Inc. pledged to the said Bank "all Stocks, Shares and
Securities which I/we may hereafter come into their possession on my/our account
and whether originally deposited for safe custody only or for any other purpose
whatever or which may hereafter be deposited by me/us in lieu of or in addition to the
Stocks, Shares, and Securities now deposited or for any other purpose whatsoever."
It should be noted that the certificate of stock in question was issued in the
name of the brokerage firm-Woo, Uy-Tioco & Naftaly and that it was duly indorsed in
blank by said firm, and that said indorsement was guaranteed by R.J. Campos & Co.,
Inc., which in turn indorsed it in blank. This certificate is what it is known as street
certificate. Upon its face, the holder was entitled to demand its transfer into his name
from the issuing corporation. The Bank was not obligated to look beyond the
certificate to ascertain the ownership of the stock at the time it received the same from
R.J. Campos & Co., Inc., for it was given to the Bank pursuant to their letter of
hypothecation. Even if said certificate had been in the name of the plaintiff but
indorsed in blank, the Bank would still have been justified in believing that R.J.
Campos & Co., Inc. had title thereto for the reason that it is a well-known practice
that a certificate of stock, indorsed in blank, is deemed quasi negotiable, and as such
the transferee thereof is justified in believing that it belongs to the holder and
transferor.

341 | P a g e
Law 321_Corporation LAW_ Case Digest

MANUEL A. TORRES, JR., (Deceased), GRACIANO J. TOBIAS, RODOLFO L.


JOCSON, JR., MELVIN S. JURISPRUDENCIA, AUGUSTUS CESAR AZURA and
EDGARDO D. PABALAN
vs.
COURT OF APPEALS, SECURITIES AND EXCHANGE COMMISSION, TORMIL
REALTY & DEVELOPMENT CORPORATION, ANTONIO P. TORRES, JR., MA.
CRISTINA T. CARLOS, MA. LUISA T. MORALES and DANTE D. MORALES.
G.R. No. 120138 September 5, 1997

FACTS:

The late Manuel A. Torres, Jr. was the major stockholder of Tormil Realty &
Development Corporation while private respondents who are the children of Judge
Torres' deceased brother Antonio A. Torres, constituted the minority stockholders. In
particular, their respective shareholdings and positions in the corporation.
In 1984, Judge Torres, in order to make substantial savings in taxes, adopted
an "estate planning" scheme under which he assigned to Tormil Realty & Development
Corporation (Tormil for brevity) various real properties he owned and his shares of
stock in other corporations in exchange for 225,972 Tormil Realty shares. Hence, on
various dates in July and August of 1984, ten (10) deeds of assignment were executed
by the late Judge Torres.Consequently, the aforelisted properties were duly recorded in
the inventory of assets of Tormil Realty and the revenues generated by the said
properties were correspondingly entered in the corporation's books of account and
financial records.
Due to the insufficient number of shares of stock issued to Judge Torres and
the alleged refusal of private respondents to approve the needed increase in the
corporation's authorized capital stock (to cover the shortage of 972 shares due to
Judge Torres under the "estate planning" scheme), on 11 September 1986, Judge
Torres revoked the two (2) deeds of assignment covering the properties in Makati and
Pasay City.

ISSUE:

Whether or not the deed of assignment executed can be revoked.

RULING:

NO.

The shortage of 972 shares would not be valid ground for respondent Torres to
unilaterally revoke the deeds of assignment he had executed on July 13, 1984 and
July 24, 1984 wherein he voluntarily assigned to TORMIL real properties covered by
TCT No. 374079 (Makati) and TCT No. 41527, 41528 and 41529 (Pasay) respectively. A
comparison of the number of shares that respondent Torres received from TORMIL by
virtue of the "deeds of assignment" and the stock certificates issued by the latter to the
former readily shows that TORMIL had substantially performed what was expected of
it. In fact, the first two issuances were in satisfaction to the properties being revoked
by respondent Torres. Hence, the shortage of 972 shares would never be a valid
ground for the revocation of the deeds covering Pasay and Quezon City properties.
Moreover, we agree with the contention of the Solicitor General that the
shortage of shares should not have affected the assignment of the Makati and Pasay
City properties which were executed in 13 and 24 July 1984 and the consideration for
which have been duly paid or fulfilled but should have been applied logically to the
last assignment of property — Judge Torres' Ayala Fund shares — which was executed
on 29 August 1984.

342 | P a g e
Law 321_Corporation LAW_ Case Digest

LEE E. WON alias RAMON LEE


vs.
WACK WACK GOLF and COUNTRY CLUB, INC.
G.R. No. L-10122 August 30, 1958

Facts:

The defendant (a non-stock corporation) issued to Iwao Teruyama Membership


Certificate No. 201 which was assigned to M. T. Reyes on April 22, 1944. Subsequently
in the same year 1944, M. T. Reyes transferred and assigned said certificate to the
plaintiff. On April 26, 1955, the plaintiff filed an action in the Court of First Instance
of Manila against the defendant, alleging that shortly after the rehabilitation of the
defendant after the war, the plaintiff asked the defendant to register in its books the
assignment in favor of the plaintiff and to issue to the latter a new certificate, but that
the defendant had refused and still refuses to do so unlawfully; and praying that the
plaintiff be declared the owner of one share of stock of the defendant and that the
latter be ordered to issue a correspondent new certificate. On June 6, 1955, the
defendant filed a motion to dismiss, alleging that from 1944, when the plaintiff's right
of action had accrued, to April 26, 1955, when the complaint was filed, eleven years
have elapsed, and that therefore the complaint was filed beyond the 5-year period
fixed in Article 1149 of the Civil Code. On July 30, 1955, the Court of First Instance of
Manila issued an order dismissing the complaint. As plaintiff's motion for
reconsideration filed on August 27, 1955 and second motion for reconsideration filed
on September 13, 1955, were both denied, the plaintiff has taken the present appeal.

Issue:

Whether or not the plaintiff is entitled to the registration of the transferred


share of stock.

RULING:

NO.

The certificate in question contains a condition to the effect that no assignment


thereof "shall be effective with respect to the club until such assignment is registered
in the books of the club, as provided in the By-Laws." The decisive question that arises
is whether the plaintiff was bound, under said condition and By-Laws of the defendant
or any statutory rule for that matter, to present and register the certificate assigned to
him in 1944 within any definite or fixed period. The defendant has not made herein
any pretense to that effect; but it contends that from the moment the certificate was
assigned to the plaintiff, the latter's right to have the assignment registered
commenced to exist. This contention is correct, but it would not follow that said right
should be exercised immediately or within a definite period. The existence of a right is
one thing, and the duration of said right is another.
On the other hand, it is stated in the appealed order of dismissal that the
plaintiff sought to register the assignment on April 13, 1955; whereas in plaintiff's
brief it is alleged that it was only in February, 1955, when the defendant refused to
recognize the plaintiff. If, as already observed, there is no fixed period for registering
an assignment, how can the complaint be considered as already barred by the Statute
of Limitations when it was filed on April 26, 1955, or barely a few days (according to
the lower court) and two months (according to the plaintiff), after the demand for
registration and its denial by the defendant. Plaintiff's right was violated only
sometime in 1955, and it could not accordingly have asserted any cause of action
against the defendant before that.

343 | P a g e
Law 321_Corporation LAW_ Case Digest

Lost or Destroy Certificates

PHILEX MINING CORPORATION


vs.
HON. DOMINGO CORONEL REYES, Presiding Judge, Court of First Instance of
Albay, 10th Judicial District, Branch IV, and RICHARD HUENEFELD.
G.R. No. L-57707 November 19, 1982

FACTS:

Richard Huenefeld, is a stockholder of petitioner Philex Mining Corporation. He


originally owned 800,000 shares of stock. On February 6, 1980, First Asian wrote
Huenefeld informing him that the stock certificate had been delivered to him at his
address at Michelle Apartment, 2030 A. Mabini Street, Manila; and that if the
certificate could not be located that Huenefeld execute an Affidavit of Loss, with the
notice of loss to be published once a week for three (3) consecutive weeks in a
newspaper of general circulation in accordance with the procedure prescribed BY
Republic Act No. 201.
Huenefeld, through counsel, replied that RA 201 is not applicable because the
stock certificate was not lost in the possession of the stockholder; that assuming it
was, the expenses of publication and premiums for the bond should be at Philex's
expense; and demanded the issuance of a replacement stock certificate. Huenefeld
also submitted an Affidavit of Loss but did not comply with the other requirements on
publication.

ISSUE:

Whether or not the Court of First Instance hasve jurisdiction over the present
controversy, which Philex contends is an intra-corporate one.

RULING:

NO.

Evident from the foregoing is that an intra-corporate controversy is one which


arises between a stockholder and the corporation. There is no distinction,
qualification, nor any exemption whatsoever. The provision is broad and covers all
kinds of controversies between stockholders and corporations. The issue of whether or
not a corporation is bound to replace a stockholder's lost certificate of stock is a
matter purely between a stockholder and the corporation. It is a typical intra-
corporate dispute. The question of damages raised is merely incidental to that main
issue.
Section 5 of Presidential Decree No. 902-A provides: In addition to the
regulatory and adjudicative functions of the Securities and Exchange Commission over
corporations, partnerships and other forms of associations registered with it as
expressly granted under existing laws and decrees; it shall have original and exclusive
jurisdiction to hear and decide cases involving: Controversies arising out of intra-
corporate or partnership relations, between and among stockholders, members, or
associates; between any or all of them and the corporation, partnership or association
of which they are stockholders, members, or associates, respectively and between
such corporation, partnership or association and the state insofar as it concerns their
individual franchise or right to exist as such entity.

344 | P a g e
Law 321_Corporation LAW_ Case Digest

CORPORATE BOOKS AND RECORDS


Stock and Transfer Book

NORA A. BITONG
vs.
COURT OF APPEALS (FIFTH DIVISION), EUGENIA D. APOSTOL, JOSE A.
APOSTOL, MR. & MS. PUBLISHING CO., LETTY J. MAGSANOC
G.R. No. 123553 July 13, 1998

FACTS:

Petitioner Alleged before the SEC that she had been the Treasurer and a
Member of the Board of Directors of Mr. & Ms. from the time it was incorporated on 29
October 1976 to 11 April 1989, and was the registered owner of 1,000 shares of stock
out of the 4,088 total outstanding shares, petitioner complained of irregularities
committed from 1983 to 1987 by Eugenia D. Apostol, President and Chairperson of
the Board of Directors. Petitioner claimed that except for the sale of the
name Philippine Inquirer to Philippine Daily Inquirer (PDI hereafter) all other
transactions and agreements entered into by Mr. & Ms. with PDI were not supported
by any bond and/or stockholders' resolution. And, upon instructions of Eugenia D.
Apostol, Mr. & Ms. made several cash advances to PDI on various occasions
amounting to P3.276 million. On some of these borrowings PDI paid no interest
whatsoever. Despite the fact that the advances made by Mr. & Ms. to PDI were booked
as advances to an affiliate, there existed no board or stockholders' resolution, contract
nor any other document which could legally authorize the creation of and support to
an affiliate.
Petitioner further alleged that respondents Eugenia and Jose Apostol were
stockholders, directors and officers in both Mr. & Ms. and PDI. In fact on 2 May 1986
respondents Eugenia D. Apostol, Leticia J. Magsanoc and Adoracion G. Nuyda
subscribed to PDI shares of stock at P50,000.00 each or a total of P150,000.00.

ISSUE:

Whether or not the petitioner is the holder of the proper certificates of share of
stock.

RULING:

NO.

The certificate of stock itself once issued is a continuing affirmation or


representation that the stock described therein is valid and genuine and is at
least prima facie evidence that it was legally issued in the absence of evidence to the
contrary. However, this presumption may be rebutted. 13 Similarly, books and records
of a corporation which include even the stock and transfer book are generally
admissible in evidence in favor of or against the corporation and its members to prove
the corporate acts, its financial status and other matters including one's status as a
stockholder. They are ordinarily the best evidence of corporate acts and proceedings.
However, the books and records of a corporation are not conclusive even
against the corporation but are prima facie evidence only. Parol evidence may be
admitted to supply omissions in the records, explain ambiguities, or show what
transpired where no records were kept, or in some cases where such records were
contradicted. The effect of entries in the books of the corporation which purport to be
regular records of the proceedings of its board of directors or stockholders can be
destroyed by testimony of a more conclusive character than mere suspicion that there
was an irregularity in the manner in which the books were kept.
Thus, while petitioner asserts in her petition that Certificate of Stock No. 008
dated 25 July 1983 was issued in her name, private respondents argue that this
certificate was signed by respondent Eugenia D. Apostol as President only in 1989 and
was fraudulently antedated by petitioner who had possession of the Certificate Book
and the Stock and Transfer Book.

345 | P a g e
Law 321_Corporation LAW_ Case Digest

Inspection of Corporate Books and Records

SY TIONG SHIOU, JUANITA TAN SY, JOLIE ROSS TAN, ROMER TAN, CHARLIE
TAN, and JESSIE JAMES TAN
vs.
SY CHIM and FELICIDAD CHAN SY
G.R. No. 174168 March 30, 2009

FACTS:

Four criminal complaints were filed by Spouses Sy against Sy Tiong Shiou,


Juanita Tan Sy, Jolie Ross Tan, Romer Tan, Charlie Tan and Jessie James Tan. Two of
the complaints, I.S. Nos. 03E-15285 and 03E-15286, were for alleged violation of
Section 74 in relation to Section 144 of the Corporation Code. In these complaints, the
Spouses Sy averred that they are stockholders and directors of Sy Siy Ho & Sons, Inc.
(the corporation) who asked Sy Tiong Shiou, et al., officers of the corporation, to allow
them to inspect the books and records of the business on three occasions to no avail.
Sy Tiong Shiou, et al. denied the request, citing civil and intra-corporate cases pending
in court.
In the two other complaints, I.S. No. 03E-15287 and 03E-15288,6 Sy Tiong
Shiou was charged with falsification under Article 172, in relation to Article 171 of the
Revised Penal Code (RPC), and perjury under Article 183 of the RPC. According to the
Spouses Sy, Sy Tiong Shiou executed under oath the 2003 General Information Sheet
(GIS) wherein he falsely stated that the shareholdings of the Spouses Sy had
decreased despite the fact that they had not executed any conveyance of their shares.

ISSUE:

Whether or not Section 74 of the Corporation Code is violated.

RULING:

YES.

In a criminal complaint for violation of Section 74 of the Corporation Code, the


defense of improper use or motive is in the nature of a justifying circumstance that
would exonerate those who raise and are able to prove the same. Accordingly, where
the corporation denies inspection on the ground of improper motive or purpose, the
burden of proof is taken from the shareholder and placed on the corporation. However,
where no such improper motive or purpose is alleged, and even though so alleged, it is
not proved by the corporation, and then there is no valid reason to deny the requested
inspection.
In the instant case, however, the Court finds that the denial of inspection was
predicated on the pending civil case against the Spouses Sy.
Even in their Joint Counter-Affidavit dated 23 September 2003, Sy Tiong Shiou,
et al. did not make any allegation that "the person demanding to examine and copy
excerpts from the corporation‘s records and minutes has improperly used any
information secured through any prior examination of the records or minutes of such
corporation or of any other corporation, or was not acting in good faith or for a
legitimate purpose in making his demand." Instead, they merely reiterated the
pendency of the civil case. There being no allegation of improper motive, and it being
undisputed that Sy Tiong Shiou, et al. denied Sy Chim and Felicidad Chan Sy‘s
request for inspection, the Court rules and so holds that the DOJ erred in dismissing
the criminal charge for violation of Section 74 in relation to Section 144 of the
Corporation Code.

346 | P a g e
Law 321_Corporation LAW_ Case Digest

VICTOR AFRICA
vs.
PRESIDENTIAL COMMISSION ON GOOD GOVERNMENT, JOSE LAURETA,
MELQUIADES GUTIERREZ, EDUARDO M. VILLANUEVA, EDUARDO DE LOS
ANGELES and ROMAN MABANTA, JR.
G.R. No. 83831 January 9, 1992

FACTS:

Victor Africa, who claims to be an employee of ETPI holding the positions of


vice-president, general counsel (on official leave without pay), corporate secretary and
special assistant to the chairman (and president), filed directly with this Court on
June 30, 1988 a petition for injunction docketed as G.R. No. 83831, seeking to enjoin
the PCGG and its nominees/designees to the board of directors and the newly-
installed officers of ETPI from implementing their alleged illegal, invalid and immoral
act of ousting him from his offices and positions at the ETPI pending the
determination of whether they have validly, legally and morally assumed their
supposed positions and offices as "directors" and/or "officers" of ETPI.
He contends that the reasons advanced by the PCGG-sponsored board of
directors for ousting him from his offices (redundancy, need to conserve company
funds and loss of confidence) are flimsy, whimsical and arbitrary, evidencing not only
the PCGG-sponsored board's discriminatory and oppressive attitude towards him but,
more importantly, its clear intent to harass him into refraining from questioning before
several tribunals all the invalid, illegal and immoral acts of said PCGG-sponsored
board which have caused and are still causing ETPI damages because they constitute
dissipation of assets.

ISSUE:

Whether or not the acts and orders of the PCGG which led to the nomination
and election of the new members of the board of directors and officers of the ETPI
correct.

RULING:

YES.

In upholding therein the right of a stockholder of a sequestered company to


inspect and/or examine the records of a corporation pursuant to Section 74 of the
Corporation Code, the Court found nothing in Executive Orders Nos. 1, 2 and 14, as
well as in BASECO, to indicate an implied amendment of the Corporation Code, much
less an implied modification of a stockholder's right of inspection as guaranteed by
Section 74 thereof. The only express limitation on the right of inspection, according to
the Court, is that (1) the right of inspection should be exercised at reasonable hours
on business days; (2) the person demanding the right to examine and copy excerpts
from the corporate records and minutes has not improperly used any information
secured through any previous examination of the records of such corporation; and (3)
the demand is made in good faith or for a legitimate purpose.

347 | P a g e
Law 321_Corporation LAW_ Case Digest

REPUBLIC OF THE PHILIPPINES


vs.
SANDIGANBAYAN (THIRD DIVISION) and MACARIO ASISTIO, JR.
G.R. No. 90529 August 16, 1991

FACTS:

In a Joint Letter-Complaint to the Ombudsman dated January 8,1989, Messrs.


Arnel Blancaflor and Rodolfo Santos, residents of Kalookan City, charged respondent
Macario Asistio, Jr., who is the incumbent Mayor of Kalookan City, with having
violated the Anti-Graft and Corrupt Practices Act (R.A. 3019), specifically Section 8
thereof.
In said Joint Sworn Letter-Complaint, they alleged that during his incumbency
as Kalookan City Mayor in 1981, 1982 and 1983, respondent Asistio acquired wealth
in the amounts of P2,142,637.50, P11,463,734.40 and P3,658,351.00, respectively, or
a total of P17,264,722.90, which he deposited in his personal account, CA-4670-
00136-3, in the Republic Planters Bank, Sangandaan Branch, Kalookan City. In
support of their allegations, they attached the original copies of the bank deposits and
receipts which indicated the various sums deposited within the three-year period and
which had been machine validated from January 5, 1981 thru December 20, 1983.
However, in his Sworn Statements of Assets and Liabilities for the period ending
December 31,1982 and December 31, 1984, said respondent had a total income of
only P234,128.68 and P255,324.02, respectively; and as against its total assets (real
and personal properties) of P2,859,327.94 as of December 31, 1982, he had loans
payable in the amount of P2,425,575.60, and against total assets of P5,143,260.98 as
of December 31, 1984, he had loans payable in the amount of P2,660,094.74. The
Preliminary Investigation was conducted by Special Prosecution Officer Margarito P.
Gervacio, Jr., before whom authenticated xerox copies of the original ledger cards of
CA-4670- 00136-3 in the name of respondent Macario Asistio, Jr. were produced and
presented by the officer-in-charge of the Republic Planters Bank, Sangandaan Branch,
Kalookan City.

ISSUE:

Whether or not the Sandiganbayan should be the proper court to try the subject
of the case.

RULING:

NO.

A perusal of the law originally creating the Office of the Ombudsman then (to be
known as the Tanodbayan), and the amendatory laws issued subsequent thereto will
show that, at its inception, the Office of the Ombudsman was already vested with the
power to investigate and prosecute civil and criminal cases before the Sandiganbayan
and even the regular courts.
Nonetheless, while we do not discount the authority of the Ombudsman, we
believe and so hold that the exercise of his correlative powers to both investigate and
initiate the proper action for the recovery of ill-gotten and/or unexplained wealth is
restricted only to cases for the recovery of ill-gotten and/or unexplained wealth which
were amassed after February 25, 1986. 18 Prior to said date, the Ombudsman is
without authority to initiate such forfeiture proceedings. We, however, uphold his
authority to investigate cases for the forfeiture or recovery of such ill-gotten and/or
unexplained wealth amassed even before the aforementioned date, pursuant to his
general investigatory power under Section 15(l) of Republic Act No. 6770.
In the case at bar, the alleged unexplained wealth of respondent Macario
Asistio, Jr. was supposed to have been acquired from 1981 to 1983. Verily, the
Ombudsman, like the Special Prosecutor, is without authority to initiate and file the
petition for forfeiture against respondent Asistio.

348 | P a g e
Law 321_Corporation LAW_ Case Digest

JOHN GOKONGWEI, JR.


vs.
SECURITIES AND EXCHANGE COMMISSION, SAN MIGUEL CORPORATION,
ANDRES M. SORIANO, JOSE M. SORIANO, ENRIQUE ZOBEL, ANTONIO ROXAS,
EMETERIO BUNAO, WALTHRODE B. CONDE, MIGUEL ORTIGAS, EMIGDIO
TANJUATCO and EDUARDO VISAYA
G.R. No. L-52129 April 21, 1980

FACTS:

Petitioner seeks to nullify and set aside the resolution en banc dated May 7,
1979 of respondent Securities and Exchange Commission in SEC Case No. 1375,
sustaining the findings of the San Miguel Corporation's Board of Directors that
petitioner is engaged in a business competitive with or antagonistic to that of the San
Miguel Corporation and, therefore, ineligible for election as director, pursuant to
Section 3, Article III of the amended by-laws. Petitioner alleges that the matter of
petitioner's disqualification should not have been heard in view of the pendency of
petitioner's motion for reconsideration with this Court; that when respondent
Commission sustained the disqualification of petitioner, it failed to consider that
private respondents are precluded from disqualifying petitioner because of the rule of
pari delicto.

ISSUE:

Whether or not respondent SEC gravely abuse its discretion in denying


petitioner's request for an examination of the records of San Miguel International Inc.,
a fully owned subsidiary of San Miguel Corporation.

RULING:

YES.

The stockholder's right of inspection of the corporation's books and records is


based upon their ownership of the assets and property of the corporation. It is,
therefore, an incident of ownership of the corporate property, whether this ownership
or interest be termed an equitable ownership, a beneficial ownership, or a ownership.
This right is predicated upon the necessity of self-protection. It is generally held by
majority of the courts that where the right is granted by statute to the stockholder, it
is given to him as such and must be exercised by him with respect to his interest as a
stockholder and for some purpose germane thereto or in the interest of the
corporation. In other words, the inspection has to be germane to the petitioner's
interest as a stockholder, and has to be proper and lawful in character and not
inimical to the interest of the corporation.
"The right to examine the books of the corporation must be exercised in good
faith, for specific and honest purpose, and not to gratify curiosity, or for specific and
honest purpose, and not to gratify curiosity, or for speculative or vexatious purposes.
The weight of judicial opinion appears to be, that on application for mandamus to
enforce the right, it is proper for the court to inquire into and consider the
stockholder's good faith and his purpose and motives in seeking inspection. Thus, it
was held that "the right given by statute is not absolute and may be refused when the
information is not sought in good faith or is used to the detriment of the corporation."
But the "impropriety of purpose such as will defeat enforcement must be set up the
corporation defensively if the Court is to take cognizance of it as a qualification. In
other words, the specific provisions take from the stockholder the burden of showing
propriety of purpose and place upon the corporation the burden of showing
impropriety of purpose or motive. It appears to be the general rule that stockholders
are entitled to full information as to the management of the corporation and the
manner of expenditure of its funds, and to inspection to obtain such information,
especially where it appears that the company is being mismanaged or that it is being
managed for the personal benefit of officers or directors or certain of the stockholders
to the exclusion of others."

349 | P a g e
Law 321_Corporation LAW_ Case Digest

RAMON A. GONZALES
vs.
THE PHILIPPINE NATIONAL BANK
G.R. No. L-33320 May 30, 1983

FACTS:

Petitioner Ramon A. Gonzales instituted in the erstwhile Court of First Instance


of Manila a special civil action for mandamus against the herein respondent praying
that the latter be ordered to allow him to look into the books and records of the
respondent bank in order to satisfy himself as to the truth of the published reports
that the respondent has guaranteed the obligation of Southern Negros Development
Corporation in the purchase of a US$ 23 million sugar-mill to be financed by Japanese
suppliers and financiers; that the respondent is financing the construction of the P 21
million Cebu-Mactan Bridge to be constructed by V.C. Ponce, Inc., and the
construction of Passi Sugar Mill at Iloilo by the Honiron Philippines, Inc., as well as to
inquire into the validity of Id transactions. The petitioner has alleged hat his written
request for such examination was denied by the respondent. The trial court having
dismissed the petition for mandamus, the instant appeal to review the said dismissal
was filed.

ISSUE:

Whether or not the denial for the request of petitioner for inspection valid.

RULING:

YES.

Although the petitioner has claimed that he has justifiable motives in seeking
the inspection of the books of the respondent bank, he has not set forth the reasons
and the purposes for which he desires such inspection, except to satisfy himself as to
the truth of published reports regarding certain transactions entered into by the
respondent bank and to inquire into their validity. The circumstances under which he
acquired one share of stock in the respondent bank purposely to exercise the right of
inspection do not argue in favor of his good faith and proper motivation. Admittedly he
sought to be a stockholder in order to pry into transactions entered into by the
respondent bank even before he became a stockholder. His obvious purpose was to
arm himself with materials which he can use against the respondent bank for acts
done by the latter when the petitioner was a total stranger to the same. He could have
been impelled by a laudable sense of civic consciousness, but it could not be said that
his purpose is germane to his interest as a stockholder.
We also find merit in the contention of the respondent bank that the inspection
sought to be exercised by the petitioner would be violative of the provisions of its
charter. The Superintendent of Banks and the Auditor General, or other officers
designated by law to inspect or investigate the condition of the National Bank, shall
not reveal to any person other than the President of the Philippines, the Secretary of
Finance, and the Board of Directors the details of the inspection or investigation, nor
shall they give any information relative to the funds in its custody, its current
accounts or deposits belonging to private individuals, corporations, or any other
entity, except by order of a Court of competent jurisdiction. Any director, officer,
employee, or agent of the Bank, who violates or permits the violation of any of the
provisions of this Act, or any person aiding or abetting the violations of any of the
provisions of this Act, shall be punished by a fine not to exceed ten thousand pesos or
by imprisonment of not more than five years, or both such fine and imprisonment.
The Philippine National Bank is not an ordinary corporation. Having a charter
of its own, it is not governed, as a rule, by the Corporation Code of the Philippines.
The provision of Section 74 of Batas Pambansa Blg. 68 of the new Corporation Code
with respect to the right of a stockholder to demand an inspection or examination of
the books of the corporation may not be reconciled with the abovequoted provisions of
the charter of the respondent bank.

350 | P a g e
Law 321_Corporation LAW_ Case Digest

ANTONIO PARDO
vs.
THE HERCULES LUMBER CO., INC., and IGNACIO FERRER
G.R. No. L-22442 August 1, 1924

FACTS:

The petitioner, Antonio Pardo, a stockholder in the Hercules Lumber Company,


Inc., one of the respondents herein, seeks by this original proceeding in the Supreme
Court to obtain a writ of mandamus to compel the respondents to permit the plaintiff
and his duly authorized agent and representative to examine the records and business
transactions of said company. To this petition the respondents interposed an answer,
in which, after admitting certain allegations of the petition, the respondents set forth
the facts upon which they mainly rely as a defense to the petition. To this answer the
petitioner in turn interposed a demurrer, and the cause is now before us for
determination of the issue thus presented.

ISSUE:

Whether or not the respondent have the right to deny inspection request by
petitioner.

RULING:

YES.

The general right given by the statute may not be lawfully abridged to the extent
attempted in this resolution. It may be admitted that the officials in charge of a
corporation may deny inspection when sought at unusual hours or under other
improper conditions; but neither the executive officers nor the board of directors have
the power to deprive a stockholder of the right altogether. A by-law unduly restricting
the right of inspection is undoubtedly invalid. Authorities to this effect are too
numerous and direct to require extended comment. Under a statute similar to our own
it has been held that the statutory right of inspection is not affected by the adoption
by the board of directors of a resolution providing for the closing of transfer books
thirty days before an election.
It will be noted that our statute declares that the right of inspection can be
exercised "at reasonable hours." This means at reasonable hours on business days
throughout the year, and not merely during some arbitrary period of a few days
chosen by the directors.
In addition to relying upon the by-law, to which reference is above made, the
answer of the respondents calls in question the motive which is supposed to prompt
the petitioner to make inspection; and in this connection it is alleged that the
information which the petitioner seeks is desired for ulterior purposes in connection
with a competitive firm with which the petitioner is alleged to be connected. It is also
insisted that one of the purposes of the petitioner is to obtain evidence preparatory to
the institution of an action which he means to bring against the corporation by reason
of a contract of employment which once existed between the corporation and himself.
These suggestions are entirely apart from the issue, as, generally speaking, the motive
of the shareholder exercising the right is immaterial.

351 | P a g e
Law 321_Corporation LAW_ Case Digest

W. G. PHILPOTTS
vs.
PHILIPPINE MANUFACTURING COMPANY and F. N. BERRY
GR. No. L-15568 November 8, 1919

FACTS:

W. G. Philpotts, a stockholder in the Philippine Manufacturing Company, one of


the respondents herein, seeks by this proceeding to obtain a writ of mandamus to
compel the respondents to permit the plaintiff, in person or by some authorized agent
or attorney, to inspect and examine the records of the business transacted by said
company since January 1, 1918. The petition is filed originally in this court under the
authority of section 515 of the Code of Civil Procedure, which gives to this tribunal
concurrent jurisdiction with the Court of First Instance in cases, among others, where
any corporation or person unlawfully excludes the plaintiff from the use and
enjoyment of some right to which he is entitled. The respondents interposed a
demurrer, and the controversy is now before us for the determination of the questions
thus presented.

ISSUE:

Whether or not the right to inspect records and transactions of the corporation
is permitted.

RULING:

YES.

Now it is our opinion, and we accordingly hold, that the right of inspection given
to a stockholder in the provision above quoted can be exercised either by himself or by
any proper representative or attorney in fact, and either with or without the
attendance of the stockholder. This is in conformity with the general rule that what a
man may do in person he may do through another; and we find nothing in the statute
that would justify us in qualifying the right in the manner suggested by the
respondents.
This conclusion is supported by the undoubted weight of authority in the
United States, where it is generally held that the provisions of law conceding the right
of inspection to stockholders of corporations are to be liberally construed and that
said right may be exercised through any other properly authorized person. As was said
in Foster vs. White (86 Ala., 467), "The right may be regarded as personal, in the sense
that only a stockholder may enjoy it; but the inspection and examination may be made
by another.
In order that the rule above stated may not be taken in too sweeping a sense,
we deem it advisable to say that there are some things which a corporation may
undoubtedly keep secret, notwithstanding the right of inspection given by law to the
stockholder; as for instance, where a corporation, engaged in the business of
manufacture, has acquired a formula or process, not generally known, which has
proved of utility to it in the manufacture of its products. It is not our intention to
declare that the authorities of the corporation, and more particularly the Board of
Directors, might not adopt measures for the protection of such process form publicity.
There is, however, nothing in the petition which would indicate that the petitioner in
this case is seeking to discover anything which the corporation is entitled to keep
secret; and if anything of the sort is involved in the case it may be brought out at a
more advanced stage of the proceedings.

352 | P a g e
Law 321_Corporation LAW_ Case Digest

REPUBLIC OF THE PHILIPPINES


vs.
SANDIGANBAYAN (THIRD DIVISION) and MACARIO ASISTIO, JR.
G.R. No. 90529 August 16, 1991

FACTS:

In a Joint Letter-Complaint to the Ombudsman dated January 8,1989 Messrs.


Arnel Blancaflor and Rodolfo Santos, charged respondent Macario Asistio, Jr., who is
the incumbent Mayor of Kalookan City, with having violated the Anti-Graft and
Corrupt Practices Act (R.A. 3019), specifically Section 8 thereof. In said Joint Sworn
Letter-Complaint, they alleged that during his incumbency as Kalookan City Mayor in
1981, 1982 and 1983, respondent Asistio acquired wealth in the amounts of
P2,142,637.50, P11,463,734.40 and P3,658,351.00, respectively, or a total
of P17,264,722.90, which he deposited in his personal account, CA-4670-00136-3, in
the Republic Planters Bank, Sangandaan Branch, Kalookan City. In support of their
allegations, they attached the original copies of the bank deposits and receipts which
indicated the various sums deposited within the three-year period and which had been
machine validated from January 5, 1981 thru December 20, 1983.
However, in his Sworn Statements of Assets and Liabilities for the period ending
December 31,1982 and December 31, 1984 said respondent had a total income of only
P234,128.68 and P255,324.02, respectively; and as against its total assets (real and
personal properties) of P2,859,327.94 as of December 31, 1982, he had loans payable
in the amount of P2,425,575.60, and against total assets of P5,143,260.98 as of
December 31, 1984, he had loans payable in the amount of P2,660,094.74.

ISSUE:

Whether or not the sequestration automatically deprive a stockholder of his


right of inspection.

RULING:

NO.

The right of a stockholder to inspect and/or examine the records of a


corporation is explicitly provided in Section 74 of the Corporation Code. The records of
all business transactions of the corporation and the minutes of any meeting shall be
open to the inspection of any director, trustee, stockholder or member of the
corporation at reasonable hours on business days and he may demand, in writing, for
a copy of excerpts from said records or minutes, at his expense.
One thing is certain, and should be stated at the outset: the PCGG cannot
exercise acts of dominion over property sequestered, frozen or provisionally taken over.
As already earlier stressed with no little insistence, the act of sequestration; freezing or
provisional takeover of property does not import or bring about a divestment of title
over said property; does not make the PCGG the owner thereof. In relation to the
property sequestered, frozen or provisionally taken over, the PCGG is a conservator,
not an owner.
The PCGG does not become, ipso facto, the owner of the shares just because the
same have been sequestered; nor does it become the stockholder of record by virtue of
such sequestration.

353 | P a g e
Law 321_Corporation LAW_ Case Digest

MERGER AND CONSOLIDATION


Effects of Merger or Consolidation

BANK OF THE PHILIPPINE ISLANDS


vs.
BPI EMPLOYEES UNION-DAVAO CHAPTER-FEDERATION OF UNIONS IN BPI
UNIBANK
G.R. No. 164301 August 10, 2010

FACTS:

The Bangko Sentral ng Pilipinas approved the Articles of Merger executed on


January 20, 2000 by and between BPI, herein petitioner, and FEBTC. This Article and
Plan of Merger was approved by the Securities and Exchange Commission on April 7,
2000. Pursuant to the Article and Plan of Merger, all the assets and liabilities of
FEBTC were transferred to and absorbed by BPI as the surviving corporation. FEBTC
employees, including those in its different branches across the country, were hired by
petitioner as its own employees, with their status and tenure recognized and salaries
and benefits maintained. Respondent BPI Employees Union-Davao Chapter -
Federation of Unions in BPI Unibank is the exclusive bargaining agent of BPI‘s rank
and file employees in Davao City. The former FEBTC rank-and-file employees in Davao
City did not belong to any labor union at the time of the merger. Prior to the effectivity
of the merger, or on March 31, 2000, respondent Union invited said FEBTC employees
to a meeting regarding the Union Shop Clause of the existing CBA between petitioner
BPI and respondent Union.
After the meeting called by the Union, some of the former FEBTC employees
joined the Union, while others refused. Later, however, some of those who initially
joined retracted their membership.

ISSUE:

Whether or not the FEBTC employees were automatically absorbed by petitioner


upon the merger between FEBTC and BPI covered by the Union Shop Clause found in
the existing CBA between petitioner and respondent Union.

RULING:

NO.

In legal parlance, however, human beings are never embraced in the term
"assets and liabilities." Moreover, BPI‘s absorption of former FEBTC employees was
neither by operation of law nor by legal consequence of contract. There was no
government regulation or law that compelled the merger of the two banks or the
absorption of the employees of the dissolved corporation by the surviving corporation.
Had there been such law or regulation, the absorption of employees of the non-
surviving entities of the merger would have been mandatory on the surviving
corporation. In the present case, the merger was voluntarily entered into by both
banks presumably for some mutually acceptable consideration. In fact, the
Corporation Code does not also mandate the absorption of the employees of the non-
surviving corporation by the surviving corporation in the case of a merger.
Significantly, too, the Articles of Merger and Plan of Merger dated April 7, 2000
did not contain any specific stipulation with respect to the employment contracts of
existing personnel of the non-surviving entity which is FEBTC. Unlike the Voluntary
Arbitrator, this Court cannot uphold the reasoning that the general stipulation
regarding transfer of FEBTC assets and liabilities to BPI as set forth in the Articles of
Merger necessarily includes the transfer of all FEBTC employees into the employ of
BPI and neither BPI nor the FEBTC employees allegedly could do anything about
it. Even if it is so, it does not follow that the absorbed employees should not be subject
to the terms and conditions of employment obtaining in the surviving corporation.

354 | P a g e
Law 321_Corporation LAW_ Case Digest

PHILIPPINE NATIONAL BANK & NATIONAL SUGAR DEVELOPMENT


CORPORATION
vs.
ANDRADA ELECTRIC & ENGINEERING COMPANY
G.R. No. 142936 April 17, 2002

FACTS:

The plaintiff alleged that it is a partnership duly organized, existing, and


operating under the laws of the Philippines, with office and principal place of business
at Nos. 794-812 Del Monte Avenue, Quezon City, while the Philippine National Bank
(herein referred to as PNB), is a semi-government corporation duly organized, existing
and operating under the laws of the Philippines, with office and principal place of
business at Escolta Street, Sta. Cruz, Manila; whereas, the other defendant, the
National Sugar Development Corporation, is also a semi-government corporation and
the sugar arm of the PNB, with office and principal place of business at the 2nd Floor,
Sampaguita Building.
The plaintiff and the defendant PASUMIL entered into a contract.That out of the
total obligation of P777,263.80, the defendant PASUMIL had paid only P250,000.00,
leaving an unpaid balance, as of June 27, 1973, amounting to P527,263.80, as shown
in the Certification of the chief accountant of the PNB, a machine copy of which is
appended as Annex ‗C‘ of the complaint; that out of said unpaid balance of
P527,263.80, the defendant PASUMIL made a partial payment to the plaintiff of
P14,000.00, in broken amounts, covering the period from January 5, 1974 up to May
23, 1974, leaving an unpaid balance of P513,263.80; that the defendant PASUMIL and
the defendant PNB, and now the defendant NASUDECO, failed and refused to pay the
plaintiff their just, valid and demandable obligation; that the President of the
NASUDECO is also the Vice-President of the PNB, and this official holds office at the
10th Floor of the PNB, Escolta, Manila, and plaintiff besought this official to pay the
outstanding obligation of the defendant PASUMIL, inasmuch as the defendant PNB
and NASUDECO now owned and possessed the assets of the defendant PASUMIL, and
these defendants all benefited from the works, and the electrical, as well as the
engineering and repairs, performed by the plaintiff; that because of the failure and
refusal of the defendants to pay their just, valid, and demandable obligations, plaintiff
suffered actual damages in the total amount of P513,263.80; and that in order to
recover these sums, the plaintiff was compelled to engage the professional services of
counsel, to whom the plaintiff agreed to pay a sum equivalent to 25% of the amount of
the obligation due by way of attorney‘s fees.

ISSUE:

Whether or not PNB liable for the unpaid debts of PASUMIL to respondent.

RULING:

NO.

A consolidation is the union of two or more existing entities to form a new entity
called the consolidated corporation. A merger, on the other hand, is a union whereby
one or more existing corporations are absorbed by another corporation that survives
and continues the combined business.
The merger, however, does not become effective upon the mere agreement of the
constituent corporations. Since a merger or consolidation involves fundamental
changes in the corporation, as well as in the rights of stockholders and creditors, there
must be an express provision of law authorizing them. For a valid merger or
consolidation, the approval by the Securities and Exchange Commission (SEC) of the
articles of merger or consolidation is required. These articles must likewise be duly
approved by a majority of the respective stockholders of the constituent corporations.
In the case at bar, we hold that there is no merger or consolidation with respect
to PASUMIL and PNB. The procedure prescribed under Title IX of the Corporation
Code was not followed.

355 | P a g e
Law 321_Corporation LAW_ Case Digest

CHESTER BABST
vs.
COURT OF APPEALS, BANK OF THE PHILIPPINE ISLANDS, ELIZALDE STEEL
CONSOLIDATED, INC., and PACIFIC MULTI-COMMERCIAL CORPORATION
G.R. No. 99398 & 104625 January 26, 2001

FACTS:

The complaint was commenced principally to enforce payment of a promissory


note and three domestic letters of credit which Elizalde Steel Consolidated, Inc.
(ELISCON) executed and opened with the Commercial Bank and Trust Company
(CBTC). On June 8, 1973, ELISCON obtained from CBTC a loan in the amount of P
8,015,900.84, with interest at the rate of 14% per annum, evidenced by a promissory
note.2 ELISCON defaulted in its payments, leaving an outstanding indebtedness in the
amount of P2,795,240.67 as of October 31, 1982.
The letters of credit, on the other hand, were opened for ELISCON by CBTC
using the credit facilities of Pacific Multi-Commercial Corporation (MULTI) with the
said bank, pursuant to the Resolution of the Board of Directors of MULTI adopted on
August 31, 1977. Subsequently, on September 26, 1978, Antonio Roxas Chua and
Chester G. Babst executed a Continuing Suretyship,5 whereby they bound themselves
jointly and severally liable to pay any existing indebtedness of MULTI to CBTC to the
extent of P8,000,000.00 each.

ISSUE:

Whether or not BPI consent to the assumption by DBP of the obligations of


ELISCON.

RULING:

YES.

Indeed, there exist clear indications that BPI was aware of the assumption by
DBP of the obligations of ELISCON. In fact, BPI admits that the Development Bank of
the Philippines (DBP), for a time, had .proposed a formula for the settlement of
Eliscon's past obligations to its creditors, including the plaintiff [BPI], but the formula
was expressly rejected by the plaintiff as not acceptable (long before the filing of the
complaint at bar).
Indeed, the authority granted by BPI to its account officer to attend the
creditors' meeting was an authority to represent the bank, such that when he failed to
object to the substitution of debtors, he did so on behalf of and for the bank. Even
granting arguendo that the said account officer was not so empowered, BPI could have
subsequently registered its objection to the substitution, especially after it had already
learned that DBP had taken over the assets and assumed the liabilities of ELISCON.
Its failure to do so can only mean an acquiescence in the assumption by DBP of
ELISCON's obligations. As repeatedly pointed out by ELISCON and MULTI, BPI's
objection was to the proposed payment formula, not to the substitution itself.
BPI gives no cogent reason in withholding its consent to the substitution, other
than its desire to preserve its causes of action and legal recourse against the sureties
of ELISCON. It must be remembered, however, that while a surety is solidarily liable
with the principal debtor, his obligation to pay only arises upon the principal debtor's
failure or refusal to pay.
In the case at bar, there was no indication that the principal debtor will default
in payment. In fact, DBP, which had stepped into the shoes of ELISCON, was capable
of payment. Its authorized capital stock was increased by the government. More
importantly, the National Development Company took over the business of ELISCON
and undertook to pay ELISCON's creditors, and earmarked for that purpose the
amount of P4,015,534.54 for payment to BPI. The original obligation having been
extinguished, the contracts of suretyship executed separately by Babst and MULTI,
being accessory obligations, are likewise extinguished.

356 | P a g e
Law 321_Corporation LAW_ Case Digest

ASSOCIATED BANK
vs.
HON. COURT OF APPEALS, PROVINCE OF TARLAC and PHILIPPINE NATIONAL
BANK
G.R. No. 107382/G.R. No. 107612 January 31, 1996

FACTS:

The Province of Tarlac maintains a current account with the Philippine National
Bank (PNB) Tarlac Branch where the provincial funds are deposited. Checks issued by
the Province are signed by the Provincial Treasurer and countersigned by the
Provincial Auditor or the Secretary of the Sangguniang Bayan. A portion of the funds
of the province is allocated to the Concepcion Emergency Hospital. The allotment
checks for said government hospital are drawn to the order of "Concepcion Emergency
Hospital, Concepcion, Tarlac" or "The Chief, Concepcion Emergency Hospital,
Concepcion, Tarlac." The checks are released by the Office of the Provincial Treasurer
and received for the hospital by its administrative officer and cashier.
In January 1981, the books of account of the Provincial Treasurer were post-
audited by the Provincial Auditor. It was then discovered that the hospital did not
receive several allotment checks drawn by the Province. On February 19, 1981, the
Provincial Treasurer requested the manager of the PNB to return all of its cleared
checks which were issued from 1977 to 1980 in order to verify the regularity of their
encashment. After the checks were examined, the Provincial Treasurer learned that 30
checks amounting to P203,300.00 were encashed by one Fausto Pangilinan, with the
Associated Bank acting as collecting bank.
It turned out that Fausto Pangilinan, who was the administrative officer and
cashier of payee hospital until his retirement on February 28, 1978, collected the
questioned checks from the office of the Provincial Treasurer. He claimed to be
assisting or helping the hospital follow up the release of the checks and had official
receipts. Pangilinan sought to encash the first check with Associated Bank.

ISSUE:

Whether or not there is merger in this case.

RULING:

YES.

Ordinarily, in the merger of two or more existing corporations, one of the


combining corporations survives and continues the combined business, while the rest
are dissolved and all their rights, properties and liabilities are acquired by the
surviving corporation. Although there is dissolution of the absorbed corporations,
there is no winding up of their affairs or liquidation of their assets, because the
surviving corporation automatically acquires all their rights, privileges and powers, as
well as their liabilities.
The merger, however, does not become effective upon the mere agreement of the
constituent corporations. The procedure to be followed is prescribed under the
Corporation Code. Section 79 of said Code requires the approval by the Securities and
Exchange Commission (SEC) of the articles of merger which, in turn, must have been
duly approved by a majority of the respective stockholders of the constituent
corporations. The effectivity date of the merger is crucial for determining when the
merged or absorbed corporation ceases to exist; and when its rights, privileges,
properties as well as liabilities pass on to the surviving corporation.
Consistent with the aforementioned Section 79, the September 16, 1975
Agreement of Merger, which Associated Banking Corporation (ABC) and Citizens Bank
and Trust Company (CBTC) entered into, provided that its effectivity "shall, for all
intents and purposes, be the date when the necessary papers to carry out this merger
shall have been approved by the Securities and Exchange Commission."

357 | P a g e
Law 321_Corporation LAW_ Case Digest

ALGER ELECTRIC, INC


vs.
COURT OF APPEALS and NORTHERN CEMENT CORPORATION
G.R. No. L-34298 February 28, 1985

FACTS:

Petitioner Alger Electric, Inc., was granted a legislative franchise for a period of
fifty (50) years from June 22, 1963 with the right, privilege, and authority to construct,
maintain and operate an electric light, heat, and power system for the generation
and/or distribution of electric light, heat, and/or power for sale within the
municipalities of Sto. Tomas, Damortis and Rosario, province of La Union, and in the
municipality of Sison, province of Pangasinan.
Respondent Northern Cement Corporation (Northern) and the National Power
Corporation (NPC) executed a contract for NPC to directly supply electric power to
Northern's cement plant located in Labayog, Sison, Pangasinan. As a result, the
petitioner filed a petition for prohibition with preliminary injunction against Northern
and NPC in the Court of First Instance of Manila. The petition alleged that the contract
was patently illegal.
The appellate court sustained the position of respondent Northern and set aside
the questioned October 24, 1969 order of the trial court. It also ordered the trial court
to act on the respondent's motion to dismiss the case. The appellate court ruled that
the Court of First Instance of Manila did not have jurisdiction over the original
complaint considering that the act sought to be enjoined was to be performed in Sison,
Pangasinan which is outside of the court's territorial jurisdiction. It, therefore, held
that the original "petition" could no longer be amended otherwise it would be in
violation of the legal prohibition of a complaint not amendable in order to confer
jurisdiction on the court in which it is filed, if the cause of action originally set forth
was not within The court's jurisdiction. This decision is now challenged in this
petition.

ISSUE:

Whether or not Northern commit illegal acts when it entered into a contract
with NPC.

RULING:

NO.

We have interpreted monopolistic claims of corporations, which want to protect


themselves through the exclusion of competitors and antagonistic parties, as
necessarily yielding to the higher claims of public interest. This interpretation is even
more called for when the exclusiveness is claimed on the basis of a public franchise.
Section 2 of Republic Act No. 3826 was obviously enacted to prevent the NPC
from distributing or selling electric power where petitioner Alger is already selling or is
able to sell its own self-generated electricity. In this case, Northern is a bulk purchaser
of power. It had never purchase's Alger's electricity before the suit was filed. It is not
the usual consumer — residential or commercial — for whom retail sales are Ideal.
Exclusivity is given by law with the understanding that the company enjoying it is self-
sufficient and capable of supplying the needed service or product at moderate or
reasonable prices. It would be against public interest where the firm granted a
monopoly is merely ail unnecessary conduit of electric power, jacking up prices as a
superfluous middleman or an inefficient producer which cannot supply cheap
electricity to power intensive industries. It is in the public interest when industries
dependent on the heavy use of electricity are given reliable and direct power at the
lowest costs thus enabling the sale of nationally marketed products at prices within
the reach of the masses. Applying the above principles to the specific facts of this case,
Northern cannot be said to have committed an act void ab initio when it concluded the
questioned contract with NPC. Accordingly, the respondent corporation is not liable for
damages to the petitioner.

358 | P a g e
Law 321_Corporation LAW_ Case Digest

COMMISSIONER OF INTERNAL REVENUE


vs.
NORTON and HARRISON COMPANY
G.R. No. L-17618 August 31, 1964

FACTS:

Norton and Harrison is a corporation organized in 1911, (1) to buy and sell at
wholesale and retail, all kinds of goods, wares, and merchandise; (2) to act as agents
of manufacturers in the United States and foreign countries; and (3) to carry on and
conduct a general wholesale and retail mercantile establishment in the Philippines.
Jackbilt is, likewise, a corporation organized on February 16, 1948 primarily for the
purpose of making, producing and manufacturing concrete blocks. Under date of July
27, 1948. Norton and Jackbilt entered into an agreement whereby Norton was made
the sole and exclusive distributor of concrete blocks manufactured by Jackbilt.
Pursuant to this agreement, whenever an order for concrete blocks was received by the
Norton & Harrison Co. from a customer, the order was transmitted to Jackbilt which
delivered the merchandise direct to the customer. Payment for the goods is, however,
made to Norton, which in turn pays Jackbilt the amount charged the customer less a
certain amount, as its compensation or profit. To exemplify the sales procedures
adopted by the Norton and Jackbilt, the following may be cited. In the case of the sale
of 420 pieces of concrete blocks to the American Builders on April 1, 1952, the
purchaser paid to Norton the sum of P189.00 the purchase price. Out of this amount
Norton paid Jackbilt P168.00, the difference obviously being its compensation. As per
records of Jackbilt, the transaction was considered a sale to Norton. It was under this
procedure that the sale of concrete blocks manufactured by Jackbilt was conducted
until May 1, 1953, when the agency agreement was terminated and a management
agreement between the parties was entered into.

ISSUE:

Whether or not the acquisition of all the stocks of the Jackbilt by the Norton &
Harrison Co., merged the two corporations into a single corporation.

RULING:

YES.

It has been settled that the ownership of all the stocks of a corporation by
another corporation does not necessarily breed an identity of corporate interest
between the two companies and be considered as a sufficient ground for disregarding
the distinct personalities. However, in the case at bar, we find sufficient grounds to
support the theory that the separate identities of the two companies should be
disregarded. There was no limit to the advances given to Jackbilt. The income tax
return of Norton for 1954 shows that as President and Treasurer of Norton and
Jackbilt, he received from Norton P56,929.95, but received from Jackbilt the measly
amount of P150.00, a circumstance which points out that remuneration of purported
officials of Jackbilt are deemed included in the salaries they received from Norton. The
same is true in the case of Eduardo Garcia, an employee of Norton but a member of
the Board of Jackbilt. His Income tax return for 1956 reveals that he received from
Norton in salaries and bonuses P4,220.00, but received from Jackbilt, by way of
entertainment, representation, travelling and transportation allowances P3,000.00.
However, in the withholding statement, it was shown that the total of P4,200.00 and
P3,000.00 was received by Garcia from Norton, thus portraying the oneness of the two
companies. The Income Tax Returns of Albert Golden and Dioscoro Ramos both
employees of Norton but board members of Jackbilt, also disclose the game method of
payment of compensation and allowances. The offices of Norton and Jackbilt are
located in the same compound. Payments were effected by Norton of accounts for
Jackbilt and vice versa. Payments were also made to Norton of accounts due or
payable to Jackbilt and vice versa.

359 | P a g e
Law 321_Corporation LAW_ Case Digest

COMMISSIONER OF INTERNAL REVENUE


vs.
VICENTE A. RUFINO, ET AL.
G.R. NOS. L-33665-68 FEBRUARY 27, 1987

FACTS:

The private respondents were the majority stockholders of the Eastern


Theatrical Co., Inc., a corporation organized for a period of twenty-five years
terminating in 1959. It was organized to engage in the business of operating theaters,
opera houses, places of amusement and other related business enterprises, more
particularly the Lyric and Capitol Theaters in Manila. The President of this corporation
during the year in question was Ernesto D. Rufino.
The private respondents are also the majority and controlling stockholders of
another corporation, the Eastern Theatrical Co Inc., which was organized in 1958, for
a term of 50 years. This corporation is engaged in the same kind of business as the
Old Corporation. The General-Manager of this corporation (hereinafter referred to as
the New Corporation) at the time was Vicente A. Rufino.
In a special meeting of stockholders of the Old Corporation it provided for the
continuation of its business after the end of its corporate life, and upon the
recommendation of its board of directors, a resolution was passed authorizing the Old
Corporation to merge with the New Corporation by transferring its business, assets,
goodwill, and liabilities to the latter, which in exchange would issue and distribute to
the shareholders of the Old Corporation one share for each share held by them in the
said Corporation.
It was expressly declared that the merger of the Old Corporation with the New
Corporation was necessary to continue the exhibition of moving pictures at the Lyric
and Capitol Theaters even after the expiration of the corporate existence of the former.

ISSUE:

Whether or not the merger of the two corporations is valid.

RULING:

YES.

The term "merger" or "consolidation," shall be understood to mean: (1) The


ordinary merger or consolidation, or (2) the acquisition by one corporation of all or
substantially all the properties of another corporation solely for stock, provided, that
for a transaction to be regarded as a merger or consolidation, it must be undertaken
for a bona fide business purpose and not solely for the purpose of escaping the burden
of taxation, provided further, that in determining whether a bona fide business
purpose exists, each and every step of the transaction shall be considered and the
whole transaction or series of transactions shall be treated as a single unit.
No taxable gain was derived by the private respondents from the questioned
transaction. Contrary to the claim of the petitioner, there was a valid merger although
the actual transfer of the properties subject of the Deed of Assignment was not made
on the date of the merger. In the nature of things, this was not possible. Obviously, it
was necessary for the Old Corporation to surrender its net assets first to the New
Corporation before the latter could issue its own stock to the shareholders of the Old
Corporation because the New Corporation had to increase its capitalization for this
purpose.
There is no impediment to the exchange of property for stock between the two
corporations being considered to have been effected on the date of the merger. That, in
fact, was the intention, and the reason why the Deed of Assignment was made
retroactive to January 1, 1959. Such retroaction provided in effect that all
transactions set forth in the merger agreement shall be deemed to be taking place
simultaneously on January 1, 1959, when the Deed of Assignment became operative.

360 | P a g e
Law 321_Corporation LAW_ Case Digest

SOLID MANILA CORPORATION


vs.
BIO HONG TRADING., INC. AND COURT OF APPEALS
G.R. NO. 90596 APRIL 8, 1991

FACTS:

The petitioner is the owner of a parcel of land. The same lies in the vicinity of
another parcel, registered in the name of the private respondent corporation. The
private respondent's title came from a prior owner, and in their deed of sale, the
parties thereto reserved as an easement of way. As a consequence, an annotation was
entered in the private respondent's title.
The petitioner claims that ever since, it had (as well as other residents of
neighboring estates) made use of the above private alley and maintained and
contributed to its upkeep, until sometime in 1983, when, and over its protests, the
private respondent constructed steel gates that precluded unhampered use. On
December 6, 1984, the petitioner commenced suit for injunction against the private
respondent, to have the gates removed and to allow full access to the easement.

ISSUE:

Whether or not the court a quo errs in holding that an easement had been
extinguished by merger.

RULING:

NO.

There is no question that an easement, as described in the deed of sale


executed between the private respondent and the seller, had been constituted on the
private respondent's property, and has been annotated. Specifically, the same charged
the private respondent as follows, that the alley shall remain open at all times, and no
obstructions whatsoever shall be placed thereon, that the owner of the lot on which
the alley has been constructed shall allow the public to use the same, and allow the
City to lay pipes for sewer and drainage purposes, and shall not ask for any indemnity
for the use thereof. Its act, therefore, of erecting steel gates across the alley was in
defiance of these conditions and a violation of the deed of sale, and, of course, the
servitude of way.
No genuine merger took place as a consequence of the sale in favor of the
private respondent corporation. According to the Civil Code, a merger exists when
ownership of the dominant and servient estates is consolidated in the same person.
Merger then, as can be seen, requires full ownership of both estates.
However, the servitude in question is a personal servitude, that is to say, one
constituted not in favor of a particular tenement (a real servitude) but rather, for the
benefit of the general public. In a personal servitude, there is therefore no "owner of a
dominant tenement" to speak of, and the easement pertains to persons without a
dominant estate, in this case, the public at large.
Merger presupposes the existence of a prior servient-dominant owner
relationship, and the termination of that relation leaves the easement of no use.
Unless the owner conveys the property in favor of the public, if that is possible, hence,
no genuine merger can take place that would terminate a personal easement. In the
case at bar, the defense of merger is, clearly, not a valid defense, indeed, a sham one,
because merger is not possible, and secondly, the sale unequivocally preserved the
existing easement. In other words, the answer does not, in reality, tender any genuine
issue on a material fact and cannot militate against the petitioner's clear cause of
action.

361 | P a g e
Law 321_Corporation LAW_ Case Digest

NON-STOCK CORPORATIONS
Purposes

CHINESE YOUNG MEN’S CHRISTIAN ASSOCIATION OF THE PHILIPPINE


ISLANDS, ET AL.
vs.
VICTOR CHING AND COURT OF APPEALS
G.R. NO. L-36929 JUNE 18, 1976

FACTS:

Respondent Victor Ching filed an action for mandamus with preliminary


injunction against the herein petitioners. He anchored his action upon the claim that
the Membership Campaign of the Chinese YMCA for 1966 held, only 175 applications
for membership were submitted, canvassed and accepted on the last day of the
membership campaign. Not more than 240 membership applications, as reported,
issue of the Chinese Commercial News, were filed.
It is to be noted that respondent Victor Ching is a member of the Board of
Directors of the Chinese YMCA, while herein petitioners, William Golangco and
Juanito K. Tan, are its president and recording secretary, respectively. In the
campaign for membership for the year 1966, a rivalry had developed between two
groups in the association, one headed by respondent Ching and the other by petitioner
Golangco.
On the last day of the membership campaign, respondent Ching and herein
petitioner Golangco were in the office of the Chinese YMCA. Respondent Ching, after it
was agreed upon that there was going to be no extension of the membership
campaign. After trial, a decision was rendered annulling the 1966 annual
membership campaign of the respondent. On appeal, the appealed decision was
affirmed.

ISSUE:

Whether or not the membership campaign is valid.

RULING:

YES.

175 membership applications were undisputedly filed within the deadline


(including the 75 withdrawn by respondent) and yet the 100 remaining unquestioned
memberships were nullified by the questioned decision without the individuals
concerned ever having been impleaded or heard (except the individual petitioners
president and secretary).
The appealed decision thus contravened the established principle that the
courts cannot strip a member of a non-stock non-profit corporation of his membership
therein without cause. Otherwise, that would be an unwarranted and undue
interference with the well-established right of a corporation to determine its
membership. In order that membership may be acquired in a non-stock corporation
and valid by-laws must be complied with, except in so far as they may be and are
waived. But provisions in the by-laws as to formal steps to be taken to acquire
membership may be waived by the corporation, or it may be estopped to assert that
they have not been taken.
Finally, the appealed decision did not give due importance to the undisputed
fact therein stated that "at the board meeting of the association held on December 7,
1965, a list of 174 applications for membership, old and new, was submitted to the
board and approved by the latter, over the objection of the petitioner who was present
at said meeting." Such action of the petitioner association's board of directors
approving the 174 membership applications of old and new members constituting its
active membership as duly processed and screened by the authorized committee just
be deemed a waiver on its part of any technicality or requirement of form.

362 | P a g e
Law 321_Corporation LAW_ Case Digest

THE COLLECTOR OF INTERNAL REVENUE


vs.
THE CLUB FILIPINO, INC. DE CEBU
G.R. No. L-12719 May 31, 1962

FACTS:

The "Club Filipino, Inc. de Cebu," is a civic corporation organized under the
laws of the Philippines with an original authorized capital stock of P22,000.00, which
was subsequently increased to P200,000.00. Neither in the articles or by-laws is there
a provision relative to dividends and their distribution, although it is covenanted that
upon its dissolution, the Club's remaining assets, after paying debts, shall be donated
to a charitable Philippine Institution in Cebu.
The Club owns and operates a club house, a bowling alley, a golf course (on a
lot leased from the government), and a bar-restaurant where it sells wines and liquors,
soft drinks, meals and short orders to its members and their guests. The bar-
restaurant was a necessary incident to the operation of the club and its golf-course.
The club is operated mainly with funds derived from membership fees and dues.
Whatever profits it had, were used to defray its overhead expenses and to improve its
golf-course. In 1951. as a result of a capital surplus, arising from the re-valuation of
its real properties, the value or price of which increased, the Club declared stock
dividends; but no actual cash dividends were distributed to the stockholders. In 1952,
a BIR agent discovered that the Club has never paid percentage tax on the gross
receipts of its bar and restaurant, although it secured B-4, B-9(a) and B-7 licenses. In
a letter dated December 22, 1852, the Collector of Internal Revenue assessed against
and demanded from the Club certain amount of tax. The Club wrote the Collector,
requesting for the cancellation of the assessment. The request having been denied, the
Club filed the instant petition for review.

ISSUE:

Whether or not the respondent Club liable for the payment of the sum of
12,068.84, as fixed and percentage taxes and surcharges prescribed in sections 182,
183 and 191 of the Tax Code, under which the assessment was made, in connection
with the operation of its bar and restaurant.

RULING:

NO.

Having found as a fact that the Club was organized to develop and cultivate
sports of all class and denomination, for the healthful recreation and entertainment of
its stockholders and members; that upon its dissolution, its remaining assets, after
paying debts, shall be donated to a charitable Philippine Institution in Cebu; that it is
operated mainly with funds derived from membership fees and dues; that the Club's
bar and restaurant catered only to its members and their guests; that there was in
fact no cash dividend distribution to its stockholders and that whatever was derived
on retail from its bar and restaurant was used to defray its overall overhead expenses
and to improve its golf-course (cost-plus-expenses-basis), it stands to reason that the
Club is not engaged in the business of an operator of bar and restaurant.
The facts that the capital stock of the respondent Club is divided into shares,
does not detract from the finding of the trial court that it is not engaged in the
business of operator of bar and restaurant. What is determinative of whether or not
the Club is engaged in such business is its object or purpose, as stated in its articles
and by-laws. It is a familiar rule that the actual purpose is not controlled by the
corporate form or by the commercial aspect of the business prosecuted, but may be
shown by extrinsic evidence, including the by-laws and the method of operation.

363 | P a g e
Law 321_Corporation LAW_ Case Digest

Voting

ANTONIO LITONJUA and ARNOLD LITONJUA


vs.
THE HON. COURT OF APPEALS, ET. AL.
G.R. No. 120294 February 10, 1998

FACTS:

Wack Wack Golf and Country Club is a non-profit corporation which offers
sports, recreational and social activities to its members. Petitioner Antonio Litonjua is
an Associate Member of said corporation and his son, co-petitioner Arnold Litonjua, is
a Junior Member thereof. The individual respondents are the members of the Board of
Directors and Membership Committee of Wack Wack. On 10 January 1985, pursuant
to its by-laws, respondent club posted the monthly list of delinquent members on its
premises. Included therein was petitioner Antonio Litonjua.
After Antonio Litonjua discovered that his name was on the January 1985
delinquent list, he proceeded to the Cashier's Office of the club and was informed
therein that the reason behind his delinquency was his failure to pay his November
1984 dues (which should have been paid before the end of December 1984 as provided
in the corporate by-laws). Antonio Litonjua alleged that he was not able to pay his
monthly bill on time because he has not received his statement of account for
November 1984. As proof, he presented a sealed envelope which he allegedly presumed
to be the November 1984 bill (but was actually the December 1984 statement of
account) and explained that he received it only on 12 January 1985.
A check with the accounting office, however, revealed that the November 1984
statement of account had already been delivered to Antonio Litonjua's office and was
received by his employee allegedly named "Aquino." Petitioner asserted that, he did not
receive said account and had no employee by the name of "Aquino." Based on the
foregoing, Antonio Litonjua was able to convince the auxiliary clerks in the Cashier's
Office to delete his name from the list of delinquent members. Consequently, Antonio
Litonjua continued to avail of the club facilities. Later, Antonio Litonjua was advised of
another outstanding balance in the amount of P9,414.00. Again, he issued a check in
payment thereof. As a result, his name was deleted from the February 1985 list of
delinquent members.

ISSUE:

Whether or not the statement of account for November 1984 was duly delivered
to and received by Antonio Litonjua's office on 12 December 1984.

RULING:

NO.

According to Mr. Limbo's testimony on record, the Court failed to find therein
any statement that he delivered the November 1984 account to Antonio Litonjua
himself. Mr. Limbo was consistent in his testimony to the effect that on 12 December
1984 he delivered the November 1984 statement of account at the office of Antonio
Litonjua and it was received by an employee of the latter who signed the Special
Delivery Receipt. On cross-examination, Mr. Limbo did not waver from his testimony
that Antonio Litonjua's November 1984 bill was duly received by the latter's
employee.Against the testimony of Mr. Victor Limbo, coupled with documentary
evidence in the form of the signed Special Delivery Receipt, petitioners presented no
proof other than the bare denial of Antonio Litonjua that he never received his
statement of account for November 1984 and that he has no "Aquino" in his employ.
Petitioners could have readily offered in evidence a record or list of Antonio Litonjua's
employees to prove that he has no employee by the name of "Aquino" but, strangely,
beyond his mere say-so no such evidence was adduced.

364 | P a g e
Law 321_Corporation LAW_ Case Digest

THE PHILIPPINE PUBLIC SCHOOL TEACHERS ASSOCIATION (PPSTA)


COMMISSION ON ELECTIONS
vs.
Honorable SERGIO A. F. APOSTOL
G.R. No. L-36966 February 28, 1974

FACTS:

On July 20, 1972, private respondent Eufemia M. San Luis as a member of the
Philippine Public School Teachers Association (PPSTA), a fraternal non-stock
association of public school teachers throughout the country, filed with respondent
court of first instance at Quezon City a complaint with preliminary injunction for the
annulment of the 1972 annual elections of the PPSTA board of directors held on June
26-28, 1972 at Teachers Camp in Baguio City for having been held outside its
principal office at Quezon City against herein petitioners as defendants.

ISSUE:

Whether or not the elections of the Board of Directors are null and void.

RULING:

NO.

The Court finds it unnecessary to rule upon the parties' above conflicting
contentions, since it finds to be decisive petitioners' contention that respondent has no
personality and standing as a single individual member out of thousands of members
of the PPSTA to bring the action below for annulment of the PPSTA 1972 annual
convention and elections, as she was not even a chapter delegate to the said
convention and she was duly represented thereat in accordance with the PPSTA's by-
laws by her duly authorized chapter delegates who have raised no question as to the
proceedings.Article IX, section 5 of the by-laws expressly provides that "only official
delegates to the representative assembly are entitled to take part in the discussions
and to vote."
Respondent's action below was in essence one of quo warranto which is
governed by Rule 66 of the Rules of Court Section 6 thereof provides that in order that
an individual may directly bring the action, he or she must claim to entitled to the
public office or position allegedly unlawfully held or usurped.Otherwise, the action
must be brought by the Solicitor General or fiscal with leave of the court upon the
complaint of the realtor under section 4 of the Rule.
The general rule is that actions for quo warranto should be brought by the
Solicitor General or a fiscal in cases of usurpation of an office established by law or by
the Constitution under color of an executive appointment, or the abuse of a public
franchise under color of a legislative grant, for these are public wrongs and not private
injuries. Since, under our system all power emanates from the people, who constitute
the sovereignty, the right to inquire into the authority by which a person assumes to
exercise the functions of a public office or franchise is regarded as inherent in the
people on the right their sovereignty. Hence, the action should be brought by the
Solicitor General or the fiscal who represents the sovereign power.
Respondent manifestly lays no claim herself to the office of PPSTA director nor
has the present action been filed with leave of court by the Solicitor General or fiscal
upon her relation as a party having an interest injuriously affected, as required by the
cited Rule. Her action must therefore fail on this score and the judgment erroneously
rendered by respondent court shall be set aside.

365 | P a g e
Law 321_Corporation LAW_ Case Digest

CLOSE CORPORATIONS
Requirements for Formation

MANUEL R. DULAY ENTERPRISES, INC


vs.
THE HONORABLE COURT OF APPEALS
G.R. No. 91889 August 27, 1993

FACTS:

Manuel R. Dulay Enterprises, Inc, a domestic corporation with the following as


members of its Board of Directors: Manuel R. Dulay with 19,960 shares and
designated as president, treasurer and general manager, Atty. Virgilio E. Dulay with
10 shares and designated as vice-president; Linda E. Dulay with 10 shares; Celia
Dulay-Mendoza with 10 shares; and Atty. Plaridel C. Jose with 10 shares and
designated as secretary, owned a property covered by TCT No. 17880 and known as
Dulay Apartment consisting of sixteen (16) apartment units on a six hundred eighty-
nine (689) square meters lot, more or less, located at Seventh Street (now Buendia
Extension) and F.B. Harrison Street, Pasay City.
Petitioner corporation through its president, Manuel Dulay, obtained various
loans for the construction of its hotel project, Dulay Continental Hotel (now Frederick
Hotel). It even had to borrow money from petitioner Virgilio Dulay to be able to
continue the hotel project. As a result of said loan, petitioner Virgilio Dulay occupied
one of the unit apartments of the subject property since property since 1973 while at
the same time managing the Dulay Apartment at his shareholdings in the corporation
was subsequently increased by his father.
Manuel Dulay by virtue of Board Resolution petitioner corporation sold the
subject property to private respondents spouses Maria Theresa and Castrense Veloso
in the amount of P300,000.00 as evidenced by the Deed of Absolute
Sale.Subsequently, private respondent Maria Veloso, without the knowledge of Manuel
Dulay, mortgaged the subject property to private respondent Manuel A. Torres for a
loan of P250,000.00 which was duly annotated.Upon the failure of private respondent
Maria Veloso to pay private respondent Torres, the subject property was sold on April
5, 1978 to private respondent Torres as the highest bidder in an extrajudicial
foreclosure sale as evidenced by the Certificate of Sheriff's Sale issued on April 20,
1978.

ISSUE:

Whether or not the doctrine of piercing the veil of corporate entity is applicable.

RULING:

NO.

Petitioner Corporation is classified as a close corporation and consequently a


board resolution authorizing the sale or mortgage of the subject property is not
necessary to bind the corporation for the action of its president. At any rate, corporate
action taken at a board meeting without proper call or notice in a close corporation is
deemed ratified by the absent director unless the latter promptly files his written
objection with the secretary of the corporation after having knowledge of the meeting
which, in his case, petitioner Virgilio Dulay failed to do.
It is relevant to note that although a corporation is an entity which has a
personality distinct and separate from its individual stockholders or members,the veil
of corporate fiction may be pierced when it is used to defeat public convenience justify
wrong, protect fraud or defend crime. The privilege of being treated as an entity
distinct and separate from its stockholder or members is therefore confined to its
legitimate uses and is subject to certain limitations to prevent the commission of fraud
or other illegal or unfair act. When the corporation is used merely as an alter ego or
business conduit of a person, the law will regard the corporation as the act of that
person.

366 | P a g e
Law 321_Corporation LAW_ Case Digest

SAN JUAN STRUCTURAL AND STEEL FABRICATORS, INC.,


vs.
COURT OF APPEALS, et.al.
G.R. No. 129459 September 29, 1998

FACTS:

San Juan Structural and Steel Fabricators, Inc.'s amended complaint alleged
that on 14 February 1989, plaintiff-appellant entered into an agreement with
defendant-appellee Motorich Sales Corporation for the transfer to it of a parcel of land.
On March 1, 1989. Mr. Andres T. Co, president of plaintiff-appellant corporation,
wrote a letter to defendant-appellee Motorich Sales Corporation requesting for a
computation of the balance to be paid: that said letter was coursed through
defendant-appellee's broker. Linda Aduca, who wrote the computation of the balance:
that on March 2, 1989, plaintiff-appellant was ready with the amount corresponding
to the balance, covered by Metrobank Cashier's Check No. 004223, payable to
defendant-appellee Motorich Sales Corporation; that plaintiff-appellant and defendant-
appellee Motorich Sales Corporation were supposed to meet in the office of plaintiff-
appellant but defendant-appellee's treasurer, Nenita Lee Gruenberg, did not appear;
that defendant-appellee Motorich Sales Corporation despite repeated demands and in
utter disregard of its commitments had refused to execute the Transfer of Rights/Deed
of Assignment which is necessary to transfer the certificate of title.

ISSUE:

Whether or not the doctrine of piercing the veil of corporate fiction be applied to
Motorich.

RULING:

NO.

First, petitioner itself concedes having raised the issue belatedly, not having
done so during the trial, but only when it filed its sur-rejoinder before the Court of
Appeals. Thus, this Court cannot entertain said issue at this late stage of the
proceedings. It is well-settled the points of law, theories and arguments not brought to
the attention of the trial court need not be, and ordinarily will not be, considered by a
reviewing court, as they cannot be raised for the first time on appeal. Allowing
petitioner to change horses in midstream, as it were, is to run roughshod over the
basic principles of fair play, justice and due process.
Second, even if the above mentioned argument were to be addressed at this
time, the Court still finds no reason to uphold it. True, one of the advantages of a
corporate form of business organization is the limitation of an investor's liability to the
amount of the investment. This feature flows from the legal theory that a corporate
entity is separate and distinct from its stockholders. However, the statutorily granted
privilege of a corporate veil may be used only for legitimate purposes. On equitable
considerations, the veil can be disregarded when it is utilized as a shield to commit
fraud, illegality or inequity; defeat public convenience; confuse legitimate issues; or
serve as a mere alter ego or business conduit of a person or an instrumentality,
agency or adjunct of another corporation.

367 | P a g e
Law 321_Corporation LAW_ Case Digest

SERGIO F. NAGUIAT
vs.
NATIONAL LABOR RELATIONS COMMISSION
G.R. No. 116123 March 13, 1997

FACTS:

CFTI held a concessionaire's contract with the Army Air Force Exchange
Services ("AAFES") for the operation of taxi services within Clark Air Base. Sergio F.
Naguiat was CFTI's president, while Antolin T. Naguiat was its vice-president. Like
Sergio F. Naguiat Enterprises, Incorporated a trading firm, it was a family-owned
corporation.
Individual respondents were previously employed by CFTI as taxicab drivers.
During their employment, they were required to pay a daily "boundary fee" in the
amount of US$26.50 for those working from 1:00 a.m. to 12:00 noon, and US$27.00
for those working from 12:00 noon to 12:00 midnight. All incidental expenses for the
maintenance of the vehicles they were driving were accounted against them, including
gasoline expenses. The drivers worked at least three to four times a week, depending
on the availability of taxicabs. They earned not less than US$15.00 daily. Due to the
phase-out of the US military bases in the Philippines, from which Clark Air Base was
not spared, the AAFES was dissolved, and the services of individual respondents were
officially terminated on November 26, 1991.

ISSUE:

Whether or not Sergio F. Naguiat Enterprises, Inc. is a separate and distinct


juridical entity which cannot be held jointly and severally liable for the obligations of
CFTI.

RULING:

YES.

From the evidence proffered by both parties, there is no substantial basis to


hold that Naguiat Enterprises is an indirect employer of individual respondents much
less a labor only contractor. On the contrary, petitioners submitted documents such
as the drivers' applications for employment with CFTI, and social security remittances
and payroll of Naguiat Enterprises showing that none of the individual respondents
were its employees. Moreover, in the contract between CFTI and AAFES, the former, as
concessionaire, agreed to purchase from AAFES for a certain amount within a
specified period a fleet of vehicles to be "kept on the road" by CFTI, pursuant to their
concessionaire's contract. This indicates that CFTI became the owner of the taxicabs
which became the principal investment and asset of the company.
Private respondents failed to substantiate their claim that Naguiat Enterprises
managed, supervised and controlled their employment. It appears that they were
confused on the personalities of Sergio F. Naguiat as an individual who was the
president of CFTI, and Sergio F. Naguiat Enterprises, Inc., as a separate corporate
entity with a separate business. They presumed that Sergio F. Naguiat, who was at the
same time a stockholder and director of Sergio F. Naguiat Enterprises, Inc., was
managing and controlling the taxi business on behalf of the latter. A closer scrutiny
and analysis of the records, however, evince the truth of the matter: that Sergio F.
Naguiat, in supervising the taxi drivers and determining their employment terms, was
rather carrying out his responsibilities as president of CFTI. Hence, Naguiat
Enterprises as a separate corporation does not appear to be involved at all in the taxi
business. From the foregoing, the ineludible conclusion is that CFTI was the actual
and direct employer of individual respondents, and that Naguiat Enterprises was
neither their indirect employer nor labor-only contractor. It was not involved at all in
the taxi business.

368 | P a g e
Law 321_Corporation LAW_ Case Digest

SPECIAL CORPORATIONS
Art. IV, Sec. 28 (3) and Art. 29 (2), 1987 Constitution

REPUBLIC OF THE PHILIPPINES


vs.
THE HONORABLE INTERMEDIATE APPELLATE COURT
G.R. No. L-68303 January 15, 1988

FACTS:

The properties in dispute number three undivided lots and Lot No. 2410-B, Psd-
864 (Lot 2461 Cad 99)] altogether consisting of a total of 1,024 hectares of ricelands.
They are all located in Tiptipon, Panamao, Sulu. The title thereto stood allegedly in the
name of Sultan Jamalul Kiram, who died in 1936. The private respondent, a niece of
the late Sultan, now claims that the original certificate of title thereto was destroyed
as a consequence of a fire that gutted the office of the Register of Deeds of Sulu
sometime in February, 1974. She likewise alleges that the owner's copy thereof was
lost on account of the same misfortune. On October 18,1979, she went to the then
Court of First Instance of Sulu, Branch I, at Jolo, now Regional Trial Court, the
Honorable Jainal D. Rasul, District Judge, presiding, for reconstitution.
The Solicitor General presented in the trial court no opposition to the
application, and based on the evidence of the private respondent, the assailed order
was issued on June 4, 1980. The Solicitor General appealed to the then Intermediate
Appellate Court, now Court of Appeals, which however affirmed in toto, on May 24,
1984, the order of the trial court.

ISSUE:

Whether or not the Petition be granted.

RULING:

YES.

It is not disputed, to begin with, that the notices (of hearing) were not posted on
the main entrances of the provincial and municipal halls of the locality in which the
lands are located. Under Section 13, of Republic Act No. 26: The court shall cause a
notice of the petition, filed under the preceding section, to be published, at the
expense of the petitioner, twice issues of the Official Gazette, and to be posted on the
main of the municipality or city in which the land is situated, at the provincial
building and of the municipal building at least thirty days prior to the date of hearing.
The court shall likewise cause a copy of the notice to be sent, by registered mail or
otherwise, at the expense of the petitioner, to every person named therein whose
address is known, at least thirty days prior to the date of hearing. Said notice shall
state, among other things, the number of the lost or destroyed certificate of title, if
known, the name of the registered owner, the names of the occupants or persons in
possession of the property, the owners of the adjoining properties and all other
interested parties, the location, area and boundaries of the property, and the date on
which all persons having any interest therein must appear and file their claim or
objections to the petition. The petitioner shall, at the hearing, submit proof of the
publication, posting and service of the notice as directed by the court.
We have held that such a mode of publication is a jurisdictional requirement.
The failure on the part of the applicant to comply with it confers no jurisdiction upon
the court. Neither is there any showing that the adjacent owners or other interested
parties were actually notified of the pending application. This too taints the petition
with a jurisdictional defect. It is not enough that there is publication in the Official
Gazette. Publication of the notice in the Official. The Republic cannot be faulted for
nursing doubts about the private respondent's assertions. In the first place, the
private respondent claims that two deeds have been lost.

369 | P a g e
Law 321_Corporation LAW_ Case Digest

THE DIRECTOR OF LANDS


vs.
THE HONORABLE COURT OF APPEALS and IGLESIA NI CRISTO
G.R. No. L-56613 March 14, 1988

FACTS:

On November 28, 1973, private respondent Iglesia ni Cristo filed an application


with the then Court of First Instance of Cavite for registration in its name of a parcel of
land with an area of 379 square meters located at Poblacion, Municipality of Amadeo,
Cavite. In said application, private respondent alleged inter alia that it was the owner
in fee simple of the land afore-described, having acquired title thereto by virtue of a
Deed of Absolute Sale executed in 1947 by Aquelina de la Cruz in its favor and that
applicant and its predecessors-in-interest had been in actual, continuous, public,
peaceful and adverse possession and occupation of said land in the concept of owner
for more than thirty [30] years. Private respondent prayed that should the Land
Registration Act not be applicable, the provisions of Chapter VIII of Commonwealth Act
No. 141, as amended by Republic Act No. 6236 be applied as applicant and its
predecessors-in-interest had been in possession of the land for more than thirty [30]
years and had introduced improvements thereon, including the fencing thereof on all
sides.
The Republic of the Philippines, represented by the Director of Lands, opposed
the application on the following grounds: 1] the applicant and its predecessors-in-
interest did not possess sufficient title to acquire ownership in fee simple of the parcel
of land applied for; 2] neither the applicant nor its predecessors-in-interest have been
in open, continuous, exclusive and notorious possession and occupation of the land in
question; and, 3] the subject parcel of land is a portion of the public domain belonging
to the Republic of the Philippines not subject to private appropriation.

ISSUE:

Whether or not the Petition be granted.

RULING:

NO.

Petitioner's heavy reliance on the case of Director of lands v. Reyes, 68 SCRA


177, is misplaced. The original tracing cloth plan was deemed essential in that case as
the lands involved were vast tracts of uncultivated, mountainous and thickly forested
lands which were necessarily difficult to Identify, unlike the land subject matter of the
instant registration case which is more readily Identifiable by reason of its location, its
comparatively smaller size of 379 square meters as well as the chapel constructed
thereon by private respondent in 1968. Moreover, the documentary evidence presented
therein consisting in the blue-prints of two [2] survey plans were not approved by the
Director of Lands unlike Exhibit "O" which bore the approval of the Land Registration
Commission at the time it was empowered by law to approve original survey plans and
which was re- verified and approved by the Bureau of Lands when the authority to
approve original survey plans was withdrawn from the Land Registration Commission
by P.D. No. 239.

370 | P a g e
Law 321_Corporation LAW_ Case Digest

Corporation Aggregate

IGLESIA EVANGELICA METODISTA EN LAS ISLAS FILIPINAS (IEMELIF)


(Corporation Sole), INC., et al
vs.
BISHOP NATHANAEL LAZARO, et al
G.R. No. 184088 July 6, 2010

FACTS:

Bishop Nicolas Zamora established the petitioner Iglesia Evangelica Metodista


En Las Islas Filipinas, Inc. (IEMELIF) as a corporation sole with Bishop Zamora acting
as its "General Superintendent." Thirty-nine years later in 1948, the IEMELIF enacted
and registered a by-laws that established a Supreme Consistory of Elders (the
Consistory), made up of church ministers, who were to serve for four years. The by-
laws empowered the Consistory to elect a General Superintendent, a General
Secretary, a General Evangelist, and a Treasurer General who would manage the
affairs of the organization. For all intents and purposes, the Consistory served as the
IEMELIF‘s board of directors.
Apparently, although the IEMELIF remained a corporation sole on paper (with
all corporate powers theoretically lodged in the hands of one member, the General
Superintendent), it had always acted like a corporation aggregate. The Consistory
exercised IEMELIF‘s decision-making powers without ever being challenged.
Subsequently, during its 1973 General Conference, the general membership voted to
put things right by changing IEMELIF‘s organizational structure from a corporation
sole to a corporation aggregate. On May 7, 1973 the Securities and Exchange
Commission (SEC) approved the vote. For some reasons, however, the corporate
papers of the IEMELIF remained unaltered as a corporation sole. Only in 2001, about
28 years later, did the issue reemerge. In answer to a query from the IEMELIF, the
SEC replied on April 3, 2001 that, although the SEC Commissioner did not in 1948
object to the conversion of the IEMELIF into a corporation aggregate, that conversion
was not properly carried out and documented.

ISSUE:

Whether or not a corporation sole be converted into a corporation aggregate by


mere amendment of its articles of incorporation.

RULING:

YES.

The Corporation Code provides no specific mechanism for amending the articles
of incorporation of a corporation sole. But, as the RTC correctly held, Section 109 of
the Corporation Code allows the application to religious corporations of the general
provisions governing non-stock corporations.
Although a non-stock corporation has a personality that is distinct from those
of its members who established it, its articles of incorporation cannot be amended
solely through the action of its board of trustees. The amendment needs the
concurrence of at least two-thirds of its membership. If such approval mechanism is
made to operate in a corporation sole, its one member in whom all the powers of the
corporation technically belongs, needs to get the concurrence of two-thirds of its
membership. The one member, here the General Superintendent, is but a trustee,
according to Section 110 of the Corporation Code, of its membership. There is no point
to dissolving the corporation sole of one member to enable the corporation aggregate to
emerge from it. Whether it is a non-stock corporation or a corporation sole, the
corporate being remains distinct from its members, whatever be their number. The
increase in the number of its corporate membership does not change the complexion
of its corporate responsibility to third parties. The one member, with the concurrence
of two-thirds of the membership of the organization for whom he acts as trustee, can
self-will the amendment.

371 | P a g e
Law 321_Corporation LAW_ Case Digest

G.R. No. 172447 September 18, 2009


IGLESIA EVANGELICA METODISTA EN LAS ISLAS FILIPINAS (IEMELIF),
INC., Petitioner,
vs.
NATANAEL B. JUANE, Respondent.
x - - - - - - - - - - - - - - - - - - - - - - -x
G.R. No. 179404
NATANAEL B. JUANE, Petitioner,
vs.
IGLESIA EVANGELICA METODISTA EN LAS ISLAS FILIPINAS (IEMELIF),
INC., Respondent.
DECISION

FACTS:

IEMELIF is a religious corporation existing and duly organized under Philippine


laws. [Juane] is a former minister or pastor of IEMELIF. He was elected as one of the
members of the Highest Consistory of Elders (or Board of Trustees) of IEMELIF in the
February 2000 IEMELIF General Conference. During the concluding Anniversary
Service of said General Conference, IEMELIF Bishop Nathanael P. Lazaro, the General
Superintendent of the whole IEMELIF Church and the General Administrator of the
IEMELIF Cathedral in Tondo, Manila, during the reading of the "IEMELIF Workers‘
Assignment", announced the appointment and assignment of Juane as Resident
Pastor of the Cathedral Congregation in Tondo, Manila. By virtue and as a
consequence of such appointment, Defendant Rev. Juane was authorized to stay at
and occupy the Resident Pastor‘s residence inside the Cathedral complex. By the same
reason, he also took charge of the Cathedral facilities and other property of the church
in said premises.

ISSUE:

Whether or not the transformation of IEMELF from corporation sole to an


aggregate one is valid.

RULING:

YES.

Juane maintains that the "IEMELIF" that filed the Complaint before the MeTC
had no personality to eject him from the subject property. The Church has remained a
corporation sole, since its transformation to a corporation aggregate was legally
defective. Juane, thus, claims that he is now the corporation sole, who is entitled to
the physical possession of the subject property as owner thereof. In fact, on the basis
of these same arguments.
Even if the transformation of IEMELIF from a corporation sole to a corporation
aggregate was legally defective, its head or governing body, i.e., Bishop Lazaro, whose
acts were approved by the Highest Consistory of Elders, still did not change. A
corporation sole is one formed by the chief archbishop, bishop, priest, minister, rabbi
or other presiding elder of a religious denomination, sect, or church, for the purpose of
administering or managing, as trustee, the affairs, properties and temporalities of
such religious denomination, sect or church. As opposed to a corporation aggregate, a
corporation sole consists of a single member, while a corporation aggregate consists of
two or more persons. If the transformation did not materialize, the corporation sole
would still be Bishop Lazaro, who himself performed the questioned acts of removing
Juane as Resident Pastor of the Tondo Congregation. If the transformation did
materialize, the corporation aggregate would be composed of the Highest Consistory of
Elders, which nevertheless approved the very same acts. As either Bishop Lazaro or
the Highest Consistory of Elders had the authority to appoint Juane as Resident
Pastor of the IEMELIF Tondo Congregation, it also had the power to remove him as
such or transfer him to another congregation.

372 | P a g e
Law 321_Corporation LAW_ Case Digest

DISSOLUTION OF CORPORATIONS
Where Creditors are Not Affected

TEODORO B. VESAGAS, AND WILFRED D. ASIS


vs.
THE HONORABLE COURT OF APPEALS
G.R. NO. 142924 DECEMBER 5, 2001

FACTS:

The respondent spouses Delfino and Helenda Raniel are members in good
standing of the Luz Villaga Tennis Clud, Inc. (club). They alleged that petitioner
Teodoro B. Vesagas, who claims to be the club's duly elected president, in conspiracy
with petitioner Wilfred D. Asis, who, in turn, claims to be its duly elected vice-
president and legal counsel, summarily stripped them of their lawful membership,
without due process of law. Thereafter, respondent spouses filed a Complaint with the
Securities and Exchange Commission (SEC) on March 26, 1997 against the
petitioners. It was docketed as SEC Case No. 03-97-5598.In this case, respondents
asked the Commission to declare as illegal their expulsion from the club as it was
allegedly done in utter disregard of the provisions of its by-laws as well as the
requirements of due process. They likewise sought the annulment of the amendments
to the by-laws made on December 8, 1996, changing the annual meeting of the club
from the last Sunday of January to November and increasing the number of trustees
from nine to fifteen. Finally, they prayed for the issuance of a Temporary Restraining
Order and Writ of Preliminary Injunction. The application for TRO was denied by SEC
Hearing Officer Soller in an Order dated April 29, 1997.

ISSUE:

Whether or not SEC has jurisdiction.

RULING:

YES.

Petitioners' attempt to impress upon this court that the club has never been a
corporation is devoid of merit. It must fail in the face of the Commission's explicit
finding that the club was duly registered and a certificate of incorporation was issued
in its favor. It ought to be remembered that the question of whether the club was
indeed registered and issued a certification or not is one which necessitates a factual
inquiry. On this score, the finding of the Commission, as the administrative agency
tasked with among others the function of registering and administering corporations,
is given great weight and accorded high respect. We therefore have no reason to
disturb this factual finding relating to the club's registration and incorporation.
Moreover, by their own admission contained in the various pleadings which
they have filed in the different stages of this case, petitioners themselves have
considered the club as a corporation. This admission, under the rules of evidence,
binds them and may be taken or used against them. Since the admission was made in
the course of the proceedings in the same case, it does not require proof, and actually
may be contradicted only by showing that it was made through palpable mistake or
that no such admission was made.

373 | P a g e
Law 321_Corporation LAW_ Case Digest

Where Creditors are Affected

AVON DALE GARMENTS, INC.


vs.
NATIONAL LABOR RELATIONS COMMISSION, ET.AL.
G.R. No. 117932 July 20, 1995

FACTS:

Private respondents were employees of petitioner Avon Dale Garments, Inc. and
its predecessor-in-interest, Avon Dale Shirt Factory. Following a dispute brought
about by the rotation of workers, a compromise agreement was entered into between
petitioner and private respondents wherein the latter were terminated from service and
given their corresponding separation pay.
However, upon refusal of the petitioner to include in the computation of private
respondents' separation pay the period during which the latter were employed by Avon
Dale Shirt Factory, private respondents filed a complaint with the labor arbiter
claiming a deficiency in their separation pay (docketed as NLRC-NCR-00-02-00810-
93). According to private respondents, their previous employment with petitioner's
predecessor-in-interest, Avon Dale Shirt Factory, should be credited in computing
their separation pay considering that Avon Dale Shirt factory was not dissolved and
they were not in turn hired as new employees by Avon Dale Garments, Inc.

ISSUE:

Whether or not the petitioner be held liable for private respondents' separation
pay from Avon Dale Shirt Factory.

RULING:

YES.

Petitioner failed to establish that Avon Dale Garments, Inc., is a separate and
distinct entity from Avon Dale Shirt Factory, absent any showing that there was
indeed an actual closure and cessation of the operations of the latter. The mere filing
of the Articles of Dissolution with the Securities and Exchange Commission, without
more, is not enough to support the conclusion that actual dissolution of an entity in
fact took place. On the contrary, the prevailing circumstances in this case indicated
that Petitioner Company is not distinct from its predecessor Avon Dale Shirt Factory,
but in fact merely continued the operations of the latter under the same owners, the
same business venture, at same address, and even continued to hire the same
employees.
Thus, conformably with established jurisprudence, the two entities cannot be
deemed as separate and distinct where there is a showing that one is merely the
continuation of the other. In fact, even a change in the corporate name does not make
a new corporation, whether effected by a special act or under a general law; it has no
effect on the identity of the corporation, or on its property, rights, or
liabilities.Respondent NLRC therefore, did not commit any grave abuse of discretion in
holding that petitioner should likewise include private respondents' employment with
Avon Dale Shirt Factory in computing private respondents' separation pay as
petitioner failed to substantiate its claim that it is a distinct entity.

374 | P a g e
Law 321_Corporation LAW_ Case Digest

DAGUHOY ENTERPRISES, INC.


vs.
RITA L. PONCE
G.R. No. L-6515 October 18, 1954

FACTS:

The Daguhoy Enterprises, Inc., a local corporation, with principal office in the
City of Manila filed in the Court of First Instance of the City Civil Case No. 15923
against Rita L. Ponce and her husband Domingo Ponce, for the collection of a loan of
P6,190 with interest at 12 per cent per annum from June 24, 1950, plus P2,500 as
attorney's fees and P34 as expenses of litigation. Defendant filed an answer admitting
practically all the allegations of the complaint, set up affirmative defenses, and a
counterclaim asking for the cancellation of the mortgage which secured the payment
of the loan of P6,190. They also filed a petition for the inclusion of Potenciano Gapol as
a third party litigant, at the same time filing a third party complaint against him
asking for damages in the amount of P25,000. The plaintiff corporation answered the
counterclaim and opposed the petition for the inclusion of a third party litigant.
Thereafter, plaintiff corporation filed a motion for judgment on the pleadings which
petition was opposed by the defendants. Then, on October 9, 1952, the trial court
rendered judgment against defendants.

ISSUE:

Whether or not the said deposit relieve the present defendants from the
payment of interests from the time of deposit, on the theory that the deposit amounted
to a payment of the loan.

RULING:

NO.

It should be remembered that Civil Case No. 13753 though in the same Court of
First Instance of Manila, is a separate and different action, for accounting not only for
the amount of the loan but for other sums. The plaintiff in that case was Gapol in
behalf of the Daguhoy Enterprises, Inc. and the defendants are Domingo Ponce and
his son Buhay M. Ponce. The parties in the present case are different. Furthermore,
when the plaintiff in said case 13753 petitioned the trial court for permission to
withdraw the deposit, presumably to pay the loan involved in the present action, his
petition was denied by the court because of the opposition of the defendants therein,
one of whom is Domingo Ponce, co-defendant of Rita Ponce in the present case. The
result was that the present plaintiff corporation could not take possession and dispose
of said amount. In other words, the loan is not yet paid.

375 | P a g e
Law 321_Corporation LAW_ Case Digest

Involuntary

PHILIPPINE NATIONAL BANK


vs.
CIF OF RIZAL
G.R. NO. 63201 MAY 27, 1992

FACTS:

On March 1, 1954, private respondents entered into a contract of lease with


Philippine Blooming Mills, Co., Inc., (PBM for brevity) whereby the letter shall lease the
aforementioned parcels of land as factory site. PBM was duly organized and
incorporated on January 19, 1952 with a corporate term of twenty-five (25) years. This
leasehold right of PBM covering the parcels of land was duly annotated at the back of
the above stated certificates of title as Entry No. 9367/T-No. 32843. The contract of
lease provides that the term of the lease is for twenty years beginning from the date of
the contract and "is extendable for another term of twenty years at the option of the
LESSEE should its term of existence be extended in accordance with law."
On October 11, 1963, PBM executed in favor of Philippine National Bank (PNB
for brevity), petitioner herein, a deed of assignment, conveying and transferring all its
rights and interests under the contract of lease which it executed with private
respondents. The assignment was for and in consideration of the loans granted by
PNB to PBM. The deed of assignment was registered and annotated at the back of the
private respondents' certificates of title as Entry No. 85215/T-No. 32843.

ISSUE:

Whether or not the cancellation of the entries on respondent's certificates of


title valid and proper.

RULING:

YES.

The contract of lease expressly provides that the term of the lease shall be
twenty years from the execution of the contract but can be extended for another period
of twenty years at the option of the lessee should the corporate term be extended in
accordance with law. Clearly, the option of the lessee to extend the lease for another
period of twenty years can be exercised only if the lessee as corporation renews or
extends its corporate term of existence in accordance with the Corporation Code which
is the applicable law. Thus, in the instant case, the initial term of the contract of lease
which commenced on March 1, 1954 ended on March 1, 1974. PBM as lessee
continued to occupy the leased premises beyond that date with the acquiescence and
consent of the respondents as lessor. Records show however, that PBM as a
corporation had a corporate life of only twenty-five (25) years which ended an January
19, 1977. It should be noted however that PBM allowed its corporate term to expire
without complying with the requirements provided by law for the extension of its
corporate term of existence.
There is no need for the institution of a proceeding for quo warranto to
determine the time or date of the dissolution of a corporation because the period of
corporate existence is provided in the articles of incorporation. When such period
expires and without any extension having been made pursuant to law, the corporation
is dissolved automatically insofar as the continuation of its business is concerned.
Considering the foregoing in relation to the contract of lease between the parties
herein, when PBM's corporate life ended on January 19, 1977 and its 3-year period for
winding up and liquidation expired on January 19, 1980, the option of extending the
lease was likewise terminated on January 19, 1977 because PBM failed to renew or
extend its corporate life in accordance with law. From then on, the respondents can
exercise their right to terminate the lease pursuant to the stipulations in the contract.

376 | P a g e
Law 321_Corporation LAW_ Case Digest

Liquidation: Methods

METROPOLITAN BANK and TRUST COMPANY


vs.
CENTRO DEVELOPMENT CORPORATION, CHONGKING KEHYENG, MANUEL CO
KEHYENG and Quirino Kehyeng
G.R. No. 180974 June 13, 2012

FACTS:

On March 20, 1990, in a special meeting of the board of directors of respondent


Centro Development Corporation, its president Go Eng Uy was authorized to mortgage
its properties and assets to secure the medium-term loan of P 84 million of Lucky Two
Corporation and Lucky Two Repacking. The properties and assets consisted of a parcel
of land with a building and improvements located at Salcedo St., Legaspi Village,
Makati City, and covered by Transfer Certificate of Title Nos. 139880 and 139881. This
authorization was subsequently approved on the same day by the stockholders. Maria
Jacinta V. Go, the corporate secretary, issued a Secretary‘s Certificate.
San Carlos failed to pay these outstanding obligations despite demand. Thus,
petitioner, as trustee of the MTI, enforced the conditions thereof and initiated
foreclosure proceedings, denominated as Foreclosure No. S-04-11, on the mortgaged
properties.

ISSUE:

Whether or not the petitioner, as creditor or as trustee, had a cause of action to


move for the extrajudicial foreclosure of the subject properties mortgaged under the
MTI.

RULING:

NO.

It is the intent of the COMPANY that the BORROWERS will obtain additional
loans or credit accommodations from certain other banking or financial institutions in
accordance with arrangements made by the BORROWERS with the CREDITORS.
All obligations covered by this indenture shall be evidenced by a mortgage
participation certificate in the form of schedule ii hereof, the issuance of which by the
trustee to the participating creditor/s shall be in accordance with section 7 of this
indenture, provided the aggregate loan values of the collateral, based on the latest
appraisal thereof, are not exceeded.
Moreover, it is worthy to note that respondents do not assail the previous MTI
executed with BPI. They do not question the validity of the mortgage constituted over
all or substantially all of respondent Centro‘s assets pursuant to the 21 March 1994
MTI in the amount of P 84 million. Nor do they question the additional loans
increasing the value of the mortgage to P 144 million; or the use of Centro‘s properties
as collateral for the loans of San Carlos, Lucky Two Corporation, and Lucky Two
Repacking.

377 | P a g e
Law 321_Corporation LAW_ Case Digest

METROPOLITAN BANK & TRUST COMPANY, INC.


vs.
THE BOARD OF TRUSTEES OF RIVERSIDE MILLS CORPORATION PROVIDENT
AND RETIREMENT FUND
G.R. No. 176959 September 8, 2010

FACTS:

RMC established a Provident and Retirement Plan for its regular employees.
Under the Plan, RMC and its employees shall each contribute 2% of the employee‘s
current basic monthly salary, with RMC‘s contribution to increase by 1% every five (5)
years up to a maximum of 5%. The contributions shall form part of the provident fund
(the Fund) which shall be held, invested and distributed by the Commercial Bank and
Trust Company. On October 15, 1979, the Board of Trustees of RMCPRF (the Board)
entered into an Investment Management Agreement with Philbank (petitioner
Metropolitan Bank and Trust Company). Pursuant to the Agreement, petitioner shall
act as an agent of the Board and shall hold, manage, invest and reinvest the Fund in
Trust Account No. 1797 in its behalf. The Agreement shall be in force for one (1) year
and shall be deemed automatically renewed unless sooner terminated either by
petitioner bank or by the Board.
In 1984, RMC ceased business operations. Nonetheless, petitioner continued to
render investment services to respondent Board. In a letter dated September 27, 1995,
petitioner informed respondent Board that Philbank‘s Board of Directors had decided
to apply the remaining trust assets held by it in the name of RMCPRF against part of
the outstanding obligations of RMC. Subsequently, respondent RMC Unpaid
Employees Association, Inc. (Association), representing the terminated employees of
RMC, learned of Trust Account No. 1797. Through counsel, they demanded payment
of their share in a letter dated February 4, 1997. When such demand went unheeded,
the Association, along with the individual members of RMCPRF, filed a complaint for
accounting against the Board and its officers.

ISSUE:

Whether or not the functions of the Board of Trustees ceased upon with RMC‘s
closure.

RULING:

NO.

Under Section 122 of the Corporation Code, a dissolved corporation shall


nevertheless continue as a body corporate for three (3) years for the purpose of
prosecuting and defending suits by or against it and enabling it to settle and close its
affairs, to dispose and convey its property and to distribute its assets, but not for the
purpose of continuing the business for which it was established. Within those three (3)
years, the corporation may appoint a trustee or receiver who shall carry out the said
purposes beyond the three (3)-year winding-up period. Thus, a trustee of a dissolved
corporation may commence a suit which can proceed to final judgment even beyond
the three (3)-year period of liquidation.
In the same manner, during and beyond the three (3)-year winding-up period of
RMC, the Board of Trustees of RMCPRF may do no more than settle and close the
affairs of the Fund. The Board retains its authority to act on behalf of its members,
albeit, in a limited capacity. It may commence suits on behalf of its members but not
continue managing the Fund for purposes of maximizing profits. Here, the Board‘s act
of issuing the Resolution authorizing petitioner to release the Fund to its beneficiaries
is still part of the liquidation process, which is, satisfaction of the liabilities of the
Plan, and does not amount to doing business. Hence, it was properly within the
Board‘s power to promulgate.

378 | P a g e
Law 321_Corporation LAW_ Case Digest

YAM
vs.
COURT OF APPEALS
GR No. 104726 11 February 1999

FACTS:

Parties entered into several loan agreements, the petitioners, Yam and Lent,
being the borrowers while the private respondent, Manphil Investment Corporaton, the
lender. In said contract, petitioners were given a loan of P500,000.00 by private
respondent. The contract provided for the payment of 12% annual interest, 2%
monthly penalty, 1 1/2% monthly service charge, and 10% attorney's fees.
Denominated the first Industrial Guarantee and Loan Fund (IGLF), the loan was
secured by a chattel mortgage on the printing machinery in petitioners' establishment.
By April 2, 1985, petitioners had paid their first loan of P500,000.00. On
November 4, 1985, private respondent was placed under receivership by the Central
Bank and Ricardo Lirio and Cristina Destajo were appointed as receiver and in-house
examiner, respectively.
A check was sent to respondent as partial payment of the second loan which
was marked as full payment in the vouchers. Demands were made for the balance of
the same, however, it was unheeded prompting respondent to file a case against the
petitioner for collection of the balance.
The trial court ruled in favor of respondents which the Court of Appeals
affirmed.

ISSUE:

Whether or not petitioner is liable to the penalties and service charges of the
loan.

RULING:

YES.

The alleged condonation of the penalties and service charges by Sobrepeñas,


president of respondent, must be in writing to be binding between and among the
parties. Since it was not reduced in writing, the same is not effective. Further, the
alleged condonation happened after the respondent corporation was placed under
receivership. As held in Villanueva v. Court of Appeals ―the appointment of a receiver
operates to suspend the authority of a corporation and of its directors and officers over
its property and effects, such authority being reposed in the receiver.‖ Thus,
Sobrepeñas had no authority to condone the debt. Petition denied.

379 | P a g e
Law 321_Corporation LAW_ Case Digest

ALHAMBRA CIGAR & CIGARETTE MANUFACTURING COMPANY, INC.


vs.
SECURITIES OF EXCHANGE COMMISSION
G.R. NO. L-23606 JULY 29, 1968

FACTS:

Petitioner Alhambra Cigar and Cigarette Manufacturing Company, Inc. was


duly incorporated under Philippine laws on January 15, 1912. By its corporate articles
it was to exist for fifty (50) years from incorporation. Its term of existence expired on
January 15, 1962. On that date, it ceased transacting business, entered into a state of
liquidation. Thereafter, a new corporation. Alhambra Industries, Inc. was formed to
carry on the business of Alhambra.
On June 20, 1963 within Alhambra's three-year statutory period for liquidation
- Republic Act 3531 was enacted into law. It amended Section 18 of the Corporation
Law; it empowered domestic private corporations to extend their corporate life beyond
the period fixed by the articles of incorporation for a term not to exceed fifty years in
any one instance. Previous to Republic Act 3531, the maximum non-extendible term of
such corporations was fifty years.
On July 15, 1963, at a special meeting, Alhambra's board of directors resolved
to amend paragraph "Fourth" of its articles of incorporation to extend its corporate life
for an additional fifty years, or a total of 100 years from its incorporation. FOURTH.
That the term for which said corporation is to exist is fifty (50) years from and after the
date of incorporation, and for an additional period of fifty (50) years thereafter.On
October 28, 1963, Alhambra's articles of incorporation as so amended certified correct
by its president and secretary and a majority of its board of directors, were filed with
respondent Securities and Exchange Commission (SEC).

ISSUE:

Whether or not the corporation can still extend its corporate term within the
three-year statutory period for liquidation.

RULING:

NO.

A corporation cannot extend its life by amendment of its articles of


incorporation effected during the three-year period for liquidation when its original
term of existence had already expired. Since the privilege of extension is purely
statutory, all of the statutory conditions precedent must be complied with in order
that the extension may be effectuated. And, generally these conditions must be
complied with, and the steps necessary to effect the extension must be taken, during
the life of the corporation, and before the expiration of the term of existence as original
fixed by its charter or the general law, since, as a rule, the corporation is ipso facto
dissolved as soon as that time expires. So where the extension is by amendment of the
articles of incorporation, the amendment must be adopted before that time.
And, similarly, the filing and recording of a certificate of extension after that
time cannot relate back to the date of the passage of a resolution by the stockholders
in favor of the extension so as to save the life of the corporation. The contrary is true,
however, and the doctrine of relation will apply, where the delay is due to the neglect
of the officer with whom the certificate is required to be filed, or to a wrongful refusal
on his part to receive it. And statutes in some states specifically provide that a renewal
may be had within a specified time before or after the time fixed for the termination of
the corporate existence.

380 | P a g e
Law 321_Corporation LAW_ Case Digest

CHUNG KA BIO
vs.
INTERMEDIATE APPELLATE COURT
G.R. NO. 71837 JULY 26, 1988

FACTS:

The Philippine Blooming Mills Company, Inc. was incorporated on January 19,
1952, for a term of 25 years which expired on January 19,1977. On May 14, 1977, the
members of its board of directors executed a deed of assignment of all of the accounts
receivables, properties, obligations and liabilities of the old PBM in favor of Chung
Siong Pek in his capacity as treasurer of the new PBM, then in the process of
reincorporation.On June 14, 1977, the new PMB was issued a certificate of
incorporation by the Securities and Exchange Commission.
On May 5, 1981, Chung Ka Bio and the other petitioners herein, all
stockholders of the old PBM, filed with the SEC a petition for liquidation (but not for
dissolution) of both the old PBM and the new PBM. The allegation was that the former
had become legally non-existent for failure to extend its corporate life and that the
latter had likewise been ipso facto dissolved for non-use of the charter and continuous
failure to operate within 2 years from incorporation.

ISSUE:

Whether or not the board of directors of an already dissolved corporation have


the inherent power, without the express consent of the stockholders, to convey all its
assets to a new corporation.

RULING:

YES.

While we agree that the board of directors is not normally permitted to


undertake any activity outside of the usual liquidation of the business of the dissolved
corporation, there is nothing to prevent the stockholders from conveying their
respective shareholdings toward the creation of a new corporation to continue the
business of the old. Winding up is the sole activity of a dissolved corporation that does
not intend to incorporate anew. If it does, however, it is not unlawful for the old board
of directors to negotiate and transfer the assets of the dissolved corporation to the new
corporation intended to be created as long as the stockholders have given their
consent. This was not prohibited by the Corporation Act. In fact, it was expressly
allowed by Section 28-1/2.
The petitioners and the private respondents are not strangers but relatives and
close business associates. The PBM office is in the heart of Metro Manila. The new
corporation, like the old, employs as many as 2,000 persons, the same personnel who
worked for the old PBM. Additionally, one of the petitioners, Chung Siong Pek was one
of the directors who executed the deed of assignment in favor of the old PBM and it
was he also who received the deeded assets on behalf and as treasurer of the new
PBM. Surely, these circumstances must operate to bar the petitioners now from
questioning the deed of assignment after this long period of inaction in the protection
of the rights they are now belatedly asserting. Laches has operated against them.

381 | P a g e
Law 321_Corporation LAW_ Case Digest

REPUBLIC OF THE PHILIPPINES


vs.
MARSMAN DEVELOPMENT COMPANY
G.R. NO. L-18956 APRIL 27, 1972

FACTS:

Sometime before October 15, 1953 an investigation was conducted on the


business operation and activities of the corporation leading to the discovery that
certain taxes were due (from) it on logs produced from its concession. The Bureau of
Internal Revenue made three assessments, totalling P59,133.78, and demanded
payment thereof. Defendants however failed to pay the taxes hence the filing of
charges in court. The defendants contend that the present action is already barred
under section 77 of the Corporation Law, Act No. 1459, as amended, which allows the
corporate existence of a corporation to continue only for three years after its
dissolution, for the purpose of presenting or defending suits by or against it, and to
settle and close its affairs.
They point out that inasmuch as the Marsman Development Co. was extra-
judicially dissolved on April 23, 1954, a fact admitted in the amended complaint, the
filing of both the original complaint on September 8, 1958 and the amended complaint
on August 26, 1956 was beyond the aforesaid three-year period.

ISSUE:

Whether or not the right of the government to collect the sums has already
prescribed.

RULING:

NO.

The stress given by appellants to the extinction of the corporate and juridical
personality as such of appellant corporation by virtue of its extra-judicial dissolution
which admittedly took place on April 23, 1954 is misdirected.
Further, at any time during said three years said corporation is authorized and
empowered to convey all of its property to trustees for the benefit of members, stock-
holders, creditors, and others interested. From and after any such conveyance by the
corporation of its property in trust for the benefit of its members, stockholders,
creditors, and others in interest, all interest which the corporation had in the property
terminates, the legal interest vests in the trustee, and the beneficial interest in the
members, stockholders, creditors, or other persons in interest.
Thus, in whatever way the matter may be viewed, the Government became the
creditor of the corporation before the completion of its dissolution by the liquidation of
its assets. Appellant F.H. Burgess, whom it chose as liquidator, became in law the
trustee of all its assets for the benefit of all persons enumerated in Section 78,
including its creditors, among whom is the Government, for the taxes herein involved.
To assume otherwise would render the extra-judicial dissolution illegal and void,
since, according to Section 62 of the Corporation Law, such kind of dissolution is
permitted only when it "does not affect the rights of any creditor having a claim
against the corporation."
It is immaterial that the present action was filed after the expiration of three
years after April 23, 1954, for at the very least, and assuming that judicial
enforcement of taxes may not be initiated after said three years despite the fact that
the actual liquidation has not been terminated and the one in charge thereof is still
holding the assets of the corporation, obviously for the benefit of all the creditors
thereof, the assessment aforementioned, made within the three years, definitely
established the Government as a creditor of the corporation for whom the liquidator is
supposed to hold assets of the corporation. And since the suit at bar is only for the
collection of taxes finally assessed against the corporation within the three years
invoked by appellants, their fourth assignment of error cannot be sustained.

382 | P a g e
Law 321_Corporation LAW_ Case Digest

TAN TIONG BIO


vs.
COMMISSION OF INTERNAL REVENUE
G.R. NO. L-15778 APRIL 23, 1962

FACTS:

On October 19, 1946, the Central Syndicate, a corporation organized under the
laws of the Philippines, thru its General Manager, David Sycip, sent a letter to the
Collector of Internal Revenue advising the latter that it purchased from Dee Hong Lue
the entire stock of surplus properties which the said Dee Hong Lue had bought from
the Foreign Liquidation Commission and that as it assumed Dee Hong Lue's obligation
to pay the 3-1/2% sales tax on said surplus goods, it was remitting the sum of
P43,750.00 in his behalf as deposit to answer for the payment of said sales tax with
the understanding that it would later be adjusted after the determination of the exact
consideration of the sale.
On January 31, 1948, the syndicate again wrote the Collector requesting the
refund of P1,103.28 representing alleged excess payment of sales tax due to the
adjustment and reduction of the purchase price in the amount of P31,522.18. The
Collector decided after a thorough investigation of the facts that the Central Syndicate
was the importer and original seller of the surplus goods in question and, therefore,
the one liable to pay the sales tax. Accordingly, on January 4, 1952, the Collector
assessed against the syndicate the amount of P33,797.88 and P300.00 as deficiency
sales tax, inclusive of the 25% surcharge and compromise penalty, respectively, and
on the same date, in a separate letter, he denied the request of the syndicate for the
refund of the sum of P1,103.28.

ISSUE:

Whether the sales tax of a dissolved corporation can be enforced against its
successors-in-interest who are the present petitioners.

RULING:

YES.

The creditor of a dissolved corporation may follow its assets once they passed
into the hands of the stockholders. And it has been stated, with reference to the effect
of dissolution upon taxes due from a corporation, "that the hands of the government
cannot, of course, collect taxes from a defunct corporation, it loses thereby none of its
rights to assess taxes which had been due from the corporation, and to collect them
from persons, who by reason of transactions with the corporation, hold property
against which the tax can be enforced and that the legal death of the corporation no
more prevents such action than would the physical death of an individual prevent the
government from assessing taxes against him and collecting them from his
administrator, who holds the property which the decedent had formerly possessed".
Bearing in mind that our corporation law is of American origin, the foregoing
authorities have persuasive effect in considering similar cases in this jurisdiction. This
must have been taken into account when in G.R. No. L-8800 this Court said that
petitioners could be held personally liable for the taxes in question as successors-in-
interest of the defunct corporation.
Considering that the Central Syndicate realized from the sale of the surplus
goods a net profit of P229,073.83, and that the sale of said goods was the only
transaction undertaken by said syndicate, there being no evidence to the contrary, the
conclusion is that said net profit remained intact and was distributed among the
stockholders when the corporation liquidated and distributed its assets on August 15,
1948, immediately after the sale of the said surplus goods. Petitioners are therefore
the beneficiaries of the defunct corporation and as such should be held liable to pay
the taxes in question. However, there being no express provision requiring the
stockholders of the corporation to be solidarily liable for its debts which liability must
be express and cannot be presumed.

383 | P a g e
Law 321_Corporation LAW_ Case Digest

Duration

REYNOLDS PHILIPPINE CORPORATION


vs.
COURT OF APPEALS
G.R. NO. L-36187 JANUARY 17, 1989

FACTS:

In its complaint of June 2, 1966, the petitioner sought to recover from the
private respondent Serg's Products, Inc. the sum of P32,565.62 representing the
unpaid price of aluminum foils and cores sold and delivered by it to the latter. The
private respondent denied liability for payment of the account on the ground that the
aluminum foils and cores were ordered or purchased by Serg's Chocolate Products, a
partnership of Antonio Goquiolay and Luis Sequia Mendoza, not Serg's Products, Inc.,
a corporation managed and controlled by Antonio Goquiolay and his wife Conchita
Goquiolay, as majority stockholders and principal officers.

ISSUE:

Whether or not the real debtor of the petitioner were the Private Respondents.

RULING:

YES.

Although the commercial documents were indeed in the name of "Serg's


Chocolate Products," the following facts proved that the true purchaser of the
aluminum foils and cores from the petitioner, was "Serg's Products, Inc." not the
partnership denominated "Serg's Chocolate Products."
The attempt to make the two factories appear as two separate businesses, when
in reality they are but one, is but a devise to defeat the ends of the law and should not
be permitted to prevail. Although the coffee factory is a corporation and, by legal
fiction, an entity existing separate and apart from persons composing it, T and his
family, it is settled that this fiction of law, which had been introduced as a matter of
convenience and to subserve the ends of justice cannot be invoked to further an end
subversive of that purpose.

384 | P a g e
Law 321_Corporation LAW_ Case Digest

MAMBULAO LUMBER COMPANY


vs.
PHILIPPINE NATIONAL BANK
GR No. L-22973 30 January 1968

FACTS:

The plaintiff applied for an industrial loan with interest with the PNB. To
secure the payment of the loan, the plaintiff mortgaged to defendant PNB a parcel of
land, together with the buildings and improvements existing thereon as well as various
sawmill equipment, rolling unit and other fixed assets of the plaintiff. However, the
plaintiff failed to pay the amortizations on the amounts released to and received by it.
Repeated demands were made upon the plaintiff to pay its obligation but it
failed or otherwise refused to do so. Upon inspection and verification made by
employees of the PNB, it was found that the plaintiff had already stopped operation
about the end of 1957 or early part of 1958. Thus, PNB requested for the foreclosure
of the real estate mortgage as well as the chattel mortgage.

ISSUE:

Whether or not petitioner foreclosure of the mortgage is tenable.

RULING:

NO.

It is clear that there was no further necessity to foreclose the mortgage of


herein appellant's chattels since the obligation has already been paid for. On this
ground alone, it may be declared that the sale of appellant's chattels, illegal and void.
The Court took into consideration the fact that the PNB must have been led to believe
that the stipulated 10% of the unpaid loan for attorney's fees in the real estate
mortgage was legally maintainable, and in accordance with such belief, herein appellee
bank insisted that the proceeds of the sale of appellant's real property was deficient to
liquidate the latter's total indebtedness. Be that as it may, however, still the
subsequent sale of herein appellant's chattels illegal and objectionable on other
grounds. The parties have agreed that in case of foreclosure, the sale should be made
elsewhere not necessarily where the properties are located. This stipulation is allowed
under the law which provides for the general rule. However, the sale was made in the
place where the properties are situated. A clear violation of the agreement of the
parties. Thus, the foreclosure is not tenable.

385 | P a g e
Law 321_Corporation LAW_ Case Digest

Powers of Corporation at Liquidation

VITALIANO N. AGUIRRES II and FIDEL N. AGUIRRE


vs.
FQB+7, INC., NATHANIEL D. BOCOBO, PRISCILA BOCOBO and ANTONIO DE
VILLA
G.R. No. 170770 January 9, 2013

FACTS:

On October 5, 2004, Vitaliano filed, in his individual capacity and on behalf of


FQB+7, Inc., a Complaint for intra-corporate dispute, injunction, inspection of
corporate books and records, and damages, against respondents Nathaniel D. Bocobo,
Priscila D. Bocobo and Antonio De Villa. The Complaint alleged that FQB+7 was
established in 1985 with the following directors and subscribers, as reflected in its
Articles of Incorporation.
The substantive changes found in the GIS, respecting the composition of
directors and subscribers of FQB+7, prompted Vitaliano to write to the "real" Board of
Directors (the directors reflected in the Articles of Incorporation), represented by Fidel
N. Aguirre. In this letter dated April 29, 2004, Vitaliano questioned the validity and
truthfulness of the alleged stockholders meeting held on September 3, 2002. He asked
the "real" Board to rectify what he perceived as erroneous entries in the GIS, and to
allow him to inspect the corporate books and records. The "real" Board allegedly
ignored Vitaliano‘s request.

ISSUE:

Whether or not dissolved corporation may continue as a body corporate for the
limited purpose of liquidating the corporate assets and distributing them to its
creditors, stockholders, and others in interest.

RULING:

YES.

A corporation‘s board of directors is not rendered functus officio by its


dissolution. Since Section 122 allows a corporation to continue its existence for a
limited purpose, necessarily there must be a board that will continue acting for and on
behalf of the dissolved corporation for that purpose. In fact, Section 122 authorizes
the dissolved corporation‘s board of directors to conduct its liquidation within three
years from its dissolution. Jurisprudence has even recognized the board‘s authority to
act as trustee for persons in interest beyond the said three-year period. Thus, the
determination of which group is the bona fide or rightful board of the dissolved
corporation will still provide practical relief to the parties involved.
The same is true with regard to Vitaliano‘s shareholdings in the dissolved
corporation. A party‘s stockholdings in a corporation, whether existing or dissolved, is
a property right which he may vindicate against another party who has deprived him
thereof. The corporation‘s dissolution does not extinguish such property right.
Further, Intra-corporate disputes remain even when the corporation is
dissolved.

386 | P a g e
Law 321_Corporation LAW_ Case Digest

CATMON SALES INTERNATIONAL CORP.


vs.
ATTY. MANUEL D. YNGSON, JR.
G.R. No. 179761 January 15, 2010

FACTS:

On February 8, 1999, petitioner Catmon Sales International Corporation filed a


Petition for Declaration in a State of Suspension of Payments with the SEC. On May
10, 2000, the SEC declared petitioner technically insolvent considering that there was
no settlement reached with its creditors and that its inability to pay its creditors had
lasted for a period longer than one year from the filing of the petition. In the same
Order, the SEC appointed respondent Manuel D. Yngson, Jr. of Receivers and
Liquidators, Inc. as petitioner‘s liquidator.
On May 31, 2001, the SEC terminated the services of respondent. Respondent,
in turn, submitted his Accomplishment Report summarizing all the activities he had
undertaken and billed the SEC the total sum of P623,214.35, representing his
liquidator‘s fee and reimbursement of out-of-pocket expenses. On December 18, 2001,
the SEC ordered that an audit be conducted to determine the proper amount to be
paid to respondent. The Corporation Finance Department noted a slight difference in
the liquidator‘s computation. On September 23, 2004, respondent manifested to the
SEC that he was willing to reduce his liquidator‘s fee provided that his request for
administrative expenses be settled in full. On June 23, 2005, the SEC, through its
General Counsel, ordered the members of the Board of Directors of petitioner to pay
respondent his claim for reimbursement of the expenses incurred in the performance
of his duties as liquidator, together with his liquidator‘s fee, for a total amount of
P398,284.40

ISSUE:

Whether or not SEC has the power to fix the amount of the liquidator‘s fee.

RULING:

YES.

However, to countenance petitioner‘s posturing would be to unduly delimit the


broad powers granted to the SEC under Presidential Decree No. 902-A, specifically the
all-encompassing provision in Section 3 that the SEC has "absolute jurisdiction,
supervision and control" over all corporations who are the grantees of primary
franchises and/or license or permit issued by the government to operate in the
Philippines. There is no gainsaying, therefore, that the SEC is authorized to determine
the fees of receivers and liquidators not only when there is "failure of agreement"
between the parties but also in the absence thereof. A contrary ruling would give
license to corporations under liquidation or receivership to refuse to participate in
negotiations for the fixing of the compensation of their liquidators or receivers so as to
evade their obligation to pay the same.
Petitioner may not have been given the chance to meet face to face with
respondent for the purpose of determining the latter‘s fee. But this fact alone should
not invalidate the amount fixed by the SEC. What matters is the reasonableness of the
fee in light of the services rendered by the liquidator. It is the policy of the SEC to
provide uniform/fair and reasonable compensation or fees for the comparable services
rendered by the duly designated members of the Management Committee (MANCOM),
rehabilitation receivers and liquidators in corporations or partnerships placed under
MANCOM/receivership or liquidation, pursuant to Section 6(d) of Presidential Decree
No. 902-A, the SEC Rules on Corporate Recovery, the Corporation Code of the
Philippines, the Securities Regulation Code, and other related laws enforced by the
SEC.
Clearly, the fee fixed by the SEC was not without basis. Besides, as correctly
held by the CA, "respondent actually rendered services in accordance with his oath of
office as liquidator for which he is entitled to be compensated by petitioner."

387 | P a g e
Law 321_Corporation LAW_ Case Digest

RENE KNECHT AND KNECHT INC.


vs.
UNITED CIGARETTE INC.
GR NO. 139370 JULY 4, 2002

FACTS:

Rose Packing Company, Inc. (Rose Packing), a domestic corporation, owns


parcels of land one of which is covered by TCT No. 73620 which was mortgaged with
the Philippine Commercial and Industrial Bank (PCIB). Said parcels of land were later
sold to United Cigarette Corporation (UCC), through its President Rene Knecht, where
Rose Packing made a warranty that the lots are free from all liens and encumbrances,
except the real estate mortgage constituted over the area covered by TCT No. 73620.
Before the deed of sale could be executed, the parties found that Rose Packing‘s
actual obligation with the PCIB far exceeded the P250,000.00 which UCC assumed to
pay under their agreement. So the PCIB demanded additional collateral from UCC as a
condition precedent for the approval of the sale of the mortgaged property. However,
UCC did not comply.
Meanwhile, Rose Packing again offered to sell the same lots to other prospective
buyers without the knowledge of UCC and without returning to the latter the earnest
money it earlier paid.

ISSUE:

Whether or not the execution of the judgment would still lie against a dissolved
corporation.

RULING:

YES.

In Reburiano vs. Court of Appeals, a case with similar facts, this Court Held:
―the trustee (of a dissolved corporation) may commence a suit which can proceed to
final judgment even beyond the three-year period (of liquidation), no reason can be
conceived why a suit already commenced by the corporation itself during its existence,
not by a mere trustee who, by fiction, merely continues the legal personality of the
dissolved corporation, should not be accorded similar treatment – to proceed to final
judgment and execution thereof.‖
The dissolution of UCC itself, or the expiration of its three-year liquidation
period, should not be a bar to the enforcement of its rights as a corporation. One of
these rights, to be sure, includes the UCC‘s right to seek from the court the execution
of a valid and final judgment in Civil Case No. 9165 – through its trustee/liquidator
Encarnacion Gonzales Wong – for the benefit of its stockholders, creditors and any
other person who may have legal claims against it. To hold otherwise would be to
allow petitioners to unjustly enrich themselves at the expense of UCC. This, in effect,
renders nugatory all the efforts and expenses of UCC in its quest to secure justice, not
to mention the undue delay in disposing of this case prejudicial to the administration
of justice.

388 | P a g e
Law 321_Corporation LAW_ Case Digest

CELIA B. CHUA
vs.
NATIONAL LABOR RELATIONS COMMISSION
G.R. NOS. 89971-75 OCTOBER 17, 1990

FACTS:

In December, 1985, Stanford Microsystems, Inc. (Stanford) filed a petition for


suspension of payments and appointment of rehabilitation receiver with the Securities
and Exchange Commission (SEC). At that time, Stanford had seven (7) secured
creditor banks and more or less seven thousand one hundred twenty-four (7,124)
employees. On February 5, 1986, the SEC declared Stanford to be in a state of
suspension of payments. It issued an order appointing Sycip Gorres & Velayo & Co.
(SGV) as the rehabilitation receiver. In view of these developments, the former
employees of Stanford filed with the Department of Labor and Employment (DOLE)
cases for money claims.In January, 1987, the SEC disapproved the Rehabilitation
Plan submitted by SGV and dismissed Stanford's Petition for Suspension of Payments
and Appointment of a Rehabilitation Receiver. Subsequently, the SEC ordered
Stanford's liquidation.
At the time Stanford filed a petition for suspension of payments and
appointment of rehabilitation receiver with SEC, Stanford had seven (7) secured
creditor banks and approximately 7,124 employees. On March 13, 1987, the seven
secured creditor banks of Stanford and 6,341 former employees executed a
Memorandum of Agreement to speed up the orderly liquidation of Stanford. All the
creditor banks and the said employees were represented by their respective counsel in
the negotiations which were supervised by Regional Director Luna C. Piezas of the
DOLE, National Capital Region.

ISSUE:

Whether or not the SEC has original and exclusive jurisdiction over the
liquidation of Stanford including the procedures for settling the money claims of
former workers and employees.

RULING:

YES.

Jurisdiction over liquidation proceedings of insolvent corporations is vested in


the Securities and Exchange Commission (SEC) pursuant to Presidential Decree No.
902-A, as amended. On the other hand, jurisdiction over money claims of employees
against their employers is vested in the Labor Arbiter whose decision may be appealed
to the National Labor Relations Commission (NLRC) pursuant to Article 217 of the
Labor Code.
An insolvency proceeding is similar to the settlement of a decedent's estate in
that it is a proceeding in rem and is binding against the whole world. Therefore, all
persons which have interest in the subject matter involved, whether or not they are
given notice are equally bound. Thus, "a liquidation of similar import or other
equivalent general liquidation must also necessarily be a proceeding in rem so that all
other interested personswhether known to the parties or not may be bound by such
proceedings."
The rule is that a declaration of bankruptcy or a judicial liquidation must be
present before preferences over various money claims may be enforced. Since a
liquidation proceeding is a proceeding in rem, all claims of creditors whether preferred
or non-preferred, the Identification of the preferred ones and the totality of the
employer's asset should be brought into the picture. There can then be an
authoritative, fair and binding adjudication. The money claims of workers pose a
special problem of jurisdiction when liquidation proceedings are on-going because of
the highly preferred nature given by law to said claims. Further, the rule is that such
money claims were correctly submitted in the course of the liquidation proceedings at
the SEC.

389 | P a g e
Law 321_Corporation LAW_ Case Digest

LUIS C. CLEMENTE
vs.
COURT OF APPEALS
G.R. NO. 82407 MARCH 27, 1995

FACTS:

The parties in this case wrestled concerning the ownership of a piece of


land.The defendants (herein private respondents), claimed ownership of the property
by virtue of acquisitive prescription. Plaintiffs on the other hand claimed ownership
based on the following allegations:
The "Sociedad Popular Calambeña" organization conceived by the parties as a
"Sociedad Anonima," was organized on or about the advent of the early American
occupation of the Philippines. Its principal business was cockfighting or the operation
and management of a cockpit.On June 8, 1911, or during its existence, the "Sociedad"
acquired by installments the parcel of land(subject of the case) above described from
the Friar Lands Estate of Calamba, Laguna at the total cost of P2,676.00.
Plaintiffs evidence also shows that Mariano Elepaño and Pablo Clemente, now
both deceased, were original stockholders of the aforesaid "sociedad." Pablo Clemente
subscribed and paid FOUR HUNDRED EIGHTEEN (418) shares of stocksworth TWO
THOUSAND (P2,000.00) PESOS. Pablo Clemente's shares of stocks were however later
distributed and apportioned to his heirs, in accordance with a Project of Partition to
Luis Clemente, shares worth P510; to Ricardo Clemente, shares worth P510; to Leonor
Clemente de Elepaño, shares also worth P510, and to Placida Clemente de Belarmino
shares worth P510. On September 24, 1932, in accordance with the aforesaid project
of .partition, the "sociedad" issued stock certificates to the aforesaid heirs of Pablo
Clemente.

ISSUE:

Whether or not petitioners can be held liable, given their submissions, to have
succeeded in establishing for themselves a firm title to the property in question.

RULING:

YES.

The Court find petitioners' evidence to be direly wanting. Except in showing that
they are the successors-in-interest of Elepaño and Clemente, petitioners have been
unable to come up with any evidence to substantiate their claim of ownership of the
corporate asset. Absent a corporate liquidation, it is the corporation, not the
stockholders, which can assert, if at all, any title to the corporate assets. The court,
even then, expressed some reservations on the corporation's being able to still validly
pursue such a claim. It said: even assuming that their parents were the only
stockholders of Sociedad, and assuming further that Sociedad has ceased to exist,
these do not ipso facto vest ownership over the property in the hands of plaintiffs-
appellants. Again, assuming that sociedad is a duly-organized entity, under the laws
of the Philippines, its corporate existence is separate and distinct from its stockholders
and from other corporations to which it may be connected (If it was not organized and
registered under Philippine laws as a private corporation, it is a de facto corporation,
as found by the court below, with the right to exercise corporate powers, and thus it is
imperative that any of the modes of transferring ownership from said entity must be
shown.
If, indeed, the sociedad has long become defunct, it should behoove petitioners,
or anyone else who may have any interest in the corporation, to take appropriate
measures before a proper forum for a peremptory settlement of its affairs. We might
invite attention to the various modes provided by the Corporation Code for dissolving,
liquidating or winding up, and terminating the life of the corporation.

390 | P a g e
Law 321_Corporation LAW_ Case Digest

CARLOS GELANO
vs.
COURT OF APPEALS
G.R. NO. L-39050 FEBRUARY 24, 1981

FACTS:

Private respondent Insular Sawmill, Inc. is a corporation organized on


September 17, 1945 with a corporate life of fifty (50) years, or up to September 17,
1995, with the primary purpose of carrying on a general lumber and sawmill business.
To carry on this business, private respondent leased the paraphernal property of
petitioner-wife Guillermina M. Gelano for P1,200.00 a month. It was while private
respondent was leasing the aforesaid property that its officers and directors had come
to know petitioner-husband Carlos Gelano who received from the corporation cash
advances on account of rentals to be paid by the corporation on the land
Out of the cash advances in the total sum of P25,950.00, petitioner Carlos
Gelano was able to pay only P5,950.00 thereby leaving an unpaid balance of
P20,000.00 which he refused to pay despite repeated demands by private respondent.
Petitioner Guillermina M. Gelano refused to pay on the ground that said amount was
for the personal account of her husband asked for by, and given to him, without her
knowledge and consent and did not benefit the family.

ISSUE:

Whether or not a dissolved corporation, could still continue prosecuting and


defending suits after its dissolution and beyond the period of three years.

RULING:

YES.

For this reason, Section 78 of the same law authorizes the corporation, "at any
time during said three years to convey all of its property to trustees for the benefit of
members, Stockholders, creditors and other interested," evidently for the purpose,
among others, of enabling said trustees to prosecute and defend suits by or against
the corporation begun before the expiration of said period.
In the case at bar, when Insular Sawmill, Inc. was dissolved on December 31, 1960,
under Section 77 of the Corporation Law, it still has the right until December 31, 1963
to prosecute in its name the present case. After the expiration of said period, the
corporation ceased to exist for all purposes and it can no longer sue or be sued.
However, a corporation that has a pending action and which cannot be
terminated within the three-year period after its dissolution is authorized under
Section 78 to convey all its property to trustees to enable it to prosecute and defend
suits by or against the corporation beyond the Three-year period although private
respondent (did not appoint any trustee, yet the counsel who prosecuted and defended
the interest of the corporation in the instant case and who in fact appeared in behalf
of the corporation may be considered a trustee of the corporation at least with respect
to the matter in litigation only. Said counsel had been handling the case when the
same was pending before the trial court until it was appealed before the Court of
Appeals and finally to this Court.
It was therefore held by the Supreme Court that there was a substantial
compliance with Section 78 of the Corporation Law and as such, private respondent
Insular Sawmill, Inc. could still continue prosecuting the present case even beyond the
period of three (3) years from the time of its dissolution.
The trustee may commence a suit which can proceed to final judgment even
beyond the three-year period. No reason can be conceived why a suit already
commenced By the corporation itself during its existence, not by a mere trustee who,
by fiction, merely continues the legal personality of the dissolved corporation should
not be accorded similar treatment allowed to proceed to final judgment and execution
thereof.

391 | P a g e
Law 321_Corporation LAW_ Case Digest

JAMES REBURIANO
vs.
COURT OF APPEALS
G.R. NO. 102965 JANUARY 21, 1999

FACTS:

A certain judgment was rendered in favor of the private respondent which


became final and executor. It appears that prior to the promulgation of the decision of
the trial court, private respondent amended its articles of incorporation to shorten its
term of existence to July 8, 1983. The amended articles of incorporation was approved
by the Securities and Exchange Commission on March 2, 1984. The trial court was
not notified of this fact. On February 13, 1991, petitioners moved to quash the writ of
execution alleging that the private respondent was no longer in existence and had no
more juridical personality and so, as such, it no longer had the capacity to sue and be
sued; That after the [private respondent], as a corporation, lost its existence and
juridical personality, Atty. Romualdo M. Jubay had no more client in this case and so
his appearance in this case was no longer possible and tenable; That in view of the
foregoing premises, therefore, the decision rendered by this Honorable Court and by
the Honorable Court of Appeals are patent nullity, for lack of jurisdiction and lack of
capacity to sue and be sued on the part of the [private respondent.]; That the above-
stated change in the situation of parties, whereby the [private respondent] ceased to
exist since 8 July 1983, renders the execution of the decision inequitable or
impossible.

ISSUE:

Whether or not a dissolved and non-existing corporation could be represented


by a lawyer as counsel.

RULING:

YES.

It is to be noted that the time during which the corporation, through its own
officers, may conduct the liquidation of its assets and sue and be sued as a
corporation is limited to three years from the time the period of dissolution
commences: but there is no time limit within which the trustees must complete a
liquidation placed in their hands. It is provided only that the conveyance to the
trustees must be made within the three-year period. It may be found impossible to
complete the work of liquidation within the three-year period or to reduce disputed
claims to judgment.
The authorities are to the effect that suits by or against a corporation abate
when it ceased to be an entity capable of suing or being sued; but trustees to whom
the corporate assets have been conveyed pursuant to the authority may sue and be
sued as such in all matters connected with the liquidation.
There is, therefore, no reason why the suit filed by private respondent should
not be allowed to proceed to execution. It is conceded by petitioners that the judgment
against them and in favor of private respondent in C.A. G.R. No. 16070 had become
final and executory. The only reason for their refusal to execute the same is that there
is no existing corporation to which they are indebted.
Such argument is untenable. The law specifically allows a trustee to manage
the affairs of the corporation in liquidation. Consequently, any supervening fact, such
as the dissolution of the corporation, repeal of a law, or any other fact of similar
nature would not serve as an effective bar to the enforcement of such right.

392 | P a g e
Law 321_Corporation LAW_ Case Digest

REPUBLIC PLANTERS BANK


vs.
COURT OF APPEALS,
G.R. No. 93073 December 21, 1992

FACTS:

Shozo Yamaguchi and Fermin Canlas were President/Chief Operating Officer


and Treasurer respectively, of Worldwide Garment Manufacturing, Inc. By virtue of
Board Resolution No.1 dated August 1, 1979, defendant Shozo Yamaguchi and private
respondent Fermin Canlas were authorized to apply for credit facilities with the
petitioner Republic Planters Bank in the forms of export advances and letters of
credit/trust receipts accommodations.
Petitioner bank issued nine promissory notes. In some promissory notes, the
name Worldwide Garment Manufacturing, Inc. was apparently rubber stamped above
the signatures of defendant and private respondent.
On December 20, 1982, Worldwide Garment Manufacturing, Inc. noted to
change its corporate name to Pinch Manufacturing Corporation. Subsequently,
petitioner bank filed a complaint for the recovery of sums of money covered among
others, by the nine promissory notes with interest thereon, plus attorney's fees and
penalty charges.
The complainant was originally brought against Worldwide Garment
Manufacturing, Inc. inter alia, but it was later amended to drop Worldwide
Manufacturing, Inc. as defendant and substitute Pinch Manufacturing Corporation it
its place. Defendants Pinch Manufacturing Corporation and Shozo Yamaguchi did not
file an Amended Answer and failed to appear at the scheduled pre-trial conference
despite due notice.
Only private respondent Fermin Canlas filed an Amended Answer wherein he
denied having issued the promissory notes in question since according to him, he was
not an officer of Pinch Manufacturing Corporation, but instead of Worldwide Garment
Manufacturing, Inc., and that when he issued said promissory notes in behalf of
Worldwide Garment Manufacturing, Inc., the same were in blank, the typewritten
entries not appearing therein prior to the time he affixed his signature.

ISSUE:

Whether or not the amendment in a corporation's Articles of Incorporation


effecting a change of corporate name extinguished the personality of the original
corporation.

RULING:

NO.

The corporation, upon such change in its name, is in no sense a new


corporation, nor the successor of the original corporation. It is the same corporation
with a different name, and its character is in no respect changed. A change in the
corporate name does not make a new corporation, and whether effected by special act
or under a general law, has no affect on the identity of the corporation, or on its
property, rights, or liabilities.
The corporation continues, as before, responsible in its new name for all debts
or other liabilities which it had previously contracted or incurred. As a general rule,
officers or directors under the old corporate name bear no personal liability for acts
done or contracts entered into by officers of the corporation, if duly authorized.
Inasmuch as such officers acted in their capacity as agent of the old corporation and
the change of name meant only the continuation of the old juridical entity, the
corporation bearing the same name is still bound by the acts of its agents if authorized
by the Board.

393 | P a g e
Law 321_Corporation LAW_ Case Digest

FOREIGN CORPORATIONS
Definition and Rights

AVON
vs.
COURT OF APPEALS et. al.
G.R. No. 97642 August 29, 1997

FACTS:

Yupangco Cotton Mills engaged to secure with Worldwide Security and


Insurance Co. Inc., a foreign corporation not doing business in the Philippines with no
office, place of business or agents in the Philippines, several of its properties for the
periods July 6, 1979 to July 6, 1980 and from October 1, 1980 to October 1, 1981,
under separate insurance policies for the same amount. Both contracts were covered
by reinsurance treaties between Worldwide Surety and Insurance and several foreign
reinsurance companies, including the petitioners.
On December 16, 1979 and May 2, 1981, within the respective effectivity
periods of the policies, the properties therein insured were razed by fire, thereby giving
rise to the obligation of the insurer to indemnify the Yupangco Cotton Mills. Partial
payments were made by Worldwide Surety and Insurance and some of the reinsurance
companies.
On May 2, 1983, Worldwide Surety and Insurance, in a Deed of Assignment,
acknowledged a remaining balance of P19,444,447.75 still due Yupangco Cotton Mills,
and assigned to the latter all reinsurance proceeds still collectible from all the foreign
reinsurance companies. Thus, in its interest as assignee and original insured,
Yupangco Cotton Mills instituted this collection suit against the petitioners.

ISSUE:

Whether or not a foreign corporation has rights under Philippine law.

RULING:

YES.

A foreign corporation, is one which owes its existence to the laws of another
state, and generally, has no legal existence within the state in which it is foreign. It
was also held that corporations have no legal status beyond the bounds of the
sovereignty by which they are created.
Nevertheless, it is widely accepted that foreign corporations are, by reason of
state comity, allowed to transact business in other states and to sue in the courts of
such fora. In the Philippines foreign corporations are allowed such privileges, subject
to certain restrictions, arising from the state's sovereign right of regulation. Before a
foreign corporation can transact business in the country, it must first obtain a license
to transact business here and secure the proper authorizations under existing law.
If a foreign corporation engages in business activities without the necessary
requirements, it opens itself to court actions against it, but it shall not be allowed to
maintain or intervene in an action, suit or proceeding for its own account in any court
or tribunal or agency in the Philippines.
The purpose of the law in requiring that foreign corporations doing business in
the country be licensed to do so, is to subject them to the jurisdiction of the Philippine
courts, otherwise, a foreign corporation illegally doing business here because of its
refusal or neglect to obtain the required license and authority to do business may
successfully though unfairly plead such neglect or illegal act so as to avoid service and
thereby impugn the jurisdiction of the local courts.
The same danger does not exist among foreign corporations that are indubitably
not doing business in the Philippines. Indeed, if a foreign corporation does not do
business here, there would be no reason for it to be subject to the State's regulation.

394 | P a g e
Law 321_Corporation LAW_ Case Digest

PALTING
vs.
SAN JOSE PETROLEUM INC.
G.R. No.L-14441. December 17, 1966

FACTS:

SAN JOSE OIL, is a domestic mining corporation, 90% of the outstanding


capital stock of which is owned by respondent SAN JOSE PETROLEUM, a foreign
(Panamanian) corporation, the majority interest of which is owned by OIL
INVESTMENTS, INC., another foreign (Panamanian) company.
This latter corporation in turn is wholly (100%) owned by PANTEPEC OIL
COMPANY, C. A., and PANCOASTAL PETROLEUM COMPANY, C. A., both organized
and existing under the laws of Venezuela. As of September 30, 1956, there were 9,979
stockholders of PANCOASTAL PETROLEUM found in 49 American states and U.S.
territories, holding 3,476,988 shares of stock; whereas, as of November 30, 1956,
PANTEPEC OIL COMPANY was said to have 3,077,916 shares held by 12,373
stockholders scattered in 49 American states.
In the two lists of stockholders, there is no indication of the citizenship of these
stockholders, or of the total number of authorized stocks of each corporation for the
purpose of determining the corresponding percentage of these listed stockholders in
relation to the respective capital stock of said corporation.

ISSUE:

Whether or not the "tie-up" between the two corporations is violative of the
Constitution, the Laurel-Langley Agreement, the Petroleum Act of 1949, and the
Corporation Law.

RULING:

YES.

The privilege to utilize, exploit, and develop the natural resources of this
country was granted, by Article III of the Constitution, to Filipino citizens or to
corporations or associations 60% of the capital of which is owned by such citizens.
With the Parity Amendment to the Constitution, the same right was extended to
citizens of the United States and business enterprises owned or controlled, directly or
indirectly, by citizens of the United States.
There could be no serious doubt as to the meaning of the word "citizens" used
in the aforementioned provisions of the Constitution. The right was granted to 2 types
of persons: natural persons (Filipino or American citizens) and juridical persons
(corporations 60% of which capital is owned by Filipinos and business enterprises
owned or controlled directly or indirectly, by citizens of the United States). In American
law, "citizen" has been defined as "one who, under the constitution and laws of the
United States, has a right to vote for representatives in congress and other public
officers, and who is qualified to fill offices in the gift of the people."
SAN JOSE PETROLEUM an American business is not entitled to parity rights in
the Philippines. In the circumstances, we have to hold that the respondent SAN JOSE
PETROLEUM, as presently constituted, is not a business enterprise that is authorized
to exercise the parity privileges under the Parity Ordinance, the Laurel-Langley
Agreement and the Petroleum Law. Its tie-up with SAN JOSE OIL is, consequently,
illegal.

395 | P a g e
Law 321_Corporation LAW_ Case Digest

Documentary

GMBH & CO.


vs.
Hon. ISNANI et al
G.R. No. 109272 August 10, 1994

FACTS:

Petitioner is a multinational company organized and existing under the laws of


the Federal Republic of Germany. On July 6, 1983, petitioner filed an application with
the SEC for the establishment of a RAH in the Philippines. The application was
approved by the BOI on Sep. 6, 1983. Consequently, on Sep. 20, 1983, the SEC issued
a Certificate of Registration and License to petitioner.
Private respondent Romana Lanchinebre was a sales representative of
petitioner. On March 1992, she secured a loan of P25,000.00 from petitioner.
Subsequently she made additional cash advances in the sum of P10,000.00. Of the
total amount, P12,170.37 remained unpaid. Despite demand, private respondent
Romana failed to settle her obligation.
On September 2, 1992, petitioner filed a Complaint for collection of sum of
money against private respondents spouses Romana and Teofilo Lanchinebre w/ the
RTC. Instead of filing their Answer, private respondents moved to dismiss the
Complaint. This was opposed by petitioner.

ISSUE:

Whether or not the petitioner has capacity to sue and be sued in the Philippines
despite the fact that petitioner is duly licensed by the SEC to set up and operate a
RAH in the country and that it has continuously operated as such for the last 9 years.

RULING:

YES.

It is clear that petitioner is a foreign corporation doing business in the


Philippines. Hence, Petitioner is covered by the Omnibus Investment Code of 1987.
Petitioner does not engage in commercial dealings or activities in the country because
it is precluded from doing so by P.D. No. 218, under which it was established.
Nonetheless, it has been continuously, since 1983, acting as supervision,
communications and coordination center for its home office's affiliates in Singapore,
and in the process has named its local agent and has employed Philippine nationals
like private respondent Romana Lanchinebre (an employee).
From this uninterrupted performance by petitioner of acts pursuant to its
primary purposes and functions as a regional/area headquarters for its home office, it
is clear that petitioner is doing business in the country. Moreover, private respondents
are estopped from assailing the personality of petitioner.
The rule is that a party is estopped to challenge the personality of a corporation
after having acknowledged the same by entering into a contract with it.
And the doctrine of estoppel to deny corporate existence applies to foreign as
well as to domestic corporations. One who has dealt with a corporation of foreign
origin as a corporate entity is estopped to deny its corporate existence and capacity.
The principle will be applied to prevent a person contracting with a foreign corporation
from later taking advantage of its noncompliance with the statutes chiefly in cases
where such person has received the benefits of the contract.

396 | P a g e
Law 321_Corporation LAW_ Case Digest

Appointment of Resident Agent

NEW YORK MARINE MANAGERS, INC.


vs.
COURT OF APPEALS
G.R. No. 111837 October 24, 1995

FACTS:

Petitioner is a foreign corporation organized under the laws of the United States
while defendant is a local domestic corporation organized under Philippine law. On 25
July 1990 American Natural Soda Ash Corporation (ANSAC) loaded in Portland ,
U.S.A., a shipment of soda ash on board the vessel "MS Abu Hanna" for delivery to
Manila . The supplier/shipper insured the shipment with petitioner. Upon arrival in
Manila the shipment was unloaded and transferred to the vessel "MV Biyayang Ginto"
owned by private respondent. Since the shipment allegedly sustained wettage,
hardening and contamination, it was rejected as total loss by the consignees. When
the supplier sought to recover the value of the cargo loss from petitioner the latter paid
the claim in the amount of US$58,323.96.
On 20 November 1991 petitioner as subrogee filed with the RTC Manila a
complaint for damages against private respondent. Thereafter, private respondent filed
a motion to dismiss the complaint one of its grounds cited being plaintiff having no
legal capacity to sue. Such was denied by the trial court upon opposition of the
petitioner.

ISSUE:

Whether or not petitioner can seek for relief from our courts.

RULING:

NO.

A foreign corporation not engaged in business in the Philippines may exercise


the right to file an action in Philippine courts for an isolated transaction. When the
allegations in the complaint have a bearing on the plaintiff's capacity to sue and
merely state that the plaintiff is a foreign corporation existing under the laws of the
United States, such averment conjures two alternative possibilities: either the
corporation is engaged in business in the Philippines, or it is not so engaged.
In the first, the corporation must have been duly licensed in order to maintain
the suit; in the second, and the transaction sued upon is singular and isolated, no
such license is required. In either case, compliance with the requirement of license, or
the fact that the suing corporation is exempt therefrom, as the case may be, cannot be
inferred from the mere fact that the party suing is a foreign corporation.
The qualifying circumstance being an essential part of the plaintiff's capacity to
sue must be affirmatively pleaded. Hence, the ultimate fact that a foreign corporation
is not doing business in the Philippines must first be disclosed for it to be allowed to
sue in Philippine courts under the isolated transaction rule. Failing in this
requirement, the complaint filed by petitioner with the trial court, it must be said, fails
to show its legal capacity to sue.
In the case at bar, petitioner's complaint is fatally defective for failing to allege
its duly authorized representative or resident agent in this jurisdiction. The pleadings
filed by counsel for petitioner do not suffice. True, a lawyer is generally presumed to be
properly authorized to represent any cause in which he appears, and no written power
of attorney is required to authorize him to appear in court for his client, but such is
disputable. Where said authority has been challenged or attacked by the adverse party
the lawyer is required to show proof of such authority or representation in order to
bind his client.
The requirement of the production of authority is essential because the client
will be bound by his acquiescence resulting from his knowledge that he was being
represented by said attorney.

397 | P a g e
Law 321_Corporation LAW_ Case Digest

Amendment of License

AETNA CASUALTY & SURETY Co.


vs.
PACIFIC STAR LINE et al
G.R. No.L-26809 December 29, 1977

FACTS:

Smith Bell & Co. (Philippines), Inc. and Aetna Surety Casualty & Surety Co.
Inc., as subrogee, instituted a case to recover the amount of US$2,300.00 representing
the value of the stolen and damaged cargo plus litigation expenses and exemplary
damages against Pacific Star Line, The Bradman Co. Inc., Manila Port Service and/or
Manila Railroad Company, Inc. alleging that Pacific Star Line, as a common carrier,
was operating the vessel SS Ampal on a commercial run between United States and
Philippine Ports including Manila; that the defendant, The Bradman Co. Inc., was the
ship agent in the Philippines for the SS Ampal and/or Pacific Star Line; that the
Manila Railroad Co. Inc. and Manila Port Service were the arrastre operators in the
port of Manila and were authorized to delivery cargoes discharged into their custody
on presentation of release papers from the Bureau of Customs and the steamship
carrier and/or its agents; that plaintiff sustained losses due to the negligence of Pacific
Star Line prior to delivery of the cargo to Manila or, in the alternative, due to the
negligence of Manila Port Service after delivery of the cargo to it by the SS Ampal.
The defendants alleged that the plaintiff, Aetna Casualty & Surety Company, is
a foreign corporation not duly licensed to do business in the Philippines and,
therefore, without capacity to sue and be sued.

ISSUE:

Whether or not Aetna Casualty & Surety Company, a foreign corporation not
doing business in the Philippines, can claim damages against defendant.

RULING:

YES.

It is settled that if a foreign corporation is not engaged in business in the


Philippines, it may not be denied the right to file an action in Philippine courts for
isolated transactions.
The object of Sections 68 and 69 of the Corporation Law was not to prevent the
foreign corporation from performing single acts, but to prevent it from acquiring a
domicile for the purpose of business without taking the steps necessary to render it
amenable to suit in the local courts. It was never the purpose of the Legislature to
exclude a foreign corporation which happens to obtain an isolated order for business
from the Philippines, from securing redress in the Philippine courts.
Aetna Casualty & Surety Company is not transacting business of insurance in
the Philippines for which it must have a license. The contract of insurance was
entered into in New York, U.S.A., and payment was made to the consignee in its New
York branch.
Since, Aetna Casualty & Surety Company is not engaged in the business of
insurance in the Philippines but is merely collecting a claim assigned to it by the
consignee, it is not barred from filing the instant case although it has not secured a
license to transact insurance business in the Philippines.
The case is remanded to the trial court for further proceedings to determine the
liability of the defendants-appellees.

398 | P a g e
Law 321_Corporation LAW_ Case Digest

HATHIBHAI BULAKHIDAS
vs.
HONORABLE PEDRO L. NAVARRO
L-49695 April 7, 1986

FACTS:

Petitioner, a foreign partnership, filed a complaint against a domestic


corporation, DSC, before the CFI of Rizal for the recovery of damages allegedly caused
by the failure of the said shipping corporation to deliver the goods shipped to it by
petitioner to their proper destination.
Paragraph 1 of said complaint alleged that plaintiff is "a foreign partnership
firm not doing business in the Philippines" and that it is "suing under an isolated
transaction." Defendant filed a motion to dismiss the complaint on the ground that
plaintiff has no capacity to sue and that the complaint does not state a valid cause of
action against defendant.
Acting on said motion to dismiss, the CFI dismissed the complaint on the
ground that plaintiff being "a foreign corporation or partnership not doing business in
the Philippines it cannot exercise the right to maintain suits before our Courts."

ISSUE:

Whether a foreign corporation ―not engaged in business in the Philippines‖ can


institute an action before Philippine courts.

RULING:

YES.

It is settled that if a foreign corporation is not engaged in business in the


Philippines, it may not be denied the right to file an action in Philippine courts for
isolated transactions. The object of Sections 68 and 69 of the Corporation law was not
to prevent the foreign corporation from performing single acts, but to prevent it from
acquiring a domicile for the purpose of business without taking the steps necessary to
render it amenable to suit in the local courts. It was never the purpose of the
Legislature to exclude a foreign corporation which happens to obtain an isolated order
for business from the Philippines, from securing redress in the Philippine courts.
No general rule or governing principle can be laid down as to what constitutes
'doing' or 'engaging' in or 'transacting' business. Indeed, each case must be judged in
the light of its peculiar environmental circumstances. The true test, however, seems to
be whether the foreign corporation is continuing the body or substance of the business
or enterprise for which it was organized or whether it has substantially retired from it
and turned it over to another. The term implies a continuity of commercial dealings
and arrangements, and contemplates, to that extent, the performance of acts or works
or the exercise of some of the functions normally incident to, and in progressive
prosecution of, the purpose and object of its organization.
While plaintiff is a foreign corporation without license to transact business in
the Philippines, it does not follow that it has no capacity to bring the present action.
Such license is not necessary because it is not engaged in business in the Philippines.
A foreign corporation not engaged in business in the Philippines is not barred from
seeking redress from the courts of the Philippines.

399 | P a g e
Law 321_Corporation LAW_ Case Digest

SCHMID & OBERLY, INC.


vs.
RJL MARTINEZ FISHING CORPORATION
G.R. No. 75198 October 18, 1988

FACTS:

RJL MARTINEZ is engaged in the business of deep-sea fishing. The


transaction, which gave rise to the present controversy, involves twelve (12) "Nagata"-
brand generators. Schmid gave RJL its Quotation dated August 19, 1975 stipulating
that payment would be made by confirming an irrevocable letter of credit in favor of
NAGATA CO . Agreeing with the terms of the Quotation, RJL opened a letter of
credit. SCHMID transmitted to NAGATA CO. an order for the twelve (12) generators to
be shipped directly to RJL MARTINEZ. NAGATA CO. thereafter sent RJL MARTINEZ
the bill of lading and its own invoice and, in accordance with the order, shipped the
generators directly to RJL MARTINEZ.
All generators subject of the two transactions burned out after continuous use.
SCHMID, upon information from RJL, brought the matter to the attention of NAGATA
CO. In July 1976, NAGATA CO. sent two technical representatives who made an
ocular inspection and conducted tests on some of the burned out generators. The
tests revealed that the generators were overrated.
As indicated both in the quotation and in the invoice, the capacity of a
generator was supposed to be 5 KVA, however, it turned out that the actual capacity
was only 4 KVA. The Japanese technicians advised RJL MARTINEZ to ship three (3)
generators to Japan, which the company did.

ISSUE:

Whether or not the transaction between the parties was an indent, not sale.

RULING:

YES.

There is no statutory definition of "indent" in this jurisdiction. However, the


Rules and Regulations to Implement Presidential Decree No. 1789 states that foreign
firm which does business through the middlemen acting in their own names, such as
indentors, commercial brokers or commission merchants, shall not be deemed doing
business in the Philippines. But such indentors, commercial brokers or commission
merchants shall be the ones deemed to be doing business in the Philippines.
Therefore, an indentor is a middleman in the same class as commercial brokers
and commission merchants. It would appear that there are three parties to an indent
transaction, namely, the buyer, the indentor, and the supplier who is usually a non-
resident manufacturer residing in the country where the goods are to be bought. An
indentor may therefore be best described as one who, for compensation, acts as a
middleman in bringing about a purchase and sale of goods between a foreign supplier
and a local purchaser.
In the case at bar, the admissions of the parties and the facts appearing on
record more than suffice to warrant the conclusion that SCHMID was not a vendor,
but was merely an indentor, in the questioned transaction. The afore-quoted penal
provision in the Corporation Law finds no application to SCHMID and its officers and
employees relative to the transactions in the instant case.
An indentor, acting in his own name, is not, however, covered by the above-
quoted provision. In fact, the provision of the Rules and Regulations implementing the
Omnibus Investments Code quoted above, which was copied from the Rules
implementing Republic Act No. 5455, recognizes the distinct role of an indentor, such
that when a foreign corporation does business through such indentor, the foreign
corporation is not deemed doing business in the Philippines. Not being the vendor,
SCHMID cannot be held liable for the implied warranty for hidden defects under the
Civil Code.

400 | P a g e
Law 321_Corporation LAW_ Case Digest

“Doing business” With or Without License: Suits By or Against Foreign Corporation

STEELCASE, INC.
vs.
DESIGN INTERNATIONAL SELECTIONS, INC.
G.R. No. 171995 April 18, 2012

FACTS:

Petitioner Steelcase, Inc. is a foreign corporation existing under the laws of


Michigan, United States of America (U.S.A.), and engaged in the manufacture of office
furniture with dealers worldwide. Respondent Design International Selections, Inc.
("DISI") is a corporation existing under Philippine Laws and engaged in the furniture
business, including the distribution of furniture.
Sometime in 1986 or 1987, Steelcase and DISI orally entered into a dealership
agreement whereby Steelcase granted DISI the right to market, sell, distribute, install,
and service its products to end-user customers within the Philippines. The business
relationship continued smoothly until it was terminated sometime in January 1999
after the agreement was breached with neither party admitting any fault. Steelcase
filed a complaint for sum of money against DISI alleging, among others, that DISI had
an unpaid account of US$600,000.00.

ISSUE:

Whether or not Steelcase is not doing business in the Philippines without


license.

RULING:

YES.

Based on this list, the Supreme Court said that the appointment of a
distributor in the Philippines is not sufficient to constitute "doing business" unless it
is under the full control of the foreign corporation. If the distributor is an independent
entity which buys and distributes products, other than those of the foreign
corporation, for its own name and its own account, the latter cannot be considered to
be doing business in the Philippines.
Applying these rules, the Supreme Court said that DISI was founded in 1979
and is independently owned and managed. In addition to Steelcase products, DISI also
distributed products of other companies including carpet tiles, relocatable walls and
theater settings. The dealership agreement between Steelcase and DISI had been
described by the owner himself as a buy and sell arrangement. This clearly belies
DISI‘s assertion that it was a mere conduit through which Steelcase conducted its
business in the country. From the preceding facts, the only reasonable conclusion that
can be reached is that DISI was an independent contractor, distributing various
products of Steelcase and of other companies, acting in its own name and for its own
account. As a result, Steelcase cannot be considered to be doing business in the
Philippines by its act of appointing a distributor as it falls under one of the exceptions
under R.A. No. 7042.
A foreign corporation doing business in the Philippines without a license may
maintain suit in the Philippines against a domestic corporation or person who is party
to a contract as the domestic corporation or person is deemed estopped from
challenging the personality of the foreign corporation.

401 | P a g e
Law 321_Corporation LAW_ Case Digest

Philippine Deposit Insurance Corporation


vs.
Citibank
GR NO.170290 April 11, 2012

FACTS:

Petitioner Philippine Deposit Insurance Corporation (PDIC) is a


government instrumentality created by virtue of Republic Act (R.A.) No. 3591, as
amended by R.A. No.9302.Respondent Citibank, N.A. (Citibank) is a banking
corporation while respondent Bank of America, S.T. & N.A. (BA) is a national banking
association, both of which are dulyorganized and existing under the laws of the United
States of America and duly licensed todo business in the Philippines, with offices in
Makati City.In 1977, PDIC conducted an examination of the books of account of
Citibank. It discovered that Citibank, in the course of its banking business, from
September 30, 1974 toJune 30, 1977, received from its head office and other foreign
branches a totalof P11,923,163,908.00 in dollars, covered by Certificates of Dollar
Time Deposit that wereinterest-bearing with corresponding maturity dates. These
funds, which were lodged inthe books of Citibank under the account ―Their Account -
Head Office/Branches-Foreign Currency,‖ were not reported to PDIC as deposit
liabilities that were subject to assessment for insurance. As such, in a
letter dated March 16, 1978, PDIC assessed Citibank fordeficiency in the sum of
P1,595,081.96.Similarly, sometime in 1979, PDIC examined the books of accounts of
BA whichrevealed that from September 30, 1976 to June 30, 1978, BA received from
its head officeand its other foreign branches a total of P629,311,869.10 in dollars,
covered by Certificatesof Dollar Time Deposit that were interest-bearing with
corresponding maturity dates and lodged in their books under the account ―Due to
Head Office/Branches.‖

ISSUE:

Whether or not a branch of a bank has a separate legal Personality.

RULING:

NO.

A branch has no separate legal personality. This Court is of the opinion that the
key to the resolution of this controversy is the relationship of the Philippine branches
of Citibank and BA to their respective head offices and their other foreign branches.
The Court begins by examining the manner by which a foreign corporation
canestablish its presence in the Philippines. It may choose to incorporate its own
subsidiary asa domestic corporation, in which case such subsidiary would have its
own separate andindependent legal personality to conduct business in the country. In
the alternative, it maycreate a branch in the Philippines, which would not be a legally
independent unit, andsimply obtain a license to do business in the Philippines.
In the case of Citibank and BA, it is apparent that they both did not incorporate
a separate domestic corporation to represent its business interests in
the Philippines. Their Philippine branches are, as the name implies, merely branches,
without a separate legal personality from their parent company, Citibank and BA.
Thus, being one and the same entity, the funds placed by the respondents in their
respective branches inthe Philippines should not be treated as deposits made by third
parties subject to deposit insurance under the PDIC Charter.
Deposit insurance is superfluous and entirely unnecessary when, as in this
case, the institution holding the funds and the one which made theplacements are one
and the same legal entity.

402 | P a g e
Law 321_Corporation LAW_ Case Digest

CARGILL, INC.
vs.
INTRA STRATA ASSURANCE CORPORATION
G.R. No. 168266 March 15, 2010

FACTS:

Cargill (foreign) is a corporation organized and existing under the laws of


theState of Delaware. Cargill executed a contract with Northern Mindanao Corporation
(NMC)(domestic), whereby NMC agreed to sell to petitioner 20,000 to 24,000
metrictons of molasses to be delivered from Jan 1 to 30 1990 for $44 per metric ton.
The contract provided that CARGILL was to open a Letter of Credit with theBPI. NMC
was permitted to draw up 500,000 representing the minimum priceof the contract.
The contract was amended 3 times (in relation to the amount and the price).But the
third amendment required NMC to put up a performance bond whichwas intended to
guarantee NMC‘s performance to deliver the molasses duringthe prescribed shipment
periods.
In compliance, INTRA STRATA issued a performance bond to guaranteeNMC‘s
delivery. NMC was only able to deliver 219551 metric tons out of the agreed
10,500.Thus CARGILL sent demand letters to INTRA claiming payment under
theperformance and surety bonds. When INTRA failed to pay, CARGILL filed
acomplaint.
CARGILL NMC and INTRA entered into a compromise agreement approvedby
the court, such provided that NMC would pay CARGILL 3 million uponsigning and
would deliver to CARGILL 6,991 metric tons of molasses. ButNMC still failed to
comply.

ISSUE:

Whether or not petitioner is doing or transacting business in the Philippines in


contemplation of the law and established jurisprudence.

RULING:

NO.

The determination of whether a foreign corporation is doing business in the


Philippines must be based on the facts of each case. In the case at bar, the
transactions entered into by the respondent with the petitioners are not a series of
commercial dealings which signify an intent on the part of the respondent to do
business in the Philippines but constitute an isolated one which does not fall under
the category of "doing business." The records show that the only reason why the
respondent entered into the second and third transactions with the petitioners was
because it wanted to recover the loss it sustained from the failure of the petitioners to
deliver the crude coconut oil under the first transaction and in order to give the latter
a chance to make good on their obligation.
In the present case, petitioner is a foreign company merely importing molasses
from a Philipine exporter. A foreign company that merely imports goods from a
Philippine exporter, without opening an office or appointing an agent in the
Philippines, is not doing business in the Philippines.

403 | P a g e
Law 321_Corporation LAW_ Case Digest

SEHWANI, INC.
vs.
IN-N-OUT BURGER
GR No. 171053 15 October 2007

FACTS:

Respondent IN-N-OUT Burger, Inc., a foreign corporation organized under the


laws of California, U.S.A., and not doing business in the Philippines, filed before the
Bureau of Legal Affairs of the IPO (BLA-IPO), an administrative complaint against
petitioners Sehwani, Inc. and Benita‘s Frites, Inc. for violation of intellectual property
rights, attorney‘s fees and damages with prayer for the issuance of a restraining order
or writ of preliminary injunction in registering, as it did, the mark claimed by
respondent.
The BLA-IPO held that petitioner is not guilty of unfair competition although
respondent has the capacity to sue and ordered petitioner to cease and desist from
using the mark which the higher courts affirmed.

ISSUE:

Whether or not respondent has legal capacity to sue.

RULING:

YES.

Respondent has the legal capacity to sue for the protection of its trademarks,
albeit it is not doing business in the Philippines. Section 160 in relation to Section 3
of R.A. No. 8293, provides:
SECTION 160. Right of Foreign Corporation to Sue in Trademark or Service
Mark Enforcement Action. — Any foreign national or juridical person who meets the
requirements of Section 3 of this Act and does not engage in business in the
Philippines may bring a civil or administrative action hereunder for opposition,
cancellation, infringement, unfair competition, or false designation of origin and false
description, whether or not it is licensed to do business in the Philippines under
existing laws.
Section 3 thereof provides:
SECTION 3. International Conventions and Reciprocity. — Any person who is
a national or who is domiciled or has a real and effective industrial establishment in a
country which is a party to any convention, treaty or agreement relating to intellectual
property rights or the repression of unfair competition, to which the Philippines is also
a party, or extends reciprocal rights to nationals of the Philippines by law, shall be
entitled to benefits to the extent necessary to give effect to any provision of such
convention, treaty or reciprocal law, in addition to the rights to which any owner of an
intellectual property right is otherwise entitled by this Act.
The question of whether or not respondent‘s trademarks are considered ―well-
known‖ is factual in nature, involving as it does the appreciation of evidence adduced
before the BLA-IPO. The settled rule is that the factual findings of quasi-judicial
agencies, like the IPO, which have acquired expertise because their jurisdiction is
confined to specific matters, are generally accorded not only respect, but, at times,
even finality if such findings are supported by substantial evidence.
This being the case, the issue on the legal capacity to sue of the respondent is
answered in the affirmative.

404 | P a g e
Law 321_Corporation LAW_ Case Digest

LORENZO SHIPPING CORP.


vs. CHUBB and SONS, Inc. et al
G.R. No. 147724 June 8, 2004

FACTS:

Lorenzo Shipping Corporation is a domestic corporation engaged in coastwise


shipping. Gearbulk ltd. is a foreign corporation licensed as a common carrier under
the laws of Norway and doing business in the Philippines through its agent Philippine
Transmarine Carrier Inc. Philippine Transmarine Carrier (PTC) is the agent of
Gearbulk in the Philippines. Sumitomo Corporation (SC), a foreign corporation
organized under the laws of the USA is the consignee. It insured the shipment with
Chubb and Sons, Inc., a foreign corporation organized and licensed to engage in
insurance business under the laws of the United States of America.
On November 21, 1987, Mayer Steel Pipe Corporation of Binondo, Manila,
loaded 581 bundles of black steel on board one of the vessels of LS for shipment to
Davao City from Manila. Upon reaching the Sasa Wharf in Davao on Dec.2, PTC
discovered that the steels were submerged in seawater. An inspector made a finding
that the steel pipes were no longer in good condition for rust were already forming.
Gearbulk noted the damage and shipped the pipes on its vessel to the USA .
Due to its heavily rusted condition, the consignee Sumitomo rejected the damaged
steel pipes and declared them unfit for the purpose they were intended. It then filed a
marine insurance claim with respondent Chubb and Sons, Inc.

ISSUE:

Whether or not Chubb and Sons, as a mere assignee of a foreign corporation


which has no authority to sue in the Philippines, has capacity to sue before the
Philippine courts.

RULING:

YES.

Subrogration contemplates full substitution such that it places the party


subrogated in the shoes of the creditor, and he may use all means which the creditor
could employ to enforce payment. The rights to which the subrogee succeeds are the
same as, but not greater than, those of the person for whom he is substituted. The law
on corporations is clear in depriving foreign corporations which are doing business in
the Philippines without a license from bringing or maintaining actions before, or
intervening in Philippine courts but a foreign corporation needs no license to sue
before Philippine courts on an isolated transaction.
For "doing business" is not really the number or the quantity of the
transactions, but more importantly, the intention of an entity to continue the body of
its business in the country. The phrase "isolated transaction" has a definite and fixed
meaning, i.e. a transaction or series of transactions set apart from the common
business of a foreign enterprise in the sense that there is no intention to engage in a
progressive pursuit of the purpose and object of the business organization. Whether a
foreign corporation is "doing business" does not necessarily depend upon the
frequency of its transactions, but more upon the nature and character of the
transactions. Furthermore, respondent insurer Chubb and Sons, by virtue of the right
of subrogation provided for in the policy of insurance, is the real party in interest in
the action for damages before the court a quo against the carrier Lorenzo Shipping to
recover for the loss sustained by its insured.
It then, thus possesses the right to enforce the claim and the significant interest
in the litigation. In the case at bar, it is clear that respondent insurer was suing on its
own behalf in order to enforce its right of subrogation.

405 | P a g e
Law 321_Corporation LAW_ Case Digest

MR HOLDINGS, Ltd.
vs.
BAJAR
G.R. No. 138104 April 11, 2002

FACTS:

ADB extended a loan to Marcopper under a Principal Loan Agreement and


Complementary Loan Agreement. A Support and Standby Credit Agreement was also
executed between ADB and Placer Dome (owner of 40% of Marcopper), whereby the
latter agreed to provide with a cash flow support for the payment of its obligations to
ADB. Marcopper also executed a Deed of Real Estate and Chattel Mortgage in favor of
ADB covering all its assets in Marinduque. Marcopper defaulted in its payment. Thus,
MR Holding, LTD (placer Dome‘s subsidiary corporation) assumed Marcopper‘s
obligation to ADB. Marcopper likewise executed a Deed of assignment in favor of
petitioner.
It appeared that SolidBank Corporation obtained a partial judgment against
Marcopper in a case filed with the RTC. A writ of execution was issued and then an
auction sale was scheduled. This event prompted petitioner to serve an "Affidavit of
Third-Party Claim" upon respondent sheriffs, asserting ownership over all the assets of
Marcopper by virtue of the Deed of Assignment. The RTC of Manila denied the
affidavit.
Petitioner filed with the RTC of Boac, Marinduque a complaint for reivindication
of properties with prayer for preliminary injunction and temporary restraining order
against respondents. The application for writ of preliminary injunction was denied.

ISSUE:

Whether or not petitioner is doing business in the Philippines.

RULING:

NO.

There are other statutes defining the term "doing business" in the same tenor as
those above-quoted, and as may be observed, one common denominator among them
all is the concept of "continuity."
The expression "doing business" should not be given such a strict and literal
construction as to make it apply to any corporate dealing whatever. At this early stage
and with petitioner‘s acts or transactions limited to the assignment contracts, it
cannot be said that it had performed acts intended to continue the business for which
it was organized. It may not be amiss to point out that the purpose or business for
which petitioner was organized is not discernible in the records. No effort was exerted
by the Court of Appeals to establish the nexus between petitioner‘s business and the
acts supposed to constitute "doing business." Thus, whether the assignment contracts
were incidental to petitioner‘s business or were continuation thereof is beyond
determination.
Significantly, a view subscribed upon by many authorities is that the mere
ownership by a foreign corporation of a property in a certain state, unaccompanied by
its active use in furtherance of the business for which it was formed, is insufficient in
itself to constitute doing business.
In the final analysis, we are convinced that petitioner was engaged only in
isolated acts or transactions. Single or isolated acts, contracts, or transactions of
foreign corporations are not regarded as a doing or carrying on of business. Typical
examples of these are the making of a single contract, sale, sale with the taking of a
note and mortgage in the state to secure payment therefor, purchase, or note, or the
mere commission of a tort. In these instances, there is no purpose to do any other
business within the country.

406 | P a g e
Law 321_Corporation LAW_ Case Digest

THE COMMISSIONER OF CUSTOMS


vs.
K.M.K. GANI, INDRAPAL & CO
G.R. No. 73722 February 26, 1990

FACTS:

Two containers loaded with 103 cartons of merchandise covered by eleven (11)
airway bills of several supposedly Singapore-based consignees arrived at the Manila
International Airport on board Philippine Air Lines (PAL) from Hongkong. The cargoes
were consigned to these different entities: K.M.K Gani (hereafter referred to as K.M.K.)
and Indrapal and Company (hereafter referred to as INDRAPAL), the private
respondents in the petition before us; and Sin Hong Lee Trading Co., Ltd., AAR TEE
Enterprises, and C. Ratilal, all purportedly based in Singapore.
The Bureau's agency on such matters, the Suspected Cargo and Anti-Narcotics
(SCAN), dispatched an agent to verify the information. The SCAN agent saw an empty
PAL van parked directly alongside the plane's belly from which cargoes were being
unloaded. When the SCAN agent asked the van's driver why he was at the site, the
driver drove away in his vehicle. The SCAN agent then sequestered the unloaded
cargoes.
These goods were transferred to the International Cargo Terminal under
Warrant of Seizure and Detention and thereafter subjected to Seizure and Forfeiture
proceedings for "technical smuggling.

ISSUE:

Whether or not the private respondents failed to establish their personality to


sue in a representative capacity, hence making their action dismissible.

RULING:

YES.

The law is clear: "No foreign corporation transacting business in the Philippines
without a license, or its successors or assigns, shall be permitted to maintain or
intervene in any action, suit or proceeding in any court or administrative agency of the
Philippines; but such corporation may be sued or proceeded against before Philippine
courts or administrative tribunals on any valid cause of action recognized under
Philippine laws."
However, the Court in a long line of cases has held that a foreign corporation
not engaged in business in the Philippines may not be denied the right to file an action
in the Philippine courts for an isolated transaction. The fact that a foreign corporation
is not doing business in the Philippines must be disclosed if it desires to sue in
Philippine courts under the "isolated transaction rule." Without this disclosure, the
court may choose to deny it the right to sue.
In the case at bar, the private respondents K.M.K. and INDRAPAL aver that they
are "suing upon a singular and isolated transaction." But they failed to prove their
legal existence or juridical personality as foreign corporations.
Under the "isolated transaction rule," only foreign corporations and not just any
business organization or entity can avail themselves of the privilege of suing before
Philippine courts even without a license. The first paragraph of their petition before
the Court, containing the allegation of their identities, does not even aver their
corporate character. On the contrary, K.M.K. alleges that it is a "single proprietorship"
while INDRAPAL hides under the vague identification as a "firm," although both
describe themselves with the phrase "doing business in accordance with the laws of
Singapore."
Absent such proof that the private respondents are corporations (foreign or not),
the respondent Court of Tax Appeals should have barred their invocation of the right
to sue within Philippine jurisdiction under the "isolated transaction rule" since they do
not qualify for the availment of such right.

407 | P a g e
Law 321_Corporation LAW_ Case Digest

COMMUNICATION MATERIALS and DESIGN, INC.


vs.
COURT OF APPEALS
G.R. No. 102223 August 22, 1996

FACTS:

Petitioners COMMUNICATION MATERIALS AND DESIGN, INC., (CMDI, for


brevity) and ASPAC MULTI-TRADE INC., (ASPAC, for brevity) are both domestic
corporations, while petitioner Francisco S. Aguirre is their President and majority
stockholder. Private Respondents ITEC, INC. and/or ITEC, INTERNATIONAL, INC.
(ITEC, for brevity) are corporations duly organized and existing under the laws of the
State of Alabama, United States of America. There is no dispute that ITEC is a foreign
corporation not licensed to do business in the Philippines.
ITEC entered into a contract with petitioner ASPAC referred to as
"Representative Agreement". ITEC engaged ASPAC as its "exclusive representative" in
the Philippines for the sale of ITEC's products, in consideration of which, ASPAC was
paid a stipulated commission. ASPAC was able to incorporate and use the name
"ITEC" in its own name. Thus , ASPAC Multi-Trade, Inc. became legally and publicly
known as ASPAC-ITEC (Philippines).
ASPAC sold electronic products, exported by ITEC, to their sole customer, the
Philippine Long Distance Telephone Company, (PLDT, for brevity). ITEC charges the
petitioners and another Philippine Corporation, DIGITAL BASE COMMUNICATIONS,
INC. (DIGITAL, for brevity), the President of which is likewise petitioner Aguirre, of
using knowledge and information of ITEC's products specifications to develop their
own line of equipment and product support, which are similar, if not identical to
ITEC's own, and offering them to ITEC's former customer.

ISSUE:

Whether or not private respondent ITEC is an unlicensed corporation doing


business in the Philippines, and if it is, whether or not this fact bars it from invoking
the injunctive authority of our courts.

RULING:

YES.

Generally, a "foreign corporation" has no legal existence within the state in


which it is foreign. This proceeds from the principle that juridical existence of a
corporation is confined within the territory of the state under whose laws it was
incorporated and organized, and it has no legal status beyond such territory. Such
foreign corporation may be excluded by any other state from doing business within its
limits, or conditions may be imposed on the exercise of such privileges.
The purpose of the law in requiring that foreign corporations doing business in
the Philippines be licensed to do so and that they appoint an agent for service of
process is to subject the foreign corporation doing business in the Philippines to the
jurisdiction of its courts. The object is not to prevent the foreign corporation from
performing single acts, but to prevent it from acquiring a domicile for the purpose of
business without taking steps necessary to render it amenable to suit in the local
courts.
The implication of the law is that it was never the purpose of the legislature to
exclude a foreign corporation which happens to obtain an isolated order for business
from the Philippines, and thus, in effect, to permit persons to avoid their contracts
made with such foreign corporations.
With the abovestated precedents in mind, we are persuaded to conclude that
private respondent had been "engaged in" or "doing business" in the Philippines for
some time now. This is the inevitable result after a scrutiny of the different contracts
and agreements entered into by ITEC with its various business contacts in the
country, particularly ASPAC and Telephone Equipment Sales and Services, Inc.

408 | P a g e
Law 321_Corporation LAW_ Case Digest

COLUMBIA PICTURES, INC.


vs.
COURT OF APPEALS
G.R. No. 110318 August 28, 1996

FACTS:

Complainants thru counsel lodged a formal complaint with the NBI for violation
of PD No. 49 and sought its assistance in their anti-film piracy drive. Agents of the
NBI and private researchers made discreet surveillance on various video
establishments in Metro Manila including Sunshine Home Video Inc., owned and
operated by Danilo A. Pelindario.
NBI Senior Agent Lauro C. Reyes applied for a search warrant with the court a
quo against Sunshine seeking the seizure, among others, of pirated video tapes of
copyrighted films all of which were enumerated in a list attached to the application;
and, television sets, video cassettes and/or laser disc recordings equipment and other
machines and paraphernalia used or intended to be used in the unlawful exhibition,
showing, reproduction, sale, lease or disposition of videograms tapes in the premises
above described.
The search warrant was served to Sunshine and/or their representatives. In
the course of the search of the premises indicated in the search warrant, the NBI
Agents found and seized various video tapes of duly copyrighted motion pictures/films
owned or exclusively distributed by private complainants, and machines, equipment,
television sets, paraphernalia, materials, accessories all of which were included in the
receipt for properties accomplished by the raiding team. Copy of the receipt was
furnished and/or tendered to Mr. Danilo A. Pelindario, registered owner-proprietor of
Sunshine Home Video.

ISSUE:

Whether or not petitioners have capacity to sue in court.

RULING:

NO.

The obtainment of a license prescribed by Section 125 of the Corporation Code


is not a condition precedent to the maintenance of any kind of action in Philippine
courts by a foreign corporation.
As thus interpreted, any foreign corporation not doing business in the
Philippines may maintain an action in our courts upon any cause of action, provided
that the subject matter and the defendant are within the jurisdiction of the court. It is
not the absence of the prescribed license but ―doing business‖ in the Philippines
without such license which debars the foreign corporation from access to our courts.
In other words, although a foreign corporation is without license to transact
business in the Philippines, it does not follow that it has no capacity to bring an
action. Such license is not necessary if it is not engaged in business in the
Philippines. Based on Article 133 of the Corporation Code and gauged by such
statutory standards, petitioners are not barred from maintaining the present action.
There is no showing that, under our statutory or case law, petitioners are doing,
transacting, engaging in or carrying on business in the Philippines as would require
obtention of a license before they can seek redress from our courts. No evidence has
been offered to show that petitioners have performed any of the enumerated acts or
any other specific act indicative of an intention to conduct or transact business in the
Philippines.
Accordingly, the certification issued by the Securities and Exchange
Commission stating that its records do not show the registration of petitioner film
companies either as corporations or partnerships or that they have been licensed to
transact business in the Philippines, while undeniably true, is of no consequence to
petitioners‘ right to bring action in the Philippines.

409 | P a g e
Law 321_Corporation LAW_ Case Digest

ERIKS PTE. LTD.


vs.
COURT OF APPEALS
G.R. No. 118843 February 6, 1997

FACTS:

Petitioner is a non-resident foreign corporation engaged in the manufacture and


sale of elements used in sealing pumps, valves and pipes for industrial purposes,
valves and control equipment used for industrial fluid control and PVC pipes and
fittings for industrial uses.
On various dates, private respondent Delfin Enriquez, Jr., doing business
under the name and style of Delrene EB Controls Center and/or EB Karmine
Commercial, ordered and received from petitioner various elements used in sealing
pumps, valves, pipes and control equipment, PVC pipes and fittings.
The transfers of goods were perfected in Singapore, for private respondent's
account, F.O.B. Singapore, with a 90-day credit term. Subsequently, demands were
made by petitioner upon private respondent to settle his account, but the latter
failed/refused to do so.

ISSUE:

Whether Petitioner Corporation may maintain an action in Philippine courts


considering that it has no license to do business in the country.

RULING:

YES.

The resolution of this issue depends on whether petitioner's business with


private respondent may be treated as isolated transactions. Granting that there is no
distributorship agreement between herein parties, yet by the mere fact that plaintiff,
each time that the defendant posts an order delivers the items as evidenced by the
several invoices and receipts of various dates only indicates that plaintiff has the
intention and desire to repeat the said transaction in the future in pursuit of its
ordinary business. Furthermore, "and if the corporation is doing that for which it was
created, the amount or volume of the business done is immaterial and a single act of
that character may constitute doing business.
More than the sheer number of transactions entered into, a clear and
unmistakable intention on the part of petitioner to continue the body of its business in
the Philippines is more than apparent. As alleged in its complaint, it is engaged in the
manufacture and sale of elements used in sealing pumps, valves, and pipes for
industrial purposes, valves and control equipment used for industrial fluid control and
PVC pipes and fittings for industrial use. Thus, the sale by petitioner of the items
covered by the receipts, which are part and parcel of its main product line, was
actually carried out in the progressive prosecution of commercial gain and the pursuit
of the purpose and object of its business, pure and simple.
Further, its grant and extension of 90-day credit terms to private respondent for
every purchase made, unarguably shows an intention to continue transacting with
private respondent, since in the usual course of commercial transactions, credit is
extended only to customers in good standing or to those on whom there is an intention
to maintain long-term relationship.
This being so, the existence of a distributorship agreement between the parties,
as alleged but not proven by private respondent, would, if duly established by
competent evidence, be merely corroborative, and failure to sufficiently prove said
allegation will not significantly affect the finding of the courts below.
Thus, the court holds that the series of transactions in question could not have
been isolated or casual transactions. What is determinative of "doing business" is not
really the number or the quantity of the transactions, but more importantly, the
intention of an entity to continue the body of its business in the country. The number
and quantity are merely evidence of such intention.

410 | P a g e
Law 321_Corporation LAW_ Case Digest

FAR EAST INTERNATIONAL


vs.
NANKAI KOGYO
L- 13525 NOVEMBER 3, 1962

FACTS:

Far East entered into a contract of sale of steel scrap with Nankai , a foreign
corporation incorporated under Japanese laws. Nankai opened a letter of credit with
the China Banking Corporation issued by Nippon Kangyo. Four days before the
expiration of the Far East license, three boats of Nankai arrived. Upon the expiration
of the export license, only 1, 058.6 metric tons of steel scrap was loaded on the SS
Mina. The license was never renewed.
On April 27, 1957, Nankai confirmed and acknowleged delivery of the 1,058.6
metric tons of steel scrap, but asked for damages amounting to $148,135.00
consisting of dead freight charges, damages, bank charges, phone and cable expenses.
On May 4, 1957, Far East wrote the Everett Steamship Corporation, requesting
the issuance of a complete set of the Bill of Lading for the shipment, in order that
payment thereof be effected against the Letter of Credit. Under date of May 7, 1957,
the Everett informed Far East that they were not in a position to comply because the
Bill of Lading was issued and signed in Tokyo by the Master of the boat, upon request
of the Charterer, defendant herein.
As repeated requests, both against the shipping agent and the buyers (Nankai),
for the issuance of the of Bill Lading were ignored, Far East filed on May 16, 1957, the
present complaint for Specific Performance, damages, a writ of preliminiry mandatory
injunction directed against Nankai and the shipping company, to issue and deliver to
the plaintiff, a complete set of negotiable of Lading for the 1,058.6 metric tons of scrap
and a writ of preliminary injunction against the China Banking Corporation and the
Nankai to maintain the Letter Credit.
Far East filed for the issuance of a complete set of Bill of Lading in order that
payment be effected against the Letter of Credit. The respondent refused.

ISSUE:

Whether or not the court had jurisdiction over the subject matter and the
person of the defendant.

RULING:

YES.

Mr. Ishida who personally signed the contract for the purpose of selling scrap in
question in behalf on Nankai Kogyo is the Trade Manager of the said corporation. Mr.
Tominaga was the Chief of the Petroleum Section of the same company and Mr.
Yoshida was the man- in- charge of the Import Section of the company‘s Tokyo
branch. All of these officers are served with summons.
The testimony of Atty. Pablo Ocampo that appellant was doing business in the
Philippines was corroborated by no less than Nabuo Yoshida, one of the appellant‘s
officers, that he was sent to the Philippines by his company to look into the operation
of mines, thereby revealing the defendant‘s desire to continue engaging business in
the Philippines, after receiving the shipment of the scrap iron under consideration,
making the Philippines a base thereof.
That a single act may bring the corporation within the purview of the statute
where it is an act of the ordinary business of the corporation. The single act or
transaction is not merely incidental or casual, but is of such character as distinctly to
indicate a purpose on the part of the foreign corporation to do other business in the
state, and to make the state a basis of operations for the conduct of a part of the
corporation‘s ordinary business.

411 | P a g e
Law 321_Corporation LAW_ Case Digest

FACILITIES MANAGEMENT CORPORATION


vs.
DE LA OSA
G.R. No. L-38649 March 26, 1979

FACTS:

Leonardo dela Osa sought his reinstatement with full backwages, as well as the
recovery of his overtime compensation, swing shift and graveyard shift differentials.
Petitioner alleged that he was employed by respondents as follows: (1) painter (2)
houseboy with an hourly rate of $1.26 from December, 1964 to November, 1965,
inclusive; (3) houseboy with an hourly rate of $1.33 from December, 1965 to August,
1966, inclusive; and (4) cashier with an hourly rate of $1.40 from August, 1966 to
March 27, 1967, inclusive.
He further averred that from December, 1965 to August, 1966, inclusive, he
rendered overtime services daily, and that this entire period was divided into swing
and graveyard shifts to which he was assigned, but he was not paid both overtime and
night shift premiums despite his repeated demands from respondents.
Respondents filed on August 7, 1967 their letter- answer without substantially
denying the material allegations of the basic petition but interposed the following
special defenses, namely: That respondents Facilities Management Corporation and J.
S. Dreyer are domiciled in Wake Island which is beyond the territorial jurisdiction of
the Philippine Government; that respondent J. V. Catuira, though an employee of
respondent corporation presently stationed in Manila, is without power and authority
of legal representation; and that the employment contract between petitioner and
respondent corporation carries -the approval of the Department of Labor of the
Philippines.

ISSUE:

Whether or not the mere act by a non-resident foreign corporation of recruiting


Filipino workers for its own use abroad is in law doing business in the Philippines.

RULING:

YES.

"Under the rules and regulations promulgated by the Board of Investments


which took effect Feb. 3, 1969, implementing Rep. Act No. 5455, which took effect
Sept. 30, 1968, the phrase 'doing business' has been exemplified with illustrations,
among them being as follows: "the performance within the Philippines of any act or
combination of acts enumerated in section 1(1) of the Act shall constitute 'doing
business' therein. In particular, 'doing business' includes: "(1)Soliciting orders,
purchases (sales) or service contracts. Concrete and specific solicitations by a foreign
firm, not acting independently of the foreign firm, amounting to negotiation or fixing of
the terms and conditions of sales or service contracts, regardless of whether the
contracts are actually reduced to writing, shall constitute doing business even if the
enterprise has no office or fixed place of business in the Philippines. "(2)Appointing a
representative or distributor who is domiciled in the Philippines, unless said
representative or distributor has an independent status, i.e., it transacts business in
its name and for its own account, and not in the name or for the account of the
principal. "(4)Opening offices, whether called 'liaison' offices, agencies or branches,
unless proved otherwise. "(10)Any other act or acts that imply a continuity of
commercial dealings or arrangements, and contemplate to that extent the performance
of acts or works, or the exercise of some of the functions normally incident to, or in the
progressive prosecution of, commercial gain or of the purpose and objective of the
business organization."

412 | P a g e
Law 321_Corporation LAW_ Case Digest

H.B. ZACHRY COMPANY INTERNATIONAL


vs.
COURT OF APPEALS
G.R. No. 106989 May 10, 1994

FACTS:

VBC entered into a written Subcontract Agreement with Zachry, a foreign


corporation. The latter had been engaged by the United States Navy to design and
construct 264 Family Housing Units at the US Naval Base at Subic, Zambales. Under
the agreement, specifically under Section 3 on Payment, VBC was to perform all the
construction work on the housing project and would be paid "for the performance of
the work the sum (U.S.$6,468,000.00)
When VBC had almost completed the project, Zachry complained of the quality
of work, making it a reason for its decision to take over the management of the project,
which paragraph c, Section 7 of the Subcontract Agreement authorized.
However, prior to such take-over, the parties executed an agreement, in
accordance with which, VBC submitted to Zachry a detailed computation of the cost to
complete the subcontract on the housing project. According to VBC's computation,
there remains a balance of $1,103,000.00 due in its favor as of 18 January 1990.
Zachry, however, not only refused to acknowledge the indebtedness but continually
failed to submit to VBC a statement of accumulated costs, as a result of which VBC
was prevented from cheking the accuracy of the said costs. On 2 March 1990, VBC
wrote Zachry a letter demanding compliance with its obligations. Zachry still failed to
do so. VBC made representations to pursue its claim, including a formal claim with
the Officer-in-Charge of Construction, NAVFAC Contracts, Southwest Pacific, which
also failed.

ISSUE:

Whether or not a writ of preliminary attachment may issue ex parte against a


defendant before acquisition of jurisdiction of the latter's person by service of
summons or his voluntary submission to the Court's authority.

RULING:

NO.

The Court rules that the question must be answered in the affirmative and that
consequently, the petition for review will have to be granted. It is incorrect to theorize
that after an action or proceeding has been commenced and jurisdiction over the
person of the plaintiff has been vested in the court, but before the acquisition of
jurisdiction over the person of the defendant (either by service of summons or his
voluntary submission to the court's authority), nothing can be validly done by the
plaintiff or the court.
It is wrong to assume that the validity of acts done during this period should be
dependent on, or held in suspension until, the actual obtention of jurisdiction over the
defendant's person. The obtention by the court of jurisdiction over the person of the
defendant is one thing; quite another is the acquisition of jurisdiction over the person
of the plaintiff or over the subject-matter or nature of the action, or the res or object
thereof.
For the guidance of all concerned, the Court reiterates and reaffirms the
proposition that writs of attachment may properly issue ex parte provided that the
Court is satisfied that the relevant requisites therefore have been fulfilled by the
applicant, although it may, in its discretion, require prior hearing on the application
with notice to the defendant; but that levy on property pursuant to the writ thus
issued may not be validly effected unless preceded, or contemporaneously
accompanied, by service on the defendant of summons, a copy of the complaint (and
of the appointment of guardian ad litem, if any), the application for attachment (if not
incorporated in but submitted separately from the complaint), the order of
attachment, and the plaintiff's attachment bond.

413 | P a g e
Law 321_Corporation LAW_ Case Digest

HUTCHISON PORTS PHILIPPINES LIMITE


vs.
SBMA
G.R. No. 131367 August 31, 2000

FACTS:

The Subic Bay Metropolitan Authority (or SBMA) advertised in leading national
daily newspapers and in one international publication, an invitation offering to the
private sector the opportunity to develop and operate a modern marine container
terminal within the Subic Bay Freeport Zone. Out of seven bidders who responded to
the published invitation, three were declared by the SBMA as qualified bidders after
passing the pre-qualification evaluation conducted by the SBMA‘s Technical
Evaluation Committee (or SBMA-TEC). Among these is the petitioner.
Thereafter, the services of three (3) international consultants recommended by
the World Bank for their expertise were hired by SBMA to evaluate the business plans
submitted by each of the bidders, and to ensure that there would be a transparent
and comprehensive review of the submitted bids. The SBMA also hired the firm of
Davis, Langdon and Seah Philippines, Inc. to assist in the evaluation of the bids and
in the negotiation process after the winning bidder is chosen. All the consultants, after
such review and evaluation unanimously concluded that HPPL‘s Business Plan was
―far superior to that of the two other bidders.‖
However, even before the sealed envelopes containing the bidders‘ proposed
royalty fees could be opened at the appointed time and place, RPSI formally protested
that ICTSI is legally barred from operating a second port in the Philippines based on
Executive Order No. 212 and Department of Transportation and Communication
(DOTC) Order 95-863.

ISSUE:

Whether or not petitioner HPPL has the legal capacity to even seek redress from
the Court.

RULING:

YES.

Admittedly, petitioner HPPL is a foreign corporation, organized and existing


under the laws of the British Virgin Islands. While the actual bidder was a consortium
composed of petitioner, and two other corporations, namely, Guoco Holdings (Phils.)
Inc. and Unicol Management Services, Inc., it is only petitioner HPPL that has brought
the controversy before the Court, arguing that it is suing only on an isolated
transaction to evade the legal requirement that foreign corporations must be licensed
to do business in the Philippines to be able to file and prosecute an action before
Philippines courts.
There is no general rule or governing principle laid down as to what constitutes
―doing‖ or ―engaging in‖ or ―transacting‖ business in the Philippines. Each case must
be judged in the light of its peculiar circumstances. Thus, it has often been held that a
single act or transaction may be considered as ―doing business‖ when a corporation
performs acts for which it was created or exercises some of the functions for which it
was organized. The amount or volume of the business is of no moment, for even a
singular act cannot be merely incidental or casual if it indicates the foreign
corporation‘s intention to do business.
Participating in the bidding process constitutes ―doing business‖ because it
shows the foreign corporation‘s intention to engage in business here. The bidding for
the concession contract is but an exercise of the corporation‘s reason for creation or
existence. Thus, it has been held that ―a foreign company invited to bid for IBRD and
ADB international projects in the Philippines will be considered as doing business in
the Philippines for which a license is required.‖

414 | P a g e
Law 321_Corporation LAW_ Case Digest

LA CHEMISE LACOSTE, S. A.
vs.
FERNANDEZ
G.R. No. L-65659 May 21, 1984

FACTS:

The petitioner is a foreign corporation, organized and existing under the laws of
France and not doing business in the Philippines. It is the actual owner of the
abovementioned trademarks used on clothings and other goods specifically sporting
apparels sold in many parts of the world and which have been marketed in the
Philippines since 1964. The main basis of the private respondent's case is its claim of
alleged prior registration.
In 1975, Hemandas & Co., a duly licensed domestic firm applied for and was
issued Reg. No. SR-2225 (SR stands for Supplemental Register) for the trademark
"CHEMISE LACOSTE & CROCODILE DEVICE" by the Philippine Patent Office for use
on T-shirts, sportswear and other garment products of the company. Two years later,
it applied for the registration of the same trademark under the Principal Register. The
Patent Office eventually issued an order which granted the application."Thereafter,
Hemandas & Co. assigned to respondent Gobindram Hemandas all rights, title, and
interest in the trademark "CHEMISE LACOSTE & DEVICE".
The petitioner filed its application for registration of the trademark "Crocodile
Device" and "Lacoste". The former was approved for publication while the latter was
opposed by Games and Garments. The petitioner filed with the National Bureau of
Investigation (NBI) a letter-complaint alleging therein the acts of unfair competition
being committed by Hemandas and requesting their assistance in his apprehension
and prosecution.

ISSUE:

Whether or not petitioner has the capacity to sue.

RULING:

YES.

The petitioner is a foreign corporation not doing business in the Philippines.


The marketing of its products in the Philippines is done through an exclusive
distributor, Rustan Commercial Corporation. The latter is an independent entity which
buys and then markets not only products of the petitioner but also many other
products bearing equally well-known and established trademarks and tradenames. In
other words, Rustan is not a mere agent or conduit of the petitioner.
The court finds and concludes that the petitioner is not doing business in the
Philippines. Rustan is actually a middleman acting and transacting business in its
own name and or its own account and not in the name or for the account of the
petitioner. More important is the nature of the case which led to this petition. What
preceded this petition for certiorari was a letter-complaint filed before the NBI charging
Hemandas with a criminal offense, i.e., violation of Article 189 of the Revised Penal
Code. If prosecution follows after the completion of the preliminary investigation being
conducted by the Special Prosecutor the information shall be in the name of the
People of the Philippines and no longer the petitioner which is only an aggrieved party
since a criminal offense is essentially an act against the State.
It is the latter which is principally the injured party although there is a private
right violated. Petitioner's capacity to sue would become, therefore, of not much
significance in the main case. We cannot allow a possible violator of our criminal
statutes to escape prosecution upon a far-fetched contention that the aggrieved party
or victim of a crime has no standing to sue. In upholding the right of the petitioner to
maintain the present suit before our courts for unfair competition or infringement of
trademarks of a foreign corporation, we are moreover recognizing our duties and the
rights of foreign states under the Paris Convention for the Protection of Industrial
Property to which the Philippines and France are parties.

415 | P a g e
Law 321_Corporation LAW_ Case Digest

MARUBENI NEDERLAND B.V.


vs.
TENSUAN
G.R. NO. 61950 SEPTEMBER 28, 1990

FACTS:

On October 23, 1976, in Tokyo, Japan, petitioner Marubeni Nederland B.V. and
D.B. Teodoro Development Corporation entered into a contract whereby petitioner
agreed to supply all the necessary equipment, machinery, materials, technical know-
how and the general design of the construction of DBT's lime plant at the Guimaras
Islands in Iloilo for a total contract price of US$5,400,000.00 on a deferred payment
basis.
Simultaneously with the supply contract, the parties entered into two financing
contracts, namely a construction loan agreement in the amount of US$1,600,000.00
and a cash loan agreement for US$1,500,000.00. The obligation of DBT to pay the
loan amortizations on their due dates under the three (3) contracts were absolutely
and unconditionally guaranteed by the National Investment and Development
Corporation (NIDC).
Pursuant to the terms of the financing contracts, the loan amortizations of DBT
fell due on January 7, 1980, July 7, 1980 and January 7, 1981. But before the first
installment became due, DBT wrote a letter to the NIDC interposing certain claims
against the petitioner and at the same time requesting NIDC for a revision of the
repayment schedule and of the amounts due under the contracts on account of
petitioner's delay in the performance of its contractual commitments. In due time, the
problems regarding the lime plant were ironed out and the parties signed a
"Settlement Agreement" on July 2, 1981.

ISSUE:

Whether or not petitioner Marubeni Nederland B.V. can be considered as "doing


business" in the Philippines and therefore subject to the jurisdiction of our courts.

RULING:

YES.

The Court reiterated that there is no general rule or principle that can be laid
down to determine what constitutes doing or engaging in business. Each case must be
judged in the light of its peculiar factual milieu and upon the language of the statute
applicable.
It ruled that petitioner can be sued in the regular courts because it is doing
business in the Philippines. The applicable law is Republic Act No. 5455 as
implemented by the following rules and regulations of the Board of Investments which
took effect on February 3, 1969. In said Act, it enumerates acts that shall constitute
―doing business‖ which includes.
It cannot be denied that petitioner had solicited the lime plant business from
DBT through the Marubeni Manila branch. Records show that the "turn-key proposal
for the 300 T/D Lime Plant" was initiated by the Manila office through its Mr. T. Hojo.
In a follow-up letter dated August 3, 1976, Hojo committed the firm to a price
reduction of $200,000.00 and submitted the proposed contract forms.
As reflected in the letterhead used, it was Marubeni Corporation, Tokyo, Japan
which assumed an active role in the initial stages of the negotiation. Petitioner
Marubeni Nederland B.V. had no visible participation until the actual signing of the
October 28, 1976 agreement in Tokyo and even there, in the space reserved for
petitioner, it was the signature. of "S. Adachi as General Manager of Marubeni
Corporation, Tokyo on behalf of Marubeni Nederland B.V." which appeared.

416 | P a g e
Law 321_Corporation LAW_ Case Digest

PHILIPPINE COLUMBIA ENTERPRISES CO.


vs.
LANTIN
G.R. No.L-29072 June 7, 1971

FACTS:

Private respondent Katoh & Co.,Ltd, alleged in its civil complaint that it is a
corporation duly organized under the laws of Japan, with head office in Tokyo, Japan.
The complaint alleged ten (10) causes of action against the defendants Philippines
Columbia Enterprises Co., with principal place of business in Manila, and the general
partners, thereof, Rufino Dy Chin and Fermin Sy, who reside in Manila.
These ten(10) causes of action are for the collection of payment of ten(10)
different shipments of angle bars, mild steel bars, and cold rolled steel sheets allegedly
ordered in May, July, October and November, 1966 by the defendants from the
plaintiff which plaintiff had duly shipped and defendants duly received but which
defendant refused to pay.
The complaint does not allege that plaintiff has secured a license to transact
business in the Philippines but its alleges that it‖ has not been and is not engaged in
business in the Philippines and that the transactions averred in this complaint were
exports made and consummated in Tokyo, Japan in pursuance of international trade.‖

ISSUE:

Whether or not plaintiff‘s allegations in its complaint, particularly in its ten (10)
causes of action, constitute by themselves an admission that it is transacting business
in the Philippines.

RULING:

NO.

An examination of complaint will show that the same expressly avers that the
transactions upon which respondent plaintiff is suing were‖ consummated in Tokyo‖
and hence, not in the Philippines. Petitioners- defendant‘s assertion that the contracts
were made in the Philippines squarely contradicts the averments in the complaint.
And the basic and well-known rule is that whether a cause of action is pleaded or not
must be ascertained solely upon the face of the complaint.
Since the petitioner‘s averment that the plaintiff‘s transactions were made in
the Philippines, being contradictory of the complaint, can not be set up in motion to
dismiss for lack of cause of action, but must be pleaded in an answer, any reception of
evidence on the point would merely duplicate the trial on the merits, and should be
deferred.
Therefore, the court below committed no abuse of discretion amounting to
excess of jurisdiction in resolving to defer action on motion to dismiss. The last
objection of the petitioners to the deferment order is that if they file a counterclaim in
their answer against respondent foreign corporation, they would be recognizing the
legal capacity of said corporation which they are precisely questioning. This fear is
without legal basis, for actions by foreign corporations are governed by rules different
from those in actions against them.
A counterclaim partakes of the nature of a complaint and/or cause of action
against the plaintiff, so that if the petitioner‘s-defendants should file a counterclaim,
the private respondent-plaintiff Katoh & Co.,Ltd., would not be maintaining a suit and,
consequently, Section 69 of the Corporation Law would not apply.

417 | P a g e
Law 321_Corporation LAW_ Case Digest

PHILIP MORRIS, INC.


vs.
FORTUNE TOBACCO
GR NO. 158589 JUNE 27,2006

FACTS:

Petitioner Philip Morris, Inc., a corporation organized under the laws of the
State of Virginia, United States of America, is, per Certificate of Registration No. 18723
issued on April 26, 1973 by the Philippine Patents Office (PPO), the registered owner of
the trademark ―MARK VII‖ for cigarettes.
Similarly, petitioner Benson & Hedges (Canada), Inc., a subsidiary of Philip
Morris, Inc., is the registered owner of the trademark ―MARK TEN‖ for cigarettes as
evidenced by PPO Certificate of Registration No. 11147. And as can be seen in
Trademark Certificate of Registration No. 19053, another subsidiary of Philip Morris,
Inc., the Swiss company Fabriques de Tabac Reunies, S.A., is the assignee of the
trademark ―LARK,‖ which was originally registered in 1964 by Ligget and Myers
Tobacco Company.
On the other hand, respondent Fortune Tobacco Corporation, a company
organized in the Philippines, manufactures and sells cigarettes using the trademark
―MARK.‖
Petitioners then filed a Complaint for Infringement of Trademark and Damages
against respondent Fortune Tobacco Corporation. In the Complaint with prayer for the
issuance of a preliminary injunction, petitioners alleged that they are foreign
corporations not doing business in the Philippines and are suing on an isolated
transaction.
They averred that the countries in which they are domiciled grant to corporate
or juristic persons of the Philippines the privilege to bring action for infringement,
without need of a license to do business in those countries. Petitioners likewise
manifested being registered owners of the trademark ―MARK VII‖ and ―MARK TEN‖
for cigarettes as evidenced by the corresponding certificates of registration and an
applicant for the registration of the trademark ―LARK MILDS‖.

ISSUE:

Whether or not Petitioner has the legal capacity to sue the respondent for
alleged infringement.

RULING:

YES.

A ―trademark‖ is any distinctive word, name, symbol, emblem, sign, or device,


or any combination thereof adopted and used by a manufacturer or merchant on his
goods to identify and distinguish them from those manufactured, sold, or dealt in by
others. Inarguably, a trademark deserves protection.
A foreign corporation may have the capacity to sue for infringement but the
question of whether they have an exclusive right over their symbol as to justify
issuance of the controversial writ will depend on actual use of their trademarks in the
Philippines in line with Sections 2 and 2-A of RA116.
It is thus incongruous for petitioners to claim that when a foreign corporation
not licensed to do business in the Philippines files a complaint for infringement, the
entity need not be actually using its trademark in commerce in the Philippines. Such
a foreign corporation may have the personality to file a suit for infringement but it may
not necessarily be entitled to protection due to absence of actual use of the emblem in
the local market.
Hence, it may be stated right off that the registration of a trademark
unaccompanied by actual use thereof in the country accords the registrant only the
standing to sue for infringement in Philippine courts. Entitlement to protection of such
trademark in the country is entirely a different matter.

418 | P a g e
Law 321_Corporation LAW_ Case Digest

PUMA SPORTS
vs.
INTERMEDIATE APPELLATE COURT
G.R. NO. 75067 FEBRUARY 26, 1988

FACTS:

On July 25, 1985, petitioner PUMA SPORTSSCHUFABRIKEN, a German


corporation manufacturing ―PUMA PRODUCTS‖, filed a complaint for infringement of
patent or trademark with a prayer for the issuance of a writ preliminary injunction
against the private respondent Mil-Oro Manufacturing Corp. before the RTC of Makati.
On July 31, 1985, the trial court issued a temporary restraining order,
restraining the private respondent and the Director of Patents from using the
trademark "PUMA' or any reproduction, counterfeit copy or colorable imitation thereof,
and to withdraw from the market all products bearing the same trademark.

ISSUE:

Whether or not the Court of Appeals erred in holding that petitioner had no
legal capacity to sue.

RULING:

YES.

Petitioner maintains that it has substantially complied with the requirements of


Section 21-A of Republic Act (RA) No. 166, as amended. According to the petitioner, its
complaint specifically alleged that it is not doing business in the Philippines and is
suing under the said Republic Act. Said Section 21-A requires that ―the country of
which the said corporation or juristic person is a citizen, or in which it is domiciled, by
treaty, convention or law, grants a similar privilege to corporate or juristic persons of
the Philippines‖ but does not mandatorily require that such reciprocity between the
Federal Republic of Germany and the Philippines be pleaded.
Such reciprocity arrangement is embodied in and supplied by the Union
Convention for the Protection of industrial Property (Paris Convention) to which both
the Philippines and Federal Republic of Germany are signatories and that since the
Paris Convention is a treaty which, pursuant to our constitution, forms part of the law
of the land, our courts are bound to take judicial notice of such treaty, and,
consequently, this fact need not be averred in the complaint.
The Convention of the Union of Paris for the Protection of Industrial Property to
which the Philippines became a party on September 27, 1965 provides in its Article 8
thereof that‖ a trade name (corporation name) shall be protected in all the countries of
the Union without the obligation of filing or registration, whether or not it forms part
of the trademark.‖ Thus, petitioner had the legal capacity to file the action.

419 | P a g e
Law 321_Corporation LAW_ Case Digest

SBMA
vs.
UNIVERSAL INTERNATIONAL GROUP OF TAIWAN
GR. NO. 131680 SEPTEMBER 14, 2000

FACTS:

In 1995, a ‗Lease and Development Agreement‘ was executed by respondent UIG


and petitioner SBMA under which respondent UIG shall lease from petitioner SBMA
the Binictican Golf Course and appurtenant facilities thereto to be transformed into a
world class 18-hole golf course, golf club/resort, commercial tourism and residential
center. The contract in pertinent part contains pre-termination clauses, which
provides in its Section 22 thereof the acts which constitute what is ―default‖.
In 1997, Petitioner SBMA sent a letter to private respondent UIG calling its
attention to its alleged several contractual violations in view of private respondent
UIG‘s failure to deliver its various contractual obligations, primarily its failure to
complete the rehabilitation of the Golf Course in time for the APEC Leader‘s Summit,
and to pay accumulated lease rentals and utilities, and to post the required
performance bond.
Respondent UIG, in its letter, interposed as an excuse the alleged default of its
main contractor FF Cruz, resulting in their filing of suit against the latter, and
committed itself to comply with its obligations within a few days. Private respondent
UIG, however, failed to comply with its undertakings.
Petitioner SBMA sent a letter to private respondent UIG declaring the latter in
default of its contractual obligations to SBMA under Section 22.1 of the Lease and
Development Agreement and required it to show cause why petitioner SBMA should
not pre-terminate the agreement. Private respondents paid the rental arrearages but
the other obligations remained unsatisfied.

ISSUE:

Whether or not respondents had the capacity to sue and possess material
interest to institute an action against petitioners.

RULING:

YES.

As a general rule, unlicensed foreign non-resident corporations cannot file suits


in the Philippines as provided in Section 133 of the Corporation Code. A corporation
has legal status only within the state or territory in which it was organized. For this
reason, a corporation organized in another country has no personality to file suits in
the Philippines. In order to subject a foreign corporation doing business in the country
to the jurisdiction of our courts, it must acquire a license from the SEC and appoint
an agent for service of process. Without such license, it cannot institute a suit in the
Philippines.
However, that the licensing requirement was ―never intended to favor domestic
corporations who enter into solitary transactions with unwary foreign firms and then
repudiate their obligations simply because the latter are not licensed to do business in
this country.‖ After contracting with a foreign corporation, a domestic firm is estopped
from denying the former‘s capacity to sue.
In this case, SBMA is estopped from questioning the capacity to sue of UIG. In
entering into the LDA with UIG, SBMA effectively recognized its personality and
capacity to institute the suit before the trial court.
The petition is partially granted. the writ of preliminary injunction is lifted and
the case is to the rtc for trial on the merits.

420 | P a g e
Law 321_Corporation LAW_ Case Digest

TIME, INC.
vs.
REYES
G.R. No.L-28882 May 31, 1971

FACTS:

This is a petition by Time, Inc. for certiorari and prohibition, with preliminary
injunctions, to annul certain orders of the respondent CFI of Rizal, issued in its Civil
Case No. 10403, entitled ― Antonio J. Villegas and Juan Ponce Enrile vs. Time, Inc.,‖
and to prohibit the said Rizal court from further proceeding with the said civil case
contending that it is the Manila CFI which has the jurisdiction.
The petition alleges that the petitioner time, Inc., is an American Corporation
with principal offices at Rockefeller Center, New York City, N.Y., and is the publisher
of ―Time‖, a weekly magazine; the petition, however, does not alleged the petitioner‘s
legal capacity to sue in the courts of the Philippines.
In said civil case, therein plaintiffs Antonio J. Villegas and Juan Ponce Enrile
seek to recover from the therein petitioner damages upon an alleged of libel arising
from a publication of time(Asia Edition) magazine, in its issue of 18 August 1967, of an
essay, entitled ―Corruption in Asia‖.

ISSUE:

Whether or not the petition will prosper.

RULING:

YES.

The dismissal of the present petition is asked on the ground that the petitioner
foreign corporation failed to allege its capacity to sue in the courts of the Philippines.
The Court failed to see how these doctrines can be a propos in the case at bar,
since the petitioner is not maintaining any suit‖ but is merely defending one against
itself; it did not file any complaint but only a corollary defensive petition to prohibit the
lower court from further proceeding with a suit that it had no jurisdiction to entertain.
Petitioner‘s failure to aver its legal capacity to institute the present petition is
not fatal, for a foreign corporation may by writ of prohibition, seek relief against the
wrongful assumption of jurisdiction. And a foreign corporation seeking a writ of
prohibition against further maintenance a suit, on the ground of want jurisdiction, is
not bound by the ruling of the court in which the suit was brought, on the motion to
quash service of summons, that it has jurisdiction‖.
The writs applied for are granted: the respondent Court of First Instance of
Rizal is declared without jurisdiction to take cognizance of its Civil Case No. 10403;
and its orders issued in connection therewith are hereby annulled and set aside.
Respondent court is further commanded to desist from further proceedings in Civil
case No. 10403 aforesaid.

421 | P a g e
Law 321_Corporation LAW_ Case Digest

UNIVERSAL RUBBER PRODUCTS, INC.


vs.
COURT OF APPEALS
G.R. No.L-30266 June 29, 1984

FACTS:

Respondent corporations, Converse Rubber Corp. and Edwardson


Manufacturing Co., sued petitioner Universal Rubber products, Inc. before the CFI of
Rizal for unfair competition with damages and attorney‘s fees.
On March 4, 1968, petitioner filed a motion in the court below praying that the
subpoena duces tecum dated February 13, 1968 be quashed which was denied. A
motion for reconsideration was likewise denied.
Consequently, on August 6, 1968, petitioner Universal Rubber Products, Inc.
filed its present petition for certiorari with preliminary injunction, alleging that in so
denying its motion to quash the subpoena duces tecum and its subsequent motion for
reconsideration, respondent Judge acted with grave abuse of discretion amounting to
an excess of jurisdiction.
Petitioner Universal Rubber Products, Inc. contends that private respondent
Converse Rubber Corp. is a foreign corporation not licensed to do business in the
Philippines and that respondent Edwardson is merely its licensee; that respondent
Converse has no goodwill to speak of and that it has no registration right over its own
name.

ISSUE:

Whether or not respondent Converse Rubber Corp. has capacity to sue.

RULING:

YES.

The Court held that ―the disability of a foreign corporation from suing in the
Philippines is limited to suits to enforce any legal or contracts rights arising from, or
growing out, of any business which it has transacted in the Philippine Islands.‖ On the
other hand, where the purpose of the suit is ―to protect its reputation, its corporate
name, its goodwill, whenever that reputation, its corporate name or goodwill have,
through the natural development of its trade, established themselves‖, an unlicensed
foreign corporation may sue in the Philippines.
Hence, it is clear that Section 29 of the Corporation Law does not disqualify
plaintiff- appellee Converse Rubber, which does not have a branch office in any part of
the Philippines and is not‖ doing business‖ in the Philippines, from filing and
prosecuting this action of unfair competition.
Therefore Converse Rubber Corp. can file and prosecute the action of unfair
competition.

422 | P a g e
Law 321_Corporation LAW_ Case Digest

VAN ZUIDEN BROS., LTD.


vs.
GTVL INDUSTRIES, INC.
G.R. NO. 147905 MAY 28, 2007

FACTS:

Petitioner Zuiden, is a corporation, incorporated under the laws of Hong Kong.


Zuiden is not engaged in business in the Philippines, but is suing before the Philippine
Courts, for the reasons hereinafter stated. It is engaged in the importation and
exportation of several products, including lace products. On several occasions, GTVL
purchased lace products from petitioner.
The procedure for these purchases, as per the instructions of GTVL, was that
Zuiden delivers the products purchased by GTVL, to a certain Hong Kong corporation,
known as Kenzar Ltd. and the products are then considered as sold, upon receipt by
Kenzar of the goods purchased by GTVL. Kenzar had the obligation to deliver the
products to the Philippines and/or to follow whatever instructions GTVL had on the
matter.
Insofar as Zuiden is concerned, upon delivery of the goods to KENZAR in Hong
Kong, the transaction is concluded; and GTVL became obligated to pay the agreed
purchase price. However, commencing October 31, 1994 up to the present, GTVL has
failed and refused to pay the agreed purchase price for several deliveries ordered by it
and delivered by Zuiden.

ISSUE:

Whether or not petitioner, an unlicensed foreign corporation, has legal capacity


to sue before Philippine courts.

RULING:

YES.

An unlicensed foreign corporation not doing business in the Philippines can sue
before Philippine courts. In the present case, the series of transactions between
petitioner and respondent cannot be classified as "doing business" in the Philippines
under Section 3(d) of RA 7042.
An essential condition to be considered as "doing business" in the Philippines is
the actual performance of specific commercial acts within the territory of the
Philippines for the plain reason that the Philippines has no jurisdiction over
commercial acts performed in foreign territories.
Here, there is no showing that petitioner performed within the Philippine
territory the specific acts of doing business mentioned in Section 3(d) of RA 7042.
Petitioner did not also open an office here in the Philippines, appoint a representative
or distributor, or manage, supervise or control a local business. While petitioner and
respondent entered into a series of transactions implying a continuity of commercial
dealings, the perfection and consummation of these transactions were done outside
the Philippines.
Further, the series of transactions between petitioner and respondent
transpired and were consummated in Hong Kong. There was no single activity which
petitioner performed here in the Philippines pursuant to its purpose and object as a
business organization. Moreover, petitioner‘s desire to do business within the
Philippines is not discernible from the allegations of the complaint or from its
attachments. Therefore, there is no basis for ruling that petitioner is doing business in
the Philippines.
Considering that petitioner is not doing business in the Philippines, it does not
need a license in order to initiate and maintain a collection suit against respondent for
the unpaid balance of respondent‘s purchases.

423 | P a g e
Law 321_Corporation LAW_ Case Digest

SECURITIES AND EXCHANGE COMMISSION LAW


(P.D. No. 902-A, as Amended by R.A. No. 8799 or Securities Regulation Code)

POWERS AND FUNCTIONS OF THE SEC

SECURITIES OF EXCHANGE COMMISSION


vs.
PERFORMANCE FOREIGN EXCHANGE CORPORATION (PFEC)
G.R. NO. 154131 JULY 20, 2006

FACTS:

Respondent Performance Foreign Exchange Corporation(PFEC) is a domestic


corporation duly registered under Securities and Exchange Commission (SEC) with its
primary purpose to operate as a broker/agent between market participants in
transactions involving, but not limited to, foreign exchange, deposits, interest rate
instruments, fixed income securities, bonds/bills, repurchased agreements of fixed
income securities, certificate of deposits, bankers acceptances, bills of exchange, over-
the-counter option of the aforementioned instruments, Lesser Developed Country‘s
(L.D.C.) debt, energy and stock indexes and all related, similar or derivative products,
other than acting as a broker for the trading of securities pursuant to the Revised
Securities Act of the Philippines. Its secondary purpose is to engage in money changer
or exchanging foreign currencies into domestic currency, Philippine currency or other
foreign currencies into another currency.
Respondent‘s officers complied and explained before the CED the nature of their
business. In 2001, Emilio B. Aquino, Director of CED, issued a Cease and Desist
Order in CED Case No. 99-2297, stating that his department conducted an inquiry on
respondent‘s business operations for possible violation of Republic Act (R.A.) No.
8799,otherwise known as The Securities Regulation Code; that the outcome of the
inquiry shows that respondent is engaged in the trading of foreign currency futures
contracts in behalf of its clients without the necessary license; that such transaction
can be deemed as a direct violation of Section 11 of R.A. No. 87994 and the related
provisions of its Implementing Rules and Regulations; and that it is imperative to
enjoin respondent from further operating as such to protect the interest of the public.

ISSUE:

Whether or not petitioner SEC has acted with grave abuse of discretion in
issuing the Cease and Desist Order and its subsequent Order making it permanent.

RULING:

NO.

Under Section 64 of R.A. No. 8799, there are two essential requirements that
must be complied with by the SEC before it may issue a cease and desist order: First,
it must conduct proper investigation or verification; and Second, there must be a
finding that the act or practice, unless restrained, will operate as a fraud on investors
or is otherwise likely to cause grave or irreparable injury or prejudice to the investing
public.
In the present case, the first requirement is not present. Petitioner did not
conduct proper investigation or verification before it issued the challenged orders. The
clarificatory conference undertaken by petitioner regarding respondent‘s business
operations cannot be considered a proper investigation or verification process to justify
the issuance of the Cease and Desist Order. It was merely an initial stage of such
process, considering that after it issued the said order following the clarificatory
conference, petitioner still sought verification from the BSP on the nature of
respondent‘s business activity.
Thus, the cease and desist order stays against the corporation until the latter
shall be able to submit the appropriate endorsement from the Bangko Sentral ng
Pilipinas that it can engage in financial derivative transactions.

424 | P a g e
Law 321_Corporation LAW_ Case Digest

ARRANZA ET.AL
vs.
B. F. HOMES, INC
G.R. NO. 131683 JUNE 19, 2000

FACTS:

Respondent BF Homes, Inc. (BFHI), is a domestic corporation engaged in


developing subdivisions and selling residential lots. One of the subdivisions that
respondent developed was the BF Homes Parañaque Subdivision, which now sprawls
across not only a portion of the City of Parañaque but also those of the adjoining cities
of Las Piñas and Muntinlupa.
When the Central Bank ordered the closure of Banco Filipino, which had
substantial investments in respondent BFHI, respondent filed with the SEC a petition
for rehabilitation and a declaration that it was in a state of suspension of payments.
On 18 March 1985, the SEC placed respondent under a management committee.
Upon that committee‘s dissolution on 2 February 1988, the SEC appointed Atty.
Florencio B. Orendain as a Receiver, and approved a Revised Rehabilitation Plan.
As a Receiver, Orendain instituted a central security system and unified the
sixty~five homeowners‘ associations into an umbrella homeowners‘ association called
United BF Homeowners‘ Associations, Inc. (UBFHAI), which was thereafter
incorporated with the Home Insurance and Guaranty Corporation (HIGC). In 1989,
respondent, through Orendain, turned over to UBFHAI control and administration of
security in the subdivision, the Clubhouse and the open spaces along Concha Cruz
Drive.
Through the Philippine Waterworks and Construction Corporation (PWCC),
respondent‘s managing company for waterworks in the various BF Homes
subdivisions, respondent entered into an agreement with UBFHAI for the annual
collection of community assessment fund and for the purchase of eight new pumps to
replace the over~capacitated pumps in the old wells.

ISSUE:

Whether or not the Securities and Exchange Commission has jurisdiction to


resolve the present controversy.

RULING:

YES.

The fact that respondent is under receivership does not divest the HLURB of
that jurisdiction. A receiver is a person appointed by the court, or in this instance, by
a quasi~judicial administrative agency, in behalf of all the parties for the purpose of
preserving and conserving the property and preventing its possible destruction or
dissipation, if it were left in the possession of any of the parties.
The appointment of a receiver does not dissolve a corporation, nor does it
interfere with the exercise of its corporate rights. In this case where there appears to
be no restraints imposed upon respondent as it undergoes rehabilitation receivership,
respondent continues to exist as a corporation and hence, continues or should
continue to perform its contractual and statutory responsibilities to petitioners as
homeowners.
No violation of the SEC order suspending payments to creditors would result as
far as petitioners‘ complaint before the HLURB is concerned. Since what petitioners
seek to enforce are respondent‘s obligations as a subdivision developer. Such claims
are basically not pecuniary in nature although it could incidentally involve monetary
considerations. All that petitioners‘ claims entail is the exercise of proper subdivision
management on the part of the SEC~appointed Board of Receivers towards the end
that homeowners shall enjoy the ideal community living that respondent portrayed
they would have when they bought real estate from it. It can not also be considered as
having claims against respondent.

425 | P a g e
Law 321_Corporation LAW_ Case Digest

QUASHA
vs.
SECURITIES OF EXCHANGE COMMISSION
G.R. NO. L-47536 MAY 31, 1978

FACTS:

Petitioner had filed on October 10, 1977 and October 17, 1977 his complaints
and continuing opposition with respondent commission against the filing of
respondent Manila Polo Club, Inc.'s Amended Articles of Incorporation and Amended
By-Laws which would convert said club into a proprietary club, assailing the
amendments as illegal, inequitable and immoral, alleging inter alia that the
amendments have the effect of enabling the members to appropriate the club's
property and to use it as their contribution to the new club; the real estate assets of
Manila Polo Club consist of 25 hectares, more or less, of prime real estate in 'he
middle of Forbes lark Makati, Metro Manila. which are conservatively valued at
present market valuation of P200 Million and its buildings, improvements, facilities
and other equipments at about P20 Million.
The more or less 2,000 actual members who will become proprietary owners of
the Club's assets under the proposed conversion will inequitably enrich themselves if
this Honorable Commission will allow the comparatively paltry of P12,500.00 to be
paid for each proprietary membership;" "the value which the Club now commands
results from the accrued contribution of past (and present) members' money, time,
effort and foresight; and the conversion plan does not in any way compensate the
predecessors of the present membership (and also those of the present membership
who do not opt for conversion) who substantially contributed to making the Club what
it is today" and further claiming that the amendments had not been duly adopted by
the required two-thirds vote.
Petitioner prayed for the disapproval and cancellation of respondent club's
amended articles and by-laws and denial of its application to register its proprietary
shares, and prayed for a restraining order meanwhile enjoining it from selling and/or
accepting any payments for the questioned proprietary shares.

ISSUE:

Whether or not the hearing officer of the SEC is empowered to issue the
questioned order denying the injunctive relief.

RULING:

YES.

The Court ruled that in view of the extremely limited time, with the
commission's hearing officer having issued his questioned order denying injunctive
relief only on December 22, 1977 at the height of the Christmas holiday with just a
few days before The scheduled deadline of December 28, 1977, petitioner properly filed
the present petition directly with this Court without going through the prescribed
procedure of filing an appeal with respondent Commission en banc within the 30-day
reglementary period since such recourse was obviously not a plain, speedy and
adequate remedy.
The questions raised by petitioner in his pending complaints with respondent
commission warrant a full-blowing trial' on the merits" after which the main issues
may be duly adjudicated as contended by him, and since respondents likewise concur
in this stand, the case will be remanded to respondent commission for such trial and
determination on the merits.
Finally, petitioner has not satisfactorily established his right to the restraining
order prayed for. Considering that petitioner submitted the incident on the basis of his
memorandum without presentation of evidence, the Court holds that respondent
commission did not act with grave abuse of discretion in denying the restraining order
prayed for. The case is ordered remanded to respondent commission for a full trial and
hearing and determination on the merits.

426 | P a g e
Law 321_Corporation LAW_ Case Digest

TRADER’S ROYAL BANK


vs.
COURT OF APPEALS
G.R. No. L-78412 September 26, 1989

FACTS:

On March 30,1982, the Philippine Blooming Mills, Inc. (PBM) and Alfredo Ching
jointly submitted to the Securities and Exchange Commission a petition for
suspension of payments (SEC No. 2250) where Alfredo Ching was joined as co-
petitioner because under the law, he was allegedly entitled, as surety, to avail of the
defenses of PBM and he was expected to raise most of the stockholders' equity of Pl00
million being required under the plan for the rehabilitation of PBM. Traders Royal
Bank was included among PBM's creditors named in Schedule A accompanying PBM's
petition for suspension of payments.
On May 13, 1983, the petitioner bank filed a case against PBM and Alfredo
Ching, to collect P22,227,794.05 exclusive of interests, penalties and other bank
charges representing PBM's outstanding obligation to the bank. Alfredo Ching, a
stockholder of PBM, was impleaded as co-defendant for having signed as a surety for
PBM's obligations to the extent of ten million pesos (Pl0,000,000) under a Deed of
Suretyship dated July 21, 1977.
In its en banc decision, the SEC declared that it had assumed jurisdiction over
petitioner Alfredo Ching pursuant to Section 6, Rule 3 of the new Rules of Procedure of
the SEC providing that "parties in interest without whom no final determination can
be had of an action shall be joined either as complainant, petitioner or respondent" to
prevent multiplicity of suits.
On July 9, 1982, the SEC issued an Order placing PBM's business, including
its assets and liabilities, under rehabilitation receivership, and ordered that "all
actions for claims listed in Schedule A of the petition pending before any court or
tribunal are hereby suspended in whatever stage the same may be, until further
orders from the Commission". As directed by the SEC, said order was published once
a week for three consecutive weeks in the Bulletin Today, Philippine Daily Express and
Times Journal at the expense of PBM and Alfredo Ching.

ISSUE:

Whether or not the court a quo could acquire jurisdiction over Ching in his
personal and individual capacity as a surety of PBM in the collection suit filed by the
bank, despite the fact that PBM's obligation to the bank had been placed under
receivership by the SEC.

RULING:

YES.

Although Ching was impleaded in SEC Case No. 2250, as a co-petitioner of


PBM, the SEC could not assume jurisdiction over his person and properties. The
Securities and Exchange Commission was empowered, as rehabilitation receiver, to
take custody and control of the assets and properties of PBM only, for the SEC has
jurisdiction over corporations only not over private individuals, except stockholders in
an intra-corporate dispute (Sec. 5, P.D. 902-A and Sec. 2 of P.D. 1758). Being a
nominal party in SEC Case No. 2250, Ching's properties were not included in the
rehabilitation receivership that the SEC constituted to take custody of PBM's assets.
Therefore, the petitioner bank was not barred from filing a suit against Ching,
as a surety for PBM. An anomalous situation would arise if individual sureties for
debtor corporations may escape liability by simply co- filing with the corporation a
petition for suspension of payments in the SEC whose jurisdiction is limited only to
corporations and their corporate assets.

427 | P a g e
Law 321_Corporation LAW_ Case Digest

VICMAR DEVELOPMENT
vs.
COURT OF APPEALS
G.R. No. 81547 May 21, 1990

FACTS:

Sometime in August, 1982, a conflict arose between petitioner Vicente


Angliongto and private respondent Rufino T. Nasser on the matter of exclusive control
and management of Petitioner Corporation. On July 7, 1983, petitioner Corporation by
petitioner Angliongto, filed a verified petition in the public respondent SEC against
private respondent Nasser, alleging, that private respondent Nasser was a Director,
Executive Vice-President and General Manager of petitioner Corporation from 1974 to
August 26, 1982 but during the annual meeting of stockholders of petitioner
corporation held on August 26, 1982, private respondent Nasser was not re-elected as
member of the Board of Directors or to his previous management positions.
In view of the result of the annual stockholders' meeting, private respondent
Nasser was then advised by the incoming president, herein petitioner Angliong to that
the latter would actively manage the corporate affairs of Petitioner Corporation.
In view thereof, private respondent Nasser was asked to turn over all corporate
books and records in his possession to the duly elected officers, among others, which
demand remained (un)heeded by private respondent Nasser as the latter continued to
hold office as Executive Vice-President and General Manager of petitioner Corporation,
performing acts and entering into transactions inimical to the interests of the
petitioner Corporation and its stockholders.
Said petition also prayed for the issuance of a restraining order and thereafter,
a permanent injunction to enjoin private respondent Nasser from representing himself
as an officer of petitioner Corporation, among other things, and for him, to surrender
all corporate books and records to the duly elected officers of said Corporation.

ISSUE:

Whether or not the Securities and Exchange Commission has abused its
discretion in recalling its Order to enforce a writ of preliminary injunction.

RULING:

NO.

The facts reveal that the writ of preliminary injunction issued on September 19,
1983 enjoined private respondent Nasser from acting as, and/or representing himself
to be, the Executive Vice-President and/or General Manager and/or officer in any
capacity of petitioner Corporation. Upon presentment of the Agreement dated
November 10, 1983 showing a transfer of ownership, control and management of
Vicmar Corporation by Vicente Angliongto unto Nasser, the SEC correctly recalled the
order of March 5, 1986 directing the implementation of the aforesaid writ, pending
hearing on the motion dated March 17, 1986.
To allow execution of the writ of preliminary injunction in favor of the
petitioners despite having transferred their rights of ownership, control and
management over said corporation to respondent Nasser would be baseless, the
contract having shown prima facie that the latter is entitled to remain as Vice-
President and General Manager of petitioner Corporation.
Thus, no grave abuse of discretion can be attributed to the SEC in recalling the
order to enforce a writ of preliminary injunction in this wise. After all, the issuance or
recall of preliminary writ of injunction is an interlocutory matter that remains at all
times within the control of the court (Alvaro v. Zapata, 118 SCRA 728 [1982]). The
grant or denial of an injunction rests upon the sound discretion of the lower tribunal,
in the exercise of which the Supreme Court will not interfere except in a clear case of
abuse.

428 | P a g e
Law 321_Corporation LAW_ Case Digest

ORIGINAL AND EXCLUSIVE JURISDICTION OF THE REGIONAL TRIAL COURTS

Orendain
vs.
BF Homes, Inc.
G.R. No. 146313 October 31, 2006

FACTS:

BF Homes, Inc. availed financial assistance from various sources to buy


properties and convert them into residential subdivisions. Despite its solvent status, it
filed a Petition for Rehabilitation and for Declaration in a State of Suspension of
Payments. SEC ordered the appointment of a rehabilitation receiver, FBO Management
Networks, Inc., with Orendain as Chairman to prevent paralyzation of BF Homes‘
business operations. Deed of Absolute Sale was executed by and between BF Homes—
represented by Orendain—as absolute and registered owner, and the Local Superior of
the Franciscan Sisters of the Immaculate Phils., Inc. (LSFSIPI) over a parcel of land.
BF Homes filed a Complaint with the RTC against LSFSIPI and Orendain for
reconveyance of the property alleging that the LSFSIPI transacted with Orendain in his
individual capacity and therefore, neither FBO Management, Inc. nor Orendain had
title to the property transferred. Moreover, it averred that the selling price was grossly
inadequate or insufficient amounting to fraud and conspiracy with the LSFSIPI.

ISSUE:

Whether or not a simple reconveyance suit is within the jurisdiction of the RTC.

RULING:

YES.

It is the RTC which has jurisdiction. Clearly, the controversy involves matters
purely civil in character and is beyond the ambit of the limited jurisdiction of the
SEC. The better policy in determining which body has jurisdiction over a case would
be to consider not only [1] the status or relationship of the parties but also [2] the
nature of the question that is the subject of their controversy.‖
More so, the first element requires that the controversy must arise out of intra-
corporate or partnership relations between any or all of the parties and the
corporation, partnership or association of which they are stockholders, members or
associates; between any or all of them and the corporation, partnership or association
of which they are stockholders, members or associates, respectively; and between
such corporation, partnership or association and the State insofar as it concerns their
individual franchises.
The second element requires that the dispute among the parties be intrinsically
connected with the regulation of the corporation. If the nature of the controversy
involves matters that are purely civil in character, necessarily, the case does not
involve an intra-corporate controversy.
The determination of whether a contract is simulated or not is an issue that
could be resolved by applying pertinent provisions of the Civil Code. Section 5 of PD
No. 902-A does not apply in the instant case.
The LSFSIPI is neither an officer nor a stockholder of BF Homes, and this case
does not involve intra-corporate proceedings. In addition, the seller Orendain, is being
sued in his individual capacity for the unauthorized sale of the property in
controversy. In addition, jurisdiction over the case for reconveyance is clearly vested in
the RTC as provided in paragraph (2), Section 19, B.P. Blg. 129.

429 | P a g e
Law 321_Corporation LAW_ Case Digest

ALFREDO P. PASCUAL and LORETA S. PASCUAL


vs.
COURT OF APPEALS (former Seventh Division), ERNESTO P. PASCUAL and HON.
ADORACION ANGELES, in her capacity as Presiding Judge, RTC, Kaloocan City,
Branch 121
G.R. No. 138542 August 25, 2000

FACTS:

Ernesto P. Pascual filed a complaint in the Regional Trial Court for "accounting,
reconveyance of real property based on implied trust resulting from fraud, declaration
of nullity of TCT, recovery of sums of money, and damages" against his brother,
petitioner Alfredo, and the latter‘s wife Loreta Pascual.
Petitioners filed a motion to dismiss on the ground that the complaint raises an
intra-corporate controversy between the parties over which original and exclusive
jurisdiction is vested in the Securities and Exchange Commission (SEC).

ISSUE:

Whether or not complaint against the Petitioners involves an intra-corporate


dispute cognizable by the SEC and, therefore, the Regional Trial Court should have
dismissed the complaint.

RULING:

NO.

The regular courts, not the SEC, have jurisdiction over this
case.1âwphi1 Petitioners and private respondent never had any corporate relations in
Phillens. It appears that private respondent was never a stockholder in Phillens, of
which the parties‘ predecessor-in-interest, Luciano Pascual, Sr., was a stockholder
and whose properties are being litigated. Private respondent‘s allegation is that, upon
the death of their father, he became co-owner in the estate left by him, and part of this
estate includes the corporate interests in Phillens. He also alleges that petitioners
repudiated the trust relationship created between them and appropriated to
themselves even the property that should have belonged to respondent. It is thus clear
that there is no corporate relationship involved here. That petitioner Alfredo Pascual
was a corporate officer holding in trust for his brother their father‘s corporate interests
did not create an intra-corporate relationship between them.
Nor is the controversy corporate in nature. As we have stated before, the grant
of jurisdiction must be viewed in the light of the nature and function of the SEC under
the law. In the case at bar, the corporation whose properties are being contested no
longer exists, it having been completely dissolved in 1993; consequently, the
supervisory authority of the SEC over the corporation has likewise come to an end.

430 | P a g e
Law 321_Corporation LAW_ Case Digest

DEVICES OF SCHEMES AMOUNTING TO FRAUD OF MISREPRESENTATION

HERNANI N. FABIA
vs.
COURT OF APPEALS
G.R. No. 132684 20 August 2000

FACTS:

Fabia was the President of private respondent MTCP, a domestic corporation


engaged in providing maritime courses and seminars to prospective overseas contract
workers and seamen. He was likewise a Director and stockholder thereof. MTCP filed
an affidavit-complaint for estafa against Hernani N. Fabia alleging that on various
occasions Fabia drew cash advances from MTCP, covered by cash vouchers which he
failed to liquidate despite repeated demands. Fabia‘s defense is that such were in the
nature of simple loans that had already been liquidated and paid. The Office of the
City Prosecutor of Manila dismissed the complaint for lack of jurisdiction for the
reason that the controversy pertained to the relationship between a corporation and a
former officer it was the Securities and Exchange Commission (SEC) which had
original and exclusive jurisdiction over the case.

ISSUE:

Whether or not SEC have jurisdiction over the case?

RULING:

YES.

The jurisdiction of the SEC to "intra-corporate disputes" defined as any act or


omission of the Board of Directors/Trustees of corporations, or of partnerships, or of
other associations, or of their stockholders, officers, or partners, including any
fraudulent devices, schemes or representations, in violation of any law or rules and
regulations administered and enforced by the Commission. Petitioner was the
President as well as a Director and stockholder in private respondent MTCP, who was
charged with the misappropriation or diversion of corporate funds after having failed
to liquidate the amount he had received as cash advances from the company. The
charge against petitioner is for estafa, an offense punishable under The Revised Penal
Code (RPC), and prosecution for the offense is presently before the regular courts.
However, jurisdiction is determined not from the law upon which the cause of action is
based, nor the type of proceedings initiated, but rather, it is gleaned from the
allegations stated in the complaint. It is evident from the complaint that the acts
charged are in the nature of an intra-corporate dispute as they involve fraud
committed by virtue of the office assumed by petitioner as President, Director, and
stockholder in MTCP, and committed against the MTCP corporation. This sufficiently
removes the action from the jurisdiction of the regular courts, and transposes it into
an intra-corporate controversy within the jurisdiction of the SEC. The fact that a
complaint for estafa, a felony punishable under the RPC, has been filed against
petitioner does not negate and nullify the intra-corporate nature of the cause of action,
nor does it transform the controversy from intra-corporate to a criminal one.
Accordingly, as the matter involves an intra-corporate dispute within the jurisdiction
of the SEC, the issue of whether prior non-accounting precludes a finding of probable
cause for the charge of estafa no longer finds relevance.
However, in conformity with RA 8799, The Securities Regulation Code,
amending PD 902-A, which has effectively transferred the jurisdiction of the Securities
and Exchange Commission over all cases enumerated under Sec. 5 of PD 902-A to the
courts of general jurisdiction or the appropriate Regional Trial Courts.

431 | P a g e
Law 321_Corporation LAW_ Case Digest

A & A CONTINENTAL COMMODITIES PHILIPPINES, INC.


vs.
SEC and ROLANDO G. AGUILA
G.R. No. L-55343 16 August 1993

FACTS:

Petitioner is a domestic corporation engaged in the commodities brokerage


business. On August 3, 1979, petitioner and private respondent entered into a
contract for the purchase or sale of commodities. On January 21, 1980, private
respondent bought, through petitioner, 7 contracts of copper. The margin
requirement: for the 7 contracts was P18,750 per contract or a total amount of
P131,250.00, which amount was earmarked from private respondent's cash deposit
with petitioner of P306,326.46. On January 23, 1980, petitioner, allegedly "without
valid and justifiable cause, maliciously, arbitrarily, wantonly, fraudulently, and
recklessly" ordered private respondent to increase his margin requirements per
contract and gave private respondent up to 5 P.M. of the same day w/in which to
deposit with petitioner the additional amount of. Private respondent requested
additional time within which to raise the amount, but petitioner informed him that it
would immediately sell his 7 copper contracts should he fail to deposit the additional
amount by 5 P.M. that same day. Private respondent then requested that should
petitioner proceed with the sale, the same be not effected immediately upon the
opening of trading if prices were low but at a later time. However, petitioner did not
accede to the request and sold 5 contracts immediately upon the opening of trading on
January 24, 1980 and the other two at a later time.

ISSUE:

Whether or not the SEC has jurisdiction over the case.

RULING:

YES.

Poring over the complaint filed by private respondent, the court find that the
complaint is praying for two reliefs based on the same set of facts. One is for the
revocation of the certificate of registration of petitioner; the other is for a sum of
money.
On the action to revoke the certificate of registration of petitioner, there is no
doubt that the SEC has jurisdiction over the same. Section 6(L) of Presidential Decree
No. 902-A clearly provides that the SEC shall possess the power to suspend or revoke,
after proper notice and hearing, the franchise or certificate registration of
corporations, partnerships or associations.
On the other aspect of the SEC's jurisdiction over the action for a sum of
money, we likewise rule that the Commission has the legal competence to decide said
issue. It is axiomatic that jurisdiction over the subject matter of a case is conferred by
law and is determined by the allegations of the complaint, irrespective of whether or
not the plaintiff is entitled to recover upon all or some of the claims asserted therein.
Considering that Petitioners' Complaints sufficiently allege acts amounting to
fraud and misrepresentation committed by Respondent Corporation, the SEC must be
held to retain its original and exclusive jurisdiction over these five (5) cases
notwithstanding the revocation by the Central Bank of Respondent Corporation's
license or permit to operate as a financing company and despite the fact that the suits
involve collections of sums of money paid to said corporation, the recovery of which
would ordinarily fall within the jurisdiction of regular Courts. The fraud committed is
detrimental to the interest of the public and, therefore, encompasses a category of
relationship within the SEC jurisdiction.

432 | P a g e
Law 321_Corporation LAW_ Case Digest

MANUEL M. ALLEJE
vs.
COURT OF APPEALS, SPORTS HEALTH AND PHYSICAL EDUCATION (SHAPE)
CENTRE, INC., and/or ARMIE E. ELMA, Presiding Judge, RTC -Pasig, Br. 153
G.R. No. 107152 25 January 1995

FACTS:

SHAPE is a duly registered non-stock, non-profit corporation the primary


purpose of which is to foster and promote health, conduct physical education and
fitness exercises as well as pleasure and recreation activities by establishing and
maintaining facilities, sports centers, and the like. Petitioner Manuel M. Alegre was
the Executive Vice President (EVP) of SHAPE until his termination of 6 June 1991.
On 16 October 1991 SHAPE filed a complaint for injunction and damages with
application for preliminary injunction and/or temporary restraining order with
preliminary attachment against petitioner Alleje before theRTC of Pasig.
On 14 December 1991, SHAPE filed a supplemental complaint alleging that
after the filing of the original complaint, it discovered that Alleje coined a new name for
his health and fitness centers and advertised the same as "SHAPE CAMP," thus
tending to misrepresent to the public that SHAPE had entered into some kind of
arrangement with Alleje when it fact it had not.
Instead of filing an answer, Alleje filed a motion to dismiss contending that the
Regional Trial Court has no jurisdiction over the present action which partakes of an
intracorporate controversy solely cognizable by the SEC.

ISSUE:

Whether or not the Regional Trial Court of Pasig, and not the SEC, has
jurisdiction over an action instituted by respondent SPORTS HEALTH AND PHYSICAL
EDUCATION (SHAPE) CENTER, INC., against one of its officers, petitioner herein, for
recovery of corporate funds and assets allegedly misappropriated by him.

RULING:

NO.

The SEC has jurisdiction. It is axiomatic that jurisdiction over the subject
matter of a case is conferred by law and is determined by the allegations of the
complaint irrespective of whether plaintiff is entitled to some or all of the claims
asserted therein. The averments in the original complaint as well as the supplemental
complaint of SHAPE sufficiently show the alleged acts committed by Alleje, an officer,
against SHAPE which amount to fraud and misrepresentation and thus detrimental to
the interest of the public. "Fraud" is defined as a generic term embracing all
multifarious means which human ingenuity can devise, and which are resorted to by
one individual to secure an advantage over another by false suggestions or by
suppression of truth and includes all surprise, trick, cunning, dissembling and any
unfair way by which another is cheated.
Clearly, the complaint alleges that as an officer of SHAPE Alleje employed
devises or schemes tantamount to fraud and misrepresentation in order to divert
corporate funds and assets for his personal use. This has transposed an otherwise
ordinary action for recovery of certain properties and sum of money with damages into
an intracorporate controversy which calls for the adjudicative powers of the SEC
pursuant to Sec. 5, par. (a), of PD 902-A. In other words, the complaint filed SHAPE
before the Pasig trial court imputes unmistakable acts of fraud to Alleje as an officer of
SHAPE which have supposedly resulted in its heavy financial losses. The fraud
committed is detrimental to the interest not only of the corporation itself but also of its
members who have unselfishly agreed among themselves that no part of the net
income of the corporation shall inure to any of them. This (fraud) encompasses a
category of relationship within the SEC jurisdiction, despite the fact that the
complaint ultimately involves collection of money, the recovery of which would
ordinarily fall within the legal competence of the regular courts.

433 | P a g e
Law 321_Corporation LAW_ Case Digest

MARIA LUISA FLOR C. BAÑEZ, SPOUSES PETRONILO ESTEVES AND LUISA


ESTEVES AND CONSUELO M. CAULBOY
vs.
DIMENSIONAL CONSTRUCTION TRADE AND DEVELOPMENT CORPORATION AND
THE RTC.
G.R. No. L-62648 22 November 1985

FACTS:

On October 13, 1980 the petitioners herein filed with the RTC a complaint
praying therein that the Dimensional Construction Trade and Development
Corporation (Dimensional), be ordered to pay them the sums of money which were
already due to them under the various promissory notes issued by said
corporation.On August 20, 1981, or 9 months thereafter, petitioners moved to have
Dimensional declared in default for failure to file a responsive pleading which the
court granted. Before a judgment in the case can be rendered, Dimensional filed on
December 16, 1981, a motion to dismiss contending that the SEC and not the CFI had
jurisdiction over the case because there was fraud committed against the stockholders
of the corporation. Petitioners filed their opposition to said omnibus motion averring
that Dimensional has no right to be heard in any manner after losing its standing in
court due to the default order issued against it. Petitioners argued that their complaint
is simply a suit for the collection of sums of money on account of promissory notes
which have already matured and therefore their case is originally and exclusively
cognizable by the CFI. The RTC dismissed the case.

ISSUE:

Whether or not the RTC correct in ruling that jurisdiction falls with the SEC
and not the RTC.

RULING:

NO.

The recitals of the complaint in the case disclose that plaintiff's cause of action
is merely for the collection of the various sums of money that have already become
payable to petitioners due to the promissory notes executed by defendant corporation
which have already matured. There is no allegation nor any mention whatsoever in
plaintiff's complaint that a device or scheme was resorted to by private respondent
corporation amounting to fraud and misrepresentation. It is, therefore, difficult to
consider that petitioners' case falls within the jurisdiction of the Securities and
Exchange Commission pursuant to PD 902-A. Paradoxically, despite the absence of
imputation of fraud and misrepresentation being alleged by plaintiff, it is the
defendant corporation itself which insinuates the existence of fraud and
misrepresentation on its part. Evidently, the defendant's challenge to the jurisdiction
of the court below is principally intended to negate the effects of the order of default
earlier issued against it as well as the evidence already adduced by petitioners in the
court below. The tactical step resorted by the private respondent in the trial court
appears to be its deliberate attempt to unduly delay the satisfaction of the reliefs
claimed for by petitioners and to avoid the effects of its failure to file any answer to the
complaint and controvert the evidence already adduced against it.
In the promissory notes issued by private Respondent Corporation, it is clearly
indicated therein that the sums of money received by private respondent were in the
nature of investments of the petitioners, agreed upon by the parties to be returned by
the corporation upon the maturity of said promissory notes. As the money received by
private respondent do not constitute payment of subscription of shares, the petitioners
herein did not become members of respondent Dimensional Trade and Development
Corporation.

434 | P a g e
Law 321_Corporation LAW_ Case Digest

RAUL SESBREÑO
vs.
HON. COURT OF APPEALS, DELTA MOTORS CORPORATION AND PILIPINAS
BANK
G.R. No. 89252 May 24, 1993

FACTS:

Hermilo Rodis, Sr. et.al. was charged with estafa before the Regional Trial Court
of Cebu. Respondents moved to quash the information on the ground that the
Securities and Exchange Commission (SEC), not the regular courts, had jurisdiction
over the offense charged and that the facts stated herein did not constitute an offense
The trial court denied the motion and private respondent elevated the case to the then
Intermediate Appellate Court . On August 16, 1983, the appellate court dismissed the
petition. Hence, trial ensued in the criminal case. However, after the prosecution had
rested its case, private respondent filed a motion to dismiss on demurrer to evidence
based on the core proposition that there was no criminal offense of estafa from the
non-payment of a money market placement.

ISSUE:

Whether or not private respondent may be held liable for estafa.

RULING:

YES.

In money market placement, the investor is a lender who loans his money to a
borrower through a middleman or dealer. Petitioner here loaned his money to a
borrower through Philfinance. When the latter failed to deliver back petitioner's
placement with the corresponding interest earned at the maturity date, the liability
incurred by Philfinance was a civil one. As such, petitioner could have instituted
against Philfinance before the ordinary courts a simple action for recovery of the
amount he had invested and he could have prayed therein for damages. It appears,
however, that petitioner did not even implead Philfinance in the complaint for damages
arising from the no return of investment with respect to the same money market
placement involved herein, which he eventually filed against Delta Motors Corporation
and Pilipinas Bank before the Regional Trial Court of Cebu City .What is involved here
in a money market transaction. As defined by Lawrence Smith, 'the money market is a
market dealing in standardized short-term credit instruments (involving large
amounts) where lenders and borrowers do not deal directly with each other but
through a middle man or dealer in the open market. It involves 'commercial papers'
which are instruments 'evidencing indebtedness of any person or entity . . . which are
issued, endorsed, sold or transferred or in any manner conveyed to another person or
entity, with or without recourse.' The fundamental function of the money market
device in its operation is to match and bring together in a most impersonal manner
both the 'fund users and the 'fund suppliers.' The money market is an 'impersonal
market', free from personal considerations. The market mechanism is intended 'to
provide quick mobility of money and securities. The Court of Appeals, therefore,
correctly ruled that a money market transaction partakes of the nature of a loan and
therefore nonpayment thereof would not give rise to criminal liability for estafa
through misappropriation or conversion.

435 | P a g e
Law 321_Corporation LAW_ Case Digest

CONTROVERSIES ARISING OUT OF INTRA-CORPORATE OF PARTNERSHIP RELATIONS

VITALIANO N. AGUIRRE II and FIDEL N. AGUIRRE


vs.
FQB+7, INC., NATHANIEL D. BOCOBO, PRISCILA BOCOBO and ANTONIO DE
VILLA
G.R. No. 170770 January 9, 2013

FACTS:

On October 5, 2004, Vitaliano filed, in his individual capacity and on behalf of


FQB+7, Inc., a Complaint for intra-corporate dispute, injunction, inspection of
corporate books and records, and damages, against respondents Nathaniel D. Bocobo,
Priscila D. Bocobo and Antonio De Villa. The Complaint alleged that FQB+7 was
established in 1985 with the following directors and subscribers, as reflected in its
Articles of Incorporation.
The substantive changes found in the GIS, respecting the composition of
directors and subscribers of FQB+7, prompted Vitaliano to write to the "real" Board of
Directors (the directors reflected in the Articles of Incorporation), represented by Fidel
N. Aguirre. In this letter dated April 29, 2004, Vitaliano questioned the validity and
truthfulness of the alleged stockholders meeting held on September 3, 2002. He asked
the "real" Board to rectify what he perceived as erroneous entries in the GIS, and to
allow him to inspect the corporate books and records.

ISSUE:

Whether the RTC has jurisdiction over an intra-corporate dispute involving a


dissolved corporation.

RULING:

YES.

Intra-corporate disputes remain even when the corporation is dissolved.


Jurisdiction over the subject matter is conferred by law. R.A. No. 8799 conferred
jurisdiction over intra-corporate controversies on courts of general jurisdiction or
RTCs, to be designated by the Supreme Court. Thus, as long as the nature of the
controversy is intra-corporate, the designated RTCs have the authority to exercise
jurisdiction over such cases.
Thus, to be considered as an intra-corporate dispute, the case: (a) must arise
out of intra-corporate or partnership relations, and (b) the nature of the question
subject of the controversy must be such that it is intrinsically connected with the
regulation of the corporation or the enforcement of the parties‘ rights and obligations
under the Corporation Code and the internal regulatory rules of the corporation.
Examining the case before us in relation to these two criteria, the Court finds
and so holds that the case is essentially an intra-corporate dispute. It obviously arose
from the intra-corporate relations between the parties, and the questions involved
pertain to their rights and obligations under the Corporation Code and matters
relating to the regulation of the corporation. We further hold that the nature of the
case as an intra-corporate dispute was not affected by the subsequent dissolution of
the corporation.

436 | P a g e
Law 321_Corporation LAW_ Case Digest

PHILIP L. GO, PACIFICO Q. LIM and ANDREW Q. LIM


vs.
DISTINCTION PROPERTIES DEVELOPMENT AND CONSTRUCTION, INC.
G.R. No. 194024 April 25, 2012

FACTS:

Petitioners are registeredindividual owners of condominium units in Phoenix


Heights Condominium developedby the respondent.In August 2008, petitioners, as
condominium unit-owners, filed a complaint beforethe HLURB against DPDCI for
unsound business practices and violation of theMDDR, alleging that DPDCI committed
misrepresentation in their circulated flyersand brochures as to the facilities or
amenities that would be available in thecondominium and failed to perform its
obligation to comply with the MDDR.
In defense, DPDCI alleged that the brochure attached to the complaint was ―a
merepreparatory draft‖. HLURB rendered its decision in favor of petitioners. DPDCI
filedwith the CA its Petition for Certiorari and Prohibition on the ground that HLURB
actedwithout or beyond its jurisdiction.The CA ruled that the HLURB had no
jurisdiction over the complaint filed bypetitioners as the controversy did not fall within
the scope of the administrative agency‘s authority.

ISSUES:

Whether or not the HLURB has jurisdiction over the complaint filed by
the petitioners.

RULING:

NO.

Jurisdiction over the subject matter of a case is conferred by law and


determinedby the allegations in the complaint which comprise a concise statement of
theultimate facts constituting the plaintiff's cause of action. The nature of an action,
aswell as which court or body has jurisdiction over it, is determined based on
theallegations contained in the complaint of the plaintiff, irrespective of whether or
notthe plaintiff is entitled to recover upon all or some of the claims asserted therein.
The averments in the complaint and the character of the relief sought are
the ones tobe consulted. Once vested by the allegations in the complaint, jurisdiction
alsoremains vested irrespective of whether or not the plaintiff is entitled to recover
uponall or some of the claims asserted therein. Thus, it was ruled that the jurisdiction
ofthe HLURB to hear and decide cases is determined by the nature of the cause
ofaction, the subject matter or property involved and the parties.In this case, the
complaint filed by petitioners alleged causes of action thatapparently are not
cognizable by the HLURB considering the nature of the action andthe reliefs sought.

437 | P a g e
Law 321_Corporation LAW_ Case Digest

STRATEGIC ALLIANCE DEVELOPMENT CORPORATION


vs.
STAR INFRASTRUCTURE DEVELOPMENT CORPORATION ET AL.
G.R. No. 187872 November 17, 2010

FACTS:

Respondents Aderito Z. Yujuico and Bonifacio C. Sumbilla, in their respective


capacities as then President and Treasurer of STRADEC, executed a Promissory Note
for and in consideration of a loan in the sum of P10,000,000.00 ostensibly extended in
favor of said corporation by respondent Robert L. Wong, one of the incorporators of
SIDC. As security for the payment of the principal as well as the stipulated interests
thereon, a pledge constituted over STRADEC‘s entire shareholdings in SIDC was
executed by respondent Yujuico on 1 April 2005. In view of STRADEC‘s repeated
default on its obligations,11 however, the shares thus pledged were sold by way of the
26 April 2005 notarial sale conducted in Makati City by respondent Raymond M.
Caraos. Having tendered the sole bid of P11,800,000.00, respondent Wong was issued
the corresponding certificates of stocks by respondent Bede S. Tabalingcos, SIDC‘s
Corporate Secretary for the years 2004 and 2005, after the transfer was recorded in
the corporation‘s stock and transfer book.
On 17 July 2006, Cezar T. Quiambao, in his capacity as President and
Chairman of the Board of Directors of STRADEC, commenced the instant suit with the
filing of the petition which was docketed as Civil Case No. 7956 before Branch 2 of the
Regional Trial Court of Batangas City, sitting as a Special Commercial Court (SCC).

ISSUE:

Whether or not the RTC has jurisdiction over the case.

RULING:

YES.

In addition to being conferred by law, it bears emphasizing that the jurisdiction


of a court or tribunal over the case is determined by the allegations in the complaint
and the character of the relief sought, irrespective of whether or not the plaintiff is
entitled to recover all or some of the claims asserted therein. Moreover, pursuant to
Section 5.2 of Republic Act No. 8799, otherwise known as the Securities Regulation
Code, the jurisdiction of the SEC over all cases enumerated under Section 5 of
Presidential Decree No. 902-A has been transferred to RTCs designated by this Court
as SCCs pursuant to A.M. No. 00-11-03-SC promulgated on 21 November 2000.
It should be noted that the SCCs are still considered courts of general
jurisdiction. Section 5.2 of R.A. No. 8799 directs merely the Supreme Court's
designation of RTC branches that shall exercise jurisdiction over intra-corporate
disputes. Nothing in the language of the law suggests the diminution of jurisdiction of
those RTCs to be designated as SCCs. The assignment of intra-corporate disputes to
SCCs is only for the purpose of streamlining the workload of the RTCs so that certain
branches thereof like the SCCs can focus only on a particular subject matter.
The RTC exercising jurisdiction over an intra-corporate dispute can be likened
to an RTC exercising its probate jurisdiction or sitting as a special agrarian court. The
designation of the SCCs as such has not in any way limited their jurisdiction to hear
and decide cases of all nature, whether civil, criminal or special proceedings.

438 | P a g e
Law 321_Corporation LAW_ Case Digest

GD EXPRESS WORLDWIDE N.V. and AMIHAN MANAGEMENT SERVICES, INC.


vs.
HON. COURT OF APPEALS (FOURTH DIVISION), HON. SECURITIES AND
EXCHANGE COMMISSION (en banc), HON. ROSITA R. GUERRERO, in her
capacity as Hearing Officer, and FILCHART AIRWAYS, INC., Respondents
G.R. No. 136978 May 8, 2009

FACTS:

Petitioner GD Express Worldwide N.V. (GD Express) is a corporation duly


organized and existing under the laws of the Netherlands. On 27 September 1990, its
predecessor-in-interest, TNT Limited (TNT) entered into a joint venture agreement with
Philippine Aerospace Development Corporation (PADC) for the establishment of a
domestic corporation as their corporate vehicle to operate as an international air
freight carrier. The joint venture agreements stipulated that PADC would own 80% of
the shares of stock of the corporate vehicle while TNT would own the remaining 20%.
The agreements essentially laid down the relationship between TNT and PADC and the
management, control and existence of the corporation. Also, pursuant to the joint
venture agreements, PADC and TNT registered with the SEC a corporation to be
known as Air Philippines Corporation (APC).
Subsequently, on 11 December 1992, APC amended its articles of incorporation
to change its corporate name to Pacific East Asia Cargo Airlines, Inc. (PEAC). On 02
April 1993, TNT transferred all its shares in PEAC to petitioner GD Express. PEAC
immediately commenced operations. Herein petitioner Amihan Management Services,
Inc. (Amihan), a domestic corporation, was contracted to undertake the daily
operations in PEAC pursuant to the joint venture agreement.
Sometime in 1994, the Office of the President mandated the Committee on
Privatization to require the Asset Privatization Trust (APT) to dispose of PADC‘s 80%
share in PEAC. Thus, petitioner GD Express and PADC executed the Terms of
Reference that would govern the disposition of PADC‘s equity comprising 12,800
subscribed shares of stock in PEAC.

ISSUE:

Whether or not an intra-corporate dispute is exclusively cognizable by the SEC.

RULING:

NO.

The designation of certain RTC branches to handle specific cases is nothing


new. For instance, pursuant to the provisions of the R.A. No. 6657 or the
Comprehensive Agrarian Reform Law, the Supreme Court has assigned certain RTC
branches to hear and decide cases under Sections 56 and 57 of R.A. No. 6657.
The RTC exercising jurisdiction over an intra-corporate dispute can be likened
to an RTC exercising its probate jurisdiction or sitting as a special agrarian court. The
designation of the SCCs as such has not in any way limited their jurisdiction to hear
and decide cases of all nature, whether civil, criminal or special proceedings.
Incidentally, not all the prayers and reliefs sought by respondent Filchart in
SEC Case No. 08-97-5746 can be characterized as intra-corporate in nature. For
instance, respondent Filchart‘s petition does not allege that the cause of action for the
nullification of the management contract between PEAC and petitioner Amihan is
being instituted as a derivative suit. It is an ordinary action for the nullification of a
contract, which is cognizable by courts of general jurisdiction.

439 | P a g e
Law 321_Corporation LAW_ Case Digest

IGLESIA EVANGELICA METODISTA EN LAS ISLAS FILIPINAS (IEMELIF), INC.


vs.
NATANAEL B. JUANE
G.R. No. 172447. September 18, 2009

FACTS:

IEMELIF is a religious corporation existing and duly organized under Philippine


laws. IEMELIF is the absolute and registered owner of a parcel of land with Transfer
Certificate of Title No. 62080 particularly described as a parcel of land with Lot No. 77-
B-2. Likewise it is the absolute and registered owner of a parcel of land with TCT No.
14366 and situated on the SE line of Calle Sande Nos. 1462-1466, District of Tondo,
Manila. On these lots the Cathedral of the Iglesia Evangelica Metodista en las Islas
Filipinas is located together with other improvements including the Pastor‘s residence
and the church‘s school.
Juane is a former minister or pastor of IEMELIF. He was elected as one of the
members of the Highest Consistory of Elders (or Board of Trustees) of IEMELIF in the
February 2000 IEMELIF General Conference. During the concluding Anniversary
Service of said General Conference, IEMELIF Bishop Nathanael P. Lazaro, the General
Superintendent of the whole IEMELIF Church and the General Administrator of the
IEMELIF Cathedral in Tondo, Manila, during the reading of the "IEMELIF Workers‘
Assignment", announced the appointment and assignment of Juane as Resident
Pastor of the Cathedral Congregation in Tondo, Manila. By virtue and as a
consequence of such appointment, Defendant Rev. Juane was authorized to stay at
and occupy the Resident Pastor‘s residence inside the Cathedral complex. By the same
reason, he also took charge of the Cathedral facilities and other property of the church
in said premises. One year thereafter, during the traditional concluding IEMELIF
Anniversary Service of the February 2001 General Conference, Juane was re-assigned
and re-appointed by Bishop Lazaro to the same position.

ISSUE:

Whether or not the transformation of IEMELIF from corporation sole to


corporation aggregate would change its head or governing body.

RULING:

NO.

As opposed to a corporation aggregate, a corporation sole consists of a single


member, while a corporation aggregate consists of two or more persons. If the
transformation did not materialize, the corporation sole would still be Bishop Lazaro,
who himself performed the questioned acts of removing Juane as Resident Pastor of
the Tondo Congregation. If the transformation did materialize, the corporation
aggregate would be composed of the Highest Consistory of Elders, which nevertheless
approved the very same acts. As either Bishop Lazaro or the Highest Consistory of
Elders had the authority to appoint Juane as Resident Pastor of the IEMELIF Tondo
Congregation, it also had the power to remove him as such or transfer him to another
congregation.

440 | P a g e
Law 321_Corporation LAW_ Case Digest

THE INTESTATE ESTATE OF ALEXANDER T. TY


vs.
COURT OF APPEALS
G.R. No. 114672 19 April 2001

FACTS:

Sylvia S. Ty was married to Alexander T. Ty. Son of private respondent


Alejandro B. Ty. When Alexander died, petitioner was appointed administratrix of her
late husband‘s intestate estate. Thereafter, petitioner filed a motion for leave to sell or
mortgage estate property in order to generate funds for the payment of deficiency
estate taxes. Private respondent Alejandro Ty then filed two complaints for the the
recovery of the pieces of property that were placed in the name of deceased Alexander
by private respondent, the same property being sought to be sold out, mortgaged, or
disposed of by petitioner. recovery of the above-mentioned property, praying for the
declaration for nullity of the deed of absolute sale of the shares of stock executed by
private respondent in favor of the deceased Alexander, and for Private respondent
claimed in both cases that even if said property were placed in the name of deceased
Alexander, they were acquired through private respondent‘s money, without any cause
or consideration from deceased Alexander.

ISSUE:

Whether or not the case is under the jurisdiction of the RTC.

RULING:

YES.

The proper forum for such a dispute is a regular trial court since in the cases at
bar, the relationship of private respondent when he sold his shares of stock to his son
was one of vendor and vendee, nothing else. The Court agrees with the ruling of the
Court of appeals that no special corporate skill is necessary in resolving the issue of
the validity of the transfer of shares from one stockholder to another of the same
corporation. Both actions, although involving different property, sought to declare the
nullity of the transfers of said property to the decedent on the ground that they were
not supported by any cause or consideration, and thus, are considered void ab initio
for being absolutely simulated or fictitious. The determination whether a contract is
simulated or not is an issue that could be resolved by applying pertinent provisions of
the Civil Code particularly those relative to obligations and contracts. Disputes
concerning the application of the Civil Code are properly cognizable by courts of
general jurisdiction.

441 | P a g e
Law 321_Corporation LAW_ Case Digest

HERNANI FABIA
vs.
COURT OF APPEALS
G.R. No. 132684 20 August 2001

FACTS:

Hernani N. Fabia was the President of private respondent MTCP, a domestic


corporation engaged in providing maritime courses and seminars to prospective
overseas contract workers and seamen. On 3 January 1996 MTCP through its new
President Exequiel B. Tamayo filed an affidavit-complaint for estafa against Hernani N.
Fabia with the Office of the City Prosecutor of Manila alleging that on various
occasions from January to July 1994 Fabia drew cash advances from MTCP, covered
by cash vouchers, which he failed to liquidate despite repeated demands. Petitioner
Fabia in his 20 March 1996 Reply-Affidavit and Motion to Dismiss admitted having
received the various amounts covered by the cash vouchers but reasoned that they
were in the nature of simple loans that had already been liquidated and paid as shown
by the receipts and vouchers which he had attached to his pleadings. On 8 April 1996
the Office of the City Prosecutor of Manila dismissed the complaint for lack of
jurisdiction for the reason that the controversy pertained to the relationship between a
corporation and a former officer thereof, hence, it was the SEC which had original and
exclusive jurisdiction over the case.

ISSUE:

Whether or not the case is an intra-corporate dispute.

RULING:

YES.

PD 902-A confines the jurisdiction of the SEC to "intra-corporate disputes"


defined as any act or omission of the Board of Directors/Trustees of corporations, or of
partnerships, or of other associations, or of their stockholders, officers, or partners,
including any fraudulent devices, schemes or representations, in violation of any law
or rules and regulations administered and enforced by the Commission. This
underscores the relationship of the party-litigants with each other, and indicates that
the nature of the cause of action should be limited to fraudulent devices, schemes or
representations, in violation of any law, rules and/or regulations administered and
enforced by the Commission for the cause of action to fall within the ambit of
authority of the SEC, elements that are both present in the instant case. Petitioner
was the President as well as a Director and stockholder in private respondent MTCP,
who was charged with the misappropriation or diversion of corporate funds after
having failed to liquidate the amount he had received as cash advances from the
company. The charge against petitioner is for estafa, an offense punishable under The
RPC, and prosecution for the offense is presently before the regular courts. However,
as correctly pointed out by private respondent MTCP, jurisdiction is determined not
from the law upon which the cause of action is based, nor the type of proceedings
initiated, but rather, it is gleaned from the allegations stated in the complaint. It is
evident from the complaint that the acts charged are in the nature of an intra-
corporate dispute as they involve fraud committed by virtue of the office assumed by
petitioner as President, Director, and stockholder in MTCP, and committed against the
MTCP Corporation. This sufficiently removes the action from the jurisdiction of the
regular courts, and transposes it into an intra-corporate controversy within the
jurisdiction of the SEC. The fact that a complaint for estafa, a felony punishable under
the RPC, has been filed against petitioner does not negate and nullify the intra-
corporate nature of the cause of action, nor does it transform the controversy from
intra-corporate to a criminal one.
Accordingly, as the matter involves an intra-corporate dispute within the
jurisdiction of the SEC, the issue of whether prior non-accounting precludes a finding
of probable cause for the charge of estafa no longer finds relevance.

442 | P a g e
Law 321_Corporation LAW_ Case Digest

TEODORO VESAGAS
vs.
COURT OF APPEALS
G.R. No. 142924 5 December 2001

FACTS:

Plaintiffs Deflin and Raniel spouses were members in good standing of Luz
VillageTennis Club alleging that club president Vesagas summarily stripped them of
their membership without due process of law. Thus, plaintiffs moved to declare as
illegal their expulsion from the club in utter disregard of the provisions of the club‘s by
laws. Respondents, on the other hand, moved to dismiss the complaint on the ground
that the SEC lacked the jurisdiction over the subject matter. They contend that since
its inception in the 1970‘s, the club in practice has not been a corporation, and that it
was only the plaintiffs who surreptitiously caused the club‘s registration with the SEC.
Further, they argued that the club has ceased to be a corporate body at any rates
thus, no intracorpoarate relationship as between the parties.

ISSUE:

Whether or not the SEC had jurisdiction over the case.

RULING:

YES.

The case falls within the SEC jurisdiction. Petitioners are estopped from
denying the personality of the corporation by their very own acts. The dispute was
considered as intra-corporate in character because the parties involved are officers
and members of the club; the conflict arose from this relation between the parties, and
the subject of complaint involved expulsions from the club membership, validity of
amendments of by laws.

443 | P a g e
Law 321_Corporation LAW_ Case Digest

SPOUSES JOSE ABEJO AND AURORA ABEJO


vs.
HON. RAFAEL DE LA CRUZ
G.R. No. L-63558 19 May 1987

FACTS:

The Spouses Abejo sold their minority shareholdings in PBPI to TSI as well as
the shares of Virginia Braga in PBPI by virtue of the stock certificates covering the
latter‘s shares endorsed in blank by Braga. TSI therefore asked that the transfer be
recorded in the books of the corporation. The corporation‘s treasurer, Norberto Braga,
son of the Spouses Braga, refused to enter the same contending that it has pre-
emptive rights over the Abejo shares and that the certificates covering the shares of
his parent were lost. Abejo and TSI prayed for Mandamus before the SEC for Braga to
enter the name of TSI in the transfer book. Norberto Braga sought to dismiss
contending that the SEC has no jurisdiction over the nature of the action since it does
not involve an intra-corporate controversy between stockholders, the principal
petitioners, TSI, not being a stockholder of record of Pocket Bell. Later, SEC ordered
that the name of TSI be entered in the books. Meanwhile, pending SEC hearing,
Braga filed a complaint before the CFI prating for nullity and rescission of the sale of
the Abejos to TSI invoking his pre-emptive right and a complaint for nullity of the sale
of Virginia‘s shares. Abejos sought to dismiss the case before the CFI invoking lack of
jurisdiction.

ISSUE:

Whether or not the SEC has jurisdiction over the controversy.

RULING:

YES.

SC held that the SEC has original and exclusive jurisdiction over the dispute
between the principal stockholders of the corporation PBPI, namely, the Abejos and
Telectronics, the purchasers of the 56% majority stock on the one hand, and the
Bragas, erstwhile majority stockholders, on the other, and that the SEC, through its
en banc Resolution of May 15, 1984 correctly ruled in dismissing the Bragas' petition
questioning its jurisdiction, that "the issue is not the ownership of shares but rather
the nonperformance by the Corporate Secretary of the ministerial duty of recording
transfers of shares of stock of the Corporation of which he is secretary."
Section 6 further grants the SEC "in order to effectively exercise such
jurisdiction," the power, inter alia, "to issue preliminary or permanent injunctions,
whether prohibitory or mandatory, in all cases in which it has jurisdiction, and in
which cases the pertinent provisions of the Rules of Court shall apply." The very
complaint of the Bragas for annulment of the sales and transfers as filed by them in
the regular court questions the validity of the transfer and endorsement of the
certificates of stock, claiming alleged preemptive rights in the case of the Abejos'
shares and alleged loss of the certificates and lack of consent and consideration in the
case of Virginia Braga's shares. Such dispute clearly involves controversies "between
and among stockholders," as to the Abejos' right to sell and dispose of their shares to
Telectronics, the validity of the latter's acquisition of Virginia Braga's shares, who
between the Bragas and the Abejos' transferee should be recognized as the controlling
shareholders of the corporation, with the right to elect the corporate officers and the
management and control of its operations. Such a dispute and case clearly fall within
the original and exclusive jurisdiction of the SEC.

444 | P a g e
Law 321_Corporation LAW_ Case Digest

AGUINALDO
vs.
SECURITIES EXCHANGE COMMISSION
G.R. NO. 102965 January 21, 1999

FACTS:

23% of the outstanding capital stock of NADECOR is owned by a U.S.


Corporation, the Sawyer Adecor International, Inc. Aytona, Aguinaldo, Calalang,
Ricafort, and five others were elected as directors of NADECOR by the stockholders at
a meeting held and during the organization meeting held on the same day, petitioners
Aytona and Aguinaldo, and one R.H. Borsoto were elected Chairman of the Board,
President, and Corporate Secretary, respectively, of the NADECOR. Pursuant to
Section 5, Article 1 of the Amended By-Laws, those present at the stockholders'
meeting on August 17, 1981, formally convened and elected private respondent
Benjamin A. Aritao as acting chairman, and appointed Atty. Eusebio V. Tan as acting
secretary, for the continuation of the said stockholders' meeting. Immediately after the
stockholders' meeting, the newly-elected Board of Directors held an organization
meeting at which the following were elected as corporate officers, namely: Conrado T.
Calalang, chairman and president; Salvador O. Rivera, treasurer; and Benjamin V.
Aritao, corporate secretary. Thereafter, private respondents submitted to the
respondent SEC the secretary's certificate attesting to the election of the above-named
directors and officers of the NADECOR. Private respondents filed a petition for
mandamus with prayer for preliminary injunction and/or restraining order against
herein petitioners with the SEC, praying that petitioners herein be directed to
recognize the individual private respondents as the lawful and duly elected directors
and officers of the corporation, among others. SEC Director and Hearing Officer
Villanueva issued a restraining order against herein petitioners.

ISSUE:

Whether or not the SEC committed grave abuse of secretion in the issuance and
continued enforcement of the TRO, and the delay of its en banc division in resolving
the petitions which also pray for the lifting of the questioned TRO.

RULING:

YES.

Section 6 of PD 902-A grants the SEC in order to effectively exercise such


jurisdiction, the power to issue preliminary or permanent injunctions, whether
prohibitory or mandatory, in all cases in which it has jurisdiction, and in which cases
the pertinent provisions of the Rules of Court shall apply. Since the SEC is at least a
co-equal body of the Regional Trial Court when it adjudicates controversies over which
it has jurisdiction, it follows that the temporary restraining order issued by SEC must
have the same life-span as that issued by the trial court. It is a well-settled rule that a
temporary restraining order issued by a trial court has a life of only 20 days. To the
extent, therefore, that the enforcement of the temporary restraining order issued by
the respondent SEC exceeded 20 days, the SEC committed grave abuse of discretion.
However, although the questioned order no longer has any force and effect, the
respondent SEC still has the jurisdiction and obligation to proceed with the hearing of
the case on the merits and to issue the appropriate orders pursuant thereto subject to
review by the CA and eventually by the SC.

445 | P a g e
Law 321_Corporation LAW_ Case Digest

JOSE PENEYRA
vs.
HON. INTERMEDIATE APPELLATE COURT
G.R. No. L-68935 January 22, 1990

FACTS:

On May 7, 1976, the Board of Trustees of the Corregidor College Inc. awarded
the management and operation of its canteen at a monthly rental of P80.00 to
petitioners herein who are stockholders of the said College. Subsequently, upon
instructions of Eulogio Dizon, Chairman of the Board of Trustees of Corregidor
College, Inc., the rental payments of petitioners were refused, and subsequently partial
demolition of the canteen was effected. Consequently, petitioners filed in the then
Court of First Instance an action against Eulogio R. Dizon for damages with
preliminary mandatory injunction. Petitioners filed their motion for reconsideration of
the order denying the admission of their amended complaint. Two days later, Eulogio
Dizon died. Thereafter, his counsel moved to dismiss the complaint by reason thereof
which was granted. Petitioners filed a special civil action of certiorariand mandamus
against respondent judge before the IAC which later on dismissed the petition holding
that the Securities and Exchange Commission (SEC) has jurisdiction over the case,
the same being an intracorporate dispute, that the amendment to include Corregidor
College, Inc. cannot be allowed and that the action for damages against Eulogio Dizon
was extinguished by his death.

ISSUE:

Whether or not the Securities and Exchange Commission has jurisdiction over
the case.

RULING:

NO.

While it is true that petitioners herein are stockholders of Corregidor College,


Inc., the. complaint in Civil Case No. 774-G did not stem directly from such
relationship, but rather from the award to petitioners of the management and
operation of its canteen at a monthly rental of P80.00. The management of a canteen,
even if awarded to a stockholder, is outside or merely incidental to the central
operations of an educational institution. Petitioners thus convincingly argue that "the
controversy is not one where petitioners are bringing the action as stockholders but
rather as operators of the canteen under an agreement with said Board. In short, the
cause of action here is for damages arising from a violation of a contract of
management operation of the College canteen by defendant Dizon. Certainly, the
present controversy cannot qualify as an intra-controversy, its root being a contractual
breach separate and distinct from the corporate relationship between petitioners and
Corregidor College, Inc., which, it must be noted, was not even named as a defendant
in the original complaint. It was therefore patent error for the Court of Appeals to
immediately rule that the present case belongs to the SEC just because petitioners
alleged that they are stockholders of Corregidor College, Inc.
Under Section 3 of Presidential Decree 902-A, the jurisdiction of the SEC is
limited to matters intrinsically connected with the regulation of corporations,
partnerships and associations and those dealing with the internal affairs of such
entities. P.D. 902-A does not confer in the SEC absolute jurisdiction and control over
all matters affecting corporations. To uphold the appellate court's ruling would remove
without legal imprimatur from the regular courts all controversies over matters
involving or affecting corporations.

446 | P a g e
Law 321_Corporation LAW_ Case Digest

MAINLAND CONSTRUCTION
vs.
MOVILLA
G. R. No. 118088 November 23, 1995

FACTS:

Mainland Construction Co., Inc. is a domestic corporation, engaged in the


construction of roads and bridges and the operation of a service shop for the
maintenance of equipment. Respondents, on the other hand, are the surviving heirs of
complainant, Ernesto Movilla, who died during the pendency of an action with the
Labor Arbiter for unpaid salaries and the non-payment of other benefits. Ernesto was
an accountant of the corporation until he became an administrative manager. The
Department of Labor and Employment conducted a routine inspection on petitioner
corporation and found that it committed such violation of non-payment of salaries and
other benefits. On the basis of this finding, petitioner corporation was ordered by
DOLE to pay to its thirteen employees which included Movilla, the total amount of
P309,435.89, representing their salaries, holiday pay, service incentive leave pay
differentials, unpaid wages and 13th month pay. All the employees listed in the
DOLE's Order were paid by petitioner corporation, except Ernesto Movilla. Ernesto
Movilla filed a case against petitioner corporation for unpaid wages, separation pay
and attorney's fees, with the Department of Labor and Employment, Regional
Arbitration, Branch XI, Davao City. The Labor Arbiter rendered judgment dismissing
the complaint on the ground of lack of jurisdiction claiming that the case is within the
jurisdiction of the Securities and Exchange Commission being an intra corporate
matter.

ISSUE:

Where or not the NLRC has jurisdiction over the controversy.

RULING:

NO.

In order that the SEC can take cognizance of a case, the controversy must
pertain to any of the following relationships: (a) between the corporation, partnership
or association and the public; (b) between the corporation, partnership or association
and its stockholders, partners, members or officers; (c) between the corporation,
partnership or association and the State as far as its franchise, permit or license to
operate is concerned; and (d) among the stockholders, partners or associates
themselves. The fact that the parties involved in the controversy are all stockholders or
that the parties involved are the stockholders and the corporation, does not
necessarily place the dispute within the ambit of the jurisdiction of SEC. The better
policy to be followed in determining jurisdiction over a case should be to consider
concurrent factors such as the status or relationship of the parties or the nature of the
question that is the subject of their controversy. In the absence of any one of these
factors, the SEC will not have jurisdiction. Furthermore, it does not necessarily follow
that every conflict between the corporation and its stockholders would involve such
corporate matters as only the SEC can resolve in the exercise of its adjudicatory or
quasi-judicial powers.
In the case at bench, the claim for unpaid wages and separation pay filed by the
complainant against Petitioner Corporation involves a labor dispute. It does not
involve an intra-corporate matter, even when it is between a stockholder and a
corporation. It relates to an employer-employee relationship which is distinct from the
corporate relationship of one with the other.
It is pertinent to note that petitioner corporation is not prohibited from hiring
its corporate officers to perform services under a circumstance which will make him
an employee. Moreover, although a director of a corporation is not, merely by virtue of
his position, its employee, said director may act as an employee or accept duties that
make him also an employee.

447 | P a g e
Law 321_Corporation LAW_ Case Digest

SECURITIES AND EXCHANGE COMMISSION


vs.
COURT OF APPEALS
G.R. No. 93832 August 23, 1991

FACTS:

Petitioner Johnny K.H. Uy and private respondents Ban Hua Uy-Flores and Ban
Ha Uy-Chua are brother and sisters. They own several corporations, including LTBS
Marketing Corporation and the Soon Kee Commercial, Inc. All the three (3) above-
named individuals, including other members of the Uy family, were interlocking
stockholders and officers of the two aforementioned corporations. Thus, private
respondents Ban Hua Uy-Flores and Ban Ha Uy-Chua were the managing directors of
the said corporations and were in custody of the corporate accounting and tax records
as well as the funds of UBS Marketing Corporation and Soon Kee Commercial, Inc.
Private respondent Roland King is the accountant of the said corporations and other
allied Uy family enterprises. Due to serious disagreements and conflicts, the members
of the Uy family, through several conciliation meetings held before their selected Board
of Mediators, agreed to divide the family business so that the UBS Marketing
Corporation would go to petitioner Johnny K.H. Uy while the Soon Kee Commercial,
Inc. would go to the rest of the Uy family, including herein private respondents Ban
Hua Uy-Flores and Ban Ha Uy-Chua. Petitioners Johnny K.H. Uy and UBS Marketing
Corporation filed with the Securities and Exchange Commission a complaint against
the private respondents for the recovery of UBS Marketing Corporation's corporate
books, books of account, and the accounting and turn over of the funds and
properties belonging to UBS Marketing Corporation. Instead of filing an answer, the
private respondents moved to dismiss the complaint 5 on the ground that the SEC had
no jurisdiction over their person and over the nature of the action because there was
no intra-corporate relationship between the parties to the suit.

ISSUE:

Whether or not the SEC has jurisdiction over the dispute.

RULING:

YES.

Under Section 5 of PD No. 902-A, as amended by PD No. 1653, the SEC has
original and exclusive jurisdiction to hear and decide cases involving controversies
arising out of intra-corporate or partnership relations, between and among
stockholders, members or associates; between any or all of them and the corporation,
partnership or association of which they are stockholders, members of associates,
respectively; and between such corporations, partnership or association and the state
insofar as it concerns their individual franchise or right to exist as such entity.
In the case at bar, at the time of the execution of the Deed of Assignment
wherein the petitioner Johnny K-H. Uy and his wife, Magdalena Uy, assigned all their
stockholdings in Soon Kee Commercial, Inc. to the private respondents Ban Hua Uy
Flores and Ban Ha Uy-Chua and other members of the UY family, and the Deed of
Assignment wherein the private respondents Ban Hua Uy-Flores and Ban Ha Uy-
Chua, assigned all their stockholdings in UBS Marketing Corporation to the petitioner
Johnny K.H. Uy or to his wife, the petitioner Johnny KH. Uy and the private
respondents Ban Hua Uy-Flores and Ban Ha Uy-Chua were all interlocking
stockholders and officers of the two (2) corporations owned by the Uy family. Hence,
the deeds of assignment were intra-corporate transactions which arose from intra-
corporate relations or between and among the stockholders of the two (2) family
corporations. The controversy subject of SEC Case No. 03328 is, therefore, an intra-
corporate controversy which falls within the original and exclusive jurisdiction of the
SEC under Section 5(b) of PD No. 902-A, as amended.

448 | P a g e
Law 321_Corporation LAW_ Case Digest

SUNSET VIEW CONDOMINIUM CORPORATION


vs.
JOSE C. CAMPOS, JR
G.R. No. L-52361 April 27, 1981

FACTS:

The private respondent, Aguilar-Bernares Realty, a sole proprietorship with


business name registered with the Bureau of Commerce, owned and operated by the
spouses Emmanuel G. Aguilar and Zenaida B. Aguilar, is the assignee of a unit,
"Solana", in the Sunset View Condominium Project with La Perla Commercial,
Incorporated, as assignor. The La Perla Commercial, Incorporated bought the "Solana"
unit on installment from the Tower Builders, Inc. The petitioner, Sunset View
Condominium Corporation, filed for the collection of assessments levied on the unit
against Aguilar-Bernares Realty, private respondent. The petitioner filed its amended
complaint for the collection of overdue accounts on assessments and insurance
premiums and the interest against the private respondent Lim Siu Leng to whom was
assigned a unit called "Alegria" of the Sunset View Condominium Project by Alfonso Uy
who had entered into a "Contract to Buy and Sell" with Tower Builders, Inc. over the
said unit on installment basis. The private respondent filed a motion to dismiss on the
ground of lack of jurisdiction, alleging that the amount sought to be collected is an
assessment. The correctness and validity of which is certain to involve a dispute
between her and the petitioner corporation; that she has automatically become, as a
purchaser of the condominium unit, a stockholder of the petitioner pursuant to
Section 2 of the Condominium Act, Republic Act No. 4726; that the dispute is intra-
corporate and is consequently under the exclusive jurisdiction of the Securities &
Exchange Commission as provided in Section 5 of P.D. No. 902-A.

ISSUES:

Whether or not the Securities & Exchange Commission has jurisdiction over
cases for collection of assessments assessed by the Condominium Corporation on
condominium units the full purchase price of which has not been paid.

RULING:

NO.

Section 5 of the Condominium Act expressly provides that the shareholding in


the Condominium Corporation will be conveyed only in a proper case. It provides that
any transfer or conveyance of a unit or an apartment, office or other space therein,
shall include the transfer or conveyance of the undivided interests in the common
areas or, in a proper case, the membership or shareholding in the condominium
corporation. It is clear then that not every purchaser of a condominium unit is a
shareholder of the condominium corporation. The Condominium Act leaves to the
Master Deed the determination of when the shareholding will be transferred to the
purchaser of a unit.
Pursuant to the above statutory provision, ownership of a unit is a condition
sine qua nonto being a shareholder in the condominium corporation. The private
respondents, therefore, who have not fully paid the purchase price of their units and
are consequently not owners of their units are not members or shareholders of the
petitioner condominium corporation.
Inasmuch as the private respondents are not shareholders of the petitioner
condominium corporation, the instant case for collection cannot be a controversy
arising out of intra corporate or partnership relations between and among
stockholders, members or associates; between any or all of them and the corporation,
partnership or association of which they are stockholders, members or associates,
respectively" which controversies are under the original and exclusive jurisdiction of
the Securities & Exchange Commission, pursuant to Section 5 (b) of P.D. No. 902- A.
The subject matters of the instant cases according to the allegations of the complaints
are under the jurisdiction of the regular courts.

449 | P a g e
Law 321_Corporation LAW_ Case Digest

WESTERN INSTITUTE OF TECHNOLOGY, INC.


vs.
SALAS
G.R. No. 113032 August 21, 1997

FACTS:

Private respondents Ricardo T. Salas, Salvador T. Salas, Soledad Salas-


Tubilleja, Antonio S. Salas, and Richard S. Salas, belonging to the same family, are
the majority and controlling members of the Board of Trustees of Western Institute of
Technology, Inc., a stock corporation engaged in the operation, among others, of an
educational institution. According to petitioners, the minority stockholders of WIT, a
Special Board Meeting was held. In attendance were other members of the Board
including one of the petitioners Reginald Villasis. In said meeting, the Board of
Trustees passed Resolution No. 48, s. 1986, granting monthly compensation to the
private respondents as corporate officers retroactive June 1, 1985. A few years later,
petitioners Homero Villasis, Prestod Villasis, Reginald Villasis and Dimas Enriquez
filed an affidavit-complaint against private respondents before the Office of the City
Prosecutor, as a result of which two (2) separate criminal informations, one for
falsification of a public document and the other for estafa, were filed before the
Regional Trial Court. The charge for falsification of public document was anchored on
the private respondents' submission of WIT's income statement for the fiscal year
1985-1986 with the Securities and Exchange Commission reflecting therein the
disbursement of corporate funds for the compensation of private respondents based
on Resolution No. 4, series of 1986, making it appear that the same was passed by the
board on March 30, 1986, when in truth, the same was actually passed on June 1,
1986, a date not covered by the corporation's fiscal year 1985-1986. Thereafter, trial
for the two criminal cases, was consolidated. After a full-blown hearing, Judge Porfirio
Parian handed down a verdict of acquittal on both counts without imposing any civil
liability against the accused therein. Petitioners filed a Motion for Reconsideration of
the civil aspect of the RTC Decision which was, however, denied in an Order.

ISSUE:

Whether or not the case is derivative suit correctly filed in the Regional Trial
Court.

RULING:

NO.

Granting, for purposes of discussion, that this is a derivative suit as insisted by


petitioners, which it is not, the same is outrightly dismissible for having been
wrongfully filed in the regular court devoid of any jurisdiction to entertain the
complaint. The ease should have been filed with the Securities and Exchange
Commission (SEC) which exercises original and exclusive jurisdiction over derivative
suits, they being intra-corporate disputes, per Section 5 (b) of P.D. No. 902-A: ―In
addition to the regulatory and adjudicative functions of the Securities and Exchange
Commission over corporations, partnerships and other forms of associations
registered with it as expressly granted under existing laws and decrees, it shall have
original and exclusive jurisdiction to hear and decide cases involving: Controversies
arising out of intra-corporate or partnership relations, between and among
stockholders, members, or associates; between any or all of them and the corporation,
partnership or association of which they are stockholders, members or associates,
respectively; and between such corporation, partnership or association and the State
insofar as it concerns their individual franchise or right to exist as such entity.

450 | P a g e
Law 321_Corporation LAW_ Case Digest

CONTROVERSIES IN THE ELECTION OR APPOINTMENT/DISMISSAL

RENATO REAL
vs.
SANGU PHILIPPINES, INC. and/ or KIICHI ABE
G.R. No. 168757. January 19, 2011

FACTS:

Petitioner Renato Real was the Manager of respondent corporation Sangu


Philippines, Inc., a corporation engaged in the business of providing manpower for
general services, like janitors, janitresses and other maintenance personnel, to various
clients. In 2001, petitioner, together with 29 others who were either janitors,
janitresses, leadmen and maintenance men, all employed by respondent corporation,
filed their respective Complaints for illegal dismissal against the latter and respondent
Kiichi Abe, the corporation‘s Vice-President and General Manager. These complaints
were later on consolidated.
With regard to petitioner, he was removed from his position as Manager
through Board Resolution 2001-033 adopted by respondent corporation‘s Board of
Directors. Petitioner complained that he was neither notified of the Board Meeting
during which said board resolution was passed nor formally charged with any
infraction. He just received from respondents a letter4 dated March 26, 2001 stating
that he has been terminated from service effective March 25, 2001 for the following
reasons: (1) continuous absences at his post at Ogino Philippines Inc. for several
months which was detrimental to the corporation‘s operation; (2) loss of trust and
confidence; and, (3) to cut down operational expenses to reduce further losses being
experienced by respondent corporation.

ISSUE:

Whether or not petitioner‘s complaint for illegal dismissal constitutes an intra-


corporate controversy and thus, beyond the jurisdiction of the Labor Arbiter.

RULING:

NO.

With the elements of intra-corporate controversy being absent in this case, we


thus hold that petitioner‘s complaint for illegal dismissal against respondents is not
intra-corporate. Rather, it is a termination dispute and, consequently, falls under the
jurisdiction of the Labor Arbiter pursuant to Section 217 of the Labor Code.
With the foregoing, it is clear that the CA erred in affirming the decision of the
NLRC which dismissed petitioner‘s complaint for lack of jurisdiction. In cases such as
this, the Court normally remands the case to the NLRC and directs it to properly
dispose of the case on the merits. "However, when there is enough bases on which a
proper evaluation of the merits of petitioner‘s case may be had, the Court may
dispense with the time-consuming procedure of remand in order to prevent further
delays in the disposition of the case." ‗It is already an accepted rule of procedure for us
to strive to settle the entire controversy in a single proceeding, leaving no root or
branch to bear the seeds of litigation. If, based on the records, the pleadings, and
other evidence, the dispute can be resolved by us, we will do so to serve the ends of
justice instead of remanding the case to the lower court for further proceedings."We
have gone over the records before us and we are convinced that we can now altogether
resolve the issue of the validity of petitioner‘s dismissal and hence, we shall proceed to
do so.

451 | P a g e
Law 321_Corporation LAW_ Case Digest

MARC II MARKETING, INC. and LUCILA V. JOSON


vs.
ALFREDO M. JOSON
G.R. No. 171993.December 12, 2011

FACTS:

Marc II Marketing, Inc. and Lucila Joson is assailing the decision of the CA for
reversing and settling aside the Resolution of the National Labor Relations
Commission. Marc II Marketing, Inc. is a corporation duly organized and existing
under and by virtue of the laws of the Philippines. It is primarily engaged in buying,
marketing, selling and distributing in retail or wholesale for export or import
household appliances and products and other items. Petitioner Lucila is the President
and majority stockholder of the corporation. Before Marc II Marketing, Inc. was
officially incorporated, Alfredo has already been engaged by Lucila, in her capacity as
President, to work as General Manager of the corporation and it was formalized
through the execution of a Management Contract dated in 1994 under Marc
Marketing, Inc., as Marc II Marketing, Inc. was yet to be incorporated. For occupying
the said position, respondent was among the corporation‘s corporate officers by the
express provision of Section 1, Article IV of its by-laws.
Alfredo was appointed as one of its officers with the designation or title of
General Manager to function as a managing director with other duties and
responsibilities that the Board may provide and authorized. However, in 1997, Marc II
Marketing Inc. decided to stop and cease its operation as evidenced by an Affidavit of
Non-Operation due to poor sales collection aggravated by the inefficient management
of its affairs. Alfredo was informed of the cessation of its business operations and the
termination of his services as General Manager. He filed action for reinstatement and
money claim against petitioners.

ISSUE:

Whether or not Marc II Marketing Inc.‘s Board of Directors could create a


position for corporate officers through an enabling clause found in its corporate by-
laws.

RULING:

YES.

Accordingly, in determining whether the SEC (now the RTC) has jurisdiction
over the controversy, the status or relationship of the parties and the nature of the
question that is the subject of their controversy must be taken into consideration.
With all the foregoing, this Court is fully convinced that, indeed, respondent, though
occupying the General Manager position, was not a corporate officer of Petitioner
Corporation rather he was merely its employee occupying a high-ranking position.
Accordingly, respondent‘s dismissal as Petitioner Corporation‘s General
Manager did not amount to an intra-corporate controversy. Jurisdiction therefor
properly belongs with the Labor Arbiter and not with the RTC.
Having established that respondent was not Petitioner Corporation‘s corporate
officer but merely its employee, and that, consequently, jurisdiction belongs to the
Labor Arbiter.

452 | P a g e
Law 321_Corporation LAW_ Case Digest

MATLING INDUSTRIAL AND COMMERCIAL CORPORATION


vs.
RICARDO R. COROS
G.R. No. 157802. October 13, 2010

FACTS:

After his dismissal by Matling as its Vice President for Finance and
Administration, the respondent filed on August 10, 2000 a complaint for illegal
suspension and illegal dismissal against Matling and some of its corporate officers
(petitioners) in the NLRC, Sub-Regional Arbitration Branch XII, Iligan City.
The petitioners moved to dismiss the complaint, raising the ground, among
others, that the complaint pertained to the jurisdiction of the Securities and Exchange
Commission (SEC) due to the controversy being intra-corporate inasmuch as the
respondent was a member of Matling‘s Board of Directors aside from being its Vice-
President for Finance and Administration prior to his termination.
The respondent opposed the petitioners‘ motion to dismiss, insisting that his
status as a member of Matling‘s Board of Directors was doubtful, considering that he
had not been formally elected as such; that he did not own a single share of stock in
Matling, considering that he had been made to sign in blank an undated indorsement
of the certificate of stock he had been given in 1992; that Matling had taken back and
retained the certificate of stock in its custody; and that even assuming that he had
been a Director of Matling, he had been removed as the Vice President for Finance and
Administration, not as a Director, a fact that the notice of his termination dated April
10, 2000 showed.

ISSUE:

Whether or not the case is considered as an intra corporate controversy.

RULING:

NO.

It appears that private respondent was appointed Accounting Clerk by the Bank
on July 14, 1963. From that position she rose to become supervisor. Then in 1982,
she was appointed Assistant Vice-President which she occupied until her illegal
dismissal on July 19, 1991. The bank‘s contention that she merely holds an elective
position and that in effect she is not a regular employee is belied by the nature of her
work and her length of service with the Bank. As earlier stated, she rose from the
ranks and has been employed with the Bank since 1963 until the termination of her
employment in 1991. As Assistant Vice President of the Foreign Department of the
Bank, she is tasked, among others, to collect checks drawn against overseas banks
payable in foreign currency and to ensure the collection of foreign bills or checks
purchased, including the signing of transmittal letters covering the same. It has been
stated that "the primary standard of determining regular employment is the
reasonable connection between the particular activity performed by the employee in
relation to the usual trade or business of the employer. Additionally, "an employee is
regular because of the nature of work and the length of service, not because of the
mode or even the reason for hiring them." As Assistant Vice-President of the Foreign
Department of the Bank she performs tasks integral to the operations of the bank and
her length of service with the bank totaling 28 years speaks volumes of her status as a
regular employee of the bank. In fine, as a regular employee, she is entitled to security
of tenure; that is, her services may be terminated only for a just or authorized cause.
This being in truth a case of illegal dismissal, it is no wonder then that the Bank
endeavored to the very end to establish loss of trust and confidence and serious
misconduct on the part of private respondent but, as will be discussed later, to no
avail.

453 | P a g e
Law 321_Corporation LAW_ Case Digest

GARCIA
vs.
EASTERN TELECOMMUNICATIONS PHILIPPINES, INC.
G.R. No. 173115 April 16, 2009

FACTS:

Atty. Virgilio R. Garcia was the Vice President and Head of Business Support
Services and Human Resource Departments of the Eastern Telecommunications
Philippines, Inc. (ETPI) while Atty. Salvador C. Hizon is the President/Chief Executive
Officer. On 16 January 2000, Atty. Garcia was placed under preventive suspension
based on three complaints for sexual harassment and was eventually dismissed
though a letter by the Atty. Hizon. A complaint-affidavit for illegal dismissal with
prayer for full backwages and recovery of moral and exemplary damages was filed by
Atty. Virgilio R. Garcia against ETPI and Atty. Salvador C. Hizon. Atty. Garcia filed a
Motions to Inhibit, praying that Labor Arbiter Libo-on inhibit himself from further
proceeding with the case, on the ground that he was a fraternity brother of Atty. Hizon
but said motions were denied. Upon appeal to the NLRC, the motion to inhibit was
granted and the case was re raffled to another Labor Arbiter who found the preventive
suspension and subsequent dismissal of Atty. Garcia illegal. An Alias writ of execution
was issued for the garnishment of the amount representing his monthly salaries for
two months and thirteenth month pay which was satisfied. Upon appeal, The
Commission ruled that the dismissal of Atty. Garcia, being ETPI‘s Vice President,
partook of the nature of an intra-corporate dispute cognizable by Regional Trial Courts
and not by Labor Arbiters.

ISSUE:

Whether or not the question of legality or illegality of the removal or termination


of employment of an officer of a corporation is an intra corporate controversy that falls
under the original exclusive jurisdiction of the Regional Trial Courts.

RULING:

YES.

A corporate officer‘s dismissal or removal is always a corporate act and/or an


intra-corporate controversy, over which the Regional Trial Court has original and
exclusive jurisdiction. Before a dismissal or removal could properly fall within the
jurisdiction of the SEC, it has to be first established that the person removed or
dismissed was a corporate officer. In the case the by-laws of ETPI.
Atty. Garcia tries to deny he is an officer of ETPI. Not being a corporate officer,
he argues that the Labor Arbiter has jurisdiction over the case. One of the corporate
officers provided for in the by-laws of ETPI is the Vice-President. It is therefore clear
from the by-laws and from Atty. Garcia himself that he is a corporate officer. One who
is included in the by-laws of a corporation in its roster of corporate officers is an
officer of said corporation and not a mere employee. Being a corporate officer, his
removal is deemed to be an intra-corporate dispute cognizable by the SEC and not by
the Labor Arbiter.
Atty. Garcia‘s ouster as Vice-President, who is a corporate officer of ETPI,
partakes of the nature of an intra-corporate controversy, jurisdiction over which is
vested in the SEC now the RTC. The Labor Arbiter thus erred in assuming jurisdiction
over the case filed by Atty. Garcia, because he had no jurisdiction over the subject
matter of the controversy.

454 | P a g e
Law 321_Corporation LAW_ Case Digest

ARMANDO T. DE ROSSI
vs.
NATIONAL LABOR RELATIONS COMMISSION
G.R. No. 108710 September 14, 1999

FACTS:

An Italian citizen, petitioner was the Executive Vice-President and General


Manager of private respondent, Matling Industrial and Commercial Corporation
(MICC). He started work on July 1, 1985. On August 10, 1988, MICC terminated his
employment. Aggrieved, petitioner filed with the NLRC, National Capital Region on
September 21, 1989, a complaint for illegal dismissal with corresponding damages.
MICC based petitioner's dismissal on the ground that the petitioner failed to secure
his employment permit, grossly mismanaged the business affairs of the company, and
misused corporate funds. However, petitioner argued that it was the duty of the
company to secure his work permit during the term of his office, and that his
termination was illegal for lack of just cause. On November 27 1991, Labor Arbiter
Asuncion rendered a decision in favor of petitioner where a writ of execution was
issued to collect the back wages of petitioner and giving MICC the option to reinstate
petitioner physically or constructively through payroll reinstatement. Upon appeal, the
NLRC dismissed the case for lack of jurisdiction.

ISSUE:

Whether or not the NLRC has jurisdiction over the dismissal case.

RULING:

NO.

The SEC, and not the NLRC, has original and exclusive jurisdiction over cases
involving the removal of corporate officers. Section 5, paragraph (c) of P.D. 902-A
unequivocally provides that SEC has jurisdiction over intra-corporate affairs regarding
the election or appointment of officers of a corporation.
An "office" is created by the charter of the corporation under which a
corporation is organized, and the officer is elected by the directors or stockholders. In
the present case, private respondents aver that the officers and their terms of office
are prescribed by the corporation's by-laws.The by-laws being in force, clearly
petitioner is considered an officer of MICC, elected and/or designated by its board of
directors.
A corporate officer's removal from his office is a corporate act. If such removal
occasions an intra-corporate controversy, its nature is not altered by the reason or
wisdom, or lack thereof, with which the Board of Directors might have in taking such
action. When petitioner, as Executive Vice-President allegedly diverted company funds
for his personal use resulting in heavy financial losses to the company, this matter
would amount to fraud. Such fraud would be detrimental to the interest not only of
the corporation but also of its members. This type of fraud encompasses controversies
in a relationship within the corporation covered by SEC jurisdiction. Perforce, the
matter would come within the area of corporate affairs and management, and such a
corporate controversy would call for the adjudicative expertise of the SEC, not the
Labor Arbiter or the NLRC.

455 | P a g e
Law 321_Corporation LAW_ Case Digest

LESLIE W. ESPINO
vs.
NATIONAL LABOR RELATIONS COMMISSION
G.R. Nos. 109642-43 January 5, 1995

FACTS:

Petitioner Leslie W. Espino was the Executive Vice President-Chief Operating


Officer of private respondent Philippine Airlines (PAL) when his services were
terminated sometime in December 1990 by the Board of Directors of PAL as a result of
the findings of the panels created by then President Corazon C. Aquino to investigate
the administrative charges filed against him and other senior officers for their
purported involvement in four, denominated "Goldair," "Robelle," "Kasbah/La
Primavera," and "Middle East" which allegedly prejudiced the interests of both PAL and
the Philippine Government. Except for the conflict of interest charges in the "Robelle"
case, petitioner and several other senior officers of PAL were uniformly charged in the
three (3) other aforementioned cases of gross incompetence, mismanagement,
inefficiency, negligence, mismanagement, dereliction of duty, failure to observe and/or
implement administrative and executive policies, and related acts or omissions
resulting in the concealment or coverup and prevention of the seasonable discovery of
anomalous transactions which, as a consequence, caused prejudice to the best
interest of PAL and the Government. As a result of his termination, petitioner Espino
filed a complaint for illegal dismissal against PAL with the National Labor Relations
Commission, praying, among others, for reinstatement with backwages, recovery of
P50 Million as moral damages, P10 Million as exemplary damages and attorney's fees
which was granted and a writ of execution was issued.

ISSUE:

Whether or not the NLRC has jurisdiction over the case.

RULING:

NO.

In intra-corporate concerning the election or appointment of officers of a


corporation, Section 5, PD 902-A specifically provides that in addition to the
regulatory and adjudicative functions of the Securities and Exchange Commission over
corporations, partnerships and other forms of associations registered with it as
expressly granted under existing laws and decrees, it shall have original and exclusive
jurisdiction to hear and decide cases involving controversies in the election or
appointments of directors, trustees, officers or managers of such corporations,
partnerships or associations.
Indisputably, the position of Executive Vice President-Chief Operating Officer
from which petitioner Espino claims to have been illegally dismissed, is an elective
office under Section 7, Article III is an elective corporate office under Section 1, Article
IV of the Amended by-Laws of PAL. He lost that position when his appointment or
election as Executive Vice President-Chief Operating Officer, together with other senior
officers who were similarly charged administratively, were deferred by the Board of
Directors in its organizational meeting on October 19, 1990. He was later considered
by the Board as resigned from the service, for reasons earlier stated, and the said
position was later abolished.
A corporate officer's dismissal is always a corporate act and/or an intra-
corporate controversy and that nature is not altered by the reason or wisdom which
the Board of Directors may have in taking such action.Furthermore, it must be noted
that the reason behind the non-election of petitioner to the position of Executive Vice
President-Chief Operating Officer arose from, or is closely connected with, his
involvement in the alleged irregularities in the aforementioned cases which, upon
investigation and recommendation, were resolved by the PAL Board of Directors
against him and other senior officers. Evidently, this intra-corporate ruling places the
instant case under the specialized competence and expertise of the SEC.

456 | P a g e
Law 321_Corporation LAW_ Case Digest

JOSEMARIA G. ESTRADA
vs.
NATIONAL LABOR RELATIONS COMMISSION
G.R. No. 106722 October 4, 1996

FACTS:

Petitioner Josemaria Estrada was the Senior Vice-President marketing Group of


private respondent Philippine Airlines Inc., responsible for the development of
corporate marketing plans and strategies of PAL and for providing direction on all
passenger and cargo sales and services activities at international and domestic
airports. In June of 1990, petitioner was implicated by then Solicitor General
Francisco Chavez in the much-publicized P2 billion anomaly in PAL. Accordingly, he
was administratively charged and thereafter preventively suspended until he was
finally dismissed.

ISSUE:

Whether or not the NLRC has jurisdiction over the case for illegal termination
filed by petitioner.

RULING:

YES.

A corporate officer‘s dismissal is always a corporate act and/or intra-corporate


controversy and that nature is not altered by the reason or wisdom which the Board of
Directors may have in taking such action. Not the least insignificant in the case at
bench is that petitioners dismissal is intertwined with still another intra-corporate
affair, earlier so ascribed as the two-billion-peso PAL scam, that inevitably places the
case under the specialized competence of the SEC and well beyond the ambit of a
labor arbiters normal jurisdiction under the general provisions of Article 217 of the
Labor Code.
The fact that petitioner sought payment of his back wages, other benefits, as
well as moral and exemplary damages and attorneys fees in his complaint for illegal
dismissal will not operate to prevent the SEC from exercising its jurisdiction under PD
902-A. While the affirmative reliefs and monetary claims sought by petitioner in his
complaint may, at first glance, mislead one into placing the case under the jurisdiction
of the Labor Arbiter, a closer examination reveals that they are actually part of the
perquisites of his elective position; hence, intimately linked with his relations with the
corporation.
Anent the issue on estoppel, suffice it to state that there is nothing on record to
show that PAL was guilty of the same. In fact, the court noted that initially at the
arbitration level, PAL already questioned the jurisdiction of the labor Arbiter on the
ground that petitioner‘s recourse should have been with the Office of the President.
While the reason therein proffered by PAL may be incorrect, it did not alter the fact
that PAL indeed questioned the jurisdiction of the labor Arbiter. At any rate, our
settled rule is that jurisdiction over the subject matter is conferred by law, and may be
questioned at anytime even on appeal.

457 | P a g e
Law 321_Corporation LAW_ Case Digest

ISLAMIC DIRECTORATE OF THE PHILIPPINES


vs.
COURT OF APPEALS
G.R. No. 117897 May 14, 1997

FACTS:

Petitioner IDP-Tamano Group alleges that Islamic leaders of all Muslim major
tribal groups in the Philippines headed by Dean Cesar Adib Majul organized and
incorporated the ISLAMIC DIRECTORATE OF THE PHILIPPINES (IDP), the primary
purpose of which is to establish an Islamic Center in Quezon City for the construction
of a Mosque, Madrasah, and other religious infrastructures so as to facilitate the
effective practice of Islamic faith in the area. The Libyan government donated money to
the IDP to purchase land at Culiat, Tandang Sora, Quezon City, to be used as a Center
for the Islamic populace. After the purchase of the land by the Libyan government in
the name of IDP, Martial Law was declared by the late President Ferdinand Marcos.
Most of the members of the 1971 Board of Trustees flew to the Middle East to escape
political persecution. Two Muslim groups sprung, the Carpizo Group, headed by
Engineer Farouk Carpizo, and the Abbas Group, led by Mrs. Zorayda Tamano and
Atty. Firdaussi Abbas. Both groups claimed to be the legitimate IDP. Significantly, on
October 3, 1986, the SEC, in a suit between these two contending groups, came out
with a Decision in SEC Case No. 2687 declaring the election of both the Carpizo Group
and the Abbas Group as IDP board members to be null and void.
Neither group, however, took the necessary steps prescribed by the SEC in its
October 3, 1986 Decision, and, thus, no valid election of the members of the Board of
Trustees of IDP was ever called. Although the Carpizo Group attempted to submit a set
of by-laws, the SEC found that, aside from Engineer Farouk Carpizo and Atty. Musib
Buat, those who prepared and adopted the by-laws were not bona fide members of the
IDP, thus rendering the adoption of the by-laws likewise null and void. without having
been properly elected as new members of the Board of Trustee of IDP.

ISSUE:

Whether or not the sale of two (2) parcels of land between the IDP-Carpizo
Group and private respondent INC null and void.

RULING:

YES.

There can be no question as to the authority of the SEC to pass upon the issue
as to who among the different contending groups is the legitimate Board of Trustees of
the IDP since this is a matter properly falling within the original and exclusive
jurisdiction of the SEC by virtue of Sections 3 and 5(c) of Presidential Decree No. 902-
A. If the SEC can declare who is the legitimate IDP Board, then by parity of reasoning,
it can also declare who is not the legitimate IDP Board. This is precisely what the SEC
did in SEC Case No. 4012 when it adjudged the election of the Carpizo Group to the
IDP Board of Trustees to be null and void. By this ruling, the SEC in effect made the
unequivocal finding that the IDP-Carpizo Group is a bogus Board of Trustees.
Consequently, the Carpizo Group is bereft of any authority whatsoever to bind IDP in
any kind of transaction including the sale or disposition of ID property.
The SEC already declared the election of the Carpizo Group as well as the
Abbas Group)to the IDP Board as null and void for being violative of the Articles of
Incorporation. In this case, the IDP, owner of the subject parcels of land, never gave its
consent, thru a legitimate Board of Trustees, to the disputed Deed of Absolute Sale
executed in favor of INC. This is, therefore, a case not only of vitiated consent, but one
where consent on the part of one of the supposed contracting parties is totally
wanting. Ineluctably, the subject sale is void and produces no effect whatsoever.

458 | P a g e
Law 321_Corporation LAW_ Case Digest

BIENVENIDO ONGKINGCO
vs.
NATIONAL LABOR RELATIONS
G.R. No. 119877 March 31, 1997

FACTS:

Petitioner Galeria de Magallanes Condominium Association, Inc. is a non-stock,


non-profit corporation formed with a primary purpose to hold title to the common
areas of the Galeria de Magallanes Condominium Project and to manage and
administer the same for the use and convenience of the residents and/or owners.
Petitioner Bienvenido Ongkingco was the president of Galeria at the time private
respondent filed his complaint. On 1 September 1990, Galeria's Board of Directors
appointed private respondent Federico B. Guilas as Administrator/Superintendent. He
was given a "monthly salary of P10,000.00 subject to review after five (5) months and
subsequently thereafter as Galeria's finances improved.
As Administrator, private respondent was tasked with the maintenance of the
performance and elegance of the common areas of the condominium and external
appearance of the compound thereof for the convenience and comfort of the residents
as well as to keep up the quality image, and hence the value of the investment for the
owners thereof. However, through a resolution passed by the Board of Directors of
Galeria, private respondent was not re-appointed as Administrator. As a result, private
respondent instituted a complaint against petitioners for illegal dismissal and non-
payment of salaries with the NLRC.

ISSUE:

Whether or not private respondent was illegally dismissed.

RULING:

YES.

While it may be true that the termination of the complainant was effected
allegedly by a resolution of the Board of Directors of the respondent association, this
did not make the dispute intracorporate in nature. The complainant is neither a
member of the association nor an officer thereof. he is an employee of the respondent
association occupying the position of administrator who is in charge with the function
of managing and administering the building or condominium owned by the members.
Indeed, there is a whale of difference between a member of the association who is a
part owner of the building and a mere employee performing managerial and
administrative functions which are necessary in the usual undertaking of the
respondent Association. The complainant falls under the second category.
It needs to be stressed that the fact that the complainant was removed by the Board of
Directors did not change the issue from an illegal dismissal case to an
intracorporate one. For, what remains to be resolved here is whether or not the
complainant's removal from his position as Administrator was for a just and valid
cause and in compliance with due process. And, as the facts now stand, the issue is
within the scope of authority of the National Labor Relations Commission to resolve.
As obtaining in this case, no intracorporate controversy exists, hence, the jurisdiction
of the NLRC should be sustained.

459 | P a g e
Law 321_Corporation LAW_ Case Digest

EFREN P. PAGUIO
vs.
NATIONAL LABOR RELATIONS COMMISSION
G.R. No. 147816 May 9, 2003

FACTS:

On 22 June 1992, respondent Metromedia Times Corporation entered, for the


fifth time, into an agreement with petitioner Efren P. Paguio, appointing the latter to
be an account executive of the firm.1 Again, petitioner was to solicit advertisements for
"The Manila Times," a newspaper of general circulation, published by respondent
company. Petitioner, for his efforts, was to receive compensation consisting of a 15%
commission on direct advertisements less withholding tax and a 10% commission on
agency advertisements based on gross revenues less agency commission and the
corresponding withholding tax. The commissions, released every fifteen days of each
month, were to be given to petitioner only after the clients would have paid for the
advertisements. Apart from commissions, petitioner was also entitled to a monthly
allowance of P2,000.00 as long as he met the P30,000.00-monthly quota. When his
services were terminated, petitioner filed for illegal dismissal.

ISSUE:

Whether or not petitioner‘s contractual relationship with respondent company


was one of a regular employment and whether his dismissal was valid.

RULING:

YES.

A regular employee is one who is engaged to perform activities which are


necessary and desirable in the usual business or trade of the employer as against
those which are undertaken for a specific project or are seasonal. Even in these latter
cases, where such person has rendered at least one year of service, regardless of the
nature of the activity performed or of whether it is continuous or intermittent, the
employment is considered regular as long as the activity exists, it not being
indispensable that he be first issued a regular appointment or be formally declared as
such before acquiring a regular status.
That petitioner performed activities which were necessary and desirable to the
business of the employer, and that the same went on for more than a year, could
hardly be denied. Petitioner was an account executive in soliciting advertisements,
clearly necessary and desirable, for the survival and continued operation of the
business of respondent corporation. Robina Gokongwei, its President, herself admitted
that the income generated from paid advertisements was the lifeblood of the
newspaper's existence. Implicitly, respondent corporation recognized petitioner's
invaluable contribution to the business when it renewed, not just once but five times,
its contract with petitioner.
A lawful dismissal must meet both substantive and procedural requirements; in
fine, the dismissal must be for a just or authorized cause and must comply with the
rudimentary due process of notice and hearing. It is not shown that respondent
company has fully bothered itself with either of these requirements in terminating the
services of petitioner. The notice of termination recites no valid or just cause for the
dismissal of petitioner nor does it appear that he has been given an opportunity to be
heard in his defense.

460 | P a g e
Law 321_Corporation LAW_ Case Digest

PEARSON & GEORGE, (S.E. ASIA), INC.


vs.
NLRC
G.R. No. 113928 February 1, 1996

FACTS:

The petitioner insists that the Labor Arbiter and the NLRC do not have
jurisdiction over the private respondent‘s complaint for illegal dismissal arising out of
his removal as Managing Director of the petitioner due to his non-reelection and the
abolition of the said position. It claims that the matter is intra-corporate and thus
falls within the exclusive jurisdiction of the Securities and Exchange Commission
pursuant to Section 5(c) of P.D. No. 902-A.
Private respondent Leopoldo Llorente was a member of the Board of Directors of
the petitioner and was elected as Vice-Chairman of the Board and as Managing
Director for a term of one year and until his successor should have been duly elected
pursuant to the petitioner‘s by-laws. On 29 January 1990, Llorente was preventively
suspended, with pay, by reason of alleged anomalous transactions entered by him,
which were prejudicial to the interest of the petitioner. Llorente, protested his
suspension and requested an examination of the supporting documents to enable him
to explain the accusations leveled against him, but to no avail.
At the regular stockholders‘ meeting on 5 March 1990, the stockholders of the
petitioner elected a new set of directors. Llorente was not reelected. On the same day,
the new Board of Directors held a meeting wherein it elected a new set of officers and
abolished the position of Managing Director. The petitioner‘s counsel informed
Llorente of his non-reelection, the abolition of the position of Managing Director, and
his termination for cause. Llorente filed with the Labor Arbiter a complaint for unfair
labor practice, illegal dismissal, and illegal suspension alleging therein that he was
dismissed without due process of law.

ISSUE:

Whether or not it is the NLRC which has jurisdiction over the complaint for
illegal dismissal which the private respondent had filed with the NLRC.

RULING:

NO.

The removal of Llorente as Managing Director is purely an intra-corporate


dispute which falls within the exclusive jurisdiction of the SEC and not of the NLRC.
In reality, Llorente was not dismissed. If he lost the position of Managing Director, it
was primarily because he was not reelected as Director during the regular
stockholders‘ meeting. The office of Managing Director presupposes that its occupant
is a Director; hence, one who is not a Director of the petitioner or who has ceased to
be a Director cannot be elected or appointed as a Managing Director. The holding of
the position of Director is a prerequisite for the election, appointment, or designation
of Managing Director. If a Managing Director should lose his position because he
ceased to be a Director for any reason, such as non-reelection as in the case of
Liorente, such loss is not dismissal but failure to qualify or to maintain a prerequisite
for that position. Then too, the position of Managing Director was abolished.
Any question relating or incident to the election of the new Board of Directors,
the non-reelection of Liorente as a Director, his loss of the position of Managing
Director, or the abolition of the said office are intra-corporate matters. Disputes
arising therefrom are intra-corporate disputes which, if unresolved within the
corporate structure of the petitioner, may be resolved in an appropriate action only by
the SEC pursuant to its authority under paragraphs (b) and (c), Section 5 of P.D. No.
902-A.

461 | P a g e
Law 321_Corporation LAW_ Case Digest

APODACA, petitioner
vs.
NATIONAL LABOR RELATIONS COMMISSION, respondent
G.R. No. 80039 April 18, 1989

FACTS:

Petitioner was employed in respondent corporation. On August 28, 1985,


respondent Jose M. Mirasol persuaded petitioner to subscribe to 1,500 shares of
respondent corporation at P100.00 per share or a total of P150,000.00. He made an
initial payment of P37,500.00. On September 1, 1975, petitioner was appointed
President and General Manager of the respondent corporation. However, on January
2, 1986, he resigned.
On December 19, 1986, petitioner instituted with the NLRC a complaint against
private respondents for the payment of his unpaid wages, his cost of living allowance,
the balance of his gasoline and representation expenses and his bonus compensation
for 1986. Petitioner and private respondents submitted their position papers to the
labor arbiter. Private respondents admitted that there is due to petitioner the amount
of P17,060.07 but this was applied to the unpaid balance of his subscription in the
amount of P95,439.93. Petitioner questioned the set-off alleging that there was no call
or notice for the payment of the unpaid subscription and that, accordingly, the alleged
obligation is not enforceable.
In a decision dated April 28, 1987, the labor arbiter sustained the claim of
petitioner for P17,060.07 on the ground that the employer has no right to withhold
payment of wages already earned under Article 103 of the Labor Code. Upon the
appeal of the private respondents to public respondent NLRC, the decision of the labor
arbiter was reversed in a decision dated September 18, 1987. The NLRC held that a
stockholder who fails to pay his unpaid subscription on call becomes a debtor of the
corporation and that the set-off of said obligation against the wages and others due to
petitioner is not contrary to law, morals and public policy.

ISSUE:

Whether or not an obligation arising from non-payment of stock subscriptions


to a corporation can be offset against a money claim of an employee against the
employer.

RULING:

NO.

The unpaid subscriptions are not due and payable until a call is made by the
corporation for payment. Private respondents have not presented a resolution of the
board of directors of Respondent Corporation calling for the payment of the unpaid
subscriptions. It does not even appear that a notice of such call has been sent to
petitioner by the respondent corporation.
What the records show is that the respondent corporation deducted the amount
due to petitioner from the amount receivable from him for the unpaid
subscriptions. No doubt such set-off was without lawful basis, if not premature. As
there was no notice or call for the payment of unpaid subscriptions, the same is not
yet due and payable.
Lastly, assuming further that there was a call for payment of the unpaid
subscription, the NLRC cannot validly set it off against the wages and other benefits
due petitioner. Article 113 of the Labor Code allows such a deduction from the wages
of the employees by the employer, only in three instances, to wit: Wage Deduction. —
No employer, in his own behalf or in behalf of any person, shall make any deduction
from the wages of his employees, except for certain instances.

462 | P a g e
Law 321_Corporation LAW_ Case Digest

PSBA
vs.
LEANO
GR L- 58468 February 24, 1984

FACTS:

Tan is one of the stockholders of PSBA. He was a director and Executive Vice-
President enjoying salaries and allowances. During a regular meeting, the Board of
Directors declared all corporate positions vacant except those of the president and
chairman and at the same time elected new set of officers. Tan was not re elected for
which he filed for illegal dismissal before the NLRC. He also instituted a one million
peso damage suit before the Court of First Instance for the illegal and oppressive
removal. He lodged another complaint with the SEC questioning the validity of the
elections and his ouster. The SEC issued a subpoena duces tecum commanding the
production of all corporate documents, records, books. The Labor Arbiter also issued a
subpoena duces tecum for the production of the same records and documents.
Petitioners moved for the dismissal of the complaint before the NLRC invoking the
principle against split jurisdiction.

ISSUE:

Whether or not the NLRC has jurisdiction over the case.

RULING:

NO.

PSBA is a domestic corporation duly organized and existing under our laws.
General management is vested in a Board of seven elected annually by stockholders
entitled to vote, who serve until the election and qualification of their successors. Any
vacancy in the board of directors is filled up by a majority vote of the subscribed
capital stock entitled to vote at a meeting generally called for the purpose, and the
directors so chosen shall hold office for the unexpired term. Corporate officers are
provided for, among them, the Executive Vice-President, who is elected by the board
from their own number. The officers receive such salaries as the board may fix. The
by-laws likewise provide that should the office be rendered vacant by reason of death,
resignation, disqualification or otherwise, the board, by a majority vote may choose a
successor who shall hold office for the unexpired term of the predecessor.
The controversy is intra-corporate in nature. It revolves around the election of
directors, officers and managers of PSBA, the relation between and among its
stockholders, and between them and the corporation. PD 902-A vests in the SEC the
original and exclusive jurisdiction to hear and decide cases involving controversies
arising out of intra-corporate relations between and among stockholders, and between
the stockholders and the corporation. It also has exclusive jurisdiction over
controversies involving the election and appointment of officers, directors, trustees or
managers of such corporation.
The case is not a case of dismissal. The case is that of a corporate office having
been declared vacant and of Tan‘s not having been re-elected thereafter. The matter of
whom to elect is a prerogative that belongs to the Board and involves the exercise of
deliberate choice and the faculty of discriminative selection. Generally speaking, the
relationship of a person to the corporation, whether as an officer or as agent or
employee, is not determined by the services performed but by the incidents of the
relationship as they actually exist.

463 | P a g e
Law 321_Corporation LAW_ Case Digest

PURIFICACION G. TABANG
vs.
NATIONAL LABOR RELATIONS COMMISSION
G.R. No. 121143 January 21, 1997

FACTS:

Purificacion Tabang was a founding member, a member of the Board of


Trustees, and the corporate secretary of private respondent Pamana Golden Care
Medical Center Foundation, Inc., a non-stock corporation engaged in extending
medical and surgical services. Medical Director and Hospital Administrator of private
respondent's Pamana Golden Care Medical Center in Calamba, Laguna. Although the
memorandum was silent as to the amount of remuneration for the position, petitioner
claims that she received a monthly retainer fee of five thousand pesos (P5,000.00)
from private respondent, but the payment thereof was allegedly stopped in November,
1991.
As medical director and hospital administrator, petitioner was tasked to run the
affairs of the aforesaid medical center and perform all acts of administration relative to
its daily operations.Petitioner was allegedly informed personally by Dr. Ernesto Naval
that in a special meeting held on April 30, 1993, the Board of Trustees passed a
resolution relieving her of her position as Medical Director and Hospital Administrator,
and appointing the latter and Dr. Benjamin Donasco as acting Medical Director and
acting Hospital Administrator, respectively. Petitioner averred that she thereafter
received a copy of said board resolution. Petitioner then filed a complaint for illegal
dismissal and non-payment of wages, allowances and 13th month pay before the labor
arbiter but the complaint was dismissed for lack of jurisdiction.

ISSUE:

Whether or not the NLRC has jurisdiction over the case.

RULING:

NO.

It is the SEC which has jurisdiction over the case. The charges against private
respondent partake of the nature of an intra-corporate controversy. Similarly, the
determination of the rights of petitioner and the concomitant liability of private
respondent arising from her ouster as a medical director and/or hospital
administrator, which are corporate offices, is an intra-corporate controversy subject to
the jurisdiction of the SEC. Contrary to the contention of petitioner, a medical director
and a hospital administrator are considered as corporate officers under the by-laws of
respondent corporation.
The president, vice-president, secretary and treasurer are commonly regarded
as the principal or executive officers of a corporation and modern corporation statutes
usually designate them as the officers of the corporation. However, other offices are
sometimes created by the charter or by-laws of a corporation or the board of directors
may be empowered under the by-laws of a corporation to create additional offices as
may be necessary.
In the case at bar, considering that herein petitioner, unlike an ordinary
employee, was appointed by Respondent Corporation‘s Board of Trustees in its
memorandum, she is deemed an officer of the corporation. Section 5(c) of Presidential
Decree No. 902-A, provides that the SEC exercises exclusive jurisdiction over
controversies in the election appointment of directors, trustees, officers or managers of
corporations, partnerships or associations, applies in the present dispute. Accordingly,
jurisdiction over the same is vested in the SEC, and not in the Labor Arbiter or the
NLRC.
A corporate officer's dismissal is always a corporate act, or an intra-corporate
controversy, and the nature is not altered by the reason or wisdom with which the
Board of Directors may have in taking such action.

464 | P a g e
Law 321_Corporation LAW_ Case Digest

UNION MOTOR CORPORATION


vs.
NATIONAL LABOR RELATIONS COMMISSION
G.R. No. 159738 December 9, 2004

FACTS:

Alejandro Etis was hired by the petitioner as an automotive mechanic at the


service department where his latest monthly salary was P6,330.00.During his
employment, he was awarded the Top Technician for the month of May in 1995 and
Technician of the Year (1995).He also became a member of the Exclusive P40,000.00
Club and received the Model Employee Award in the same year. On September 22,
1997, the respondent made a phone call to Rosita dela Cruz, the company nurse, and
informed her that he had to take a sick leave as he had a painful and unbearable
toothache. Finding that the respondents ailment was due to a tooth inflammation, the
doctor referred him to a dentist for further management where he was scheduled to
have a tooth extraction but the date was extended because there was still
inflammation. Upon instructions from the management, Mr. Dumagan, a company
security guard, visited the respondent in his house and confirmed that the latter was
ill. The petitioner issued an Inter Office Memorandum through the manager of its
Human Resources Department, terminating the services of the respondent for having
incurred more than five (5) consecutive absences without proper notification. The
petitioner considered the consecutive absences of the respondent as abandonment of
office under Section 6.1.1, Article III of the Company Rules. On October 4, 1997, Dr.
Pamor successfully extracted the respondents tooth. As soon as he had recovered, the
respondent reported for work, but was denied entry into the company‘s premises. He
was also informed that his employment had already been terminated.

ISSUE:

Whether or not private respondent was illegally dismissed.

RULING:

YES

The company rules do not require that the notice of an employee‘s absence and
the reasons therefore be in writing and for such notice to be given to any specific office
and/or employee of the petitioner. Hence, the notice may be verbal; it is enough then
that an officer or employee of the petitioner, competent and responsible enough to
receive such notice for and in behalf of the petitioner, was informed of such absence
and the corresponding reason. The evidence on record shows that the respondent
informed the petitioner of his illness through the company nurse. The security guard
who was dispatched by the petitioner to verify the information received by the
company nurse, confirmed the respondents illness. Respondent complied with the
requisite of giving notice of his illness and the reason for his absences to the
petitioner.
From these disquisitions, it is clear that the absences of private respondent are
justifiable. The petitioner, likewise, failed to prove the factual basis for its dismissal of
the respondent on the ground of gross and habitual negligence under Article 282(b) of
the Labor Code of the Philippines, or even under Section 6.1.1, Rule III of the
Company Rules. Dismissal is the ultimate penalty that can be meted to an employee.
Thus, it must be based on just cause and must be supported by clear and convincing
evidence. To effect a valid dismissal, the law requires not only that there be just and
valid cause for termination; it, likewise, enjoins the employer to afford the employee
the opportunity to be heard and to defend himself.

465 | P a g e
Law 321_Corporation LAW_ Case Digest

PETITIONS FOR DECLARATION IN THE STATE OF SUSPENSION OF PAYMENTS

G.R. NOS. 174457-59


EXPRESS INVESTMENTS III PRIVATE LTD. AND EXPORT DEVELOPMENT
CANADA
vs.
DAYAN TELECOMMUNICATIONS, INC., THE BANK OF NEW YORK (AS TRUSTEE
FOR THE HOLDERS OF THE US$200,000,000 13.5% SENIOR NOTES OF DAYAN
TELECOMMUNICATIONS, INC.) AND ATTY. REMIGIO A. NOVAL (AS THE COURT-
APPOINTED REHABILITATION RECEIVER OF BAYANTEL)

FACTS:

Respondent Bayantel is a duly organized domestic corporation engaged in the


business of providing telecommunication services. It is 98.6% owned by Bayan
Telecommunications Holdings Corporation (BTHC), which in turn is 85.4% owned by
the Lopez Group of Companies and Benpres Holdings Corporation.
On various dates between the years 1995 and 2001, Bayantel entered into
several credit agreements. In July 1999, Bayantel issued US$200 million worth of
13.5% Senior Notes pursuant to an Indenture dated July 22, 1999 that it entered into
with The Bank of New York as trustee for the holders of said notes.

ISSUES:

Whether or not the claims of secured and unsecured creditors should be


treated pari passu during rehabilitation.

RULING:

YES.

As between the creditors, the key phrase is "equality is equity." When a


corporation threatened by bankruptcy is taken over by a receiver, all the creditors
should stand on equal footing. Not anyone of them should be given any preference by
paying one or some of them ahead of the others. This is precisely the reason for the
suspension of all pending claims against the corporation under receivership. Instead
of creditors vexing the courts with suits against the distressed firm, they are directed
to file their claims with the receiver who is a duly appointed officer of the SEC.
Since then, the principle of equality in equity has been cited as the basis for
placing secured and unsecured creditors in equal footing or in pari passu with each
other during rehabilitation. In legal parlance, pari passu is used especially of creditors
who, in marshaling assets, are entitled to receive out of the same fund without any
precedence over each other.

466 | P a g e
Law 321_Corporation LAW_ Case Digest

ADVENT CAPITAL AND FINANCE CORPORATION


vs.
NICASIO I. ALCANTARA and EDITHA I. ALCANTARA
G.R. No. 183050 January 25, 2012

FACTS:
Petitioner Advent Capital and Finance Corporation (Advent Capital) filed a
petition for rehabilitation withthe Regional Trial Court (RTC) of Makati City and the
RTC named Atty. Danilo L. Concepcion as rehabilitation receiver. Upon audit of Advent
Capital‘s books, Atty. Concepcion found that respondents Nicasio and Editha
Alcantara (collectively, the Alcantaras) owed Advent Capital representing trust feesthat
it supposedly earned for managing their several trust accounts. Then, Atty.
Concepcion requested Belson Securities, Inc. (Belson) to deliver to him, as Advent
Capital‘s rehabilitation receiver, the cash dividends that Belson held under the
Alcantaras‘ Trust Account. Belson refused, however, citing the Alcantara‘ objections as
well as the absence of an appropriate order from the rehabilitation Court.
Thus, Atty. Concepcion filed a motion before the rehabilitation court to direct
Belson to release the money to him and thereafter the rehabilitation court granted
Atty. Concepcion‘s motion and in compliance to the order, Belson turned over the
subject dividends to him.Thereafter, the Alcantaras filed a special civil action of
certiorari before the Court of Appeals (CA), seeking to annul the rehabilitation court‘s
order and the CA granted the petition and directing Atty. Concepcion to account for
the dividends and deliver them to the Alcantaras.

ISSUE:

Whether or not the cash dividends held by Belson and claimed by both the
petitioner and therespondents, could be claimed by the Advent Capital upon the order
of the rehabilitation court.

RULING:

NO.

The rehabilitation court has no jurisdiction to hear and adjudicate the


conflicting claims of theparties over the dividends that Belson held in trust for their
owners. The rehabilitation court has notbeen given the power to resolve ownership
disputes between Advent Capital and third parties. Neither Belson nor the Alcantaras
are its debtors or creditors with interest in the rehabilitation.
Advent Capital must file a separate action for collection to recover the trust fees
that it allegedly earned and, with the trial court‘s authorization if warranted, put the
money in escrow for payment towhoever it rightly belongs. Having failed to collect the
trust fees at the end of each calendar quarter asstated in the contract, all it had
against the Alcantaras was a claim for payment which is a proper subject for an
ordinary action for collection. It cannot enforce its money claim by simply filing a
motionin the rehabilitation case for delivery of money belonging to the Alcantaras but
in the possession of athird party.
Rehabilitation proceedings are summary and non-adversarial in nature, and do
not contemplateadjudication of claims that must be threshed out in ordinary court
proceedings. Adversarial proceedingssimilar to that in ordinary courts are inconsistent
with the commercial nature of a rehabilitation case.The latter must be resolved quickly
and expeditiously for the sake of the corporate debtor, its creditorsand other interested
parties. Thus, the "incorporate the concept of prohibited pleadings,affidavit evidence in
lieu of oral testimony, clarificatory hearings instead of the traditional approach
of receiving evidence, and the grant of authority to the court to decide the case, or any
incident, on thebasis of affidavits and documentary evidence."
Here, Advent Capital‘s claim is disputed and requires a full trial on the merits.
It must be resolvedin a separate action where the Alcantaras‘ claim and defenses may
also be presented and heard.

467 | P a g e
Law 321_Corporation LAW_ Case Digest

SIOCHI FISHERY ENTERPRISES, INC., JUN-JUN FISHING CORPORATION, DEDE


FISHING CORPORATION, BLUE CREST AQUA-FARMS, INC., and ILOILO
PROPERTY VENTURES, INC.,
vs.
BANK OF THE PHILIPPINE ISLANDS
G.R. No. 193872 October 19, 2011

FACTS:

Petitioners Siochi Fishery Enterprises, Inc., Jun-Jun Fishing Corporation, Dede


Fishing Corporation, Blue Crest Aqua-Farms, Inc. and Iloilo Property Ventures, Inc.
(petitioners) are domestic corporations of the Siochi family. Petitioners are engaged in
various businesses and have interlocking stockholders and directors. Their principal
office is located at 31 Don B. Bautista Boulevard, Dampalit, Malabon City.
In the course of their business, petitioners borrowed from respondent Bank of
the Philippine Islands (BPI) and from Ayala Life Assurance, Inc. As of 30 June 2004,
petitioners‘ total obligation amounted to P85,362,262.05.
On 15 July 2004, petitioners filed with the RTC a petition for corporate
rehabilitation. Petitioners prayed that the RTC (1) issue a stay order; (2) declare
petitioners in a state of suspension of payments; (3) approve petitioners‘ proposed
rehabilitation plan; and (4) appoint a rehabilitation receiver.

ISSUE:

Whether or not the Court of Appeals erred in setting aside the RTC‘s Order
because it is within the RTC‘s discretion to disregard the procedural formalities, and
the lower court has factual basis in its finding that [petitioners] are capable of
rehabilitation.

RULING:

NO.

The rehabilitation plan is an indispensable requirement in corporate


rehabilitation proceedings. Section 5 of the Rules enumerates the essential requisites
of a rehabilitation plan: The rehabilitation plan shall include (a) the desired business
targets or goals and the duration and coverage of the rehabilitation; (b) the terms and
conditions of such rehabilitation which shall include the manner of its
implementation, giving due regard to the interests of secured creditors; (c) the material
financial commitments to support the rehabilitation plan; (d) the means for the
execution of the rehabilitation plan, which may include conversion of the debts or any
portion thereof to equity, restructuring of the debts, dacion en pago, or sale of assets
or of the controlling interest; (e) a liquidation analysis that estimates the proportion of
the claims that the creditors and shareholders would receive if the debtor‘s properties
were liquidated; and (f) such other relevant information to enable a reasonable
investor to make an informed decision on the feasibility of the rehabilitation plan.
With respect to the Appraisal Report, it bears to stress that the same was
commissioned by respondent corporations and petitioner was not afforded the
opportunity to contest the same. Also, it is extant from the records that some of the
properties included therein do not belong to respondent corporations but to their
officers, namely, Ferdinand Siochi, Mario Siochi, Jr., Gerald Siochi and Jose Patrick
Siochi. Thus, these properties should not be considered as part of respondent
corporations‘ assets as their officers have a separate personality from the corporation
itself. In turn, this renders doubtful their declaration in their Rehabilitation Plan that
they have "sufficient collaterals to back-up their bank loans.

468 | P a g e
Law 321_Corporation LAW_ Case Digest

JOSE MARCEL PANLILIO, ERLINDA PANLILIO, NICOLE MORRIS and MARIO T.


CRISTOBAL
vs.
REGIONAL TRIAL COURT, BRANCH 51, CITY OF MANILA, represented by HON.
PRESIDING JUDGE ANTONIO M. ROSALES; PEOPLE OF THE PHILIPPINES; and
the SOCIAL SECURITY SYSTEM
G.R. No. 173846. February 2, 2011

FACTS:

On October 15, 2004, Jose Marcel Panlilio, Erlinda Panlilio, Nicole Morris and
Marlo Cristobal (petitioners), as corporate officers of Silahis International Hotel, Inc.
(SIHI), filed with the Regional Trial Court (RTC) of Manila, Branch 24, a petition for
Suspension of Payments and Rehabilitation4 in SEC Corp. Case No. 04-111180.
On October 18, 2004, the RTC of Manila, Branch 24, issued an Orderstaying all
claims against SIHI upon finding the petition sufficient in form and substance.
The Court shares the view of the private complainants and the SSS that the
said stay order does not include the prosecution of criminal offenses. Precisely, the law
"criminalizes" the non-remittance of SSS contributions by an employer to protect the
employees from unscrupulous employers. Clearly, in these cases, public interest
requires that the said criminal acts be immediately investigated and prosecuted for the
protection of society.

ISSUE:

Whether or not the suspension of "all claims" as an incident to a corporate


rehabilitation also contemplate the suspension of criminal charges filed against the
corporate officers of the distressed corporation.

RULING:

NO.

The prosecution of the officers of the corporation has no bearing on the pending
rehabilitation of the corporation, especially since they are charged in their individual
capacities. Such being the case, the purpose of the law for the issuance of the stay
order is not compromised, since the appointed rehabilitation receiver can still fully
discharge his functions as mandated by law. It bears to stress that the rehabilitation
receiver is not charged to defend the officers of the corporation. If there is anything
that the rehabilitation receiver might be remotely interested in is whether the court
also rules that petitioners are civilly liable. Such a scenario, however, is not a reason
to suspend the criminal proceedings, because as aptly discussed in Rosario, should
the court prosecuting the officers of the corporation find that an award or
indemnification is warranted, such award would fall under the category of claims, the
execution of which would be subject to the stay order issued by the rehabilitation
court. The penal sanctions as a consequence of violation of the SSS law, in relation to
the revised penal code can therefore be implemented if petitioners are found guilty
after trial.
However, any civil indemnity awarded as a result of their conviction would be
subject to the stay order issued by the rehabilitation court. Only to this extent can the
order of suspension be considered obligatory upon any court, tribunal, branch or body
where there are pending actions for claims against the distressed corporation.

469 | P a g e
Law 321_Corporation LAW_ Case Digest

RICARDO V. CASTILLO
vs.
UNIWIDE WAREHOUSE CLUB, INC
G.R. No. 169725 April 30, 2010

FACTS:

Petitioner filed for illegal dismissal against respondents Uniwide Warehouse


Club, Inc. and its president, Jimmy N. Gow. The complaint contained a prayer for the
payment of worked Saturdays for the year 2001; holiday pay; separation pay; actual,
moral and exemplary damages; and attorney's fees. However, almost two months from
the filing of the Complaint, respondents submitted a Motion to Suspend Proceedings
on the ground that in June 1999, the Uniwide Group of Companies had petitioned the
Securities and Exchange Commission for suspension of payments and for approval of
its proposed rehabilitation plan. It appears that on June 29, 1999, the SEC had ruled
favorably on the petition and ordered that all claims, actions and proceedings against
herein respondents pending before any court, tribunal, board, office, body or
commission be suspended, and that following the appointment of an interim receiver,
the suspension order had been extended to until February 7, 2000. On April 11, 2000,
the SEC declared the Uniwide Group of Companies to be in a state of suspension of
payments and approved its rehabilitation plan. Labor Arbiter Lilia S. Savari denied the
Motion to Suspend Proceedings in the present case. Respondents lodged an appeal
with the NLRC which sustained the Labor Arbiter and held that as early as February
7, 2000 the suspension order of the SEC should be considered lifted already and that
with the approval of the rehabilitation plan, the suspension of the proceedings in the
instant labor case would no longer be necessary.

ISSUE:

Whether or not the illegal dismissal case proceedings should be suspended.

RULING:

NO.

Corporate rehabilitation connotes the restoration of the debtor to a position of


successful operation and solvency, if it is shown that its continued operation is
economically feasible and its creditors can recover by way of the present value of
payments projected in the rehabilitation plan, more if the corporation continues as a
going concern than if it is immediately liquidated. It contemplates a continuance of
corporate life and activities in an effort to restore and reinstate the corporation to its
former position of successful operation and solvency, the purpose being to enable the
company to gain a new lease on life and allow its creditors to be paid their claims out
of its earnings.
An essential function of corporate rehabilitation is the mechanism of
suspension of all actions and claims against the distressed corporation, which
operates upon the due appointment of a management committee or rehabilitation
receiver. Jurisprudence is settled that the suspension of proceedings referred to in the
law uniformly applies to "all actions for claims" filed against a corporation, partnership
or association under management or receivership, without distinction, except only
those expenses incurred in the ordinary course of business. The reason behind the
imperative nature of a suspension or stay order in relation to the creditors' claims
cannot be downplayed, for indeed the indiscriminate suspension of actions for claims
intends to expedite the rehabilitation of the distressed corporation by enabling the
management committee or the rehabilitation receiver to effectively exercise its/his
powers free from any judicial or extrajudicial interference that might unduly hinder or
prevent the rescue of the debtor company. To allow such other actions to continue
would only add to the burden of the management committee or rehabilitation receiver,
whose time, effort and resources would be wasted in defending claims against the
corporation, instead of being directed toward its restructuring and rehabilitation.

470 | P a g e
Law 321_Corporation LAW_ Case Digest

PACIFIC WIDE REALTY AND DEVELOPMENT CORPORATION


vs.
PUERTO AZUL LAND, INC.
G.R. No. 178768. November 25, 2009

FACTS:

Puerto Azul Land, Inc. (PALI) is the owner and developer of the Puerto Azul
Complex situated in Ternate, Cavite. Its business involves the development of Puerto
Azul into a satellite city with residential areas, resort, tourism and retail commercial
centers with recreational areas. In order to finance its operations, it obtained loans
from various banks, the principal amount of which amounted to Six Hundred Forty
Million Two Hundred Twenty-Five Thousand Three Hundred Twenty-Four Pesos
(P640,225,324.00). PALI and its accommodation mortgagors, i.e., Ternate
Development Corporation (TDC), Ternate Utilities, Inc. (TUI), and Mrs. Trinidad Diaz-
Enriquez, secured the loans.
In the beginning, PALI‘s business did very well. However, it started
encountering problems when the Philippine Stock Exchange rejected the listing of its
shares in its initial public offering which sent a bad signal to the real estate market.
This resulted in potential investors and real estate buyers shying away from the
business venture. The situation was aggravated by the 1997 Asian financial crisis and
the decline of the real estate market. Consequently, PALI was unable to keep up with
the payment of its obligations, both current and those that were about to fall due. One
of its creditors, the Export and Industry Bank (EIB), later substituted by Pacific Wide
Realty and Development Corporation (PWRDC), filed foreclosure proceedings on PALI‘s
mortgaged properties. Thrust to a corner, PALI filed a petition for suspension of
payments and rehabilitation, accompanied by a proposed rehabilitation plan and three
(3) nominees for the appointment of a rehabilitation receiver.
On December 13, 2005, the RTC rendered a Decision approving PALI‘s petition
for suspension of payments and rehabilitation.

ISSUE:

Whether or not the terms of the rehabilitation plan are unreasonable and in
violation of the non-impairment clause.

RULING:

NO.

An indispensable requirement in the rehabilitation of a distressed corporation is the


rehabilitation plan, and Section 5 of the Interim Rules of Procedure on Corporate
Rehabilitation provides the requisites thereof. The Court finds nothing onerous in the
terms of PALI‘s rehabilitation plan. The Interim Rules on Corporate Rehabilitation
provides for means of execution of the rehabilitation plan, which may include, among
others, the conversion of the debts or any portion thereof to equity, restructuring of
the debts, dacion en pago, or sale of assets or of the controlling interest.
The restructuring of the debts of PALI is part and parcel of its rehabilitation.
Moreover, per findings of fact of the RTC and as affirmed by the CA, the restructuring
of the debts of PALI would not be prejudicial to the interest of PWRDC as a secured
creditor. There is nothing unreasonable or onerous about the 50% reduction of the
principal amount when, as found by the court a quo, a Special Purpose Vehicle (SPV)
acquired the credits of PALI from its creditors at deep discounts of as much as 85%.
Meaning, PALI‘s creditors accepted only 15% of their credit‘s value. Stated otherwise, if
PALI‘s creditors are in a position to accept 15% of their credit‘s value, with more
reason that they should be able to accept 50% thereof as full settlement by their
debtor. They also find no merit in PWRDC‘s contention that there is a violation of the
impairment clause. Section 10, Article III of the Constitution mandates that no law
impairing the obligations of contract shall be passed. This case does not involve a law
or an executive issuance declaring the modification of the contract among debtor PALI.
or have opposed the plan or whether or not their claims have been scheduled.

471 | P a g e
Law 321_Corporation LAW_ Case Digest

PHILIPPINE NATIONAL BANK and EQUITABLE PCI BANK


vs.
COURT OF APPEALS
G.R. No. 165571 January 20, 2009

FACTS:

Petitioners are members of the consortium of creditor banks acting as trustee


for the consortium. Private respondents filed with the SEC a verified petition for
rehabilitation with prayer for suspension of actions and proceedings pending
rehabilitation stating that they possess sufficient properties to cover their obligations
but foresee inability to pay them within a period of one year. They cited the sudden
non-renewal and/or massive withdrawal by creditors of their loans to ASB Holdings,
the glut in the real estate market, severe drop in the sale of real properties, peso
devaluation, and decreased investor confidence in the economy which resulted in the
non-completion of and failure to sell their projects and default in the servicing of their
credits as they fell due. The ASB Group had assets worth PhP 19,410,000,000 and
liabilities worth PhP 12,700,000,000.
Faced with at least 712 creditors, 317 contractors/suppliers, and 492
condominium unit buyers, and the prospect of having secured and non-secured
creditors press for payments and threaten to initiate foreclosure proceedings, the ASB
Group pleaded for suspension of payments while working for rehabilitation with the
help of the SEC. Finding the petition sufficient in form and substance, the SEC
Hearing Panel issued an order suspending for 60 days all actions for claims against
the ASB Group, enjoining the latter from disposing its properties in any manner
except in the ordinary course of business and from paying outstanding liabilities, and
appointing Atty. Monico V. Jacob as interim receiver of the ASB Group. Atty. Jacob
was later replaced by Atty. Fortunato Cruz as interim receiver. The consortium of
creditor banks, which included petitioners, filed their Opposition praying for the
dismissal of the petition.

ISSUE:

Whether or not the rehabilitation proceedings is merely a temporary suspension


of payments of obligations falling due by the distressed corporation and not
cancellation or repudiation of those contractual obligations.

RULING:

NO.

The mere fact that the ASB Group averred that it has sufficient assets to cover
its obligations does not make it "solvent" enough to prevent it from filing a petition for
rehabilitation. A corporation may have considerable assets but if it foresees the
impossibility of meeting its obligations for more than one year, it is considered as
technically insolvent. The period referred to the corporation‘s inability to pay its
obligations; when such inability extends beyond one year, the corporation is
considered technically insolvent. Said inability may be established from the start by
way of a petition for rehabilitation, or it may be proved during the proceedings for
suspension of payments, if the latter was the first remedy chosen by the ailing
corporation. If the corporation opts for a direct petition for rehabilitation on the
ground of technical insolvency, it should show in its petition and later prove during
the proceedings that it will not be able to meet its obligations for longer than one year
from the filing of the petition.
Appointment of an interim receiver becomes automatic. By that statutory
provision, it is clear that the approval of the Rehabilitation Plan and the appointment
of a rehabilitation receiver merely suspend the actions for claims against respondent
corporations. Petitioner bank‘s preferred status over the unsecured creditors relative
to the mortgage liens is retained, but the enforcement of such preference is
suspended.

472 | P a g e
Law 321_Corporation LAW_ Case Digest

PRYCE CORPORATION
vs.
COURT OF APPEALS
G.R. No. 172302 February 04, 2008

FACTS:

Pryce Corporation has its primary purpose to develop real estate in Mindanao.
It engaged in the development of memorial parks, operated a major hotel in Cagayan
de Oro City, and produced industrial gases. Asian financial crisis, however, badly
affected petitioner‘s operations, resulting in heavy losses. It could not meet its
obligations as they became due. It incurred losses of P943.09 million in 2001, P479.05
million in 2002, and P125.86 million in 2003. Thus petitioner filed a petition for
rehabilitation where it prayed for the appointment of a Rehabilitation Receiver from
among the nominees named therein and the staying of the enforcement of all claims,
monetary or otherwise against it. Petitioner also prayed that after due hearing, its
proposed Rehabilitation Plan be approved. Some of the proposed rehabilitation plans
were that the bank creditors will be paid through dacion en pago of assets already
mortgaged to them, in case of insufficiency, the deficiency shall be settled by way of
dacion of memorial park lots owned by the petitioner, and that all penalties shall be
waived by the creditors. The creditors opposed the petition.

ISSUE:

Whether or not the petition for rehabilitation of petitioner Pryce Corporation be


granted.

RULING:

YES.

Section 6 of the Interim Rules of Procedure on Corporate Rehabilitation


provides among others that if the court finds the petition to be sufficient in form and
substance, it shall, not later than five days from the filing of the petition, issue an
Order fixing the initial hearing on the petition not earlier than forty five days but not
later than sixty days from the filing thereof; directing all creditors and all interested
parties to file and serve on the debtor a verified comment on or opposition to the
petition, with supporting affidavits and documents, not later than ten days before the
date of the initial hearing and putting them on notice that their failure to do so will bar
them from participating in the proceedings; and directing the creditors and interested
parties to secure from the court copies of the petition and its annexes within such
time as to enable themselves to file their comment on or opposition to the petition and
to prepare for the initial hearing of the petition.
In the case at bench, when the commercial court appointed a rehabilitation
receiver, the very next day after the filing of the Petition for Rehabilitation, it is highly
doubtful and well-nigh impossible, that, without any hearing yet held, the commercial
court could have already gathered enough evidence before it to determine whether
there was any imminent danger of dissipation of assets or of paralization of business
operations to warrant the appointment of a rehabilitation receiver.In determining
whether petitioner‘s financial situation is serious and whether there is a clear and
imminent danger that it will lose its corporate assets, the RTC, acting as commercial
court, should conduct a hearing wherein both parties can present their respective
evidence. Hence, a remand of the records of this case to the RTC is imperative.

473 | P a g e
Law 321_Corporation LAW_ Case Digest

UNIWIDE HOLDINGS INC.


vs.
JANDECS TRANSPORTATION
GR 168522 DECEMBER 19, 2007

FACTS:

Petitioner and respondent Jandecs Transportation Co., Inc. entered into a


contract of Assignment of Leasehold Rights under which the latter was to operate food
and snack stalls at petitioner's Uniwide Coastal Mall in Parañaque City. The contract
was for a period of 18 years, commencing October 1, 1997 up to September 30, 2015,
for a consideration of P2,460,630.15. The parties also agreed that respondent's stalls
would be located near the movie houses and would be the only stalls to sell food and
beverages in that area.Respondent paid the contract price in full. Petitioner, however,
failed to turn over the stall units as agreed upon. Respondent sought the rescission of
the contract and the refund of its payment. Petitioner refused both.Respondent filed a
complaint for breach of contract, rescission of contract, damages and issuance of a
writ of preliminary attachment. Petitioner filed for the suspension of proceedings
praying that the action in Court be held in abeyance in view of the SEC's order of
suspension of payments and approval of its rehabilitation plan.

ISSUE:

Whether or not the proceedings be suspended.

RULING:

NO.

The reason for suspending actions for claims against the corporation is not
really to enable the management committee or the rehabilitation receiver to substitute
the corporation in any pending action against it before any court, tribunal, board or
body but to enable the management committee or the rehabilitation receiver to
effectively exercise its powers free from any judicial or extra-judicial interference that
might unduly hinder or prevent the ―rescue‖ of the debtor corporation. To allow such
other action to continue would only add to the burden of the management committee
or rehabilitation receiver, whose time, effort and resources would be wasted in
defending claims against the corporation instead of being directed toward its
restructuring and rehabilitation.
The power to rescind obligations is implied in reciprocal ones, in case one of the
obligors should not comply with what is incumbent upon him. The injured party may
choose between the fulfillment and the rescission of the obligation, with the payment
of damages in either case. He may also seek rescission, even after he had chosen
fulfillment, if the latter should become impossible.
Certainly, petitioner's failure to deliver the units on the commencement date of
the lease on October 1, 1997 gave respondent the right to rescind the contract after
the latter had already paid the contract price in full. Furthermore, respondent's right
to rescind the contract cannot be prevented by the fact that petitioner had the option
to substitute the stalls. Even if petitioner had that option, it did not, however, mean
that it could insist on the continuance of the contract by forcing respondent to accept
the substitution. Neither did it mean that its previous default had been obliterated
completely by the exercise of that option.

474 | P a g e
Law 321_Corporation LAW_ Case Digest

BANK OF THE PHILIPPINE ISLANDS


vs.
SECURITIES OF EXCHANGE COMMISSION
G.R. No. 164641 December 20, 2007

FACTS:

The Bank of the Philippine Islands through its predecessor-in- interest, Far
East Bank and Trust Company extended credit accommodations to the ASB Group
with an outstanding aggregate principal amount of P86,800,000.00, secured by a real
estate mortgage over two properties. The ASB Group filed a petition for rehabilitation
and suspension of payments before the SEC. The Rehabilitation Plan provides, among
others, a dacion en pagoby the ASB Group to BPI of one of the properties mortgaged to
the latter at the ASB Group as selling value of P84,000,000.00 against the total
amount of the ASB Group‘s exposure to the bank. In turn, ASB Group would require
the release of the other property mortgaged to BPI, to be thereafter placed in the asset
pool. BPI opposed the Rehabilitation Plan and moved for the dismissal of the ASB
Group‘s petition for rehabilitation as it violates its rights as creditor. However, the SEC
hearing panel issued an order approving ASB Group‘s proposed rehabilitation plan
and appointed Mr. Fortunato Cruz as rehabilitation receiver.

ISSUE:

Whether or not the Rehabilitation plan violates the rights of BPI as creditor.

RULING:

NO.

Rehabilitation proceedings in our jurisdiction, much like the bankruptcy laws of


the United States, have equitable and rehabilitative purposes. On the one hand, they
attempt to provide for the efficient and equitable distribution of an insolvent debtor‘s
remaining assets to its creditors; and on the other, to provide debtors with a "fresh
start" by relieving them of the weight of their outstanding debts and permitting them
to reorganize their affairs. It is to effect a feasible and viable rehabilitation by
preserving a foundering business as going concern, because the assets of a business
are often more valuable when so maintained than they would be when liquidated.
The Court reiterates that the SEC‘s approval of the Rehabilitation Plan did not
impair BPI‘s right to contract. Besides, the mere fact that the Rehabilitation Plan
proposes a dacion en pago approach does not render it defective on the ground of
impairment of the right to contract. Dacion en pago is a special mode of payment
where the debtor offers another thing to the creditor who accepts it as equivalent of
payment of an outstanding debt. The undertaking really partakes in a sense of the
nature of sale, that is, the creditor is really buying the thing or property of the debtor,
the payment for which is to be charged against the debtor‘s debt. As such, the
essential elements of a contract of sale, namely; consent, object certain, and cause or
consideration must be present. Being a form of contract, the dacion en pago
agreement cannot be perfected without the consent of the parties involved.
Thus, if BPI does not find the dacion en pagomodality acceptable, the ASB
Group can propose to settle its debts at such amount as is equivalent to the selling
price of the mortgaged properties. If BPI still refuses this option, it can assert its rights
in the liquidation and distribution of the ASB Group‘s assets. It will not lose its status
as a secured creditor, retaining its preference over the other creditors.

475 | P a g e
Law 321_Corporation LAW_ Case Digest

PHILIPPINE AIRLINES
vs.
HEIRS of ZAMORA
G.R. No. 164267 November 23, 2007

FACTS:

Zamora was a cargo representative assigned at the International Cargo


Operations - Import Operations Division of petitioner Philippine Airlines, Inc. Zamora
received a Memorandum informing him of his temporary transfer to the Domestic
Cargo Operations (DCO) Zamora refused to follow the directive because: first, there
was no valid and legal reason for his transfer; second, the transfer violated the
collective bargaining agreement between the management and the employees union
that no employee shall be transferred without just and proper cause; and third, the
transfer did not comply with the 15-day prior notice rule. The transfer was for the
purpose of diffusing the tension between him and his immediate superior. The
management issued several directives informing Zamora of his transfer. However,
Zamora refused to receive these and continued reporting to the ICO-
IOD. Consequently, he was reported absent at the DCO. His salaries were
subsequently withheld. He also ignored the management‘s directive requiring him to
explain in writing his continued absence. He was informed of his termination due to
Insubordination/Neglect of Customer, Disrespect to Authority, and AWOL.

ISSUES:

Whether or not the proceedings should be suspended for the rehabilitation of


the petitioner.

RULING:

YES.

Petitioner had been placed by the Securities and Exchange Commission under a
Permanent Rehabilitation Receiver. Such being the case, a suspension of all actions
for claims against petitioner pending before any court, tribunal or board was, ipso
jure, in order. The suspension of all actions for claims against a corporation embraces
all phases of the suit, be it before the trial court or any tribunal or before this Court.
No other action may be taken, including the rendition of judgment during the state of
suspension. It must be stressed that what are automatically stayed or suspended are
the proceedings of a suit and not just the payment of claims during the execution
stage after the case had become final and executory. Once the process of
rehabilitation, however, is completed, this Court will proceed to complete the
proceedings on the suspended actions.
Furthermore, the actions that are suspended cover all claims against the
corporation whether for damages founded on a breach of contract of carriage, labor
cases, collection suits or any other claims of a pecuniary nature. No exception in favor
of labor claims is mentioned in the law.

476 | P a g e
Law 321_Corporation LAW_ Case Digest

ALEMAR'S SIBAL & SONS, INC.


vs.
JESUS M. ELBINIAS
G.R. No. 75414 June 4, 1990

FACTS:

Private respondent G.A. Yupangco and Co. Inc. filed an action for collection of
a sum of money with prayer for damages and preliminary attachment against Alemar's
Bookstore, a business entity owned and managed by petitioner Alemar's Sibal & Sons,
Inc. Subsequently Ledesma, Saludo and Associates, as intervenor-movant, filed an
omnibus motion informing the respondent trial court that the petitioner Alemar's has
been placed under rehabilitation receivership by the Securities and Exchange
Commission and that movant has been appointed as its receiver. In its opposition,
G.A. Yupangco maintained that it received notice of the receivership only on January
10, 1985 or after one month after the collection suit. It further averred that the motion
to intervene by the receiver was not seasonably made. G.A. Yupangco urged the
issuance of a writ of execution to implement the default judgment which had become
final and executory, there being no motion for reconsideration or appeal. The
corresponding writ was issued on January. Petitioner Alemar's moved for the
discharge of the writ on the ground that its issuance was improper since the
proceedings have been suspended pursuant to the court order.

ISSUE:

Whether or not respondent court can validly proceed with the execution of a
final decision for the payment of a sum of money despite the fact that the judgment
debtor has been placed under receivership.

RULING:

YES.

It is the general rule that once a decision becomes final and executory, its
enforcement becomes the ministerial duty of the court. Equally settled is that the rule
admits of certain exceptions, one of which is where it becomes imperative in the higher
interest of justice to direct the deferment of execution. In the instant case, the stay of
execution is warranted by the fact that petitioner has been placed under rehabilitation
receivership. It must be stressed that the SEC had earlier ordered the suspension of
all actions for claims against Alemar's in order that all the assets of said petitioner
could be inventoried and kept intact for the purpose of ascertaining an equitable
scheme of distribution among its creditors.
During rehabilitation receivership, the assets are held in trust for the equal
benefit of all creditors to preclude one from obtaining an advantage or preference over
another by the expediency of an attachment, execution or otherwise. As between
creditors, the key phrase is equality is equity. When a corporation threatened by
bankruptcy is taken over by a receiver, all the creditors should stand on an equal
footing. Not anyone of them should be given any preference by paying one or some of
them ahead of the others. This is precisely the reason for the suspension of all
pending claims against the corporation under receivership. Instead of creditors vexing
the courts with suits against the distressed firm, they are directed to file their claims
with the receiver who is a duly appointed officer of the SEC.

477 | P a g e
Law 321_Corporation LAW_ Case Digest

BAROTAC SUGAR MILLS, INC.


vs.
COURT OF APPEALS
G.R. No. 123379 July 15, 1997

FACTS:

Private respondent Pittsburgh Trade Center Co., Inc., filed a complaint for a
sum of money against BAROTAC. Instead of filing an answer, BAROTAC filed a Motion
to Suspend Proceedings on the ground that a Petition for Suspension of Payments
With Prayer for the Appointment of a Management or Rehabilitation Committee had
been filed with the Securities and Exchange Commission. This motion met opposition
from PITTSBURGH. The court issued an Order denying petitioner‘s motion ruling that
upon the filing of the petition, the SEC has not yet placed it under receivership. At the
time the Complaint in the instant case was filed with the respondent court, there was
no order yet from the SEC for the appointment of a management or rehabilitation
committee or that which will indicate that petitioner had been placed under
management or receivership.

ISSUE:

Whether or not the mere filing with the SEC of such petition suspends the
proceedings in the RTC.

RULING:

NO.

The appointment of a management committee or rehabilitation receiver may


only take place after the filing with the SEC of an appropriate petition for suspension
of payments. This is clear from a reading of sub-paragraph (d) of Section 5 and sub-
paragraph (d) of Section 6 of P.D. No. 902-A, as amended by P.D. Nos. 1653 and 1758.
The conclusion then is inevitable that pursuant to the underscored proviso in
sub-paragraph (c) of the aforementioned Section 6, taken together with sub-paragraph
(d) of Section 5 and sub-paragraph (d) of Section 6, a court is ipsojure suspended only
upon the appointment of a management committee or a rehabilitation receiver. Since
there is no showing at all that a management committee or rehabilitation receiver for
BAROTAC has been appointed by the SEC, suspension of the proceedings before the
RTC of Quezon City is not warranted.

478 | P a g e
Law 321_Corporation LAW_ Case Digest

BF HOMES, INCORPORATED
vs.
COURT OF APPEALS
G.R.No. 77143 October 3, 1990
G.R. No. 76879 October 3, 1990

FACTS:

BF Homes, Inc. is a domestic corporation previously engaged in the business of


developing and selling residential lots and houses and other related realty matters. On
July 19, 1984, BF contracted a loan from Rosalinda R. Roa and Vicente Mendoza in
the amount of P250,000.00 with interest at the rate of 33% per annum payable after
32 days. The obligation was embodied in a promissory note and secured by two post-
dated checks issued by BF in favor of the lenders. On September 25, 1984, BF filed a
Petition for Rehabilitation and for a Declaration in a State of Suspension of Payments
under Sec. 5(d) of P.D. No. 902-A with a prayer that upon the filing of the petition and
in the meantime, all claims against it for any and all accounts or indebtedness be
suspended, but allowing petitioner to continue with its normal operations. It also
asked for the approval of the proposed rehabilitation plan. Roa and Mendoza filed a
complaint against BF for the recovery of the loan of P250,000.00, with interest and
attorney's fees. The complaint also prayed for the issuance of a writ of preliminary
attachment against the properties of BF. The trial court issued the writ against
properties of BF sufficient to satisfy the principal claim in the amount of P257,333.33.
The SEC, finding an urgent need to rehabilitate BF issued an order creating a
management committee and suspending all actions for claims against BF pending
before any court, tribunal or board.

ISSUE:

Whether or not the action for collection be suspended pending the outcome of
the rehabilitation proceedings.

RULING:

YES.

Under Sec. 6(d) of P.D. No. 902-A, the management committee or rehabilitation
receiver is empowered to take custody and control of all existing assets and properties
of such corporations under management; to evaluate the existing assets and liabilities,
earnings and operations of such corporations; to determine the best way to salvage
and protect the interest of investors and creditors; to study, review and evaluate the
feasibility of continuing operations and restructure and rehabilitate such entities if
determined to be feasible by the SEC.
In light of these powers, the reason for suspending actions for claims against
the corporation should not be difficult to discover. It is not really to enable the
management committee or the rehabilitation receiver to substitute the defendant in
any pending action against it before any court, tribunal, board or body. Obviously, the
real justification is to enable the management committee or rehabilitation receiver to
effectively exercise its/his powers free from any judicial or extra-judicial interference
that might unduly hinder or prevent the "rescue" of the debtor company. To allow
such other action to continue would only add to the burden of the management
committee or rehabilitation receiver, whose time, effort and resources would be wasted
in defending claims against the corporation instead of being directed toward its
restructuring and rehabilitation.

479 | P a g e
Law 321_Corporation LAW_ Case Digest

BANK OF THE PHILIPPINE ISLANDS


vs.
COURT OF APPEALS
G.R. No. 97178 January 10, 1994

FACTS:

Petitioner Bank of Philippine filed with the Regional Trial Court a complaint
against respondent Ruby Industrial Corporation for foreclosure of real estate
mortgage. After filing its answer with counterclaim, respondent RUBY submitted to the
trial court a motion for suspension of the proceedings on the ground that the
Securities and Exchange Commission issued an Order placing RUBY under a
rehabilitation plan.Petitioner BPI filed a motion for reopening of the proceedings,
invoking our ruling order of suspension of payments of Philfinance as well as for all
actions or claims against Philfinance could only be applied to claims of unsecured
creditors. Such order can not extend to creditors holding a mortgage, pledge or any
lien on the property unless they give up the property, security or lien in favor of all the
creditors of Philfinance. The trial court denied the motion of BPI on the basis that the
suspension of payment applies to all creditors, whether secured or unsecured, in order
to place them on equal footing.
Petitioner then filed with the Court of Appeals a petition for certiorari and
mandamus to set aside the Orders of 22 August 1990 and 19 October 1990, alleging
grave abuse of discretion on the part of the trial judge in refusing to reopen the case.
In the instant petition, it is alleged that the Court of Appeals has decided a question of
substance not in accord with the applicable decision of this Court and/or sanctioned a
departure by the trial court from the accepted and usual course of judicial proceedings
as to call for the exercise by this Court of its power of supervision.

ISSUE:

Whether or not petitioner, which is a secured creditor of respondent RUBY, may


still judicially enforce its claim against the latter which has already been placed by
SEC under rehabilitation pursuant to Sec. 5 and Sec. 6, pars. (c) and (d), P.D. 902-A.

RULING:

YES.

In the instant case, the action of petitioner for foreclosure of real estate
mortgage had been filed against respondent RUBY and was pending with the trial
court when RUBY was placed by SEC under rehabilitation through the creation of a
management committee pursuant to Sec. 6, par. (d), P.D. 902-A. In its order of 10
August 1984, SEC directed that all actions or claims against RUBY pending before any
court, tribunal, branch or body be deemed suspended. On the basis of this order, the
jurisdiction of this trial court over the case was also considered suspended. As a
result, SEC acquired jurisdiction, which is bolstered by the fact that it had already
appointed a rehabilitation receiver for the distressed corporation and had directed that
all proceedings or claims against it be suspended.
While it is recognized that petitioner is a preferred creditor whose claim is
secured by real estate mortgage on the properties of respondent RUBY, its right to
enforce its claim in court is suspended with the placing by SEC of respondent under
rehabilitation. This rule will enable the management committee or rehabilitation
receiver to effectively exercise his/its power free from any judicial or extrajudicial
interference that might unduly hinder the rescue of the distressed company.

480 | P a g e
Law 321_Corporation LAW_ Case Digest

CHING
vs.
LBP
G.R. No. 73123 September 2, 1991

FACTS:

On September 19, 1980, private respondents Filand Manufacturing and Estate


Development Co., Inc. and Emilio Ching obtained from petitioner Land Bank of the
Philippines a loan in the amount of Ten Million Pesos (P10,000,000.00). Private
respondents having failed to pay the loan on its due date, petitioner instituted a
complaint for recovery thereof. During the pendency of the collection suit, private
respondents filed a petition for declaration of insolvency. Cited as ground therefore
was their inability to pay the various debts and liabilities incurred by them, either
jointly or solidarily or guaranteed by one for the other, in the course of their
businesses, such inability being due to business reserves brought about by the fire on
January 2, 1984 which gutted the old Holiday Plaza Building then owned and
operated by Filand Manufacturing, as well as the economic crisis which gripped the
country following the assassination of former Senator Benigno S. Aquino in 1983.

ISSUE:

Whether or not it is the SEC which has jurisdiction over proceedings for
suspension of payments and voluntary and involuntary insolvency.

RULING:

YES.

Section 5, par. (d) should be construed as vesting upon the SEC original and
exclusive jurisdiction only over petitions to be declared in a state of suspension of
payments. This qualification effectively circumscribes the jurisdiction of the SEC over
insolvent corporations, partnerships and associations, and consequently, over
proceedings for the declaration of insolvency. It demonstrates beyond doubt that
jurisdiction over insolvency proceedings pertains neither in the first instance nor
exclusively to the SEC but only in continuation of or as an incident to the exercise of
its jurisdiction over petitions to be declared in a state of suspension of payments
wherein the petitioning corporation, partnership or association had previously been
placed under a rehabilitation receiver or management committee by the SEC itself.
Viewed differently, where the petition filed is one for declaration of a state of
suspension of payments due to a recognition of the inability to pay one's debts and
liabilities, and where the petitioning corporation either: (a) has sufficient property to
cover all its debts but foresees the impossibility of meeting them when they fall due.
However, if the petitioning corporation has no sufficient assets to cover its liabilities
and is not under a rehabilitation receiver or a management committee created under
P.D. No. 902-A and does not seek merely to have the payments of its debts suspended,
but seeks a declaration of insolvency, as in this case.
As declared by the law itself, these are merely ancillary powers to enable the
SEC to effectively exercise its jurisdiction. These additional ancillary powers can be
exercised only in connection with an action pending before the SEC and therefore had
to be viewed in relation to Section 5 which defines the SEC's original and exclusive
jurisdiction. Section 6 does not enlarge or add to the exclusive and original jurisdiction
of the SEC as particularly enumerated under Section 5 of said Presidential Decree, as
amended. Construing P.D. 902-A, as amended, in relation to Act 1956, the court ruled
that insofar as petitions for declaration of insolvency of private corporations are
concerned, it is the regular court that has exclusive and original jurisdiction thereon.
The SEC may entertain such petitions only as an incident of and in continuation of its
already acquired jurisdiction over petitions to be declared in the state of suspension of
payments in the two cases provided in Section 5 (d) of P.D. 902-A, as amended.

481 | P a g e
Law 321_Corporation LAW_ Case Digest

PHILIPPINE COMMERCIAL INTERNATIONAL BANK


vs.
COURT OF APPEALS
G.R. No. L-76853 April 18, 1989

FACTS:

On March 3, 1981, Philippine Underwriters Finance Corporation executed a


pledge agreement involving certain shares of stocks and bonds in favor of Insular
Bank of Asia and America now PCIB as a security for its outstanding obligation. On
June 18, 1981, the Securities and Exchange Commission placed Philfinance under
suspension of payments upon the directive of the President of the Philippines to
conserve the assets of the Corporation and obtain an equitable payment to all its
creditors. On August 7, 1981, SEC appointed a Receivership Committee to conserve
the assets of Phil-finance and determine the best way to protect the creditors, as well
as make the necessary representations with any court or other body for the
consolidation of all claims against Philfinance which are pending before such court or
body in order to forestall the probability of inequitable disposition/satisfaction of said
claims. Upon the basis of the findings of the Receivership Committee and on its own,
SEC ordered the dissolution and liquidation of Philfinance. From this order, some
concerned parties appealed to this Court and respondent Court of Appeals.
Meanwhile, Philfinance failed to satisfy its outstanding obligation with PCIB
which prompted the latter to post a Notice of Auction Sale of the pledged shares of
stocks and bonds on August 18, 1986 by the other petitioner, Notary Public Melchor
B. Francisco. On August 15, 1986, the Receiver filed a petition for a writ of preliminary
injunction with the Regional Trial Court to stop the aforementioned auction sale which
the trial court denied but which denial was reversed by the CA.

ISSUE:

Whether or not suspension of payments apply only to unsecured creditors.

RULING:

NO.

SEC's order for suspension of payments of Philfinance as well as for all actions
of claims against Philfinance could only be applied to claims of unsecured creditors.
Such order can not extend to creditors holding a mortgage, pledge or any lien on the
property unless they give up the property, security or lien in favor of all the creditors
of Philfinance. The rights of a preferred creditor remain to be respected and recognized
in every existing situation. To hold otherwise would render the said rights inutile and
illusory. Besides, there was no substantial difference between the suspension of
actions in the instant case and that under the Insolvency Law. Consequently, the
herein order of suspension, could not have a different interpretation as regards
secured credits than that already given by this Court. The records show that PCIB
neither surrendered the pledged shares of stock and bonds nor participated in the
proceedings before the SEC regarding the suspension of payments or actions of claims
against Philfinance or in the latter's subsequent dissolution and liquidation. The
pledged properties being still in PCIB's possession, the Receiver could not possess the
same for equitable distribution to the creditors of Philfinance.
The fact that the SEC order for the dissolution and liquidation of Philfinance
has already been upheld by the court had been taken judicial notice of. In view of this
development, it appears that the Rehabilitation Receiver has no more right to enjoin
the auction sale since its prayer for injunctive relief was based on the order for
suspension of payments which was in turn based on the directive of the President of
the Philippines to conserve the assets of the corporation and obtain an equitable
payment to all its creditors.

482 | P a g e
Law 321_Corporation LAW_ Case Digest

RADIOLA-TOSHIBA PHILIPPINES, INC.


vs.
INTERMEDIATE APPELLATE COURT
G.R. No. 75222 July 18, 1991

FACTS:

On July 2, 1980, three creditors filed a petition for the involuntary insolvency of
Carlos Gatmaytan and Teresita Gatmaytan, the private respondents herein. On July 9,
1980, the respondent court issued an order taking cognizance of the said petition and
statingthat the Court forbids the payment of any debts, and the delivery of any
property owing and belonging to said respondents-debtors from other persons, or, to
any other persons for the use and benefit of the same respondents-debtors and/or the
transfer of any property by and for the said respondents-debtors to another, upon
petitioners' putting up a bond by way of certified and reputable sureties. On April 12,
1983, petitioners-creditors filed second urgent motion for issuance of insolvency order
and resolution of the case, alleging among other things, they caused to be
investigated the real properties in the names of Carlos Gatmaytan and Teresita
Gatmaytan and they were surprised to find out that some of the aforesaid properties
were already transferred to Radiola-Toshiba Phil. Inc. Judgment was rendered
declaring the insolvency of respondents-debtors Carlos Gatmaytan and Teresita
Gatmaytan. Petitioner filed a supplemental opposition to the same second urgent
motion and motion to direct respondent sheriff to issue a final certificate of sale for the
properties in its favor. On September 21, 1982, the court ordered the consolidation of
ownership of petitioner over said properties but respondent sheriff of Angeles City
refused to issue a final certificate of sale in favor of petitioner.

ISSUE:

Whether or not the levy on attachment in favor of the petitioner is dissolved by


the insolvency proceedings against respondent spouses commenced four months after
said attachment.

RULING:

NO.

Sec. 32 states that as soon as an assignee is elected or appointed and qualified,


the clerk of the court shall, by an instrument under his hand and seal of the court,
assign and convey to the assignee all the real and personal property, estate, and
effects of the debtor with all his deeds, books, and papers relating thereto, and such
assignment shall relate back to the commencement of the proceedings in insolvency,
and shall relate back to the acts upon the adjudication was founded, and by operation
of law shall vest the title to all such property, estate, and effects in the assignee,
although the same is then attached on process, as the property of the debtor. Such
assignment shall operate to vest in the assignee all of the estate of the insolvent debtor
not exempts by law from execution. The provision is very clear that attachments
dissolved are those levied within one month next preceding the commencement of the
insolvency proceedings and judgments vacated and set aside are judgments entered in
any action, including judgment entered by default or consent of the debtor, where the
action was filed within thirty days immediately prior to the commencement of the
insolvency proceedings.
There is a cut off period of one month in attachment cases and thirty days in
judgments entered in actions commenced prior to the insolvency proceedings. Section
79, on the other hand, relied upon by private respondents provides for the right of the
plaintiff if the attachment is not dissolved before the commencement of proceedings in
insolvency, or is dissolved by an undertaking given by the defendant, if the claim upon
which the attachment suit was commenced is proved against the estate of the debtor.
Therefore, there is no conflict between the two provisions.

483 | P a g e
Law 321_Corporation LAW_ Case Digest

RIZAL COMMERCIAL BANKING CORPORATION


vs.
INTERMEDIATE APPELLATE COURT
G.R. No. 74851 December 9, 1999

FACTS:

On September 28, 1984, BF Homes filed a Petition for Rehabilitation and for
Declaration of Suspension of Payments with the SEC. One of the creditors listed in its
inventory of creditors and liabilities was RCBC. On October 26, 1984, RCBC requested
the Provincial Sheriff of Rizal to extra-judicially foreclose its real estate mortgage on
some properties of BF Homes. A notice of extra-judicial foreclosure sale was issued by
the Sheriff on October 29, 1984, scheduled on November 29, 1984, copies furnished
both BF Homes as mortgagor and RCBC as mortgagee. On motion of BF Homes, the
SEC a temporary restraining order effective for 20 days, enjoining RCBC and the
sheriff from proceeding with the public auction sale. The sale was rescheduled to
January 29, 1985. On January 25, 1985, the SEC ordered the issuance of a writ of
preliminary injunction upon petitioners filing of a bond. However, petitioner did not
file a bond until January 29, 1985, the very day of the auction sale, so no writ of
preliminary injunction was issued by the SEC. Presumably, unaware of the filing of
the bond, the sheriffs proceeded with the public auction sale on January 29, 1985, in
which RCBC was the highest bidder for the properties auctioned. On February 5,
1985, BF Homes filed in the SEC a consolidated motion to annul the auction sale and
to cite RCBC and the sheriff for contempt. RCBC opposed the motion.
Because of the proceedings in the SEC, the sheriff withheld the delivery to
RCBC of a certificate of sale covering the auctioned properties. On February 13, 1985,
the SEC belatedly issued a writ of preliminary injunction stopping the auction sale
which had been conducted by the sheriff two weeks earlier.

ISSUE:

Whether or not preferred creditors of distressed corporations stand on the same


footing with all other creditors.

RULING:

YES.

The issue of whether or not preferred creditors of distressed corporations stand


on equal footing with all other creditors gains relevance and materiality only upon the
appointment of a management committee, rehabilitation receiver, board, or body.
Insofar as petitioner RCBC is concerned, the provisions of Presidential Decree No. 902-
A are not yet applicable and it may still be allowed to assert its preferred status
because it foreclosed on the mortgage prior to the appointment of the management
committee on March 18, 1985. Paragraph (c), Section 6 of Presidential Decree 902-A,
providesthat upon appointment of a management committee, rehabilitation receiver,
board or body, pursuant to this Decree, all actions for claims against corporations,
partnerships or associations under management or receivership pending before any
court, tribunal, board or body shall be suspended accordingly. It is thus adequately
clear that suspension of claims against a corporation under rehabilitation is counted
or figured up only upon the appointment of a management committee or a
rehabilitation receiver.
In other words, once a management committee, rehabilitation receiver, board or
body is appointed pursuant to P.D. 902-A, all actions for claims against a distressed
corporation pending before any court, tribunal, board or body shall be suspended
accordingly. This suspension shall not prejudice or render ineffective the status of a
secured creditor as compared to a totally unsecured creditor. P.D. 902-A does not
state anything to this effect. What it merely provides is that all actions for claims
against the corporation, partnership or association shall be suspended. This should
give the receiver a chance to rehabilitate the corporation if there should still be a
possibility for doing so.

484 | P a g e
Law 321_Corporation LAW_ Case Digest

RUBBERWORLD (PHILS.), INC.


vs.
NATIONAL LABOR RELATIONS COMMISSION
G.R. No. 126773 April 14, 1999

FACTS:

Petitioner is a domestic corporation which used to be in the business of


manufacturing footwear, bags and garments. It filed with the Securities and Exchange
Commission a petition for suspension of payments praying that it be declared in a
state of suspension of payments and that the SEC accordingly issue an order
restraining its creditors from enforcing their claims against petitioner corporation. It
further prayed for the creation of a management committee as well as for the approval
of the proposed rehabilitation plan and memorandum of agreement between Petitioner
Corporation and its creditors. The SEC favorably ruled on the petition for suspension
of payments. Private respondents, who claim to be employees of Petitioner
Corporation, filed against petitioners their respective complaints for illegal dismissal,
unfair labor practice, damages and payment of separation pay, retirement benefits,
13th month pay and service incentive pay. Petitioners moved to suspend the
proceedings in the above labor cases on the strength of the SEC Order.
The Labor Arbiter denied the motion holding that the injunction contained in
the SEC Order applied only to the enforcement of established rights and did not
include the suspension of proceedings involving claims against petitioner which have
yet to be ascertained. The Labor Arbiter further held that the order of the SEC
suspending all actions for claims against petitioners does not cover the claims of
private respondents in the labor cases because said claims and the concomitant
liability of petitioners still had to be determined, thus carrying no dissipation of the
assets of petitioners.

ISSUE:

Whether or not the labor proceedings should be suspended.

RULING:

NO.

The applicable law is PD 902-A, as amended which provides that upon the
appointment by, the SEC of a management committee or a rehabilitation receiver, all
actions for claims against the corporation pending before any court, tribunal or board
shall ipso jurebe suspended. The justification for the automatic stay of all pending
actions for claims is to enable the management committee or the rehabilitation
receiver to effectively exercise its/his powers free from any judicial or extra-judicial
interference that might unduly hinder or prevent the 'rescue' of the debtor company.
To allow such other actions to continue would only add to the burden of the
management committee or rehabilitation receiver, whose time, effort and resources
would be wasted in defending claims against the corporation instead of being directed
toward its restructuring and rehabilitation.
Parenthetically, the rehabilitation of a financially distressed corporation benefits
its employees, creditors, stockholders and, in a larger sense, the general public. And
in considering whether to rehabilitate or not, the SEC gives preference to the interest
of creditors, including employees. The reason that shareholders can recover their
investments only upon liquidation of' the corporation, and only if there are assets
remaining after all corporate creditors are paid. No exception in favor of labor claims is
mentioned in the law. Since the law makes no distinction or exemptions, neither
should the Court. Allowing labor cases to proceed clearly defeats the purpose of the
automatic stay and severely encumbers the management committee's time and
resources. The said committee would need to defend against these suits, to the
detriment of its primary and urgent duty to work towards rehabilitating the
corporation and making it viable again. To rule otherwise would open the floodgates to
other similarly situated claimants and forestall if not defeat the rescue efforts.

485 | P a g e
Law 321_Corporation LAW_ Case Digest

UNION BANK OF THE PHILIPPINES


vs.
HONORABLE COURT OF APPEALS
G.R. No. 131729 May 19, 1998

FACTS:

Private respondents EYCO Group of Companies filed with the SEC a Petition for
the Declaration of Suspension of Payment, Formation and Appointment of
Rehabilitation Receiver/Committee, Approval of Rehabilitation Plan with Alternative
Prayer for Liquidation and Dissolution of Corporations alleging, among other things,
that the present combined financial condition of the petitioners clearly indicates that
their assets are more than enough to pay off the credits but that due to factors beyond
the control and anticipation of the management the inability of the EYCO Group of
Companies to meet the obligations as they fall due on the schedule agreed with the
creditors has now become a stark reality. The SEC Hearing Panel then issued an order
setting its hearing. At the same time, said panel also directed the suspension of all
actions, claims and proceedings against private respondents pending before any court,
tribunal, office, board and/or commission.Without notifying the members of the
consortium, petitioner, however, decided to break away from the group by suing
private respondents in the regular courts. Aggrieved, petitioner immediately took
recourse to the Court of Appeals by filing therewith a Petition for Certiorari with Prayer
for the Issuance of a Temporary Restraining Order and/or Writ of Preliminary
Injunction.

ISSUE:

Whether or not the SEC can validly acquire jurisdiction over a petition for
suspension of payments filed pursuant to Section 5 (d) of P.D. No. 902- A, as
amended, when such petition joins as co-petitioners the petitioning corporate entities
and individual stockholders thereof.

RULING:

NO.

The SEC's jurisdiction on matters of suspension of payments is confined only to


those initiated by corporations, partnerships or associations. Administrative agencies
like the SEC are tribunals of limited jurisdiction and, as such, can exercise only those
powers which are specifically granted to them by their enabling statutes.
Consequently, where no authority is granted to hear petitions of individuals for
suspension of payments, such petitions are beyond the competence of the SEC.
In a case of misjoinder of parties which in this case is the co-filing of the
petition for suspension of payments by both the Yutingcos and the EYCO group, the
remedy has never been to dismiss the petition in its entirety but to dismiss it only as
against the party upon whom the tribunal or body cannot acquire jurisdiction. The
result, therefore, is that the petition with respect to EYCO shall subsist and may be
validly acted upon by the SEC. The Yutingcos, on the other hand, shall be dropped
from the petition and be required to pursue their remedies in the regular courts of
competent jurisdiction.
Aside from the fact that these allegations are evidentiary in nature and still
remains to be proved, we have likewise consistently ruled that what determines the
nature of an action, as well as which court or body has jurisdiction over it, are the
allegations of the complaint, or a petition as in this case, and the character of the
relief sought. That the merits of the case after due proceedings are later found to veer
away from the claims asserted by EYCO in its petition, as when it is shown later that
it is actually insolvent and may not be entitled to suspension of payments, does not
divest the SEC at all of its jurisdiction already acquired at its inception through the
allegations made in the petition.

486 | P a g e
Law 321_Corporation LAW_ Case Digest

SECURITIES REGULATION CODE


(Republic Act No. 8799)

ELEMENTS OF AN INVESTMENT CONTRACT

WHAT SECURITIES ARE REQUIRED TO BE REGISTERED

EXEMPT TRANSACTIONS

PUBLIC COMPANIES

TRADING IN SECURITIES: MARGIN REQUIREMENTS

FRAUDULENT TRANSACTIONS

SIGNIFICANT FACTS ON INSIDER’S DUTY TO DISCLOSE WHEN TRADING

DISCLOSURE REGULATIONS FOR PUBLICLY-LISTED SHARES

487 | P a g e
Law 321_Corporation LAW_ Case Digest

TENDER OFFERS

488 | P a g e

Anda mungkin juga menyukai